Você está na página 1de 495

Bidtd

Cti6'p:'l/f
C [ili ,',,' 'r'':Vi $eS
ffi.hdtioh in
A.
Biological:Moleculs

I.

2. 3. 8., EukarJrotic rCelld l. Cellular ReproductioR ': , ,, ;, 1 , :


a. Eukaryotic Chromosomes b. The Cell Cycte, Mitosis, & Meiosis

Arnino Acids & P,foteifls Carbohydrates & Lipids Nucleic Acids

2. Cellular & Moiecular Organization a. Biomembranes & Memhrane Transport I

b. Nucleus, Nucleolus, & Ribosomes c. Endoplasmicr Reticulurn,,,Col$i Atrrpaf atu s.


"

lrr

Lyqosomes.Fq{o:-liisr}mep,.&vtiioChondlia

d.

Microtubules, Microfilaments,

& Tntermedia te Fila ments

C. Prok4ry.atie Cetl$,: ,r
l.
Cellular Reproduction

a. b.

Prokaryotic Chromosomes
The Cell Cycle Biomembranes & Membrane Transport

2.

Cellular & Molecular Organization

a. b.

Nucleoid & Ribosomes

D.

Viruses

l.

2. 5. Practice Passageo &i Angwers

Architecture & Cenomes Infection & Cenomic Expression Assembly & Release of Progeny Virons

Specializing in MCAT Preparation

RE -{AffiY l)n.n v.r.'"8.\ I'

Structure & Function in Cells & Viruses


Top lO Section Goals

o? Be familiar with the basic biotoqical molecules of cells and viruses.


Be able to recognize

The 4 general classes of bioJogical,molecules are_a.mino acids,nucleic acids, carbohydrates, and Iipids.

their

ba-sic differences

and know their functional significanie.

standard amino acids that av lQotqllg2q B".bl" t" "r-.,.t" rrnt"" ".

are used to synthesize proteins.

Arg, & His) that can have charged polar side chains.

@v
@w

Understand the difference between mitosis and meiosis at the cellular level.
Know the different stages of the cell cycle and how they relate to one another. Be able to describe the chromosome nirmber and ptoiay level ai any stage of mitosis or meiosis. Be familiar with the different tvoes of membrane transoort processes. Have a feel for the differences between simple diffusion, facilitated diffusion, active transport, and bulk transport. Be aware of symports, antiports, uniports, and membrane charge'balance. Be familiar with the different organelles in a eukaryotic cell. Be aware of how they relate to one another wiihin their cellular environment. Know their general structural characteiistics (e.g., single membrane, double membrane, contains DNA", etc.). Be familiar with the functions of eukaryotie oryanelles. -For each of the major organelles in a eukaryotic cell, be ablc to define its [unction in general terms. Are they involved in metabolism? Cellular packaging and sorting? Replication? Protein synthesis?

"? "?
ov
oB

Know where the different metabolic processes occur in a cell. The general metabolic reactions of glycoiysis, Krebs cycle, electron transport, and oxidativc phosphorylation are common to eukaryotes and prokaryotes. Where do they occur in each cell?

Know the general differences between Gram-negative and Gram-positive bacteria.


Bacl.erial cells, for the most part, can be divided into Cram- negative and Cram-positive cells. It is

important to have a feel f or the structural differences between the- two cell types.

t?
@v

Be familiar with the reproductive cycle of a virus. The biological diversity within viruses is enormous._ Just have a general idea of ihe strategy that an enveloped and a non-enveloped virus uses when taking over a host cell.

Know the general differences between eukaryotes, prokaryotes, and viruses.


Just as it is important to know which galaxy our solar system is located in, it also important to know

the basic differences between eukaryotic celis, prokarvotic

ce11s,

and viruses.

Biology

Structure & Function in Cells & Viruses

Biological Molecules

ffi ffi

fi

fi

llffi Gilff $i

:iiffi

iiiiFrot#fi

ffi

Amino Acids
Even though there are more than 400 naturally occurring amino acids, there are a standqrd set of 20 amino acids that comprise the proteins of all living species. As

we

will see, proteins are simply individual amino acid residues linked together in a head-to-tail fashion. All of the standard 20 amino acids are referred to as cramino acids (i.e., a 2-amino acid), except for proline which is referred to as an o(imino acid.
o

An amino acid is composed of a basic amino group (-NHz), an acidic carboxyl group (-COOH), a hydrogen (H) atom, and a side chain which is characteristic to each amino acid. If no one particular amino acid is being discussed, the side
chain is usually designated as -R. The amino group of an amino acid is covalently attached to the cr-carbon atom of the amino acid--hence the name cr-amino acid. The a-carbon atom (Figure 6-1) is the carbon atom next to the carboxyl carbon atom. Each of the 20 standard amino acids has a particular three letter and one letter abbreviation. For example, proline can be abbreviated either as Pro or as P. Each

H,N_ CR

@l

coo

lc

Figure 6.1
An u,-amino acid.

amino acid also has a defined molecular weight (M.) and can be placed, according to its R group (at pH 7.0), into one of three main families of amino acids. The first family (Figure 6-2) consists of those amino acids with nonpolar R groups. These amino acids are hydrophobic.
Nonpolar R Groups

HO @tilo HrN-l C- C- O
H Glycine (Gly)[G] M. = 75

HrN- C- CI

HO orilo
cHs

H3N- C- CH1CCH

HO orilo

H3N- Cs CCH,

HO @tlO
t-

HO OrilO H3N- C- CHlC_ -l


CH CH,

CH: Alanine (AIa)[A]

H3C-

CH

t-

M'=

89

CH:
Valine (ValXVl

CH:
Isoleucine (Ile)[I]

Mr=

Leucine (Leu)[L]

117

M.=

131

M,=

131

HO @rrO H2N- q- C-

U
M,.

H,N_ C_ C-

orro
I

HO
O CHz

'l

HO @rilO H3N- C- C-

HO @rilo H3N- C- C> O

CH" l'
S
I

Proline (Pro)[P]

115

CH: Methionine (Met)[M]

d
Phenylalanine

M'=

149

M,=

165

(Phe)lFl

Tryptophan

(Trp)twl M,=204

Figure 6-2
Amino acids with nonpolar side chains

Copyright O by The Berkeley Review

The Berkeley Review Specializing in MCAT Preparation

BiOlOgy

Structure & Function in Cells & Viruses

Biological Molecules

The second family consists of those amino acids with polar uncharged R groups (Figure 6-3). These amino acids are hydrophilic because they contain side chain functional groups (e.g., -oH, -sH, or -NHz) that can hydrogen bond with water. Polar Uncharged R Groups
H3N- C- CI

HO OrrO
CHz
I

HO @rrO H3N- C- CI
HO_
CH
I

H3N- C- CI

HO @rilO
CH,

OH
Serine (Ser)[S]

t-

CH:
Threonine

SH Cysteine (Cys)tcl

M.= l05

(Th)tTl

M.= ll9

M'=

121

HO @rilO HrN- C- C- O
CH"

HO @rriO HrN'l C- CCHz


I

HO @ruO HlNc- C- O -l
CH,

t'

O= C-

NHz

t-

CH,

a
OH
Tyrosine (Tyr)tYl

t-

O= CAsparagine (Asn)[N]

NH2

Glutamine (Gln)tQl

M. = 132

M' =

146

M.= l8l

Figure 6-5
Amino acids with uncharged polar side chains.

The third family (Figure 6-4) consists of those amino acids with charged R groups. Note that two of the amino acids (Asp and Glu) have side chains which are carboxylic acids and three of the amino acids (Lys, Arg, and His) have side chains which are amine bases. The side chains of Asp, Glu, Lys, and Arg are highly ionized at neutral pH. The side chain of His is weakly ionized at neutral pH. we will see that the degree of ionization of an amino acid depends on its
acidic and basic properties. Charged R Groups

H3N- C- C-

@l

ilo
O

HO orilo H,N''l C- C_ O
CH.

H O otilol H3N- C- C- O tl CH, 'O


O=

@T?o HrN-C-C-O
CHz

CHu
I

l'

CH"

t' CH, tCH,

r C-O

CH,

O= C-O
M,=
147

r-O

6l

t' CH, tNH r@


H2N- C=
NH2

CH,

HrNC- C-l
CHz

HO @rilo

NH:
146

H-N\/,N-H
M' = 155

/,'_\o
I

Aspartate(AspXDl Glutamate (Clu)[E]


M. =

133

Lysine (Lys)[K]

M' =

Arginine (Arg)[R] M,.= 174

Histidine (HisXHl

Figure 6-4
Amino acids with charged side chains.

Copyright @ by The Berkeley Review

The Berkeley Review Specializing in MCAT Preparation

Biology

Structure & Function in Cells & Viruses

Biological Molecules

Although the standard 20 amino acids can exist in either the D or the L configuration (with the exception of glycine because it is non-chiral), only the Lamino acids are found in proteins. It is not known why evolution chose to incorporate L-amino acids into proteins instead of D-amino acids. As an example, the two non-superimposable mirror images of the o-amino acid alanine are shown in Figure 6-5. Note that the o-carbon of alanine is chiral (there are 4 different substituents attached to that u-carbon). The D isomer and the L isomer of alanine are called enantiomers. The D and L isomers of the different amino acids are based on the absolute configuration of D and L-glyceraldehyde. [All amino acids mentioned in these readings will be in the L-configuration, unless otherwise
stated.l

H3N>.C<
cHl

oi

coo

o
H

coo

o
NHj

H>. C<
Mirror
CHr

tO

L-alanine
Figure 6-5

D-alanine

Non-superimposable minor images.

Even though D-amino acids are never found in proteins, they do exist in many organisms. For example, D-alanine and D-glutamate are found within the rigid cell walls of some bacteria. The presence of these D-amino acids helps to prevent the degradation of the bacterial cell wall by specific enzymes called proteases (more on this later). D-Valine is found in the antibiotic valinomycin, a carrier of K+ ions across membranes.

Amino acids found within proteins can be modified as shown by the examples given in Figure 6-6. Two important amino acid modifications can be found in collagen, the most abundant protein in mammals. Some of the proline and lysine amino acid residues found within nascent molecules of collagen are hydroxylated to give -hydroxyproline and S-hydroxylysine, respectively. These modified amino acids help to give collagen its high tensile strength. Histamine, a powerful vasodilator, is formed from the decarboxylation of the amino acid histidine. S-Adenosylmethionine is an activated form of the amino acid
methionine and can methylate protein or nucleic acid substrates.

HO Orll O HrN- C- C- O
I

CHz
I

H3N- C-H
CHz
I

@r

CHz

ll o c-o

@t

H-CCH2

OH

H1N-

H-N./N
HO OH

4-Hydroxyproline 5-Hydroxylysine
Figure 6-6
Modified amino acids.

Histamine

S-Adenosylmethionine

Acid-Base Properties Body fluids in biological systems generally have a pH range of 6.5 to 8.0. This is considered to be physiological pH. At these pH ranges in the cell the amino and carboxyl groups of the standard amino acids are ionized. In other words, the cramino group bears a positive charge while the a-carboxyl group bears a negative charge. The dipolar or zwitterionic nature of the amino acids is due to the fact that the pKu value for the s-amino terminal is about 9.4 while the pKu for the scarboxyl terminal is about 2.2. Since amino acids can act as either an acid or a base they are referred to as ampholytes. Table 6-1 shows the pKu values for the
Copyright @ by The Berkeley Review
D

The Berkeley Review Specializing in MCAT Preparation

BiOlOgy

Structure 6r Function in Ceils & Viruses

Biological Molecules

values refer to a weak acid. Amino Acid


Glycine Alanine Aspartic Acid Glutamic Acid Histidine
Cysteine

o-carboxyl and s-amino groups of a selection of the 20 standard amino acids and their ionizable side chains. Small pKu values refer to a strong acid while large pKu

pKu (u-COOH) 2.4


2.3

pKu (a-NH3+) 9.8 9.9 9.9 9.1 9.2


10.8 9.1

pKu (side chain groups)

2.0 2.2
1.8 1.8

3.e (p-cooH) 4.2 (y-cOOH) 6.0 Imidazole) 8.3 (Sulftydryl)


10.1 (Phenol) 10.8 (e-amino) 12.5 (Guanidino) 13.0 (Hydroxyl)

Tyrosine Lysine

2.2

2.2
1.8

Arginine
Serine

2.2

9.2 9.0 9.2

Table

6-l

The pK values for the ionizing groups of a few of the standard 20 amino acids.

The tlenderson-flasselbalch Equation

of either the g-amino, G-carboxyl, or side chain groups that are ionized at a particular pH. In this equation [Ae] is the concentration of conjugate base while [HA] is the concentration of conjugate acid.

The Henderson-Hasselbalch equation (6-1) can be used to determine the fraction

(6-1)

pH=pKu+rosffi

fl

enderson-llasselbalch Equation

pH - PKa
J 2
1

lAl/HA
0.001 0.01
0.1
1

Let's consider the ratio of the protonated to the unprotonated cr-amino group of a general amino acid at a pH of 7.0. The pKu for a typical s-amino gtonp is ibout 9.4. using the Henderson-Hasselbalch equation, we find that the ratio of the protonated cx-amino group to the unprotonated o-amino group is about 102.4 or 251:1. This tells us that the predominant form of the u-amino group at a pH of 7.0 is the protonated form. The same type of analysis for the cr-carboxyl group tells us that the ratio of the o-carboxylate anion to the protonated cr-carboxyl group is about 63,000:1.

I
2
-)

10

(6-2)

.o =

100

[R-NHz] 9.4* ,on "

102.4

tR-NHil

tR-NHil _2sr

lR-NH2l

(6-3)

1000

Table 6-2
Difference between pH and pKa and the relationship to the ratio of conjugate base to conjugate acid.

similarly, at a neutral pH of 7.0 the ratio of the protonated to non-protonated side chain of histidine is about 1:10. If we know the difference between the pH and the pKu (i.e., pH - pKu), then we can establish a ratio (Table 6-2) between the concentration of conjugate base to conjugate acid. If a particular amino acid is required at the catalytic site 6f an enzyrne, knowing the ratio between conjugate base and conjugate acid will help determine the maximum potential activity of the enzyme.

Copyright @ by The Berkeley Review

The Berkeley Review Specializing in MCAT Preparation

Biology

Structure & Function in Cells & Viruses

Biological Molecules

Isoelectric Point (pI) The isoelectric point (pI)'is that p H at which an amino acid (or a molecule) carries no net electric charge. By manipulating the Henderson-Hasselbalch
equation we can obtain an expression (6-a) that states that the isoelectric point is simply the arithmetic mean of two pKu values.

(6-4)

pI=

lpKar + pKa2]
2

For example, the isoelectric point for the amino acid glycine is 6.1 while the isoelectric point for the amino acid lysine is 1.0.0. If we were to place these two amino acids in an electric field, we would find that at any pH above their isoelectric points they would migrate toward the anode (positive electrode) while at any pH below their isoelectric points they would migrate toward the cathode (negative electrode). This characteristic allows for separation of amino acids by a process called electrophoresis. Paper electrophoresis is generally used for separating mixtures that contain charged molecules which are small while gel electrophoresis is used for separating proteins and nucleic acids.

Titration Curves At physiological pH glycine exists in solution as the dipolar ion. The net charge on this amino acid would be 0. However, if we were to add a strong acid to the dipolar solution, the carboxylate group would become protonated and the overall charge of the amino acid would be +1. Similarly, addition of a strong base to the dipolar solution would remove a proton from the protonated cr-amino group, giving the amino acid a net charge of -L. As shown in Figure 6-7,the ionization of glycine depends on the pH of the solution.
pH=1
pH = 6.1

pIJ=14

oT3 s,Nc- oH
HPK"H
[Net charge is +1]

-H

HO @rrio

-H

HO rilo
I

?-

a^

PKo
9.8

[Net charge is 0]

[Net charge is -1]

Figure 6-7
Ionization of glycine.

The ionization of glycine can be followed by using a titration curve (Figure 6-8). We can start our titration by adding equivalents of base (such as NaOH) to a solution of glycine which is fully protonated. The initial pH of this solution might be in the neighborhood of 1 and the overall net charge on glycine is roughly +1.0.

The first midpoint of the titration curve comes when [NH3@CH2COOH] = [NH3oCH2COOO].The Henderson-Hasselbalch equation tells us that at this first midpoint the pH = pKa. In other words, the pH at this point is 2.4 (which is the pKu of the first ionizable proton), and half of the solution is composed of NH3@CH2COOH while the other half of the solution is composed of NH3oCH2COOo. The overall net charge on glycine in solution at this point is about +0.5.
Copyright @ by The Berkeley Review

The Berkeley Review Specializing in MCAT Preparation

BiOlOgy

Structure & Function in Cetts & Viruses

Biological Molecules

Addition of more base will eventually convert all of the NH3@CH2COOH to NH3@CH2Cooo. At this point the first equivalence point of the solution has been reached. The overall net charge on glycine is 0. The isoelectric point has
been reached and the pH is about 6.1.

In order to reach the second midpoint in the titration we add more base. At the second midpoint we find that [NH3@CH2COOO] = [NH2CH2COOO].Again, the Henderson-Hasselbalch equation tells us that at the second midpoint the pH = pKu. The pH at this point is 9.8 and the overall net charge on glycine in solution is about -0.5.
Second Midpoint

INH3+CH2COO-] = tNH2CH2COO-l

PKa = 9'8
8

lNH2CH2 First Midpoirtt lNH3+CH2COOHI


il

coo-l

EquivaLence Point

pH6
4

tNH3+CH2COO-I PKa = 6'1

lNH3+CH2COO-I

Buffer Region

PKa = 2'4

tN H3+cH2cooHl

+
0E-

0.5

1.0

1.5

Equivalence of NaOH Added

Figure 6.8
Titration of glycine.

By the time the second equivalence point is reached enough base has been added to completely titrate all the ionizable protons on glycine. The dominant species in solution is now NH2CH2COOO. The overall net charge on the amino acid is -1.

Buffers
The ability of a solution to resist a change in its pH when either an acid or a base is added is the principle behind the buffer capacity of a solution. In Figure 6-8 note that the slope of the titration curve is much less near the first and second midpoints than near the equivalence point. For each increment of base added in the vicinity of the pKa's of glycine the pH changes very little. This means that

the numerical value of the log ([Ae]/[HA]) in the Henderson-Hasselbalch equation does not change that much. In other words, [Ae] approximately equals [HA] in the region of the pKa. If a weak acid is within 1pH unit of its pK3 value, it resides within a good buffering range.

Copyright @ by The Berkeley Review

The Berkeley Review Specializing in MCAT Preparation

Biology
Proteins

Structure & Function in Cells & Viruses

Biological Molecules

The 20 standard amino acids that we have mentioned can be linked together to form long polypeptide chains. Amino acids can be joined to one another by a special type of amide bond called a peptide linkage. For example, in Figure 6-9 two amino acids are joined together to form a dipeptide. As we will see in a later discussion, this reaction requires an input of free energy. This means that the hydrolysis of the dipeptide would be more favorable than its synthesis. All hydrolysis reactions are favorable reactions.
Peptide Bond

H3N- C- CR1

@l

ilo
O

HrN_c_c_o

o I

Ho il

Hzo
o /

0
I lil @ "[o-l
I

_;:tj,__i,l.-ir

HO rilo C-C-O
I

R2

Figure 6-9
Formation of a dipeptide from two amino acids.

A polypeptide is a convenient term for any length of a polymer of amino acids. If we put two amino acids together, we have a dipeptide. If we put three amino acids together, we have a tripeptide. We might run into the word oligopeptide. An

oligopeptide is simply 10 or so amino acids linked together. A protein is


generally described by 100 or more amino acids linked together.
Amino terminal
end

H O Cysteine Iil C_N-C_C-N-C tt HCH,H tI

SH SH

Carboxyl terminal
end

H CH" H I tC_ N- C- C_N I I

HOHOHOHOHO oriltilrtttttlllo C- C- N- C- C- N- C- CHlN'rtt C- C- N- C- C- Nll tl ll


RTHRzHRjHR+HR5
Figure 6-lO
Modified amino acids.

il

Cysteine

reduction

1l

ll

"*,o"u.r

Amino acid units are often referred to as residues. Consider the pentapeptide shown in Figure 6-10. This peptide is composed of 5 amino acid residues and is synthesized (and written) from the amino terminal residue to the carboxyl terminal residue, left to right, respectively. In other words, if we had the pentapeptide Tyr-Gly-Gly-Phe-Met, then Tyr would be the amino terminal and Met would be the carboxyl terminal.
X-ray crystallographic studies performed by Linus Pauling and Robert Corey in the late 1930s showed that the peptide unit (i.e., the O-C-N-H bonding) is planar and rigid due to the partial double bond character of the C-N bond (i.e., the peptide bond). However, there can be rotation about the bonds between the acarbon and the carbonyl carbon and the s-carbon and the nitrogen.

Iil C-N-C_C-N-C tt I HCH^H t'


S
I

HO

S
I

HCH"H tt"l C-N-C_C_N-C lll HO


Cystine

Figure 6-l

Formation of a disulfide linkage between two cysteine


residues.

disulfide bonds as shown of bovine insulin acid sequence the amino within in Figure 6-11. For example, the ability to form three have residues These there are 6 cysteine residues.
Some proteins that contain cysteine residues can form Copyright O by The Berkeley Review

The Berkeley Review Specializing in MCAT Preparation

BiOIOgy
(a)

Structure & Function in Cetls & Viruses

Biological Molecules

cystine.

distinct sets of disulfide cross-links. When two cysteine residues are oxidized they form a disulfide called cystine. Be careful of the difference between cvsteine and

away from the helical axis (Figure 6-12b).


Hydrogen bonds between the C=O and N-H groups help to stabilize the cr-helix.

t-9st Pauling and Corey suggested that polypeptide chains have the ability to tn fold into a-helices and B-pleated sheets. the cr-netix is stabilized by hydrogen bonding between the Co and NH groups as shown in Figure 6-12a (that are f6ur residues apart). There are about 3.6 amino acids per turn of the cr-helix. They are separated from one another by a translational distance of 1.5 A and a rotation of 100 degrees. The side chains of the amino acid residues extend outward and

ft)

In the B-pleated sheet the hydrogen bonding between the co and NH groups occurs between different polypeptide chains as shown in Figure 6-13. These polypeptide chains can either be parallel (running in the same direction) or antiparallel (running in opposite directions). In the situation of the antiparallel Bpleated sheet the polypeptide chain is almost fully extended and the distance between amino acid residues is about s.s A. As a polypeptide chain folds back to itr the opposite direction, it reverses direction by making a _tnt B-Turn. Hydrogen bonding at the B-Turn occurs between the Co group of one amino acid residue and the NH group of an amino acid residue which is three residues away.

Looking down the o-helix


we see the side chains pointing outward.

ROHRO l ll H l t ilH .c. c. r-N_^_c_---c_'i-N. rro / i-1l1--tc r c N C' c F l ilHt Ht t \ HROHN\ Hydrogenbond+: n_C_R : !

Figure 6- l2
Two views of an s-helix. (a) Side view. (b) Top view.

I + \ Hfl t-attHilttHil RHORHO

^,f-

c.

N-.

THP )"./
!.c.
N,?-

".

Figure 6-13
Hydrogen bonding in antiparallel B-sheet.

In 1957 John Kendrew worked out the three-dimensional structure of the protein myoglobin (Figure 6-14). Human myoglobin consists of 153 amino acid reiidues. Roughly 75"/, of this polypeptide chain is in an a,-helical conformation. There are 8 major helices called helix A, B, c, D, E, F, G, and H. within a crevice of the protein is a heme group which can bind oxygen. The interior of the myoglobin protein is composed almost entirely of hydrophobic non-polar residues.
Not all proteins have such a high percentage of o-helices within their framework. For example, ribonuclease S (an enzyme secreted from the pancreas that hydrolyzes RNA) is a protein that is composed of 124 amino acid residues which are arranged in a number of B-pleated sheets. Proteins with a single polypeptide chain can be defined by their primnry, secondary, and tertiary structure. The primary structure represents the sequence of amino acids in a protein and includes the location of disulfide bonds. The secondary structure represents the spatial arrangement of amino acids that are close to one another while the tertiary structure represents the spatial arrangement of amino acids that are far from one another. A protein with tertiary

Figure 6myoglobin.

l4
of

Three dimensional structure

Copyright @ by The Berkeley Review

to

The Berkeley Keview Specializing in MCAT Preparation

Biology

Structure & Function in Cells & Viruses

Biological Molecules

structure can be referred to as a subunit. If various subunits associate with one another, then the protein is said to have quaternary structure. For example, myoglobin is a protein with tertiary structure but hemoglobin, because it contains four polypeptide subunits, is a protein with quaternary structure.

cr-helix

The level of structure in a protein can best be understood by considering hemoglobin. The four polypeptide (two a and two p) subunits of hemoglobin form a tetramer which is spheroidal in shape. Associations between these subunits are formed through hydrophobic, ionic, hydrogen bonding, or polar interactions. It is the interactions between these four polypeptide subunits that gives hemoglobin its quaternary structure.
However, if we were to examine just one of the polypeptide subunits, we would find that it would primarily contain a number of cr-helical domains (given the letters A-H). The complete spatial arrangement of all the amino acids to one
Primary

another within this polypeptide subunit defines the tertiary structure. The secondary structure is simply the spatial arrangement of amino acids adjacent to one another within the polypeptide chain. The primary structure includes the primary sequence of the polypeptide chain. These four levels of protein structure are depicted in Figure 6-15. In particular, we have focused in on a set of amino acids in the E helix of one of the B subunits of hemoglobin.

Secondary

It tums out that it is the primary structure that determines the way in which a protein will fold up on itself. Insight into this phenomenon came from the work that Christian Anfinsen did with bovine ribonuclease. He found that in the presence of p-mercaptoethanol (which cleaves disulfide bonds) and 8 M urea, the ribonuclease enzyme was denatured and lost its enzymatic activity.
Enzymatic activity was restored to this protein after the sulfhydryl groups on the cysteine residues were oxidized and dialysis of the p-mercaptoethanol and urea was complete (Figure 6-16). This experiment confirmed that it is the primary

structure of a polypeptide that determines the three-dimensional tertiary


structure of a protein.
Quaternary Chemical treatment Removal of chemical treatment
SH SH Denatured Ribonuclease

Figure 6-

l5

The four levels of protein structure in the molecule hemoglobin.

Ribonuclease (active)
Figure 6- l6

(inactive)

Chemical denaturation of ribonuclease using B-mercaptoethanol.

Out of the many possible ways in which the ribonuclease protein can fold into its tertiary structure, only one will lead to the correct enzymatic activity. It turns out that protein folding is not a random search. Why? Because it would take too long to complete the process. In fact, if a protein had 100 amino acid residues, each having the possibility of assuming 3 different positions,then there would 6" 3100 possible stiuctures. This would take about 'L.6 x L027 yearsl The universe, however, is only about 1.5 x 1010 years old (give or take a few billion years).

Copyright @ by The Berkeley Review

ll

The Berkeley Review Specializing in MCAT Preparation

Biology

structure & Function in ceils & viruses

Biological Molecules

How is it thai polypeptides fold into a tertiary structure? one theory says that sections of the primary structure fluctuate between being in the natlve linear (primary) arrangement and either an s-helix or a B-pleated'sheet and that when two such forms of secondary structure interact wiih one another by diffusion they tend to stabilize each other. The exact mechanism as to how this happens is not known.
The idea of two helices Filg side by side is rather interesting, especially in proteins that bind to DNA. If two cx-helices, each containing af leasl 4 leucine residues and located on separate proteins, come together throirgh interdigitation of those leucine resid'es, tfen a leucine zipper wilt form bnainjthe two iroteins tgsetfel Even though the leucine zipper does not directly inteict with the DNA double helix, it does allow for the binding of some DNA regulatory proteins. we will examine this in a little more detail when we discuss thJexpressi-on of genetic information.
Essentially all reactions in a cell are cataryzed by proteins called enzymes. when into contact with the active rit" or catalytic site of an enzqe, it will be bound at that site in such a way that the enzyme will be able to convert
a substrate comes

is a site that is different from the catalytic site. Enzymes of thiJ naturi are generally referred to as allosteric enzymes (from the ireek allo for',other" and stereos for "space" or "site").
We will find that many different proteins have these allosteric regulatory sites and that these sites can control the binding of various substrate or modulator molecules. A good example of an allosteric protein is hemoglobin. Hemoglobin has four polypeptide subunits (it has quatemary structure), -each having u"h"-" gro-up that has the capability of binding an oxygen molecule. when or,J o*yg"r, molecule binds to the heme group of one of the polypeptide subunits, tlat binding information is somehow transmitted to the otn". tnr"" polypeptide subunits, thus facilitating their ability to bind oxygen.

that substrate into a product molecule. There ur" oth", enzymes that, besides having a catalytic site, have-a regulatory site or allosteric site.'The regulatory site

Copyright @ by The Berkeley Review

t2

The Berkeley Review Specializing in MCAT Preparation

Biology
Carbohydrates

Structure & Function in Cells & Viruses

Biological Molecules

Carbohydrates E(i,Lipids
The name carbohydrate is misleading. It was coined at a time when the formulae of carbohydrates was just becoming known. Almost all carbohydrates had a similar empirical formula which looked like Cn(H2O)tr' This formula leads us to believe that there is a water molecule attached to each carbon atom that apPears in the structure. However, this is not the case. A better empirical formula to use is (CH2O)n.

oH -\/
C
I

H- CI

OH

cH2-oH Glyceraldehyde cH2-oH


I

gto.rpi. Carbohydrates are simply polyhydroxylated aldehydes or ketones. If we iollow this rule, we find that the simplest carbohydrate is obtained when n = 3 in the empirical formula (CH2O)1. Glyceraldehyde and dihydroryacetone are two simpleit carbohydrates that can be formed (Figure 6-17). Glyceraldehyde has an aldehyde functional group while dihydroxyacetone has a ketone functional group. Each has two alcohol groups. If the carbohydrate contains an aldehyde, it is referred to as an aldose. If it contains a ketone, it is referred to as a ketose.
As a group carbohydrates are the most abundant biological molecules on earth. Theyian be divided into three major classes: monosaccharides, oligosaccharides, and polysaccharides.

In order for a molecule to be classified as a carbohydrate, it must have an aldehyde or a ketone functional gloup and two or more alcohol functional

C=O
I

cH2-oH Dihydroxyacetone

Figure 6-17
Molecules with the molecular formula (CHZO)n.

Monosaccharides

monosaccharide, besides being a polyhydroxylated aldehyde or ketone, contains a backbone of unbranched carbon atoms that are covalently linked to

one another. Glyceraldehyde and dihydroxyacetone are the two simplest monosaccharides. If the carbohydrate has a 3-carbon backbone, it is called a triose. Besides the triose sugars, we will also find sugars which are tetroses (4-C),
pentoses (5-C), hexoses (6-C), and heptoses (7-C).

At the turn of the twentieth century Emil Fisher realized that he could devise a system for naming carbohydrates (and amino acids) based on the structure of glyceraldehyde. He chose glyceraldehyde because it has just one chiral center (a iuiUon atom to which four different substituents are attached). If a molecule has n chiral centers, it will have 2n stereoisomers. Glyceraldehyde has two stereoisomers (Figure 6-L8). Fisher said that if the hydroxyl group on glyceraldehyde's chiral carbon is to the right, the molecule is in the form of the D-isomer. If the hydroxyl group is to the left, the molecule is in the form of the L-isomer.
In order to tel| if a monosaccharide drawn in the Fisher projection is in the D or L isomeric form, we only need to look at that chiral carbon which is most distant trom the carbonyl carbon. That particular chiral carbon atom is often referred to as the reference carbon. If the hydroxyl group attached to that reference carbon is to the right, the molecule is the D isomer. If the hydroxyl group is to the left, the molecule is the L isomer. Most of the naturally occurring sugars are found in their D form (while most of the naturally occurring amino acids are found in their L form).

H.
C
I

,f

Mirror oat ,H
C
I

"o-!-,
L

n7!\ou
cH2-oH

cH2-oH

D
Glyceraldehyde

Figure 6-lB
D-Glyceraldehyde and its
enantiomer.

\lonosaccharides can either be found in the linear form or in the cyclic form. \fany sugars that contain five or more carbon atoms in their backbone prefer to re rn the cyclic form. For example, the aldohexose D-glucose (Figure 6-19) can
Copyright @ by The BerkeleY Review

r5

The BerkeleY Review Specializing in MCAT PreParation

Biology
C-l
C.2 C-4

structure & Function in celrs & viruses

Biological Molecutes

o.H r\z -c
H_C-OH
I I

c-3 Ho- c-H


H-C-OH
I I

C-5 H-c-oH c-6 cH2oH


D-Glucose

form a six-membered ring referred to as a pyranose. In the formation of the ring (Figure 5-20), the oxygen atom of the C-5 hydroxyl group reacts with the carbonyl carbon atom to form two different diastereomeir, a-D-gl.r.opyranose and p-D-glucopyranose. Because these two molecules diffe-r or,ly in th" configuration about the C-1 carbon atom, they are referred. to as anomers, and the C-1 carbon atom itself is referred to as the anomeric carbon. L:r the o( anomer note that the oH group at the anomeric carbon is on the opposite side of the ring f1o*_!" CH2OH group that is attached to the referet .iibon. In the B anome"r "" side of the ring as that tle oH group at the anomeric carbon is on the same CH2OH group.

Figure
Glucose.

6-lg
o
t

The straight chain form of D-

.H

Ht

n-H \ o

HOH
B-D-Glucopyranose

HOH

C-l C-2

u,-D-Glucopyranose

cH"oH
l"
I

C= o
I

c-3

Figure 6.2O
Formation of the two anomers of D-Glucopyranose.

Ho- c* H C-4 H-C_OH I C-5 H-c-oH c-6 cH2oH


I

D-Fructose

Carbohydrates can also exist in a five membered ring. ln this case the cyclic sugar would be called a furanose. The ketohexose D-fructose (Figure 6-21) can.y"i"" to form a five membered ring (Figure 6-22). There are two diastereomers, o-Dfructofu ranose and p-D-fructofuranose.

Figure 6-21
The straight chain form of DFructose.

*"W:
OHH
d,-D-Fructofuranose

l''"[?{::]'ry'".
OH

OHH
B-D-Fructofuranose

Figure 6-22
Formation of the two anomers of D-Fructofuranose.

Formation of the pyranose ring in D-glucopyranose results from a general reaction involving an alcohol and an aldehyde. The resulting derivative is referred to as a hemiacetal. similarly, formation of the furanose ring in Dfructofuranose results from the general reaction involving an alcohoi and a ketone. In this case a hemiketal is formed. Formation of these derivatives is shown in Figure 6-23.
Since D-glucose and D-fructose readily interconvert between the linear and cyclic forms, they can undergo reactions which are typical to aldehydes and ketones, respectively. Two oxidizing agents used to identify the functional groups of carbohydrates is the Tollens'reagent (containing an Ag@ complex) and the Benedict's reagent (containing a Cu2@ complex). Ir tne rouens' reagent is used to Copyright @ by The Berkeley Review

t4

The Berkeley Review Specializing in MCAT Preparation

Biology

Structure & Function in CeIIs & Viruses

Biological Molecules

identify an aldose or a ketose, the sugar is oxidized and the Ag@ ion is reduced to silver metal (which precipitates as a siluer mirror on the sides of the reaction vessel. If the Benedict's reagent is used, the sugar is oxidized and the Cu2@ ion is reduced to give a brick-red precipitate. If an aldose or a ketose is capable of reducing these ions, those sugars are referred to as reducing sugars. Carbohydrates that contain a hemiacetal or a hemiketal group give positive tests with Tollens' and Benedict's reagents.
o
H +
R2_

il Rr-

O\

C-

HO-

R2

=*

/O_

RrH
Aldehyde

,C,

Rl- C-

o il

R2_
R.3

O\

HO- R. - -i*

/o_
R:

Rr

/\
C

Alcohol

Hemiacetal

Ketone

Alcohol

Hemiketal

Figure 6-23
Hemiacetai and hemiketal formation.

Oligosaccharides
Oligosaccharides are relatively short chains of monosaccharides linked to one another by a glycosidic bond. Disaccharides are the most common oligosaccharides. The disaccharide sucrose (table sugar) is formed by the linkage of aD-glucopyranose and B-D-fructofuranose. The bond connecting the two monosaccharides is termed an O-glycosidic bond, and it is the carbohydrate analog of the peptide bond found in proteins.
The systematic naming of disaccharides (and larger sugars) follows a few simple rules. Let's consider sucrose as an example (Figure 6-24). First, notice how the two monosaccharides in sucrose are joined together. They are linked through an

oxygen atom, which means that the bond is an O-glycosidic bond. If we were to hydrolyze that bond with H2O, we would get the two individual monosaccharides (glucose and fructose).
CH,OH

OH
CI

HOH
G-o-D-glucopyranosyl-(
1

OHH
(Sucrose)

HOHOHH
cr-D-Glucopyranose
B-D-Fructofuranose

-r2)-B-D-fructofuranoside

Figure 6-24
Sucrose is also systematically called O-a-D-glucopyranosyl-(1-+2)-$D-fructofuranoside.

Locate the anomeric carbon for glucose. Is the OH group attached to that anomeric carbon in the cr or B conformation? It is s because the hydroxyl group at the anomeric carbon is on the opposite side of the ring from the CH2OH group that is attached to the reference carbon. Is the OH group attached to the anomeric carbon of fructose in the cr or B conformation? It is p because the hydroxyl group at the anomeric carbon is on the same side of the ring from the CH2OH group that is attached to the reference carbon. This is an unusual linkage in that the Oglycosidic bond is formed between two anomeric carbons. If we view this linkage
Copyright @ by The Berkeley Review

l5

The Berkeley Keview Specializing in MCAT Preparation

Biology

structure & Function in cells & viruses

Biological Molecules

from the point of view of the glucose molecule, we will see that the bond is formed between the C-1 atom of glucose and the C-2 atom of fructose. This is written as (1-+2) in the naming of the molecule. putting what we have learned so far together, we could name sucrose as o -a-n-glucopyranosyl-(L-+2)-B-Dfructofuranoside' The O in front of the name is to remind us that the linkage is an o-glycosidic linkage. The -ide at the end of the name tells us that sucroie is not a reducing sugar.
We know that a sugar is a non-reducing sugar if its hemiacetal or hemiketal group has been converted to an acetal or ketal group, respectively. we can form an acetal or a ketal as outlined in Figure 6-2s.If a carbohydrate contains only an

acetal or a ketal group,


Benedict's reagent.

it will not react with either the Tollens' ,"ug"rri o,


O\

R.-O O-H ,"t +


RrH

R2-

HO- R:

/-\ RrH
Acetal

P- R3

R.-O O-H CHO-R4 : ,/\ Rr R3


Hemiketal

R2- O,

,ORs

Rc

Rr

Hemiacetal Alcohol

Alcohol

Ketal

Figure 6-25
Acetal and Ketal formation.

It is composed -D-galactopyranose and p-D-glucopyranose linked together through a B(l-+a) O-glycosidic linkage (Figure 5-26). Notice that the anorieric cu.bon o-f the galactose residue is tied up in an acetal linkage with glucose. However, the anomeric carbon of the glucose residue is just part of a hemiacetal group, and since it can be oxidized,lactose is a reducing sugar. [The structure oflactose has been given on the MCAT a number of times.l
9f
B cH2oH

Lactose (milk sugar) is a disaccharide that is found only in milk.

OH H'I
2

al-

HOH
(Lactose)

HOH

HOH

o-B-D-galactopyranosyl-( I -;4)-p-D-glucopyranose B-D-Galactopyranose B-D-Glucopyranose

Figure 6.26
The disaccharide lactose and its two monosaccharide components, D-galactose and D-glucose.

Polysaccharides
Polysaccharides (glycans) are large molecules which contain many monosaccharide residues linked together through glycosidic linkages. Polysaccharides can consist of just one type of monosaccharide, in which case they are called homopolysaccharides, or they can consist of different types of monosaccharid.es, in which case they are called heteropolysaccharides. Each type can be either unbranched or branched. Two important (storage) polysaccharides are starch and glycogen.

Copyright @ by The Berkeley Review

l6

The Berkeley Review Specializing in MCAT Preparation

Biology

Structure & Function in Cells & Viruses

Biological Molecules

Starch is a food reserve in plants. It can be found as cr-amylose, a linear polymer of unbranched D-glucose residues linked together in o(1+4) linkages, or as amyldpectin, a branched polymer of D-glucose residues linked together primarily in a(1+4) linkages but at the branch points in o(1-+5) linkages. Examples of both polymers are shown in Figure 6-27. Tll.e branch points in amylopectin occur roughly every 24 to 30 glucose residues.
CH"OH

QH
H

OHH HOH
cH2oH

HOHO cH2oH 6 cH2


I

H
HOH
c-amylose

\r
fLoJ\

23

HOH

amylopectin

Figure 6-27
The two forms of starch:. cx-amylose and amylopectin.

Starch can be found in wheat, rice, corn, and potatoes and is the major carbohydrate source in the human diet. Many of the starches contain about 15% cr-amylose and 85% amylopectin. Digestion of starches begins in the mouth with the enzyme saliaary cr-amylase. This enzyme hydrolyzes many of the o(1-+a) linkages in starch and degrades large polysaccharides into smaller oligosaccharides. Once the oligosaccharides pass through the stomach and into the duodenum of the small intestine, they are degraded even further by pancreatic uamylase to disaccharides, trisaccharides, and small branched oligosaccharides referred to as dextrins. Enzymes within the intestinal system degrade these oiigosaccharides into individual monosaccharides, which are then absorbed by the intestinal epithelial cells and passed to the blood'
Glycogen is the storage polysaccharide common to all animals and is located primarily in skeletal muscle and liver tissue. Glycogen has a structure similar to that of amylopectin, except that the branch points in glycogen come about every 8 to 12 glucose residues.

Lipids

Biological lipids are molecules that can readily dissolve in nonpolar solvents but are relatively insoluble in water. These compounds are chemically and lunctionally quite diverse. Some of the classes of lipids that we will examine are the fatty acids and triacylglycerols, glycerophospholipids, sphingolipids, and
cholesterol.

Copyright @ by The Berkeley Review

The Berkeley Review Specializing in MCAT Preparation

Biotogy
Ho.
H"C

structure & Function in cells & viruses


Fatty Acids & Triacylgtycerots Ho.
H"C

Biological Molecules

-o c'

-o C/

t'
H,C

CH,

CH. H,C
CH"

CH. t' H"C

H,C
CH, H"C

however, will contain double bonds between specific carbon atoms. Two common fatty acids are shown in Figure 6-2g.
backbone to form a triacylglycerol (Figure 6-29). In animals these compo,inds are synthesized and stored in adipocytes (fat cells). Triacylglycerols arc-neutral fats and serve as storage depots for fuel used in metabolism. Triacylglycerols ihat contain a high degree of saturated fatty acids (e.g., butter) pac{ t*ofether quite well and form crystalline structures that havefairty high meltiig pointJ. This is due to the large van der waals attractions between ttre rnethyle"J (-cuz-) groups of the hydrocarbon portion of the fatty acid. As the molecular weight of a iaturated fatty acid incteases, so does the melting point. The fluidity of tIi" tlpia decreases and at room temperature tend to become solids referrei to as fats. A double bond in an unsaturated fatty acid places a kink in the hydrocarbon chain, thereby decreasing the van der waals interactions and lowering the melting poini. Triacylglycerols that contain a high degree of unsaturated or polyunsiturated fatty acids (e.g., olive oil) tend to be oils at room temperature.

Fatty acids are carboxylic acids with a hydrocarbon side chain. The majority of fatty acids have side chains that are unbranched and contain an even number of garbon atoms' In plants and animals the more common fatty acids contain either 16 carbons or 18 carbons. If a fatty acid is saturated, ali of its carbon atoms (except the carbonyl carbon) will have a full complement of hydrogen atoms. we will not find any double bonds between carbon uio*s. An unsaturfted fatty acid,

H"C

CH, /'

'\^_H
L

In nature, fatty acids are rarely free. Rather, they are esterified to a glycerol

CH, t' H-C CH" H"C

H.C

/H

c-

il

CH"

H.C H:C

/'

CH.

H"C CH"

/"

CH.

HNC

CH, H"C

Palmitic acid
(Cro)

CH: Oleic acid (Cra)

Figure 6-28
Two common fatty acids.

H,CF OH

"oAAz\o

H,c-

ln

oA-'\'/\

"l- "/--..-,,^--H2C- OH
Glycerol Fatty Acids

",Lo-[^^o
Triacylglycerol

Figure 6'29
The formation of a triacylglycerol.

Glycerophospholipids

The glycerophospholipids (or phosphoglycerides) are the predominant lipid component of all biological membranes. Glycerophospholipids (rigure 6-30) have two fatty acid residues esterified to the C-1 and e-2 cirbons ol glycerol. Esterified to the c-3 carbon of glycerol is a phosphate group which-bears a negative charge at pH 7.0. Three of the more common molecules that can be attached to the phosphoryl hpad group are ethanolamine (HocH2cHzNHg@), choline (HoCH2CH2N(CH3)3o), and serine. Glycerophospholipids u.e1*pniphilic in that they have nonpolar tails (the hydrocarbon chiins of the fatty acids) and a polar heads(the phosphate group and its associated attachment). Beca1,s" glycerophospholipids contain a phosphate group, they are also referred to as phospholipids.

Copyright @ by The Berkeley Review

l8

The Berkeley Review Specializing in MCAT preparation

Biology

Structure & Function in CeIIs & Viruses

Biological Molecules

Sphingolipids The sphingolipids are not based on a glycerol backbone. Instead, they are derivatives of amino alcohols. One common derivative is sphingosine. Attached to the C-2 carbon is an amino group that can be linked to a fatty acid through an

amide linkage. This molecule is called a ceramide (Figure 6-31,a), and is the structural residue that is common to all sphingolipids.
a phosphoethanolamine or a phosphocholine group is attached to the C-1 carbon of the ceramide, the sphingolipid becomes a sphingomyelin (Figure 631b), and sphingomyelins are abundant in the myelin sheaths that surrounds the axons of nerve cells in the nervous system.
(a)
H
I

s,c-on 'l
n

il

If

,L-"A^^.o
H"C- O-

I oo

'r-n-VT-tn'
cHl

6\l

CHr

(b)
H
(CH2)r2-

Figure 6-3O
Phosphatidylcholine.
(CHz)rz- CHr

C-3 HO- CH-

t= "HO
I

CHi

HO- CH-

f= HO
I

C-

c-2
H

"L-*A-H

o
il

c-l

Hz(F

OH

H,C'l
Ceramide

O- P- O-

CH,

CH2

NH.r

o^ v
Sphingomyelin

Figure 6-3 I
Different types of sphingolipids.

\Vhen a single monosaccharide like glucose or galactose is attached to the C-1 carbon of the ceramide, the sphingolipid becomes a cerebroside. Cerebrosides are the simplest of the sphingoglycolipids. The more complex sphingoglycolipids are the gangliosides. In this case several sugar residues (i.e., an oligosaccharide) are attached to the C-1 carbon of the ceramide (Figure 6-32). The extended oligosaccharide residues of a ganglioside are quite important as they are thought to be involved in recognition of other molecules at the surface of the cell. For example, specific gangliosides act as the antigenic determinants in the human ABO blood group system. Note that cerebrosides and gangliosides do ,:of contain a phosphate group. Cholesterol is synthesized in the cytosol and generally exists as the cholesterol ester. This lipid is a major constituent of eukaryotic animal plasma membranes and is an intermediate in the biosynthesis of all human steroids. It has a fairly rigid set of four fused nonpolar rings and a polar hydroxyl group that gives it a :Li ghtly amphiphilic attribute.
There are five major classes of steroids (Table 6-3), all of which are synthesized tn the mitochondrion, Synthesis of a steroid hormone begins with the hydrolysis of :holesterol esters in the cytosol and the subsequent transport of cholesterol into
Cholesterol

Figure 6-52
A typicai ganglioside.

Steroid Hormones
1. Progesterone 2. Glucocorticoids 3. Mineralocorticoids 4. Androgens 5. Estrogens

ihe mitochondrion. Adrenocorticotropic hormone (ACTH) stimulates the conversion of cholesterol to pregnenolone, an intermediate in the pathway of
steroid synthesis (Figure 6-33).
Copyright @ by The Berkeley Review

Table 6-5
Steroid classes.

l9

The BerkeleY Review Specializing in MCAT Preparation

Biology

Structure & Function in Cells & Viruses Biotogical Molecules


Later, when we discuss basic physiological processes, we will return to these steroids and discuss them in more detail. But for the moment, let's consider their general function.

it prepares the uterine lining for implantation of an ovum. If implantation occurs, this steroid is
Progesterone has a primary function in women where
necessary to maintain the endometrial lining of the uterus and hence maintain pregnancy. During pregnancy progesterone stimulates mammary tissue growth in preparation for parturition. Progesterone is also synthesized in low levels in the testes of the male and in the adrenal cortex of the adrenal glands (located on top of the kidneys) of both sexes.
Cholesterol
I

f
Pregnenolone

i
Progesterone

Cortisol is synthesized and secreted from cells in the cortex of the adrenal glands. In the liver cortisol acts to increase both glycogen sy'rthesis and gluconeogenesis. In skeletal muscle this glucocorticoid acts to decrease both glucose uptake and protein synthesis, and increases protein catabolism. In adipose tissue cortisol increases lipid mobilization and decreases glucose uptake. Cortisol is also known as hydrocortisone. Aldosterone is also synthesized and released from cells in the adrenal cortex. It acts to increase the reabsorption of sodium (Na@) at the level of the kidney, intestines, salivary glands, and sweat glands. The net effect is to cause retention of Nao in the extracellular fluid (ECF) thereby increasing ECF volume. Not only will this lead to an increase in blood volume, but it will also lead to an increase in blood pressure and blood flow.
Testosterone is synthesized in the male in the Leydig cells of the testes and aids in

Testosterone

"4\ tl
I

Cortisol

I
I

nto.,.t

"".,r," sperm maturation. This androgen can also reach the blood and circulates throughout the body, promoting a variety of biological effects, including development of the secondary sex characteristics
Bstradiol is the primary estrogen in women and is synthesized in the theca cells of ihe ovarian follicles. Estrogens have a variety of biological effects, including development of secondary sex characteristics, regulation of the ovarian cycle, and control of certain metabolic processes.

I
Estradiol Figure 6-33
Synthesis of the major steroids from cholesterol.

Copyright @ by The Berkeley Review

20

The Berkeley Review Specializing in MCAT Preparation

Biology
Nucleic Acids

Structure & Function in Cells & Viruses

Biological Molecules

Nucleic,,,Acids
Genetic information in organisms is carried in the form of nucleic acids. Nucleic

acids occur as either deoxyribonucleic acid (DNA) or ribonucleic acid (RNA). Both DNA and RNA are made from molecular units called nucleotides, and each nucleotide consists of (1) a nitrogenous base, (2) a pentose sugar, and (3) phosphoric acid. DNA and RNA are polynucleotides of high molecular weight.

Nitrogenous Bases
DNA contains four different nitrogenous bases. Adenine (A) and guanine (G) are referred to as purines while thymine (T) and cytosine (C) are referred to as pyrimidines. These bases are essentially planar and relatively insoluble in water. RNA also contains four different nitrogenous bases. The only difference between the bases of DNA and RNA is that in RNA thymine is replaced by uracil (U). The structures of these bases are shown in Figure 6-34.
NHz

*4.-.-*. 'it ;il 1) \* A'*,


i
H

HzN

t.\p
Guanine

*,,u oN

,*4. cHr -4
H

NHz

Adenine

Thymine

"^.)
HH
Cytosine

./-^j,l
Uracil

"*A

Figure 6-54
The nitrogenous bases of DNA and RNA.

Pentose Sugar DNA is 2'-deoxy-D-ribose while in RNA it is simply D-ribose (Figure 6-35). In order to avoid ambiguity between the numbering system in the nitrogenous bases and the numbering system in the ribose ring, the carbon atoms of the furanose ring are given a number immediately followed by a prime (') marking. Note that the C-1'hydroxyl group in both 2'-deoxy-D-ribose and D-ribose is in the B configuration.
The pentose sugar found in
5'

HOCH"
4'

OH

HH

l'
H

3'
HOH
2'-deoxy-D-ribose
HO

z',

D-ribose

Figure 6-55
The p-furanose rings of

DNA and RNA.

Phosphorus
The pH of a cell's cytoplasm is approxim ately 7.2. Phosphoric acid (H3Po4) is an

:mportant molecule in the cell and helps in the buffering of the cytoplasm. There ut" three hydrogen atoms which are capable of dissociating, each at a different :K, value (Table 6-4).
Copyright @ by The Berkeley Review

2l

The BerkeleY Keview Specializing in MCAT Preparation

Biology

Structure & Function in Cells & Viruses

Biologicat Molecutes

H3Po4
HrPOoo

: --i-i+

Hrpooo *
HpO42o

H@

pKa

= =
=

z.l j.2

+ H@

pKr

Ol o-P=O rO
o
I

HPO42o

PO43O + H@

pKu

12.7

Table 6,4

5'

rt

K"Jr V---y
9Hr

The three ionizations of phosphoric acid.

-o

ol o-

OH
f=

11

n--Glycosidic tinkage

T",

i",^ft,

,T, t" p-H of the_cytoplasrr-isT.2,so we set the pH in our equation at 7.2. atlo know which pKu value to use. It is thai pKu {1to value which is closest the pH value *e ut" given. we then solve the Henderson-Hasselbalch equation (6-1) for the rela"tive concentrations of the conjugate base (HPo42e) and weak acid (H2poae) in solution. we find that the ratios of the two species are equal (6-5).
equation. We know t .2 =

In order to determine this we need to use the Henderson-Hasselbalch

\A/hat is the relative concentration of free phosphoric acid in the cytoplasm?

t.2* ro, -[!!o?l


tH2PoJl

roo={gle?l
lH2poa)

1= trrPo?l tH2PoJl

(6-s)

why is it important to have an understanding of phosphoric acid? First, the molecule has an excellent buffering capaciiy. since ptro"phoric acid. has three pKu values, it means there are lhtel .egio's wheie the conjugate base (A) is going to equal the weak acid (HA). Iismall amounts of acid or base are added to any one of these three regions, the pH of the solution will essentially remain constant. This is what defines the buffering capacity of a solution.
0

.9o r00 to

backbone.
3'Ho
Figure 6-36
A polynucleotide sequence of DNA showing the bases, phosphodiester bonds, and polarity of the polymer.
H

second, phosphoric acid is a component of the nucleotide unit in both DNA and RNA. Each nucleotide is linked to the next by a phosphodiester bond. The backbone of DNA and RNA consists of itternating pentose sugars and phosphate groups. Since the hydrogen on the hydroiyi group of each phosphate residue has a pK3 close-to 3, we find that each phosphate in DNA and RNA has a negative charge associated with it, making the backbone quite polar. The nitrogenous bises that are attached to the sugar residues are hydrophobic and. direct them-selves awav from the )

DNA The short segment of single-stranded DNA shown in Figure 6,36 contains four nucleotide units and three phosphodiester linkages b'etween the ribose sugars. The nitrogenous bases are attached to the sugar residues by an Nglycosidic linkage. Both of these rinkages, the phosphodiester and the Nglycosidic, were formed by a pariicular dehydration reaction, and this means that those b""9:. can be hydrolyzed. Note that the N-glycosidic linkage is also an acetal linkage.

Copyright @ by The Berkeley Review

22

The Berkeley Review Specializing in MCAT preparation

Biology

Structure & Function in Cells & Viruses

Biological Molecules

The synthesis of a polynucleotide chain has direction or polarity. Each strand of nucleic acid will have a 5' end and a 3' end. Single strands of nucleic acids are (by definition) written with the 5' end at the left and the 3' end at the right. This is classically expressed in the notation 5'+3'. If we are discussing a double-stranded of DNA (i.e., a DNA double helix or duplex DNA), then the top strand polymer -of the duplex is written with the 5' end at the left and the 3' end at the right while the bottom strand takes on the opposite orientation, 3' end on the left and 5' end on the right. This can be represented by segment of DNA shown in Figure 6-37.

3.4 A

Hffi*rjfly'm ;:x,l'#o'I ! ,f [],r*:", ""4,{,4,{l


Daes

Figure 6-37
Duplex DNA showing antiparallel arrangement.

DNA double helix is joined to the other through interactions of the nitrogenous bases. Base pairing rules established by Watson and Crick allow for adenine and thymine to be joined by two hydrogen bonds and guanine and cytosine to be joined by three hydrogen bonds (Figure 6-38). As the two nucleic acld polymers wrap around each other in a helical fashion, the bases begin to
Each strand of a stack on top of one another, adding to the stability of the duplex'
H

(\(n-,.,,,,,,,? "ono
*,ri:Y

Hydrogen
Ribose

f", -

rr'v""'i"' arron'inn"""-"4-,,ll-) -,.1*,-^


i
Figure 6-38

rL _. o"" 'T^ir\
.,

N""",,,,,

/yo"',,,,,,,, **y

",,,,,,,,,,1X

""'T3\
t*^*'
*,Jo..

Ribose

Guanine

Adenine

Hi.og.n

bonding between adenine and thymine and guanine and cytosine.

i* r-o)'right @

by The Berkeley Review

25

The BerkeleY Review Specializing in MCAT PreParation

Biology

Structure & Function in Cells & Viruses

Biological Molecules

The structures of nucleic acids can become quite a pain to write out, especially if the nucleotide sequence is hundreds or thousandi of bases long. A number of different nomenclatures are used to solve this problem. we can let the bases be symbolized by their respective single letter abbreviations (e.g., G, c, A, & T). Each pentose sugar unit can be s)rmbolized by a aerticnl line and each phosphate by a p within a circle. Finally, we can place the 3' and 5' numerals on the vertical lines to indicate where the phosphate attachment takes place. This type of abbreviated nomenclature is shown in Figure 6-39.

5'

Figure 6-39
Abbreviated nomenclature for writing poiynucleotide sequences.

the 3' end. If we want to just consider the bases in a polynucleotide, the sequlnce on the right is the easiest and simplest to write. Based on this nomenclaiure, a DNA duplex should not be difficult to imagine.

It is understood that when we write a nucleotide sequence from the 5' to the 3' will be an unreacted phosphate group at the 5' end and an rrnreacted hydroxyl group at the 3' end. we can simplify our notation even more as shown below. Both sequences are identical to the nomenclature given in Figure 6-36. ht the sequence on the left it is understood that there is an unreacted hydroxyl at
end, there

pCpCpApT

GCAT

RNA As we have mentioned, RNA contains all of the same components as DNA with the exception that uracil is found in RNA instead of thymine and the ribose ring of RNA is hydroxylated at the 2'position whereas in DNA it is not.
There are three types of RNA, and all are slmthesizedby a process called DNA transcription. Each RNA transcript that is made carries out a specific function in the cell. RNA polymers that allow for the synthesis of proteinJare referred. to as

transcripts of messenger RNA (mRNA). The building blocks of proteins are amino acids. Amino acids are brought to the site of protein synthesis by an RNA polymer called transfer RNA (tRNA). The site of protein synthesis is the ribosome, which itself is composed of protein (i.e., amino acids) and RNA. In particular, the RNA polymers that help define the ribosome is ribosomal RNA (rRNA).
We will come back to both DNA and RNA in a later discussion and consider the reguiatory aspects that allow for DNA replication, transcription, and translation.

Copyright @ by The Berkeley Review

24

The Berkeley Review Specializing in MCAT Preparation

Biology

Structure & Function in Cells & Viruses

Eukaryotic Cells

Cel!ffi,ffii1i fddfibti;uil
Eukar5rotic Chromosomes
lvithin the membrane-bound nucleus of eukaryotic cells resides the majority of the genetic information needed for cellular growth and division. This genetic information is found in the nondividing (interphase) cell in the form of
chromatin, a complex of linear, double-stranded DNA, and protein (histones). As the cell prepares for division (mitosis and cytokinesis) the chromatin becomes

highly condensed into chromosomes.

:.umber of chromosome types. During fertilization the fusion of a male and :emale gamete produces a fertilized egg, or zygote, which now has a chromo-rorre Coffrplement of 46. Cells bearing this number of chromosomes are diploid ::om the Greek diplous, meaning double), or 2n. Human somatic cells (i.e., cells ',"'-nich are not gametes) are diploid in that they have 23 pairs of chromosomes. :ach pair of chromosomes is referred to as a homologous pair of chromosomes. -:.er- generally have similar shapes and sizes and carry the same hereditary traits ::'ntro11ed by segments of DNA called genes. Individual members of each pair i:- referred to as homologs.

gamete (eggs and sperm) is 23. There are22 autosomes and l sex chromosome either an X or a Y). Since each gamete has just one set of 23 chromosomes it is said to be haploid (from the'Greekhaplous, meaning single), or n, where n is the

Haploid & Diptoid In humans the characteristic number of chromosomes in the nuclei of

each

Sister Chromatids

" \L/ a"nttor"r"tf\

ftlt
s7

in the nuclei of non-dividing cells are not visible under the light ::_::oscope. However, when cells do begin to divide, either through mitosis or : =lrrsis, the chromosomes become highly condensed and can easily take up a :::-etr. of stains, making them quite visible under the light microscope. As we , -.- see in a few moments, the best phase of the cell cycle to view chromosomes -" ;-::hg metaphase.

^::omosomes

Metacentric

-, . rmportant

to remember that the chromosomes in metaphase have previously .: ':.:::.ted during the synthetic phase (S phase) of the cell cycle. These chromo, : ::.es have a characteristic size and shape that define the karyotype of the . r-:sm. In the human karyotype the chromosomes are numbered, ihe largest ,:-: :i homologous chromosomes being assigned the number 1 and the smallest -,-: 'leing assigned the number 22. These are the autosomes. The number 23 is ,-..,a;red to the sex chromosome.

,V

t\
/\

Acrocentric

Chromatids & Centromeres

,\J
Telocentric

:. r-. :rromosome arrested at metaphase consists of two sister chromatids, each -.-1 i..gether through a constricted region called the centromere. Individual -,---::::atids will become individual chromosomes when the centromere joining '--. :.sler chromatids divides and the chromatids are allowed to separate. By
;.--:,:ron chromosomes containing just one chromatid do not exist. This is an -: ::::ant point as it will help us understand exactly how much DNA there is ,-- i . r'.r- many chromosomes there are at any given moment in the life of a cell.

Figure 6-4O
General classification of eukaryotic chromosomes based on centromere position.

r:ht

O by The Berkeley Review

25

The Berkeley Review Specializing in MCAT Preparation

Biology

Structure & Function in CeIIs & Viruses

Eukaryotic Cens

The position of the centromere along the length of the chromosome allows for a general classification of eukaryotic chromosomes (Figure 5-40) as either being metacentric, acrocentric, or telocentric.

llistones & Nonhistones If all the DNA of all the chromosomes within a nucleus of a single human cell
were placed end-to-end, it would stretch to about 6 feet. How is it that all of this 6 x 109 base pairs, can be so successfully confined to the diploid nucleus of a cell and still retain its functionality?

DNA, roughly

The two major types of proteins associated with the structure of DNA are histones and nonhistones. The most abundant are the histones, which are basic proteins consisting of a high percentage of lysine (Lys) and arginine (Arg) residues. At physiotogical pH the side chain amino groups (-NHSO) of these amino acids bear a positive charge. An electrostatic relationship can be easily formed with the negatively charged DNA polymer. Nonhistones are proteins that associate with DNA, but are not histone proteins. Rather than being basic, these proteins are acidic and bear a net negative charge. A classic nonhistone protein is RNA polymerase. This enzyme synthesizes an RNA transcript from a DNA
template. There are five types of histones, designated as H1, HZA,IIZB, H3, and H4. The association of these histones with a specific length of DNA defines a structure referred to as a nucleosome. Each nucleosome repeats itself, on average, about every 200 base pairs (Figure 6-41). The region that separates each nucleosome is called the linker. There are about 30 million nucleosomes associated with the DNA in each nucleus.

K
DNA

B
Many packing
steps

s tl v s

A string of nucleosomes

Figure 6.41
Nucleosomes containing DNA and histones has the appearance of beads on a srring.

A replicated Figure 6.42

and

condensed chromosome
There are many different levels of packing that are presumed to give rise to the final, highly condensed chromosome

The core histones (H2A,F{ZB,I{3, and H4) bind roughly one-and-three-quarters turns of DNA, or about 146 base pairs. This association allows the length of the DNA to decrease by about a factor of six.

The H1 histone is though to play an important role in pulling the individual


nucleosomes together to create a 30-nm chromatin fiber. This fiber, which is about

L mm in length, is still much larger than the diameter of the cell's nucleus. Further packing of the DNA is needed, and this is where current speculation begins. It could be that there is an ever increasing number of coils until a particular higher-order structure of chromatin is established (Figure 6-42).

Copyright @ by The Berkeley Review

26

The Berkeley Review Specializing in MCAT Preparation

Biotogy
The Cell Cycle

Structure & Function in CeIIs & Viruses

Eukaryotic Cells

The Cell Cycle, Mitosis, & Meiosis


The cells of a eukaryotic organism complete a cell cycle for either reproductive, growth, or replacement purposes (Figure 6-43). The cycle occurs in both haploid and diploid cells and is defined as the sum of all events that occur between the completion of one cell division and the next. The rate of cell division varies between different cells, and some, like striated muscle cells and nerve cells, never divide. A typical eukaryotic cell cycle lasts anywhere from 16 hours to 24 hours. Let's briefly consider the events of a cell cycle.

The Phases of the Cell Cycle


The three major stages of the ce1l cycle are interphase, mitosis, and cytokinesis. Interphase can be divided into the G1, S, and G2 phases. Mitosis (M) is divided

urto prophase, metaphase, anaphase, and telophase. Mitosis is usually followed :r' cytokinesis, that portion of the cell cycle that allows for partitioning of the :ontents of the cytoplasm into two new daughter cells.

Figure 6'43
A generalized eukaryotic cell cycle.

tnterphase During the first growth phase (Gi), which lasts about 10 hours, RNA and :roteins are actively being synthesized. The centriole pair separates in :reparation for the synthesis of daughter centrioles in the S phase. The cell and ::s nucleus begins to increase in volume. If a cell is destined to never divide .gain, it will remain arrested in G1.
:ach of the 46 strands of chromatin, with the exception of the centromeres, are :eplicated by the end of the synthetic phase (S). This event lasts between 6 to 8 r.crurs. If we could visualize the chromatin at this stage, we would find that it ', ould be extended and loosely coiled. Even though the DNA has doubled to . -rm 92 sister chromatids, the chromosome number has remained the same at 46. l,aughter centrioles are synthesized at right angles to the parental centrioles. This
::ccess is completed by mitosis.

-:e
--

second growth phase (C2) lasts about 2 to 6 hours. During this phase the :omatin is beginning to condense and become more tightly coiled. Protein
cel1 prepares

i .:.ihesis is quite active and the

for mitosis.

![itosis
l,

titosis is simply nuclear division, and involves the equal partitioning of genetic a:erial to two new daughter cells. The phases of mitosis are outlined below.
Microtubules

Prophase -,.:he beginning of prophase the two centriole pairs begin to move apart. :-:iotubules begin to radiate from each pair in all directions, forming a star-like ,::-:.ture called an aster. The region from which the microtubules extend , -.:',.''ard is called the centrosome, or microtubule-organizing center (MTOC). .1.::otubules leaving the centrosome will eventually form the mitotic spindle, a :r:--::ture involved in the separation of the chromosomes during anaphase. By -= :nd of prophase the condensation of the chromatin is complete and the ,.-:-i:rlosomes, each containing a pair of sister chromatids held together at the :r irorrrr, can be visualized using different staining techniques. Many of the - ,::rtubules leaving the centrosome attach at the kinetochore (Figure 6-44), a ,:-:-alized area closely associated with the centromere. Other microtubules
: . rrght
@

Kinetoc
Metaphase chromosome

Figure 6-44
Microtubules attaching to the kinetochores.

by The Berkeley Review

27

The Berkeley Review Specializing in MCAT Preparation

Biology

Structure & Function in Cells & Viruses

Eukaryotic Cells

extend from the centrosomes to the equatorial region of the nucleus without
contacting the kinetochores. The nucleolus disappears and the nuclear membrane begins to break down.

Metaphase The fully condensed chromosomes align themselves along the equator of the cell, a region also referred to as the metaphase plate. By this time the nuclear
membrane has completely disappeared.

Centromere

\';?-*

XN\

]tr

5
(h

Sister

I-ate Interphase (2N)

Prophase (2N)

Metaphase (2N)

^^. I

Cytokinesis is complete and two new daughter cells are formed

Daughter chromosomes

if

_,

Nuclear envelope begins to form

Early Interphase (2N)

Telophase (4N)

Anaphase (4N)

Figure 6-45
Mitosis showing a nucleus containing four nonhomologous chromosomes. In this diploid cell, N
refers to the number of chromosomes.

Anaphase
The centromeres of each chromosome aligned on the metaphase plate diaide and

the two sister chromatids, which can now officially be called daughter chromosomes, move towards opposite poles. At this moment in the cell cycle we
should have a total of 92 chromosomes; 46 are moving towards one pole and 46 are moving towards the opposite pole. Movement occurs because of microtubule depolymerization at the region of the kinetochore. Cytokinesis, the cleavage of one cell into two daughter cells, usually begins late in anaphase.

Copyright @ by The Berkeley Review

2A

The Berkeley Review Specializing in MCAT Preparation

Biology

Structure & Function in Cells & Viruses

Eukaryotic Cells

Telophase ,"':lr the start of telophase all daughter chromosomes have reached their :=.:e:tive poles. Each chromosome begins to uncoil and extend itself. The ::-::olubules of the spindle apparatus begin to disappear as a nuclear membrane - :::r-s around each of the two daughter nuclei. The nucleolus (or nucleoli) :::::ears. The cleavage furrow of cytokinesis continues to deepen.

Cltokinesis
--"'::'srnesis simply involves the cytoplasmic division of a cell into two daughter Ti,e process generally begins during late anaphase and when it is complete, =i. ::.a j the end of mitosis (or meiosis). Mitosis is outlined in Figure 5-45.
:

!!eiosis :- -:::r.: diploid cells (2N) which are destined to become haploid gametes (N) --:::;r DNA replication and then two successive nuclear divisions. This ::::-:s rs called meiosis and the formation of the gametes is called
;.;:-.etoqenesis (discussed in the section onReproduction and Deaelopment). In the : r l-: --i.3 gametes are called eggs (ova) and are produced in the ovaries. The :".i.: .::-:e:es are called sperm (spermatozoa) and are produced in the testes.
"

: I . successive nuclear divisions in meiosis are called meiosis I and meiosis : , --r :,erotic divisions follow the same four phases in mitosis: prophase,
r.i r

' l--: i

r:-.e. alaphase, and telophase. In meiosis we distinguish those phases by --:.e Roman numeral I or II after the phase name (e.g., prophase I or :-;. IIi. Meiosis I and meiosis II are each preceded by an interphase. The ',r-:-, --: -r,terphase before meiosis I proceeds through the familiar G1, S, and - ;.':'- -';. During the S period the DNA replicates and during the G2 period the :'-:, ==i'.: to meiosis takes place. After the first nuclear division one cell with .": :.::orLes becomes two cells, each with 23 chromosomes. This is ' r' : i-i-:i :alied a reductive division. Each of these two cells will enter into the :-r -::.: ::-.at precedes meiosis II. During this second interphase the S period '" - .: : '-.: and so the DNA cannot be replicated again. After these cells leave r:1-- - '-nev enter into meiosis II. At the end of the second nuclear division :--r r=rr.iln, each with 23 chromosomes. What we would like to do is : :: j ::,e events of meiosis at a basic level and then, during a later
:

""r

Prophase I s::ge of meiosis (Figure 6-46), prophase I is quite long and ::r,pared to prophase of mitosis. Prophase I can be divided into -:lc:ene, zygotene, pachytene, diplotene, and diakinesis. Each of : -:.:ains terminology which is important to meiosis and important : troughout prophase I we will find 46 chromosomes or 92
- r- -^tl -e11.

-,sptotene. The replicated


'-

chromosomes have already started to

:::*... a:-Li nolv become visible. If we wanted to, we could assign 23 of -' ,:: - i - rr,-rSorrlS (i.e., 46 chromatids) as being maternal and 23 as being -l:,
I- :

-:. --,ilglrl.

t]]ilrni

fl

":"'

l'-:

Berkelev Review

29

The Berkeley Review Specializing in MCAT Preparation

Biology

Structure & Function in Cells & Viruses

Eukaryotic Cells

Zygotene. Homologous chromosomes begin to pair up longitudinally in preparation for crossing-over (genetic recombination). This is referred

to as synapsis. A specialized protein and RNA scaffold called

the

t
Chromatin

hemeiotic
Interphase

Synapsis

Prophase (2N)

(2N) centromere

synaptonemal complex (Figure 6-47a) appears between the pairing chromosomes and facilitates their union. The homologous chromosomes which have undergone synapsis are referred to as a bivalent, and because there are four chromatids in a bivalent the paired homologous chromosomes are also referred to as tetrads. How many tetrads are there at this stage? Only 23. How many centromeres are there in each tetrad? Just two. How many chromosomes would we find? Based on the number of centromeres, we would fl::.d 46. How many pairs of chromosomes are there? We could count 23 pairs at this stage.
(a)

D;. ads are

(b)

p'.rlled r: poles

TzlS *P,
\TT

,,:*ffir.r\wl;,:r:
-fP=A.> | c"nro^rK ,Kn ol
Figure 6,47

K.w-="::l:'*..
-\\;!--,
chiasma

Microtubules-1l-

t\\ri,v?
Anaphase

F,r\ l/,+
(2N)
Nuclear envelope begins to fonn Cleavage furrow
deepens

Formation of the synaptonemal complex between a pair of homologous chromosomes.

Pachytene. The chromosomes continue to condense and become much more distinct. Genetic recombination occurs through a process called
crossing-over.

il

Diplotene. As the homologous chromosomes begin to

separate, the

events of crossing-over become visible at structures called chiasmata (singular: chiasma). At each chiasma any one of the four chromatids may be involved in a cross-over event. For each bivalent, there is an average of about 10 cross-overs, and this leads to a very thorough mixing of the maternal and paternal chromatids.

/ I

Two new daughter


cells are formed

In the section on Reproduction snd Deuelopmenl we learned that the developing eggs (oocytes) in a female are arrested in diplotene of prophase I by about the seventh month of gestation (fetal development)Those eggs wilt not be able to continue their meiotic adventure until the onset of puberty. Even then just one egg per month will have the capacity to develop and be fertilized. The rest will be left in diplotene,
some residing in that state for more than 50 years.

Meiotic Interphase (N)

Diakinesis. While the nuclear envelope begins to break down and the nucleoli disappear, the chiasmata move along the lengths of the chromatids until they reach the ends. As the homologous chromosomes
begin to separate they appear to be joined to each other at their ends.

Figure 6-46
Meiosis I showing the different stages that lead to the formation of two haploid cells.

Metaphase I
The homologous chromosomes align on the metaphase plate. The centromeres of each pair appear to be quite distant from each other. It seems as if the only parts

of the chromosomes touching one another are the ends. Microtubules from the spindle apparatus are attached at the centromeres and the nuclear membrane has
disappeared.

Copyright @ by The BerkeleY Review

30

The BerkeleY Review Specializing in MCAT Preparation

Biology

Structure & Function in Cells & Viruses

Eukaryotic Cells

Anaphase I As the microtubules puII the homologous chromosomes apart, the centromeres do not divide. The pull on each chromosome is from one direction only. Each chromosome that migrates toward a pole is still composed of sister chromatids and this pair is referred to as a dyad
(Figure 6-47b). Cytokinesis beings.

@*;*@ + +
(N)

Telophase I The migrating chromosomes (dyads) complete their movement to


-rpposite poles and the nuclear membrane reforms. Cytokinesis is more :ronounced.

'rZAr\'/d"*\\\ \lt \*'ll'll \\\tr"b/1 \\\\ /-r',/ \\)i$:j-l-,

/z=A\\

G+N
Prophase

Cytokinesis -fter cytokinesis is complete we find that a dipioid cell with :lromosomes has divided into two haploid cells, each with
-:-.rs

46 23

\'\iFZl

N\\="94 \\\\ /-.7 / tZ)

(N)

-jlromosomes. Since there is a reduction in the number of chromosomes,


is aiso called a reductive division.

7N\i

Interphase II
,:.e second (meiotic) interphase is usually brief. DNA repiication ..,ie place during the S phase of this interphase.
does not

'leiosis II: Prophase II , - - stages of meiosis II (Figure


:.'-:.'rsis. As

\\4 I
+

,fl, N\ \ \\\

',trifl*)r,
Metaphase

,17,

(N)

{4,t

6-48) are quite similar to the stages of in prophase of mitosis, the chromosomes in prophase II of -,osis II begin to condense. The microtubules of the spindle apparatus

,: :--h to the kinetochores of each chromosome.


trfetaphase II '.,.mosomes line up along the metaphase plate as the haploid cells : :::3re for division.

#N
(2N)

Zzf;N

Anaphase

{naphase II
,-:',e centromeres begin to divide the chromatids of each of the 23 ,:rlosomes are pulled to opposite poles of the dividing cell. : -<rnesis is in progress. The chromatids which separate from one - :-'.er can officially be called chromosomes. There are now 46 r ::'.osomes in the dividing cell. An interesting technicality arises -,--:: anaphase IL The chromatids of each chromosome that are : , :::tng cannot be referred to as sister chromatids. Why? Think about , -i-:ierence in DNA between the replication of the chromatin during ' : rSi interphase and the events of crossing-over during pachytene of ' ::.:.se I of the first meiosis. The DNA is no longer the same as there , ir::r1 genetic recombination. -:

rfi \li \ifl A# \\#2 I

..

r\W2,,

(:W,t

-:lophase II
..

Telophase

(2N)

--=ar membrane forms around the decondensing chromosomes. The ,r t furrow created by cytokinesis deepens.

@@@@
Haploid Gametes (N)

't,rkinesis
four haploid cells, each having 23 ' . - somes, will have been derived from one diploid cell. Each of -: , -ir ce11s enters into interphase where they are arrested at the G1 - - lhese cells will remain quiescent until fertilization takes place.
:,- :.-.e completion of cytokinesis
O by The Berkeley Review

Figure 6-48
Meiosis II.

The Berkeley Review Specializing in MCAT Preparation

Bionogy

Structure & Function in Cells & Viruses

Eukaryotic Cells

Asexual & Sexual Reproduction

In asexual reproduction, a new organism develops without a sexual process, either from a single cell or a group of cells. It can occur in either haploid or diploid organisms. The offspring produced are identical to the parents. \Atrhat is an example of a eukaryote that produces asexually (Figure 6-49)? Yeast.
Animals that reproduce sexually have two types of cells: germ cells and somatic cells. As we have seen, haploid germ cells are produced by the process of meiosis while diploid somatic cells are produced by the process of mitosis. \Atrhen haploid gametes from a male and a female fuse together, a diploid zygote is formed. This process, termed sexual reproduction, involves the alternation of diploid and haploid phases of an organism's life cycle. It occurs only in diploid organisms. The offspring that rcsult are nof identical to the parents due to extensive genetic recombination. What is an example of a eukaryote.that produces sexually (Figure
6-50X Humans.

n Sperm ftil

fA)
t_UJ

fure

6-a9

Srmrmm"! oi asexual reproduction.

W1 f ;*,3tU

fl[--1[*$\
Figure 6-5O
Summary of sexual reproduction.

ilQ..,.*wrr,/^t8 Tt-*'
chromosomes

r0r

ffi \a/B ffi' c11 o


Meiosis

Copyright @ by The Berkeley Review

32

The Berkeley Review Specializing in MCAT Preparation

Biotogy

Structure & Function in Cells & Viruses

Eukaryotic Cells

fldtr,ebu[ iii:OfghiliZatidil
Biomembranes & Membrane Transport
The major components of both eukaryotic and prokaryotic membranes are lipids, proteins, and carbohydrates. Even though there will be differences in the specific

molecular components of eukaryotic and prokaryotic membranes, we will find that their structural organization is similar. in both cases we will see membranes that are dynamic.

Biomembranes
Glycerophospholipids, sphingolipids, and cholesterol are the three major lipid components of eukaryotic biological membranes. Amphiphilic molecules like the

H,c-oA.'v-

lo

glycerophospholipids and sphingolipids present a shape similar to that of a rectangle. At one end of the rectangle the molecule is polar (hydrophilic) while at the other end it is nonpolar (hydrophobic). In biological membranes the two most abundant glycerophospholipids are phosphatidylcholine (Figure 6-30) and lhosphatidylethanolamine (Figure 6-51). We can let these amphiphilic molecules be represented by the drawing shown in Figure 6-52. The hydrophobic tails of :he phospholipids represent the fatty acyl side chains containing the methylene *roups while the polar head is represented by the phosphate group and its associated molecular attachments.

"LH2C-

"A--^,^ O?_o{.", oo

Figure 6.5 I
Phosphatidylethanolamine.

Lipid Structures
The three structures that can be formed by phospholipids are micelles, lipid :Llayers, and liposomes. Micelles are spherical structures that are formed when =;rough phospholipids congregate together such that the polar heads interact

,",-ith water while the hydrophobic tails exclude water (Figure 6-53a). In our :iscussion on gastrointestinal physiology, we will see how bile acids form :rrcelles and act to solublize fats during digestion.

Polar

gl
Figure 6'55
l:oss sections of a (a) micelle, (b) lipid bilayer, and (c) liposome.

Head

Hydrophobic

Tail

Figure 6-52
General representation of a

phospholipid.

Lipid bilayers are formed when the hydrocarbon tails of two phospholipid .:.eets interact with one another to exclude water (Figure 6-53b). These structures ::e stabilized through hydrogen bonding and electrostatic interactions of the ::ad groups with water and by the hydrophobic van der Waals interactions --:trveen the hydrocarbon side chains. The average thickness of a lipid bilayer is :out 60A (or 6 nanometers). if a lipid bilayer folds back on itself, a hollow ::ueous-filled structure will form called a liposome (Figure 6-53c). Lipid Mobility -:.e individual phospholipids in the plane of a bilayer have a great deal of :.obi1ity. Neighboring phospholipids can easily exchange places by a process -: led lateral diffusion. However, movement of a phospholipid from one lipid : -ane to the next, a process called transverse diffusion or flip-flop, is a very rare
=
"

ent (Figure 6-54).


pr right O by The Berkeley Review

.'J

The Berkeley Review Specializing in MCAT Preparation

Biology

S,ttnncture Er Function

in Cetls & Viruses

Eukaryotic Cells

mf''
lr -:.; -::-s:,rn 1

Phospholipids that have shorlcharn fatty acids and fatty acids with increasing sites of cis unsaturation (i.e., double bonds) tend to increase their mobility (i.el fluidity) in the membrane. The greater the motion of the fatty acid side chains,
decrease fluiditr-. The planar.steroid ring inserts between neighboring fatty acid side charns and interferes with the movement of those chains."Memfrine Rliaity

the more fluid the membrane. Addition of cholesterol to a'membrane acts to

ats
,itllll

:"St]

membrane

is also dependent on temperature. Cooler temperatures act to


iluidity while higher temperatures
act to increase it.

decrease

_,_rJluuu

mffifif\ffin

Membrane Proteins A r-arietv of different proteins can be associated with the lipid bilayer. However, each of these proteins will have an operational classification. For example, integral membrane proteins are embedded in the lipid bilayer. These proteins can either span the bilayer, in which case they aie called trans-membrane proteins, or they can be embedded in the membrane and exposed at just the exterior surface or interior surface of the bilayer. Integral membrane proteins are tightly bound to the lipid bilayer by hydrophobic forces and harsh uru "o.,ditions needed to remove them. once removed the membrane is usually disrupted.
Peripheral membrane proteins are weakly attached to the surface of the lipid bilayer, either through hydrogen bonding or electrostatic associations. Removal of these proteins occurs under mild conditions and the membrane is usuallv left intact. In Figure 6-55 we see a schematic diagram of these general classls of
proteins.

[uuruus

Glycolipid
Integral

Figure 6-54
Diffusion of phospholipids in lipid bilayer.
a

Figure 6.55
A schematic diagram showing the relationship between integral and peripheral proteins.

Glycolipids & Glycoproteins


If a membrane lipid is attached to a carbohydrate, it is referred to as a glycolipid. The majority of glycolipids are found o.r ih" exterior surface of a lip"id bilayer. Carbohydrates that are attached to membrane proteins are called glycoproteins.

WU

fiq

sugar residues are found attached to proteins that face the exterior of the membrane. one important class of glycolipids and glycoproteins that we will be discussing in a later section are the human blood group antigens that make up the A, B, and O blood groups.

All

ti
l:ir,'gM
wl,..,r[1

lm

Membrane Transpotr
The cell membrane acts as a permeability barrier that selectively allows molecules to enter and exit the celi. There are five processes by which this can occur:

,41
M

lIWH

dhl*
"!F}tm

(1) simple diffusion, (2) facilitated diffusion, (3) active transport, (4) group

3 by The Berkeley Review

34

The Berkeley Review Specializing in MCAT preparation

Biology

Structure & Function in Cells & Viruses

Eukar5rotic Cells

-:"nslocation, and (5) bulk transport. Let's take a brief look at each of these types -: rembrane transport.

Simple Difftrsion --:.-:irer name for simple diffusion is passive diffusion. Simple diffusion -- , ,:-r-es the spontaneous movement of solute molecules through a lipid bilayer, r: l. an area of high concentration to an area of low concentration, down a :,:.:entration gradient. The relative rates of equilibration of a few molecules r :r-:s a lipid bilayer is shown in Table 6-5. Fa,cilitated Difftrsion ln;rliiated diffusion (or carrier-mediated diffusion) is similar to simple diffusion ,- --:.1: rt also takes place down a concentration gradient. However, in this case ',,-:::',olecules cannot simply pass through a lipid bilayer. Instead, diffusion -=:=::s on the interaction of the solute molecules with integral membrane r.i r. r;-rLS :hat are embedded in the lipid bilayer. If one type of solute molecule ; lr::r -:r:ough a transmembrane protein in one direction, the transporter is r-,:: : uniport. If two different types of solute moiecules pass through a r':rrrrrr-rc rn the same direction, the transporter is called a symport. If a :,--.:: -::er al1orvs for the passage of two different solute molecules through a -r :_ri;i,e in opposite directions, it is referred to as an antiport (Figure 6-56). :. --.-- ::ar*<porters that allow for the movement of a solute across a membrane ; : - . --;rili'lr- called permeases. Symports and antiports are also referred to as :;a::sporters. Both facilitated diffusion and simple diffusion are examples of a
r.-

Solute/Diffusion Rate
Hydrophobic

o,2,N2,coz
Diffuse very fast Polar Uncharged (small)
Water, Urea, Ethanol

Diffuse fast Polar Uncharged (large)


Glucose, Galactose, Fructose

Diffuse

s1ow1y

Ions
yu@, g@, gu2@

Diffuse very slowly

Table 6-5
Lipid solubility of select solutes.

r ,1rli.

"{

e t:ansport system.

Antiport
$il$flmq rl" 56
,
. rli

|:

"

,:..it

, -::'::,:::i:t.lr oi three

different types of transport systems.

4|nnthrw'e

1flr-alrsport

. --.,
,rrriiil

"':

: -: -::
.':

::ansported against its concentration gradient, energy -: --=--.: :


"': ::.

will be :--:n or another. We will divide active transport into primnry

'rlirrr' rr'r-lr

::--iidnry

ACtiae trAnSpOrt.

"' i-llrm.;l*l a:C.-e transport the classic energy source is adenosine triphosphate -":i :l .r :-*= :-:-.sr. erample involves the Na@-K@ pump.This pump is also 1il,l ,rl'i ri i:i -*. \a3-K3 ATPase and its primary responsibility is to set and Illr ;-l -'"i: --it::-l1.ilar concentrations of Na@ and KO. The concentrations of ttlrilrl : - - :.-.---:. -.ns are quite different between the intracellular and 'r l:*iiL i I r.-i,: :,.; - :.-. c,: ne ce1l (Table 6-6). The intracellular K@ concentration is
ilrLiii;iui: . ,,

tllllllil

t'

nr"l

.'5

The Berkeley Review Specializing in MCAT Preparation

Biology
llnsidel

Structure & Function in Cells & Viruses

Eukaryotic Cells
K@

about 140 mM while that of Na@ is about 5 mM to 15 mM. The extracellular concentration is about 5 mM while that of Na@ is about L45 mM.

lon (mM)
Na@ 5-15 6@ t4o ca2@ l-2

IOutside]

(mM)
145
5

3-5 110

The imbalance of these two ions across the plasma membrane of a cell is maintained by the Na@-K@ ATPase. This_pump is an antiport that transports two K@ ions into the cell for every three Na@ ions out of the cell (Figure 6-57). The energy required for this transport comes from the hydrolysis of ATP to ADP and li (iiorganic phosphate). It has been calculated that in nerve cells about 70% of the cell's energy is used to maintain these pumps.
Note that as three Na@ ions move out of the cell and two K@ ions move into the cell, a net separation of charge (an electrical potential) is established across the plasma *"*btut". The inside of the cell becomes negatiue with respect to the Lutside. The membrane potential that is established is generally in the range of -50 millivolts (mV) to -70 mV. We will come back to this point during a later
discussion.

cro
Table 6-6

Common ionic concentrations inside and outside the cell.

Another important membrane pump is the Ca2o ATPase' This protein ensules that the Ca2b concentration within the cell is always at a low level by pumping two Ca2@ ions out of the cytosol for every ATP hydrolyzed. Ionic gradients that have been established by primary active transport systems can piovide a driving force that allows for the cotransport of other molecules against their concentration gradients. This process is called secondary active tiansport and a classic e*arttple involves the cotransport of Nao with glucose into specialized intestinal or kidney cells (Figure 6-53). The symport protein that facilitates this process has two binding sites; one for NaO and one for glucose' Both need to be occupied in order for the translocation to occur. The transport of Na@ into the cell o."irc because of (1) the concentration difference of Nao across the cell's membrane (high outside and low inside) and (2) the attractive net negative charge inside the cell. Once Na@ and glucose are brought into the cell, Nab is pnttlp"a out through the Na@-K@ ATPase. This helps to reestablish the electroclhemical gradient across the plasma membrane. Other secondary active transport systems cotransport Nao and amino acids into the cell'

Figure 6-57
The Na@-Ko ATPase.

Glucose

Na@

Group TYanslocation

Examples of group translocation can be found in certain bacteria. In this process u ,rrgu, residire liie glucose is phosphorylated as it is being transported through the ilasma membrane. This type of transport is coupled to cellular metabolism. We will return to it when we examine the regulation and metabolism of lactose.

Bulk TransPort Many animil cells will show an invagination of a portion of their plasma *"rrrbrunu that will eventually pinch off to form an internalized vesicle' This is
Figure 6-58
An example of secondarY active
transport.

called endocytosis and the vesicles that are formed are called endosomes or endocytotic vesicles. Within these vesicles is a portion of the extracellular environment. If the invagination resulted in a vesicle that contains the liquid portion of the extracellutai environment, the process is referred to as pinocytosis ior cell drinking). F{owever, if the vesicle contains some type of particulate matter, the process is referred to as phagocytosis (or cell eating). As these vesicles move toward the interior of the cell some will fuse with others to form larger structures. Material can also be released from the cell through a plocess .utLa exocytosis. Endocytosis and exocytosis are both examples of bulk transport.

Copyright @ by The BerkeleY Review

36

The BerkeleY Review Specializing in MCAT PreParation

Biology
SucIeus

Structure & Function in Cells & Viruses

Dukaryotic Cells

l\ucleus, Nucleolus, & Kibosomes


.:.e most prominent structure in a eukaryotic cell is a double membrane-bound ::sanelle called the nucleus (Figure 6-59). The inner membrane of this nuclear =:.-,-elope surrounds the majority of the cell's genetic material (DNA). The :enLaining genetic material in an animal cell can be found in the mitochondria. l:.e outer membrane becomes part of an extensive membranous system found in re cytoplasm called the rough endoplasmic reticulum (RER). Between the inner ::.d outer membrane is the perinuclear space.

).rrrng interphase of the cell cycle the DNA is referred to as chromatin, and the :-irture of chromatin and the aqueous phase of the nucleus is referred to as the nucleoplasm. During mitosis the extended chromatin begins to condense into :ghtly coiled structures called chromosomes. We will see that within the nucleus
is involved in two important events. The first is DNA replication while the .econd is the transcription of DNA into messenger RNA (mRNA). Translation of nRNA into protein (i.e., protein symthesis) occurs in the cytoplasm of the cell.

fNA

Figure 6-59
The nucleus.

Scattered throughout the nuclear envelope are thousands of nuclear pores with :hannel diameters that range from 10 nm to 20 nm. These pores are areas where :he inner and outer nuclear membranes come together to form passage ways that allow for the two-way flow of selected materials between the cytoplasm of the cell and the nucleoplasm of the nucleus. Since protein synthesis cannot take place

-n the nucleus, all nuclear proteins found within the nucleoplasm must be imported from the cytosol. The exact mechanism that allows these proteins to be transported into the nucleus us unknown, but it has been postulated that the
transport process is energy-dependent and involves protein signal sequences. The major type of protein associated with nuclear DNA are histones. These proteins are rich in the amino acids lysine and arginine. At physiological pH these basic amino acids bear a positiae charge on their side chain and can associate rvith the negatiuely charged DNA through electrostatic linkages. Histones are thought to be involved in DNA folding and the highty condensed chromosomal
structures observed during metaphase of mitosis.

Nucleolus
Within the nucleus is a highly organized region called the nucleolus. The nucleolus, which is not a membrane-bound organelle, is centered around certain chromosomes that contain nucleolus organizer regions and is involved in the

synthesis of ribosomal RNA (rRNA). The rRNA which is transcribed from specific genes in the DNA of the nucleolus organizer regions associates with ribosomal proteins in the nucleoplasm. Together the ribosomal proteins and rRNA form two different ribosomal subunits, one small and one large, that are then transported out of the nucleus through the nuclear pores and assembled in the cytoplasm into complete and functional ribosomes that carry out translation. If a cell is quite actively involved in protein synthesis, one would expect the nucleolus to be larger than if a cell were not as actively involved in protein
synthesis.

Ribosomes
Eukaryotic ribosomes are the sites of protein synthesis. They are composed of two subunits, each differing in size and content of RNA and protein. The size of a
Copyright @ by The Berkeley Review J/

The Berkeley Review Specializing in MCAT Preparation

Biology

Structure & Function in Cells & Viruses

Eukar5rotic Cells

mass (i.e., molecular weight). The sedimentation coefficient of a complete eukaryotic ribosome is expressed as 80S.

ribosome and each of its component parts is based on a sedimentation coefficient, s, that is expressed in svedberg units (s), where one s = 10-13 s. The rate at which a molecule sediments in an ultracentrifuge tells us something about its

If we separate this ribosome into its two component parts (Figure 6-60), we find that the large subunil sediments at 605 while the small subunil sediments at 40S. These values are not additive. In other words, 605 + 40s does not equal \00s. Sedimentation coefficients are not linearly related to molecular weight as they
80s Ribosome

depend on the size and the shape of the molecule.

{,

The overall dimensions of a complete ribosome is about 20 nm by 30 nm and contains roughly 60% rRNA and 40h protein. The small subunit is about 9 nm in diameter and contains roughly half rRNA and half protein. The large subunit is roughly 25 nm in diameter and contains about 65% rRNA and 35% protein.
The point here is not to memorize values but rather to think about dimensions and content. We just learned that ribosomal subunits are transported from the nucleus to the cytoplasm where they are assembled into complete and functional ribosomes. Based on what we have discussed we would e*pect the small subunit to pass through a nuclear pore with relative ease. The large subunit seems to be at least as large as the diameter of a nuclear pore. what would need to happen to allow for its passage into the cytoplasm?

605 Subunit

rRNA
Protein
40S Subunit

Figure 6-6O
The components of a eukaryotic ribosome.

What are the two major types of biological molecules contained in a ribosome? Nucleic acids and amino acids. \zVhich nucleic acids are directly associated with ribosomes? In the section on Expression of Genetic Information we will learn that during protein synthesis ribosomes are associated with IRNA, mRNA and transfer RNA (IRNA).

In any eukaryotic cell that is involved in protein synthesis there are hundreds of thousands if not millions of ribosomes within the cell. In eukaryotic animal cells ribosomes are not only found floating free in the cytoplasm, but they are also found associated with a membranous structure called the endoplasmic reticulum. what determines if a ribosome remains free in the cytoplaim or is bound to the endoplasmic reticulum? It depends on what happens during the initial stages of protein synthesis. All ribosomes involved in the translation of nuclear genetic information begin polypeptide synthesis in the cytoplasm. If a particular ribosome is destined to be bound to the endoplasmic reticulum, then during the initial stages of protein synthesis a short signal peptide is made which directs the ribosome to the endoplasmic reticulum. This model, called the signal hypothesis, will be discussed shortly.
Ribosomes are also found in the matrix of the mitochondria. Ribosomes found in the mitochondrial matrix not only differ in RNA and protein content from those found in the cytoplasm, but they are also smaller and sediment at about 55S. This

smaller sedimentation coefficient is close to the sedimentation coefficient of prokaryotic ribosomes, which is about 70S.

fr

Copyright @ by The Berkeley Review

3a

The Berkeley Review Specializing in MCAT Preparation

Biology

Structure & Function in Cells & Viruses

Eukaryotic Cells

Endoplasmic Keticulum & Golgi Apparatus


Endoplasmic Reticulum
The endoplasmic reticulum (ER) is a network of membranes that is found within the cytoplasm of most eukaryotic cells and is continuous with the outer nuclear membrane. The space enclosed by this membranous system is referred to as the lumen. The ER can either be smooth, in which case it is called the smooth endoplasmic reticulum (SER), or it can be studded with ribosomes, in which case it is called the rough endoplasmic reticulum (RER). The ER is the largest membrane system in a eukaryotic cel1.

Smooth Endoplasmic Reticulum


The SER has a membrane system that lacks ribosomes and appears more tubular

in

shape. The SER

is involved in the synthesis of a majority of the cell's

membrane lipids, including the neutral fats, phospholipids, prostaglandins, and steroid hormones. Specific integral membrane proteins embedded in the SER act as enzymes that help catalyze these reactions.

The SER, especially of hepatocytes (liver cells), is involved in hydroxylation reactions that aid in the detoxification of drugs. Hydroxylating a compound makes it more water soluble, thereby making it easier to be eliminated from the
body.

In hepatocytes the SER also plays an important role in the catabolism of liver glycogen. Recall that glycogen is a storage polysaccharide consisting of many ;lucose residues linked together in o(1-+a) and u(1+6) linkages. If blood glucose -evels fall below normal values, a hormonally mediated set of events occur that allow for the breakdown of glycogen and the release of glucose. One of those :i-ents involves the enzyme glucose-6-phosphatase. As glycogen is being broken iown, a molecule of glucose-6-phosphate will eventually be produced. The :1zyme glucose-6-phosphatase is embedded in the membrane of the SER and :atalyzes the removal of the phosphate group at the C-6 position of glucose-6::.osphate. The desired product is glucose, as shown in (6-6). Because glucose is a eutral compound (i.e., it no longer has any negative charges due to the :rosphate group) it can readily pass from the cytoplasm of the hepatocyte, --:ough a permease in the plasma membrane, and into the blood.
G luc o s e - 6 -p ho s p

hatas e

Glucose-6-phosphate + Hzo

Glucose

P;

(6-6)

-- :ertain cell types the SER regulates Ca2@ levels. For example, in muscle cells : = SER is referred to as the sarcoplasmic reticulum (SR). The SR sequesters Ca2@ .-.s and when stimulated by a nerve impulse releases those ions into the : .:plasm of the muscle cell. The result is a contraction of actin and myosin ' -:rents in the muscle cell.
Kough Endoplasmic Reticulum -- = RER has a membrane system that is generally flat and sheet-like. Ribosomes ,-': line the cytoplasmic face of the RER are bound to the membrane by their ::;e (605) subunit. The ribosomes synthesize membrane and secretory proteins r':: are then passed through the membrane of the RER and into the lumen where
:reht @ by The Berkeley Review

39

The Berkeley Keview Specializing in MCAT Preparation

Biology

Structure Er Function in CeIIs & Viruses

Eukaryotic Cells

post-translational modification begins. In other words, after synthesis of the polypeptide chain by the ribosome, certain amino acids are modified by hydroxylation and glycosylation events. After a brief period of time in the lumen
of the RER, these proteins are shuttled to the Golgi apparatus (see below) where post-translational modification continues. These modified proteins are directed to their final destinations by the Golgi apparatus.

Signal llypothesis
What is the general mechanism that accounts for the passage of proteins into the lumen of the RER? Recall that proteins encoded by nuclear genes begin the initial stages of their synthesis in the cytoplasm of the cell. Some of these proteins have at their N-terminus a sequence of amino acids that act as a signal sequence which binds to a signal recognition particle (SRP) in the cytoplasm, Translation is temporarily halted as the SRP directs the large subunit of the ribosome to a signal sequence receptor embedded in the membrane of the RER. Once the ribosome docks at the membrane receptor protein, polypeptide synthesis continues and the signal sequence is inserted through the membrane of the RER. The released SRP is free to bind to another developing signal sequence in the cytoplasm. Meanwhile, the signal sequence that was passed into the lumen of the RER is cleaved and degraded by the enzyme signal peptidase. Modification of the growing polypeptide chain begins. After completion of protein synthesis, the ribosome dissociates from the RER (Figure 6-61). Non-membrane proteins synthesized by the ribosome are released into the lumen of the RER. These proteins are usually destined for secretion or incorporation into lysosomes. Other proteins may span the membrane of the RER, becoming integral membrane proteins that are destined for the plasma membrane.

Signal
sequence

-) SRP.-----=
@

OL--J
\
a
Ribosome

7--.\ ,'

6os

5'

4os

,.qi.n..

o*(, 'r**ffi'oo" J,,, potypeptide tlW


RER

Lumen

Modified protein - ---

?^

Figure 6-61 Synthesis of secretory proteins in the lumen of the RER. O Ribosomal subunits associate on mRNA. @ Signal sequence leaves large subunit. O SRP binds signal sequence and ribosome.
Protein synthesis stops. @ Ribosome-SRP complex binds SRP receptor on RER. SRP dissociates and protein synthesis starts. Signal sequence guides growing polypeptide into RER lumen. l Signal peptidase cleaves signal sequence. @ Polypeptide continues to grow. @ After protein is formed ribosome dissociates and is recycled. @ Protein modification continues.

Copyright @ by The Berkeley Review

40

The Berkeley Review Speciatizing in MCAT Preparation

Biology
Golgi Apparatus

Structure & Function in Cells & Viruses

Eukaryotic Cells

\lost proteins synthesized in the RER are transported in small vesicles to the
Golgi apparatus, a complex of flattened membranous sacs called cisternae. The cisternae of the Golgi complex (Figure 6-62) is divided into three distinct regions, each containing an environment that has a relatively neutral pH. The cis cisterna tace the nucleus and endoplasmic reticulum; the trans cisterna face the plasma membrane; the medial cisterna are located between the cis and trans cisternae.
As a protein passes from the cis cisterna to the trans cisterna, different chemical modifications occur along the way. The principal chemical modification involves

clsFace

:he addition of carbohydrates (glycosylation) to the maturing protein in a .equential fashion. Other modifications involve sulfation (addition of inorganic .ulfate) and proteolysis (reducing the size of the protein).
Once the proteins reach the trans face of the Golgi complex they appear to be :orted and concentrated into vesicles destined for different regions of the cell.
@ Trans Face

Lysosomes
Specific glycosylated proteins found in the lumen of the RER are transported to :he cis Golgi where individual (mannose) sugar residues are phosphorylated. In

nigure 6-62
The Golgi complex. The cis lace is olten called the lorming
face while the trans face is

:he lumen of the trans Golgi these phosphorylated glycoproteins bind to a mannose-6-phosphate) membrane-bound receptor that directs them to sorting ,.-esicles. Due to a low pH within the sorting vesicles the phosphorylated ilvcoproteins are released from their membrane-bound receptors. Enzymes :alled phosphatases remove the phosphate groups from the sugar residues. The :lr-coproteins are transported to lysosomes while the membrane-bound receptors : j the sorting vesicles are recycled back to the trans Golgi. Lysosomes number in .re hundreds in most cells.
-1,e glycoproteins that reach the spherically-shaped, single membrane-bound . sosomes are actually inactive precursors of lysosome enzymes. These inactive

often called the maturing face.

:recursors are called proenzymes and they undergo proteolytic cleavage in an '.idic environment (pH = 5) to form the mature and active enzyme.
:'"

-,.'sosomes are unique in their composition in that they contain more that 50 pes of hydrolytic enzymes. Since all of these enzymes function at acidic pH ,:lues, they are often referred to as acid hydrolases. The general reaction that -.ese enzymes catalyze is A-B + H2O -+ A-H + B-OH. In other words, hydrolytic

::.z\:mes degrade polymers into individual monomeric subunits. Table 6-7 lists .:,rre acid hydrolases common to lysosomes.
E:zyme
FROTEASES
Peptidase Peptides Peptide

Substrate

Bond Hydrolyzed

r-\-COSIDASES
j-herosandnidase

Glycolipids
Phospholipids

Glycoside Carboxylic ester Phosphoric diester


Phosphoric monoester

,.P \SES
Flnsph.oli.pase

].;L'CLEASES .t, c i tl de o xy r ib o nuc l,e as e

DNA
Phosphomonoesters

::IOSPHATASES

\cid phosphatase

Table 6-7
:

:-:rted lysosomal enzymes.

- l'right

O by The Berkeley Review

4t

The Berkeley Review Specializing in MCAT Preparation

Biology
Cytoplasm

Structure & Function in Celts & Viruses

Eukaryotic Cells

Lysosomal enzymes are inactive at neutral pH' Therefll":


ADP

tl"

environment

+Pi
2H@ Lumen proton pump

withinthelysosomeismaintainedatacidpHvalues.Thisisaccomplished in the lysosomal ,nr""gn,n.'action of an ATPase-driven proton pump located pump that protein the of membrane (Figure 6,63). ATP binds to u po.iiot-, to- enter protons two allows extends into the cytoplasm and as it is hydrolyzed
across the membrane' ir,,o ,f," lysosome. In Lrder to maintain electrical neutrality

achloridetranspolterworksinconjunctionwiththeprotonpumptoallow
anions into the organelle'

of
Lysosome

Figure 6.65
Lysosomal Proton PumP.

Whathappenstothehydrolysisproductscatalyzedbythedifferentenzymatic and are utilized in a variety of reactions? They simply hif.rr" out of the organeile expect to see lysis of this would *",uUofi. pro""rr"r. Ii this were not the .ur*u, -" increase the solute would ;rg^";ii;: why? rne proa.rcts. of hydrolysis entry of water' The osmotic the concentration in the tyro'ro*" and wouli leadto it means observed' not is this would sweil and eventually lyse. Since organelle' "rg;rr" inlt tn" hydrolysis products are removed from the
an inborn error of If the activity of a particular hydrolase enzyme is absent, then lysosomal storage discussed metabolism can result. One oi the more commonly

disorder is due to the absence abnormalities is Tay-sachs disease. This recessive accumulation of a ,i tn" enzyme hexosaminidase A, andresults in the excessive cerebral cortex' the of lysosomes of ganglion cells ;;i;;r'glycolipid in the occurs by usually Death result' Severe mental retard.ation and motor impaiime|t
the age of 5'

Peroxisomes

cellular peroxisomes are single membrane-bound organelles found within the This catalase' being cvtoolasm that contain a variety of enzymes, tf,e most notable (6-7). r,yar"gen peroxide (il2o2) according to equation

;i;;#;r"i",
(6-7)

zE2oz

Catalase vorqruov

> 2H2o +

02

a potentially damaging oxidant, The peroxisome generates hydrogen peroxide, acid

fatty iypr.auct o-f both amino aJid oxidation u.d (very) iong chain ", " oxidation.Peroxisomes,whichnumberinthehundredsinmostcells,arealso
heat' thought to be involved in the dissipation of

Mitochondria
Inanimals,mitochondria(Figure6-64)arefoundinallcellsexcept"'ytllo.-y-T: 1500 ur" oti" of the iargest organelles in the cel1, roughly i;;;;i;'.ells). They s much as occupying nm long by 500 n* *idu, and can numbei in thJthousands'
is characterizedby a double membrane 25oh ofa cell's cytoplasm. This organelle space

,yri"r"
Figure 6-64
The mitochondrion.

6-65). (Figure ""a

two distinct compar"tments, the matrix and intermembrane

Theoutermembraneiscomposedofroughly50%proteinand50%lipid.The a trarismembrane protein which major protein in the outer *"^*brun" is pJrin, with molecular weights of less than forms channels that allows small molecules
10,000 to freelY Pass'

nd2S"hlipidandis TheinnermembraneiscomposedofroughlyT5To-p.rotein p.i:j:t::::::i:t"*,:t:: ine of essentially i*p"r*"uUt"" t"lif-*tf"tt't"''"lvtu'ly. il""H;##ur"""lt" involved in an electrontransport process that is coupled
Copyright @ bY The BerkeleY Review

42

The BerkeleY Review Specializing in MCAT PreParation

Biology

Structure & Function in Cells

Er

Viruses

Eukaryotic Cells

to the generation of ATP from ADP and Pi. Other proteins act as carriers or channels that allow for the transport of certain molecules across this highly impermeable membrane. The number of reactions that occur along the inner
membrane is greatly increased by the presence of numerous convoluted foldings called cristae. These foldings extend into the matrix of the mitochondrion.

Within the matrix some of the cell's most important biochemical reactions occur. The majority of the cell's ATP is produced in the matrix. The Krebs (tricarboxylic acid) cycle, B-oxidation of fatty acids, ketone body metabolism, gluconeogenesis, and some of the reactions of the urea cvcle also occur in the matrix.

t I t I
Malrix Figure 6-65
.\
section of the membrane system and compartments of the mitochondrion

Outer
Membrane

o aa
s re a'c lJ

o Inner
Membrane

Er.en though most of a mitochondrion's proteins are encoded by nuciear genes,


',r'e

do find that mitochondria have their own circular, double-stranded DNA referred to as mtDNA) and their own ribosomes (sedimenting at 55S) and :ransfer RNA molecules that allow for the synthesis of certain mitochondrial ':roteins. Within the small mtDNA genome are 37 genes, and almost all of the -6,569 base pairs are transcribed and translated, making the presence of introns a :emote possibility. Later, when we consider the endosymbiont hypothesis for -.rganelle evolution, we will see that the mitochondrial genome and ribosomes :esembles that of a prokaryote rather than that of a eukaryote.

llicrotubules, Microfi laments,

Ee

Intermediate Filaments

:rlending throughout the cytoplasm of a eukaryotic cell is the cytoskeleton, an -:',tricate protein network of microtubules, microfilaments, and intermediate
--,aments that help define the shape of the cell and allow for ceil movement and

-:il division.

llicrotubules
I.licrotubules are composed of 13 protofilaments arranged side-by side to create a :llow tube-like structure that is about 25 nm in diameter. Each protofilament is - -,nposed of alternating cx-tubulin and B-tubulin proteins, linked together as r.-lers and arranged in the same direction. This gives polarity to the molecule.
- :'e growth of microtubules occurs from regions cailed microtubule organizing :enters (MTOCs). Three common centers are the centrosome (cell center),

rr li-sht O by The Berkeley Review

43

The Berkeley Keview Specializing in MCAT Preparation

Biology

Structure & Function in Cells & Viruses

Eukaryotic Cells

kinetochores (spindle attachment sites on chromosomes), and centrioles. As the microtubules being to polymerize, tubulin dimers add to the fast growing plus (+) end and extend toward the periphery of the cell. The opposite end is "ultea the minus (-) end.

Microfilaments
Microfilaments are about 7 nm in diameter and consist of the protein G-actin (where "G" refers to globular). As the individual G-actin *onb*ers begin to polymetize, they form a long, double-helical, thread-like structure called F-actin (where "F" refers to filamentous). Each G-actin monomer in the polymer is arranged in the same direction, giving the microfilament polarity. Polymerization occurs faster at the plus (+) end of the filament while depolymerization occurs faster at the minus (-) end.

Intermediate Filaments
composition, depending upon what tissue they are located in. For example, the intermediate filaments in epithelial cells are composed of keratins while in muscle cells they are composed of desmin.

Intermediate filaments are about 8 to 12 nm in diameter and differ in their

Copyright @ by The Berkeley Review

44

The Berkeley Review Specializing in MCAT Preparation

Biology

Structure & Function in Cells & Viruses

Prokaryrotic Cells

Bdctffiffiiilrchi Gtffi$ticc
Size and Shape Bacteria come in all shapes and size. The two most frequently encountered
bacteria are the cocci (singular, coccus) and the rods (sometimes called a bacillus). Cocci are essentially spherical in shape while rods generally resemble that of a tube. Bacteria that have a rigid twist to their rod-like structure are called spirilla. If their twisted structure is more flexible, they are called spirochetes. Bacteria not only differ in shape, but they also vary in size. They can be as small as the largest virus or as large as an erythrocyte.

Cell Structure
Prokaryotic cells have a variety of structures important for survival. They all have a plasma membrane which acts as a selectively permeable barrier to metabolites entering and leaving the cell. Ribosomes are found within the cytoplasm and are important for protein synthesis. Inclusion bodies aid in the storage of a wide variety of substances. The genetic material is found in an amorphous region of the cell called a nucleoid. Almost all bacteria contain a cell rvall which helps to prevent cell lysis. Some bacteria have layers external to their cell wall called capsules; others have slime layers. Bacterial movement is brought about by the use of flagella (singular, flagellum).

Plasma Membrane
Prokaryotic cells are fairly consistent in their cellular structure. They all have a plasma membrane which bounds the cytoplasm. Within this plasma membrane are both proteins and lipids. Even though bacterial membranes do not contain cholesterol, many do contain a sterol-like molecule that probably functions in much the same manner as cholesterol. Invaginations of bacterial cell membranes are called mesosomes. The function of these structures is not known.

Cytoplasm & Ribosomes


The cytoplasm of bacterial cells lack membrane-bound organelles (e.g., nucleus,

nitochondria, Golgi, etc.) that we often associate with eukaryotic cells. Within *re cytoplasm are structures called inclusion bodies. These structures can rontain organic molecules like glycogen or inorganic molecules like phosphate
*ranules.

lhe ribosomes found in the cytoplasm of prokaryotic cells are much small than

ie

ribosomes of eukaryotic cells. Complete prokaryotic ribosomes are referred to rs being 70S ribosomes. These ribosomes are composed of a large 50S subunit

'nd

small 30S subunit.

l{ucleoid of DNA
?rokaryotic cells do not have a nucleus. Instead, their circular, double-stranded fNA chromosome is confined to a region in the cell called a nucleoid. These ::gions are often associated with either the plasma membrane or mesosomes. :ome bacteria also have other chromosomes which are circular and double.=anded called plasmids. These additional DNA molecules allow bacteria to be ::sistant to some drugs, and give the bacteria a selective advantage.

J,rpyright @ by The Berkeley Review

45

The Berkeley Review Specializing in MCAT Preparation

Biology

Structure & Function in Cells & Viruses

Dukaryotic Cells

outside cen ile ^^^^^{ ;-\-, ifittrlfll?l'ffixlffi'tff

uuuuuuiiffijuu

it d d d d eGi i-ri i,( ,c ,( ,( ,(

M M-M

I
1

? 9Xo
(D

The Cell Wall In 1884 Christian Gram developed a stain that allowed bacteria to be classified into two groups. Bacteria that stained purple were called Gram positive (+) bacteria while those that stained red or pink were called Gram negative (-) bacteria. The difference between the two types of bacteria has to do with their cell walls. structures outside of the plasmf membrane are usually referred to as the cell wall or envelope. Gram positive bacteria have a rather thick homogeneous peptidoglycan or murein layer (20 nm to 80 nm) just outside their plasma membrane. They also contain a large amount of teichoic acids that extend to the surface of the cell from either the plasma membrane or the peptidoglycan layer.
Gram negative bacteria (Figure 6-66) have a much thinner peptidoglycan layer (about 1 nm to 3 nm) surround their plasma membrane. Flowever, surrounding

M-.-M

ri

0c

? o tu

the peptidoglycan layer is an outer membrane that contains lipopolysaccharides and porins. Not only does the polysaccharide help to stabilize the membrane, but it also acts as an endotoxin and provides a defense mechanism for the cell. The porins allow for passage of materials smaller than 700 daltons. Larger materials must be transported across the outer membrane.
The peptidoglycan itself layer is composed

Figure 6-66
Gram negative bacterial membrane.

and D-alanine. Attached to each N-acetylmuramic acid residue is


tetrapeptide side chain.

of two acetylated amino sugars, Nacetylglucosamine (G) and N-acetylmuramic acid (M), linked together in a B(1,a) [nkage, and a small number of amino acids, including D-glutamic acid
a

Capsules & Slime Layers In general, both capsules and slime layers are composed of polysaccharides that extend from the surface of the cell. This polysaccharide layer is sometimes called a glycocalyx. The distinguishing features between the a capsule and a
slime layer is that slime layers are easier to wash off. Capsules are much more resistant. In fact, they help protect the bacterium from foreign invaders. We will come back to this point when we consider the Griffith experiment involving the bacterium Streptococcus pneumoniae.

Flagella

a counterclockwise direction that propels the bacterium through its environment. The major protein component of the flagellar structure is
flagellin.

Many bacteria use flagella for movement. These protein structures extend from the plasma membrane and cell wall and provide a propeller-like movement in

Why do bacteria need to move in their environment? Nutrients. Bacteria are attracted by chemical nutrients like sugars and amino acids. Attraction towards
or repulsion away from certain chemicals is referred to as chernotaxis.

Life Cycle
The process of cell division in bacteria is rather simple. The DNA, which is attached at some point to the inside of the plasma membrane, undergoes replication at a site called the replication origin. Once the DNA has been replicated a new plasma membrane and cell wall begin to enclose the newly synthesized chromosome. The two bacterial cells will eventually separate by a process called binary fission.
Copyright @ by The Berkeley Review

46

The Berkeley Review Specializing in MCAT Preparation

Biology

Structure E( Function in CeIIs & Viruses

Prokaryotic Cells

DNA Transfer Genetic material can be passed from one bacterial cell to the next by the simple process of binary fission. However, genetic information can also be transferred to a bacterial cell by either bacterial conjugation, transformation, or transduction.

In bacterial conjugation the transfer of genetic information occurs by cell-cell .ontact. Donor strains of bacteria are referred to as being F+ (male) while the :ecipient bacteria are F- (female). The "F" refers to the fertility plasmid. During :onjugation the F factor replicates by a process called the rolling circle mechanism of replication and the newly synthesized DNA is passed to the recipient bacterium. Transformation involves the uptake of genetic material from the surrounding :-Ledium. Usually this new genetic material is incorporated into the host ;hromosome.
Transduction is simply the transfer of bacterial genes by viruses. How do viruses :et bacterial genes in the first place? In general, when a virus infects a host cell it ncorporates its own DNA into the DNA genome of the host. When it is time for ie virus to leave, it removes its own DNA from the host genome. It is during 'jris process that errors can occur. Sometimes the virus removes a few host genes as well. Once the virus has replicated and the progeny phage have been :ssembled and released, they are able to infect other bacteria. Those progeny :hage will bring to those new bacteria a few genes from previously infected host
:eils.

will be returning to these different types of DNA transfer when we begin our liscussion on Genetic lnformation.
,\-e

- rpvright @ by The Berkeley Review

47

The Berkeley Keview Specializing in MCAT Preparation

Biology

Structure & Function in Cells & Viruses

Viruses

Architecture
\-iruses show a wide range of biological diversity and are quite successful at :arasitizing other organisms. Quite simply, at the genetic level, viruses are
.-bligate intracellular parasites that cause infected host cells to produce viral gene :roducts rather than host gene products.
Lipid membrane

lrre architecture of a virus (Latin for poison) is usually based upon one of two .ructural motifs (Figure 6-67); those which are isometric (usually in the form of :,.. icosahedron) or those which are helical. In its simplest form a virus is

tu,,,*
protein

-. iormed from capsomers, which are building blocks composed of a specific :,'-rmber of individual proteins. If there is no nucleic acid within the protein coat :r shell, then the empty shell is referred to as a capsid. Flowever, if there is :-ucleic acid within the protein shell, the complex is called a nucleocapsid.
-: -]re nucleocapsid of the virus is not surrounded by a lipid membrane, the virus ;o referred to as a non-enveloped (naked) virus. However, if the nucleocapsid is .;rror-rnded by a lipid membrane, the virus is referred to as an enveloped virus. li.e membrane of an enveloped virus is derived from the host cell that the virus ::ected and is attached to the nucleocapsid by matrix proteins. Transmembrane ::oteins which have been glycosylated (i.e., glycoproteins) act as antigens and

--lmposed of a nucleic acid that is surrounded by a protein coat. The protein coat

L'N
Nucleocapsid
Capsomer

(b)

.'-on. the virus to communicate with its environment. These complexes are
>:,=retirnes called spikes.

:"::re viruses, like the bacteriophage T4 that infects the bacterium Escherichia coli Z :':li), are quite complex. This phage not only has an elongate iscoshedral head, ::: it also has a helical tail section with tail fibers that can attach to the ';".polysaccharide portion of the host's membrane.

Genome l:e genetic information within the genome of a virus may be encoded in either :-e nanguage of DNA or RNA. The nucleic acid can either be linear or circular,
;::rl.e-stranded or double stranded, and it can even be segmented. However, no :'Jer what type of nucleic acid is found in a viral genome, the translational :f :..ess involved in the expression of that genome uses mRNA as a template. -}.=:efore, by convention, we define that nRNA as being a positive (+) strand - ::leic acid. DNA and RNA polymers that have a base sequence identical to this ;,:s:ti\re mRNA strand are also designated as being positive (+) strands' i,:::ember, in DNA we find that the base thymine is used instead of the base ,*.:-1. DNA and RNA polymers with a base sequence that is complementary to the :,-srdve mRNA strand are referred to as being negative (J strands. In order to , :.fr.esize a positive mRNA strand, the nucleic acid template must either be a ri:":aLive DNA or RNA strand.

Figure 6,67
Viruses with (a) enveloped isometric and (b) nonenveloped helical geometry.

l:=

relationship between the positive mRNA strand and the different nucleic allows us to organize viruses into six classes based on a scheme proposed :,, lar-id Baltimore. The Baltimore classification for the six viral classes and some ',*::esentative viral families are shown in Table 6-8.
.r:- j:'

:r "i-isht

by The Berkeley Review

49

The Berkeley Review Specializing in MCAT Preparation

Biotogy
Virus
T4 phage
), phage Herpes

Structure & Function in Cells & Viruses


Host
Bacteria Bacteria

Viruses

Genome Linear (+/-) ds DNA Linear (+/-) ds DNA Linear (+/-) ds DNA

Class

Size (kb)

Mo
No No
Yes

t70

Elongated icosahedral head Helical tail; Tail fibers Icosahedral head

I
I
IIa IIa

46
150

simplex

Helical tail Animal


Bacreria

M13

QX174

phage phage

Icosahedral

Bacreria

Circular (+) Circular (+)

ss ss

DNA DNA

6.4 5.4

No No

Helical rod
Icosahedral

Rotavirus Poliovirus Rhinovirus


Rabies

Mammals Mammals Mammals

Segmented (+/-) ds RNA

m
IV IV
V

1.2

1.4

No No No
Yes Yes Yes

Icosahedral Icosahedral
Icosahedral

Linear (+) ss RNA Linear (+) ss RNA

7.2 - 8.5 7.2 - 8.5

lnfluenza

HIV

Vertebrates Linear (-) ss RNA Vertebrates Segmented C) ss RNA Vertebrates Dimer (+) ss RNA
= 1000 nucleotides

t2
13.6 9.2

Helical rod
Helical rod Icosahedral

VI

$ Approximate sizes in kilobases where 1 kb

Table 6-8
The six viral classes as proposed by David Baltimore.

we will examine the coliphages T4, 1., and Qx{74, as well as the human imunodeficiency virus (HIV), a retrovirus implicated in the pathogenesis of human acquired immunodeficiency syndrome (AIDS).

The information presented in Table 6-g might seem a bit overwhelming. It is not meant to be. And above all, this information is not to be memorized. Rither, it is presented to give you a feel for the rich biological diversity in the viral world. As we proceed in our discussions we will be considering a number of these viruses.

Copyright O by The Berkeley Review

50

The Berkeley Review Specializing in MCAT preparation

Biology
Adsorption

Structure & Function in Cells & Viruses

Viruses

Ad#U#pffi#itri1,,"'WffH, ffiffii|ffi11# s#i$ffi


\'-iral infections are generally restricted to specific organisms or types of cells, usually referred to as the host range of the virus. Viral specificity for a host cell depends on the interaction of a particular virus with the host cell's surface proteins, glycoproteins, and glycolipids.
For example, the bacteriophage T4 has a complex of tail fibers that recognizes a specific lipopolysaccharide structure as well as a particular protein porin, both of rrhich are located on the cell surface of the bacterium E. coli. The bacteriophage l, nas a single tail fiber that recognizes an integral membrane protein responsible
Bacteriophage

tor the transport of the sugar maltose into E. coli. The HIV retrovirus has a glvcoprotein (gp120) associated with its envelope that appears to bind to a specific protein receptor (CD4) found on the surface of helper T cells and monocytes, two cellular constituents of blood, and glial cells, a cellular
c,f,nstituent of the central nervous system.

Penetration
Once a virus adsorbs to the surface of a suitable host, its next step is to introduce 1ts genome into that host, a process called penetration. The type of virus and the

attachment to the host membrane determines how this

will occur.
Viral DNA

Bacteriophages
-{fter a bacteriophage like T4 has adsorbed to the membrane of the host cell, the orotein sheath that defines the tail begins to reorganize and penetrates the cell ltali of the bacterium until it contacts the plasma membrane. An opening is iormed in the plasma membrae and phage DNA is passed from the viral capsid, lhrough the tail section, and into the bacterial cytoplasm (Figure 6-68). The llershey-Chase Experiment -\s shown in Figure 6-68 only the viral DNA enters into the host. The protein components of the capsid, tail, and tail fibers remains outisde the bacterial cell. This finding was demonstrated in a classic experiment performed by Alfred Hershey an-d Martha Chase in 1952. They infecte d E. coli bacteria with 32P and 35S labeled T2 bacteriophages. After allowing for phage adsorption the infected 'tacterial cells were separated from any unattached phages by centrifugation and then placed in a Waring blendand subjected to violent shearing forces. The fluid ',ras again centrifuged. Hershey^ and Chase found that the bacterial cell pellets containeda high percentage of 32P while^the supernatant, containing the sheared phages, contained a high percentage of 35S. Since protein contains sulfur (in the amino acids cysteine and methionine) and DNA contains phosphorous (in the :tnsphodiester bonds of the backbone), Hershey and Chase concluded that it was :he viral genome, and not the viral protein, that entered the bacterial cells and caused infection.
(c)

capsid

s"--,{n

"K
,rn"'&
Flgure 6-68
Sequence of events showing (a) adsorption of phage to bacterial cell, (b) penetration, and (c) release of viral

DNA into host

ilaked (Non-Envetoped) Viruses \aked viruses gain access to the host's cytoplasm only by

cytoplasm.

recptor-mediated

.:fiocytosis. The receptors on the cell surface of the host are usually located near specialized depressions called clathrin- co ated pits. Clathrin, a non-glycosylated piotein, is believed to act as a scaffold that promotes vesicle formation. After the -;irus has been taken into the host by endocytosis, the pH of the vesicle's interior s lowered as hydrogen ions are transported into the vesicle by proton pumps in -Jre vesicle membrane. The clathrin coat depolymerizes and the smooth-surfaced
t/y

Copyright @ by The Berkeley Review

5l

The Berkeley Review Specializing in MCAT Preparation

Biology

Structure & Function in Cells & Viruses

Viruses

vesicle is now referred to as an endosome. As the pH of the vesicle falls, the nucleocapsid disrupts the membrane of the end.osorile and is released into the cytoplasm. The naked virus has been brought into the host cell (Figure 6-69).
virus binding to membrane receptor
Plasma membrane of host

U.Naked

Viral genome

cytoplasm
coated pit

(b)

Clathrin/

'{w{ Y
..(
Receptor-mediated endocytosis Endosome

Figure 6-69
Penetration of a naked virus into a host cell. (a) Binding of the virus to a membrane receptor is followed by (b) endocytosis and (c) enclosure of the viral g-"no-e in an endosome.

cytoplasm.

Enveloped Viruses viruses which are enveloped can either enter their host through receptormediated endocytosis or by direct fusion with the plasma membrane. If an enveloped virus enters-thrgrigh receptor-mediated endtcytosis, a process similar to the one just outlined is followed. The enveloped virus attaches to the plasma membrane of the host and is brought into the ."tt uy end.ocytosis (rigure o-zo). After a low enough pH is reached in the endocytotic vesicle, the vira'i envelope fuses with the vesicle's membrane and the nucleocapsid is released into the

Enveloped virus binding to membrane receptor

+
Cytoplasm
coated pit

Plasma membrane of host

Viral genome

(b)

Clathrin/

-4t {
Receptor-mediated endocytosis

\)
.-(

Endosome

Figure 6-7O
Penetration of an enveloped virus into a host cell by receptor-mediated endocytosis. (a) Binding of the viral envelope to a membrane receptor is followed by (b) endocytosis and (c) enclosure of-the enveloped virus in an endosome.

Copyright @ by The Berkeley Review

52

The Berkeley Review Specializing in MCAT preparation

Biology

Structure & Function in CeIIs & Viruses

Viruses

-{n alternate approach is for the envelope of the virus to directly fuse with the plasma membrane upon initial contact with a specific receptor (Figure 6-71). This is how the HIV retrovirus is thought to enter its host cell. The fusion of the viral membrane with the plasma membrane, whether it is through receptor-mediated endocytosis or direct membrane fusion, is mediated by fusion proteins in the

liral

membrane.

Enveloped virus binding to membrane receptor

q)
Cytoplasm

Viral genome

Plas

o.,4t

Figure 6-7

Penetration of an enveloped virus into a host cell by membrane fusion. (a) Binding of the viral envelope to a membrane receptor is followed by (b) fusion of the viral and plasma membranes and (c) release of the nucleocapsid into the cytoplasm.

Once the virus has entered the host cell, its genome must have access to the cellular processes that allow for nucleic acid replication and protein synthesis. This is brought about by a process called uncoating. Here the protein capsid is removed from the viral genome, thereby allowing the viral nucleic acid to enter the host cell's cytoplasm.

Expression
We have mentioned that viruses are obligate intracellular parasites that cause infected host cells to produce viral gene products rather than host gene products. How does this occur? In order to answer that question, we must first determine if the infected cell is prokaryotic or eukaryotic; if the viral nucleic acid is DNA or RNA; and if that nucleic acid is double stranded or single stranded.

Viral Replication & Expression in Prokaryotes


There are many examples of viral replication and expression in prokarytotic cells. One of the most widely studied groups of viruses are those that comprise the Tseries of. bacteriophages that infect E. coli. Prcbably that most famous coliphage fuom this series is T4.
The genome of the T4 phage is represented by linear, double-stranded DNA with a rather unique feature. The nitrogenous base cytosine has been modified with a hydroxymethyt group at the C-5 position of the ring. This modified base, 5hydroxymethylcytosine (HMC), is present only in the viral DNA. It is not found in the bacterial DNA. The importance of this modified base to the viral genome

is that viral enzymes will easily be able to determine which is viral DNA and which is bacterial DNA.
The

DNA of T4 could contain as many as 150 genes, of which about half code for regulatory proteins and degradative enzymes. The bacterial enzyme RNA
Copyright @ by The Berkeley Review
DC

The BerkeleY Review Specializing in MCAT Preparation

Biology

Structure & Function in Cells & Viruses

Viruses

polymerase, which is continually being modified by viral proteins, is used to transcribe different genes in the viral genome. viral gene products not only disrupt the replication, transcription, and translation of the host genome, but they also aid in the production and assembly of progeny virions. For example, within 2 minutes after infection the immediate eailq viral genes are transcribed by the bacterial errzyrr.e RNA polymerase. Translation of thil plus (+)

mRNA transcript produces viral proteins that have a variety of actions. The pol}rnerase enzyme is subsequently modified in order to transcribe the delayed early, quasi-late, and late viral genes. Translation of these plus (+) mnlvR transcriptsproduces viral proteins that eventually act to digest bacterial DNA
we will be returning to viruses in our discussion on Genetic Information.

Copyright @ by The Berkeley Review

54

The Berkeley Keview Specializing in MCAT Preparation

Structure and Function in Cells and Viruses


15 Passages

100 Questions

Passage Titles

Questions
12-17

I. U. III. IV. V. VL VII. V[I. IX. X. XI. XU. XI[. XIV. XV.

Sepsis

EndoplasmicReticulumMicrosomes Essential Amino Acids Endotoxin Spinal Meningitis Nuclear Envelopes and Pores Lipids and Membranes Nonstandard Amino Acids Amino Acid Characteristics Fluorescent Recovery after Photobleaching (FRAP) Gram-Negative and Gram-Positive Bacteria Raffinose Mitosis and Meiosis Lipids: Four Groups BacteriophageLambda

6-tr

1-.5

18 - 23

24-29 30-36 37 -43 44-50 51-58 59-65 66-12 73 -79 80-87 88-94 95 - 100

,]fu

REBI{ELAY
Speci aLtztng

L)R.E-u.l .-b.w*
in MCAT Preparation

Suggestions
The passages that follow are designed to get you to think in a conceptual manner about the processes of molecular biology at the organismal level. If you already have a solid foundation in moleculai biology, many of the questions you read here will seem to be very straight forward and easy to answer. But if you are new to the subject or if you have not had a pleasant experience with molecular biology in the past, some of them might appear to come from the void that spreads out beyond the Oort field ii the edges of our solar system. Pick a few passage topics at random. For these initial few passages, do not worry about the time. just focus o1 what is expected of you. First, read the passage. Second, look at any diagrams, charts, or graphs in it. Third, read each question and the accompanying answers carefully. Fourth, answer the questibns the best you can. Check the solutions and see how you did. \A/hether you got the answers right or wrong, it is important to read the explanations and see if you understand (and agree with) what is being explained. Keep a record of your results.

approaches to a passage. For example, you might feel well versed enough to read the questions first and then try to answer some of them, without ever having read the passage. Maybe you can answer some of the questions by just looking at the diagrams, charts, or graphs that are presented in a particular passage. Remember, there are many effective learning styles. You need to begin to develop alormat thit works best for you. Keep a record of your results. The last block of passages might contain at least a few topics that are unfamiliar even to those who know a good deal about molecular biology. Find a place where the level of distraction is at a minimum. Get out your watch and time yourself on these passages, either individually or as a group. It is important to have a feel for time, and an awareness of how much is passing as you try to answer each question. Never let a question get you flustered. If you cannot figure out what the answer is from information given to you in the passage, or from your own knowledge base, dump it and move on to the next question. As you do this, make a note of that pesky question and come back to it when you have more time. When you are finished, check your answers and make sure you understand the solutions. Be inquisitive. If you do not know the answer to something, look it up. The solution tends to stay with you longer that way. (For example, what ls the Oort field, anyway?)

After you feel comfortable with the format of those initial few passages, pick another block of try to do them in one sitting. Be aware that time is going to become important. On average, you have about 1 minute and 15 seconds to complete a question. Be creative in how you approach tils next group. If you feel comfortable with the outline presented above, fine. If not, then iry different
Passages and

The estimated score conversions for 100 questions are shown below. At best, these are rough approximations and should be used only to give one a feel for which ballpark they are sitting in.

Section VI Dstimated Score Conversions


Scaled Score

Raw Score

>13

ll-12
9- l0 7 -8 5-6
<4

- 100 10-79 60-69 50-59 40-49 0-39


80

Biology
Passage

Sepsis

Passage

(Questions 1-5)

1.

The Gram stain used to differentiate between two main types of bacteria takes advantage of the fact
that:

Sepsis

is an extremely complex disease that disrupts

all physiological regulatory mechanisms involved in maintaining homeostasis. This disorder, which in its extreme form, may be characterized by organ malfunction, is brought about by the interaction of
microbial products with host cells. When a host organism first succumbs to the effects of an acute bacterial infection, the inflammation response initiated by the body of the host is meant to preserve its health. Normally, the inflammatory response is turned off by a variety of mechanisms. Septic shock develops when this homeostasis is disrupted.

A. B. C.

neither type has a membrane-bound nucleus.

Gram-positive bacteria do not have a plasma


membrane.

bacteria differ from one another


structures of their cell walls.

in

the

D. different bacteria possess different


characteristic shapes, such as rod and spiral.

,-aused

Septic shock has generally been considered to be by Gram-negative bacteria. The major sepsisinducing factor of a Gram-negative bacterium is its
surface lipopolysaccharide (LPS). This bacterial product \timulates the production of a number of cytokines and -rther inflammatory mediators that have both prornflammatory and anti-inflammatory functions. Septic .hock is associated with the excess production of pro.nflammatory mediators. Among these cytokines are :umor necrosis factor (TNF) and interleukin-1. The target ri the inflammatory activation pathway is the endothelial
, e11.

2.

According to the passage, cytokine generation and the inflammatory response are normally repressed by a number of mechanisms. Which of the following events would be LEAST likely to repress the
inflammatory response?

A. B. C. D.

Down-regulation of cytokine receptors on host


cells.

Once LPSs are found in the host in quantities that :rnnot be managed by its defense mechanisms, they ::cruit monocytes, macrophages, polymorphonuclear -ells, and endothelial cells to initiate an inflammatory -:scade. This response may occur locally in tissue or
.rroughout the bloodstream.

Activation of the complement system.


Endogenously generatedcytokineantagonists. Suppression of the immune system by
hormones.

,;

In blood, LPS binds to a circulating molecule known LPS-binding protein (LBP). Once formed, these :rmplexes then bind CD14, a surface molecule on -.onocytes. Binding of LPS/LBP to CD14 leads to

:'.

-.onocyte activation. Endothelial cells are also stimulated

:-.,rrvever,

LPSs, acting with LBP and CD14 molecules. in this case, endothelial cells lack a membrane-

rund form of the CD14 molecule.


3.

It is difficult to study sepsis, because most mediators are capable of promoting the release of other mediators and even of themselves. This selfstimulation is BEST described as:

A.
B. C. D.

an endocrine function.
a a

exocrine function.
paracrine function.

an autocrine function.

- :,ryright O by The Berkeley Review

The Berkeley Review


Specializing in MCAT PreParation

Biology
4.

Sepsis

Passage

Septic shock is not a new disease, yet its incidence is on the rise. Which of the following would be a likely reason for this increase?

A.
B. C. D.

Less invasive procedures are being used in


modern medicine.

decline in immunosuppresive therapy has

been noted.

The use of antibiotics has created highly


resistant strains of bacteria.

There has been a decline in the population of


persons with treatable serious diseases.

5.

In

endothelial cells, LPS is also capable of stimulation through LBP and CD14. Analysis of endothelial cells shows a lack of a membrane-bound form of CD14. This protein is MOST likely to be
found:

A.
B.

bound bound

to the extracellular face of to the cytoplasmic face of

the the

membrane on the endothelial cell.

c.
D.

membrane on the endothelial cell. within the cytosol of the endothelial cell. within the blood and extracellular fluids of the
tissues.

Copyright

by The BerkeleY Review

5a

The Berkeley Review Specializing in MCAT PreParation

Biology
Passage

Endoplasmic Keticulum Microsomes


6.

Passage tr

II

(Questions 6-11)

:omogenization, the endoplasmic reticulum (ER) is


-ra_smented into many closed vesicles called microsomes.

When tissues or cells are disrupted

by

gradient is used with the following result:

To separate smooth microsomes from rough microsomes, centrifugation within a sucrose

\licrosomes derived from rough endoplasmic reticulum re called rough microsomes. The ribosomes are always
:,rund on the outside surface of these microsomes.

Smooth 1 mlcrosomes

J
Signal peptides were discovered in secreted proteins :irt are translocated across the ER membrane prior to .:eir transport to the Golgi apparatus and eventual ':,-retion from the cell. The signal hypothesis postulates :,:t a leader peptide serves as a signal peptide, directing .-: eventual secreted protein to the ER membrane. Once
.Rough mlcrosomes

-:

'

peptide is inside the ER, the signal peptide is cleaved

br

a special protease.

This result can BEST be explained by the fact that:

,=:.rde and moves between the cytosol and the ER -.:''rrane. The other component is the SRP receptor, an - =rral membrane protein found within the ER ;:frane. The signal-recognition particle binds to the : "1 peptide as soon as the peptide emerges from the ' i,rme. A subsequent pause in protein synthesis is -,=:,.ed untii the ribosome carrying SRP binds to the :,: ::ceptor. The translational arrest is then lifted with * -.:rneous translocation of the protein into the :- , :-rsmic reticulum.

*::rbrane by two components. One is the signal'-:.-snition particle (SRP), which binds ro the signal

The signal peptide is thought to be guided ro rhe ER

A. B. C.

rough microsomes are made


smooth microsomes have

of

different
higher

proteins than smooth microsomes. ribosomes contain large amounts of RNA.

a much

D. rough

density than rough microsomes,

microsomes have much smaller diameters when compared to smooth

microsomes.

The exterior of a rough microsome is equivalent to


the:

A.
B.
C.

D.

cytosol. lumen of the nucleus. lumen of the rough ER. lumen of the Golgi.

8.

The mRNA of a hormone normally produced by an endocrine cell is translated by a free ribosome in the presence and absence of microsomes. In this cell free system, the hormone is found to be:

A.
B.
C.

D.

the same size in the presence and absence of microsomes. longer in the absence of microsomes. shorter in the absence of microsomes. longer in the presence of microsomes.

r
m

bv The Berkeley Review

59

The Berkeley Review Specializing in MCAT Preparation

Biology
9.

Endoplasmic Reticulum Microsomes

Passage tr

It was discovered that washing microsomes with salt


removes the SRP as part of the salt extract. This would be MOST likely to result in:

A. B.

a quicker translational arrest.

elimination of the ability to import protein into


the ER.

C. the loss of the signal peptide. D. an increase in the frequency of secreted


proteins.

10.

One reason for the translational arrest observed the peptide emerges from the ribosome might be:

as

A. B. C.

D. to allow the ribosome time to proofread the


signal peptide before it binds to the SRP.

to prevent the release of the newly synthesized protein into the cytosol. an indication that the ribosome is nonfunctional. to ensure that all translation must occurs on the membrane of the ER.

11. During

the course of a typical secretory pathway, a molecule leaving the lumen of the ER will next come in contact with the:

A. B. C. D.

lateral region of the Golgi complex' cis region of the Golgi comPlex. medial region of the Golgi complex. trans region of the Golgi complex.

Copyright @ by The BerkeleY Review

60

The BerkeleY Review Specializing in MCAT PreParation

Biology
Passage

Essential Amino Acids

Passage Itr

III

(Questions 12-17)

12. According to Table 1, which of the following


statements must be TRUE?

Nine amino acids are required in the diet of adult


human beings. They are histidine (His), isoleucine (Ile), leucine (Leu), lysine (Lys), methionine (Met), threonine 'Thr), valine (Val), tryptophan (Trp), and proline (Pro). Each of these amino acids has a distinctive side chain that ,-annot be formed by enzymes in the human body.

I.

All animal proteins meet the ideal protein


requirements. Soybeans meet ideal protein requirements.

II.

III. Lentils are low in sulfur-containing


acids.

amino

In the 1950s and

1960s, researchers studied

A.
B.

I only

.equirements of each amino acid and produced an ideal :rofile of amino acids for human nutrition. Generally, the :-1esh of other animals, birds, and fish contains all nine

C. D.

'mino acids in patterns that fulfill the "ideal" protein ::quirement. Except for soybeans, the proteins of . egetable sources are usually lacking one or two of the :ssential amino acids. Grains and nuts are usually low in
'.

II only II and III only I, II, and III

.:uno acids. The combination of plant proteins of

sine. Beans and peas tend to be low in sulfur-containing

: rd are complemented by the other

:rrino acids the body requires for growth


:rntenanCe.

---ierent classes (so that amino acid weaknesses in one food) is called 'nrein complementation. An amino acid is termed niting" if it does not meet or exceed the ideal protein ,..:dard. The standard is based on the proportions of
and

For which amino acid are beans characteristically


lower than the ideal protein standard?

A.
B. C. D.

Methionine Lysine Leucine


Phenylalanine

Protein
Food Source

Lys
5.5
6.4 1.8 8.7 8.8 6.9

Met / Cys
3.5
5.5
-1

Thr
4.0
5.0 4.6

Trp
1.0

Leu
7.0 8.8 9.8 8.2 7.2 8.4 8.7 7.0 3.0
7.5
-J.t,

.::al
',1r11

t.6

--J

t.4
1.2
1.2 1.5

:..+

3.8

4.4
4.3 4.3 3.4 3.6 4.0 J.J
1.8

l:.icken
:
.,

4.0
3.5

beans

14. Which

of the following structures indicates lysine?

6.4

2.6
1.5

1.0

-.rti

ls

6.1

t.0
0.6
1.3

:lmeal
neal
n

2.9 3.7
-1.4

3.5
3.6

HjN- C- CCHt

HO @rilo

H3N-

HO orilo C- C*
CH"

0.9

0.0

t'

jeal reference protein.

l.

Arnino acids as percent protein in foods and compared

o+"
H1N- C- CI

HO otilo
CH"

t" CH" t'


CH:
I

HjN- C- CCH:

@rilo

or NH:

CH,

H-N-7,N-H

/:\o

lH'

on

-_:ht O by The Berkeley Review

The Berkeley Review Specializing in MCAT Preparation

Biology
15.
Of the choices listed in Table limiting amino acids?

Essential Amino Acids


1, which has only two

Passage Itr

A. B. C. D.

Collagen
Soybeans

Cornmeal

Lentils

16. An unknown food is


observed:

examined, and each

of its

amino acids is analyzed by gas chromatography with flame-ionization detection. The following pattern is

Lys Met/ Cys Thr Tip Leu


5.^/,

3.t

3.7

1.0

3.2

What is the MOST probable food category of this


unknown sample?

A. Bean B. Grain c. Milk D. Fish

17.

Which of the following amino acids is nonessential?

A. B. C. D.

Phenylalanine

Valine
Isoleucine

Histidine

Copyright @ by The Berkeley Review

62

The Berkeley Review Specializing in MCAT Preparation

Biology
Passage

Endotoxin
18-23)

Passage IV
Gram-negative bacteria are distinguished from

fV (Questions

18.

::rnains with the acute inflammatory response to


rndotoxin released from these bacteria. The endotoxin is a .-popolysaccharide, often called LpS. The acute -flammatory response to endotoxin is called septic shock.

_ Many potent antibiotics exist that are effective against Gram-negative bacteria, but a potential clinical problem

Gram-positive bacteria by:

A. B. C. D.
19. A

an anaerobic culturing process. an immunoassay.


a differential staining technique. high performance liquid chromatography.

The death

of

Gram-negative bacteria leads

::lease of endotoxin, a lipid component of the outer cell ;- ill of most Gram-negative bacteria. Humans respond to with a release of cytokines and other cellular 'rdotoxin :---'diators. These mediators include tumor necrosis factor :r TNF-o), interleukin-1 (L-1), IL-6, leukotrienes, and :::omboxane A2. At high levels, these compounds may .:--gter the responses of septic shock, including fever, l.-,lck. hypotension, clotting, and organ failure.

to

the

person has an infection caused by a Gramnegative bacterium. What effect would an antibiotic effective against Gram-negative bacteria have on short-term concentration of endotoxin in the blood?

A. B. C. D.

Endotoxin levels would not change with


antibiotic treatment.

Endotoxin levels would decrease rapidly as the antibiotic bound and neutralized endotoxin.

:":r

In the endotoxin molecule, a terminal disaccharide ::,rspholipid (Lipid A) contains the key strucrural feature
is responsible for toxicity in Gram-negative bacteria.

Endotoxin levels would decrease briefly


bacteria were killed by the antibiotic.

as as

Endotoxin levels would increase brieflv


bacteria were killed bv the antibiotic.

oH

(Ho)rP(o)oY NHR,
oRr

\^r"+^*
OH

.A' *i'

oP(oxoH),

20.

The following table shows data from an experiment on mice, using the experimental agent E5531 given simultaneously with 3 prg of E. coliLpS.
E5531 (pglmouse Plasma TNF Plasma TNF Mortality inhibition (%) (Vo)
0
6 100

(ng/ml) 582+ 20
547

0 (control

E. coli Lipid

t45

80 20
0r!

J 10

432+ 40*
259 + 28+ 198
71

26
55

- . I A. called E5531. It was designed to work as an i;:sonist of endotoxin to avoid the cascade of events
,r'.,j1119

i.:searchers tested
to septic shock.

synthetic structural analog of

30
100

!241 + l8t

66
88

0* 0*

* p < 0.01 versus control.

p < 0.001 versus control.

Which of the following statements is FALSE


oP(oxoH)"

regarding these data?

Meo

nl./
oH

'u"*'

A. A dose of 10 prg E5531 allowed all the mice B. C. D.


tested to survive without complete suppression of plasma TNF changes due to LPS. The selected dosage of LPS given to the mice was lethal to about 7570 of those tested. Plasma TNF responded inversely to the dose of E553 1 .

,Ho)rp(o)oY NHR.
oRr

+^"-,

E5531

Plasma TNF inhibition led


survival rates for test animals.

to increases in

lright

by The Berkeley Review

tr.t

The Berkeley Keview Specializing in MCAT Preparation

Biology
21.
experiments involving administration
intraperitoneally to mice:

Endotoxin
23.

Passage IV

The following graph shows the results of a series of

of E. coli

85531 might act as an antagonist of Lipid A in an organism infected with Gram-negative bacteria by:

I. Itr.

competing for cellular surface receptors in the


host.

U. directly
No treatment

cleaving the terminal portion of

released Lipid A to promote inactivity. interfering with bacterial release of Lipid A.

FA

Y>.
(

60

640
r1

A. I only B. I and II only C. I and III only D. I,II, and trI

01020304050
Time after E coll infection (hrs)

Which conclusion is supported by the data in the


graph?

A. The B. C.

antagonist alone provided effective

treatrnent in promoting long-term survival. This bacterial strain is resistant to antibiotics.

The combination of antibiotic and endotoxin antagonist was most effective in promoting
long-term survival.

D. The antibiotic alone provided

effective

treatment in promoting long-term survival.

22. What is the purpose of

administering both an antibiotic and an endotoxin antagonist during an


infection involving Gram-negative bacteria?

To

increase blood bacterial count while

U. To increase concentrations of ilI. To destroy strains of bacteria


A.
B. C. D. mediators to attack the bacteria.
resistant to the antibiotic that is used.

decreasing plasma endotoxin concentration.

cellular

that may be

I only

I and II only I, II, and III


None of the above

Copyright @ by The Berkeley Review

64

The Berkeley Specializing in MCAT

tsiology
Ir';rss;age

Spinal Meningitis
25.
as

Passage V

\r (Questions 24-29)

According to the passage, the epithelial layer of the


throat:

. :' :neningitis, is caused by the bacterium Neisseria :' .*. i:iidis. The disease is known for its extremely rapid : ,,: - .\ infected person first develops fever and malaise. - ;,:.,:r. within hours, these symptoms evolve into
-

\f:lingococcal meningitis, commonly known

A. B. C. D.

traps bacteria and promotes infection. serves as a barrier against infection bacteria.

by

the

r ::: :eadache, neck rigidity, and an aversion to bright .: ": Ii these symptoms go untreated, a patient can lapse " : ,:ra and a fatal form of shock.
,{:rrngococci normally colonize the lining of the
^::

secretes mucus, which destroys bacteria. becomes infected in spinal meningitis, and is the ultimate cause of death.

- '; -, , '-: of healthy people carry meningococci. Spinal -r- rlrris begins when the organism invades the blood :'.::- :nd crosses the meninges (the membranes which .: ::re spinal cord and brain) into the cerebral spinal

.:

So common is the organism that at any given time,

26.

'

-.

".-

. :::tefia.

This fluid acts as a culture medium for rapid growth

In an experiment, tissue samples are taken from both the spinal cord and brain of a human subject with spinal meningitis. Which of the following statements is MOST likely to be true?

'":: :rrJocci. The group

- - 960, researchers from the Army Institute - ,;.-:ated the importance of host defense against
-

A. In the brain and spinal cord, gray matter is B. In the brain and spinal cord, gray matter is C.
matter is found in the interior region.

found in the exterior region, while white


found in the interior region, while white matter is found in the exterior region. In the brain, white matter is found in the exterior, while in the spinal cord white matter is found in the interior.

drew blood from thousands of : ::s as they entered basic training and followed them "' -i:- ut this period. As men became ill, their stored ,,r"r ::e tested for their ability to kill meningococci and - : -:J with anti-meningococcal activity from the sera :::.:jr! men. The group found that the disease occurred -:.:-.r in recruits who had low anti-meningococci . . '' ln sera before they became ill. Most adults have

D. In the brain, white

matter is found

in

the

' :-: ;.:.DoSed to meningococci stood a high chance of : , , r.-re i11. It appears that most people develop :;..',e antibodies after their first exposure to ' r - r:,;occi, which keeps them from becoming ill.
Lr

' :::1r,e antibodies against meningococci. The recruit ": .r,Jicated that individuals lacking antibodies who

interior, while in the spinal cord white matter is found in the exterior.

27, People with a genetic deficiency in

their

complement system are unusually susceptible to spinal meningitis. Since the disease is rare in the United States, a substantial proportion of cases seen here may result from this deficiency. However, the defect is unlikely to be a significant cause of an

epidemic. Which
supports this claim?

of the following

statements

A. In Nigeria and Gambia


.,1, i'r-;h of the following organelles ' -:--n i,{eisseria meningitidis?

(countries with

repeated spinal meningitis epidemics), patients

with complement deficiencies are commonly


would be found

B. In Nigeria and Gambia, patients C. A D. A


encountered during epidemics.

encountered during epidemics.

with

complement deficiencies are not commonly

\" E, i--, tt

\ucleus \Iitochondria
Ribosomes

population's level

of

antibodies against

Endoplasmicreticulum

population's level of antibodies against meningococci declines before an epidemic.

meningococci rises after an epidemic.

;
m

65

The Berkeley Keview Speciatizing in MCAT Preparation

Biology

Spinal Meningitis

Passaqe V

28. Neisseria lactamicq, a relative of N. meningitidis, is part of the normal oral flora in humans. Young
children who acquire N. Iactamica throat infections are frequently protected against meningococci infections. The MOST likely reason for this
protection is that:

A.

toxins released from N. lactamica destroy N.


meningitidis.

B. N. Iactamica colonizes in the throat,


eliminating any viable environment for
N.

C. D.

meningitidis. antibodies produced against N. Iactamica are effective against N. meningitidis.

antigenic determinants on N. lactamica are different from N. meningitidis.

29.

According to the passage, neck rigidity is a symptom of spinal meningitis. This symptom is a result of:

A. C. D.

B. rapid growth of the bacteria in the CSF,


causing inflammation in the meningeal lining.

endotoxins released from the bacteria, which affect only neck muscle.

N. meningitidis over stimulating the nerves


innervating neck muscle.

N. meningitidls under stimulating the nerves


innervating neck muscle.

Copyright @ by The Berkeley Review

66

The BerkeleY Review Specializing in MCAT Preparation

Biology
Passage

Nuclear Dnvelope and Pores


30.

Passage VI

VI (Questions 30-36)

The nuclear envelope of a cell encloses its DNA and :efines the nuclear compartment. The spherical inner

According to the passage, the perinuclear space is continuous with the:

:uclear membrane contacts the chromosomes and nuclear R-\As. The inner membrane is surrounded by the outer
.irosomes engaged in protein synthesis.
The nuclear envelope in all eukaryotes is perforated by lclear pores. A typical mammalian cell contains 3000 .-;h pores. The pore is embedded in a large disk-like ).:ucture known as the nuclear pore complex. The ,:mp1ex is formed by protein granules arranged in an

-uclear membrane, which

is usually studded

with

A. B. C. D.

nuclear lumen. cytosol. endoplasmic reticulum lumen.

mitochondriamatrix.

,.:ich water-soluble molecules shuttle between the ---leus and cytosol. The effective size of the pore has ::in determined by measuring the rates of diffusion of
=::led, non-nuclear components. The results show the :e of the pore to be 9 nanometers in diameter and 15 - ::,rmeters long. To offer a comparison of size, the - ::reter of a ribosome is approximately 30 nanometers.
-

,:risonal arrangement. The pore is an aqueous channel in

31.

According to the passage, a nuclear envelope:

A. B. C. D.

ensures DNA replication.

confines translation to the cytosol.

prevents synthesis of ribosomal RNA inside


the nucleus.

breaks down during interphase


cycle.

of the

cell

:r:lragrn that opens to just the right size when activated si-unal on an appropriate protein. The selectivity of --:.:ar transport lies in the presence of nuclear import . .1s. which are present only in nuclear proteins. The .,.1. located anywhere within the protein, generally . -:.sts of four to eight amino acids that bear positively. -.:_.:d side chains.

'- .

\uclear pores can open to accommodate objects larger '- 9 nanometers in diameter. The pore is like a

32. The MOST likely reason that

non-nuclear

components are used to measure the effective size the nuclear pore is that nuclear components:

of

' r. :'le molecules every three minutes to package the -- -nation molecule correctly. During mitosis, the
' -:.:.rs. many soluble proteins are excluded, including -. '. iormer residents of the nucleus. In time, these

-- a cell is replicating its DNA, then

it requires

106

A. B. C. D.

cannot be radiolabeled. are difficult to isolate. are accommodated by nuclear pores. are smaller than the pore size.

-:.:ar envelope breaks down, and the contents of the -: :us mix with the cytosol. Upon reformation of the
-:-:r residents find their way back
inside the nucleus.

33. In an experiment,

nucleoplasmin (an abundant nuclear protein) is proteolytically cleaved into a tail and a head component. Both are attached to a 20-nm piece of colloidal gold, which is easily visible under the electron microscope, and placed in the cytosol. In time, only the gold attached to the tail piece is found near the nucleolus. The nuclear import signal is found on the:
A.
B.
C. head, and contains Phe and Trp. head, and contains Lys and Arg. tail, and contains Phe and Trp. tail, and contains Lys and Arg.

D.

t'

-:ht

O by The Berkeley Review

The Berkeley Review Specializing in MCAT Preparation

Biology
34. In an experiment, a signal
enzyme:

Nuclear Envelope and Pores


sequence is chemically

Passage VI

attached to random amino acids on phosphofructokinase. The result of the experiment is that the

r (

A. B. C. D.

remains in the cytosol, because the location of the nuclear import signal is unimportant. remains in the cytosol, because the location of the nuclear import signal is important. is transported to the nucleus, because the location of the nuclear import signal is unimportant. is transported to the nucleus, because the location of the nuclear import signal is important.

d n

ti s
d

&

q fr

35. According to the passage, nuclear


MOST likely to:

proteins are

,il

fi

OI

A. B. C. D.

be resynthesized after mitosis. be destroyed during mitosis.

lose their nuclear import signal after transport into the perinuclear space. keep their nuclear import signal after transport into the perinuclear space.

fr I

lfii

6! m

&

36. In an experiment,

small RNA molecules injected into a frog oocyte nucleus are rapidly transported into the cytosol. When these same RNA molecules are injected into the cytoplasm, they remain there. The BEST explanation for this is that receptors on
the:

h
,t
frru

T:

fi
nucleoplasmic face of the pore recognize the
nuclear import signal.

A.
B.

cytoplasmic face of the pore recognize the


nuclear import signal. nucleoplasmic face recognize a nuclear export signal.

C. D.

cytoplasmic face recognize a nuclear export


signal.

Copyright

by The Berkeley Review

68

The BerkeleY Review Specializing in MCAT Preparation

Biology
Passage

Lipids and Membranes


Experiment

Practice Passage Vtr

VII

(Questions 37-43)

Biological membranes are composed

of

lipids,

In a double-labeling experiment, researchers gave a


suspension of growing bacterial cells a l-minute pulse of radioactively labeled 32pgo3-. This pulse label was

proteins, and carbohydrates. Membranes control the flow of molecules into and out of a cell through the use of

exocytosis, endocytosis, ionophores, pumps, gates,


junctions, and other membrane-embedded proteins. Simple lipids include the terpenes and the cholesterolderived steroids. Complex lipids include fatty acids, triglycerides, waxes, phosphoglycerides and sphingolipids. Lipids are soluble in nonpolar organic solvents,
such as chloroform. They are marginally soluble in water.

designed to add radioactively labeled phosphoryl groups only to phospholipids currently being synthesized by the cell. After the 1-minute pulse, a reagent specific for PE,

trinitrobenzenesulfonic acid (TNBS), was immediately


added to the cellular suspension.

Because

of the relative insolubility of lipids in

water,

HzC-

amphipathic membrane lipids like the phosphoglycerides and sphingolipids form lipid bilayers. The hydrophilic portion of the bilayer faces the extracellular space and cytoplasm of the cell, while the hydrophobic portion tbrms the interior of the membrane.
The transverse diffusion (flip-flop) of a lipid molecule across the membrane is quite rare, but the lateral diffusion

lo

o il O- C-

Rl

HC- O- C- R2

Irr
lo

HlC- O- P- O' lo

liloozN

CH.CH2NHI

PE o-

of a lipid molecule along the entire length of a bacterial :ell like Escherichia coli takes about one second.

in biological membranes primarily on the source of the membrane and, to : small extent, the diet of the organism producing the nembrane. For example, in Escherichia coli, the weight :ercent of total lipid of the phosphoglyceride phospha:idylethanolamine (PE) is about 65Vo, while that of :holesterol is }Vo. Even though most membranes are
The composition of lipids
depends

HzCHC-

Irr O- Cloo.N Irr HrC_ O_ P_ O*


R2

lo

il O- C- Rl

o+

\-

nrso.

Noz
CH2CH2

NH

sg
Experiment 2

:oughly one-half protein, the protein-to-lipid ratio varies,


:epending on the function of the membrane.

Membrane proteins can be integral or peripheral' Integral proteins are tightly bound to membranes by

:ldrophobic forces, and peripheral proteins are loosely :,,rund to membranes through electrostatic interactions :nd hydrogen bonding. Unlike lipid molecules, membrane ::oteins cannot undergo transverse diffusion. They can :rfuse through the membrane only in a lateral fashion. Horvever, both the lipids and the proteins found in a
:embrane are asymmetrically distributed.

In this double-labeling experiment, the procedure is exactly the same as in Experiment 1, except that a 3minute time period is allowed to pass between the end of
the l-minute pulse of radioactively labeled 32pgo3- un6
the addition of TNBS.

.{symmetric membranes are synthesized on the ::inbrane synthesize membrane lipids while membrane:,:und ribosomes synthesize protein to be incorporated ::o the membrane itself. Selective labeling experiments, , -:h as the one described below, confirm that membranes :.:r be constructed on the framework of preexisting
:r:mbranes.

37,

The bacterium Escherichia coli is classified as:

-'::rnework

of preexisting

membranes. Enzymes

in

the

A. B. C. D.

Gram-positive,

with a plasma

membrane

surrounded by a thick cell wall'

Gram-negative, with a plasma membrane surrounded by a thin cell wall and an outer
membrane.

Gram-negative, with a plasma membrane Gram-positive, with a plasma membrane surrounded by a thin cell wall and an outer
membrane. surrounded by a thick cell wall.

l:pvright

by The Berkeley Review

69

The BerkeleY Review Specializing in MCAT PreParation

Biology
38.

Lipids and Membranes


of the phosphoglyceride
4t.

Practice Passage Vtr

Complete hydrolysis
products?

shown below by a eukaryotic cell yields how many

The inability of an integral protein to diftuse transversely in a membrane is MOST likely due to:

A. strong hydrophilic forces between


o il

the

HrC- O- C- (CH2)r4cH3 loHH Ittll c- (cHz):- c= c-

B. C.
(cH2)7cH3

membrane and the Protein. the protein's asymmetric distribution.

the protein's numerous hydrophobic amino acid residues in the membrane's nonpolar
interior. strong hydrogen-bonding forces between the protein and the membrane.

lo o Itt H,C-O-P-o-cH2cH2NH3 -l
o9

D.

A phosphatidYlethanolamine

A.0 8.3
D.5

c.4
In Experiment 1 it is observed that none of
with TNBS. This indicates that:
the

42.

The fluidity of a membrane is controlled by the composition of fatty acids in it and by its cholesterol content. A decrease in the membrane fluidity of a bacterial cell is caused by:

39.

radioactively labeled PE molecules was labeled

L II. III.

the presence of cholesterol'

long-chain fatty acids with a cis double bond' an ln"rease in the length of a saturated hydrocarbon chain on a fattY acid'

I. il. III.

TNBS can readily cross the bacterial


membrane.

synthesis

of PE occurs on the extracellular

face of the membrane.

synthesis

of PE occurs on the cytoplasmic

A. I only B. II only C. III onlY D. I and III onlY

face of the membrane.

A. I only B. II only C. III only D. I and III onlY 43.


40.

In Experiment 2 it is observed that roughly half of the radioactively labeled PE was labeled with TNBS. This indicates that:

I.

TNBS can readilY


membrane. membrane.

cross

the bacterial the


bacterial

Sperm whales (Physeter catadon) can dive to depths of 1100 m. A mature whale's head contains about 4 tons of different waxes (commonly referred to as whale oil). One such wax is spermaceti' an ester of a 16-carbon fatty acid and a 16-carbon alcohol. This wax has a melting point between 424? 'F. Whales use these waxes to: fi

II. PE can fliP-floP

across

cH3(cH?)14

III. synthesis of PE occurs on both the


the membrane.

- c- oSpermaceti

(cH2)15cH3

extracellular face and the cytoplasmic face of

A. I only B. II only C. III only D. II and III onlY


Copyright
@ bY

A. B. C. D.

increase their average density'

control their buoyancy in sea water'


conserve energy at great depths' all of the above.

The BerkeleY Review

70

The BerkeleY Keview Specializing in MCAT PreParation

Biology
Passage

Nonstandard Amino Acids


46. How many
codons does

Passage VItr

VIII

(Questions 44-50)

it

take to specify the

nonstandard amino acids?

The twenty common amino acids dictated by the


standard coding tables are not the only amino acids to occur in biological systems. Nonstandard amino acids occur in proteins, in biologically active peptides, such as neurotransmitters, and in several metabolic pathways.

A. B. C. D.
47.

Three

Four Five
None of the above

collagen fibers, are formed

by modification of the standard amino acids after a polypeptide is synthesized. Collagen is the most abundant protein in vertebrates. It is present in connective tissue, such as bone, teeth, ligaments, tendons, and epithelial tissue.
Some neurotransmitters are made by modifications

4-hydroxyproline and 5-hydroxylysine, components of

The enzyme ornithine transcarbamoylase transfers the carbamoyl group of carbamoyl phosphate to
diagrammed below:

ornithine, yielding citrulline. This reaction is

of

2 ATP+HCO3

O,I

single amino acids. For example, y-amino butyric acid r'GABA) is made from glutamic acid by the enzyme glutamic acid decarboxylase. Dopamine is made from the amino acid tyrosine. Other nonstandard amino acids lunction in urea biosynthesis (citrulline and ornithine), rmino acid metabolism (homocysteine), and methylation
reactions
(S

+NH3

,r /

-qi+ 2ADP+P; o\ tlrN - "to C- O- Por

Carbamoyl phosphate

-adenosylmethionine).

in -^n' I ' ?", CH" "-i-',P", loo.


Omithine

l*-;'""']tt
I

f",

T"

lttt
''

Fn,.,

?,

+t.

The lysine residue modified to make hydroxylysine in the collagen polypeptide is modified during:

["-t--t
Citrulline

CH,

l"-["i']tn

A. B. C. D.

post-translation. post-transcription. post-replication.


post-degradation.

The carbamoyl moiety is indicated by:

45. Which of the following


l
-i

amino acids is proline? B.

A. I B. II c.m D. IV
48. In a series of experiments,
one group of rats is fed laC-radiolabeled proline as part of their diet. A second group of rats is fed laC-radiolabeled

A.

HIN-C-C-O
n
I

HO OrnO
CHz
I

HO @rll O H3N-C-C-O
CHz
I

CH"

CH,
OH

II

H- C_
CHt

lCH. t'
CH"

hydroxyproline. The rats are killed, and their collagen is separated and analyzed. In which group
is radioactive collagen synthesized?

.c!l

NHr

^l' " NH,

A. Both, because the Proline and B. Only the rats receiving proline
collagen polypeptide.

the hydroxyproline are both incorporated into the


have

C.

D.

oIl oocr.-i-

ooc.
on

""*H
g,s

Acs, c I'

C.

radioactive collagen.

Only the rats receiving hydroxyproline have


radioactive collagen.

,t""/ H

D. Neither, because the Proline and hydroxyproline are digested in the small
intestine.

7l

on

The BerkeleY Review Specializing in MCAT PreParation

Biology
49.
organ is urea synthesized?

Nonstandard Amino Acids

Passage VItr

Urea is a vehicle for removing nitrogenous waste from the body in a relatively nontoxic form. In what

A. B. C. D.

The liver The kidney

The gall bladder


The pancreas

50. Prolyl hydroxylase,


scurvy?

the enzyme that hydroxylates

proline, requires vitamin C as a coenzyme. What would be the effects of vitamin C deficiency, called

I.

il.

Easy bruising

Itr.
A.
B. C. D.

Poor wound healing Loose teeth

fi

fr

I only

I and II only II and III only I, II, and III

il
0

ri

fr q

iG

Copyright

by The Berkeley Review

72

The BerkeleY Reviev Specializing in MCAT Preparation

Biology
Passage

Amino Acid Qharacteristics


51.

Passage IX

IX (Questions 51-58)

The pH of a solution can be expressed as:

Proteins are initially synthesized from the amino (N) terminus to the carboxyl (C) terminus during translation as polypeptide chains from a pool of 20 common amino acids. Each of these amino acids is coded for by at least

one codon in the genetic code. Once an amino acid is incorporated into a growing polypeptide chain, it can undergo post-translational modification, thus allowing for.
an even richer diversity in protein structure.

A. B. C. D.

- ln [H+].
lo916 [H+].

ln [H+].
- log1s [H+1.

Each of the common 20 amino acids contains an ctcarbon atom, to which four substituents are bonded: a hydrogen atom, an amino group, a carboxyl group, and a
side chain.

52. A mixture

of the common 20 amino acids in solution at physiological pH would predominately take on


the characteristic of being:

It is the side chain that places each amino acid

rn a unique grouping. Nine amino acids have nonpolar side chains, six have uncharged polar side chains, and five
have charged side chains.

A.
B.
C.

nonpolar.

positively charged and monovalent.


negatively charged and monovalent. dipolar.

The cr-amino and cx-carboxyl group of each amino acid 'iear a dissociable hydrogen atom with a characteristic :Ku value. For leucine, an amino acid with a nonpolar

D.

.ide chain, the pKa for the cx-amino group is 9.6, while :re pK. for the o-carboxyl group is 2.4. The addition of rne equivalent of a base (NaOH) titrates the cr-carboxyl ;roup and converts the fully protonated form of leucine :to the zwitterion form whose net charge is zero (Figure

53. All

The addition of a second equivalent of base titrates the

of the following amino acids are found near the center of a long polypeptide chain. Which amino acid in this protein molecule is not ionizable?

,r:-amino group.

- cooH (9t :lrN 'I - C- H


t' HIC- CI

*oHo u,N-g-H *oHo


+H @ lHrC-C-H 'l
CH:

o
9oo

coo
I
I

H2N- C-H
CH,

A. B. C. D.

Histidine
Isoleucine Lysine

Arginine

CH" H

+H

HIC-l C-H
CH:

t-

CHr

Figure

54. The titration of leucine with sodium

hydroxide

-:
.

-'.:otropin-releasing hormone (TRH) is an example of a eptide containing three amino acids joined by peptide - --Js (Figure 2). Proteins generally contain 50 or more

If a small number of amino acids are linked together, molecule is called a peptide or apolypeptide.

generates the titration curve shown below. The isoelectric point for this amino acid is close to:

-.-:ro acids.

* 2.0 o d_ z t.J
E
C rn o r.v

I ll @l - N - C- C-

HHOHOHO
N

I ll I ll -C - C -N - C- C-

'=
NH2

IV

tD

cl

0.5

N-u

il "-N\
Thyrotropin Releasing Hormone (TRH)

A.
B. C.

1.0

3.0 6.0
10.0

Figure 2

D.

*
oo
-ght

by The Berkeley Review

1C

The BerkeleY Review Specializing in MCAT Preparation

Biology
55.

Amino Acid Characteristics

Passage IX

Phenylalanine, an essential amino acid, is converted to tyrosine in the following reaction:

58. In Figure 2, the N-terminus nitrogen in TRH


indicated by which Roman numeral?

is

o
H3N- C-

@ foo
H

A. I B. II c.ru D. IV

cH"

o
Phenylalanine

t"

This reaction can be described as a:

A.
B.
C.

carboxylation. hydrolysis. hydration.

D.

hydroxylation.

56.

Proteins are molecules that contain 50 or more


amino acid residues, and they can act as excellent
buffers because of: their hydrogen-bonding capabilities in forming
secondary and tertiary structures.

A.
B. C. D.

the ease with which H+ and OH- ions can be absorbed once the protein is hydrolyzed. the ability of the terminal regions of the protein to accept or donate H+ ions. the wide range of pKu values found within the protein.

5t. Complete hydrolysis of the tripeptide thyrotropinreleasing hormone (TRH) requires how many moles

of water?

A.2 8.3 c.4 D.5

Copyright @ by The Berkeley Review

74

The BerkeleY Review Specializing in MCAT Preparation

BiOlOgy
Passage

Fluorescent Kecovery aft,er Photobleaching (FKAP)


]l
I

Passage X

X (Questions 59-65)

Experiment

6)
C)

In 1970, a hybrid cell was artificially produced by the fusion of mouse cells with human cells. Two differently labeled antibodies were used to differentiate between human and mouse plasma membrane proteins. At t = 0 minutes, the labeled antibodies were confined to their respective halves. At t = 40 minutes, the two different .ntibodies were mixed over the entire surface of the
hvbrid cell. This experiment is shown in Figure
1:

O O

Time

---------->

Figure 3

59. In Experiment

1, the incubation temperature was

lowered significantly. The time required for the


human and mouse plasma membrane proteins to
reach the state pictured in Figure

lb would

be:

A.
Figure I

B. C. D.

:,:eriment

increased, due to increased fluidity of the plasma membrane. increased, due to decreased fluidity ofthe plasma membrane. decreased, due to increased fluidity ofthe plasma membrane. decreased, due to decreased fluidity ofthe plasma membrane.

To study the lateral diffusion rates of membrane :eins, one can use fluorescence recovery after : -:tobleaching (FRAP). Most commonly, fluorescent

:-

- .:lovalent antibodies are attached to selective membrane :-"::ins. The tightly bound fluorescent ligands are

"=.:hed in a small area by a laser beam, and the time , r,.in for adjacent membrane proteins carrying -::l:ached fluorescent antibody molecules to diffuse into
"= rleached area is measured. This experiment is shown ' - .gure 2 and Figure 3:

60.

The monovalent antibodies used in Experiment 2 are fragments of antibodies that have only one antigenantibody binding site. The MOST likely reason for using such antibodies is that:

A.
B.

only monovalent antibodies can


fluorescence. membrane proteins.

carry

only monovalent antibodies can attach to


the use of monovalent antibodies prevents cross-linking between neighboring molecules. a bivalent antibody would attach to more than one antigenic determinant.

c.
D.

lur',"l

{}

[l

:L

[-l *".nu.ru

61.

The results of Experiment

I provide

evidence for:

Figure 2

A. B. C. D.
75

protein flip-flop. lateral diffusion. simple diffusion.

rotationaldiffusion.

ii

:'_ri.lt

O by The Berkeley Review

The Berkeley Review Specializing in MCAT Preparation

BiOlOgy
62.

Fluorescent Kecovery aft,erPhotobleaching (FRAP)

Passage X

The graph shown in Figure 3 shows that the highest level of recovery is slightly lower than the beginning level offluorescence. This is because;

65. In eukaryotic cells, the plasma membrane


concentrations, cholesterol has the effect of:

contains

relatively large amounts of cholesterol. At such high

A. not enough time was allowed during


recovery process to obtain the
fluorescence.

the

full level of

A. B. C. D.

decreasing the

inhibiting hydrocarbon chains from

fluidity of the membrane by

B. the level of
declined.

fluorescence had naturally


a

C.

the laser beam only temporarily bleaches


population of the fluorescent ligand.

crystallizing. decreasing the fluidity of the membrane by promoting hydrocarbon-chain crystallization. increasing the fluidity of the membrane by crystallizing.

inhibiting hydrocarbon chains from


of the
membrane by

D. the laser

beam permanently bleaches

population of fluorescent ligand.

increasing the fluidity

promoting hydrocarbon-chain crystallization.

63.

The graph below depicts the FRAP results for four

different surface glycoproteins under similar


conditions:

l
I

o
C)

o o !

tr

The rate of diffusion is lowest for:

A.
B. C. D.

Glycoprotein Glycoprotein Glycoprotein Glycoprotein

A.
B.
C.

D.

64.

In an experiment, a single glycoprotein is inserted into a synthetic lipid bilayer. Using FRAP, it is determined that the rate of diffusion of the
glycoprotein is significantly higher in vito than in vivo. The MOST likely explanation for this increase in the rate of diffusion is that: A.
B.

antibodies

of a lighter weight

are used for

C.

tagging in this synthetic system. cross-linking occurs between antibodies in this synthetic system. the lack of bulky intracellular oligosaccharide

chain interaction increases the rate of


D.

chain interaction increases the rate of


diffusion.
Copyright @ by The Berkeley Review

diffusion. the lack of bulky extracellular oligosaccharide

76

The Berkeley Revier Specializing in MCAT Preparatia

Biology
Passage

Gram-Negative and Gram-Positive Bacteria

Passage Xl

XI (Questions 66-72)

-\n important difference between prokaryotic cells and -riaryotic animal cells is the presence (in prokaryotes) of : protective cell wall sunounding the plasma membrane.

o^ to a)r
(D

Bacteria can be divided into two groups: Gram:,:sitive and Gram-negative. Division into these two _.:rups is based on their reaction to the Gram stain, as
:

"::lined in the procedure given below:

Step

suspension

of

bacterial cells

is

stained with

crystal violet. This procedure takes about I minute.


Step 2

Face2
o|!
o

,G

//////// M'LM M-LM M-LM dt d d d d d d '/////// M' M-M M-LM M-LM /////// GGGGGGG
M

cll

rJ (D rd

oO <o (Doq

Iodine (I2) is added to the suspension and complexes with the crystal violet to fix the cells.
This procedure takes about 3 minutes. Step 3

Face

Alcohol is added to the suspension and removes :o1or from the cells. This procedure takes about 30
seconds.

Transmembrane

protein

Figure I
Step 4

Safranin, a red-colored counterstain, is added to the suspension of cells. This procedure takes about 1 to

reactions, while enzymes

minutes.

Staining techniques can generally be divided into three - :,es: positive staining, negative staining, and differential .'-:ing. Positive staining employs dyes to stain cells in ':-.r to increase their contrast. Negative staining allows ;:-is to be seen in outline against a stained background.

Antibiotics like penicillin inhibit transpeptidation like lysozyme hydrolyze polymers of N-acetylglucosamine and N-acetylmuramic acid. Both actions weaken the ce1l wall and lead to eventual osmotic lysis. However, if the cell is in osmotic equilibrium with its environment, intact spherical structures called spheroplasts and protoplasts can form. Spheroplasts retain some fragments of the cell wall, but
protoplasts do not.

-:isvsnsitl staining can involve both positive and


:.rative staining.
---' another

Gram-positive and Gram-negative cells differ from in the structure of their cell walls. The cell *:.1 of Gram-positive bacteria contains peptidoglycan, , ,.,trted polysaccharides, and teichoic acids. The cell wall

66.

Escherichia coli has the characteristic shape of


Gram-negative bacterium, namely a:

Gram-negative bacteria contains peptidoglycan,

: : -' spholipids, lipopolysaccharides, and assorted proteins'


-.:, Gram-positive cells, peptidoglycan accounts for about : , :o 90 percent of the weight of the wall; in Gram:=::rive cells, it accounts for about 10 percent.

A. B. C. D.

coccus. rod. spiral. spiral helix.

: *::Lber of amino acids, including D-glutamic acid and D-

Peptidoglycan is composed of two acetylated amino -;ars. N-acetylglucosamine (G) and N-acetylmuramic p(1,4) linkage, and a small '.:-: iM), linked together in a

67.

,!.:rine (Figure 1). Attached to each N-acetylmuramic -,:-J residue is a tetrapeptide side chain. Transpeptidase ::rrmes catalyze cross-linking between some of these
:e chains and increase the stability of the peptidoglycan.

Invariant cell structures in prokaryotes include all of the following, EXCEPT:

:-rrmes called autolysins :::ng cellular growth.


-

open up the peptidoglycan

A. B. C. D.
/l

cell wall.

a nuclear region.

ribosomes. a plasma membrane.

:lyright

by The Berkeley Review

The BerkeleY Review Specializing in MCAT PreParation

Biology

Gram-llsgative and Gram.Positive Bacteria


staining

Passage XI

68. After being subjected to the Gram

procedure, Gram-positive and Gram-negative cells show which of the following colors, respectively?

A. B. C. D.

Violet; colorless

Blue;violet
Colorless; red Blue; red

69. The Gram staining procedure is a:

I. rI. III.

positive staining technique. negative staining technique. differential staining technique.

A. I only B. II only C. III only D. I and III only


70.
Based on Figure 1, the enzyme transpeptidase would be expected to be found near which membrane face in both Gram-positive and Gram-negative bacteria?

A. B. C. D.

Face I

Face2
Face 3
Face 4

7t. The antibiotic penicillin weakens the peptidoglycan layer in virtually all species of prokaryotes by inhibiting transpeptidation. This can lead to lysis hnd eventual death of the cell. Penicillin-induced
lysis can be prevented by:

A. B. C. D.
72.

inhibiting enzymes called autolysins.


increasing the growth rate of the cells.

inhibitingprotoplastformation.
decreasing the solute concentration outside the

cell.

One would expect a bacterial cell with the structures shown in Figure 1 to be:

A. B. C. D.

to penicillin, because the antibiotic can cross the outer membrane. resistant to lysozyme, because the enzyme
nonresistant
cannot cross the outer membrane.

resistant

to both penicillin

and lysozyme,

because neither can cross the outer membrane. nonresistant to lysozyme, because the enzyme can cross the outer membrane.

Copyright @ by The Berkeley Review

78

The Berkeley Review Specializing in MCAT Preparation

Biology
Passage

Kaffinose
73.

Practice Passage XII


Raffinose can be described as

XII

(Questions 73-79)

a:

primary groups on the basis of their mode of


reproduction . The monotremes are the only mammals that lay eggs. The embryo of the marsuplal develops in the uterus for a very short time after fertilization. As soon as the immature marsupial is able to survive outside the

Mammals first arose some 200 million years ago during the Mesozoic Era. Today, there are at least 4500 species of mammals. They can be divided into three

A. B. C. D.

reducing sugar. nonreducing sugar.


disaccharide.

glycoprotein.

it leaves and crawls into a marsupium (pouch) where the mother's mammary glands are located to finish the developmental process . Placental mammals develop in the uterus to a much later stage of maturity before parturition.
mother's uterus,

74.

Fructose can be classified as:

The koala (Phascolarctos cinereus) is an arboreal


marsupial native to Australia that feeds exclusively on the

I.

leaves

of the eucalyptus tree. One of the naturally

il. ilI.
IV.
A.
B.

an aldose. a ketose.
a furanosea

occurring carbohydrates found in eucalyptus leaves is the oligosaccharide raffinose (see structure below). Placental

pyranose.

mammals, such as humans (Homo sapiens), typically obtain raffinose from leguminous seeds like peas and
beans.
CH,OH

I only

c.
D.

II only II and III only I and IV only

Bond

0
HOH
Galactose
H

o-

9ut

OH

HOH

't5.

The bond between glucose and fructose is in the:

Glucose

Fructose

For all mammals, carbohydrates provide a large


:crtion of the daily caloric requirement. However, before :r oligosaccharide can be utilized by an organism, it must ..rst be degraded by enzymes (salivary amylase and ,: ancreatic amylase) into smaller units called ':anosaccharides. Monosaccharides such as galactose, :.ucose, and fructose are rapidly absorbed across the :ucosal cells of the wall of the duodenum and ileum of
--.e

I. IL III. ry.

o(l-+2)configuration.

a(l-+5) configuration.
9(5-+1) configuration.

B(2-+1)configuration.

A. I only B. I and II only C. III only D. III and IV only

small intestine by a carrier-mediated transport system.

I'iigosaccharides that are

not hydrolyzed are

not
The bond between galactose and glucose is in the:

. : sorbed.

Raffinose is an example of an oligosaccharide that .nnot be hydrolyzed in the small intestine. However,
.:e this carbohydrate reaches the terminal portion of the

76.

A.
B.

cr(6+1) configuration.

:um and the beginning of the large intestine, it can be , drolyzed into its constituent monosaccharides by :srdent bacteria. Many of these monosaccharides can be rerobically metabolized by the bacteria to produce : npounds like lactate, methane, carbon dioxide, and
,

F(6+1) configuration.

C. D.

p(l+6)

configuration.

cr( I

-+6) configuration.

Jrogen gas.

The Berkeley Keview Specializing in MCAT Preparation

Biology
77.
about which carbon atom?

Raffinose

Practice Passage XII

Galactose and glucose are epimers of one another

A.
B.
C.

c-l
c-4 c-5 c-6

D.

78.

If the galactose residue were to be hydrolyzed from


raffinose, the disaccharide left over would be:

A.
B.
C.

D.

maltose. lactose. sucrose. none of the above.

I I
79.

t
After ingestion of leguminous
seeds, many people

I!

experience a period of flatulence. This problem can be traced to:

1o

A. B. C. D.

the loss of bacterial enzymes that degrade


oligosaccharides.

,i

oligosaccharides that cannot be hydrolyzed by human intestinal enzymes. a decrease in the products of anaerobic metabolism. a decrease in intestinal motility.

o d

ft d
h
I

U ni

u
h
II
iil

n[

h fl

ni

Copyright @ by The Berkeley Review

ao

The BerkeleY Keview Specializing in MCAT Preparation

Biology
Passage

Mitosis and Meiosis

Passage XItr

X[II

(Questions 80-87)

Normal Down's

Mitosis and meiosis are essential for all eukaryotic


organisms. After fertilization, the zygote depends on mitosis for growth and development. Mitosis is also important for replacement of certain tissue cell types during the life of the organism.
0)

Mother Father
6

Child
I

Child

rzg

N^

After cytoplasmic division (cytokinesis) of a cell during mitosis, the resulting two daughter cells enter interphase of the cell cycle. Interphase is a period of intense metabolic activity. During interphase, each daughter cell proceeds through an initial growth period, followed by a period of DNA synthesis (S phase), and then another growth period before mitosis and
cytokinesis. Mitosis occurs in somatic cells and results in daughter cells, which are diploid.

c oo 6.t bo.- ,!'t I

I I

I I
1

Autoradiograph of Family

Normal Down's

Mother Father
6

Child
I I -

Child

6g
C61 o3

o N^

In contrast, meiosis occurs in the sex cells and results


in the production of gametes which are haploid. Meiosis consists of two cellular divisions called meiosis I and meiosis II. Before the onset of meiosis I, chromosomes have duplicated during the S phase. Homologous chromosomes pair (synapse) and form a tetrad consisting of four chromatids. During meiosis I, there is a reductive division in which each tetrad separates to form dyads, each containing two sister chromatids. The dyads are separated during meiosis II into monads. Each monad represents a single chromosome, the carrier of genetic information in the form of genes.
There are many techniques in genetics that can be used '.o analyze genes. In Southern blotting, DNA from a ;articular chromosome is cut into fragments using a

E-

b0.2
1

I I

Autoradiograph of Family 2

Figure

80.

Which of the following sequences represents the


correct order of the phases of the cell cycle?

A. B. C. D.
81.

Metaphase, telophase, S, G2, prophase Interphase, S, G1, metaphase, prophase Telophase, anaphase, Gr, Gz, metaphase G2, anaphase, telophase, S, G1

:estriction endonuclease and then separated according to size using agarose gel electrophoresis. The DNA in the sel is next treated with alkali and denatured to yield

.ingle-stranded DNA.

nitrocellulose paper (used

Which of the following mature cell types rematns ln the G1 phase of the cell cycle?

iecause of its ability to bind single-stranded DNA) is :laced on top of the gel, and a buffer flow is used to -:ansfer the DNA from the gel to the nitrocellulose paper. The exact DNA fragment pattern in the gel is maintained rn the nitrocellulose paper. Next, a probe containing a ,.nown sequence of radioactively labeled DNA is used for
rr bridization to the DNA on the nitrocellulose paper. The :osition of the DNA fragments that anneal to the probes

I. IL IfI.

Kidney epithelial cells


Skeletal muscle cells

Nerve cells

.:e revealed by autoradiography. In Northern blotting'

A. I only B. I and II only C. III only D. II and III only


82. DNA synthesis during the S phase of the cell cycle
can be detected by using radioactively labeled DNA precursors. The BEST technique for monitoring the incorporation of these labeled precursors is:

l*\A

fragments can be transferred to nitrocellulose paper.

l.r Western blotting, proteins can be analyzed using an


SDS gel.

The Southern blotting technique is used to analyze the

l,NA of two different families. Each family has a child


a child who is unaff'ected. The '. ith Down syndrome and .rtoradiographs of the DNA from the mother, the father, .:d the children of each family are shown in Figure 1:

A. B. C. D.

Northern blotting. Southern blotting. Western blotting.


autoradiography.

;
n

l,rpyright

by The Berkeley Review

al

The BerkeleY Review Specializing in MCAT Preparation

Biology
83. Which radioactively

Mitosis and Meiosis

Passage Xftr

labeled precursor would be BEST for monitoring DNA synthesis during the S phase of the cell cycle?

86. In the autoradiographs of Family I


gametogenesis?

and Family 2, which parents show an abnormal division during

A. B. C. D.

3H-thymine 3H-cytosine

A.
B. C. D.

Family
Family Family Family

3H-uracil
3H-adenine

(maternal); Family 2 (maternal) (paternal); Family 2 (maternal) (maternal); Family 2 (paternal) (paternal); Family 2 (paternal)

84.

Down syndrome is due to trisomy:

87. The mother of Family 2 has a haploid chromosome number of 23. How many chromatids are present in
metaphase of the second meiotic division?

A. B. c. D.

13. 18.

21. 22.

A. B. c. D.
or

23

46 69 92

85. During nuclear division, chromosomes

chromatids move to the opposite poles of the cell. This is called disjunction.If disjunction were to fail, two chromosomes or two chromatids would go to one pole and none would go to the other pole. This is called nondisjunction. Primary nondisjunction occurs at the first meiotic division, while secondary

nondisjunction occurs
spermatogenesis?

at the second meiotic division. Which of the following diagrams represents secondary nondisjunction in
A.

@@@@
B.

@@oo
o@@@

c.

D.

@ooo
@

Copyright

by The Berkeley Review

a2

The BerkeleY Revier Specializing in MCAT PreParation

Biology
Passage

Lipids: Four Groups

Passage XIV

XfV (Questions 88-94)

Lipids can be divided into four distinct groups: fats rnd waxes, complex lipids, steroids, and prostaglandins .ind leukotrienes. In animals, an important use for lipids is :he storage of energy, especially in the form of fat. If all ;hree alcohol groups of glycerol are esterified to the :ndividual carboxylic acid groups of various fatty acids, a

The most abundant steroid in animals is cholesterol (Figure 3). Cholesterol is found in cellular membranes, and it is also the precursor to many steroid hormones and bile salts.

iat called a triglyceride is formed (Figure


Tnglycerides are simple lipids.
o H?c-

1).

lo
I

on-

cH' cH' cH' A Triglyceride

"lto

fl

Figure 3 Prostaglandins and leukotrienes are synthesized from arachidonic acid, a C20 unsaturated fatty acid (Figure 4).
o

".1-o-!
Figure I

se- [
Arachidonic acid

cH'

Figure 4

Fatty acids attached to the glycerol backbone can be :-ther saturated or unsaturated, and they can be of varying ,:rgths. The Crs saturated fatty acid attached to the ,.1cerol backbone in Figure 1 is stearic acid. Saturated ."ts are solids at room temperature. Unsaturated fats are ;;nerally liquids at room temperature.

Prostaglandins and leukotrienes mediate hormonal


action, and both cause inflammation and fever.

The membranes that surround cells and cellular ,:"anelles contain complex lipids. These lipids can be ::'.ided into two groups: phospholipids and glycolipids.
?:ospholipids contain an alcohol, fatty acids, a phosphate ::lup, and either choline, ethanolamine, or serine. If the

88.

The fat shown in Figure 1 can be described as:

.l:oho1 is glycerol, the phospholipid is called a :;:osphoglyceride. A common phosphoglyceride is :rosphatidyl choline (Figure 2). It the alcohol is .:hingosine, the phospholipid is called a sphingolipid. ,,i-' colipids are sphingosine-based complex lipids that ::rtain carbohydrates like glucose and galactose.
o

A. B. C. D.
89.

carboxylic acid.

an alcohol. an ester.
a ketone.

:,-c-

lo -,|-o-'j I
I o

o-rj

ctt'

The chemical structures shown in the passage are a triglyceride (Figure 1), phosphatidylcholine (Figure 2), cholesterol (Figure 3), and arachidonic acid (Figure 4). The arrangement of these lipids in order

of decreasing polarity is:

t"'
or
CH, -

A. B. C. D.

phosphatidyl choline > arachidonic acid >

. .ICH: !-n-'J-o&T-t"' oo
PhosPhatidYl choline

Figure 2

triglyceride > cholesterol. cholesterol > a triglyceride > arachidonic acid > phosphatidyl choline. arachidonic acid > a triglyceride > cholesterol > phosphatidyl choline. phosphatidyl choline > a triglyceride > arachidonic acid > cholesterol.

lew

--:p1'right @ by The Berkeley Review

a5

ron

The BerkeleY Review Specializing in MCAT PreParation

Biology
90. The following series of

Lipids: Four Groups


92.

tucryeXIV

diagrams represents the hydrocarbon portion of individual fatty acids that

Arachidonic acid can contribute to the formation of:

make up different triglycerides. What is the increasing order of the melting points for the
following four triglycerides?

I. IL Itr. A. B. C. D.

simple lipids. complex lipids.


prostaglandins.
and II only II only III only I,II and Itr

Tryglyceride

I
CHr

93.

cHl cHr

A bacterial culture growing at 38 'C is transferred to an environment where the temperature is 25 'C. All of the following initial responses should be
observed, EXCEPT:

Tryglyceride

II

A.
B.

an increase in the synthesis of unsaturated fatty acids.

decrease

in the fluidity of the bacterial

c.
Tryglyceride

membranes. an increase in the synthesis of long chain fatty


acids. an increase

III
cHr
CH: CH:

D.

in the synthesis of short chain fatty

acids.

94.

Tryglyceride IV A.
B. C.

Hydrolysis of membrane phosphoglycerides might yield all of the following compounds, EXCEPT:

IV,Itr, II II, I, W, III III,IV,I, II


I,

D.

III, I, IV,

il

A. B. C. D.

glycerol.
glucose.

choline.'
serine.

9t.

Elevated levels of cholesterol in the blood serum can lead to plaqueJike deposits on the inner walls of the

arteries, a condition called atherosclerosis. This


disease reduces the diameter of the blood vessels and leads to adverse clinical conditions, such as a stroke or heart attack. Atherosclerosis results from
the:

A.
B.

increased synthesis

of bile salts

from

C. D.

cholesterol. increased synthesis of steroid hormones from cholesterol. removal of excess cholesterol from cellular lipid bilayers. insolubility of cholesterol in water.

Copyright @ by The Berkeley Review

84

The BerkeleY Review Specializing in MCAT Preparation

Biology
Passage

Bacteriophage Lambda
95.

Passage XV

XV (Questions 95'100)

The lambda phage is largely incapable of infecting bacterial species other than E. coli. This is because:

Bacteriophage lambda is a nonenveloped DNA virus begins when the virus is adsorbed to the surface of the hoit cell. Once attached, the phage injects its DNA across the plasma membrane and into the cytoplasm of the E'
rvhose host is the bacterium E. coli. The infection process

A. C. D.

B. other bacterial species have


chromosomes, while E. coli does not'

E. coti has a circular chromosome, while other bacterial species do not.

circular

it is transformed from a linear strand to a :ircular one. After this point, the infection may take one rf two different pathways:

:oli

cell, where

other bacterial species lack a chromosomal


integration site for lambda DNA.

other bacterial species lack specific cell


surface proteins that lambda normally binds.

Lttic Pathway

After injection, the circularized DNA is transcribed ind translated by the host cell's DNA. The resulting products make up the future viral capsid. The ::otein ..mbda DNA then undergoes several rounds of ::plication, after which it is packaged into the newly :ssembled capsids. Multiple new viral particles are : rrmed, ultimitely causing the lysis, or bursting, of the
:.ieased.
-',

-,:st E. coli cell. During lysis, the new lambda phages are 96.

strain of lambda phage exists that produces a defective integrase enzyme, meaning it is unable
to:

sogenic Pathway

::rduction

-{fter injection, the circularized DNA directs the bf a viral protein known as integrase'This ;ir\me causes the circular lambda DNA sequence to -:eErate itself into the bacterial chromosome' Once .:ited, the lambda DNA (now termed a provirus) is "::Licated along with the E' coli host cell. The provirus , remain hidden and inactive in this latent state for *.": -l:ip1e replications. If the E. coli host is damaged by UV

A. B. C. D.

avoid causing the lysis


infection. to infect E. coli. to replicate its DNA. to enter the lytic PathwaY.

of E. coli

upon

;:t -.,.r'.

or ionizing radiation, the latent lambda provirus can itself from the bacterial chromosome and enter ":: -'. tic pathway.

97.

Which of the following statements would represent


an evolutionary selective advantage for the lambda virus?

L II. III.

Upon damage to the host E' coli cell, a lambda provirus can excise itself and enter
the lYtic PathwaY'

Lambda-does not always

kill its host cell

immediatelY' Lambda DNA is almost completely resistant to mutation.

A. I only
B. II only C. I and II only

D. I, II, and

III

_Jrt O by The BerkeleY Review

85

The BerkeleY Keview Specializing in MCAT PreParation

Biology
98. A lambda

Bacteriophage Lambda

Passage XV

f,

DNA strand is labeled with radioactive 32P and packaged into a viral capsid. The newly made virus is allowed to infect an E. coli cell. The

cell does not lyse, but instead begins normal mitosis. At this stage, radioactivity would MOST
likely be detected:

A. B. C. D.

in the nucleus of the E. coli host cell. in the cytoplasm of the E. coli host cell. in the ribosomes of the E. coli host cell. outside of the E coll host cell.

99.

A certain strain of E. coLi produces a cytosolic


endonuclease that cleaves lambda DNA at certain
sites. This should inactivate:

A. only the lytic pathway of infection. B. only the lysogenic pathway of infection.
C.

both the lytic and lysogenic pathways of


infection. neither infection pathway.

D.

100. Lambda phages entering the lytic or lysogenic


infection pathways do NOT differ from each other in their:

A. mode of viral DNA injection. B. mode of viral DNA replication. C. immediate lethality to the host cell. D. ability to remain latent for long periods.

Copyright O by The Berkeley Review

a6

The Berkeley Re Specializing in MCAT

Biology

Structure and Function in Cells and Viruses

Section VI Answers

1.

C is correct. Recall that Gram-positive bacteria do not have an outer membrane, while Gram-negative bacteria do. In addition, the peptidoglycan layer in Gram-positive bacteria is much thicker than the peptidoglycan layer in Gramnegative bacteria. The stain does take advantage of the fact that bacteria have different cell wall structures. The correct choice is C.

B is correct. Activation of the complement system (a system of proteins found in the blood, which participates in the immune response) brings about diffusible factors that stimulate the secretion of histamine from mast cells and basophils. Even if we did not know the details of what complement did, we should be aware of the fact that it takes part in the immune system response. Therefore, it is unlikely that activation of complement is part of the repression
of the inflammatory response. The rest of the answers all lead to suppression of response. The correct choice is B.

3.

C is correct. This function involves the release of chemical messengers that are secreted into the extracellular fluid. Those messengers then act upon the cell that secreted them. This is best describing the self-stimulation addressed in the question. Paracrine function is secretion of chemical messengers that act locally, but not on the cell that secreted the signal. The correct choice is C. C is correct. We should rcalize that sepsis is the result of an acute bacterial infection. Therefore, if an individual is infected with a bacterium that is unresponsive to antibiotics, the infection may becomes acute and lead to septic shock. Thus, we are looking for an explanation for the rise in sepsis. The creation of highly resistant bacteria through the overuse of antibiotics certainly would be a valid explanation for the increase in sepsis. The rest of the choices would all lead to the conclusion that sepsis should be declining: Less invasive procedures should lead to a smaller possibility of infection. Less frequent use of immunosuppresive therapy would not increase a person's susceptibility to infection. Finally, a decline in treatable diseases would not create the potential for resistant strains. The correct choice is C. D is correct. We are told that this CD14 molecule lacks a membrane-binding domain. Therefore, it does not bind to any plasma membrane, regardless of the face. We can automatically eliminate choices A and B. To distinguish between choices C and D, we need to think about the function of CD14. It helps transduce a signal from the outside wodd into the endothelial cell. Therefore, it is not likely to be found already inside the cytosol of the endothelial cell. It is found instead in the blood and extracellular fluid of the tissue. The correct choice is D.

4.

5.

6.

B is correct. From the picture in the question, we learn that the rough microsomes fall below in the sucrose gradient when compared to smooth microsomes. This must mean that they are more dense. Choice C is eliminated. The rough microsomes have ribosomes attached all over them. They are not made from radically different proteins. Recall that ribosomes are made up of RNA and protein. This will certainly have the effect to add more mass without affecting the volume of the ribosome as much. The net effect is that the rough microsomes are more dense and fall lower than the smooth microsomes in a sucrose gradient. The correct choice is B.

7.

A is correct. In the passage, it is stated that the rough microsome always has on its outside surface the ribosomes. This is implying that the topology of the ER is conserved. In other words, the outside of the rough ER in an intact cell is the cytosol. The ribosomes that are attached to the ER are attached on the cytoplasmic face of the ER membrane. This case is no different for the microsomes. They have not been turned inside out. Therefore, it becomes clear that the exterior of a rough microsome is equivalent to the cytosol. Consider the other choices. The lumen of the nucleus is not correct. The lumen of the nucleus is enclosed by the nuclear membrane, and it is not
continuous with any other cellular fluid. The lumen of the ER is of course equivalent to the inside of the microsome. That is why we take advantage of these mini-ER systems for experiments. Finally, it is evident that the lumen of the Golgi has little to do with the exterior of the microsome. Choice D can be eliminated. The correct choice is A.

8.

B is correct. This,problem requires that we think about the processing that occurs on a protein that is being transported into the'ER. It is stated in the passage that the signal peptide that is responsible for bringing that protein into the ER is cleaved once in the ER. Therefore, proteins that make their way into the ER will be shorter than they would have been if they had remained in the cytoplasm. Since microsomes are mini-ERs, the same holds. In the
absence of the microsome, the protein should be longer, and indeed this is the case. Again, the signal peptide is not cleaved if the protein does not enter the ER. Based on this information, all other answers can be dismissed. The correct choice is B.

rpyright

by The Berkeley Review

a7

The Berkeley Review Specializing in MCAT Preparation

Biology
9.

Structure and Function in Qells and Viruses

Section W Answers

B is correct. We are told from the passage that the SRP binds to the signal peptide and is ultimately responsible for bringing the protein to the ER membrane. Let us think about what would happen if the SRP was no longer there. Would *. t." an increase in the frequency of the secreted proteins? No, we would see a decrease since the proteins that are being translated cannot make their way to the ER. This eliminates choice D. Would we see a loss in the signal peptid"Z No, nothing would happen to this sequence on the protein except that it would no longer bind to the Snp U"cause there is not one available. Therefore, we can choice C. Would we see a quicker translational arrest?

Assuming there is no SRP around, we would not see any translational anest because binding of the SRP to the signal peptide ii responsible for the arrest. We can eliminate choice A. We would see the elimination of the ability to import proteins into the ER. The correct choice is B.

10.

A is correct. By having a translational arrest, the cell is ensuring that a protein that is needed on the outside of the cell (or some organellelnside the cell) will not be released into the cytosol. If the protein is no longer being made on

not the case for all of those proteins that carry out their function within the cytosol. We can eliminate choice C. There is no evidence in the- passage or our body of knowledge that such a translational arrest is a means of proofreading. Therefore, the most likely reason lies in the idea of ensuring proteins that are destined to be secreted are not inappropriately released into the cytosol. The correct choice is A.

the ribosome, this is a solid means of insurance. Consider the other choices. There is no evidence that the ribosome is non-functional. It will translate the protein just fine, but its function is regulated by the SRP. Therefore, we can eliminate choice B. The notion that all translation must occur on the ER membrane is ridiculous. This is obviously

ll.

B is correct. The Golgi stack has two distinct faces: a cis face (or entry face) and a trans face (or exit face). The cis face is closely associated with the transitional elements of the ER while the trans face is distended into a tubular reticulum known as the trans Golgi network. Proteins and lipids enter a Golgi stack in small vesicles from the ER on the cis side and exit for various destinations in vesicles from the trans side of the Golgi. Based on this information, one can easily eliminate choice D. The medial region of the Golgi is the region in between the cis and the trans face. Therefore, on" can easily eliminate choice C. There is no lateral region of the Golgi complex, so this choice can easily be eliminated. The correct choice is B.

12.

C is correct. Collagen is an animal connective tissue protein. It is at the bottom of the list in the table. It does not
meet the ideal protein requirements. Statement I is not true. Soybeans do meet the ideal protein requirements, as seen in the table. Statement II is true. Lentils do not meet the ideal protein requirements for sulfur-containing amino acids. Statement III is true. The correct choice is C. amino acid that containJ sulfur. Choice A is correct. Neither choices B, C, nor D contain sulfur. The correct choice

13.

A is correct. The
is A.

passage indicates that beans are usually low in the sulfur-containing amino acids. Methionine is an

14.
15.

C is correct. Choice A is phenylalanine. Choice B is tryptophan. Choice C is lysine. Choice D is histidine. The
correct choice is C. D is correct. We can read this answer from the table. Compare the values for each protein food to the ideal standard given at the top. Collagen is deficient in all five of the listed amino acids. Choice A is incorrect. Soybeans have idequate levels of the l]sted amino acids. Choice B is incorrect. Cornmeal has 3 limiting amino acids. Choice C is incorrect. Lentils have2limiting amino acids (count the sulfur-containing amino acids as one choice, since only methionine is actually needed by the body). The correct choice is D. B is correct. First, compare the amino acid pattern to the ideal. This is not an ideal protein, so it is probably not fishwhich is a "complete" piotein. Choice D is incorrect. Beans, as we learned in the passage, are usually low in sulfurcontaining aminb acidi. The unknown is not deficient in these, so choice A is incorrect. Milk also meets the ideal protein st-andards, not the pattern shown. Choice C is incorrect. Grains are low in lysine, as is the unknown. The bther values are similar to cornmeal and oatmeal, both of which are grains. The correct choice is B.

16.

17.

A is correct. The opposite of essential is nonessential. In the passage, valine, isoleucine, and histidine are indicated
as essential.

This means phenylalanine is nonessential. The correct choice is A.

Copyright @ by The Berkeley Review

a8

The Berkeley Reviev Specializing in MCAT Preparation

Biology
18.

Structure and Function in Cells and Viruses

Section VI Answers

incorrect. HPLC is used to separate compounds based on their molecular characteristics. Bactbria are not separated this way. Choice D is incorrect. The correct choice is C.
19.

C is correct. The Gram stain is commonly used in microbiology. It involves staining samples with a purple dye, such as crystal violet, using iodine as a mordant, and then staining with a pinkdye, safranin.-The differen."i in..tt wall structure allow characterization of Gram-positive and Gram-negative bacteiia. This gives the clinician a broad approach for treatment. Not all bacteria would grow under anaerobic conditions. Choice A is incorrect. An immunoassay binds specific molecules, but would not be used to classify Gram-negative or positive. Choice B is

correct choice is D.
:r).

D is correct. The death of Gram-negative bacteria leads to the release of endotoxin from the bacterial cell wall. As the antibiotic kills bacteria, they would release more endotoxin. At some point, endotoxin levels would rise and then fall if the antibiotic were able to control the bacterial population. Choice C is incorrect. Choice B is incorrect. The antibiotic does not bind or neutralize the endotoxin. Choice A is incorrect. Choice D is the correct choice. The

B is correct. At the dosage of l0 ug, the mortality was \Vo, so all the mice in this group survived, even though TNF was inhibited only 55Vo. Choice A is true. As E5531 levels were increased by group, the plasma TNF concentration decreased. Choice C is true. Inhibiting TNF increased survival rates. Choice blr t.u.. The dose of LpS given killed all the control group. The dose was selected to do this, not to kill only 7 5vo of the control group. choice B is false, and is the answer we want. The correct choice is B.

:1.

C is conect. Neither the antibiotic nor the antagonist alone provided any benefit in terms of long term survival. Choices A and D are incorrect. In the combined treatment, the antibiotic seemed to work together with E553 l. We cannot make a judgment on the resistance of the bacterial strain based on this data. Choice B is incorrect. The combination of E5531 and the antibiotic provided the only beneficial effects in terms of survival 50 hours later. Choice C is correct. The correct choice is C.

D is correct. The purpose of the antibiotic is to decrease the number of bacteria. The purpose of the endotoxin antagonist is to decrease unfavorable changes caused by the response of cellular -"diuiorr to the endotoxin produced. Blood bacterial count and endotoxin concentration should both decrease with this regime. Choice I is incorrect. The goal is to decrease cellular mediators such as TNF. Choice II is incorrect. This stratigy will not work if the bacteria are resistant to the antibiotic. The endotoxin antagonist has no direct effects on bacierial number or reproduction. Choice III is incorrect. Choice D is the correct choice, none ofthe above. The correct choice is D. .{ is correct. The similarity in structure between Lipid A and E553 I should suggest that E553 I can bind to the same cellular surface receptors in the host. It is probably a competitor that does not elicit the same cellular responses. Statement I is correct. Usually enzymes (proteins) are responsible for cleavage, not lipopolysaccharides. Statement II is incorrect. The release of Lipid A as part of the LPS is usually as a response io ieit damage or death (see passage). E5531 combats the effects of release of LPS, not its release directly. Statement III ii inconect. The correct choice is A.

;JA,

:rcept ribosomes. The correct choice is C.

C is correct. The question is asking us to think about bacterial structure and organization with reference to -rganelles. The cytoplasm of bacteria, unlike eukaryotes, contains no internal compartments and no organelles

B is correct. The answer can be derived from a careful read of the passage. We are told that the bacteria that causes :rs disease normally colonizes on the human throat. However, the disease begins only when the bacteria invades the :-ood stream and eventually reaches the cerebral spinal fluid. This statement impliis that the bacteria must cross : 'er the lining of the throat to reach the blood stream. Therefore, the layer of epithelial cells that lines that r rrnally serves as a barrier of penetration to the blood stream. Once that barriei is broken, then the disease throat takes :," 1.J. The correct choice is B.
.ffiitrr,,

D is correct. The concentration of myelin sheaths make tracts of nerve axons white. The rest of the CNS

the brain, white matter is located in the inner region, whereas in the spinal cord, white matter is located ;: the exterior. This answer cannot be derived from the passage (meaning th-e question tests your personally s-';-rired knowledge), and in fact is true of all brain and spinal cord tissue, regirdless-of whether a person-has spinal merrngitis. The correct choice is D.

*-.r. Within

appears

by The Berkeley Review

89

The Berkeley Review Specializing in MCAT Preparation

Biotogy
)'l

Structure and Function in Cells and Viruses

Section VI Answers

B is correct. We are told from the question that this lack of complement is unlikely to be a cause of an epidemic. We are now looking for a statement that will back this claim up. The second answer claims that in countries where there is a high frequency of epidemics, persons with this lack of complement are not commonly found during the epidemic. This certainly qualifies as evidence for the claim that the genetics is not a significant cause of an
epidemic. Statement A is completely the opposite, so it can easily be eliminated. The statements about the rise and fall of antibody levels are true, and research into the cause of these fluctuations continues. However, these statements do not offer us support for the claim made in the question. Again, our best evidence comes from the fact that we do not see a lot of persons afflicted with this lack of complement during spinal meningitis outbreaks in countries where epidemics are common. The correct choice is B.

28.

C is correct. N. Iactamica is a relative of N. meningitidis. Therefore, it is not highly unlikely that they may be similar in structure in some ways. Children who are infected by N. lactamica produce antibodies against this
organism. We are told that children who have had this infection are shown to be protected against spinal meningitis. Therefore, it is logical to claim that those antibodies against N. lactamica are effective against N. meningitidis. Since they are relatives of each other, they very well may have similar antigenic determinants (eliminate choice D) and therefore antibodies against one may be effective against the other. There simply is no evidence in the passage or logic to lead us to believe that choices A and B are correct. The correct choice is C.

to

B is correct. One should realize from that passage that the cerebral spinal fluid is a culture medium for the rapid growth of the bacteria. This increase in the population of the bacteria causes inflammation of the meningeal lining. The swelling of the membrane is what causes the stiff neck. It also causes fever, headache, and potentially coma.
While this answer cannof be directly ]ifted from the passage, one should be able to relate the growth of the bacteria to swelling of tissue. Furthermore, one may take a process of elimination to this answer. For choice A, one has to ask themselves why the endotoxin from the bacteria affect only neck muscle. This is not very likely. For choices C and D, one has to ask themselves how the bacteria would cause stimulation of nerve cell innervating the neck muscle. There is no evidence in the passage for such interaction, while there is information regarding the growth potential of bacteria in the cerebral spinal fluid. The correct choice is B.

30. C is conect.

The perinuclear space is the space between the inner nuclear membrane and the outer nuclear membrane. It is stated in the passage that the outer nuclear membrane is studded with ribosomes. This should indicate that the membrane of the ER is continuous with the outer nuclear membrane. Therefore, the space inside the "ER, the lumen, should be continuous with the periplasmic space. The periplasmic space cannot be continuous with the nucleoplasm, because of the division by the inner nuclear membrane. Thus, choice A is eliminated. Furthermorethe periplasmic space is divided from the cytosol by the outer nuclear membrane. Therefore, this eliminates choice B. Finally, the mitochondrial matrix is in no way continuous with the periplasmic space. They are separated by many membranes and should not be construed as continuous. The correct choice is C.

31.

B is correct. The nuclear envelope ensures that translation of a particular mRNA polymer into a protein occurs in the cytosol and not the nucleoplasm. One can gather this from the passage by considering the resting size of the pore and size given for the ribosome. The ribosomes is clearly too large to fit into a nuclear pore, and there would not be a nuclear import signal on the ribosome. Therefore, translation is separated and confined from the nucleus. Consider the other answers. There is no evidence that the nuclear envelope ensures DNA replication. While nuclear envelope breakdown is a part of the mitotic cycle, the envelope does not provide a means of insurance. We can eliminatc choice A. Choice C is false. We know that ribosomal RNA is produced in the nucleus and the nuclear envelope harr no known control over that process. Therefore, this answer can easily be eliminated. Finally, the nuclear enve as stated above, breaks down during mitosis, not the interphase portion of the cell cycle. This makes choice D incorrect. The nuclear envelope ensures that translation remains in the cytosol. The correct choice is B.

32.

C is correct. First, the statement that nuclear components cannot be radiolabeled is simply not true. Think labeling nucleotides that will eventually be incorporated into a nucleic acid polymer. Since this statement obviously false, it can easily be eliminated. We have no evidence from the passage that nuclear components difficult to isolate. We cannot make this assumption. Therefore, this is not the best answer, and choice B can eliminated. Let us look at choice D, which states that nuclear components are smaller than the nuclear implying that they would not be good measures of the size of the pore. This statement is also false. Think a DNA and RNA polymerase. These molecules are very large and must be imported into the nucleus. Since
@

Copyright

by The Berkeley Review

90

The Berkeley Specializing in MCAT Prepara

Biology

Structure and Function in Cells and Viruses

Section VI Answers

statement is false, choice D can easily be eliminated. This leaves us with choice C. The nuclear pore, according to the passage, opens up beyond its resting size to accommodate those molecules that are larger than the pore size, yet are necessary inside the nucleus for proper function. Therefbre, it would not be a wise experimental choice (if one is trying to measure the non-accommodating resting size of the pore) to use nuclear components. The correct choice is C.
33.

D is correct. It is clear from the question that only the tail portion entered into the nucleus. Since the tail portion is attached to a 20 nm piece of gold, the nuclear pore must have opened up to accommodate the piece of gold. Therefore, we can safely assume that the nuclear import signal is found on the tail piece. Since the gold attached to the head piece did not make it into the nucleus, we can saf'ely say there is no nuclear import signal on the head portion of the protein. Therefore, we can eliminate choices A and B. The next question is to decide the makeup of the amino acids in the nuclear import signal. The passage tells us the signal is rich in positively charged amino acids. The amino acids Phe and Trp are not positively charged. However, the amino acids Lys and Arg are basic, positively charged amino acids. Therefore, they are likely candidates fbr the residues making up the nuclear in-rport signal. The correct choice is D. C is correct. We are told from the passage that the nuclear import signal can be found anywhere within the protein and still be functional. This implies that the location of the signal is unimportant. Based on this informarion alone, we can eliminate choices B and D, because they claim the location of the signal is important. The question is telling us that a nuclear import signal is attached to random amino acids onto an enzyme that usually finds its home in the cytosol. In time, that enzyme should be inside the nucleus, as the protein now has a nuclear import signal. Based on that information, we can eliminate choice A, which claims the molecule remains within the cytosol. Therefore, the molecule ends up inside the nucleus, because the location of the nuclear import signal is unimportant. The correct choice is C. D is correct. The passage tells us that during mitosis, the nuclear envelope breaks down. It goes on to inform us that after the nuclear division has taken place, the nuclear envelope begins to refbrm around the chromosomes. The passage states that many of the previous molecules of the nucleus are exiled when the envelope reforms. However, in time, these previous nucleus dwellers find their way back into the nucleus. It is that last sentence which is the most important in answering this question correctly. If those molecules that were once inside the nucleus flnd their way back to the nucleus after the envelope reforms, the nuclear import signal must not be lost on these molecules. In
other words, they must have the signal to get back into the nucleus. If these indeed are the same molecules that were previously in the nucleus (meaning they were not resynthesized), the signal must have remained. Consider the other answers. We cannot believe that all molecules that reside inside the nucleus are resynthesized after every nuclear division. We have no evidence for this claim; and furthermore, it makes little sense. because it seems a terrible waste of energy. Thus, we can eliminate choice A. Choice B can be eliminated, because it is false. The molecules from the nucleus are not destroyed. Based on the discussion above, we can eliminate choice C, because the molecules most likely retain their nuclear import signal. The correct choice is D. C is correct. We are told from the question that a mcllecule that is placed in the nucleus of a frog oocyte is exported out into the cytoplasm. Next, we are told that the same molecule, when placed directly in the cytosol, remains there. This implies that the molecule has a nuclear export signal and it does not have a nuclear import signal. In other uords, it can only leave the nucleus, and not reenter. Based on this information, one can easily eliminate choices A and B. The question then becomes where is the receptor that will recognize this nuclear export signal. Will it be on the cytoplasmic face of the nuclear membrane, or the nucleoplasmic face? The answer seems obvious. In order to recognize the nuclear export signal, the receptor must be facing the inside of the nucleus. Therefore, the receptor must be on the nucleoplasmic face of the nuclear membrane. The correct choice is C.

.i.{.

B is correct. This question is designed to test your basic knowledge of the bacterium Escherichia coli (abbreviated t> E. coli). This prokaryotic organism is rod-like (bacilli), with a length of about 2 prm and a diameter of about lm. Bacteria are classified as either being Gram-positive or Gram-negative. If a bacterium takes up the Gram stain a crystal violet dye and iodine), then it is said to be Gram-positive. If it does not take up the Gram stain, it is Gram:-gative. Surrounding the cytoplasm of a prokaryotic cell is the plasma membrane. Surrounding the plasma rrembrane is a peptidoglycan (murein or cell wall) layer which consists of covalently linked polysaccharide and :.rl)'peptide chains.
1

;
,mr

by The Berkeley Review

9l

The Berkeley Review Specializing in MCAT Preparation

Biology

Structure and Function in Cells and Viruses


Extracellular Face

Section VI Answers

Peptidoglycan

(cellwall)

f 1 Plasma I memb.ane f
Gram-positive bacterial
(e.g., Staphylococcus

Periplasmic gel

Cytoplasmic Face

wall aureus)

Gram-negative bacterial wall (e.g., Escherichia coli)

In the Gram-positive bacterium the peptidoglycan layer is quite thick (about 250 A) while in the Gram-negative layer it is rather thin (about :041. fnis is where the basic similarities between the two types of bacteria end Surrounding the peptidoglycan layer of Gram-negative bacteria is an outer membrane. The space between the
plasma membrane and the outer membrane of a Gram-negative bacterium is often called the periplasmic space (or" more correctly, the periplasmic gel). The correct choice is B. 38.

D is correct. Al1 hydrolysis reactions are favorable. Hydrolysis of this phosphatidylethanolamine will give palmiuc
acid, oleic acid, phosphate, ethanolamine, and glycerol. These individual structures are shown below. As we learn later, each of these components will become important in metabolism. The correct choice is D.
o o ll o il

uill

HrCHC-

lft

il O- CO

(CH2)r4CHj

H)C4 H.O
(CH2)7CH3

OH

HO- C-

(CH2)|4CH1

Palmitic acid Oleic acid

HH - C(CH2h

lo Itt HrC"l

- C: C-

tt

HH
(CHz)r

HCI

OH

HO- CHO_ P_ o il
I

- C: CHO-

tl

(CHzhCH3
@

o
O- P- OCH2CH2NH-,

H2C-

OH

OH

CH2CH2NH3

o6
A phosphatidylethanolamine

Ethanol

oq
Phosphate

Ethanolamine

39.

C is correct. In bacteria, membrane lipids are synthesized primarily by integral membrane proteins. The question m whether that synthesis occurs on the extracellular face (outside) or cytoplasmic face (inside) of the membrane. As u membrane lipid is being synthesized it begins to incorporate molecules like glycerol, fatty acids, ethanolamine, anC phosphate. In this case the phosphate which is incorporated into the membrane lipids is radioactively labeled.
Extracellular Face

Q rNnS complexed with


membrane lipids that

,o*ion
rinio

of

ulaverl

contain PE

Q
Cytoplasmic Face

Rudiou"tively labeled
membrane lipid

would be expected to be radioactively labeled and some would bind TNBS. This was not observed. If TNBS c cross the cell's plasma membrane, it should be able to bind to radioactively labeled PE which was being syn on the cytoplasmic face. Since it was observed that none of the PE molecules which were radioactively labeled '* labeled with TNBS, it must mean that TNBS cannol cross the cell's plasma membrane and that the membrane lin Copyright @ by The Berkeley Review

If TNBS is added to the cell suspension, we would expect it to bind to PE on the extracellular surface as well as PE on the cytoplasmic face. If membrane lipids like PE were synthesized on the extracellular face, some of
t

92

The Berkeley Re Specializing in MCAT

Biology

Structure and Function in Cells and Viruses

Section VI Answers

Remember, membrane lipids have head groups which are polar and part of that polarity is due to the negatively charged phosphate groups. Like charges repel one another. The correct choice is C.
40.

are being synthesized on the cytoplasmic face. TNBS cannot cross the membrane because is bears a negative charge.

B is correct. First, go back and read the previous answer. TNBS is still negatively charged when it is added to the bacterial suspension. Therefore, it still cannot cross the bacterial membrane. This will eliminate choice A. Membrane lipids like PE are synthesized on the cytoplasmic face, not the extracellular face. This eliminates choice C and D. This leaves us (by default) with choice B.
Extracellular Face

fNas

complexed with

membrane lipids that

contain radioactively

dilTil:.{
Q
Cytoplasmic Face

labeled PE

Rudiou.tively labeled
membrane lipid

As outlined in the passage, phospholipids in membrane bilayers will take days to undergo a flip-flop (i.e., transverse diffusion). Just because the phospholipids take days to flip-flop does not mean that membrane lipids like PE cannot flip-flop in a shorter period of time. A relevant diagram for Experiment 2 is shown below. The correct choice is B.
11.

B is correct. The passage states that integral proteins are tightly bound to membranes by hydrophobic forces. Those hydrophobic forces arise from the interaction of the non-polar interior of the lipid bilayer and the hydrophobic amino acid residues in that portion of the protein which is in contact with that non-polar interior. The only hydrophilic forces of interest between the integral protein and the membrane will occur at the polar regions of the lipid molecules. These forces, if they exist, are not as great as the interior hydrophobic forces (i.e., the exclusion of water). Hydrogen bonds are mainly electrostatic interactions, and they occur in the limited polar region of the membrane. Choice A and D are eliminated as possibilities, and choice C becomes a good candidate for the answer.

Outside

However, let's consider choice B. As stated in the passage, both the lipids and the proteins found in a membrane are asymmetrically distributed. This means that an integral protein is different on one side of the membrane than it is on the other side of the membrane. For example, the C-terminal region of a protein can be found on the inside of a cell, rvhile the N-terminal region can be found on the outside. The N-terminal region on the outside of the membrane might have more amino acids associated with it while the C-terminal region might have fewer amino acids. Also, carbohydrate residues can be found attached to the N-terminal domain (on the outside of the cell).

It turns out that it is the asymmetry of the integral protein that makes it so difficult for transverse diffusion to occur. The hydrophilic domains of the integral protein not associated with the interior of the membrane have a difficult time rotating and trying to pass through a hydrophobic interior. If transverse diffusion were to begin, the hydrophobic portion of the protein and the hydrophobic portion of the membrane would still (for the most part) remain in contact with one another. The correct choice is B.

irr",*:-:t O by The Berkeley Review

93

The Berkeley Review Specializing in MCAT Preparation

Biology
42.

Structure and Function in Cells and Viruses

Section VI Answers

According to the pasiage, the bacterium Escherichia coli contains 0% cholesterol in its membrane. Since cholesterol choices A and D as answers. A decrease in the membrane's fluidity means that the is not present, "ii.ninit" stiffer. If there were long-chain fatty acids with cis double bonds in the membrane, this membrane is becoming would introduce a kinl in the fatty acid and not allow for close packing of the hydrocarbon tails. The membrane tends to be a bit more fluid. Eliminate choice B. If we increase the length of the hydrocarbon tail of a saturated fatty acid, this would allow for more hydrophobic interactions between the individual methylene (-CHz-) groups. The more hydrophobic interactions, the tighter the packing of the hydrocarbon tails, and the less fluid the membraneThe correct choice is C. 43.

C is correct. The question asks about the fluidity in the membrane of a bacterial cell, not a eukaryotic cell.

D is correct. Spermaceti has two long hydrocarbon tails, which can pack together quite well. As mentioned in the question, this wix melts between 42-41 "F. It tends to be more of a fluid than a solid above this temperature rangeAs the sperm whale dives to great depths, the temperature of the water begins to drop to the low 30' F range. Cold
volume de-creasis. A decrease in volume is related to an increase in density. Recall that the density is the ratio of an object's mass to its volume. As the whale's density increases, it is able to stay at these great depths for longer periods of time without expending considerable energy.

sea watei is circulated through special chambers in the whale's head. Since the cold water has a temperature below the melting temperature of the wix, the wax begins to solidify. As the wax goes from a fluid state to a solid state, its

In other words, by controlling the shape of the wax the whale controls its own buoyancy. This is analogous to thc weights that a scuba diver uses on her weight belt as she descends into the water. When the whale ascends, the temperature of the cold water in the chambers increases because of the increased circulation of warm blood to the
head. The correct choice is D.

44. A is correct. The passage states the amino acid residue is modified

after the protein is synthesized. Only postproduction of a template m-RNA for proteir is the Transcription is complete. translation indicates synlhesis synthesis. Choice B is incorrect. Replication is the duplication of DNA in the nucleus. Choice C is incorrecl
Iiegradation means the protein is destroyed. Choice D is incorrect. The correct choice is A.

45.
46.

D is correct. Choice A is hydroxylysine. Choice B is lysine. Choice C is hydroxyproline. Choice D is proline.


correct choice is D.

D is correct. The nonstandard amino acids are ones not specified by the standard tRNA codon table. They
modified versions of the standard amino acids (modified after translation). Therefore, there are no tRNA corresponding to the nonstandard amino acids. The correct choice is D.
c

47.
48.

B is correct. The box marked II indicates the carbamoyl moiety. Look at the molecule carbamoyl phosphate: C=Ol-phosphate. The carbamoyl moiety is in brackets. That's your answer. The correct choice is B.

Thus, proline is incoiporaGd and then modified to hydroxyproline after the peptide is made. This

B is correct. From the passage, we learned that hydroxyproline is formed by modification of a synthesized

hydroxyproline itself would not be incorporated. Choices A and C are incorrect. Proline is not further broken in the digestive ffact, so choice D is incorrect. The correct choice is B.

49.

A is correct. The liver contains the enzymes of the urea cycle and produces urea from

excess amino not make urea kidney does urine, the in the excretion to its way Although urea passes through the kidney on C and D are i Choices in urea biosynthesis. has a role pancreas the gill or bladder the B is inJorrect. Neither The correct choice is A.

50.

D is correct. Collagen would not be synthesized properly if hydroxyproline were not available due to impair activity of prolyl hydroxylase. Collagen is a major component of connective tissue and plays an important role
blood vessel str.ngth, healing of wounds, and healthy gum tissue. The correct choice is D.

All

the symptoms listed are symptoms

of

Copyright @ by The BerkeleY Review

94

The Berkeley Specializing in MCAT Pre

Biology

Structure and Function in Cells and Viruses

Section VI Answers

D is correct. This relationship is quite important and stems from the fact that it is often necessary to express the
concentrations of hydrogen and hydroxide ions in an aqueous solution. These concentrations can be quite awkward to work with (e.g., 10 M to l0-14 M) and so a common logarithmic notation (based on logro) is used for simplicity. One of the rules of logarithms states that for a given quantity X,

pX=log!=-logX
X
This allows the [Ho] to be expressed as, pH = log " ' or even the [OHe] as, = - log [H* -f [H*l

pOH-log-l
IOH
]

=-loglOH-l

Natural logarithms have as their base e = 2.'l 18 and are exponents to which e must be raised to give a number. These are quite different from common logarithms. The correct choice is D.

it

D is correct. Physiological pH is considered to be 7.4. All of the common 20 amino acids at physiological pH will
have their s-amino group in the protonated state and their cr-carboxyl group in the ionized (deprotonated) state. This is because the average pKn of the c-amino group is about 9.5 and the average pIQ of the cr-carboxyl group is about 2.2. At physiological pH many of these amino acids will be in their zwitterionic form (where the net charge on the amino acid adds to zero). Since each amino acid will bear at least two charges at physiological pH, they cannot be nonpolar and they cannot be monovalent. The correct choice is D.

53.

B is correct. Since all of these amino acids will be found near the center of a long polypeptide chain, their s-amino group and a-carboxyl group will be tied up in a peptide linkage. The only protons that are able to ionize are those on amino acids with ionizable side chains. There are only 7 amino acids with ionizable side chains, and three of them are listed as answers. The one amino acid which does not have an ionizable side chain is isoleucine. The correct
choice is B.

-i.1.

C is correct. The two pKa values for leucine were given in the passage. Leucine's cr-amino group has a pKa of 9.6 while its c,-carboxyl group has a pKa of 2.4. The pI can be calculated as follows:

ot=W=9Lr2a=6l
Note that leucine will carry no net electric charge at its pI. In the case of leucine this is its zwitterionic form (it is a dipolar ion). The correct choice is C.

i5.

D is correct. The only difference between phenylalanine and tyrosine is the presence of a hydroxyl (OH) group on
the ring.

-L:lli

A carboxylation reaction would have added a CO2 group. A hydrolysis reaction would have used water to break
(iyse) a molecule. The elements of water would have also been added to the products. A hydration reaction involves

t|eu

':

.,ght O by The Berkeley Review

95

toll

The Berkeley Keview Specializing in MCAT Preparation

Biology
D. 56.

Structure and Ftrnction in Cells and Viruses

Section VI Answers

the addition of water to a molecule without lysing that molecule. Since we do not see any of these three types of reactions in the conversion of phenylalanine to tyrosine, it must be a hydroxylation reaction. The correct choice is

D is correct. This means that certain amino acids within a protein can have side chains with dissociable hydrogen atoms. The pKn's of these amino acid side chains can range from about 3.9 (aspartic acid) to about 12.5 (arginine).
This is quite a wide range with which a protein can donate or accept hydrogen ions (i.e., act as a buffer). Proteins can form secondary and tertiary structure through internal hydrogen bonding interactions. This occurs through amino acid interaction within the polypeptide chain itself. Once a protein is in its tertiary form it will stay in that form until it is denatured. Protons in the medium surrounding the protein are (for the most part) not involved in the hydrogen bonding that dictates the secondary and tertiary structure of a protein. If a protein were denatured (by hydrolysis), the element of water would add across the peptide bond being broken. Once this happened the ability of protons to bind to a free terminus would be diminished. However, protons can add to the N-terminus and the Cterminus of a protein. This action only absorbs (at most) two protons. Many more protons could be absorbed by amino acids that have ionizable side chains. The correct choice is D.

61

62

57.

C is correct. The key to answering this question is to locate all of the amide bonds (see the dots . in the structure below). Each amide bond will require I molecule of water. After hydrolysis we will get 4 compounds. They are
glutamic acid (Glu), histidine (His), proline (Pro), and ammonia (NH:).

pyro-Glu His
HHOHOHO
\ I

pro

r lla I lla ot r il. H- N- C- C- N_ C- C-N - C- C- NH"

o=c.,./ "

t.

Fn,
I

u- N\zN -H
Note that glutamic acid in the structure is referred to as pyro-Glu. This is because its side chain has condensed with its own o-amino group to form a ring structure involving an internal amide bond. Hydrolysis of this bond simply exposes the glutamic acid side chain. The correct choice is C.
58.

o /-\

A is correct. Hydrolysis of the internal amide bond in pyro-Glu and the terminal amide bond of Pro will allow
see the

us to

tripeptide in its "true" linear form.


N-teminus C-terminus

o HHOHO il I r rl r il or H- N_ C- C- N_ C- C_ N- C- c-

Ii l" i f", (9 f"' H-N\zN-H . l=


"o."_ro

oH +

NH3

IV

The N-terminus is the cr-amino group of Glu. The correct choice is A.

EE
ffi.
I I

59.

B is correct. To answer this question, one must be aware of the following idea: A drop in temperature will cause bilayer to become more viscous. The increased viscosity will cause a decrease in the fluidity of the membrane. Sincc the proteins are embedded inside the lipid bilayer, the time it takes to react the state in Figure 2 should increase to the decreased fluidity of the membrane. In fact, organisms such as bacteria and yeast will change the compositi of the lipids making up their biological membrane in responses to drops in temperature. Based on this informati all other answers are easily eliminated. The correct choice is B.

Copyright @ by The Berkeley Review

96

The Berkeley Specializing in MCAT

Biology
60.

Structure and Function in eelts and Viruses

Section VI Answers

C is correct' From the passage' we known that monovalent antibodies are used in this process. Normally, antibodies are bivalent, that is, they have two arms, each of which is capable of binding t" i"iig""ic determinant. In our immune system response' such anatomy is beneficial because antibodies "" cari cross-link"antigen molecules into a large lattice as long as.each antigen molecule has three or more antigenic,determinants they usually do). i*hi"r, However, in this experiment, one can imagine^that_cross-linking between fluorescent antibodies can greatly affect the movement of proteins and thus the resulis of the lateral diffustn rates. The correct choice is C.

61.
I

B is correct. At t = 0 minutes, the labeled antibodies of the mouse and human are on their own halves of the hybrid cell' At t = 40 minutes' it is clear that the antibodies are mixed. The only way the .ouio occur would be if the -i*lng lipid molecules readily exchange places with their neighbors within a monollyer. This riovement is known as lateral diffusion, and this experiment provides evidence for such a process. The correct choice is B. D is correct. This answer requires one to think about the experiment and interpret the graph. The graph shows that the asymptotic level of fluorescence reached is lower than ihe original level of fluore"scerrc". why could this be? Look for the obvious. In the experiment, we use a laser beam to bleach some of the fluorescent ligand. We will not be able to recover our original amount of fluorescent because we have permanently bleached the ligand. we can assume that the bleaching is permanent and not temporary on two accounts. There ii no evidence from the passage that demonstrates that the bleaching is only temporiry. Furthermore, if the bleachi"g i; ;"i; remporary, over time we should see the level recover to its original levei. we do not see this in the lraph, a, ti,e level reached is asymptotic' Therefore, we can eliminate choice c. There is no evidence for choices i utio b in either the passage or our body of knowledge. The correct choice is D.

62.

63.

D is correct' This question requires graphical interpretation. The longer it takes for the recovery process to occur, the smaller the diffusion rate. Since all of the experiments *"." .u.ii"d out in simila. .onditionr, we are really looking a diffusion coefficien-ts. The greater thq rate of recovery, the greater the diffusion coefricient of the membrane glycoprotein. Therefore, it becomes clear that it takes glycoproteln D the longest u*oun, of time to reach the full rate of recovery. Therefore, glycoprotein D must have th-eiowest diffusion rate and thus smallest diffusion coefficient. The correct choice is D. D is-correct. The question requires us to think about several things. First, there is no evidence that the antibodies used are lighter in the in vitro system. Furthermore, we have no-evidence for a correlation between weight and diffusion rates. Since we do not have this information, we cannot make the assumption. it.r.ior", we can eliminate choice A' Next, we are not told of any switching to a bivalent antibody. Therefore, we have no reason to believe that cross-linking would be occurring. we can eliminate choice B. So ntw we are left to think about oligosaccharide interaction on the surface of cell membranes. First, the oligosaccharides are found on the extracellular face, and not the intracellular face, of the plasma membrane. This alonJ eliminates choice c. Let us finish the conclusion. with only one glycoprotein, there is no interaction.betwe.en^ these bulky oligosacchario" groufr as the proteins are difTusing laterally. The less interaction leads to higher diffusion rates. The iorrect choicJis D.
C is correct' The question requires us to know a little about cholesterol and its role in our biological membranes. At the high concentrations that cholesterol is found in eukaryotic cells, it acts to increase the fluidity of the membrane. while this may seem counter-intuitive, it is indeed the case. Therefore, one can easily eliminate choices A and B. bulky cholesterol as getting in the way with its large, hydrophobic, classic"steroid rings. Therefbre, cholesterol acts to increase the fluidity of the membrane by inhibiting hydroc-arbon chain crystallization. The correct choice is C.

64.

irh
Pl).

65.

Now the question becomes how does cholesterol increase the fluidity of the membrane. well, it prevents the hydrocarbon chains making.up the lipids from crystallizing by preveniing their interaction. One can think of the

66'

se the

edlr
rsitiom

Since

worth your while to remember at least one or two of the ciassic prokaryotic cells and"their characteristi" ,t up.sl

B is correct. This question is not based on any information in the passage, but rather tests you on information you might have learned in some of your science. courses. Many bacteria hav-e rather distinctive shapes and sizer Even though it would be rather difficult to associate all the species of-bacteria *itn u !iu.n characieristic, it mignt ue

,ation-

Streptococcus pneumoniae (causes pneumonia)

The two species of streptococci that cause most of the streptococcal diseases associated with humans are
and,

Bacteria shaped like spheres are referred to as cocci. The best known examples comes from the genus Streptococcus.

Streptococcus pyogenes (causes impetigo, a superficial skin

opyright @ by The Berkeley Review

97

The Berkeley Review Specializing in MCAT preparation

Biology

Structure and Function in Cells and Viruses

Section VI Answers

infection commonly found in children). Bacteria shaped like cylinders are referred to as rods (also called bacilli). The best known example comes from the genus Escherichia and has the species name Escherichia coli (which can cause bladder and kidney infections). Bacteria shaped like spirals come in different sizes as well. There are those bacteria which are short and twisted into a rigid spiral, and there are those bacteria which are elongated and twisted into a flexible spiral helix. Representatives of these two forms of bacteria are not as commonly mentioned as the ones above, but we will give two examples for completeness. A typical spiral bacterium comes from the genus Spirillum. The species Spirillum volutans is one of the largest bacteria known. One of the better known bacteria that represents a spiral helix comes from the spirochete genus,Treponema.The species Treponema pallidum causes syphilis in humans. The correct choice is B.
67.

A is correct. Invariant simply

means that something (in this case a characteristic) is unchanging or constant. Another way to look at this question is to ask yourself which are the most important features of a prokaryotic cell. Certainly the nuclear region which contains the DNA is important. If new proteins need to be made to help in cellular function, then the DNA must be transcribed into mRNA. This message must then be translated at the level of the ribosome into a particular protein. Variation in either the nuclear region or the ribosomes could be deadly to the cell. The same can be said about the plasma membrane. This is the membrane that prevents the cytoplasm from leaving the interior of the cell. Lysis of the plasma membrane leads to immediate destruction of the cell.

The cell wall is the only structure mentioned which can be invariant. If the cell wall were to be removed from a given prokaryotic cell, that cell would form a protoplast (see the passage). These cellular structures are still capable of carrying out metabolic processes given the right environment. In fact, one genus of bacteria (Mycoplasma) lacks a cell wall entirely, simply because it cannot synthesize the precursors needed for the formation of the peptidoglycan layer. The correct choice is A.
68.

D is correct. Recall that Gram-positive bacteria do not have an outer membrane, while Gram-negative bacteria do. Also, the peptidoglycan layer in Gram-positive bacteria is much thicker than the peptidoglycan layer in Gramnegative bacteria. In Step I both types of cells are treated with crystal violet. This procedure will stain both cell

fix the crystal violet in the cells. A complex of crystal violet and iodine is formed. Again, both cell types will remain purple. In Step 3 alcohol is added and acts to decolorize the cells. However, only the Gram-negative cells are decolorized by the alcohol. The Gram-positive cells remain purple. This is due to the thickness of the peptidoglycan layer of the Gram-positive cells. Alcohol tends to dehydrate this layer. thus making any pores within the layer itself rather small. These small pores hinder the passage of the crystal violetiodine complex during the extraction process. Since the crystal violet-iodine complex remains trapped in the peptidoglycan layer, Gram-positive cells still display a violet color at this stage. In Gram-negative bacteria the thin peptidoglycan layer does not significantly hinder the extraction process, and these cells therefore display no color (i.e., they are colorless) at this stage. In Step 4 a counterstain (safranin) is added to the suspension. This red-colored stain is added so the Gram-negative bacterial cells can be visualized. They now display a red color. The purple colored Gram-positive bacteria also pick up the red stain and now appear blue. The correct choice is D.
types purple. In Step 2 iodine is added to

69.

D is correct. In Gram-positive bacteria the peptidoglycan layer is rather thick, while in Gram-negative bacteria the
peptidoglycan layer is rather thin. The Gram-staining procedure takes advantage of these different characteristics. It is important to understand that the designations "Gram-positive" and Gram-negative" refer to the fact that a particular bacterial cell can resist decolorization or be decolorized, respectively. The technique employed by the Gram procedure is an entirely different matter. In order for a cell to be clearly seen under a light microscope, organic dyes are used to stain cells. This technique is referred to as positive staining.The dyes used can either be cations (positively charged) or anions (negatively charged). Crystal violet and safranin ara both examples of dyes which are cations. One of the reasons that these two dyes work rather well in staining cells i-r because the membranes ofcells show a high degree ofnegative charge. Negative staining allows cells to be visualized in an outline form. This technique calls for the staining of the cell background while leaving the cell itself stain-free.
s

E,ither positive or negative stains are used in this procedure. Therefore, we find that the Gram staining procedure is both a positive staining procedure anC a differential staining procedure. The correct choice is D.

Dffirential staining is a procedure that does not stain bacterial cells equally.

Copyright

by The Berkeley Review

9a

The Berkeley Revier Specializing in MCAT Preparatior

Biology
70.

Structure and Function in Cells and Viruses

Section VI Answers

B is correct. Recall that the enzyme transpeptidase links amino acids together between adjacent glycan chains. This linkage helps to stabilize the fbrming peptidoglycan layer. If the enzyme were found near Face 1, it would not be
close to the peptidoglycan layer and would therefore be unable to catalyze the cross-linkage between adjacent amino

acids of the glycan chains. We can rule out choice A in both types of bacteria. We can immediately rule out two other choices if we recall that Gram-positive bacteria do not have an outer membrane, and hence do not have membrane Face 3 or membrane Face 4. Remember, the question asks for location of the enzyme in both Grampositive and Gram-negative bacteria. Choices C and D are eliminated. By default, this leaves choice B. Notice that if the enzyme were near membrane Face B, it could easily catalyze the joining of amino acids in the peptidoglycan layer. The correct choice is B.

7I.

A is correct. In order for bacterial cell growth to occur, the peptidoglycan layer must first be opened up by

the

action of autolysins. Specific sugar derivatives and amino acids are then added to the peptidoglycan layer as the cell begins to grow. The strength in the peptidoglycan layer comes fiom the cross-linking of peptide bonds between individual glycan chains by enzymes called transpeptidases.

If these enzymes are inhibited (by penicillin), then cross-linking can not be completed and the peptidoglycan layer remains weak. The cell becomes susceptible to lysis. If the autolysins were inhibited, the peptidoglycan layer would not be able to grow and therefore the cell would stop growing. Since the peptidoglycan layer would not be opened up, the action of the transpeptidases would not be needed. Penicillin is not able to induce lysis in non-growing cells, because inhibition of the transpeptidase enzyme would have no effect.
Increasing the growth rate of the cells would mean that penicillin could interact with the transpeptidases and inhibit the action of this enzyme. Even if the growth rate were to increase, the growing cells would still lyse.

Under normal conditions, the solute concentration within a bacterial cell is much higher than the solute concentration outside a bacterial cell. Water wants to flow down its concentration gradient and into the cell, causing
the cell to swell and eventually lyse. Lysis by penicillin can also be prevented by adding a solute such as sucrose to the medium in which the cell resides. If the solute concentration inside the cell balances the solute concentration outside the cell, then structures called protoplasts form when the cell wall of the bacterium becomes too weak. Protoplasts are (spherical) cellular structures that do not have a cell wall. They have lost the ability to retain it.

However, theses structures still have their plasma membrane and all of their intracellular components. Inhibition of protoplast formation wouid mean that the solute concentrations across the bacterial cell membrane(s) are not the same. If penicillin is administered, and the bacterial cells cannot form protoplasts, they will lyse.
Decreasing the solute concentration outside the cell sets up an even larger solute concentration gradient between the inside and the outside of the cell. More water will want to flow into the cell. If this happens, the cell swells and eventually lyses. The correct choice is A.

C is correct. First, look at Figure l. Note that there is an outer membrane (with a pore), a peptidoglycan layer, and then a plasma membrane. The action of both penicillin and lysozyme is described in the last paragraph of the passage. Both work on the peptidoglycan layer. In order to work on the peptidoglycan layer, they both must have 3ccess to it. The only types of cells that will allow for such access are Gram-positive cells. Gram-negative cells have that extra outer membrane and tend to be (more) resistant to penicillin and lysozyme. Choice A states that the cell is nonresistant to penicillin, because the antibiotic can cross the outer membrane. Penicillin is a large molecule and would have a difficult time crossing the lipid bilayer of the outer membrane. What ,rbout passage through the pore? Again, penicillin would be too large to pass through the pore. The same is true for .-r sozyme. Enzymes are generally quite large (much larger than penicillin). If penicillin cannot pass through the :cre, either can lysozyme. This allows us to eliminate choice A and choice D. Even though choice B is a true :ietement, there is a better answer. Do not just sequentially read down the answers until you find one that is correct. ) ou oed to flnd the best answer. The correct choice is C.

B is correct. As shown by the molecular structure, rafTinose is clearly not a disaccharide. It is considered to be an ..gosaccharide (or even a polysaccharide). Since there are no amino acids attached to and of the three sugar --f1dues, it cannot be a glycoprotein. This means that raffinose is either a reducing sugar or a non-reducing sugar.
m
ffMl

. by The Berkeley Review

99

The Berkeley Review Specializing in MCAT Preparation

Biology

Structure and Function in Cells and Viruses

Section VI Answers

Sugars having anomeric carbon atoms that have not formed glycosides (containing an acetal linkage) are called reducing sugars. Recall that the cyclic and linear forms ofboth aldoses and ketoses are readily interconverted. The aldehyde function in galactose and glucose can easily be oxidized by an oxidizing agent like Benedict's reagent (a solution ofcopper(Il) sulfate and sodium citrate in aqueous base) to the corresponding carboxylic acid (see below).
cHo

H-C-OH I HO_ C- H H-C-OH I H- C- OH


I I

2cl2+ +

5 oHo

"ooo H- C- OH HO-C-H
I I I

Cu2Olry
red

3H2O

Benedict's reagent

'

H_C-OH H- CI I

OH

precipitate

cH2-oH

(blue)

cH2-oH

D-Glucose
(open chain)

D-Gluconate

The Benedict's reagent, which is a blue solution, is reduced to a red precipitate.

Fructose is an a-hydroxy ketone and a-hydroxy ketones are easily oxidized to the diketone by Benedict's reagent.

functional groups remain tied up in a glycosidic bond (as shown in the structure ofraffinose), then they cannot react with the Benedict's reagent. The correct choice is B.
74.

If

the ;ldehyde and

ke-tone

C is correct. Sugars with an aldehyde function are referred to as aldoses, while those with ketone functions are referred to as ketoses. The open chain form of glucose can undergo intramolecular hemiacetal formation. In this case, the hydroxyl oxygen of the C-5 carbon attacks the carbonyl carbon of the aldehyde to form the stable hemiacetal. This hemiacetal, which forms a six-membered ring, is referred to as a pyrinose. With this much information, we can eliminate choices A and D. The open chain form of fructose cin undergo intramolecular hemiketal formation to produce a five-membered ring referred to as a furanose. The hydroxyl oxygen of the C-6 carbon attacks the carbonyl carbon ofthe ketone to give the hemiketal. The correct choice is i.
B is correct. This answer depends on how you view the linkage between glucose and fructose. Locate the anomeric carbons of each sugar residue. The anomeric carbon for glucose is at the C-l position while the anomeric carbon fm fructose is at the C-2 position. If we are looking at this linkage from the point of view of glucose, then the linkage will be 1-+2. Is this linkage in the a or the p position? The bond stemming from the anomiric carbon of glucose-is pointing down, below the plane of the Haworth projection. This represents the c-configuration. Therefore, we catr say, that from the point of view of glucose, the bond is in the cr(l-+2) configurarion. But what if we look at the bond from the point of view of fructose? In this case, the linkage is 2-+1. Since the bond at the anomeric carbon is pointing up, above the plane of the Haworth projection, it is in the p-configuration. Therefore, the 6x(l-+21 configuration by itself is not good enough. We also need to consider the p(2-+1) configuration as well. The correct
choice is B.

't5.

76.

D is correcL First, you must find the anomeric carbon of each sugar. The anomeric carbons of galactose and glucosc are both at the C- I position. The bond leading form the C- I position of galactose is connecring with the C-6 positim of glucose. Note that the C-l bond of glucose is connecting with fructose. This allows us to eliminate choicei A ard B. Now, is the l-+6 linkage in the a or the B position? In order to determine this we need to consider the carbon that links the two molecules. That anomeric carbon belongs to galactose. Note that the bond stemming fn the anomeric carbon of galactose is pointing down, below the plane of the Haworth projection. This represenithe configuration. If the bond were pointing up, it would represent the p-configuration. The correct choice is D.
B is correct. Recall that when sugars differ from one another about one carbon atom they are referred to as epimr The only place where the carbon atoms of galactose and glucose differ from one another is at the C-4 position. galactose the hydroxyl at the C-4 position is above the plane of the ring, while in glucose it is below the plane of ring. The correct choice is B.

77.

78.

C is correct. Hydrolysis of just the galactose residue from raffinose leaves a single disaccharide composed glucose and fructose. Sucrose is composed of both glucose and fructose. However, in order to call this disacchar sucrose, the two monosaccharides, glucose and fructose, must be linked together in the correct fashion. Note that linkage between these two sugars is glucose-o(l+2)-fructose. This is the correct linkage between these two residues. Lactose is a disaccharide composed of galactose and glucose while maltose is a disaccharide composed two glucose residues. The correct choice is C.
@

Copyright

by The Berkeley Review

too

The Berkeley Specializing in MCAT

Biology
79,

Structure and Function in Cells and Viruses

Section VI Answers

(CH:CHOHCOOo), methane (CF{a), carbon dioxide (COz), and hydrogen gas (H2). It is the gases rhar are produced that lead to flatulence. These compounds can also cause an increase in the motility of the intestinal system, fluid secretions, and cramps. The correct choice is B.

B is correct. If there had been a loss of bacterial enzymes that degrade oligosaccharides, then the bacteria would not be able to produce the monosaccharides which (when metabolized) lead to the production of lactate

80.

A is correct. If we start the cell cycle just after cytokinesis (division of the cytoplasm), we will find the following
order:

Gr

iir;il;

Phase *

oNi^sviliri.rirt

Phase *

Cr

ii;ffi;

Phase

Prophase--+ Metaphase

Anaphase

-+ Telophase--> Cytokinesis

Note that in choice B the S phase is shown to follow interphase. The S phase does not follow interphase, rather it is a component of interphase. In choice C it says that anaphase comes after telophase. This is not the case as outlined above. Finally, in choice C the Gt phase comes before the S phase, not after it. The Gz phase comes after the S phase. The correct choice is A.

sl,

D is correct. Two mature cell types in the body which do not replicate at all are skeletal muscle cells and nerve cells. These cell types are said to be in their "resting" state in which they are maintaining the metabolic functions of the cell. The resting state for these cell is the Gt phase of the cell cycle. It is in the Gr ptrase that transcription and translation for the various cell functions take place. The Gt phase is part ofinterphase. Interphase also includes the G2 phase (where the chromatin condenses) and the S phase (where DNA is replicated). After interphase is completed at the end of the Gz phase, a cell enters into the M phase. This is where mitosis and eventual division of the cytoplasm (cytokinesis) occurs. The correct choice is D.
D is correct. We are interested in following labeled precursors which are incorporated into DNA. Northern blotting involves RNA analysis, while Western blotting involves analysis of proteins. This allows us to eliminate choices A and C. As outlined in the passage, Southern blotting is used to locate certain genes in a segment of double-stranded DNA. The question is asking us for a way to monitor the incorporation of labeled precursors into DNA, not to locate a particular gene in the DNA. Autoradiography is employed to locate radioactively labeled molecules. Radioactive decay from an isotope that has been incorporated into a molecule would reduce silver grains in the emulsion of a sheet of film. Development of the film would show a deposition of silver grains, indicating the location of the Iabeled precursors. The correct choice is D.

!:.

uracil (U) in_place of T. If we were to use 3H-Uracil, the label would show up in RNA. If we were to use either 3Hcytosine or 3H-adenine, the label would show up both in DNA and in RNA. This allows us to eliminate choices B, C, and D. If we just use 3H-thymine, the label shows up in the DNA and not in the RNA. The correct choice is A. C is correct. Down syndrome (trisomy 21) is the most common type of human aneuploidy (the change in number of one or more chromosomes during gametogenesis). It results from the non-disjunction of chromosome 2l from either the father or the mother. Individuals with Down syndrome show a number of physical and mental abnormalities. Life expectancy is relatively short, with a mean at about l7 years. Individuals with trisomy l3 (Patau syndrome) and trisomy l8 (Edwards syndrome) show severe physical and mental abnormalities and usually die soon after birth. Trisomy 22 is common in abortuses. Since there was no mention of the cause of Down syndrome in the passage, it u as assumed that you would have learned about this genetic condition in your general science classes. The correct choice is C.

A is correct. This question is designed to see if you understand the differences between the bases in DNA and RNA. Recall that DNA contains adenine (A), thymine (T), guanine (G), and cytosine (C). RNA contains A, G, C, but uses

C is correct. The best way to see the difference between primary and secondary nondisjunction is to consider diagrams of each case. Primary nondisjunction (as stated in the question) occurs at the first meiotic division while secondary nondisjunction occurs at the second meiotic division. These types of disjunctions are drawn below. By :omparing the answers with these diagrams, we see that C is the correct choice.

p,-;fi1 O by The Berkeley Review

lol

The Berkeley Review Specializing in MCAT Preparation

Biology

Structure and Function in Cells and Viruses

Section VI Answers

Meiosis I

Meiosis I

Meiosis

II

@@oo
Primary Nondisjunction

/ tt

Secondary Nondisjunction

If a normal gamete were to unite with one of the gametes of primary nondisjunction during fertilization, half the zygotes that would arise which would show trisomy (the gain of one chromosome) and half would show monosomy (the loss of one chromosome). However, if that normal gamete were to unite with one of the gametes of secondary nondisjunction, then half of the zygotes would be normal, a quarter would show trisomy, and a quarter would show
monosomy. The correct choice is C.
86.

C is correct. Maternal simply refers to the mother, and paternal refers to the father. The procedure for establishing the autoradiograms is well outlined in the passage. All we need to do is read the results. In both families we are looking for that parent which contributes to DNA fragments to the Down's child. In Family I
note that the mother contributes the two fragments (one at about 5 kilobases and the other at about

kilobase).

Normal Down's

Mother Father
6
N^ v) a
O

Child

Normal Down's

Child
6

Mother Father

Child

Child

E? o-o
oo .-

r53

N^

<

.t-t

I I

Ed4
il:i

CJ-o

_i:J

I -

I I

Autoradiograph of Family I

I 2

Autoradiograph of Family

The father only contributes one fragment (at about 2 kilobases). In Family 2 it is the father who contributes the tu,o fragments to the Down child (one at about 3 kilobases and the other at about 2 kilobases). The mother onlv contributes one fragment (at about 4 kilobases). The correct choice is C.

87.

B is correct. In order to understand where this answer came from, it is important to understand meiosis. Before the actual onset of meiosis, DNA has already been synthesized during the S phase of the cell cycle. In prophase I of the first meiotic division the chromosomes become visible and centromeres begin to develop. Homologous chromosomes begin to pair or synapse at the centromere. This structure of two sister chromatids is referred to as a bivalent. When two bivalents come together the complex is referred to as a tetrad. A tetrad consists of four chromatids. The number of tetrads is equal to the number of haploid chromosomes. This means that if we have 2-1 haploid chromosomes, we must have 4 times that many chromatids. In other words, we have 23 x 4 = 96 chromatids. After telophase I of the first meiotic division has occurred, two haploid nuclei are formed. These nuclei are haploii by definition, even though they contain pairs of sister chromatids. Since the sister chromatids are attached by the same centromere they are considered to be part of one chromosome. The total number of chromosomes in each nuclei has been reduced by one-half. This is why the first meiotic division is referred to as a reduction division. The
second meiotic division is similar to mitosis and is therefore referred to as a equational division.

Copyright @ by The Berkeley Review

t02

The Berkeley Revieu Specializing in MCAT Preparation

Biology

Structure and Function in Cells and Viruses

Section VI Answers

Since the total number of chromosomes in each nuclei has been reduced by one-half, we now have 46 chromatids in each nuclei as we enter into the second meiotic division. During metaphase II of the second meiotic division the chromosomes align along the equatorial plane. If we stop at metaphase II, we find that the daughter chromatids have not yet separated. Therefore, we will still have 46 chromatids present in each of the two nuclei at metaphase II. It is not until anaphase II that the centromeres begin to divide and the sister chromatids move to the opposite poles. After telophase II there will be 23 chromatids in each of the four meiotic nuclei. The correct choice is B.

88.

C is comect. Fats are esters. Recall that esters result from the union of an alcohol and a carboxylic acid. The backbone of all fats is glycerol, a three-carbon molecule containing a hydroxyl group on each of the carbon atoms. Fatty acids contain a carboxylic acid group.
o H)CI

OH

"o"o16-

!
o

cH,

HCI

OH

!
o

cH'

H,C-

OH

!
3 Fatty Acids (Stearic acid)

cH,

Glycerol

$.tt.-

3 H2o

H,c-o-!

"l- o- f
I

ln

""'
"",
ctt, A Triglyceride

H2c-

tfl o- 6

Since glycerol contains 3 hydroxyl groups, there is the possibility of joining 3 fatty acids ro one molecule of glycerol. This will form a fat called a triglyceride. Each linkage will be an ester linkage. The correct choice is C.
{l'q

passage and determining the order of polarity. We want to arrange the polarity from the most polar molecule to the least. The most polar molecule bears a full charge or charges. The only structure in our four diagrams that bears full charges is phosphatidyl choline. The remaining three structures do not bear a full charge. However, they all contain oxygen atoms, which are highly electronegative. We first look for that structure with the most oxygen atoms. It turns out that both the triglyceride and arachidonic acid have two oxygen atoms. Which is more polar? At physiological pH, arachidonic acid has its carboxyl hydrogen ionized (not shown in the diagram). The two oxygen atoms on the carboxylic acid group will share the negative charge between them (tkough resonance). This leads to stabilization of that functional group and an increased polarity. The triglyceride contains ester linkages. Because one of the oxygen atoms is tied up with two other carbon atoms, the polarity of the ester linkage is reduced. However, it is not as reduced as the polarity of the single oxygen atom of the hydroxyl group of cholesterol. With this arrangement of polarity, we can oider the lipids in terms of decreasing polarity. The correct choice is A.

A is correct. This question involves looking at the four structures in the

C is correct. The more saturated the fatty acid, the more likely it is a solid at room temperature. The more unsaturated a fatty acid, the more likely it will be a liquid at room temperature. Triglyceride II contains three hydrocarbon chains which are completely saturated. This allows these three chains to pack close together in a tight parallel arrangement, thereby increasing the amount of attractive forces between the individual chains. If there are more attractive forces between the aliphatic chains, then it will require more energy (heat) to pull them apart. We
,,:

,:.:ht O by The Berkeley Review

l03

The Berkeley Review Specializing in MCAT Preparation

Biology

Structure and Function in Cells and Viruses

Section VI Answers

would expect the melting temperature to be the highest for triglyceride II. Remember, we are being asked for the order of the increasing melting point for the four triglycerides? We can immediately eliminate choice B.
We can distinguish between choices A, C, and D by looking for that triglyceride with the most sites of unsaturation (i.e., double bonds). This turns out to be triglyceride III with 3 sites of unsaturation. The more double bonds there are in a fatty acid, the more disorder there will be in the packing of the hydrocarbon chains. The more disorder in the packing, the less interaction there will be between individual regions of each chain. Therefore, less energy will be required to pull them apart. We would expect the melting temperature to be the lowest for triglyceride III. We can now eliminate choice A.

All that remains is to locate that triglycerides with the next two highest degree of unsaturation. They are triglyceride IV (2 sites of unsaturation) and triglyceride I (1 site of unsaturation). Therefore, the order for increasing melting point of the four triglycerides is III, IV, I, and II. The correct choice is C.
91.

D is correct. The structure of cholesterol is shown in the passage in Figure 3. Note that there is just one hydroxyl group on the molecule. The rest of the molecule is composed of a non-polar cyclohexane and cyclopentane ring
system. Attached to the cyclopentane ring is a long non-polar hydrocarbon chain. Diets high in cholesterol increase the blood serum concentrations of cholesterol. Because this compound is quite insoluble in water they begin to adhere to the walls of arteries and help in the formation of plaques.

If there were an increase in the synthesis of bile

preparation for digestion. Similarly, an increase in the synthesis of steroid hormones means a reduction of cholesterol in the blood serum. Both conditions would act to reduce the formation of plaques. Once cholesterol is integrated into a lipid bilayer it usually stays there until the membrane is degraded. There is no (known) mechanism to remove excess cholesterol from cellular lipid bilayers. Once the membrane is degraded, the excess cholesterol in the membrane can contribute to plaque formation, but it is the insolubility of cholesterol in water that allows for this. The correct choice is D.
92-

salts from cholesterol, then less cholesterol would be in the blood serum and more would be excreted in the feces. As we will learn a bit later, bile salts act to emulsify dietary lipids in

nothing to say that we cannot attach arachidonic acid to the alcohol group of a complex lipid. Thus, arichidonic acid can contribute to the formation of complex lipids as well. Finally, it was stated in the paisage that arachidonic acid is a precursor to the synthesis of prostaglandins and leukotrienes. The correct choice iJD.
93.

D is correct. The structure of a simple lipid is shown in Figure I of the passage. Arachidonic acid has a carboxylic acid group which can react with an alcohol group of glycerol to form an ester linkage. Therefore, arachidonic acid can contribute to the formation of simple lipids. As outlined in the passage, the only difference between a simple lipid and a complex lipid is that there is a phosphate group involved in the formation of complex lipids. There is

C is correct. The question states the temperature is being changed from 38 "C to 25 "C. The bacterial cultures are going from a warm environment to a cold environment. At high temperatures membranes are rather fluid; at low temperatures they tend to be less fluid. However, it is the desire of every cell to maintain some degree of fluidity in their membranes. This will allow for transport across that membrane. Which of the three answeis in the quesiion lead to an increase in fluidity? If the synthesis of unsaturated fatty acids is increased, there will be less packing between the fatty acid side chains and hence more freedom of movement. Choice A will occur. Initially there will be a decrease in the bacterial membranes. This is simply because the cells will not have had enough time to begin synthesis of the components needed to maintain membrane fluidity. Choice B will occur. The synthesis of short chain fatty acids will increase. Why? The shorter the chain, the less packing between the chains. The less packing between the chains, the less of an attractive force holding the chains together. The membrane becomes more fluid-. Choice D will occur. Choice C will not occur because the bacterial cell does not want to decrease the fluidity of its membrane. If the fatty acid chains are longer, there will be more attractive forces holding the chains together. This will lead to a more solid membrane. The correct choice is C.

94.

choline). However, as outlined in the passage, glucose is not part of a phosphoglyceride simply beiause phosphoglycerides are based on a glycerol backbone. Sphingolipids are based on a sphingosine back6one. In this
case, glucose would be associated with sphingolipids and not with phosphoglycerides. The correct choice is B.

glucose and serine? The amino acid serine can be part of a phosphoglyceride (attached to the phosphate group like

B is correct. A typical membrane phosphoglyceride is shown in the passage in Figure 2. We can hydrolyze this compound as shown below. The hydrolysis products are glycerol, stearic acid, phosphate, and choline. Whit about

Copyright @ by The Berkeley Review

to,4

The Berkeley Review Specializing in MCAT Preparation

Biology

Structure and Function in Cells and Viruses


o

Section VI Answers

H2c-

6l V' N-CH,' H,C-o-p-o'lCH. oo


I il

[ "lI o I
n-

lo

o-

[
cH,

tn'

t",
Phosphatidyl choline

+ 4 H2o

{2

fl

uydrotysis
Futty Acid (Stearic acid) cH,

H2C- OH
I

"o
"o

o - 'j
o

HCI

OH

'j
o
ll
+
I

cH,

HzC- OH Glycerol

+ HO-P-OH

oo
Phosphate

"o& f- ""'
CH:

Ol

CH,

Choline

95.

D is correct. The question asks why the lambda virus is incapable of infecting other bacterial species. If

these

Answer choice A is incorrect because most bacterial species have circular chromosomes, not just E. coli. Additionally, having a circular bacterial chromosome has little to do with whether the lambda phage can infect a cell. Don't be confused by the passage, which states that the viral DNA is circular. Likewise, answer choice B is incorrect, as we know that E. coli has a circular chromosome (as opposed to the linear ones seen in eukaryotes).
Choice C is tempting, because we know from the passage that lack of a chromosomal integration site would prevent the lysogenic infection pathway from functioning. We cannot say, however, that this would interf'ere with the lytic pathway; the virus might still be able to infect the cell and lead to lysis. The correct choice is D.
Sh.

species lack appropriate cell surface proteins, lambda phage may never be able to become adsorbed, or attached, to the surface of these cells. According to the passage, this attachment is the first step in the process of viral infection. If it can't occur, the virus doesn't infect the cell. We can approach this problem by a process of elimination as well.

A is correct. If the integrase protein were def'ective in a certain strain of lambda phage, that strain would be unable to enter the lysogenic infection pathway. This is because the viral DNA would never be able to integrate into the bacterial chromosome; functional integrase is normally required for this insertion process. Therefore, this viral strain would be able to enter only the lytic pathway, always leading to the lysis of the host cell. Choice B is incorrect, because the integrase-defective viral strain would still be able to infect E. coli; it just wouldn't be able to adopt the lysogenic pathway. Choice C is likewise invalid, as the lytic pathway does lead to viral DNA replication. We can also eliminate choice D, because a lack of integrase would not prevent the virus from entering the lytic infection pathway. The correct choice is A.
C is correct. The question essentially asks which statements represent evolutionarily advantageous situations for the lambda virus. An evolutionary selective advantage is basically anything that better enables an organism to survive, reproduce, or adapt to change. Statement I represents just such an advantage. Recall, a provirus is lambda DNA that has integrated into the bacterial chromosome. If the host cell were damaged and later died, the lambda provirus riould also perish. Therefore, it is a selective advantage for the lambda provirus to excise itself and enter the lytic ;ycle when the host cell is initially damaged. In this fashion, it can quickly replicate new viral particles and leave the d1,ing host cell, potentially to infect new neighboring E. coll cells. This is an evolutionary advantage. Statement II is likewise beneficial evolutionarily. If lambda always killed its host cell immediately (i.e., via the lytic pathway), it nould soon run out ofviable host cells. It would also have less chance ofbeing dispersed to new areas. If itdidn't always kill its host cell immediately (i.e., the lysogenic pathway), it could replicate along with the bacterium and

'rr,i1:-nt

O by The Berkeley Review

los

The Berkeley Review Specializing in MCAT Preparation

Biology

Structure and Function in Cells and Viruses

Section VI Answers

therefore be more successful at spreading to new areas. This is an evolutionary selective advantage. Statement III, on the contrary, does not represent an advantage. If lambda DNA were almost completely resistant to mutation, it could never evolve to meet new environmental challenges. In essence, it would be incapable of adapting to change. This is not an evolutionary advantage. The correct choice is C.

98. B is correct.

The experiment described in the question involves radioactively labeling the lambda viral DNA, packaging this labeled DNA into a phage protein capsid, and allowing this new virus to infect an E. colicell. The question effectively asks where the viral DNA would be after lysogenic infection not lytic infection (the cell does not lyse). Recall from the passage, the lysogenic infection pathway leads to the integration of viral DNA into the host's chromosome. In prokaryotes such as bacteria, chromosomes are located in the cytoplasm of the bacterial cell. This is were we would detect radiation from the integrated, labeled viral DNA. Choice A is incorrect because bacteria do not have nuclei. Choice C is invalid, because we would only see labeled viral DNA bound to ribosomes if it were being translated. After integration via the lysogenic pathway, the viral DNA does not direct protein synthesis and is therefore latent. Choice D is wrong, because we would only see labeled viral DNA outside fhe cell if lysis, or cell bursting, had occurred, releasing new virus particles. Since the virus in question does not enter the lytic pathway, this would not occur. The correct choice is B.

99.

insert into the bacterial chromosome. This information leads us to eliminates answer choiceJA, B, and D. The correct choice is C.

C is correct. A cytosolic endonuclease is an enzyme which can cleave DNA at specific sequences within the strand. Endonucleases (also called restriction enzymes) are therefore capable of cleaving circular viral DNA. Such cleavage would prevent viral DNA from being either translated directly or integrated into the bacterial chromosome. Injection of lambda DNA is a step common to both infection pathways; the DNA injected at this step is degraded "by the bacterial endonuclease. Therefore, both lytic and lysogenic pathways of infection would be inactivated. The lytic pathway is prevented because lambda DNA is cleaved before it can direct synthesis of the protein capsid. ihe lysogenic pathway is inhibited because lambda DNA is cleaved before it can either direct integrase production or

100. A is correct.

'

The qtrestion essentially requires us to know the differences and similarities between a lambda phage which adopts the lytic pathway of infection and a lambda phage which adopts the lysogenic pathway of infection. The question asks about the similarities between the two pathways. From the passage, we learn thal the common step to both pathways is the adsorption ofthe phage to the bacterial cell surface and the subsequent injection of viral DNA. This makes choice A correct; the two pathways do NOT differ in this regard. We cin also proceed via a process of elimination. Choice B can be ruled out, because the lysogenic and lytic pathways both adopt dffirent methods of replicating viral DNA. The lysogenic approach involves replication along wittr the host chromosome, while the lytic approach involves replication independent of the host chromosome. Therefore, this answer choice represents a way in which the two pathways dffir. Choice C is likewise incorrect, because it also represents a difference between the two pathways. While the lytic pathway is immediately lethal to the infected cell ldue to lysis), the lysogenic pathway may allow the cell to undergo multiple replications unharmed. In the same sense, choice D can be eliminated. Only viruses entering the lysogenic pathway can remain latent for long periods. This is another difference between viruses entering each of the two respective pathways. The correct choicels A.

Copyright @ by The Berkeley Review

l06

The Berkeley Revier Specializing in MCAT Preparation

ology
$bff,$fi.....VII
lA
A. '..,,E m l{inetics
,,

'. ,

tid

''l.r'. Li.r .''

'1,

&.'

l. t'., 5.

Transition State
Mic,haelis-&{enten E'.quation

Lineweaver-BurkPlot
Enzynre

Inhibition

,'.

B.,,,,'

lfinryme1,:MgchanlsmS .',

Chymotrypsinl\zlechanism RN-*A as an Active Enzyme Inactive Enzymatic Precurso.s Molecular Evolution ':' , ,5; ,, Trmsition State Analog ,'tt, , llrstabteTetrahedral
s*t.s.Hiss;

l. 2. 3. 4.

C.
H- lj-- R--1/

Metabolic Molecules

--ll'',

'],,,,,'

]],,,.

@ -_! .\ ''t\;n-":*--.*o-c-Asfto2

t.
Aminoproduct:,,.,;':3'),,
rea'es rhe scene

AdenosineTriphosphate

',,r:','!,.',Cofaciors anA,Coaneyrnes

Acyf rEn@,i

htefhrdiafF:
li, ,,,
..
,,, ,l l,i

T f\

,:'_ :',,,. r

':.,

r,'.'.::ii

'I

i:.
i l:i.,

:.. trJi:ffinine
l:

IiliCotinamideAdenineDinucleotide Dinucleotide

1..'':,1,',..

',.t,il

lill : r I

i....;.r:."..lrf

K'i,Ei.VtI.EtW* Spec,ializin$ irn' k[C Preparation


:

Metabolic Components
Top lO Section Goals

Li

n
:J_ ::t:. t::

oB

-{r
Be familiar with the concept of transition states. It is important to understand the difference between the activation energy of an uncatalyzed reaction and thdt of a catalyzed reaction.

-:

:ri_-

av
@?

Understand how to read a Plichaelis-Menten graph.


Don't,worry about the algebra involved in deriving the Michaelis-Menten equation. lnstead, be able to understand the components of that equation in relationship to its graphi-al form.

lt_:

-rL:
x

]:;

Understand how to read a Lineweaver.Burk graph.


Be able to

i^

s;
lrr
$r:triL

sure you understand the meaning of the different pararieters.

nF

@v

Be familiar with the concept of enzvme inhibition. Enzyme inhibition is a favorite topic in biology. Know the ditferenc" U"t*""t"t u .ot rp"titi* una non-competitive inhibitor and how they function in the presence or absence of substiate.

Understand the basics of th" chy-otry


The details of this mechanism have been on the MCAT before. Those who took this exam needed

only a basic understanding of enzymatic mechanisms. Keep track of yoo,

"i*.tronri

ov
@w

Be familiar with the concept of molecular evolution. On occasion, the MCAT has had a few questions concerning evolution. Be aware of the terms co nue rge n t euol u t io n and d i ue rge n t euol u t i o i.

Know the basic components and functions of the molecule ATP. ATP is the energy currency of the cell. We will encounter this metabolic component throughout our
entire review session. Know its components and understand its uses in metabolism.

"v
@v

Know the basic components and functions of the coenz;nnes NAD and FAD, NAD and _FAD_are both coenzymes. They are not enzymes. A coenzyme helps an enzyme function properly. NAD and FAD are frobably ttie two best known coenzym-es.
Be familiar with coenzyme A and its uses in metabotism.
Coenzyme A is another importart coenzyme, especially in the Krebs cycle and in fatty acid metabolism. Have some understanding of where this molecule is used in the cell and why.

Biology

Metabolic Components

Enz;rme Kinetics

ftffi$iffi$ffilii$
Chemical reactions occur at a particular set of rates. In the reaction shown in equation (7-1) there will be a rate under specified conditions at which molecule A .un b""o*" molecule B or molecule B can become molecule A. Usually k1 is referred to as the forward reaction rate while k2 is referred to as the reverse :eaction rate. The point of equilibrium is where [A] and [B] no longer change ',r-ith respect to one another. If k1 > k2, then at equilibrium [B] > [A]'

Activation Energy & CatalYsts

AA-<:

kr

B
k2

(7-1)

this is true, then why don't all reactions come to equilibrium immediately? In :rder to answer this question we need to consider the energy diagram shown in Figure 7-'1,.Ifwe have two energies for A and B, we will find that at equilibrium ts ivitl be in higher concentration. It is a bit more stable than A. It has less energy :rd is therefot" *ot" stable. In order to get from A to B we have to go through a ::gher energy state. This is often referred to as the transition state (TS). Do not --:jnk of this reaction as one molecule of A and something else, but rather think of
-: as a

population of molecules.

^*r"J;;;.;; of the
Uncatalyzed Reaction

a ,!

Activation EnergY
of the Catalyzed Reaction

-r'l .J
!

A
(Substrate)

B
(Product)

Progress of Reaction Sgure 7-f


ir

""u.s:r:ion States and the Effect of a Catalyst.

--:,tn this population of molecules some will have more energy than others r:lr- because some have been "heated" more recently than others. They have * **i rr contact with something that has given them energy. The population as.a , * : -e d"oes not have a particular energetic state but rather a range of energetic ,ril::s. In order to make the trek up to the transition state, some proportion of the t;:':Lation of molecules of species A must have enough energy to be at that : ir-.:5on state. If it were u r"iy small proportion of the population of species A,
:T,*:
-,.,-e

would expect to see t-he forwirdreaction occurring very slowly. If ryg t:= :o look at the population of molecules represented by species B' we would the greater fir,* -i.at it would be a harder trek to go back to species A because of from molecules of population Some A. B and between in energy -:i::::ence difference lli.r,,r'::.s B, though, would be expected to have an energy equal to the
-,.,q,,

Review -qht O by The BerkeleY

l09

The BerkeleY Review Specializing in MCAT PreParation

Biology

Metabolic Components

Dnryme Kinetics

between the transition state and the average thermodynamic state of species B. This will be different at different temperatures. At low temperatures fewer members of the population of A and B will have enough energy to make the trek across the transition state. This is why chemists, in order to achieve equilibrium,

often raise the temperature.

population of species A, per unit time, will cross the barrier.

If we raise the temperature, then more of the

How9v,er, in biological materials raising the temperature is usually not a possibility. For any one particular organism there is a range of temperatures which is fairly restricted (i.e., around body temperature). A diff"rent method must be employed and that is to use a catalyst. A catalyst is a general name for any substance (or substances) which can provide an alternative route or an alternative transition state. A catalyst can change the nature of the intermediate state to some lower energy state, If the transition state (TS) is lower in terms of its own energy, then the nature of the relative proportion of the population of A and B that can obtain a transition state at a-paiticular temperaiure is increased (without an increase in the temperature). This is indicated in Figure 7-1.
as being like a tunnel through a mountain (the energy barrier). Instead of expending a lot of energy to go over the mountain, r,r,e can go through the mountain to the other side (products) by way of the tunnel. As our molecule A goes through the tunnei (i.e., is worked on by the enzyme) it is converted to B. Our tunnel allows A to be converted into B. Wlttrin the iunnet. though, molecule A goes through a high energy transition state (less stable before it becomes molecule B. It turns out that one can synthesize molecules that

ffi

W
I@m

we can think of the enzyme

ffim

ilh,

iM

look like the transition states of molecules that are being examined. These synthetic structures are called transition state analogs, and they can easil,-interact with our enzyme (fit into the tunnel). Since this transition state analos has nothing to do wiih A going to B, it actually impedes the process. In other words, transition state analogs are excellent inhibitors of the catilytic process.
Thus, enzymes are catalysts which are often employed to loz.oer the transitio: state's activation energy. By definition, a catalyst will not alter the equilibrium ci a reaction. However, a catalyst will alter the rate of a reaction. In other words, a: enzyme accelerates the forward reaction as well as the reverse reaction t,-, precisely the same factor.

illicftddili5r,M6iltEH EHdhtion
Derivation

&

Meaning

How do enzymes carry out their catalysis of a reaction? Enzymes contain spec:il regions called active sites to which a specific substrate molecule will bin; Enzymes also stabilize the transition state and they carry out acid-base catalr sirs by precisely positioning the catalytic groups of certairramino acids (e.g., ,i=, Glu, Lys, Arg, His, Ser, Cys, etc.) found within the active site pocket.

For a number of enzymes, the catalytic rate (V) changes with the substi concentration (lsl). In equation (7-2) the enzyme (E) combines with the substr' (S) to form an enzyme-substrate complex (ES) with some rate k1. This complex can continue on to form the product with some rate k3 orit"
dissociate to the substrate and enzyme at some rate k2.

Copyright @ by The Berkeley Review

llo

The Berkeley Rer Specializing in MCAT Prepar

Biology
E+Sr5-2)
muhen

Metabolic Components

Enzyme Kinetics

k1
kt

S----.-..-.-.-E+P

kr

[S] is small and the [E] is constant, then V is essentially proportional to [S], :::.'-:ing a first order reaction. However, when [S] is large, then V is essentially i:nrJependent of [S], giving a zero order reaction. This can be seen in Figure 7-2. 1}e tvpe of curve obtained is hyperbolic.
l}Le velocitlz of the reaction given by equation (7-2) is shown in equation (7-3). }e only way that we can obtain the product is through the ES complex.
lt, -J,

V=

kslESl
Substrate Concentration IS]

The total concentration of enzyme in the system in question is equal to the ,smcentration of the free enzyme plus the concentration of the enzyme bound "w:h substrate in the ES complex. This is shown in equation (7-4).
fir-41
ltur

Figure 7-2
Reaction rates as a function of substrate concentration.

Etotul = [E]fr""

[ES]6orr,.,6

he following discussion we will consider the derivation of the Michaeliswish, you can skip this derivation and proceed

,\ufislten equation (7-14').It is important to know how to use the Michaelis-Menten eumiation but not how to deriae it. The only reason the derivation is presented here

ur :or completeness. If you dinecfly to equation (7-'1.4).

We rvould like to describe the rate of an enzyme reaction in terms of some flrmrameter that we can measure. It would be nice to measure ES. Unfortunately fitinrnq !5 13*1gr difficult. Consider the rate of change of [ES] over any given time. Th:rs can be seen in equation (7-5). This is simply the rate of formation of ES munus the rate at which ES is broken dovrm.
iN{I-5}

dlEsl/dt = krlEllSl - (kz+k3)lEsl

lftamon of the intermediates

Michaelis and Maud Menten reasoned (circa 1913) that the concenin a steady state process remain the same while the umrrentration of the reactants and products change. In other words, d[ESUdt = 0. "lhr:s r!'ill occur when the rate of formation of the ES complex and breakdown of
ll,,cqnnor
dftE

ES complex are equal. This can be seen in equations (7-6) and (7-7).

{ilf-61

dlEsl/dt

=0=

kltEltsl -

(k2 + k3)[ES]

lu

- |

kltEllsl

(k2 +k3)[ES]

M*arranging equation (7-7) gives equation (7-8).


illli",-fi

lESl

= tEltsl/(k2 +k3)/k1

$lhrli*

tfms constant
'iffi-9[

can now define (k2 + k3)/ft1 to be the Michaelis constant, KN1. Substituting into equation (7-8) gives equation (7-9).

lESl = [E][S]/Krra

tfiaUrright @ by The Berkeley Review

tll

The Berkeley Review Specializing in MCAT Preparation

BioIo gv

Metabolic Components

Enz5rme

Kinetics

Now, rearanging equation (7-4) gives equation (7-10). And substitution of (7-10) into (7-9) gives equation (7-11).

[E]r."" = [E]totut
tES]

[ES]uo.,r,a

(7-10)

(lE]totur

[ES])[S]/KM

(7-t1)

Solve equation (7-11) for [ES]. This gives equation (7-12).


[ES]

[E]total

(tsl(tsl

+ Kria))

(7-12',)

Substituting (7-12) nto equation (7-3) gives equation (7-13).

V=

k3[El16tar ([S]/([S] +

Ku))

(7-73)

Let's define the maximal rate of a reaction as v-u*. This reaction rate can be obtained when all the enzyme's active sites are saturated with substrate. In other words, vmax = kglE]total. substituting this expression into equation (7-13) yields equation (7-1'4). This equation is the Michaelis-Menten equation and it is an important equation in enzyme kinetics.

Michaelis-Menten Equation

V-

V-u^ [S]

[S] +

Krra

(7-14)

If we plot the velocity of a reaction as a function of the substrate concentration, we will get ahyperbolic curoe (Figure 7-3). This is the same type of curve that we will see when we examine the binding of oxygen to myoglobin. In the case of an enzyme, we find that when we saturate that enzyme with substrate, the enzyme
is operating at its maximal velocity (i.e., Voru*). In the case of an enzyme, when [S] = KM, then V = Vmax/2. In other words, the Ky is equal to the substrate concentration at uhich the reaction rate is half of its maximal ualue.
The Ky can also be related to the rate constants in a given reaction. Suppose we have a situation in which kz>>>kg. This means that the enzyme-substrate complex (ES) will have a tendency to dissociate to E and s rather than form E and P. In this situation, KM = kZ/kt. It is only when this situation is true that Kg is a measure of the binding strength of the ES complex.

v-u*

x
O

v-u*
2

.J

Substrate Concentration IS]

In other words, a high K14 value indicates a weak binding of the ES complex while a low Kyl value indicates a strong binding of
the ES complex.

Figure 7-5
Michaelis-Menten kinetics showing variation of Reaction Velocity with Substrate Concentration.

In practice it is not very convenient to estimate V*ar, in the manner just


suggested. This is because as we increase the amount of substrate the solution becomes more and more viscous (e.g., like honey). The only values that are really much use to us are those in the area where things arc first order.

In some cases we will not have a perfect hyperbola. For example, if we had a protein that had a structure like that of hemoglobin, where each of the four subunits has an active site, then those active sites might act independently of one
Copyright @ by The Berkeley Review

tt2

The Berkeley Specializing in MCAT

Biology

Metabolic Components

Enz5mre

Kinetics

another or they might influence one another. If one of those subunits was nteracting with a substrate, it could cause a change in the affinity (the Ky) of the rther active sites with respect to their interaction with other substrates. Since this :equently happens it turns out that there is cooperativity in multimeric proteins -,rith more than one active site. Allosteric enzymes have a non-hyperbolic curve i.e., more sigmoidal) when plotted as velocity versus substrate concentration.

Lineweaver-Burk Plot
Meaning
try and find some way of dealing with Ky that did not rr-olve plotting the hyperbola and then estimating the Vmaxlz. Lineweaver and Burk developed a plot which is now referred to as a double-reciprocal plot. In -:js plot we graph l/V as a function of 1ilSl. If we were to plot the hyperbola .hown in Figure 7-3 onto this graph, we would end up with a straight line.We :an obtain an equation for this plot if we take the reciprocal of the Michaelisl-fenten equation. This is shown as equation (7-15). Note that this equation is in -:.e form of y 11 + b, the equation for a straight line. =
-: became of interest to

--15)

1-[r*lIrl, v - t-v,""J t-rsr] v-*


I

Lineweaver-Burk Equation

-i"e solid dots on the Lineweaver-Burk graph are data points (Figure 7-4).If we ,h:ere to take the best fit through those data points and then extrapolate the line,
",e ivill intercept the Y-axis at L/V*u*. If we continue to the X- axis, we will .::ercept at -1lKp1. Note the region of high IS] and low[S] values. If 1/[S] is :!:proaching zero, then it must mean that [SI is approaching infinity. At the r:ersection of the X-axis and Y-axis the [S] would be equal to infinity. This is the :,:int at which V became V*u* in Figure 7-3.If the line in Figure 7'4 is not ':aight, it indicates that we do not have a hyperbola. If we do not have a r-.-l'erbola, then we have an enzyme which is undergoing some type of :,-:ernative reaction. This is very often a sign that we are dealing with a r-:ltimeric protein which binds more than one molecule of substrate. Slope =

KMff-u*
r

tlir
Figure 7"4
Lineweaver-Burk plot.

,,1en an enzyme is completely saturated with substrate, then the number of '-lstrate moleculei which are converted to product per unit time is referred to as
r-e furnover number.

Enzyme:Inhibition,' ::::

il.eversible & Irreversible I- -re are two major types of enzyme inhib.ition: reaersible and irreoersible rdribition. One example of an irreversible inhibitor is diisopropylfluorophosphate
DIIPF), a potent nerve gas. DIPF inhibits the enzyme acetylcholinesterase, which i :'l-olved in the hydrolysis of the neurotransmitter acetylcholine to acetate and -::'j:re. In other words, DIPF blocks cholinergic nerve impulses within the body. : :j-:Trs out that DlPF-inhibited enzymes have an unusually reactiae serine residue r: -:re active site. Other enzymes like elastase, trypsin, and chymotrypsin all have '::;tive serine residues at their active sites as well. Thus, these enzymes are ::uped into a class of enzymes called serine proteases.

:,:.,right

by The Berkeley Review

rl5

The Berkeley Review Specializing in MCAT Preparation

Biology
E + s=5 ES k3-E + P +k2

Metabolic Components

Enzyme Kinetics

There are two types of reversible inhibition. They are competitiae inhibition and non-competitiae inhibition (of which allosteric inhibition is a subset).

1l*,

EI
Figure 7-5
Competitive Inhibition,

['

Competitive Inhibition A competitive inhibitor (K1) usually resembles the normal substrate molecule 11d can readily bind at the active site of the enzyme to form an enzymeinhibitor (EI) complex as shown in Figure 7-5.
T there is a competitiae inhibitor around, it will compete with the substrate for the active site on the enzyme. In Figure z-s we see that the enzyme is being utilized in two pathways. one pathway utilizes the normal substrate and the

e e

,&

other pathway utilizes the competitive inhibitor. The consequence of that competition is a decrease in the rate of catalysis of the enzyme. The rate of formatio_n of the product is dp/dt = ks[ES]. If some of the IE'sl is removed to become [EI], we essentially have a lower concentration of ES.

,&

,ds,

&
m0

Uninhibired

vru*
v\/

b
o

'rn4x

concentration of the competitive inhibitor becomes negligibte and the enzyme


catalyzed reaction

A competitive inhibitor can be overcome at high concentrations of substrate (Figure 7-6). Initially we have a louter vmaxlo, u gr""" *bstrate concentration, but as we approach an infinite substrite concentration, the will
once again approach

W
@
um ME

oa aL

the,u*u

f_i.

when a

0
Figure 7-6

Ktr,t

isl
Competitive Inhibition "appears" to
increase KM.

both Figure 7-6 and tn Figure

"u.h the "^rymehas for a given substrate. In other words, Ky of a competitiverrinhibited enzyme appears to shift to a higher varue. 'Fio*"rr"r, if we add enough substrate, we will eventually reach trie same vrr,u*. This can be seen in
7-7
.

:lry*:. Ky-val1e

competitiae inhibitor is present it changes the appareni Ky of the The word "apparent" is used because a characteristic

,,diil

fi

q@

,@m

ne@

Lineweaver-Burk plot.

that the Vrnu* remains constant while the apparent K1,4 changes. Note that a competitive inhibitor will increase the slope of tne tines" found in the

each plot the concentration of inhibitor and enzyme is constant. Again, we find

shown in Figure 7-7 is the Lineweaver-Burk representation of Figure 7-6. In

[I] = 2Kt

lll

= Ki

Uninhibited

Slope

Kv
V*u*

V.o*

_1
Krra

Apparen

?
tt4

oo

_1
Kv

Figure 7-7
Lineweaver-Burk Plot for Competitive Inhibition.

Copyright @ by The Berkeley Review

The Berkeley Revieu Specializing in MCAT preparatior

Biology

Metabolic Components
a

Enz5rme Kinetics

An example of a competitiue inhibitor would be ethanol. Ethylene glycol,

constituent of antifreeze, can be converted via a.number of reactions in the body to oxalic acid (which causes kidney damage and often results in death). One of those reactions involves the enzyme alcohol dehydrogenase. It turns out that alcohol dehydrogenase will also convert ethanol to acetaldehyde irvhich can then be converted to pyruvate--more on this in a later discussion). If ethylene glycot is ingested, all one needs to do is drink intoxicating levels of ethanol. The end result is that ethanol will compete with ethylene glycol for the active site on the alcohol dehydrogenase enzyme, and since there is more ethanol :han ethylene glycol around, the enzyme will convert much more ethanol to :ceialdehyde than it will ethylene glycol to oxalic acid.

,\nother example of a competitive inhibitor is methotrexate, an anticancer drug. As we will see later, rapidly dividing cancer cells require a continuous stream of :recursor molecules (called deoxyribonucleotides) for the synthesis of DNA. I.lethotrexate resembles a molecule called dihydrofolate, an intermediate in the ::ductive synthesis of tetrahydrofolate (Figure 7-8).
-

re enzyme that carries out this reaction is dihydrofolate

reductase'

l.lethotrexate binds to dihydrofolate reductase and prevents the conversion of :ivdrofolate to tetrahydrofolate, which is not only an essential cofactor for the .-,-nthesis of one of the bases in DNA (i.e., deoxythymidylate (abbreviated as :TMP)) but it is also a cofactor in the synthesis of all other reactions that depend ,:. tetrahydrofolate (e.g., the amino acids methionine, histidine, and the purine : :,.svnthetic pathways).
n
H
I

C-

NH

+i,:. T +i::"
- N-H

- C-

COO

C-NH_C_COO

"1r ____1 A.-,[ H.N*fH ,k"


"

''-.]/
H

l,

{ .",-

,i

"

Dihydrofolate Reductase

Tetrahydrofolate (THF)

nrgLrre 7-B
:

--

::3xate is a Competitive Inhibitor.

ght O by The Berkeley Review

I l5

The Berkeley Review Specializing in MCAT Preparation

Biology
kr Ba$:ES
+

Metabolic Components
Non-competitive Inhibition
k3

Enzyme Kinetics

k2+ I
ESI

__+E+P

I
EI + S:

lfn k1 filn
k2

Non-competitive inhibitors do not compete at the active site of an enzyme, but rather bind to some other site on the enzyme. Binding at this ,'ailosteric site,, will alter the conformation of the enzyme ihus giving rlversible inactivation of the catalytic site. These inhibitors can bind tJbottlthe free enzyme and the enz).nne-substrate complex (Figure 7-9).
One characteristic of non-competitive inhibition is that it cannot be overcome by increasing the substrate concentration. This means that we will have

t I

il

Figure ?-9
Non-competitive Inhibition.

apparent

v*"*

for the reaction. However, the

V,nu*

Consider one specific type of enzyme. The non-competitive inhibitor that we add to the reaction mixture containing this specific ut iy-" will bind to the enzyme at some allosteric site and cause a conformational ihange to alter the enzyme,s

Ky will not change. why?


unit. However,

alower

:V 'max ts]

g2

()

Non-competitive

Inhibition

oKu
tsl

"itulytic there are still other enzymes in the reaction solution that may not have had an encounter with this non-competitive inhibitor. Those unry*", have active sites that are just as active as they ever were, and they still have the same Kp1 value as their brothers and sisters (because the K1,4 is a relationship b"i.,"""r, substrate and actiue enzyme). But since their numbers have been reduced (by non-competitive inhibition), they have a lower overall vmax. This can be seen
in Figure 7-10.
The Lineweaver-Burk representation of the plot shown in Figure 7-10 isshown in Figure 7-11. Note that the apparent K1,4 remains constait while the v-6y

active site. This enzyme is no longer functional u" u

Figure 7-lO
Non-Competitive Inhibition Decreases Vmax.

clecreases.

Uninhibited

Figure 7-l I
Lineweaver-Burk Plot for Non-Competitive Inhibition.

Copyright @ by The Berkeley Review

116

The Berkeley Specializing in MCAT

Biology

Metabolic Components

Enryme Mechanisms

biochemistry one usually learns the mechanism for at least one of four classic ::lzymes. Lysozyme is an enzyme that hydrolyzes a specific glycosidic bond in re polysaccharide component of certain bacterial cell walls. Ribonuclease is an :lzyme that catalyzes the hydrolysis of phosphodiester bonds in RNA polymers. Carboxypeptidase, is a zinc-containing digestive enzyme secreted by the .rocrine cells of the pancreas. This enzyme hydrolyzes the carboxyl terminal ::ptide bond of protein polypeptide chains. Chymotrypsin catalyzes the :'" drolysis of either ester or peptide bonds. All of these enzymes are referred to :: hydrolases. Water is introduced between a particular bond that had originally :'=en formed from the loss of water. In this section we will consider the reaction :- echanisms for chymotrypsin.

chtffiff

ffi,sflft...iiffiHili bm

-:.r'motrypsin catalyzes the hydrolysis of either ester or p"pt![" bonds. If a ',::::le is hydrolyzed, the products will be an amine and an acid. If an ester is " , 3ro1yzed, the resulting products will be an alcohol and an acid.
-r:-,r- rvas this catalysis at the active site revealed? One of the amino acid residues ,: -r-e active site was identified using diisopropylphosphofluoridate (DIPF), an r':=..-ersible inhibitor. It turns out that chymotrypsin has 28 serine residues, yet : -.' one of those residues, Ser-195, reacts with DIPF. Another of the amino acid ':s-jues at the active site, His-57, was identified using tosyl-L-phenylalanine -:" -:omethyl ketone (TPCK) and a process called ffinity labeling. Knowing the : . . peptide sequence of chymotrypsin, making use of the different types of ,,i:,=-::rg techniques, and using x-ray studies, it was discovered that His-57 is not --" adjacent to Ser-195 but also to another amino acid residue, Asp-102. These :'':- residues, shown in Figure 7-12, lorm what is called a catalytic triad at the - = site of chymotrypsin.

-:=: physiological conditions, where the pH is about 7.4,5ev195 will have a " , ::ound 13. What this means is that for all practical purposes, Ser-195 will
"
',::,

li,t ::tain its hydrogen atom on the hydroxyl group of its side chain. (Remember, ,": :t',? pH of the solution is greater than the pKaof the ionizing side chain, the '" ;":*'.:nnnt species is the conjugate base of the side chain, and when the pH of the i r -" . ": is less than the pKa of the ionizing side chain, the predominant species is the u.,:",,i .:,u,other way to think about this is as follows: pH ) pKq, then [HA] < [A-1 and iJ r i < vtl(o, then [HA] > tA-1.) Therefore, at a physiological pH of about 7.4, only l -: 1 r 10-6 molecules of Ser-195 will be in the form of the conjugate base.
i'e:-195 is reacting with DIPF, there must be something that is allowing the i : : -lrat serine residue to be lowered. If the pKa of serine is lowered, there will rr; r ::: of a chance of finding it in the form of the conjugate base. In this case the rr -'u-. >:de chain would be in the oxyanion form. When alcohols lose their :r:, :.ns they become alkoxide ions--and an alkoxide ion is a stronger base and 1"",,,:r'i.f , rE a better nucleophile--and nucleophiles love to give away electrons.
xu

r,i-:*.

"llr::: r
rrr

.-ut that the His-57 residue is unprotonsted when there is no substrate the active site. Remember, histidine has a pKx of about 6.0, and since
Berkelev Review

i- --f 9 bv The

tt7

The Berkeley Review Specializing in MCAT Preparation

Biology

Metabolic Components

Enzyme Mechanisms

we are at a pH of about 7.4, roughly 96% of it will be unprotonated. The Asp-102 residue that is part of this triad has a pKa of about 4.4. At physiological pH it will essentially be in the anionic form. It therefore can act as a base and interact with His-57, thus making His-57 an even stronger base than it would have been if the Asp-102 residue were not present. What this means is that His-57 can abstract a proton from Ser-195 as the substrate comes into the active site. Once Ser-195 loses the proton from its hydroxyl function, it becomes a rather reactive alkoxide ion.This portion of the mechanism is shown in Figure 7-12.
His57

Ser1e5-

o- H ------ -!------: *V"1t

/-;4

\ I
H

----------------o-

fl

c-

Asproz

I
)
I
H

His57

Ser1e5- O:-------------'

oF.oR- *\Z*H

---------------

g- C- Asproz

Figure 7-12
We have mentioned that chymotrypsin catalyzes the hydrolysis of either ester or

peptide bonds. Let's consider the hydrolysis of a peptide bond in a typical


protein substrate (Figure 7-13).
His57

Ser1e5-

Or O ) i./ i.. R'C' llr NH o)


Substrate

u
Rr

N.l ,,
lt

/4. Nil
His57

)
I

---------------:

ofl o- C-

Asproz

Ser195-

t..

F' N 7, NI

--------------- O- C-

ofl

Asproz

Unstable Tetrahderal Transition State

Figure 7-13

Copyright @ by The Berkeley Review

lra

The Berkeley Revier Specializing in MCAT Preparatior

Biology

Metabolic Components

Enz5rme Mechanisms

-\lkoxide ions are strong bases and quite reactive. When the Ser-195 residue is in re alkoxide ion form it is a strong nucleophile, and nucleophiles love to seek out :arbonyl carbons and pass electrons to them. (Carbonyl carbons are partially :csitively charged while the oxygen is partially negatively charged.) Flowever, -:is will place too many electrons on the carbonyl carbon. The result is that the =-ectrons in the double bond of the carbonyl will move to the oxygen.

lrce the electrons move to the oxygen we form an unstable tetrahedral :".tttsition state. The function of the enzyme is to stabilize this transition state. - ris is accomplished by hydrogen bond interaction of the negatively charged ':bonyl oxygen with the N-H groups of the main chain Ser-195 residue and a l-r-193 residue located in the vicinity.
thi.s point the electrons essentially have two paths they can follow. The first is ": --a: they can return to where they came from (reversing Figure 7-13) or they can

::-:eed in such away that there is collapse of the tetrahedral intermediate to


::n the acylenzyme. This occurs with expulsion of the leaving group, which in ,- -: case is the amino product (Figure 7-14).
His57

Ser195-

I r---. | /" -" R'C: ' l\HN: R, o) v

..,-"-.-)..Z*1t

J.
His57

--------------- O- C-

L?

Asp1s2

nstable Tetrahedral Transition State

ie119-i-

:NVN- H --------------- O- C- Asp162


Acyl Dnryme Intermediate
H

Rr-

I d - N- R, '

Amino product
leaves the scene

tilgure 7-I4

' . ;:-,'1 enzyme


--,",

intermediate that we have just formed provides a lower energy

ar-

to get from the substrate to the product. Once the amino product is
by Mozart).

: - red the First Act in the opera


' -n
r-r: opera Die ZauberJldte

=: it is free to leave the scene. We could consider the steps that we have just Chymotrypsin (abit like the First Act in the

- ::.e beginning of the Second Act we need to introduce water in order to , -;. our acyl enzyme intermediate. As the His-57 function removes a :,- : ::n atom from water, there will be a nucleophilic attack on the carbonyl of the acyl enzyme intermediate by the oxygen atom on the water , : --e. Again, a transient tetrahedral intermediate is formed as shown in
.:

.-:l

-l-5.

::rt

O by The Berkeley Review

t19

The Berkeley Keview Specializing in MCAT Preparation

Biology

Metabolic Components
His57

Enzyne Mechanisms

o
H

----------- -"o-'J-

Asproz

Hll I
His57

Ser195-

O Rr- C-

) o /t-/' ]O ll O HH ------------Y9- C- Asp1s2 ^\z^O

J^ e ,! C '
Figure
7-

Unstabte Tetrahedrat Transition State

l5

The His-57 residue is now in a position to donate a hydrogen atom to the ser-L95 residue. Again, we have an electron shift. The transient tetrahedral intermed.iate collapses and the acid component of the substrate that we started with is free to leave the scene. Note that we now have the enzyme in its original starting condition--ready to accept another substrate molecule for catalysis (Figure 7-16)His57

\ I

l--\o ---ser1e5-o----.------s -q\zN" J


R1- QI

o ll -- o- c-

AsP162

lll oo'

g
H

il

I
llis57

Ser195- O-

o t *- H --------------- o!
O

o ll

C-

Asp162

R1- C-

ilt OH

'

Acid Product can leave the scene

Figure 7.16

In the analysis of this mechanism we looked at the hydrolysis of a generic peptide bond. However, chymotrypsin is rather selective in the peptide bonds that it will cleave. For example, chymotrypsin will hydroryze the peptide bond on the carboxyl side of Phe, Tyr, and Trp, three aromatic amino acid residues.
Copyright @ by The Berkeley Review

120

The Berkeley Review Specializing in MCAT Preparation

Biology
residues like Leu, Ile, and Met.

Metabolic Components

Enz5rme Mechanisms

chymotrypsin will also hydroryze on the carboxyr side of large hydrophobic It turns out that the side chains of these'amino residues can fit into a special nonpolar pocket near the ser-195 residue of the
active site. This pocket allows {or chymotrypsin,s specificity.

ffil#dtiv.$Ll

.ilil"'l'i'ltltti

, ,,,.,,,,,,,

.,::.,

::r.

:,ii,:i:i

*.

During the early 1980s Thomas Cech and his colleagues discovered that RNA can act as a catalytic enzyme. In the ciliated protozoan Tetrahymena thermophila, a 414 nucleotide intron is excised from a 6.4 kb ribosomal RNA precursor in the Dresence of a cofactor which proved to be a guanosine (G) residue. The release of ihis 414 nucleotide intron and subsequent splicing oi th" juxtaposed exons ;howed that an RNA molecule can have cataiytic a"ctivity. in" nsnucleotide ntron undergoes two more rounds of self-splicing, first ltsing a 15 nucleotide :ragment and then losing a 4 nucleotide fragment."The linear fiagment of rRNA rat is left is called L19 RNA. (The "L" stanJs for the fact that it is linear and the 19" means that 19 intervening nucleotide sequences were removed since the :crmation of the 414 nucleotidsintron. It is now 395 nucleotides long.) L19 RNA *' rather stable and can act on other substrates. As we will see later, Ihis enzyme
both a nuclease and a polymerase.

InactiV
-

$iFr$6ut$#C
hrorder ior these

-'"-rI1ogens are inactiae precurslrs of the active enzyme.

'lany enzymes are initially synthesized in the form of a proenzyme or zymogen.

-r-:e.chymotrypsinogen is synthesiled in the pancreas it is secreted into the r:-stinal tract where it is activated by a proteolytic enzyme called trypsin. The ':::"'ated enzyme, chymotrypsin, can then hydroryze proteins found in the "-:-:tine. If the pancreas were to become damaged, the pancreatic secretions ':ld rapidly become activated and digest the pancreas, giving rise to the . :. :ition called acute pancreatitis.
-"

"or,rid", -- "-motrypsin is the qctiT)e form of the enzyme. The inactive form of this enzvme -:e zymogen) is synthesized in the pancrlas and is caned chymotrypst"G;.

"rrryi.r", so*e type of proteolytic activation "' rich is an irreversible step). As an examprl, chymotrypsin.
become active, they must first undergo

=:e is a naturally occurring substance called a1-antitrypsin which is an :-mely effective. inhibitor of trypsin and an even better inhibitor of elastase ,,:;- called antielastase).Th1s inhibitor prevents these digestive enzymes from ',:=sting the rest of your body. For exampre, antitrypsin iJpresent in the tissues ::. lung and prevents elastase from digesting connective iissue proteins in the = - -ar walls of the lungs. Genetic disorderi leading to a deficiency in this '"' :-tor can result in a clinical condition known as eiphysemq. Aperson with : :--,','sema breathes much harder than the normal individual. Individuals who --i , r,:e cigarettes also have the possibility of developing emphysema. It turns out "I -i{arette smoke oxidizes a specific Met residue on antitrypsin and converts 'nethionine sulfoxide. The Met residue is essential for bindLg elastase. Once 1: .s converted into methionine sulfoxide, elastase no longer binds to the
'Lr

."

,:::tr.

Mlotr

eculru,,,E#otutidfi

'' :lastase. These three enzymes are secreted by the

-:rer digestive enzymes that are quite similar to chymotrypsin are trypsin p^rl"."ui and all have the
O by The Berkeley Review

" *::rr

t2t

The Berkeley Review Specializing in MCAT preparation

Biotogy

Metabolic Components

Enzyme Mechanisms

ser-His-Asp catalytic triad that we have been discussing. They are serine proteases and their mechanisms of action are also similar. It turns out that about 40'/. of the overall amino acid sequences in these proteins are identical. why? Because these three enzymes evolved (after mutation and duplication) from a similar ancestral enzyme. The evolutionary process that allowed for these three distinctive enzymes is called divergent evolution.
The bacterial enzyme subtilisin (isolated fuornBacillus subtilus) is also a serine protease.If you compare the sequences of amino acids in subtilisin with those in chymotrypsin (or trypsin or elastase), you will find a remarkably different composition. These two enzymes (probably) did not have a common ancestral enzyme and therefore were evolutionarily independent of each other. Even though subtilisin is a serine protease (as are chymotrypsin, trypsin, and elastase),

the amino acid residues located at the active site of the enzyme are in functionally different positions than the amino acid residues at the active sites of, say, chymotrypsin or elastase. \A/hat this means is that both the bacterial enzyme and the mammalian enzyme have found a similar way to catalyze a particular reaction. These two independent processes are probably due to an evolutionary process called convergent evolution.

ffi il$iti$rHii| $tfi tCIffi

ffi

fi t6g

Transition state analogs are synthesized with the hopes that they will occupy the active site of an enzyme. The role of an enzyme is to bind the transition state and stabilize it. A transiiion state analog is thus a competitiae inhibitor of the actual reaction.
The vertebrate immune system's single most important function is to recognize self from non-self. If a foreign substance (an antigen) enters into the body, the immune system will recognize that substance as being foreign and begin to synthesize antibodies or immunoglobulins against it. Suppose the foreign antigm that entered the body was a transition state analog. The antibody synthesized in response to the transition state analog will bind to that analog in a very specifi,c

way. This is exactly analogous to the way in which an enzyme would bind a substrate. What we have done is to make the immune system produce an "enzyme," which is the antibody. If we remove the transition state analog from, the antibod/, what will be able to fit into the space left behind? How about substrate or the product from the reaction in which the transition state was competitively inhibiting? This would suggest that such antibodies w have catalytic activity. It turns out that they do! These synthetic ant "enzymes" are not necessarily as good as the naturally occurring enzymes, they do have enzymatic activity. This might be a way to design and synthesi your own protein(s).

Copyright @ by The Berkeley Review

122

The Berkeley Specializing in MCAT

Biology

Metabolic Components

Metabolic Molecules

\Ietabolism is the generalized word for all of the processes which occur inside -:r'ing organisms. Metabolism can be divided into catabolism and, anabolism. catabolism means the breakdown of complex molecules into smaller and simpler products accompanied by the release of. energy.In most instances the :ody attempts to harness this energy in the form of adenosine triphosphate -LTP). Anabolism means to build up or become more complex. It is the :iosynthesis of small precursor molecules into larger and more complex :-olecules. Entropy is reduced in this energy-requiring organizational process.
l\-e can diagram these two metabolic processes as shown in Figure 7-17.

Complex Complex biomolecules

"food"
molecules

ADP

+Pi

Adenosine Triphosp62f,g,,,,(ATP
'",-e
"

\il" I

)
ATP
Simple nutrient materials

will first concern ourselves with catabolic processes and then after that we rll look at selective anabolic processes. In order to begin our discussion of ,::abolism, we need to look at the chemical properties of ATp. As we will see, {'TP is the cunency exchange in the cell. A good analogy for ATp are dollar bills;
--u make

/ ll.
u
Less

lll-".

Il8

l3

them and you spend them in the course of a lifetime.

\TP is composed of adenine (a base), D-ribose (a sugar), and three phosphate .:,rups. Adenine and D-ribose taken together make up the molecule adenosine. ,:-,',-e add one phosphate to adenosine, we get adenosine monophosphate (AMp). :'"',-e add two phosphates to adenosine, we get adenosine diphosphate (ADp). '--,d if we add three phosphates to adenosine, we get adenosine triphosphate \T?). Note the numbering of the two rings shown in Figure 7-18. The adenine ** : system (called a purine ring) has the numbers 1 though 9 as shown. The -:."::se ring has the numbers 1' through 5' (the "prime" markings are important).

complex "waste" products

Figure 7.17
Catabolism and anabolism are the components of metabolism.

AcidI anh Lnhydrid de


kages tiinka5

o il il5', -.P - O_ CH 2tO P- O-,P-r -P-O -P^t ,o oJ -t )o


rosphon Phos monoester mon (age link kase

e{l e{i I

,.r
Adenosine is just

Jo)Jfi LJ

Adenine & D-Ribose

ADP
ATP
ilfltiumure

7-IB
Triphosphate (ATP)

,.i*jr. :-ne

li" *ght @ by The Berkeley Review

123

The Berkeley Review Specializing in MCAT Preparation

Biology

Metabolic Components

Dnzyrme Kinetics

HjC- C- O- C+H.O ll -H"O


4t

oo ilil
'I

The linkage between the C1' of the ribose ring and the N9 of the adenine ring is an N-acetal linkage. It is also referred to as an N-glycosidic linkage. The linkage between the C5' of the ribose ring and the first phosphate group is a phosphomonoester linkage. The linkages between the first and the second phosphates and the second and third phosphates are phosphoric anhydride linkages.
CH3

Acetic Anhydride

A similar situation

stems from organic chemistry where you may have encountered acetic anhydride. Aceti<- anhydride is the anhydride between two molecules of acetic acid as shown in Figure 7-19.

I
OH

o il 2 H3C-CFigure 7-19

Acetic Acid

The hydrolysis of acetic anhydride to two molecules of


acetic acid.

bonds. If you were to hydrolyze acetic anhydride, the equilibrium constant would favor the acetic acid products more than you would have expected had you made it analogous to an ester where you would get back an acetic acid and an alcohol product. one of the reasons for this is that in the acid anhydride situation the acyl groups do not have the option of becoming stabilized by resonance in their acid anion form. \Atrhen acetic anhydride is hydrolyzed you get two molecules of acetic acid which can be stabilized by resonance. This stabilization of both molecules of (product) acid probably leads to this large release of energy that we have mentioned.

Both esters and anhydrides can be hydrolyzed with water. However, anhydride bonds hydroryze in a manner that releases more energy than hydrotysis of ester

ffipffiGfi$

iiilffi:i:ffi# si,rl,

Enzymes often require substances other than amino acids to carry out tl functions. These substances are called cofactors. Consider an enzyme with active site. This active site is isolated from the rest of the enzyme and within cavity are lrydrophobic side chains. If you have a substrate (s) which is in solu it tells you that it must have some degree of polarity-otherwise it would not soluble in that solution. There are a number of reactions that must be carried in an anhydrozs condition (without water). Suppose you wanted to do to the substrate that required a non-polar environment. one way to achieve situation in a cell that is full of water is to get the substrate to settle down within the pocket of the active site (which is surrounded by hydrophobic residues). If this happens, the substrate can be converted into product. once the substrate settles down at the active site, what will happen? Recall that we have a "tool kit" of 20 amino acids. Some of these amino acids are acidic (they can catalyze a reaction by donating a proton) while others are basic (they can accept a proton). These types of amino acids are often involved in acid/ba catalysis. There are also amino acids which can be oxidized or reduced li cysteine. There are also amino acids with hydrophobic side chains which good for excluding polar compounds such as water.

cannot be done with our 20 amino acid tool kit. For this you need additional apparatus at the active site. In hemoglobin that additional app was the heme prosthetic group. other prosthetic groups are necessary for simi reasons. You can do things with these prosthetic groups that you cannot do w any of the 20 amino acid side chains. Therefore, a prosthetic group at the acti site may be necessary to achieve some particular type of catalytic process. are a whole series of prosthetic groups that need not be bond in a cov manner at the active site. These are the cofactors that we just mentioned. If cofactors are required by an enzyme, they are called coenzymes. A coenzyme is
Copyright @ by The Berkeley Review

There may be other things that you would like to do to this substrate--things

124

The Berkeley Specializing in MCAT

Biology

Metabolic Components

Metabolic Molecules

, :n-amino acid, non-polypeptide, non-protein substance which is required for


-:-e activity of an enzyme. In many cases the coenzyme is bound to the enzyme in ; :.on-covalent manner but there are also many cases in which the coenzyme is
:

:i-'gle enzyme.

--"'alently attached. There also may be more than one coenzyme involved with a

lTicotilild

d';Huffi tfi Diffi il;GlEutiffi lltil.ffi il

Business end

NAD
'N

il

-:re coenzyme that will become very important to us is nicotinamide adenine :.:rtucleotide (NAD) whose structure is shown in Figure 7-20. Note the reactive o ^ -.:t or "business end" of the molecule. " n - 'J -

o-cH,

::':etv of NAD is one nucleotide while the other

,1ere does "dinucleotide" in NAD come from? The phosphorylated adenosine phosphorylated moiety (the *-:.'se and the pyridine ring) of NAD is the other nucleotide. The "businesi end" : \AD contains a pyridine ring linked to an amide. This is called nicotinamide ,: niacinamide). If we add a carboxylic acid function to the pyridine ring, we " - have nicotinic acid. This is also referred to as niacin. why call it niacini
=re is a carboxylic acid function in the structure of nicotinic acid, but what : - rt the "nicotin" prefix? Tobacco was brought back from the New world to the

p
"

HO

OH

-:
,

"

- : ivorld by a Frenchman by the name of Jean Nicot.


-:Cnic acid.

'

The tobacco plant was .:.ed Nicotiqna tqbscum after Nicot in his honor. Nicotiana contains a iubstance '--ti nicotine. If you were to oxidize nicotine with HNo3, you would get

Figure 7-2O
Nicotinamide Adenine Dinucleotide (NAD) NHz

O O"-"" O'
Pyridine

N
I

CH:

Nicotinic
(or

acid niacin)

Nicotinamide (or niacinamide)

'\,E:rre 7-21
.-.

.:n-based ring structures.

have the ability to synthesize the nicotinamide (or nicotinic acid) ::-rn of the NAD molecule provided they have enough of the amino acid ::",:tophan in their diet. Tryptophan is an essential amino acid. Once we have ': :;ced nicotinamide the rest of the NAD molecule can be synthesized. If we ,.: : diet low in tryptophan, we would not be able to synthesize as much - :--:,amide and therefore would have a deficiency in the NAD coenzyme. Since .:ramide is an amine and it is "vital" to the diet, we would call it a "vital - -:=." Today these are known as "vitamins" (from "vital amine"). If we do not - = enough of the NAD coenzyme to carry out our metabolic reactions, then we -- :.ot function as well. This is the basis of a vitamin deficiency.

- :.ans

:,:r,s out that nicotinamide was one of the first vitamins ever discovered. '.- *:.d the same time that this vitamin was being discovered, it was also , :- -'-i:rced that the nicotine in cigarette smoking was bad for you. The general ": : had a rather hard time distinguishing between the words nicotine and .namide. Nutritionists decided that the name "nicotinamide" had to be
::sht
@

by The Berkeley Review

125

The Berkeley Review Specializing in MCAT Preparation

Biology

Metabolic Components

Enzyme Kinetics

changed in order to make consumption of nicotinamide more psychologically acceptable. So, instead of calling it nicotinamide, the same substance was renamed niacinamide. Nicotinic acid was renamed niacin (Figure 7-2L)
The coenzyme NAD is held at a specific site on the surface of the active site of an enzyme. NAD is held there by its R group (the adenine dinucleotide portion). When you use a screwdriver the part that engages the screw is just the head of the screwdriver itself. Without the handle the screwdriver would be useless. The enzyme is holding the NAD coenzyme by its handle (the R group portion which

o
O
Substrate

has negative and positive charges) and positions the nicotinamide portion toward the incoming substrate. This can be seen inFigweT-22.

iffiHiffiiii ilffffiiiiiffiffiiffiffiftfiffiliiiift,H#ffiil,,:,,,,',,,,,,,,,,',r,,','
In Figure
7-23a we see the structure of

Figure 7-22
Enzy mel co enzyme i nteracti on

riboflavin (vitamin Bz), a coenzyme component. We cannot synthesize riboflavin. Instead, we must obtain it elsewhere (e.g., our diet). Riboflavin is a component in the structure of a
molecule called flaain mononucleoflde (FMN), which is shown in Figure 7-23bRiboflavin has been phosphorylated to give FMN, the active coenzyme. Thc bond involving the phosphate and oxygen atoms in FMN is a phosphate monoester bond. Phosphorylation of riboflavin has increased the size of thc "handle" that we mentioned concerning NAD. FMN is another type of enzFrrn that is often involved in redox reactions.
o il
o

at the business end of NAD.

u^c2'Y**
VNN
I

"- " Lo A ",.-lo-[


(a)
H- C|
I

":.\,,1|,.4-,to
(b)
H_C-H H- C- OH
I

^'-2'{*\A
I
r

^- "

"-f-"
H_ C.

OH

H- C- OH
OH

H_ CI

OH

H_ C_ OH

a_

cH2-oH

H2C-

PO3-

Riboflavin
o

Flavin Mononucleotide il

,.r2-{*$nI
I

","-!,ll-*A*!o
H-C_H

"

NHt

(c)

H- c- oH H- C- OH H-C-OH
I I

#r
o
CH"

,/i cH2-o-l-o-,t-o
tllll

oo
Figure 7-25
The Flavin Derivatives

oo

Flavin Adenine Dinucleotide

Copyright @ by The Berkeley Review

t26.

The Berkeley Specializing in MCAT Pre

Biology

Metabolic Qomponents

Metabolic Molecules

If we were to add a phosphate, ribose ring, and an adenine base to FMN, we -.r'ould get the coenzyme flaain adenine dinucleotide (FAD). This is shown in Figure 7-23c. The flaain portion of FMN and FAD is the collection of three rings :t the top of the molecule. Recall that we have mentioned that when you connect : base (in this case, adenine), a ribose ring, and a phosphate together, it is :eferred to as a nucleotide. The FMN molecule itself looks a bit like a nucleotide. lhus, FMN is called flavin mononucleotide while FAD is called flavin adenine
:rnucleotide.

Coenzyme A (GoA)

,i:

,,,,,,

,,,i,,i,,;,

--nother coenzyme that will be important to us is Coenzyme A (often abbreviated -oA). The terminal sulfhydryl group is the active site of CoA. The rest of the : rlecule can be thought of as being like a handle, similar to the handle on a .::ervdriver. This is shown in Figure 7-24.

{.cvl groups can be linked to the sulfhydryl function to form acyl coenzyme A :: acyl CoA). A common acyl group that is linked to CoA is an acetyl moiety, 1" :-. giving acetyl CoA. Because the AGo'for acetyl CoA is -7.5Kcals/mol, this r . -ecule has a high acetyl group transfer potential. In other words, acetyl groups r,,:- :e easily transferred from acetyl CoA to other molecules. :*':der CoA as shown in Figure 7-24 for a moment. where did the components come from? If we hydrolyze the amide bond towards the sulfhydryl *: ::.rn, and then carboxylate the CH2 moiety next to the nitrogen, we will have ''' sieir. If we hydrolyze the second amide bond in from the sulfhydryl function, ii- r --r.in carboxylate the CH2 moiety next to the nitrogen, we will have aspartic :: Ii rve hydrolyze the first phosphoester bond closest to the sulfhydryl : :i -::':'r. we find a segment that is synthesized fuorrr three acetate residues. The " r;-: ler of the molecule is just resembles ADP.

, lr-{

,rrrL

coenzyme

A
cH2

|1- .":

{
I

- cHz -

N-

end" -, :.i ':'!rru / -.: -er..'ed from

I c-

cHz- cHz- N- c-

iH HCH:
f
Derived

9?"t", I o- I o- oP-

N' N4't ) -JL

TN,

P-

Oo oo

"f

from :; s:eine by aspartic acid by -)\u;- :oxylation decarboxylation


Derived

fiom 3 acetates (many -) steps)

AI

" o-

Zo.. K } fJ

oH

"

"o-P=o

:," The Berkeley Review

t27

The Berkeley Review Specializing in MCAT Preparation

Metabolic Components
15 Passages 100 Questions

Passage Titles

I. II. IrI. IV. V. VI. VII. VIII. IX. X. XI. XII. XIII. XI\,'. XV.

Questions
6-

Enzyme Inhibitors Enzymes, Coenzymes, and Vitamins Plasma Glucose Measurement Brown Adipose Tissue Heme Metabolism Enzyme Kinetics I Vitamin Brz Enzyme Nomenclature Lysozyme Mechanism Enzyme Kinetics II Adenosine Triphosphate (ATP)

1-5
11

5l-58

24-29 30-36 37-43 44-50

]8-23

12-11

ProteinDegradation(Ubiquitin) Nutrient and Proximate Analysis Niacin Experiment Diffusion-Limited, Enzyme-Catalyzed Reactions

59-65 66-12 13-19 80-87 88-94 95 - 100

Suggestions
The passages that follow are designed to get you to think in a conceptual manner about the Processes of moleclhr biology at the organismal level. If you already have a solid foundation in molecular biology, many of the questionr yo, r"ud here will seem to be very straight forward and easy to answer. But if you ,r" ,r"* to the subject or if you have not had a pleasant experience with molecular biology in the past, io come from the void that spreads out beyond the Oort field at the edges of some of them might "pp""r our solar system.

pick a few passage topics at random. For these initial few passages, do not worry about the time. Just focus on what is exp6cted of you. First, read the passage. Second, look at any diagrams, charts, or graphs in it. Third, read eich question and the accompanying answers carefully. Fourth, answer the questions the best you can. Checkthe solutions and see how you did. \Atrhether you got the answers right or wrong, it is important to read the explanations and see if you understand (and agree with) what is being explained. Keep a record of your results.

After you feel comfortable with the format of those initial few passages, pick another block of passages and try to do them in one sitting. Be aware that time is going to become important. On average, yo.t nlve abouf 1 minute and 15 seconds to complete a question. Be creative in how you aPProach this next group. If you feel comfortable with the outline presented above, fine. If not, then try different

appro-achei to a passage. For example, you might feel well versed enough to read the questions first andthen try to answ-er sorie of them, without ever having read the Passage. Maybe you can answer some of the questions by just looking at the diagrams, charts, or graphs that are presented in a particular passag. Rem-ember, there are many effective leaming styles. You need to begin to develop a format that works best for you. Keep a record of your results.

The last block of passages might contain at least a few topics that are unfamiliar even to those who know a good deal about molecular biology. Find a place where the level of distraction is at a minimum' Get out your watch and time yourself on these passages, either individually or as a grouP. It is important to have a feel for time, and an awareness of how much is passing as you try to answer each question. Never let a question get you flustered. If you cannot figure out what the answer is from information , given to you in the pu"rlg", or from your own knowledge base, dump it and move on to the next and come back to it when you have more [uestion. As you do [his, make a note of that pesky question make sure you understand the solutions. Be time. When you are finished, check your answers and look it up. The solution tends to stay with you inquisitive. If you do not know the answer to something, longer that way. (For example, what ls the Oort field, anyway?) approximations and should be used only to give one a feel for which ballpark they are sitting in.

The estimated score conversions for 100 questions are shown below. At best, these are rough

Section Vtr Dstimated Score Conversions


Scaled Score Raw Score 80

>13

- 100

tt-12
9- l0 '7 -8 5-6
<4

70 -'79

60-69 50-59 40-49

0-39

Biology
Passage

Enz5nne

Inhibitors

Passage I

(Questions L-5)

r:rzyme

In the case of a simple enzyme-catalyzed reaction, an E binds a substrate S with a high degree of

.tt-rnity, proceeds through a transition state involving the :nzyme-substrate complex ES, and converts that substrate -rto a product P (or products). This type of reaction is :escribed by Equation 1:

Kv and Vmar and cannot be overcome at high [S]. Noncompetitive inhibitors bind to either the enzyme or enzyme-substrate complex. They decrease the Vmax and cannot be overcome at high [S].

Competitive inhibitors resemble the substrate and bind reversibly at the active site. They increase the Ky and can be overcome at high [S]. Uncompetitive inhibitors bind only to the enzyme-substrate complex. They decrease the

E+S

ES k''

E+P

The rate (velocity v) of this reaction is dependent not

,'{::abolic reactions in the cell occur at higher rates when r,.:re enzyme is available. However, the rate of a reaction :i :enerally determined by [S]. When [S] is low, the rate - i : reaction increases quite rapidly in a linear fashion. ;:en [S] is high, the rate of the reaction begins to
1:in:tion

::-r' on the enzyme concentration [E], but also on the , -bstrate concentration [S] in the cell. Initially, the ':l,rcity of the reaction is directly proportional to [E].

Many cellular enzymes are composed of more than a single polypeptide chain. Instead of having just tertiary structure, these enzymes have quaternary structure and multiple binding sites that can cooperatively interact with one another. Instead of having hyperbolic kinetics, these
enzymes show sigmoidal or S-shaped saturation kinetics.

is maximal and is given the term V*u*. The ,l-:haelis constant, KM, is the [S] at which the initial :. -'ciry of the reaction is one-half the maximal velocity. l:e ;urve developed from this type of saturation kinetics
, :::at of a hyperbola (Figure \',n*
1):

i:,;:ease less rapidly until saturation of the enzyme with L:e substrate is reached. At this point, the rate of the

1.

The diagram shown below represents an interaction between enzyme (E), substrate (S), and inhibitor (I):

g 4ffii dffi+,r \ V
/t

1l^
zr-illll\
(::::::,i'ri',,:'l
\:,!!!!!!!!4!!!

lln
I
fal /f;Ill\
fi:..:...iin'iji:lfSl
\::::iljillili::l::i:li-

I2 >

: rr tmax =

YI\!!,,,t

=r!r-

KM

tsl

Which of the following reversible enzyme inhibition graphs BEST represents the diagram shown above?
[Note; Curve C represents the control.] Figure 2 B.

Figure
ii

: tollows classical
in Equation (2):

Michaelis-Menten kinetics

,:.,r;T--ised

.. 1 _ V.u*

[S]

Kn,,.LSl

/Y4 1
0
1/tsl

,,V,/

-.-:ce it is difficult to measure V6a1 irccurstely from a l.::i:: plot of v as a function of [S], the reciprocal of i;i l[-:on (2) is used to generate a straight line of the form : :x + b. A double reciprocal plot (Figure 2) allows for
iL

D.

l/vl

:r :rr3 accurate

determination of both Vmar and Kvt.

,iur,:"1",

inhibition of an enzyme-catalyzed reaction can be led in terms of Michaelis-Menten kinetics. The three rn I'n'i :-r' types of reversible inhibition are competitive,
rliilL

l:e

-%
l4sl

l/tsl

:n ptitive, and noncompetitive.

lry',:r,sht @ by The Berkeley Review

l3r

The Berkeley Review Specializing in MCAT Preparation

Biology
2.
the Michaelis constant

Enzyme lnhibitors

Passage I

Which of the following is NOT a characreristic of

(Ky)?

5.

in eukaryotic cells it occurs in the matrix of

The Krebs cycle requires eight enzymatic steps, and


the

A. B. C. D.

It is the substrate concentration atYmw<12. Itcharacterizes substrate-enzymeinteraction. It is equal toYmu</2. It is not altered by noncompetitive inhibitors.

mitochondrion. One rate-controlling step involves the conversion of isocitrate to cr-ketoglutarate:


Isocitrate

NAD+1excesr,

.3
+ NADH + H+ +
CO2

cx-Ketoglutarate

Three reaction mixtures are examined:

3.

What is the initial velocity of a reaction catalyzed by the enzyme urease, if the concentration of urea in solution is 2.5 x 10-1KM?

A. B. c. D.

Mixture Mixture Mixture

1:

Substrate andNAD+. Substrate, NAD+, and 2 mM ADP. Substrate, NAD+, and 5 mM ADP.

0.20 vmar,( 0.25 VmarK 0.75 VmaK

2: 3:

The initial reaction velocity (v) as a function substrate concentration [Sl of each mixture is
plotted. Based on this analysis one might conc
that:

0.80 Vmax

4.

Three separate enzyme-catalyzed reactions are run with the same substrate and enzyme at a constant enzyme concentration. The only difference between the three reactions is the type of inhibitor used
during catalysis. Reaction Reaction Reaction

>| .n

1: 2: 3:

No inhibitor added.

Competitive inhibitor added in


increasing concentrations.

Substrate concentration

Noncompetitive inhibitor added in


increasing concentrations.

A.
B. C.

ADP increases the Ku of the enzyme. ADP is an allosteric inhibitor of the enzyme.
a high [NADH]/[NAD+] ratio stimulates
enzyme.
a

The data from each reaction was plotted on the


graph shown below:

D.

high [ATP]/[ADP] ratio inhibits the

l4sl

The point that is common to both Reaction 1 and


Reaction 2 is:

A.
B. C. D.

Point Point Point Point

A.
B.
C.

D.

Copyright @ by The Berkeley Review

132

The Berkeley Specializing in MCAT

Biology
Pa"ssage

EnzSrmes, Coenz;imes, and Vitamins

Passage tr

II

(Questions 6-11)

enzyme to another. They are involved in the :.::sfer of hydrogens and electrons in oxidation-reduction ' :.;tiOns.
The only oxidation step in glycolysis is catalyzed by a :-':r'drogenase enzyme. The proposed mechanism for this ':.;tion is outlined in Figures 1-4:

'::,n one

:-,zvmes called dehydrogenases and can easily move

){icotinic acid is also referred to as niacin. The amide : nicotinic acid is called nicotinamide, a component of :- coenzymes nicotinamide adenine dinucleotide (NAD) .-i nicotinamide adenine dinucleotide phosphate l;.\DP). NAD and NADP are loosely bound to a class of

H-dV,NH
nslo

ll^ tt-b-o"
Product
Figure 4

e \r"5t"ro-]

rj_no 60

N,Lo

\/v
ll^ HrN
I

Zbn
H -uL

A niacin deficiency leads to the disease pellagra, a condition characterized by dementia, dermatitis, and
dianhea. Meat, fish, beans, and nuts are good sources of niacin. Very little niacin is found in either milk or eggs.

; r "-d\r*" .z----\&o s.'


."
H_C_OH
cHroPo;

Riboflavin (vitamin Bz) has a complex isoalloxazine

ring system showing conjugation of multiple bonds (Figure 5). In general, the more conjugated bonds a
molecule has, the longer the wavelength at which the
molecule absorbs light.
Oxidized
isoalloxazine

rSubstrate

Figure

:,.n:*"
R

ring

ll

,r* *

,.-

:"gltr"l
RH Figure 5

Reduced

isoalloxazine

ring

Riboflavin is a component of two closely related

}.T{D
H-

-H

"r,l

..9F

coenzymes, flavin adenine dinucleotide (FAD) and flavin mononucleotide (FMN). These coenzymes are covalently bound to a class of enzymes called dehydrogenases, and they participate in the transfer of hydrogens and electrons in oxidation-reduction reactions.

A vitamin Bz deficiency is rare, but it can occur during pregnancy, in growing children, and during times of stress. If a deficiency does occur, it usually occurs with
Figure
3

pellagra. Milk, eggs, liver, and meat are good sources of vitamin Bz.

:';nght

by The Berkeley Review

133

The Berkeley Review Specializing in MCAT Preparation

Biology
6.
on the:

Enrymes, CoenzSmes, and Vitamins


11. In solution,

Passage Il

Electrons on the nitrogen atom of the imidazolium ring of histidine (His) in Figure 2 ultimately end up A.
B.

succinate is oxidized to fumarate by the

enzyme succinate dehydrogenase.

c.
D.

carbonyl oxygen of nicotinamide. nitrogen of the nicotinamide ring. sulfur atom of cysteine (Cys). carbonyl oxygen of the substrate.

coo
I

FAD FADH2

coo
I

CH" CH"

coo
Succinate

t' l'g

CH

il Succinate dehydrogenase

CH

coo
Fumarate

l6

7.

What is the name of the linkage at Q in Figure 4?

The dehydrogenase enzyme contains a covalently


bound prosthetic group, flavin adenine dinucleotide (FAD). Oxidized FAD absorbs light in the visible

A. B. C. D.

Phosphodiester Peptide

Mixed acid anhydride Amide

region of the electromagnetic spectrum at 460 nmBased on the reaction shown above and the graph shown below, the color change during the course d the reaction is from:
Blue
cone

Green Red

cone cone

8.

The sulftrydryl group of Cys loses its hydrogen atom between Figure I and Figure 2. This is best explained by the need for the sulfhydryl group to
become:

i1 X

A. B. C. D.

a nucleophile, attack the aldehyde carbon of the substrate, and form an acyl thioester bond. an electrophile, attack the aldehyde carbon of the substrate, and form a hemiacetal. a nucleophile, attack the aldehyde carbon of the substrate, and form a hemiacetal. an electrophile, attack the aldehyde carbon of the substrate, and form an acyl thioester bond.

Pk 6O
boo '-o Frb a

400
A.
B.

500

600

Wavelength (nm)

c.
D.

blue to red. yellow to colorless. yellow to blue. colorless to red.

9,

The BEST dietary source for precursors to the


dehydrogenise enzyme would be:

A. B. C. D.

eggs.

fish.
meat.
beans.

10.

Al1 of the following oxidation-reduction pairs could be found at the active site of a dehydrogenase

enzyme, EXCEPT:

A. B. C. D.

reduced NAD; reduced cysteine. reduced NAD; reduced product. oxidized NAD; reduced cysteine. oxidized NAD; oxidized cysteine.

Copyright @ by The Berkeley Review

134

Biology
Passage

Plasma Glucose Measurement


12. Which of these is the structure
a

Passage III

III

(Questions 12-17)

of glucono-6-lactone,
B.

product of Reaction t?

Researchers often need to measure plasma glucose :'"els for reports on clinical experiments. A plasma .:-,.:cose analyzer is a machine containing reagents and :lzvmes that allows rapid analysis of plasma glucose :',e1s. A blood sample is introduced into the analyzer,

A.

o.\ -c- H
I

0t'a o -

_o

.:J

a probe determines the glucose concentration. The , *-icture of the probe is shown in Figure 1:

H_C-OH I HO- C- H

Platinum Reaction 2

H-C-OH I H-C-OH
I

HO-CH- CH- CI

*- i- o'
H OH OH

cH2oH

67cD.

oo

Reaction

C.

CH.OH

"""'YU:""
lmmobilized
Oxidase

)-o

t'

o
d

"J(f1>"
OH

U)

13.

o-Ri ng

The glucose analyzer could be used to follow


changes in plasma glucose over time. Which of the

following graphs indicates how a person's blood glucose would change over the three hours
Figure
1

following
A.

a meal?

The first reaction occurs on the membrane containing

B.

:-:obilized glucose oxidase. Glucose oxidase catalyzes ": : -ollowing reaction:


:-D-Glucose

*02 +

Glucono-d-lactone +H2 O Reaction I

o o o

o o

Tre second reaction occurs at the platinum electrode


.-:: :s shown below: HzOz 2H + 02 + 2e

01234
Time Fotrlowing A Meal (hours)

01234
Time Following A Meal
(hours)

Oo

c.
Reaction 2
c)

D.

.i thin cellulose filter blocks chemical compounds ,' ,:. molecular weights above 200 between Reaction I
.":

C)

o
rFl

2. The electrons produced at the electrode ,:nearly proportional to the H2O2 concentration, and -:'r::ore to the concqntration of the substrate. The current ':: --red on the silver electrode to keep the electrode pair

i':

: Reaction

::-::r1 is recorded as electron flow. Electron flow is *::;ied and reported by the probe to a computer, which
into
a plasma

01234
Time Following A Meal (hours)

01234
Time Following A Meal
(hours)

r.-:::.:i-ies and converts the electrical signal : -: rse concentration.

;
n

.:. rght

by The Berkeley Review

135

The Berkeley Review Specializing in MCAT Preparation

BioIo gv
14.

Plasma Glucose Measurement


17.

Passage Itr

The user of the glucose analyzer first generates a standard curve like the one shown below. The

The function of the platinum electrode in this probe is to act as:

computer then has an internal reference for


interpolating the actual glucose samples from the subject. If the electron reading were 1250, what glucose concentration would be displayed on the computer screen ofthe glucose analyzer ?

A. B. C. D.

a cathode.

an anode. an oxidizing agent. an oxidase.

bo

o o ! o
C)

g.l

60

80 100 120 140 160 180 200


Blood Glucose (mg/dl)

A. B. C. D.
15.

110

mg/dl
mg/dl

220mgldL
330

440mgldL

How would the measurement of the plasma glucose concentration change, if a technician accidentally added hydrogen peroxide to the plasma sample instead of saline buffer?

A. B. C. D.
16.

The glucose reading would be erroneously


high. The glucose reading would not be changed by this addition. The glucose reading would be erroneously

low.

The glucose reading would be zero, because the hydrogen peroxide would destroy the
glucose.

What is the purpose of the cellulose filter between the immobilized oxidase and the platinum electrode?

A. The filter blocks hydrogen peroxide. B. The cellulose allows passage of C. D.


accurate.

many

oxidizabl'e substances, so that the readings are

The cellulose blocks many other oxidizable


substances, so that the readings are accurate. The filter protects the immobilized enzyme.

Copyright @ by The Berkeley Review

r36

The Berkeley Specializing in MCAT

Biology
Passage

Brown Adipose Tissue


19. The most abundant adipose

Passage IV

IV (Questions 18-23)

White adipose tissue (WAT) is the most common fat storage medium in adults and children. The small amounts of brown adipose tissue (BAT) between the shoulder blades and along the nape of the neck of newborn infants is metabolically important for their
survival. The primary difference between the two tissues is that tsAT has several times the mitochondria of WAT. Also, the inner mitochondrial membranes in BAT are more
permeable to protons than those of WAT. The reactions

tissue is called "white," although in humans and many other animals, it is actually yellow, due to stored carotenoids. What provides the brown color in brown adipose tissue

(BArX

A. B. C. D.

Ribosomes in the cytosol DNA in the nucleus Cytochromes in the mitochondria Bacteria in the lysosomes

of

:ridative phosphorylation, particularly the generation of {TP QV ATP synthetase, depend on an inner ::tochondrial membrane that is impermeable to protons. --, the case of BAT, this means that oxidative ::osphorylation is uncoupled, so that fewer ATP :rllecules are made when protons are pumped out of the :ritochondria. This uncoupling is caused by a protein :Aled thermogenin that is present in the inner membrane. l:iermogenin allows the passage of protons back into the
-rtochondrial matrix, bypassing the ATP synthetase. This .rcoupling produces beneficial effects for the infant. The adipocytes in BAT are controlled by extensive i-"mpathetic enervation. When stimulated, these nerves

20.

Norepinephrine first binds to the beta-adrenergic receptors and then acts through a second messenger system to cause changes inside the adipocyte. This sequence of events is represented in the following diagram:

Hormone

Act'ivated

Extracellular

receptor

-:1ease norepinephrine directly at the adipocytes. \rrepinephrine stimulates lipolysis and oxidation of the lerated fatty acids. Through oxidation, both NADH + :--: and FADHz are produced. These give rise to the ::rtons that are pumped across the mitochondrial
::,embrane. The intracellular metabolic pathways and their ::izvmes are not significantly different between BAT and
G protein

cAMP
ATP
Intracellular

A'AT.

GTP

In the diagram shown above, which of the following

intracellular events follows the binding of


norepinephrine to the p-adrenergic receptor?

IE. Which of the following would result from


uncoupling of fat oxidation and ATP production?

the

A. B. C. D.

The hormone is transported into the cytosol. The production ofthe By subunit is increased. Adenylate cyclase is inhibited. Concentrations of cAMP increase.

A. B. C. D.

The production of heat

A local cooling effect Conservation of body fuel stores A lowered resting metabolic rate

21. What is the principal

storage component

of

an

adipose cell in white adipose tissue?

A. B. C. D.
137

Fatty acids Triglycerides Phospholipids Glycerol

- :n1'right @ by The Berkeley Review

The Berkeley Review Specializing in MCAT Preparation

Biotogy
22.

Brown Adipose Tissue

Passage IV

What role can BAT play in glucose homeostasis for the whole body?

A. B. C. D.

BAT provides glycerol for gluconeogenesis. BAT provides glucose for gluconeogenesis.
BAT contains glycogen for glycogenolysis. BAT provides fatty acids for gluconeogenesis.

23.

Thermogenin, the uncoupling protein, is probably MOST similar in structure to:

A. B. C. D.

hemoglobin. chymotrypsinogen.

ATP-ADP synthetase.

Nao-Ko pump.

Copyright @ by The Berkeley Review

r38

The Berkeley Specializing in MCAT

Biology
Passage

Ileme Metabolism
25.
The conversion
described as a:

Passage V

V (Questions 24-29)

of biliverdin to bilirubin can

be

The ability to transport oxygen throughout the body


results from the oxygen-binding capability of hemoglobin Hb). In particular, it is the heme prosthetic group of Hb

;:rat carries out this essential function. The prosthetic :roup consists of a porphyrin ring bound to a central iron ::om, which in turn directly binds oxygen.

A. B. C. D.

one-electronoxidation. one-electronreduction.

two-electronoxidation.
two-electronreduction.

In a disease of porphyrin anabolism, the enzyme -:oporphyrinogen synthase is deficient and a non"-nctional porphyrin skeleton is formed. As a result,
::egular erythrocytes are made and eliminated. This
:-sease, congenital erythropoietic porphyria (CEP), is ::lieved to be transmitted in an autosomal recessive

26. In the absence of


m).

uroporphyrinogen synthase,

uroporphyrinogen I (uro

I)

the biologically active uroporphyrinogen


coo

is synthesized in place of III (uro

..hion.

,--.e

The breakdown of the heme prosthetic group produces end product bilirubin, and the first step involves the

:.:avage of an alpha-methane bridge to form biliverdin. reaction involves the release of carbon monoxide. :-lrverdin is then converted into bilirubin, a reaction

l:rs

::lined in Figure 1. In the liver, bilirubin is attached to ' r glucuronate residues, and the complex is secreted into .--: is released along with bile. Studies show that bilirubin

".:,e degradation.

, an anti-oxidant and has the ability to eliminate " :roperoxy radicals. For this reason, it is postulated that : :lution has selected bilirubin as the end product of
coo
NADPH

H+ Biliverdin \
+
Figure
1

NADP+

'.Bilirubin
NH
N H HN H N

ol

ooc

Conversion of uro I into uro

III requires:

-j

The breakdown of heme to bilirubin takes place in


-:1e:

A. an C. D.

isomerase, converting

symmetric

B. an isomerase,
liver.
spleen. pancreas.

molecule into an asymmetric molecule.

converting an asymmetric

.\. B. C, D.

gall bladder.

molecule into an asymmetric molecule. a kinase, converting a symmetric molecule into an asymmetric molecule. a kinase, converting an asymmetric molecule into a symmetric molecule.

,r.

-_:ht @ by The Berkeley Review

159

The Berkeley Review Specializing in MCAT Preparation

Biology
27. Which of the following
statements
regarding the heme prosthetic group?

Ileme Metabolism
is
TRUE

Passage V

A. B.

Carbon monoxide has a greater affinity for Hb


than for an isolated heme group. Oxygen has a greater affinity for Hb than does carbon monoxide.
be bound to carbon monoxide.

C. A small percentage of Hb can be expected to D. Hb is known to exhibit a noncooperative


binding mechanism,

28.

Based on information

in the

passage, which

of the

following statements is FALSE?

A. B. C. D.

Persons suffering from CEP should exhibit a


decreased blood viscosity.

Persons suffering from CEP exhibit a retarded oxygen transport system. Persons suffering from CEP exhibit a lowered arterial pO2.

Persons suffering from CEP should exhibit a decreased resistance to blood flow.

29. A

man heterozygous for CEP has a child with a woman who is homozygous dominant for CEP. What is the probability that their first son will show
the disease phenotype?

A. B. C. D.

jVo
25Vo 50Vo
IOOVo

Copyright @ by The Berkeley Review

140

The Berkeley Specializing in MCAT

Biology
Passage

Enryme Kinetics I

Practice P2ssage VI

VI (Questions 30-36)

Enzymes can also undergo one

of three types of

In

1913 Leonor Michaelis and Maude Menten

proposed that reactions catalyzed by enzymes proceed in two steps. First, an enzyme (E) rapidly binds a substrate (S) to form an enzyme-substrate complex (ES). Second, the enzyme-substrate complex is converted to product (P)

in a rate-determining step. The rates of enzymatically catalyzed reactions, in comparison to uncatalyzed


reactions, can easily exceed a factor of a million. Since an enzyme is a catalyst, it does not alter the equilibrium of a chemical reaction.

reversible inhibition. Competitive inhibitors resemble the substrate and bind to the active site of the enzyme, thus preventing the substrate from binding to that active site. The initial tendency of lowering the velocity at which the enzyme converts substrate to product can be overcome at high substrate concentrations. Noncompetitive inhibitors can bind to either the free enzyme or the enzymesubstrate complex. This action also lowers the rate at which substrate is converted to product. Uncompetitive inhibitors bind only to the enzyme-substrate complex, thus lowering the rate at which substrate is converted to

FrS-FS>ErP Lrvk2

k1

k1

product. Unlike competitive inhibition, uncompetitive inhibition and noncompetitive inhibition cannot be
overcome at high substrate concentrations.

Almost all known enzymes are proteins, and all of the amino acid residues found in proteins are L-amino acids

rr (S)-amino acids. The exception is that L-cysteine is designated as (R)-cysteine. The actions of enzymes can be :egulated, and they offer a high degree of specificity and

30.

Trypsin is a digestive enzyme generally referred to


as a peptidase, because it hydrolyzes polypeptides. The scissile peptide bond to be cleaved in the polypeptide is on the carboxyl side of the amino acid residues arginine (Arg) and lysine (Lys).

:rample, enzymes readily degrade polypeptides


,r'nthesized from L-amino acids, but not from D-amino

:atalysis towards a substrate or reactant molecule. For

.:ids.
When a substrate binds to the active site of an enzyme, . usually does so by non-covalent interactions. Typical :-nding forces involve hydrogen bonding, hydrophobic rteractions, electrostatic interactions, and van der Waals :teractions. The active sites of enzymes generally form :.efts or crevices in the molecule. Those crevices are lined ,i rth specific amino acid residues that help to stabilize the ':rnsition state of the substrate and allow catalysis to ::cur. Even though different enzymes can have ,:ecificities for different side chains next to the peptide :,:nd to be cleaved, they can have similar active sites.

Which of the following peptide bonds is cleaved by


trypsin? ^HOH c)rrtOO f c- N- c- coo fr, H cH3 i*' Alanine (Ala)
CH"

nrN-

H O H^ tl19 ooc- c- N - c*",

H:c

?I

"

t",
CHz

:', l'
@ NH3

II

CH' LHr

1..'

t-

@ NH3

Lysine (Lys)

.-

Enzymes like chymotrypsin, trypsin, elastase, and rtilisin are referred to as serine proteases, because they -.','e a highly reactive serine residue at their active site. ..:e presence of active site serine residues can be
--.sopropylphosphofluoridate (DIPF)
as a

-::ermined by using an irreversible inhibitor like


chemical label.

CH,OH HOCH. @rrr l' a' ir ro o o ooc- c- N - c- c- NHj n.NcN- c- coo ttl ? HHCHU ?t,HH
I

cH"

(H1C)2HC-

i O-

NH

?t'
I

t'

cHz
I

III

IV

f"'
NH

io

O-

CH(CHt2

HzN

,caa

NHz

Oc - ta-\ HuN

NHz

DIPF

Arginine (Arg)

:orming a diisopropylphosphoryl complex with the ,-:',me. This irreversible inhibitor has a similar action on ":..'.'lcholinesterase, a serine esterase involved in the
::olysis of the neurotransmitter acetylcholine to acetate
-.:

'

DIPF reacts only with serine residues at the active site

A.
B. C. D.

I only

II only I and III IV only

: ;holine.
:r right O by The Berkeley Review

!t
0n

t4t

The Berkeley Review Specializing in MCAT Preparation

Biology
31.
serine proteases depends upon:

Enzyme Kinetics I
34.

Practice Passage VI

The action of the nerve gas diisopropylphospho-

fluoridate (DIPF) on acetylcholinesterase and

A. B. C. D.

allostericinhibition. competitiveinhibition.
phosphorylation of ADP. phosphorylation of the active site.

residues, aspartate 52 and glutamate 35, at the catalytic site with pK6 values of approximately 4.0 and 6.0, respectively. Ifthese are the only ionizable groups involved in catalysis, then the velocity of the reaction varies with pH according to which
graph?

The lysozyme enzyme has two amino

acid

32. In the following graph, if

reaction velocity (V) as a function of substrate

Curve

reprsents the

o o o o

concentration (S) plot for an enzyme, then Curve II represents the reaction of the same substrate in the presence of a fixed concentration of:

&

V-"*
o o o
o

&

C)

A.
B. C. D.

an irreversible inhibitor. a competitive inhibitor. an uncompetitive inhibitor. a noncompetitive inhibitor.

o
0)

33.

Of the four

compounds tested at the same concentration, the MOST effective competitive

&

C)

inhibitor of the common enzyme is:

il
o
o

&

A. B. C. D.

Compound I. Compound II. Compound III. Compound IV.

A. B.
C.

D.

Graph I. Graph II. Graph III. Graph IV.

Copyright @ by The Berkeley Review

142

The Berkeley Specializing in MCAT

Biology

Enzyme Kinetics I

Practice Passage VI

35. At a pH of about 5, which of the amino acid


residues in the active site of the lysozyme enzyme provides the general base catalysis and which

provides the general acid catalysis?

A.

B.

C.

Aspartate 52 provides the general acid catalysis, because its side chain p-carboxyl group is about 9lVo deprotonated at a pH of 5. Glutamate 35 provides the general base catalysis, because its side chain y-carboxyl group is abott9l7o protonated at apH of5. Glutamate 35 provides the general acid catalysis, because its side chain 1-carboxyl group is about9Vo deprotonated at a pH of 5. Aspartate 52 provides the general base catalysis, because its side chain B-carboxyl group is about9Vo protonated at apH of5. Aspartate 52 provides the general base catalysis, because its side chain B-carboxyl group is about9lVo protonated at a pH of 5. Glutamate 35 provides the general acid catalysis, because its side chain y-carboxyl
group is about 9Vo deprotonated at
a

pH of 5.

D.

Glutamate 35 provides the general base


catalysis, because its side chain y-carboxyl group is about9Vo deprotonated at a pH of 5. Aspartate 52 provides the general acid catalysis, because its side chain p-carboxyl group is about9lVo deprotonated at apH of5.

According to the following graph, the Vmax of the


enzyme reaction is:

A. 3tz 8.2 D.

c.

rtz

2tz

I ";r'nr,,:tght @

by The Berkeley Review

146

The Berkeley Keview Specializing in MCAT Preparation

Biology
Passage

Vitamin Bl z
37.

Passage Vtr

VII

(Questions 37-43)

Based on the reaction shown in Figure 1, what type of enzyme is methylmalonyl-CoA mutase?

Vitamin Brz is a large cobalt-containing molecule that has a ring structure similar to heme, with Co as the central

ion instead of Fe. Vitamin Brz participates in only two known reactions in the human body, catalyzed by the
enzymes methylmalonyl-CoA mutase and homocysteine
methyltransferase.

A. B. C. D.

Oxidoreductase

Hydrolase
Transferase Isomerase

38.
coo
I I

Methylmalonyl-CoA
mutase

coo
I

H-C_CH.

CH"
I

o=c-s-coA
Methylmalonyl CoA

'rC-S-CoA
Succinyl CoA Figure
1

.-l

CH"

In the enzyme methionine synthetase, both Brz and tetrahydrofolate (THF) are cofactors. If B rz is low, then THF is trapped as methyl-THF at this enzyme and is not recycled. This is called the methyl-TllF trap. How can 812 status affect folate status?
A.
B.

A primary B 12 deficiency can lead A primary folate


secondary folate deficiency.

deficiency can lead

C. D.

secondary B12 deficiency. Folate and Bn are independent of each other. B 12 is required to synthesize folate.

to H-p-NHj
I

coo
CH"
I

Homocysteine methyltransferase

t@ H_C_NH,
I

coo
CH.
I

39.

The production of IF often decreases with age. To


prevent B12 deficiency in older adults, which of following would be the BEST treatrnent?

CH.
I

cH"
I

tb

SH

S
I

CH:

Figure 2

I. II. ilI. A. B. C. D.

Intramuscular injection of Brz Increase dietary Bt2 Increase dietary Co

I only

Btz is synthesized by anaerobic microorganisms. In the diet, it is found in animal products, particularly in the meat and milk of ruminants, because their first stomachs
contain microorganisms that synthesize Btz for the animal. Complete vegetarians need to include either a
microbiological or a synthetic source of Btz in their diets. B12 is absorbed in the ileum of the small intestine. Intrinsic factor (IF), a glycoprotein, is produced in the stomach. In the intestinal lumen, IF binds cobalamin, and the complex is taken up by receptors in the ileum of the
small intestine.
40.

II only I and II only I and III only

Bacteria in the human colon produce B tz. How this production affect Brz status in the individual?

A. Colonic Brz is readily absorbed B.


vegetarians.

Colonic Btz is not absorbed, because is


distal to the ileal receptor.

A deficiency of IF lbads to Brz deficiency, even if the diet is rich in B12. Signs of a Btz deficiency include: increased urinary methyl malonic acid, neurological damage due to abnormal membrane lipids, and
macrocytic anemia.

C. Colonic B 12 decreases production of IF. D. Colonic Bt2 decreases activity of


malonyl CoA mutase.

Copyright

by The Berkeley Review

144

The Berkeley Specializing in MCAT

Biology
{1.

Vitamin 812

Passage Vtr

Why is increased methyl malonic acid (MMA) in the urine a sign of B12 deficiency?
coo

H- C-

CH3

o=c-oH
A.

The blocked metabolic pathway to succinyl CoA pushes the reactants into the MMA
pathway.

B.

Brz is a competitive inhibitor of the MMA


pathway.

c.
D.

MMA is a by-product of the methionine


synthetase reaction.

MMA results from breakdown of surplus IF.

{2" Which of the following


methionine (Met) is FALSE?

statements regarding

A. B. C. D.

Met is an essential amino acid. Met has a sulfur-containing side chain. The carbon skeleton of Met can be used in
gluconeogenesis.

Met is the precursor of Tyr, a nonessential


amino acid.

.{"1. Why is IF resistant to digestion in the stomach?

A. Proteins are not digested by gastric enzymes. B. IF is protected by nondigestible surface C. D.


carbohydrates.

IF inhibits the activation of pepsin. IF is not resistant to digestion in the stomach.

l"rpyright

by The Berkeley Review

145

The Berkeley Review Specializing in MCAT Preparation

Biology
Passage

Enz4yme Nomenclature

Passage VIII

VtrI

(Questions 44-50)

Enzymes are catalysts that are quite specific in their choice of reactants (substrates) and in the reactions they catalyze. They are able to lower the energy barrier to a

The substrate-binding site of a protein enzyme is called the active site. Lining the active site are specific amino acid side chains, which attract the substrate and enhance catalysis. If the pH of the catalytic environment
is optimal, the rate of reaction tends to be maximized. The

reaction, thereby increasing the rate (velocity) of the reaction in either the forward or the reverse direction. Catalysts have no effect on the position of equilibrium of
the reaction.

optimal operating pH of an enzyme therefore depends on the pKa values of the amino acid side chains (Table 2) at
the active site.

There are thousands of different types of enzymes that

Table 2. pKu Values of Select Ionizable Groups

involved in the catalytic process. The suffix -ase is attached to the common name to indicate an enzyme.
Urease, for example, increases the rate ofurea hydrolysis.

catalyze a wide variety of reactions. Many of these enzymes have common names that are descriptive of their functions and they are derived from the principal reactant

o
H2N- C- NH2
Urea

lt

H2O

Urease-

NH3 + CO'

Some enzymes have names that do not end in -ase. Pepsin and trypsin, for example, are proteolytic enzymes.

Acid Arginine Aspartic acid Cysteine Glutamic acid Histidine Lysine Tyrosine
a-Amino

cl-cooH cr.NH3+ Side chain

p&r 1.8 2.0 1.8 2.2 1.8 2.2 2.2

p&z 9.0 10.0 10.8 9.5 9.2 9.2 9.I

plq3
12.5

3.9
8.3 4.1

6.0
10.8
10.1

In order to

assign a name unambiguously

to an
44. Using the expression AG'' = -2.3 RT log determine the change in the free energy for
if the concentration values the reactants and products at equilibrium is
reaction shown below,
moles.

enzyme, the Enzyme Commission (EC) of IUPAC places each enzyme into one of six enzyme classes (Table 1), based on the type of reaction they catalyza Within each of these six classes are subclasses and sub-subclasses. Each enzyme is assigned a specific, four-integer EC number, along with a common name and a systematic
name.

A+B-i.-C

Table 1. Enzyme Classification


Enzyme

Class

A.
B.

Type of Reaction Catalyzed Oxidation-reduction Transfer offunctional groups

Oxidoreductase Transferase .

c.
D. 45.

2.3 + 2.3 - 4.6 + 4.6

RT RT RT RT

Hydrolase Lyase Isomerase Ligase


Enzymes can

Hydrolytic cleavage Addition of groups to double


bonds or the reverse

In the Krebs cycle, fumarate is converted


by the enzyme fumarase.

Transfer of groups within molecules to give isomeric forms Bond formation coupled with ATP hydrolysis

coo
I

o
+ HnO

coo
I
I

CH

ll

HO- C- H
CH'

CH

lg coo
Fumarate

coo
Malate
a:

l-6

eiist in alternate forms

called

This enzyme is

isoenzymes. Lactate dehydrogenase (LDH), for example, has two different types of subunits designated as M and

H. LDH with M subunits is found primarily in muscle tissue, while LDH with H subunits is found primarily in
heart tissue.

A. B. C. D.
r46,

lyase involved in a hydration reaction. hydrolase involved in a hydrolysis reactioalyase involved in a hydrolysis reaction. hydrolase involved in a hydration reaction-

Copyright

by The Berkeley Review

The Berkeley Specializing in MCAT Prel

Biology
hexokinase.

Enryme Nomenclature
49. The active

Passage VItr

16. The first step in glycolysis is the conversion of glucose to glucose-6-phosphate by the enzyme
o o t/ .>l 1 p- oo/ P- oo
It

sites of different enzymes can contain unique catalytic groups. The properties of these generally show bell-shaped curves.

groups are pH-sensitive, and the initial rates


Vrnu*
() {)
d

o
AMP

lt

Ol

o
I

o
H

o-P=o
o

Hrc

)' ;b:

*s._(t
H,C

J-o

os

ADP
Glucose

OH

8 9 10 ll1213
pH

Glucose-6
phosphate

Hexokinase is
called:

member

of a class of

enzymes

represents the catalytic group


above?

Which of the following amino acid pairs BEST for the curve shown

A. B. C. D. .{-.

ligases. hydrolases. transferases. oxidoreductases.

A. B. C. D.

Histidine and aspartic acid Cysteine and tyrosine Arginine and lysine Lysine and cysteine

Hexokinase also has the ability to transfer the 1phosphoryl group of ATP to water, but at a rate which is about 40,000 times slower than its transfer to the C-6 hydroxyl of glucose, even though the nucleophilic properties of the C-6 hydroxyl group and water are similar. The large difference in rate can be explained by all of the following EXCEPT:

50.

The isoenzymes of LDH can be resolved by using electrophoresis. In the polyacrylamide gel shown

below, Lane 1 represents one type


Lane

of

LDH

isoenzyme, while Lane 3 represents another type:

1 Lane2 Lane 3
(+)

A. B. C.

the introduction of a large conformational


change in the enzyme by glucose.

the reduction of active site polarity by the


exclusion of water. the presence of water in the active site at the position occupied by the C-6 hydroxyl group.

D. the

hydroxyl of glucose and the increased electrophilicity of the y-phosphoryl group of

increased nucleophilicity

of the C-6

Ij:@

Origin
C)

ATP.

Equal amounts of the two isoenzymes are mixed


together. The subunits are next dissociated from one another and then allowed to reassociate randomly as indicated in Lane 2. This indicates that LDH is a:

*ri,

The second step of the glycolytic pathway involves

:he conversion of an aldose sugar into a ketose sugar. Enzymes that catalyze this type of reaction
:re called:

\. B. C. D.
: ':,ght

transferases. isomerases. hydrolases. ligases.

A. B. C. D.

dimer.

trimer.
tetramer. pentamer.

by The Berkeley Review

147

The BerkeleY Review Specializing in MCAT Preparation

Biology
Passage

Lysozyms Flssftanisrn
Main Chain of Lysozyme Enzyme

Passage

IX (Questions 51-58)

The cell wall polysaccharide of bacteria is composed of two types of sugars, N-acetylmuramate (NAM) and Nacetylglucosamine (NAG). These sugars are joined to
each other by a glycosidic linkage.

Cfu:S

o'rc-oo
Ring F

Lysozyme is composed of 129 amino acids, linked together through peptide bonds to form a protein that displays cr-helical and p-sheet regions in its tertiary structure (Figure 1). This enzyme, which is found in human saliva and lacrimal fluid, and in hen egg whites, is an antimicrobial substance that aids in the hydrolysis of a
specific substrate bearing a NAM-NAG sugar linkage.

NAM N-H lal

o=f oi^._o o=F


CH: f
I

NAG N-H
c",

Asp 52

Active Site

Main Chain of Lysozyme Enzyme

Figure

Main Chain of Lysozyme Enzyme

Cru:S

o',t-oo
psheet

v'

Lysozyme Figure I
Rings A-B-C

The proposed mechanism for the catalytic hydrolysis of bacterial cell walls by lysozyme is outlined in Figures 2-5. ln this mechanism, a portion of the substrate containing six glycosidic sugar residues fits into the active site of the enzyme at positions labeled A, B, C, D, E, and F. Two catalytic groups, glutamic acid 35 (Glu 35) and aspartic acid 52 (Asp 52), reside close to and on opposite sides of the glycosidic linkage to be cleaved. The neighboring environments of Glu 35 and Asp 52 are quite different from each other.
Main Chain of Lysozyme Enzyme

NAM N-H
u

^-l -L

cHr i^._o Asp 52


I
I

o_

Main Chain of Lysozyme Enzyme

Figure 4

Main Chain of Lysozyme Enzyme

Cru:S

cru
I

rs

ntrt'no
Rings A-B-C

t
cH20H

o',C- o o

o
Ring F Rings A-B-C

NAM N-H lal


o=

NAG of

N_H

NAM N-H
u

F ^,-o cHr I
Figure 2

o=f
c",

^-l -L

CH, I
I

o or ^-_o

I t' Aso 52

Asp 52

Main Chain of Lysozyme Enzyme

Main Chain of Lysozyme Enzyme

Figure 5

Copyright

by The Berkeley Review

t4a

The Berkeley Specializing in MCAT Prepar

Biology
51.
I.
Gram-positive. Gram-negative. Missing their cell walls.

Lysoryme Mechanism
55.

Passage IX

Lysozyme works BEST on bacteria that are:

II.

III.
A.
B. C. D.

The pKa values for the side chain carboxyl groups on aspartic acid and glutamic acid are usually cited as 3.9 and 4.1, respectively. Analysis of lysozyme's active site indicates that the pKa of Asp 52 is still about 3.9, but the pKn of Glu 35 is now about 6.6. Glu 35 shows a 2.5 fold increase in pKa, because:

I only

II only III only II and III only

A.

the carboxyl group of Asp 52 is located in


polar environment.

B. the ionized carboxyl group of Asp C. D.

52

destabilizes the protonated carboxyl group of

Glu 35. the carboxyl group of Glu 35 is located in


nonpolar environment.

<)

The configuration of the linkage between rings D


and E in Figure 2 is:

the carboxyl group of Glu 35 is located in a polar environment, where it is stabilized by


hydrogen bonding.

A. B. C. D.

cr(l-+4).

p(a-+l).
cx(4-+1).

56. In

p(1+4).

order to establish clearly which bond is being cleaved by lysozyme, enzymatic hydrolysis using water labeled with 18O is used. This heavy isotope of oxygen is found attached to sugar residue D at
the:

53.

Which of the following general classes of enzymes BEST represents lysozyme?

A. B. C. D.
57.

reference carbon. anomeric carbon. C-5 carbon. C-6 carbon.

A. B. C. D.

Hydrolase
Transferase

According to the proposed mechanism outlined in the passage, lysozyme is involved in acid catalysis.
This mechanism involves:

Oxidoreductase Ligase

A. B. C. D.

noncovalent catalysis coupled with heterolytic


bond cleavage.

covalent catalysis coupled with heterolytic


bond cleavage.

noncovalent catalysis coupled with homolytic


bond cleavage.

51. Amino acid charge distribution


an enzyme is arranged to:

at the active site of

covalent catalysis coupled with homolytic


bond cleavage.

A. C. D.

destabilize the transition state


increase the rate of catalYsis.
decrease the rate of catalysis.

in order to

58.

B. stabilize the transition state in order to


destabilize the transition state

Based on the representation of lysozyme shown in Figure 1, glutamic acid would be expected to be found in the:

in order to

I. active site.

decrease the rate of catalYsis. stabilize the transition state in order to increase the rate of catalYsis'

II. IIr.
A.
B. C.

cx,-helical regions.

interior of the protein.

I only

D.

II only I and II only I and III only The BerkeleY Keview Specializing in MCAT PreParation

l:n:"rright

by The Berkeley Review

149

Biology
Passage X (Questions 59-65)

Dnzyme Kinetics II

Passage

Enzyme activity depends not only on temperature and pH, but also on the available concentrations of substrates [S] and inhibitors [I]. As [S] in an enzyme-catalyzed reaction increases, so does the rate of catalysis and the

The turnover number of an enzyme is the number d substrate molecules processed per enzyme per secooil when the enzyme is completely saturated with substrar-

This value is equal to k3 and is sometimes called ftc

catalytic constant, k.ut.


kcat/Ku. Table
1

measure

formation
(Figure 1).

concentration [E], the relationship between the initial reaction rate (velocity v) and [S] describes a hyperbola

of

products P.

At a

constant enzyme

catalytic efficiency can be obtained from the ratio


lists some values for kcat and Ku.

of an enzymeh d

Table 1. Enzyme and Substrate Parameters


Enzyme
(and

substrate) KM

V-u"

Acetylcholine
(Acetylcholine)

(M) k.", (s-1) esterase 9.5 x 10 -5 1.4 x 10 a 2.6xlo-2


6.6x10-4
5.ox1o-6
3.0

Carbonic anhydrase
(Bicarbonate)
O
0)

4.ox1os
1.9x102

V-u*
2

Chymotrypsin
(N-Acetyltyrosine ethyl ester)

Fumarase
(Fumarate)

8.0x102
5.0 x 10 -l

Pepsin
(Phe-Gly)

x 1o

-a

Ribonuclease

'7.9'x

Km
Figure I

(Cytidine 2',3' cy clic phosphate)

rc'3

i.9 x lo2
1.ox1oa

tsl

Urease
(Urea)

2.5x10-2

When an enzyme reacts with a substrate, it forms the enzyme-substrate complex ES, which then breaks down to reform the enzyme and release the product. This reaction

is governed by individual rate constants kl, kz, and kt. Rate constant kr has limits between 108 to 199 14-1r-1.

E+S

ES

k3 .E +p

59.

When the substrate concentration is very much lower than the Michaelis constant, the MichaeUsMenten equation reduces to:

The relationship between the initial reaction velocity and [S] was examined during the early 1900's by Leonor Michaelis and Maude Menten. They concluded that the initial reaction velocity v can be given by:

A. ,r-V**. Ky vt* B. ,n ' - Ky + [S]' ^


(-.

(r)

u=

J'aduK* + [S]

,. v

where [S] is the initial substrate concentration, Vmax is the

--,

V*u* [S]
Krra

maximal velocity of the reaction under saturating [S], and Kv is that [S] where the reaction operates at one-half its maximal velocity. The lower the value of Ku, the higher the affinity the enzyme has for its substrate. Ku is called the Michaelis constant and Equation (1) is called the
M ichaelis - M enten e quation.

D.

"=+P
The Berkeley Keviev Specializing in MCAT Preparation

Copyright @ by The Berkeley Review

150

Biotogy
Menten equation reduces to:

Enz5rme Kinetics

II

Passage X

6lD. When the substrate concentration is very much higher than the Michaelis constant, the Michaelis-

63. By

taking the reciprocal of the Michaelis-Menten

{. r, ' = Ky[S] + [S]' B. v = Vmax.

equation, a straight line can be graphed on a double reciprocal plot. This graph (shown below) more accurately represents the parameters of the curve in Figure L

c'

u=

rst

isr'
V-u* [S]

D.'v=-.

tsl

-4 -3 -2 -1 0
The

2 3 4
lrtsl

6,1.

When the substrate concentration is equal to the Michaelis constant, the Michaelis-Menten equation
reduces to:

Ku of the enzyme with

these kinetic data is:

A. ,, - vtu* 2Krtr

B. u-2V-u*. C. D. u-Vtu*. 2
ttrL

A. + 0.5. B. - 2.0. C. +2.0. D. + 5.0.

Ky V = 2V-r*.

64. Which of the following pairs of enzymes


achieved a state of virtual catalytic perfection?

have

The relationship between IEl, tsl, [ES], and [P] can be expressed graphically in terms of steady-state

enzyme kinetics. Which

of the following
B.

graphs

A. B. C. D.

Carbonic anhydrase and chymotrypsin Acetylcholine esterase and carbonic anhydrase Fumarase and acetylcholine esterase Chymotrypsin and pepsin

BEST represents this relationship?

A.

65.
o

The enzyme triose phosphate isomerase catalyzes

C)

o o

U
Time C. o
c6

of glyceraldehyde-3-phosphate into dihydroxyacetone phosphate in both yeasts and humans. This enzyme has a reported efficiency of 2.4 x log M-1s-1, which suggests that it:
the conversion
Time

A. B. C. D.

D.

evolved to near maximum efficiency early in its evolutionary history and has changed very little since then.
evolved to near maximum efficiency late in its evolutionary history and has changed very

H
C)

little since then. did not evolve to near maximum efficiency during its evolutionary history, because it is
found in distinctly different species.

U
Time

U
Time

did not evolve to near maximum efficiency during its evolutionary history, because of its
inability to change.

l:nlright

by The Berkeley Review

r5l

The BerkeleY Keview Specializing in MCAT Preparation

Biology
Passage

Adenosine Triphosphate (ATp)


67.
in Figure l,

Passage XI

XI (Questions 66-72)

Which of the following cellular transport processes


requires ATP?

Adenosine triphosphate (ATp), shown

higher-energy phosphate compounds and the lowerenergy phosphate compounds, Other energy-rich nucleoside triphosphates that function in this capacity are guanosine triphosphare (GTP), cytidine triphosphate
(CTP), and uridine triphosphate (UTp).

serves as the primary energy intermediary between the

A. B. C. D.
68.

Osmosis

Diffusion Active transport


Passive transport

Table I indicates ATP's energetically central position. The hydrolysis of ATP to ADP (adenosine diphosphate) and Pi (inorganic phosphate), or to AMP (adenosine

in the cells of an organism, what happens to that


organism?

When a compound is used to inhibit ATp production

A. It B.
C.

monophosphate) and PPi (pyrophosphate), provides

continues regular metabolic processes without change by switching ro a higher-

muscle contraction and transport


concentration

energy for many endergonic reactions in the body, such as

energy phosphate compound.

gradients.

of ions

against

It continues regular metabolic processes using


production.

o"" oo .,o ort o-P-o-P-o-T-o-r",


HO Figure
1

r | !nffs
^
OH

/i f (o{.o/

i*

It is unaffected by the cessation of ATp


and

GTP.

D.

It slows all metabolic activity


dies.

69. Which compound would release the MOST energy after the phosphorylation of ADp?

A. B. C. D.

Phosphoenolpyruvate 1,3-bisphosphoglycerare Acetyl phosphate Phosphocreatine

Compound
Phosphoenolpyruvate
1,3-Bisphosphoglycerate Acetyl phosphate Phosphocreatine PPi -+ 2 Pt ATP -+ AMP + PP1

AG" (kJ/mol)
- 61.9 - 49.4 - 43.1 - 43.1
- 33.5 - 32.2 - 30.5 - 20.9

70. Which of the following metabolic processes NOT generate a net yield of ATp or GTp?

A. B. C. D.
71.

Glycolysis

Citric acid cycle Protein synthesis Oxidativephosphorylation

AIP -+ ADP + P1
Glucose- 1,-phosphate Fructose-6-phosphate Glucose-6-phosphate

What type of chemical bond is indicated by


arrow in Figure l?

- 13.8 - 13.8
- 9.2

Glycerol-3-phosphate

Table 1. Standard free energies of phosphate hydrolysis of some compounds.

A. B. C. D.
72.

Phosphoanhydridebond Phosphoesterbond O-Glycosidic bond


Peptide bond

When ATP is hydrolyzed to AMp and ppi, which the following statements must be TRUE?

66. Based on the data in Table 1, which

of the following compounds can be phosphorylated exergonically by ATP?

I. II.

A. B. C. D.

Phosphoenolpyruvate
Glucose-1-phosphate Acetyl phosphate Pyrophosphate

IfI. A. I only B. II only C. II and trI only D. I,II. and III


1|52

is very stable due to electrostatic interactions between its phosphate residuesMore energy is released than in the hyd of ATP to ADP. PPi is insoluble in the cytosol of the cell.
PP1

Copyright @ by The Berkeley Review

Biology
Passage

Protein Degradation (Ubiquitin)


73.

Passage XII

XII (Questions 73-79)

Why do lysosomal enzymes have pH optima close to


5?

.bout fifty hydrolytic enzymes, including several types of ;roteases. The lysosomal enzymes have pH opiima of .rout 5. Lysosomes fuse with vacuoles inside the cell and
: rmponents and

:roteolysis. These membrane-bound organelles contain

:onselective means. Lysosomes are used ior nonselective

Proteins are degraded inside cells by selective and

A.
B.

.'''drolyze the contents, thereby recycling intracellular


digesting foreign particles.

C. D.

So that they will be active when released to the cytoplasm So that they will be inactive when released ro the cytoplasm To degrade basic proteins more efTiciently To degrade acidic proteins more efficientiy

,,'ry selective. The molecule ubiquitin, a 76_amino acid .-.ot"rl,- _binds to lysine residues on condemned proteins.
-

Conversely, the ATP-dependent proteolytic system is

te DNA
An

sequence coding

rnserved throughout eukaryotes.

for ubiquitin is

trignty
74.

.:gs and destroys the labeled protein. The ubiquitin )tem is used to destroy abnormal proteins and short ,'.ed enzymes that are at control points in metabolic :.thways. As shown in Table 1, the N-terminal residue of --e protein determines how quickly it is ubiquitinated and
:'.

_enzyme system called the ubiquitin_conjugate ::grading enzyme (UCDEN) recogniies ttre ubiqiitin

[!1t is meant by saying rhar a DNA sequence is highly conserved in eukaryotes?


A.
B.

T!"

sequence varies

eukaryotes studied. The sequence differs


eukaryotes studied.

widely among all

only among phyla of

:;stroyed.

C.
D.

eukaryotes studied. The sequence is completely identical eukaryotes studied.

The sequence is virtually identical in all

in

all

Cytochrome c
.{ protein in the eleclron-ranspon chain

50 hours

Clyceraldehyde-3-phosphate
Jehydrogenase (GAPDH)
A protein required for glycolysis

130 hours

.\ldolase

I 1 8 hours

t5.

protein required for glycolysis

\-Terminal residue (stabilizing)


\Iethionine (Met), Serine (Ser), \lanine (Ala), Threonine (Thr),
> 20 hours

be modified quickly by ubiquitin?

Based on information given in the passage and in Table 1 about its role at metabolic iontrot points, which.of the following proteins is MOST likely to Cytochrome c Tyrosineaminotransferase Ornithinedecarboxylase
Glyceraldehyde-3-phosphate dehydrogenase

\aline (Val), Glycine (Gly)

shof$ill+ i6,i ffil#iiiir


T1'rosine aminotransferase .{ protein required for amino acid catoblism R\A Polymerase I
.{ eukaryotic protein that synthesizes rRNA

120 minutes 78 minutes


12 minutes

A. B. C. D.

Cmithine decarboxylase
.\ protein required for polyamine synthesis

\-Terminal residue (destabilizing)


I.oleucine (Ile), Glutamic acid (Glu) Tvrosine (Tyr), Gluramine (Gln) ?henylalanine (Phe), Leucine (Leu)
= 30 minutes = 10 minutes

76. Of the following

= 3 minutes = 2 minutes

acid provides the MOST stability for an enzyme?

choices, which N-terminal amino

\spartic acid (Asp), Lysine (Lys) erginine (Arg)

Table 1. Half-lives of long-lived and short-lived cellular :lzymes as a function of their (potential) N-terminal residues.

A. B. C. D.

Lysine Glycine
Glutamate

Leucine

- .,,-vright @ by The Berkeley Review

153

The Berkeley Review Specializing in MCAT preparation

Biology

Protein Degradation (Ubiquitin)


would

Passage Xtr

3
Prs

77. Under what physiological condition


lysosomal protein degradation be the highest?

79,

The antimalarial drug chloroquine, pictured below,


penetrates the lysosome and accumulates as a weak

A. B. C. D.

During a fast Following a meal During exercise During pregnancy

base inside. What effect does this have on the lysosomal degradation of proteins?

,*fu
frunl
&rE
fr"r*1.'

ha['lr

il78.
The following diagram indicates the steps involved in the attachment of ubiquitin to a protein:

a,r

- 1cH2)r-

.l
N(c2Hs)2

cHr

f llifti ltofu

Chloroquine

'lfl'mrfilMt

dfiMffi

A.
B.

The rate ofprotein degradation is decreased. The rate ofprotein degradation is increased.

firuffit

'@mfueil;

c.
ubiquitin-

Chloroquine inhibits the proteases by


modifying their active sites.

'mitEm

'llrdhT

3-

oO

+ BI-SH

D.

Chloroquine activates the proteases


modifying their active sites.

try

o" , llr ll\ aue* ee' v


o
il

Ubiquitin-

C- S-

Er

10fi

,flcp
Ubiquitinn

lrq m
E2

C- S-

il

l16

ConOemned protein (CP)

u 'o

ll\

ez-sn
il

Ubiquitin- C-

N-

Lys-

CP

What is the purpose of the reaction involving ATP in Step 1?

A. The hydrolysis of ATP to AMP and the


subsequent hydrolysis

of

PP1 provides the

B.

energy ofthree high-energy phosphate bonds. The hydrolysis of ATP to AMP provides the energy of two high-energy phosphate bonds.

C. The hydrolysis of ATP to AMP and the


subsequent hydrolysis

of

PP1 provides the

D.

energy of two high-energy phosphate bonds. The hydrolysis of ATP to AMP provides the energy of three high-energy phosphate bonds.

Copyright

by The Berkeley Review

154

The Berkeley Specializing in MCAT

Biology
Fassage

Nutrients and Proximate Analysis


80.

Passage XItr

XIII

(Questions 80-87)

The dehydration step of proximate analysis is often

Nutrients are grouped into six classes: protein, :..:bohydrate (starches and sugars), and lipid, all of which
::ovide energy; and water, vitamins, and minerals, which .:: noncaloric. Protein and carbohydrate each provide 4 , - al/gram, and triglyceride (a type of lipid) provides 9
i:aVgram. Average daily water intake for an adult is about 1.5 to iiters from liquids, foods, and metabolic water. -.-tamins are divided into water-soluble and fat-soluble :umins. The water-soluble vitamins are thiamin (Br),

by lyophilization (freeze-drying). The sample

is

frozen, and the water is evaporated from the frozen sample in a vacuum. What is this process called?

A. B. C. D.
81.

Melting
Vaporization
Condensation

Sublimation

-::oflavin (82), niacin (B3), pyridoxine (Bo), folate, ,.:alamine (Btz), folate, biotin, pantothenic acid and -,:orbic acid (vitamin C). The fat-soluble vitamins are "amin A (retinol), vitamin E (alpha-tocopherol), vitamin - dihydroxy-cholecalciferol), and vitamin K. Minerals
-

When vitamins and minerals are combusted, which compounds produce the gas phase and which
produce the ash?

:;;ude the major or macrominerals: calcium, phosphorus, -lorine, potassium, sulfur, and magnesium; and the trace :,nerals: iron, zinc, iodine, copper, manganese, fluoride, -::omium, selenium, molybdenum, cobalt, vanadium, tin,
,r--on, and nickel.

A. B. C. D.

Both are in the gas phase. Both are in the ash. Vitamins are in the ash, while minerals are in
the gas phase. Vitamins are in the gas phase, while minerals are in the ash.

Vitamins and minerals function in specific ways in Some are important as cofactors or coenzymes, ;'..rle others maintain acid-base balance, promote nerve -- j muscle activity, or maintain fluid and electrolyte -.-.:nce. Vitamins are organic molecules, while minerals *: inorganic ions and atoms. Many of the B vitamins

.:: body.

82.

Which of the following is NOT a trace mineral?

, :;tion as cofactors

::-abolism, such

in the reactions of

energy
as

A. B. C. D.
83. A

Magnesium

Zinc
Manganese

Iodine

as niacin

in NAD. Vitamin C works

in a reduced form in some r::i'mes. The fat-soluble vitamins are also important. .:rmin A is crucial for vision. Vitamin D helps regulate -.:lum homeostasis. Vitamin K is required for blood
-

"": :ntioxidant to keep metals

-ino

portion of chicken is analyzed by the Kjeldhal technique and found to contain 5 grams of nitrogen. How many grams of protein are in it?

The nutrient content of food can be determined by rimate analysis, whish involves a series of chemical ,::..i ses. First, the sample is dehydrated, and the mass of " .::r is calculated by difference. Next, a lipid extraction
,

A. B. C. D.

31 grams protein 0.8 grams protein 80 grams protein 3.1 grams protein

' : organic solvent is performed to calculate the mass of " :. A Kjeldhal analysis determines nitrogen content. '.: mass of protein is determined based on the i :-:nption that a protein contains 16%o nitrogen Fiber is :, :esidue unaffected by acid and base hydrolysis.
:, -idng

84. If you were to analyze a new food,


content?

which

components would you test to determine the calorie

in a gas phase and an ash. Finally, carbohydrate :l.culated as the total weight minus all other nutrients

"nins and minerals are

separated

by

combustion,

,::

: :lber.

I. il. ilI. A. B. C. D.

Carbohydrates

Vitamins

Lipids

I and III only II only II and III only I, II, and III

:''right

by The Berkeley Review

155

The Berkeley Review Specializing in MCAT Prepmation

Biology
85.

Nutrients and Proximate AnalYsis

Passage )fltr

!
L
rf
@

Which of the following statements is FALSE?

A. B. C. D.

Excess water-soluble vitamins are excreted in the urine. Fat-soluble vitamins require binding proteins for transPort in the blood. Ascorbic acid is a fat-soluble vitamin' An antagonist for Vitamin K could lead to longer blood-clotting times.

&

86.

If

a person consumes 1800 kcals, and

if

3070

of the

energy in that food is from triglyceride, how many grams of triglyceride are consumed?

A. B. C. D.
87.

540 grams
60 grams 67 grams 54 grams

About 250 mL of water is produced through

metabolism and is called metabolic water. Which of

the following reactions would NOT


metabolic water?

produce

A.
B.
C.

The last step ofthe electron-transport chain Synthesis of a protein from amino acids

Glycogen production
The breakdown of triglYcerides

D.

Copyright @ bY The BerkeleY Review

156

The BerkeleY Speciatizing in MCAT Pre

Biology
Passage

Niacin Experiment
88. If
none

Passage XfV

XIV (Questions 88-94)

quantities

enzymes of energy metabolism. It is also used in gram quantities, at pharmacological levels, to lower levels of cholesterol and triglycerides in the blood. This beneficial effect is believed to result from reduced fatty acids leaving the adipose tissue and entering the liver. Once in the liver, free fatty acids are believed to influence the hepatic assembly of lipoproteins.
89. o
il

Niacin is a B vitamin that is required in milligram by the body as a cofactor in many oI th"

triglycerides in the adipocyte, what ratio of glycerol to fatty acids would be seen in the blood?

of the fatty acids were reesterified into

A. B. C. D.

2:3

l:2
1:3

l:4

Niacin

ry

C-

OH

. A research group is testing the following :bout niacin's


mechanism of action:

isotopic enrichment? hypothesis

from the blood must be measured. tf tne isotopically enriched molecules use-d were 11,23,4:t3Cipalmitate and 11,1,2,3,3-2Hl-glycerol, then what instrument would be used for measurement of

For the turnover measurements, the isotopic enrichment of fatty acids and glycerol extracted

!{rpothesis

Inside adipocytes, niacin inhibits reesterification of :atty acids that were liberated by hormone-sensitive
-pase.

A. B. C. D.

Scintillationcounter
Mass spectrometer Geiger counter UV spectroscope

:ryeriment

I
palmitate and

90.

.:; first studied with isotopically labeled


;11'cerol

Six hyperlipidemic and hypercholesterolemic subjects

:-r'cerol. The isotopicalty labeled compoundi are used as :3cers to study the entry of fatty acids and glycerol into "":e blood from adipose stores. The subjects ihen receive :-acin for 2 months, building up to 3 grams per day. After ;'s period, the subjects repeat the turnover study with rjicates the data obtained from this study:
Phase
Pre-

to quantify turnover of free faity acids and

To extract glycerol from the blood, a student uses a series of cation and anion exchange resins. A portion of deproteinized blood is adjusted to pH 7.b. The blood is applied to a cation resin therto an anion resin. Following a water rinse through the exchange
resins, where is the glycerol?

Blood sample

:reled palmitate and glycerol. The following table


Fatty acid turnover
fumol/kg/min)
niacin
Post-

Cation exchange resin

Glycerol turnover
(pmoVkg/min)

W
Ed

Anion exchange
3.4
2.1 1.2

resin

q.rl,'.#

t#

niacin

1.3

Table I:_Turnover of fatry acids and glycerol (pmol/kglmin.1 in subiects before and after niacin intervirition.

Collection
tube

A. B. C. D.

The glycerol is attached to both the cation and the anion resins. The glycerol is attached to the cation resin. The glycerol is attached to the anion resin. The glycerol is in the collection tube.

tll:]'right

by The Berkeley Review

157

The Berkeley Review Specializing in MCAT preparation

Biology

Niacin Experiment

Passage XIV

91. If fatty acids are released from the adipose tissue,


repackaged in the liver, and sent back for storage in

the adipose tissue, what metabolic effect does this


cause?

A.
B. C. D.

uses energy through a substrate cycle. creates energy through a substrate cycle. Fatty acid cycling damages the liver. Fatty acid cycling damages the adipose tissue.

It It

92.

Which of the following statements is supported by


the data in Table 1?

A.
B.

The pre-niacin period involved no lipolysis.

C. The pre-niacin period did not


D. free fatty acids in the adipose tissue.

Niacin treatment promoted both lipolysis and reesterification. involve


reesterification. Niacin treatment increased reesterification of

93.

If a new experimental drug were studied


increased fatty acid reesterification effect would this have on the body?

that

to

1007o, what

A. B. C. D.

The body would switch to ketone bodies for


peripheral tissue energy metabolism. The brain and nervous tissue would have no ready supply offuel. The muscle and heart would have no ready supply of fuel.

The body would switch to cholesterol for


peripheral tissue energy metabolism.

94.

In a separat6

experiment

to learn more

about

lipolysis, the blood of a group of subjects is studied before and after being given caffeine. Caffeine is a stimulator of hormone-sensitive lipase, an enzyme in the adipose tissue that hydrolyzes triglycerides into free fatty acids and glycerol. What effect would caffeine have on glycerol and fatty acid concentrations in the blood? A.
B.

Glycerol and fatty acid concentrations would


both increase.

Glycerol concentrations would increase, and


fatty acid ioncentrations would decrease.

C. D.

Glycerol concentrations would decrease, and


fatty acid concentrations would increase. Glycerol and fatty acid concentrations would
both decease.

Copyright @ by The Berkeley Review

r5a

The Berkeley Specializing in MCAT

Biology
lTrssage

Dif frrsio n. Limited, E;rrn1rm,e' Qataly zed Reacti o ns


97.

Passage XV

XV (Questions 95-100)

l:r metabolic pathways, the product of one reaction -r::, serves as a reactant for a subsequent reaction. For :i: i-,1son, many enzyme-catalyzed reactions need to be ,:- :n rn one direction. To obtain a negative Gibbs free r::r and thereby drive a reaction in one direction, many inr.,-ilSS are coupled to the hydrolysis of ATP into ADP r: P,. The ratio of ATP to ADP is high, which ensures riL: : ";rlable cellular source of energy.
-- :ell's livelihood is dependent upon its ability to ,l:*' rut its metabolism efficiently. Efficient metabolism
--:es efficient enzymes, working at rates greater than r:r3S of unavoidable, competing side reactions. The rr ; irich is rate-determining in many enzyme-catalyzed r:.!d:-rls is due to the collision frequency of enzyme and ;i..'1,n::::e. rather than the actual enzymatic function of the lr,: :r:. These reactions are termed diffusion-limited, and
rr

Based on information in the passage, which of the following statements is TRUE?

A. Many enzymes B. C. D.
98.

have inefficient catalytic

properties.

Diffusion-limited reactions are faster in the


cytosol than in the nucleus. Multienzyme complexes should increase the rate of diffusion-limited reactions. The hydrolysis of ATP into ADP and ! has a positive Gibbs free energy.

u L:

It is discovered that in a particular cell, a membranebound compartment is 20Vo of the total cellular
volume. The concentration of reactants inside the
compartment can be:

:: "::rr],, they can be accelerated by increasing the :.;rtration of the reactants. However, the cell's '' r":-rsm has limits on the volume of reactants it can . : and most diffusion-limited reactions are not

A. B. C. D.
99.

5 times greater than in the cytosol. 10 times greater than in the cytosol. 15 times greater than in the cytosol. 20 times greater than in the cytosol.

rllr;: ::Ered in this fashion.

The following table shows experimental data for


diffusion-limited reactions taking place outside and within a cell membrane:
Internal
Time required for substrate to hit target
15

-:;
lilL:
tilu

,iLu

t"l ,:. Diffusion-limited reactions that are carried out n r[]': Ihe membrane itself, using membrane-anchored nr,"r':-:s. are also accelerated. The reason is that the ,ii ff,,r::les and enzymes are limited to movement in only \r :imensions. By eliminating a dimension of IIr u , ::,ent, there is an enhanced opportunity for substrate r :rzvme collision.

:: ieactions limited by diffusion. The formation of iiriirle compartments concentrates substrates and r i' :,3s. and it minimizes the distance molecules must

presence of intracellular membranes increases the

Tlial
A B
C

Membrane
None Large Small

minutes

l minute
9 seconds

Based on information presented in the table, it can be concluded that the frequency of collisions in:

uu 'i-hich of the following statements


-

describes an

rEanism approaching equilibrium?

.{. It has a high intracellular ratio of ATP/ADP. B. It attains the most efficient state of C. D.
tiillltr
metabolism. There is a constant input
organism.

A. B. C. D.

Trial Trial Trial Trial

A is

15 times greater than in Trial B. C is l0 times greater than in Trial B. C is 100 times greater than in Trial A. B is 10 times greatff than in Trial C.

of energy into the

100. According to the passage, an efficient enzymecalalyzed reaction requires that DNA-binding
proteins should:

It experiences intracellular death and decay.

-:

rhe process

of diffusion, the distance

a molecule

:rvels from its origin is directly proportional to the :quare root of the time traveled. Based on this riormation, it can be concluded that diffusion is:

A. B. C. D.

contain many regions of acidic amino acids.

randomly jump onto and off of a DNA molecule until they hit an appropriate binding
site.

be translated in the cell's nucleus.

.{. B. C. D.

efficient over long and short distances.

bind to any region of the chromosome and then

efficient over long distances, but inefficient


over short distances

slide along the DNA until they reach


appropriate binding site.

an

inefficient over long and short distances.

inefficient over long distances, but efficient


over short distances.

irr',,:-,sht @

by The Berkeley Review

159

The Berkeley Review Specializing in MCAT Preparation

Biology
l.

Metabolic Components

Section VII Answers

B is correct. We first need to consider the diagram with the enzyme interacting with the substrate and inhibitor (Figure I below). Notice that there are two sites at which binding can take place. The substrate (square) and the
the enzyme-substrate complex.

inhibitor (diamond) bind at different locations. This is an indication that the inhibition is not competitive. Instead, it should look like noncompetitive inhibition. A noncompetitive inhibitor can bind to the free enzyme, or it can bind to

uJ-- txljr
Fl

.'-\

/',---\A

1t^

lln
I
l-s-l

@
)

G-b VY
1

0
Figure 2

l/tsl

Figure

Now that we know the interaction is noncompetitive, we must choose the correct graph. A noncompetitive i decreases the maximal velocity of a reaction (V.u*). The more inhibitor added to the reaction, the more the Vmax decreased. The slope of the line becomes steeper. However, one important characteristic of a noncompeti inhibitor is that the KM remains the same. The constant Ky is important, because it allows us to eliminate e choice except B (see Figure 2 above). The correct choice is B. C is correct. The Michaelis constant, Ku, tells us something about a given enzyme and its relationship to a substrate. Kl,r has a simple definition. It is the substrate concentration, [S], that gives a half-maximal reac velocity. In other words, when [S] = KM, the enzyme is said to be half-saturated with substrate (i.e., vo = y This is what is given in choice A. The Klt also characterizes the substrate-enzyme interaction. It is a measure of enzyme's affinity for a substrate only when the step leading from the enzyme-substrate (ES) complex to the has a rate (k3) that is much smaller than the rate (le) of the ES complex dissociating back to the free substrate enzyme. A small Kvr indicates a strong binding, while a high Krvr indicates a weak binding between enzyme
substrate. This is indicated by choice B.

kr E+STES+E+P

k3

The Krra value between enzyme and substrate is not changed if a noncompetitive inhibitor is added. Only the V changed (it.decreases). This is what we see in choice D. The Ku is not numerically equal to V mu4/2, because [S] and not the value at Ymaxl2. The value of Ymaxl2 might be given in units of pM/min, while the value of might be given in units of mM. The correct choice is C.

3.

A is correct. This question is asking you to rearrange the Michaelis-Menten equation so the initial velocity
solved for in terms of Vrnu*. v

V'* [S] when rearranged


Ku + [S]

gives:

[u*lll

V.u* - V.*
Krur

lsl tsl
V*u*
5

tsl

In the question we are told that [S] = 2.5 x 10-l Ku. This is [S] = 0.25 Ku. Substitution of this value
equation shown above gives the following:
.,

V*u*

K"*l
tsl

Kr *1 0.25 Krra

Vtu* =-J43r- -

I +1

4+1

o.2o V*u*

0.25

The correct choice is A. Copyright @ by The Berkeley Review

160

The Berkeley Specializing in MCAT

Biology

Metabolic Components

Section VII Answers

B is correct. It is important to understand the difference between competitive and noncompetitive inhibitors, how they act, and what factors they affect in comparison to a control (i.e., with no inhibitor). A competitive inhibitor binds reversibly to the active site of an enzyme. It competes with the substrate for the active site and increases the Kna of the enzyme. At a substrate concentration that is high enough, the substrate outcompetes the competitive
inhibitor. Therefore, at a high substrate concentration, in the presence of competitive inhibitor, the Vmax is the same as that of the control. If we increase the concentration of inhibitor, the Vmax remains the same (at a substrate concentration that is high), but the Klvr continues to increase. This is indicated in Figure I below.
2x inhibitor

2x inhibitor

(^-l .{-

Inhibitnt

llv

4t r

Inhibi,o.

A0
Figure I

A0
Figure 2

A noncompetitive inhibitor binds at a site other than the active site and does not compete at the active site for
substrate binding. Binding of a noncompetitive inhibitor inactivates the enzyme and therefore decreases the Vmax. It does not alter the Kru. It we continue to add noncompetitive inhibitor, the Vmax continues to decrease, but the Ku remains the same. This is indicated in Figure 2 above. Combining both of these diagrams gives the graph shown in the question. Reaction I is the control, and Reaction 2 involves the competitive inhibitor. The graph of the competitive inhibitor is shown above in Figure 1. The point that is common to both the control and the competitive inhibitor is Point B. The correct choice is B.

D is correct. Note that the graphs for each mixture are not hyperbolic. They are sigmoidal. They follow nonMichaelis-Menten kinetics. However, we can use the ideas put forth by Michaelis and Menten to answer the question. Let's consider the Krebs cycle reaction and the graphs as they are given in the question.
Isocitrate

NAD+ (excess)

Enzyme

o-Ketoglutarate

+ NADH + H+ +

CO2

Based on the reaction equation and the three mixtures we see that excess NAD@ is used in each case. We also see that as we move from Mixture I to Mixture 2 to Mixture 3, the concentration of ADP in solution increases. What can we conclude from this? Does ADP increase the Ku of the enzyme? No, because as we increase the concentration of ADP in solution, the curves move to the left. This is characteristic of a decrease in Krra. Eliminate choice A. Is ADP an allosteric inhibitor of the enzyme? No, because as we add more ADP, the curve moves to the left, indicating that /ess substrate is needed to reach half-maximal velocity. Eliminate choice B. Does a high [NADH]/[NADo] ratio stimulate the enzyme? This is telling us that we have more NADH (the reduced form) than NADo (the oxidized form). If we have less NADo (i.e., it is not in excess), then the reaction rate will slow down. There will not be enough of the NAD coenzyme to facilitate catalysis. Eliminate choice C. A high IATP]4ADPI ratio means that there is less ADP than ATP. This would mean that the graph would resemble that of Mixture l. In other words, a high ATP concentration inhibits this reaction. This makes sense, because if we have plenty of ATP in the cell, why waste energy making more? The cell makes more ATP only if its concentrations of this nucleotide have been depleted. The correct choice is D.

'f*"

B is correct. In order to go from Figure 2 to Figure 3 in the passage, we must have a movement of electrons from the imidazolium nitrogen of histidine to the nitrogen atom of the nicotinamide ring. We can see this if we consider the flow of electrons in Figure A below. [This diagram corresponds to Figure 2 in the passage.] After electron movement and the formation of new bonds, we get Figure B. [This corresponds to Figure 3 in the passage.]

- ru1'right @ by The Berkeley Review

r6l

The Berkeley Review Specializing in MCAT Preparation

Biology

Metabolic Components

Section VII Answers

In Figure A, the electrons on the nitrogen atom of the histidine ring move to an area of electron deficiency. Areas of electron deficiency are indicated by atoms bearing a positive charge (or partial positive charges). Areas of electron deficiencies are sometimes referred to as electron slnks. In Figure B, we see that there isnow a pair of electrons or the nitrogen atom of the nicotinamide ring and that the nitrogen atom of the imidazolium ring of histidine bears a positive charge. Even though the electrons could end up on the atoms indicated in any of the other choices (and ir theory they might, for a fleeting moment), they do not produce the stablest end product. The correct choice is B.

Figure A
7.

Figure B

C is correct. Consider the bond in the question as they are shown in Figure 1 below. If we were to hydrolyze bond with water, we would get a carboxylic acid functional group and a phosphoric acid functional group (Fi below). These are two different acids. If we mix them together and lose the element of water between them anhydrous reaction), we get a mixed acid anhydride linkage. Note that this linkage does not resemble the lin either a phosphodiester bond, a peptide bond, or an amide bond.

tf- c- oH ocHroroi Figure


1

\9no I
'6g-

o
+

o
I

p- gv

HO- PU

ll

l,n

l"

H-C-OH
la
cH20Po3

lo

Figure 2

is

Examples of a phosphodiester linkage, peptide linkage, and an amide are given below. Notice that a peptide just a special class of an amide linkage.
R

li

lllllCH' O-p-O-p-O-p-O-R

ooclol

r r r o I o I o

--

R-C-N-R'

l__J Phosphodiester

I Peptide

| As

"

HtN

,C-I

l-o

Amide

The correct choice is C.


8.

C is correct. If the sulfhydryl sulfur were to become an electrophile, it would bear a positive charge and be able to attack a (partially) positively charged carbon atom of the aldehyde functional group. Therefore, eliminate choices B and D. When the sulftrydryl sulfur loses its hydrogen atom, it becomes a nucleophile, a that seeks out electron-deficient centers (like the carbonyl carbon atom) and pass electrons to them. The now becomes one of what is formed after this happens. Look at Figure 2 in the passage. Do we see an acylthi or a hemiacetal? Consider the word "acylthioester" for a moment. Let's break this into its components. acyl, thio, ar'd ester. We know what an esler looks like--it is an R-CO-O-R'linkage (see below). The prefix from thiol, which is just R-SH. Thiols are the sulfur analogs of hydroxyl groups. A thioester would then look CO-S-R' (see below). An acyl group is derived from an acetyl group (CH:-CO-R) and looks like R-CO. It is carbonyl group attached to something. Notice that an acylthioester needs a carbonyl group in the i structure in Figure 2 of the passage. We do not see that. Therefore, we can eliminate choice A.

Copyright @ by The Berkeley Review

162

The Berkeley Specializing in MCAT

ffiflology

Metabolic Components
o

Section VII Answers

R-C-O-R'
Ester

il

o il R_C-S-R'
Thioester

\\.hat is a hemiacetal? A hemiacetal can be formed when an aldehyde undergoes a nucleophilic attack by a hydroxyl group (or in this case, a sulfhydryl group). The general reaction is shown below:

fl)
,"t-\ RH

t
:

"' i

R':R*C-S-R':R-C-S-R' I I HH

ooH rl
I HH
I

Hemiacetal

he hemiacetal that is formed is usually too unstable to isolate. However, as shown in Figure 2 of the passage, this is

ire structure that is linked to the enzyme. The correct choice is C. C is correct. As you read in the passage, dehydrogenase enzymes can contain either niacin or vitamin Bz. If the :nzyme contains niacin, it is referred to as an NAD-linked dehydrogenase. If the enzyme contains vitamin B2, it is .;lerred [o as an FAD-linked dehydrogenase. Yery little niacin is found in either milk or eggs. This allows us to :liminate choice A. Even though fish is a good source of niacin, it is not mentioned as being a good source of ";tamin B2. Eliminate choice B. Similarly, beans are a good source of niacin but are not listed as being a good i -'urce of vitamin Bz. Eliminate choice D. This leaves meat as a common source of both niacin and vitamin Bz. The correct choice is C.
B is correct. This question is asking whether you can recognize the difference between something that is oxidized "".i something that is reduced. Only one of the oxidation-reduction pairs is not found at the active site. Consider the --.gures I -4 in the passage. In choice A, we find the reduced form of NAD (i.e., NADH) and the reduced form of ; i steine (i.e., R-SH) at the active site in Figure 4. Eliminate choice A. In choice C, we find the oxidized NAD (i.e., \AD@) and the reduced cysteine at the active site of Figure 1. Eliminate choice C. In choice D, we find the oxidized
5

\D

and the oxidized cysteine (i.e., R-S-R') at the active site of Figure 2. Eliminate choice D.

i: choice B, we also find the reduced form of NAD at the active site of both Figure 3 and Figure 4. However, the ::,rduct, which is in the active site of Figure 4, has been oxidized (and not reduced). How can we tell whether the ::lduct has been oxidized? The oxidation state of the carbonyl carbon in the substrate in Figure I is +1. The :tidation state of the carbonyl carbon in the product in Figure 4 is +3. As we move from the substrate to the :r-rduct, there is a 2-electron change at that carbonyl carbon. Ifthere has been a loss ofelectrons, then it signifies an - ri.lation. A gain of electrons would signify a reduction.
{nother way to consider this is to think of just the substrate and the coenzyme. Since they are both involved in an :riidation-reduction reaction, then one must start off in the oxidized form, while the other is in the reduced form. {tter the reaction is over, the one that was originally in the oxidized form becomes reduced, and the one originally .: the reduced form becomes oxidized. In Figure l, the coenzyme NAD is in the oxidized form (NAD@). Therefore, :he substrate must be in the reduced form. At the end of the reaction, NAD is in the reduced form (NADH), and the ::oduct is in the oxidized form. The correct choice is B. B is correct. The reaction in the question tells us we need to consider the coenzyme FAD. The structure of this :r-IlZltrle is given in the passage in Figure 5. Note that the oxidized form is highly conjugated. We are told in the :L\sage that "...the more conjugated bonds a molecule has, the longer the wavelength at which the molecule absorbs ;rght." In the question we are told that "oxidized FAD absorbs light in the visible region of the electromagneric ipectrum at 460 nm." We are also given a graph of light absorption at wavelengths in the visible region of the :lectromagnetic spectrum.

-\t

the beginning"of the reaction, we have a solution of succinate (reduced) and FAD (oxidized and highly

;urnjugated). The oxidized form of FAD is absorbing light at 460 nm. By looking at the graph, we see that rhe blue ;one is being absorbed. If a wavelength of light is being absorbed, we perceive its complementary color. In this case, :he complementary color to blue is yellow (a mixture of the green and red cones). Therefore, our starting solution is 'n ellow. We can eliminate choice A and choice D.

i1 ur,,*:ht O by The Berkeley Review

163

The Berkeley Review Specializing in MCAT Preparation

Biology

Metabolic Components

Section VII Answerg

At the end of the reaction, we have an oxidized product (fumarate) and a reduced coenzyme IFADHz). Note that fu structure of FADHz in Figure 5 of the passage is not as highly conjugated as the structure in the oxidized state. Thc fewer conjugated bonds a molecule has, the shorter the wavelength at which that molecule absorbs light. Bec FADHz has so few sites of conjugation, we would expect absorption to take place outside the visible range of electromagnetic spectrum. The final solution will be colorless. The correct choice is B.

12.
13.

D is correct. Choice A is p-D-glucose. Choice B is saccharic acid. Choice C is fructose. Choice D is gl lactone. A lactone is recognized by the fact that it is a cyclic ester. The correct choice is D.
C is correct. Blood glucose would rise in response to a meal, then fall when insulin was released by the pancreas response to elevated blood glucose. A slight dip is common when the glucose level is lower than at the state, and then it corrects itself. Blood glucose neither rises linearly nor falls linearly. Choices A and D are i Blood glucose does change in response to a meal, so choice B is incorrect. The correct choice is C.

14.

A is correct. If we read the standard curve, this is an easy question. Find 1250 on the y-axis. Move horizontally u you intersect the line of the standard curve. Move down to touch the x-axis, and read off the correct answer, I
The correct choice is A.

15.

A is correct. HzOz from any source would reach the electrode and be oxidized, although the person reading the results assumes that only HzOz from the glucose oxidase reaction is reacting. However, if extra HzOz somehou into the machine, then the reading would be misleadingly high. Choices B and C are incorrect. HzOz does oxidize glucose. The correct choice is A.

16.

If the filter blocked hydrogen peroxide, then nothing would reach the electrode and be oxidizsd make a signal. Choice A is incorrect. If many oxidizable substances passed through the filter, then the readi the electrode would be amplified by other oxidation reactions. This would make an incorrect reading of p glucose. Choice B is incorrect. Choice C is the correct answer: The filter keeps large oxidizable substances from the electrode. Choice D is incorrect. The filter in question is downstream from the immobilized enzyme. does not protect the enzyme. The correct choice is C.
C is correct.
B is correct. Oxidation occurs at the anode. The hydrogen peroxide is oxidized. The platinum acts as a agent. Platinum acts as an anode. Choices A and C are incorrect. An oxidase is an enzyme. It plays a role in the reaction. Choice D is incorrect. The correct choice is B.

17.

18. A is correct. In the uncoupling

situation, fewer ATPs are produced per unit of fuel. The dissipation of electrochemical H@ gradient produces heat, which is probably important in regulation of body temperarlr neonate.s. Choice B is incorrect. More fuel must be burned to make ATP than in the regular, coupled state. Bodl' stores would be recruited, not conserved. Choice C is incorrect. All this oxidation would lead to an increase in resting metabolic rate. Choice D is incorrect. The correct choice is A.

19.

C is correct. The passages tells you that the number of mitochondria is greatly increased in BAT versus
"Chrome" usually refers to a colored substance. The cytochromes contain iron and are brown in color. The WA the same cytosolic components as the BAT, so they are not causing differences in color between the two tt tissue. Choices A, B, and D are incorrect. The correct choice is C.

20.

D is correct. The diagram is a second messenger system. The hormone does not enter the cell, but relies i its communication with the cell interior via the second messenger, which is a G protein, in this case. Choice incorrect. The s-subunit is involved in activating adenylate cyclase. The p- andy-subunits are neutral in this
Choices B and C are both incorrect. The correct choice is D.

21.

B is corrett. The fat cell is a storage depot for triglyceride. A triglyceride contains glycerol esterified to three acids. It is neutral and hydrophobic. This is the form that is used for storage in fat cells. Fatty acids are presenl blood and are oxidized for fuel in the tissues. Phospholipids are located in the membranes, predominantly. Gly is a component of triglycerides, and it also exists free (unbound to anything) in the blood. The comect choice fu

Copyright @ by The Berkeley Review

16,4

The Berkeley Specializing in MCAT

Biology
1)

Metabolic Components

Section VII Answers

A is correct. Although the adipose tissue does not perform gluconeogenesis, the liver tissue does and can use the released glycerol for the production of glucose. This plays a role in glucose homeostasis. BAT does not contain
glycogen or release glucose. Choices

B and C are incorrect. Fatty acids cannot be made into glucose,

by

gluconeogenesis or any other pathway in the human body. Choice D is incorrect. The correct choice is A.
11

C is correct. Thermogenin is probably most like a protein with a similar role in the body. Chymotrypsinogen is a hormone that, when cleaved properly, digests certain peptide bonds in food. Choice B is incorrect. Hemoglobin is a transporter of oxygen in the blood. Choice A is incorrect. Choices C and D exist in the membrane of cells and allow things to transport through the membrane. However, ATP-ADP synthetase is located in the mitochondrial inner membrane and allows protons to cross, driving ATP synthesis. Thermogenin allows protons to cross the inner mitochondrial membrane for free (i.e., without an input of energy). The sodium-potassium pump requires energy. Choice D is incorrect. The correct choice is C.

t1.

B is correct. This question can be answered using previously acquired knowledge. The human red blood cell contains hemoglobin molecules. Therefore, if we know where the human red blood cell is degraded, we know where the breakdown of heme occurs. A typical human red blood cell has a life span of 120 days. Old cells are removed
from the circulatory system and degraded in the spleen. The correct choice is B.

{lflll

ilIfi
rfl0

l, we see that the way to determine whether the conversion is a reduction or an oxidation is to make note of the cofactor involved (in this case, NADPH). The cofactor itself is oxidized as a result of the reaction, and the oxidation involved two electrons. The two electrons given off had to go somewhere, and they went to the reduction of biliverdin into bilirubin. Remember, we cannot have an oxidation without a reduction. Therefore, we can conclude that the conversion is a two-electron reduction. The correct choice is D.
D is correct. Looking at Figure A is correct. We have to look
at the two structures given in the question. First, there is no difference in the chemical constituents of the two molecules. However, the atoms are arranged differently. For this reason, an isomerase is the enzyme that would be involved. Next, in looking at the picture one should arrive at the conclusion that uro I is a

80tI
ritml

ml
lml

symmetric molecule, symmetric around the center point of the molecule (where the iron would be bound). By srvitching one set of constituents, uro III is produced. Switching these constituents produces an asymmetric molecule. The correct choice is A.

C is correct. The passage informs us that carbon monoxide is released when the heme group is converted to biliverdin. From one's previous knowledge of Hb, one should remember that carbon monoxide has a very strong allinity for the iron atoms of the hemoglobin molecule. That is why the gas is poisonous. Judging from this reaction, \\ e can assume that the body naturally produces a small amount of this gas. However, the amount is very small relative to the amount of oxygen in our body. Nonetheless, we cannot ignore the fact that CO is indeed produced, and a small percentage will indeed bind to Hb. The correct choice is C.
:$"

C is correct. The passage informs us that persons suffering from this disease have abnormal red blood cells that are liminated by the body. Therefore, we can assume individuals with this disease have a lower than normal red blood :ell count. Does the red blood cell level contribute to the partial pressure of oxygen? The answer is no. The partial Jressure of oxygen is determined only by the amount of oxygen dissolved in the blood, not by the amount of oxygen :ound to hemoglobin. For this reason, persons with CEP would not have a lowered partial pressure of oxygen. The correct choice is C.
e

.\

is correct. This is a very straightforward genetics question. We know from the passage that the disease in question, CEP, is transmitted in a autosomal recessive fashion. The father is heterozygous for this trait, making his

:.notype Cc (letters chosen arbitrarily). The mother is a homozygous dominant, making her genotype CC. The llrestion asks for the probability that a son will suffer from the disease. Doing the cross between parents, it becomes rpparent that no child will suffer from the disease, as no recessive allele is contributed by the mother. Therefore, ::ere is aOTo chance. The correct choice is A.

frlt

B is correct. Trypsin is an enzyme that cleaves peptide bonds on the carboxyl side of the amino acids arginine (Arg) r:rJ lysine (Lys). The important clue to answering this question comes from the last sentence of the second :aragraph. It says, "...enzymes readily degrade polypeptides synthesized from L-amino acids, but not D-amino ,;ids." This tells us that we must find that structure with the two amino acids in the L configuration. How do we do ::.,s? We need to recall our stereochemistry from organic chemistry.
"*

il.r r))'i!

-ci'. 9 by The Berkeley Review

165

The Berkeley Review Specializing in MCAT Preparation

Biology

Metabolic Components

Section VII Answers

Recall that a Fisher projection is a way of representing a tetrahedral carbon (and its substituents) in three
dimensions. The structures of the dipeptides in the question are drawn in Fisher projections. Each dipeptide has trr " chiral carbon atoms, each with four different substituents. Focus on the four bonds attached to those chiral carbon' Vertical lines represent bonds pointing away from your point of view, while horizontal lines represent bontJ: pointing towards your point of view.

Rank the four substituents on each chiral carbon in order of decreasing priority. Arrange the order so that thr substituent with the lowest priority is on top. In each case the substituent with the lowest priority is hydrogen (H). Ii two substituents need to be exchanged so that hydrogen is on top, then do not forget to exchange another pair cr substituents so that the absolute configuration is retained. Next, trace the order of priority of the three remaining groups. If thetracing is in the clockwise direction, the stereocenter is designated as R (rectus, Latin for right). If the tracing is in the counterclockwise direction, the stereocenter is designated as S (sinister, Latin for left). However" since amino acids are designated by the older D and L prefixes, we must make a conversion. It turns out that the oltj D prefix is analogous to the old R prefix, and the L prefix is analogous to the S prefix. We are now ready to find the
answer.

Consider Structure I (see below). Both chiral carbon atoms have their hydrogens pointing up. All we need to do is arrange the substituents in order of priority. Let's do lysine first. Nitrogen (N) has an atomic number of 7, while carbon (C) has an atomic number of 6. Nitrogen is assigned the highest priority. Next, we need to distinguish between the carbons attached to the chiral carbon. The carbon atom with the oxygen atom attached has the nerl highest priority. This is followed by the methylene (-CHz-) carbon atom. The tracing is from nitrogen to carbonln carbon to methylene carbon, giving a clockwise direction. This stereocenter is R, which in the old nomenclature i: D. Right away, we know that Structure I cannot be the answer, because we need both chiral centers to be in the L configuration. We just found that one chiral center is in the D configuration.
R (D) R (D) H1N- C- C-

@ru

HO

;rb

"?1 CH,
tI

N_ C_ COO ta I HCHr_ b
Alanine (Ala)

ro

CH. CH,

t'

NH3

Lysine (Lys)

Let's continue this example by finding the configuration about alanine's chiral carbon. Again, the hydrogen atom r$ pointing up and therefore has the lowest priority. Nitrogen still has the highest priority, followed by the carbor$ carbon, andfinally the methyl carbon. The tracing is again clockwise, giving the D configuration. D olll HOH^ lo HOCH, ottt D

OOC_

C- N - C_ C_ trl H.C H CH. 't'


CHt
I

NH"

H1N-C-C-N-C-COO rtt CH,HH


I

rO

CH.OH lll l(9 O ooc- c- N - c- c- NHI llt


HHCH,
I

CH.
I

CH,

II
O

CHt

i"'
I

fn, ilI
NH
I

IV

tI

fn'
NH

NH3

HzN

,caa

NHz

OC .a\ H:N

NH,

Arginine (Arg)

Following this same procedure for Structure II, we get a configuration of L-Lys and L-Ala. Structure III has configuration of D-Arg and L-Ala. Structure IV has a configuration of D-Ala and L-Arg. The correct choice is B

Copyright @ by The Berkeley Review

166

The Berkeley Specializing in MCAT Prepar

Biology
31.

Metabolic Components

Section VII Answers

D is correct. As stated in the fourth paragraph of the passage, DIPF is an irreversible inhibitor. Since this inhibitor reacts with active site serine residues, it must enter into the active site, bind to a specific serine residue (see below), and shut the enzyme down. Allosteric inhibition refers to the fact that binding is taking place at a site other than the active site. Since this is not the case here, we can eliminate choice A.
HrCF Serine

o
CH(CHj}2

{HrC)2HC-

O- P- Oil

"p Serine-CH'OH +

(H3C)2HC-O-P-O-CH(CH3)2

o
DIPF

il

+ HF

o
Phosphorylated active site

If DIPF were a competitive inhibitor, it would mean that it is of the reversible type, which is not what is stated in the
passage. We can eliminate choice B. What about phosphorylation of ADP?

If DIPF

could phosphorylate ADP (and

in the process form HF), then it would be unable to react with the active site serine residue. This allows us to eliminate choice C. In order for DIPF to bind irreversibly to the active site serine residue, it must phosphorylate that
residue as indicated above. The correct choice is D.

1'

B is correct. Notice that initially (at low [S]) the reaction velocity for Curve II is lower than the reaction velocity for Curve I. However, as the [S] is gradually increased, the reaction velocity of Curve II approaches that of Curve I. Eventually, the two reaction velocities (in theory) will reach the same velocity (i.e., V63*). This is exactly what is mentioned in the last paragraph of the passage. The correct choice is B.

11

A is correct. As outlined in the last


eliminate choice B and choice D.

paragraph of the passage, competitive inhibitors show the same maximal velocity. This means that the maximal velocity of the competitive inhibitor crosses the y-axis on the graph at the same point as the maximal velocity of the control. Only Lines I and III intersect at that point. This allows us to

The most effective competitive inhibitor is that inhibitor that requires more substrate to reach half-maximal saturation. In other words, the most effective competitive inhibitor is that inhibitor with the largest apparent Kvr. This would give the smallest value for -llKrrl. Another way to look at this is that the most effective competitive
inhibitor is indicated by the line with the steepest slope. The correct choice is A.
3-t.

C is correct. Almost all enzymes are proteins. Since proteins are composed of amino acids, they are quite sensitive to pH changes. We are told in the question that aspartate 52 (Asp-52) has a pKn of about 4.0, while glutamate 35
(Glu-35) has a pKs of about 6.0. Based on our discussion of pKns we know that at the pIG of a particular amino acid side chain the dissociable hydrogen atom is half on and half off. The dissociable hydrogen on the side chain spends half of its time in the protonated state and half of its time in the deprotonated state.

At low pH values (close to l), we would expect both the Asp-52 and the Glu-35 side chain carboxyl groups to be protonated. As the pH begins to increase and approach the pK2 of Asp-52, we find that the side chain carboxyl of Asp-52 begins to lose its hydrogen atom. That carboxyl group begins to take on a negative charge. By the time we are one full pH unit away from the pKn for Asp-52 (at a pH of 5), we find that'the side chain carboxyl group spends most of its time negatively charged. This negative charge stabilizes the substrate at the active site. At a pH of 5, we are one full pH unit below the pKa for Glu-35. This means that the side chain carboxyl of Glu-35 still has its dissociable hydrogen atom. It is this hydrogen atom that eventually gets transferred to the substrate at the active site. As we increase the pH even more, say, to one full pH unit above the pK3 for Glu-35, we find that the side chain carboxyl of Glu-35 now spends most of its time negatively charged. Since we do not have a proton to donate to the substrate anymore, the reaction rate begins to decrease. Therefore, we would expect an increase in reaction rate as we approach a pH of 5, and a decrease in the reaction rate as we move to pH values higher than 5. This is exactly what we see in the bell-shaped curve in Graph III. With this information, you should be able to reason why Graphs I, II and IV are invalid solutions. The correct choice is C.
35.

B is correct. Aspartate-52 provides the general base catalysis, because its side chain p-carboxyl group is about 97o protonated at a pH of 5. At a pH of approximately 5, we find that Asp-52 would acts as the general base (-COOo), while Glu-35 would act as the general acid (-COOH). Without even dealing with percentages, we can immediately
eliminate choice A and choice D.

lnght

by The Berkeley Review

167

The Berkeley Keview Specializing in MCAT Preparation

Biology

Metabolic Components

Section VII Answers

Let's consider choice B and choice C. In choice B, we are told that G1u-35 acts as the general acid, while Asp-52 acts as the general base. We know this is correct. Are the percentages correct? Will the side chain carboxyl group of Glu-35 be about 9Vo deprotonated at a pH of 5? Let's give a rough estimation first. At a pH of 6, we know that the side chain carboxyl of Glu-35 is 507o protonated and 50Vo deprotonated. This is because the pKa of the side chain carboxyl of Glu-35 is given as 6. We can see this quantitatively by using the Henderson-Hasselbalch equation, as shown in the first column below:
Using pH = 6

Using pH =

pH = pK2 + tog

J4I

pH = pKn + tog [A-]

tHA]

6=6+log
o=

[A-]

5=6+log
-t
=

[A-]

tHA]

[HA]

1o,

[A-]

1o, [A ]
tHA]

IHAI

100- [A ]
IHA]

t'-t _
tO

] or t'r= [HA] [HA] tAl


[A

1=

tHA]

[Al or [HA]=[A]

[Ha] or l0 [HA] l tAl tA-l

Let's try this same approach with a pH of 5. This is shown in the second column above. At a pH of 5, the side carboxyl of Glu-35 is more on the protonated side and less on the deprotonated side. After applying the Hasselbalch equation, we end up with l0 = [HA]/[Ao]. This states that there is ten times more of the proto (HA) form of the carboxyl side chain than the deprotonated (Ae) form. In other words, Glu-35 is about 9l protonated or about 9Vo deprotonated. A similar calculation can be made for Asp-52. The best way to get a feel as to whether or not an amino acid is protonated or deprotonated, and by how much, is use the Henderson-Hasselbalch equation in a series of simple calculations. Start like we did in the first col where the pH is equal to the pKn of the dissociable hydrogen of interest. Next, move one full pH unit away from pKa (in either direction). You will find results similar to the ones in the second column. Move two full pH away from the pK6 of interest. Try three full pH units away. You should begin to see a pattern develop. In fact, you understand this pattern, you will not need to use the Henderson-Hasselbalch equation for the calculation. Y will just be able to do a very quick and fairly accurate estimation. The correct choice is B. 36.

C is correct. The maximal velocity of the reaction is taken from the intersection of the line with the y-axis.
value of the point at that intersection is 2. ll2.The correct choice is C.

All we

need to do is remember that we have a reciprocal. Therefore,

37.

D is correct. An oxidoreductase catalyzes a reaction in which one substrate is oxidized and another is 'Choice A is incorrect. A hydrolase catalyzes the hydrolytic cleavage of singe bonds. Choice B is transferase moves groups from one substrate to another. Choice C is incorrect. An isomerase moves a group
one position to another within the same molecule. The correct choice is D.

38.

A is correct.This is a question of which came first, Btz or folate deficiency. If Btz is low (i.e., a Btz defic the enzyme homocysteine methyltransferase can't work properly. Any THF that is around is trapped as methylat this enzyme. Folate functions as THF in many aspects of one-carbon metabolism; and in this case, it is deli

if folate status were normal, this trap due to The correct choice is A. deficiency would cause a secondary folate deficiency.
a methyl group to homocysteine methyl transferase. Even

39.

bypasses the problem with IF and makes the vitamin available to the blood. Statement I is correct. An increase in dietary B tz would not help if the IF were not because the vitamin is absorbed as a complex with IF. Statement II is incorrect. Co itself is not the vitaminvitamin contains Co in the Btz molecule, but Co by itself would be of no benefit. Statement III is incorrecl correct answer is A.
@

A is correct. An intramuscular injection of Brz

Copyright

by The Berkeley Review

l6a

The Berkeley Specializing in MCAT Pre

Biology
40.

Metabolic Components

Section VII Answers

B is correct. If you remember the anatomy of the GI tract, the ileum is the last section of the small intestine, and the colon or large intestine follows it. The receptor of Btz is in the ileum, so any B12 produced past this receptor (in the colon) is not absorbed. The makes choice A incorrect. Choice B is correct. Choices C and D imply that the colonic B tz is absorbed to have reactions on enzymes in the body, and are therefore incorrect. The correct choice is B.

41.

A is correct. If methyl malonyl-CoA cannot be metabolized into succinyl-CoA, then the CoA is hydrolyzed and the excretion product is MMA. Choice A is correct. Brz is not a competitive inhibitor of the MMA pathway. Choice B is incorrect. MMA is not a by-product of homocysteine methyl transferase; this is another enzyme altogether. Choice C is incorrect. MMA is not made from IF. Choice D is incorrect. The correct choice is A. D is correct. Met is an essential amino acid. This may
seem like a trick because of the reaction with homocysteine methyl transferase. However, this homocysteine had to be made using a Met in the first place. Choice A is true. We can see from the diagram that choice B is true. Met is used in gluconeogenesis, so choice C is true. The precursor of Tyr is Phe, so choice D is false. Since you want the false answer, choice D is it. The correct choice is D.

42.

43.

B is correct. IF is a glycoprotein, which is a protein-carbohydrate combination. Just like the protective mucus of the GI tract, glycoproteins have nondigestible carbohydrate bonds. IF is protected from digestion, because our bodies do not have the digestive enzymes to break it down. Choice A is incorrect, because pepsin (a protein-digesting enzyme) is present in the stomach. Choice C is incorrect, because IF does not influence the activation of pepsin. Choice D is incorrect, because the passage tells us IF makes it to the ileum from the stomach. It must be resistant to gastric enzymes. The correct choice is B.

44.

B is correct. The values of the reactants and products at equilibrium is 10 moles. Using the equation AG'' = -2.3 RT log Ksq, where.Ksq = [C]/[A][B], we find that AG"'= + 2.3 RT. AG'' = - 2.3 RT log AG'' = - 2.3 RT log

K". 0.10

-)
--)

AG"'= - 2.3 RT

log [C]
tAl

IBI

= - 2.3RT

log

[10] [10]

tl0t

AG"'= - 2.3 RT (- 1.0) = + 2.3 RT

The standard free-energy change is an important concept to understand, because it will be able to tell us whether a reaction is spontaneous (exergonic) or nonspontaneous (endergonic). The correct choice is B. {5.

A is correct. Lyases are involved in the cleavage of bonds like C-C, C-N, and C-O. Lyases can cleave double bonds
to make single bonds, or single bonds to make double bonds. Functional groups can be either added across or taken away from these bonds, respectively.

ill

OO coo coo Fumarate


CH ll +H"O+l ' cH lolg Fumarase
HO

coo

- CCHt

coo
Malate

In the example of fumarate going to malate, we see that water is being added across a double bond. This type of reaction occurs quite frequently in biochemistry. If we add the element of water to a bond, it is referred to as a hydrationreaction. Ahydrolysis reaction involves the use of water (hydro-) to break (-lysls) abond. The correct choice is A.
16.

C is correct. Hexokinase aligns the 1-phosphoryl group of ATP with the C-6 hydroxyl of glucose and catalyzes a transfer of that y-phosphate group to the C-6 position of the sugar. Since this reaction involves a chemical group transfer, the enzyme is a transferase.
Ligases involves the hydrolysis of a high-energy bond, such as those found in ATP; but the energy in that highenergy bond is used to drive the condensation of two molecules via a ligation reaction. Hydrolases involve the use of water to break a specific bond. Oxidoreductases are enzymes involved in electron transfer through oxidationreduction reactions. The correct choice is C.

rr:''right @ by The Berkeley Review

169

The Berkeley Review Specializing in MCAT Preparation

Biology

Metabolic Components
o o
O-

Section VII Answers

(o
HzC

t/> o I o- P- o- P,I
lt

o
H

AMP ol oHzC

P= o
I

;o.'
on

o on
I

"l\Y
OH

1n\

l-o

T. "si?
OH

Hexokinase

"r. ,/f__ o

Glucose

Glucose-6
phosphate

47.

C is correct. We can think of the nucleophilic properties of the C-6 hydroxyl of glucose and water as being similar, we consider both molecules as having the R-OH format. The R for water is a hydrogen atom, while the R fr glucose is the rest of the molecule other than the C-6 hydroxyl. Even though the hydroxyl functional group in botl cases acts as a nucleophile, the R group in each case is quite different.

if

The presence of glucose at the active site (along with ATP) induces a large conformational change in the enzym and allows the active site to close around the substrates with the exclusion of water. If water is excluded from rh active site, the polarity of the active site's environment is reduced. The active site becomes hydrophobic. This that the C-6 hydroxyl of glucose and the y-phosphoryl group of ATP are not solvated. Therefore, the C-6 hr group's oxygen atom becomes more nucleophilic, while the y-phosphoryl group's phosphate atom becomes electrophilic. These events allow for an accelerated transfer ofthe phosphoryl group to glucose. If water were al position occupied by the C-6 hydroxyl group, it would be able to attack the y-phosphoryl group of ATP. The would be the hydrolysis of ATP to ADP and (inorganic) phosphate. In other words, the rate of ATP hydrolyri would increase. As stated in the question, this is not what is observed. The correct choice is C.
48.

B is correct. It is important to know the differences between an aldose sugar and a ketose sugar. Sugars saccharides, polyhydroxyl carbonyl compounds that contain an aldehyde or a ketone functional group and at I two hydroxyl groups. Sugars with aldehyde functional groups are referred to as aldoses, while sugars with ket functional groups are referred to as ketoses. Even though you do not need to know the complete structures of aldose and the ketose in the second step of glycolysis, you do need to know (recognize) the aldehyde and functional groups of a sugar. We can draw them as shown below:

_lt

o-H --c, EnzYme cH2oH ; H_C_OH + C=O l- H Ho-l "o* "

Aldose

Ketose

What type of enzyme assists in the conversion of an aldose into a ketose? Notice that in going from an aldose ketose, there is no loss or gain of atoms. These two molecules differ in the sequences in which their atoms are together. The empirical formula is the same in both cases. These molecules are isomers of one another. the type of enzyme that catalyzes this reaction is an isomerase. The characteristics of the other three choices outlined in the passage in Table l. The correct choice is B.
49.

B is correct. The pK2s for these amino acid pairs are given in Table 2 in the passage. The optimal activity of enzyme is where the initial velocity is the greatest, which is at V6ax (i.e., the maximal velocity). The pH at V about 9.0. We want that amino acid pair whose pKa values border as close as possible to a pH of 9.0.
We can get a'rough estimate of this by adding the two pKa values of the amino acid pair together and then di by 2. For choice A, we get a pH value of 4.95, which is way to the left of the graph and nowhere near the activity of the enzyme. Choice B gives a pH value of 9.2, which is extremely close. Choice C gives a pH v I 1.65, which is a little further away. Choice D gives a pH value of 9.6, which is close to the optimal activity, as close as choice B. The correct choice is B.

Copyright @ by The Berkeley Review

170

The Berkeley Specializing in MCAT

Biology
50.

Metabolic Components

Section VII Answers

C is correct. The passage mentions that lactate dehydrogenase (LDH) has /wo different types of subunits, designated as M (for muscle) and H (for heart). Let's assume that the band shown in Lane I is composed of all M
subunits (i.e., Mx, where x is the number of subunits), while the band shown in Lane 3 is composed of all H subunits (i.e., Hx). Therefore, the bottom band in Lane2 is all M and the top band is all H. The three bands in the middle are mixtures. This is indicated in Figure I below.
Lane

1 Lane2
a

Lane

Lane (+)
5

1 Lane2

Lane 3 (+)

ll:rrr:]:] flir:l]l
r------------r

gl
=J

\./ \
I

,/

4
J 2

Hx
I

L:--= Frl -= r-\,/ \,/ gL H,


lttt

z
Origin

x\t

L_____J

>
Origin ".r::-:"

x\-

sEE H4 + !! u1H, r !! tvt2H,


2
1

Lt-----(-)
MX

!l
lI ll

vr.H'
rr t"t4

1----.

c)
MX

Figure

Figure 2

Consider Band 3 in Lane 2. This band falls halfway between the isoenzyme that is all M and the isoenzyme that is all H. We would expect the isoenzyme indicated by Band 3 to contain an equal mixture of M and H subunits. If

Band3weretocontainoneMandoneHsubunit,thenBandlwouldcontain2Msubunits,andBand5would contain 2 H subunits. This means that Band 2 would have to contain 1.5 M subunits and 0.5 H subunits. Similarly, Band 4 would have to contain 0.5 M subunits and 1.5 H subunits. Since we cannot have anything but a whole number for a subunit (otherwise, the subunit would be nonfunctional), we need to make an adjustment in our
analysis.

Instead of a 1:1 mixture of M to H in Band 3, let's assume a 2'.2 mixture of M to H. This would allow Band 1 to

contain4MsubunitsandBand5tocontain4Hsubunits.Band2nowcontainsaratioof3M:lHsubunits,while Band 4 will contain a ratio of 1M:3H subunits. This is shown in Figure 2 above. This analysis fits the gel pattern and
indicates that there are 5 isoenzymes of LDH, each being a tetramer. The correct choice is C.

A is correct. Bacterial cells that are Gram-positive have a plasma membrane surrounded by a thick peptidoglycan layer, the cell wall. Within this layer are two types of sugar residues, N-acetylmuramate (NAM) and Nacetylglucosamine (NAG), which can act as a substrate for lysozyme. This enzyme has a clear path to these sugars. In a-Giam-negative bacterium, there is an additional outer membrane that contributes to the bacterial cell wall. This outer membrane acts as a barrier that helps to prevent lysozyme from degrading the peptidoglycan layer. If bacteria are missing,their cell walls, chances are that they have already lysed due to osmotic differences between the inside of the cell ind the outside of the cell. If they have not lysed but have formed protoplasts instead, then the lysozyme enzyme is facing a plasma membrane that does not have the NAM and NAG sugar residues linked as they were in the peptidoglycan layer. The correct choice is Ai,]

D is correct. You need to recall your carbohydrate chemistry to answer this question. Rings D and E have been labeled with the C-1 and the C-4 positions already (see Figure 2 in the passage). In order to determine the
configuration of the linkage, we first need to locate the anomeric carbon atom of the ring' The anomeric carbon in the open-chained form is the most oxidized carbon atom. That turns out to be the C- I carbon or the carbonyl carbon of the aldehyde functional group. When the ring begins to close, the oxygen of the C-5 hydroxyl group attacks the C-1 carbon. The carbonyl oxygen at the C-l position picks up a hydrogen atom to form the C-i hydroxyl. That hydroxyl can reside either above or below the plane ofthe_ring, depending on the direction of attack. If the C-l hydroxyl is above the plane of the ring, it is said to be in the p-configuration; if it is below the plane of the ring, it is in the cr-configuration. By examining the linkage between rings D and E, we see that the hydroxyl at the C- 1 position of the D ring is in the ring E. Hence, the 0lconfiguration. The linkage extends from the C-l carbon of ring D to the C-4 carbon of configuration of the linkage is said to be 0(1+a). The correct choice is D.

: . right

by The Berkeley Review

l7t

The Berkeley Keview Specializing in MCAT Preparation

Biologr
53.

Metabolic Components

Section VII Answers

A is correct. All enzymes are based on the type of reaction they catalyze. Many enzymes are known by a common name, usually derived from the principal reactant involved in the catalysis. For example, arginase reacts with the
amino acid arginine.

Arginase falls within a specific class of enzymes, just as lysozyme does. There are six classes of enzymes established by the Enzyme Commission of the IUPAC. In the answers to this question, we are presented with enzymes from four of those classes. A hydrolase enzyme is involved in hydrolysis reactions. Water iJ used to cleave bonds like C-C, C-N, or C-O. This is exactly what we see in the case of lysozyme (and what we will see in the case of arginase in a later discussion). A transferase enzyme transfers a functional group. An oxidoreductase is involved in an oxidation-reduction reaction. A ligase is involved in bond formation (think of ligation) that is coupled to the hydrolysis of ATP. We do not see the last three examples involving lysozyme catalysis. The correct choice is A.
54.

through an intermediate species (the transition state). The transition state has a free energy that is higher than either the substrate or the product. The charge distribution at the active site of an enzyme stabilizes this transition state, thereby lowering the barrier to activation lAGf ) between substrate and transition state. If the barrier to activation is lowered, the rate of a reaction can be accelerated significantly. The correct choice is D.

D is correct. Almost all enzymes are sequences of more than 100 amino acids. However, there are only a few amino acids that can be used to make up the catalytic group of amino acids at the active site. These few amino acids form an enzyme-substrate complex through a variety of bonding interactions. As the substrate forms product, it goes

C is correct. The last sentence of the third paragraph of the passage reads, "The neighboring environments of Glu 35 and Asp 52 are quite different from each other." In the question it states that the pku valuls for Asp 52 and Glu 35 are usually cited as 3.9 and 4.1, respectively. The difference between the two side chains is the additional methylene (-CHz-) group in aspartic acid. Under normal conditions, the influence of this additional methylene g1oup has negligible consequences. Thus, under normal conditions we would expect both aspartic acid and glutamic acil to have similar pKa values for their side chains. Since both side chains are on polar (charged) amino acids, ,yys lryerrldl
expect their environments also to be polar.

polar and the other nonpolar. And we know which is which, based on information contained in the question: "Analysis of lysozyme's active site indicates that the pKn of Asp 52 is still about 3.9, but the pK' of Glu 33 is nom
about 6.6." The pK6 value of the aspartic acid side chain has not changed. The pKs of the glutamic acid side chair has increased by 2.5 times. This tells us that Asp 52 is in a polar environment, and that Glu 35 is in a non environment. In order to remove the dissociable hydrogen from the side chain of Glu 35, the active site must bi u pH close to a pKn of 6.6. In other words, it is harder to remove the dissociable hydrogen from the carboxyl si chain of Glu 35 than it is from the carboxyl side chain of Asp 52. Recall from organic chemistry that dissociation a hydrogen from a carboxyl group is more favored in a polar environment than it is in a nonpolar environment.

However, we know that the two environments are different from each other. We can assume that one environmenr

Even though choice A is a correct answer, it does not address the question. What about choice B? The carboxyl group ofAsp 52 is in a different environment than the protonated carboxyl group of Glu 35 (see Figure 2 the passage). If anything, the negative charge of the ionized carboxyl group would help stabilize the lparti positive charge of the hydrogen atom of the protonated carboxyl group. However, in this situation the two groups are distant from one another, and a stabilizing or destabilizing influence is negligible. In choice D, we that if the carboxyl group of Glu 35 were in a polar environment, then its pKa value would be around 4.1. Hydrc bonding would help in stabilization. But we already know that Glu 35 is in a nonpolar environmenr. With reasoning, we can eliminate choices A, B, and D. The correct choice is C.
50.

2, the C-l and C-4 carbons are identified. Based on this, we can establish where the C-5 and C-6 carbons

B is correct. This question involves careful inspection of Figure 2, Figure 4, and Figure 5 in the passage. In Fi,

located. We know where to find the anomeric carbon (at the C-1 position). The reference carbon is that carbon establishes whether a sugar is in the D or L configuration. By definition, the reference carbon is the last carbon farthest from the most oxidized carbon. In the case of sugar ring D, the most oxidized carbon is at C- l, w the last chiral carbon is at C-5. Therefore, C-5 is the reference carbon. This is great, because it says that choice and choice C are one in the same. Since there should not be two correct answers to the same question, we c eliminate both df them.

We are left with choice B and choice D. After examination of the hydroxyl groups on the C-6 carbons of Figurc and Figure 5, we can conclude that they are one in the same. We can eliminate choice D. By default, we are left ni
choice B.

Copyright @ by The Berkeley Review

172

The Berkeley Specializing in MCAT

Biology
Glu 35
I

Metabolic Components
Main Chain of Lysozyme Enzyme
|

Section VII Answers

Main Chain of Lysozyme Enzyme

.c- O o tttrrrtrrttrtt
_

cru
H

ls

CH"OH
o
Rings lrttttt"ttttttttt'

'rt

o',Ct o o
Rings A-B-C,

t1'

A-B-C

NAM N-H

^-l u -L

NAM N-H
o_ ^._o

l^

CHr i
I

o=

Asp 52

ol^-_o f cHr

Main Chain of Lysozyme Enzyme

Main Chain of Lysozyme Enzyme

i I

nsn s2

However, by examining Figure 4 and Figure 5, you should be able to see how the hydroxyl group of the water molecule forms a bond with the C-l anomeric carbon, and how the remaining hydrogen of the water molecule forms a bond with the carboxylate oxygen of Glu 35. This is indicated by the dashed lines in the two diagrams shown above. The correct choice is B.

:/.

A is correct. General acid-base catalysis involves the transfer of

a proton in the transition state (see the protonated form of Glu 35 in Figure 2 in the passage). This transfer does not involve covalent bond formation.

A covalent bond between two atoms like C and H involves the sharing of a pair of electrons. When a C-H bond breaks, the electrons must do something. If one electron leaves with the hydrogen and the other stays with the carbon, the reaction is referred to as homolytic cleavage, and two radicals are produced (e.g., H. and C.). If both electrons leave with one atom, the reaction is referred to as heterolytic cleavage (e.g., .6.e and Ho or Co and '11'o). In Figure 3 of the passage, we see a carbocation at the anomeric position. The correct choice is A.
58.

C is correct. Glutamic acid is an acidic amino acid with a negatively charged side chain atpH7.4. The side chain is polar and tends to interact with other polar molecules (like water). We would expect this amino acid to be found on the outer surface of the enzyme, where it can be in contact with the aqueous medium. Note that in the diagram, the a-helical region is towards the outer surface of the enzyme. We would not expect glutamic acid to be buried in the interior of the protein, where the amino acid residues tend to be quite hydrophobic and nonpolar. Finally, as outlined in the passage, Glu 35 (glutamic acid) is part of the catalytic group at the active site of the enzyme. The correct choice is C.

:9.

denominator, Krra + [S] becomes Krr,r + [0], which is just Ky. What this is telling us is that the initial reaction velocity is almost proportional to [S] at very low values of [S]. In other words, at low [S] we observe a straight line in the graph in Figure 1 of the passage. This is characteristic of a first-order reaction. The correct choice is C.
'ultrt-

C is correct. When the substrate concentration is quite low, [S] is insignificant compared to Ku. In

the

B is correct. When [S] is quite high, it means that Ky is quite small. In the denominator, Ku + [S] becomes [0] + [S], which is just [S]. This reduces the Michaelis-Menten equation to the following:
v=

Vt^* [Sl=

vmax

tsl

In other words, at very high [S] the reaction approaches some maximal velocity (V.ur). Note that the velocity of this reaction is now independent of [S]. This is characteristic ofa zero-order reaction. The correct choice is B.
n,[-

D is correct.

If

[SJ = Krra, then substitution into the Michaelis-Menten equation reduces it to:

V.u* [S]

lsl

tsl 2tsl
173

V.u* [S]

V*u*
2

This expression is telling us that correct choice is D.


|,.'flr'!nsht @ by The Berkeley Review

Ky

is that [S] where the reaction operates at one-half its maximal velocity. The

The Berkeley Review Specializing in MCAT Preparation

Biology
62.

Metabolic Components

Section VII Answem

C is comect. This question can best be answered by considering the reaction of E and S. As E and S combine, [Sl and [E] both decrease in order to make the ES complex. Therefore, we would expect the [ES] to increase. As t]c [ES] begins to increase, some of it begins to get converted to P. As a result, the tpl begins io increase. The corrett choice is C.

E
C)

kr E+S:.ES-E+p
kz Time

ks

63.

A is correct. The reciprocal of the Michaelis-Menten equation is given below. This formula is called Lineweaver-Burke equation. Note that it is in the form of the general y-iniercept equation for a straight line: (y = + b).

t=/ru\ t + I v \v.u*/[Sl v-^,


l/v. The slope of the line is Ku/Vmax. The x-axis is l/[S]. The y-intercept is l/Vmax. How do we the value of Klt? In Figure I of the passage, Ku is a particular [S] at which the enzyme is half-maximally satt In this case, Ku is also a particular [S]. However, the Ku here is a reciprocal value (i.e., l/Krvr). And since we a quadrant that has a negative sign notation for the x-axis, we place a negative sign in front of l/Ky. It now l/Krra. All we need to do now is extrapolate the line from the y-axis to the x-axis as shown below.
The y-axis is

e^vapolation/

/i (e/
r2345
14sl
The rest is just algebra. We find that:

-4 -3 -2 -1 0
-

l/Kv

2 3 4

l{sl

l-.
Krr'r

K1a=-1 =+0.5
-2

The
64.

Krr,l value in Figure I of the passage is a positive value. Likewise, the also be a positive value. The correct choice is A.

Ku of the double reciprocal plot

is correct. In order to answer this question, we must consider the values for Ku and kcat in Table I in the it must mean that it is extremely efficient at what it does. A an enzyme's efficiency is given by kcat/Ku (from the passage). We also know that rate constant kt has I between 108 to lOe M-lsec-I. This is tetling us that if kr has these limits, then kt must have them as well amount of product that can be made is determined by how much of the ES complex there is available; and availability is determined by how fast E and S can come together. You do not ne;d a calculator to do this pr All you need to work with are the exponents.

c
If

an enzyme is (almost) catalytically perfect,

Copyright @ by The Berkeley Review

t74

The Berkeley Specializing in MCAT

Biotogy

Metabolic Components

Section VII Answers

Consider choice A: Carbonic anhydrase has a Kv =2.6x 10-2 M and a kqal = 4.0 x 105 sec-1.If we consider the exponents alone, we find that kcat/Ku = 195716-2 = 107 M-lsec-I. The actual value is 1.5 x 197 14-1r""-1. Chymotrypsin has a Kv = 6.6 x 10-4 M and a kcat= 1.9 x ld sec-1.If we considertheexponents alone, we find that k"^t /Kr,r = 192119-+ - 106 M-lsec-1. The actual value is 2.9 x 1gs 14-1t."-1. Therefore, for carbonic anhydrase, the efficiency is about 197 14-1t."-1. For chymotrypsin, the efficiency is about 196 14-1t""-1. These exponents (107 and 106) are not too far away from the limits of 108 to 199 14-1r".-1. Therefore, this is a pretty efficient enzyme pair. But we do not know if it is the ruosr efficient pair.

ConsiderchoiceB: AcetylcholineesterasehasaKlr=9.5x10-5Mandakcat=1.4x104sec-1. Ifweconsiderthe exponents alone, kcat /Krr,r - 104i10-5 - 109 M-tsec-l. From choice A, we know that the efficiency for carbonic
anhydrase is 107 M-lsec-1. These two sets of exponents (109 and 107) are even closer to catalytic perfection.

ConsiderchoiceC: FumarasehasaKu=5.0x 10-6Mandak,r.r1 =8.0x 102sec-l.Therefore,kcat/Ku -l}2lrc'6 168 14-1r""-1. From choice B, we know that the efficiency of acetylcholine esterase is 109 M-lsec-I. So far, these two sets ofexponents (108 and 109) appear to be the most effictent.

ConsiderchoiceD:PepsinhasaKl,,r=3.0x10-4Mandakqnl=5.0x10-lsec-l.Thekcat/Ku=16-1719-4,whichis 193 14-1t..-1. From choice A, we know that the efficiency of chymotrypsin is 196 14-1r""-1. These two sets of
exponents (106 and 103) make this enzyme pair the least efficient. The correct choice is C.
b5.

A is correct. According to the


1ge 14-lr-1.

passage, the rate constant k1 (for the reaction shown below) has limits between 108 to

E+s--I!ES
k2

kr,h,+p

This value is predicted from diffusion theory and represents a very extreme case, where (k: >>> k1) and every substrate that collides with an enzyme is converted into an ES complex. This means that the efficiency of the process is determined by how fast a substrate can be placed into the active site of the enzyme (forming the ES -omplex). Having limits between 108 to 169 14-lt-1*"ans that the enzyme is near catalytic perfection. Since triose phosphate isomerase has an efficiency of 2.4 x 108 M-ls-1, it means that this enzyme is almost perfect. There is little reason for it to change. This leads us to choices A and B.
Now, did this change occur early or late in evolution? The passage does not help you with this part of the answer.
Common sense will. Organisms are believed to have gained the ability to respire (use Oz) some 2 billion years ago (the numbers are not important here). Multicellular organisms are believed to have appeared some 700 million years ago. Humans are multicellular organisms; yeasts are not. Long ago (early in evolution), there was a divergence in our ancestral relationship with yeasts. At that divergence point, the isomerase enzyme was passed to each organism. The enzyme must have been quite catalytically efficient at that time; otherwise, both lineages might have gone the way of extinction. Since each lineage did not become extinct, the same enzyme (essentially) is present today. If the maximal efficiency of the enzyme came about late in evolution, we would expect that one lineage might have a nearly perfect enzyme, while the other lineage would not. This is because of different environmental forces acting
on the two species.

Even though both choice C and choice D are incorrect (primarily because of the first part of the statement), let's briefly look at the last part of choice D. The isomerase enzyme does have the ability to change through amino acid substitution (most likely through point mutations or single base changes). If a mutation is to be passed on to future generations, it must allow the organism to survive and reproduce. Organisms do have the ability to change. The correct choice is A.

66.

B is correct. ATP is the major carrier of chemical energy in all living cells. Many of the reactions in which ATP is an intermediate involve a phosphoryl-group transfer from ATP to another molecule, or from an energy-rich molecule to adenosine diphosphate (ADP) to form ATP. The simplest example of this type of group transfer reaction can be taken from Table I in the passage. This table summarizes the phosphoryl-group transfer potentials of hydrolysis of some important compounds in metabolism. If ATP is hydrolyzed to ADP and Pi (inorganic phosphate), water acts as the phosphoryl-group acceptor. The standard free energy of hydrolysis (AG") for this reaction is -30.5 kJ/mol. This value lies between that of the hydrolysis of phosphoenolpyruvate (-61.9 kJ/mol) and the hydrolysis of
glycerol-3-phosphate (-9.2 kJ/mo1).

ri right

by The Berkeley Review

t75

The Berkeley Review Specializing in MCAT Preparation

Biology

Metabotic Components

Section VII Answers

Molecules that have a phosphoryl-group transfer potential greater than that of ATp (e.g., phosphoenolpyruvate) transfer a phosphoryl group to ADP to form ATP. Molecules that have a phosphoryl-grJup't.unif"r potential lesi than that of ATP (e.g., glycerol-3-phosphate) acquire a phosphoryl group from ATpio Torm ADp and the molecule being phosphorylated. Which of the compounds listed in the answerihoices can be phosphorylated exergonically by ATP? We know from general chemistry that reactions that are exergonic are spontaneoui und proceed with a rellas! of free energy. In this case, the change in free energy (AG) is negative, because the products of these types of reactions have less free energy than the reactants. Reactions that are endergonic are not spontaneous, anO ttrey require an input of free energy. The AG for these reactions is positive, becaus6 the products huu" free energy -o." than the reactants. We are therefore looking for compounds that have a Lower AG" value than AT-f. Phosphoenolpyruvate, acetl phosphate and pyrophosphate all have AG"' values that are higher than ATp, allowing us to eliminate choices A, C, and D. Glucose- I -phosphate has a AG" value of -20.9 kJ/moj, which is clearly low& than the AG" value for the hydrolysis of ATP. The correct choice is B.
67-

C is correct. Molecules can move across cell membranes by several processes. Diffusion is the net movement of molecules from a high concentration to a low concentration. Osmosis ii the net diffusion of water from a region d low solute concentration (i.e., high water concentration) to a region of high solute concentration (i.e., lorri water concentration). Passive transport (i.e., simple diffusion) does not require carrier molecules or an expenditure energy' but rather is the net movement of molecules down their concentration gradient across a membrane. Acti transport is a carrier-mediated process that requires the input of cellular energylnd makes possible the transporr molecules across a membrane against their electrochemical gradient. Onl-y active t.unrport works against t concentration gradient and requires an input of energy in the form of ATP. Ciroices n, S, and D are incorrect. correct choice is C.
D is correct. Any compound that inhibits ATP production, such as cyanide in the electron-transport chain, is c it the cell'i metabolic processes slow down and the eventually dies. A human being has an energy reserve of about four minuGs of ATp and other phosphe compounds in every cell. This is one of the reasons brain damage is likely to occur after someone has been withorl heartbeat for about four minutes and then is revived. The cell is set up to use ATP, so it cannot suddenly switch other phosphate compounds and survive indefinitely. Choices A, B, and C are incorrect. The correct choice is D.
a poison. ATP is the cell's energy culrency, and without

68.

69.

A is correctt If the phosphoanhydride bond in any one of the answer choices is hydrolyzed to yield the prod compound plus Pi, the energy release is in the form of heat. For example, if tli,e phtsphoaniydride bond phosphoenolpyruvate is hydrolyzed to give pyruvate and Pi, the standard frei-energy .trung" of hydiolysis is kJ/mol. If the hydrolysis of an energy-rich compound releases more free energy t[an the -hydrolysis of A1'p, thguq! phosphoryl-group transfer
hydrolysis of This means that the hydrolysis of an energy-rich compound can be coupled to the synthesis of ATp. We can see in the coupled example shown below:
i-tre

ATP. Note that the reaction for the phosphorylation of ADP is the reverse of the ieaction tor

these energy-rich compounds can transfer

phosphoryt

lroup to ADp

to

ooo. o-U-no \"/ | ' 6^ +Hro + it .c(7i


/\ H' 'H
Phosphoenolpyruvate

cOO

AG-

t-o *
cHr

no-lJ-oo
o
I

Pyruvate

Phosphate

ililo Adenosine-O-P-O-P-O -

oo lt oo
1'

o
+

oo

"o

-!-oo
o
I

----------------

Adenosine-O-P-O-P-O-p-O

ttt ooo

ilililo

ooo

HnO

+7))

ooo

Adenosine diphosphate (ADP)

Adenosine triphosphate (ATp)

Phosphoenolpyruvate

+ ADP +

Pyruvate

ATp

The compound among those in the answer choices that releases the most energy upon hydrolysis would also the most heat after the phosphorylation of ADP. This compound is phosphoenolpyruvaie. Choices B, C, an all lower in energy released upon hydrolysis and are incorrect. The correct choice is A. Copyright @ by The Berkeley Review

176

The Berkeley Specializing in MCAT

Biology
70.

Metabolic Components

Section VII Answers

C is correct. Of the metabolic processes presented to us, three generate high-energy phosphate compounds (i.e.,
ATP or GTP), and one does not. We are looking for the one that does not. Three of the metabolic processes involve the oxidation of fuel molecules: glycolysis, citric acid cycle, and oxidative phosphorylation. All of these pathways produce either ATP or GTP. These metabolic processes are catabolic, and they release energy in the form of ATP and heat. Protein synthesis, being anabolic, requires an enormous input of energy (in the form of GTP)-possibly as much as 90Vo of the chemical energy used by a cell during the course of its biosynthetic reactions. Choices A, B, and D are incorrect. The correct choice is C.

71.

B is correct. The bond indicated by the arrow in the ATP molecule, between the 5' carbon of the ribose ring and the o-phosphate, is a phosphoester bond. There are two phosphoanhydride bonds. One is located between the a- and Bphosphates, and the other is located between the p- and y-phosphates. Eliminate choice A.

ypGbbndl

Phosphoester

NH:

-o ^o ^o /
T
"J

""_i_,_i_"_i-"{:-,, Jl jfl-i{ L-<"> /- \N-crY'*idi' u "'+ ll I t


? Phosphoranhvdride
!"
The only type of glycosidic linkage found in ATP is an N-glycosidic linkage, located between the I' carbon of the ribose ring and the N-9 nitrogen of the nitrogenous base. Eliminate choice C. Peptide bonds are formed between an N-terminus nitrogen and a C-terminus carbon of amino acids. Eliminate choice D. The correct choice is B.

z)-fl \,3J\"-,

72,

B is correct. Pyrophosphate (PPi) is a compound that contains two phosphate residues, and depending upon the pH of the solution, can exist in several ionic forms. For example, there are times when the molecule might bear two negative charges, and there are times when it might bear three or four negative charges. The electrostatic repulsion between the negatively charged oxygen atoms of PPi provide a portion of the driving force tbr its hydrolysis to two molecules of inorganic phosphate (Pi). Since PPi is rather unstable, statement I is inconect. Based on the values for the standard free energies of phosphate hydrolysis in Table I in the passage, we see that more energy is released for the hydrolysis of ATP to AMP and PPi than for the hydrolysis of ATP to ADP and Pi. Statement II is correct. Since PPi can bear a variety of negative charges in the cell, it is quite soluble. Statement III is theretbre incorrect. The correct choice is B.

11

B is correct. The pH of the cytoplasm is approximately 7. These lysosomal enzymes would be virtually inactive in the cytoplasm. This is a protective mechanism, so that a rupture in a lysosome does not lead to the destruction of all intracellular components. Choice A is incorrect. The acidic or basic nature of the proteins does not matter. They are all degraded with the same degree of efficiency. Choices C and D are incorrect. The correct choice is B. C is copect. A region of DNA is considered to be highly conserved if it is very similar in the genetic material of two or more organisms. That means choice A is incorrect. The phyla have nothing to do with conservation in this question. Choice B is incorrect. "Highly conserved" means virtually identical, while "completely conserved" would mean completely identical, so choice D is inconect. The correct choice is CC is correct. Ubiquitin attaches most quickly to those enzymes that are at metabolic control points in a biochemical reaction. Some pathways operate pretty much constitutively (all the time), such as glycolysis, the Krebs cycle, the electron-transport chain, and oxidative phosphorylation. Conversely, your body closely regulates pathways involving gluconeogenesis, amino acid catabolism, nucleic acid synthesis, or the synthesis of spermine and spermidine, two polyamines used in DNA-packaging. Cytochrome c ig a peripheral membrane protein found on the side of the inner mitochondrial membrane that faces the intermembrane space. This protein passes electrons from Complex III of the electron-transport chain to Complex IV. Under normal cellular conditions, this enzyme has a half-life of about 150 hours and, as indicated in Table I in the passage, is a long-lived enzyme that has a stabilizing N-terminal amino acid residue. It turns out that the Nterminal amino acid in human cytochrome c is glycine (Gly). Since the ubiquitin system is used to destroy abnormal proteins and short-lived enzymes, we can eliminate choice A.

'{.

:r right

@ by The

Berkeley Review

t77

The Berkeley Review Specializing in MCAT Preparation

Biology

Metabolic Components

Section VII Answers

Glyceraldehyde-3-phosphate dehydrogenase is a glycolytic enzyme involved in the conversion of glyceraldehyde-3phosphate to--1,3-bisphosphoglycerate. Its long half-life of 130 hours indicates that the N-terminal amino acid has a stabilizing effect. Eliminate choice D.

The remaining two proteins are both short-lived enzymes. Tyrosine aminotransferase has a half-life of l2O minutes and is involved in the catabolism of the amino acids phenylalanine and tyrosine to acetoacetate and fumarateOrnithine decarboxylase has a half-life of about 12 minutes and is involved in the synthesis of spermine ard spermidine, two polyamines used in the packaging of nucleic acid. Even though uoin of these pioteins havc relatively short half-lives, and they each have a destabilizing N-terminal amino acid,"we want to select the one that ir most likely to be modifiea by ubiquitin. The protein we want is the one with the shorrest half-life, ornithine 9u19_ttv decarboxylase. A short half-tife usually means that a molecule is being turned over quickly, and in the case of rapidly dividing cells, ornithine carboxylase is required in large amounts. With such a short half-life, UCDEN ir probably degrading ornithine decarboxylase just as fast as new proteins are being synthesized to replace it. ThG correct choice is C. 76.

B is correct. Look at Table I in the passage. The stablest enzyme has the longest half-life. Lysine and leucine have half-lives of about 3 minutes. Eliminate choices A and b. Glutamic acii has a half-life of abour 30 min Eliminate choice C. Glycine has a half-life of more than 20 hours. The correct choice is B.

77.

A is correct. During a fast, amino acids are still needed for the repair and synthesis of proteins. Since the person i not eating, these amino acids must come from thebody's storedprotein, ihe muscles. Following a meal, diet4 amino acids are readily available, and lysosomal degradation of proteins should be low. Choice B is incorre During exercise, some proteins may be degraded, but not ur rnuny as are degraded during a fast. Choice C incorrect. During pregnancy, women are usually well-fed and not fasiing. ChoicJD is incorrec"t. fhe correct is A.
phosphoanhydride bond between the B- and y-phosphates of ATP. Th; pyrophosphate thus formed is rap hydrolyzed to two molecules of P1 (inorganic phosphite). In this reaction t"qu"n"", i*o prrorpr1oanhydride have been hydrolyzed. choices A, B, and D are incorrect. The correct choice is c.

78.

C is correct. The conversion of ATP to AMP and PPi (pyrophosphate) involves the hydrolysis of

79.

enzymes requires a pH of approximately 5. Chloroquine, in the unch form, diffuses across the single membrane of the lysbsorne and accumulates inside. Due to the acidic medium i the-lysosome, chloroquine becomes protonated and begins to accumulate in the charged form. This increases the within the lysosome, leading to inactivation of the enrym"s that require a tow optimum and a subseqr iH decrease in protein degradation. Choice B therefore can be eliminated. Since there is no indication in the pa! that chloroquine modifies the active site of a protease, we must assume that it does not, so choices C and D can be eliminated. The correct choice is A.

A is correct. The action of lysosomal

80.

81.

remain in the ash portion. The correct choice is D.

is, correct. Melting is the change of a solid to a liquid, so choice A is incorrect. Vaporization is the change liquid to 3 vapor phase, and that is not what is involved in freeze-drying. Choice B is therefore incu Condensation is the change of a vapor to a liquid or a solid. Choice C is incotect. Sublimation is the change solid to a vapor, which is an accurate description ofthe freeze-drying process. The correct choice is D. D is correct. In the third-paragraph of the passage, we learn that vitamins are organic compounds. Therefore" can be combusted to yield COz and H2o, among other gases. Minerals are just in6rganic ions and atoms and q

82. A is correct. In the second paragraph of the passage,


83.

we find that magnesium is not a trace mineral; macromineral. Zinc, manganese, and iodine are trace minerals. The correct choice is A.

il

84.

A is.correct. According to the last paragraph in the passage, a protein is about 16To niftogen The correct obtained by dividing 5 grams nitrogen by 167o n_itrogen per sample of protein, which givJs 31 grams of p answer for choice B is obtained by multiplying 5 grams by 16To. This gives 0.8 grams"of protein, which is i Choices c anu n are the same answers mutiipfi"a Uy ro*" power of to] rne coriect choice is A. A is correct. The calorie content of food is another way of saying its energy content. Only proteins, carboh and lipids provide energy. Statements I and III are correct, which means thit choice A is also correct. Vita not provide energy themselves, although they are involved in metabolic reactions. This means that incorrect, which allows us to eliminate choices B, c, and D. The correct choice is A.

Copyright @ by The Berkeley Review

t7a

The Berkeley Specializing in MCAT prel

Biology
85.

Metabolic Components

Section VII Answers

C is correct. Choices A and B make you think about solubility. The blood is an aqueous medium. Water-soluble
vitamins could travel freely in it, while fat-soluble vitamins (like fats) would need a transport protein. Excess watersoluble vitamins would be excreted into the urine by the kidney. Vitamin K is important for blood clotting, so an antagonist would decrease the ability to clot, leading to longer clotting times. Choices A, C, and D are thus all true. The false (and best) answer is choice C. Ascorbic acid (vitamin C) is water-soluble. The correct choice is C.

86.

B is correct. In the first paragraph of the passage, we learned that a triglyceride contains 9 kcal/gram. 30% of 1800 kcal = 0.30 x 1800 = 540 kcal. 540 kcall9 kcal per gr&rn = 60 grams triglyceride. The answers in choices A, C, and D are the results of various incorrect manipulations of the data. The correct choice is B. D is correct. We are looking for the answer that does not produce water. The last step of the electron-transport chain uses electrons to reduce oxygen, producing water. Choice A is true. Synthesis of protein or glycogen involves condensation reactions and the production of water. Choices B and C are true. The breakdown of a triglyceride requires the use (not the production) of water in a hydrolysis reaction. Choice D is the false answer, because it does not produce water. The correct choice is D.

87,

88.

C is correct. This question cannot be answered based on information in the passage and requires previous knowledge. The release of fatty acids from glycerol is regulated by the actions of insulin, epinephrine, and glucagon through a reaction catalyzed by an enzyme called hormone-sensitive triacylglycerol lipase. Adipocytes hydrolyze triglycerides (i.e., triacylglycerols) to free fatty acids and glycerol, as shown in the reaction below;

O H,C_O_ H,C- OH C_ R "l 'l lt .3 H,o R-c-o-c-H Ho-c-H o -------:--------ll I I Linase ' H?C-OC- R
H2C- OH

o il

3 HO- C-R
Fatty acid

o il

Triglyceride

Glycerol

One of the intermediates in the synthesis of triglycerides is the molecule glycerol-3-phosphate. If the concentration of this molecule is low in the adipocyte, the free fatty acids produced in the reaction shown above are released into the bloodstream. They are not reesterified to triglycerides. The ratio of glycerol to fatty acids would therefore be l:3. The correct choice is C.

!p.

B is correct. The isotopically labeled molecules Iisted in the question are not radioactive. Therefore, scintillation counting or using a Geiger counter would not work. Both l3C and 2H ate stable isotopes of their parent atoms. Choices A and C are incorrect. The only difference between l2C and l3C is one neutron in the nucleus. The same is true of hydrogen and deuterium (2H). The molecules can be separated by mass only, using a mass spectrometer. Choice B is correct. A UV spectroscope would not help the situation either, since mass is the only critical piece of data to gather. Choice D is incorrect. The correct choice is B.
so that glycerol is not charged. Therefore, glycerol would not interact with either the cation exchange resin (where the beads are negatively charged) or the anion exchange resin (where the beads are positively charged). Glycerol would pass through the apparatus and end up in the water in the collection tube. Choice A, B, and C are incorrect. The comect choice is D.

',l'l],

D is correct. The pH is adjusted to 7.0

A is correct. Since this cycling is well-regulated and operates almost 24 hours per day, it is doubtful that the normally cycling quantities of fatty acid damage either the liver or the adipose tissue. Choice C and D are incorrect. The cycle does, however, require energy to operate. It is a futile cycle, one that operates in a circle but uses energy. Choice B is incorrect. The correct choice is A.
D is correct. The turnover of fatty acids and glycerol means they are entering the bloodstream and exiting the bloodstream. Turnover is a measure of how rapidly this entering and exiting occurs. In the pre-niacin state, you "an see that lipolysis of triglycerides in the adipose tissue must be occurring, since there is a non-zero value ior both fatty acid and glycerol turnover. Choice A is incorrect. You can also see that some fatty acids are being reesterified, even in the pre-niacin state. If there were no reesterification, then the fatty acid turnover value would be three times the glycerol turnover value. Since this is not the case (1.3 x 3 = 3.9), then there must be some reesterification in the pre-niacin state. Choice C is incorrect. Niacin treatment did not increase lipolysis, since the glycerol value did not change in the pre and post states. Only the value for the fatty acid turnover decreased, so the change was in the reesterification of fatty acids. Choice B is incorrect. The correct choice is D.

': " nght @ by The Berkeley Review

179

The Berkeley Review Specializing in MCAT Preparation

Biology
93.

Metabolic Components

Section VII Answers

C is correct. These free fatty acids are crucial for use as energy during rest and during exercise. The hean preferentially uses free fatty acids as fuel, as does the skeletal muscle at rest. This drug probably would be called a poison, since a large dose of it would mean death. Anyway, the body could not switch to metabolizing ketone bodies, since they are formed from fatty acids. Choice A is incorrect. The brain and nervous tissue rely primarily on glucose as fuel, so choice B is also incorrect. Cholesterol does not contain usable energy for human beings, so choice D is incorrect. The correct choice is C.
A is correct. Hormone-sensitive lipase hydrolyzes triglycerides into glycerol and fatty acids. These are both released from the cell. Blood levels of both would increase following the ingestion of caffeine. Choices B, C, and D are
incorrect. The correct choice is A.

94.

95.
96.

D is correct. Equilibrium is a stable condition, a state where no further net change is occurring. In a living syster4 this condition means vital reactions are no longer netting vital products. Without the expenditure of energy and rhe net production ofvital products, the organism will decay and approach death. The correct choice is D.

D is correct. The question describes the relationship between time and distance traveled when molecules ac diffusing through a medium: If a molecule requires I second to travel I micron, then it requires 9 seconds to travel
microns, and 16 seconds to travel 4 microns. From this information, we can see that as the distance increases. efficiency of diffusion decreases. Therefore, diffusion is inefficient over long distances, but efficient over distances. The correct choice is D.

97.

C is correct. We are told in the passage that it is the frequency of collisions between substrate and enzyme thar the rate-determining step in diffusion-limited reactions. It should follow that if we can bring the substrate and enzyme closer together, we can increase the rate of reactions. A multienzyme complex is exactly what its implies, a group of enzymes involved in a reaction pathway. By bringing the enzymes close together, we de
the distance one product has to travel to the next enzyme to continue the reaction chain. By decreasing the di we increase the frequency ofcollisions and increase the rate ofreaction. The correct choice is C.

98.

A is correct. The question informs us of a membrane-bound compartment that occupies 20Vo of the total volume. If we put all of the substrate into this compartment, its concentration could be as much as 5 times
than it is in the cytosol. How do we arrive at this figure? Let us assume that the concentration in the cytosol is I to I unit of volume. In the compartment, the volume is reduced to 0.2. Now, the concentration is I unir solute to 0.2 units of volume, which gives us a concentration that is 5 times greater than in the cytosol. The choice is A.

of solute

99.

C is correct. This answer can be arrived at simply by looking at the time it takes for these molecules to hir target. In Trial C, it takes 9 seconds for the molecule to find its target. If we convert 9 seconds into minutes. sr 0. l5 minutes, so the frequency of collisions in Trial C is 100 times greater than in Trial A. The correct choice ig

100.

D is correct. This answer can be arrived at by thinking about movement in different dimensions. Recall
membrane3 accelerate diffusion-limited reactions by limiting movement to only two dimensions. The same ki thinking can be applied to DNA-binding proteins, as they diffuse through the nucleoplasm in search of their bi site(s). It is not efficient for the binding protein to jump onto and off the DNA molecule at random places to binding site, because it might miss something. It is more efficient for the binding protein to limit its move

going up and down the DNA molecule until affinity. The correct choice is D.

it finds its appropriate binding site, indicated by a strong

bi

Copyright @ by The Berkeley Review

rBo

The Berkeley Specializing in MCAT

A.
,t,, ','.'

Metabolic Pathways

:'i; ' ,,biSaiihuride

Metabolism
t,,.,,,'.

::,,,,i,r:,,,ri.,,: ,::,

Metabolic Pathways

1.,.,'',.
,1

1.,

5;,',.,,fhe.Kgehs'Cyile,., rl

,,..,

r:,.,.,,,,,,.',t'''r0*idativeP'hosbhorylati, ..',',.', 6.,,,, Pen@ pho$ghate,Faih#a y.,,,,,,,,,,,. l,:,t.,,t.,:7'"rt,,.Glwbneogenesis ''lrr.'.''*rl


....''.:l.::$;.:::.:l.sa*'5r'Aci,do.xidation:;.l''',,;;:'...

g.

The Urea Cycle

.
.,,,,,
1,:

eg',Pa$sgget,,,*-'O**.*t"

:)t:

.:..1.

uzo_-ll7

11:1;l|:ffi1*l-El ':4tt ,t:t.::

ti.

.Fumat'ate,".r.

raoHl*$41',,,,,,,;'
FAD]I -,/.11i:: ir".r.
:'

",'r

9?ttYi,.'. :tr;"6

-4\ -r\ '}NADH+H+


:., ':,'

.&

'rr4rNtiDt, +rebA
:
,'-.

,{9?,.,t'.:'

.,,

., r'

,. ',

$$a

n
R'.

arvr;'I;h'i w*

Specialiiin$ in' MC$'Preparation

Metabolic Pathways
Top lO Section Goals
Understand the behind the

Glycolysis is one of the pathways central to metabolisnr. You should have

;f'h;i"= it

operates, ivhut

ii

generates, and how

it is tied in with the Krebs cycle'

a basic

understanding

that can be metabolized. of Be famillar with the common Suears other than glucose can be used for metabolism. Sucrose, lactose, fructose, & galactose are Uu? a few sugais that are commonly metabolized by a wide variety of organisms.
Be able to link the concePts of glycolysis with tlre l{rgbs cycte. Once the commitment to aerobic respiration is made, pyruvate is converted to acetyl-CoA which then enters the Krebs cycle. ThJs link unites glycolysis and the Krebs cycle and is important.

of the Understand the im NADH is generated in both glycolysis and

NADII

ET

NADII2.

the Krebs rycle. |AD_H2 is generated.iljl" Krebs cycle. are important for the generation of ATP. c6mponents reduiing Understan? how these

"3

Know hory the electron transport chain operates. It is important to have an understanding of how electrons flow from NADH and FADH2 down the
electron transport chain to oxygen, the rfltimate electron acceptor.

'v
ov
^% - V t(

Understand the chemiosmotic hypothgsis and the concept of a proton gradient.


Understand how a proton gradient is generated, and how it leads to the eventual synthesis of ATP in oxidative phospfiorylad6n. Be famiiiar with inhibitors and how they work. Be

familiar with the general concepts of the pentose phosphate pathway.

Do not memorize the structures of the pentose phosphate pathway. Just understand why this ^also found in the Calvin iycle. pathway is important. Many of its component} are

Understand the relationship


Be aware that gluconeogenesis is not the complete reverse oi glycolysis. There are irreoersible step*

which must be"bypassed. Know the importance of gluconeogenesis in metabolism.

"v

Know the concepts involved in the p-oxidation pathway.

thiJ pathway that ilhistrate the importance of undersianding organic


Be familiar with the urea cycle and its relationship to the

Fatty acid oxidation is important in metabolism. There are many classic biochemical reactions:n

chemistir--

lite. LE44ey:-

reactions. Understand how nitlogenous w-astes are removed from the body and in what for:n',

Biology

Metabolic pathways

Glycolysis

cl#,c
Phase

I l
I

put to immediate use by :xamining the reactions in glycolysis. The first st?p - giirryrrr"is shown in Step 1 in Figure 8-1. The most common and wide-spread enzyme that makes use of qlucose when it enters the ceil is hexokinase. A kinase is an enzyme that invorves :he transfer of a termina.l phosphar"-g;o"p of an ATp ,.,it to some other ;ompound. In this case.it is b-grucos". ivtr"" ATp transfe* ii, phorphate onto D-glucose it does so atthe C-o p"osition. we will end up with a compound calred clucose-6-phosphate. Note that we have usea r arp. rr,i, i, ur., inaestment step and is an important contror point in giy"otysir. why did we bother to attach a :hosphate onto glucose in the first pra?& Gr,r.oru dols not il";;"y charges and :an pass back and forth across the cel's membrane reratively eas'y. However, rnce it is phosphorylated,it picks_up some negative charges and can no longer sass across the cel|s membrine. Grucose, in.th"e.form ,r gi".;-6_phosphate, is inside the cell. The AGo, for this ,uu.tior, in =apped Step 1 is _4.0 kcal/mol. Ho \ /tC

The ideas that we have recentry been considering can be

HO \ az-

,
HO

H_C_ OH
I

AGo'= -4.0
Hexokinase

C
I

AGo'= +0.4
OH

H.C-

C_H
I

HOOH

Phosphogluco isomerase
HO

t' c=o
I

CH"-OH

C.H
I

H.CI I

H.C_ OH cH'-oH
Glucose

ADP

H.CI

OH

Step 2

C.H
I I

H.C_ OH
H-C_

Step

H-C_ OH
I

cH2- o- P_ O

-o
I

OH O IrrO cH2- o_ P_ O
I

n
I

Glucose-6-phosphate

Fructose-6-phosphate

Figure B- I

reto sugar). The AGo'for

step 2 in glycorysis irwolves an equilibrium between two isomers, glucose_6:hosphate and fructose-6-phosphatd. rr," reuction takes prace at the C-l and C_2 :arbons of glucose-6-phosphaie. and proceeds through an enediolintermediate. lhe compound that emeiges from ihis reaction is- Fructose-6-phosphate as ;hown in Figure g-1. Thelnryme involvedhere is called phosphoglucose isomerase. Fructose-6-phosphate is the phosphorylated versio'n-of D-fructose (a

thii reaction is +0.+ kcal/mol.

ir;allmol

general.class of enzyme involved here is :iif:t:flf:Y.'_'::l* a transferase ructokinar:. enzyme cataryzes the conversi;Hll":r';r"-T Ti: phosphate to Fructose-$6-diphosplate.
called pho sphof
as

Step 3 involves a second inrsestment of an ATp molecure and is yet another control

shown in Figure g_i.

The

lGo' for this reaction is -3.4

opyright @ by The Berkeley Review

185

The Berkeley Review Specializing in MCAT freparation

Biology

Metabolic Pathways
o ilo - o- P- o
OO

Glycolysis

AGo'= -3.4 cH2 cH2-oH t"t'l Phosphofructo C= O C= O I kinase HO- C.H HO- C-H a.'ip o" io* o"
I

tl "_l-

H-c-

oH I . cH2- o- P- o

Step

H-c-

"_l-

oo

cH2-

oH I

o- P- o oo

Fructose-6-phosphate
Figure 8-2

Fructose- 1,6-diphosphate

In Step 4 fructose-1,6-diphosphate undergoes an interesting reaction in which bond between the C-3 and C-4 carbons is broken. This reaction involves a aldol condensation and the products are dihydroxyacetonephosphate ( and glyceraldehyde-3-phosphate. DHAP and glyceraldehyde-3-phosphate the two simplest and smallest carbohydrates. Both are trioses. If DHAP were react with glyceraldehyde-3-phosphate, forming fructose-1,6-diphosphate, reaction would be called an aldol condensation. The enzyme involved here aldolase. It is in the class of enzymes referred to as lyases. The AGo'for reaction is +5.7 kcal/mol as shown in Figure 8-3.

AG" = +5.7

^, Aldolase

C=O
|

OO

cH2-oH

cH.o-P 'l
Fructose- 1,6-diphosphate

Iilo

-o oo

Glyceraldehyde-3-phosphate

Figure B-3

In Step 5 DHAP can be converted into glyceraldehyde-3-phosphate. compounds are isomers and the reaction is catalyzed by triose p isomerase. All of the other significant reactions in glycolysis proceed glyceraldehyde-3-phosphate. Even though about 96% ol the of the phosphate molecule at equilibrium is DHAP, once glyceraldehyde-3is removed, DHAP is isomerized to glyceraldehyde-3-phosphate (recall ChAtelier's principle). In other words, from one molecule of fructosediphosphate you can get two molecules of glyceraldehyde-3-phosphate. AGo' for this reaction is +1.8 kcal/mol as shown in Figure 8-4.

Copyright @ by The Berkeley Review

ta4

The Berkeley Specializing in MCAT

Biotogy
o rr O cH2- O- P- O rl C=O O^ r s,,,
cH2-oH

Metabolic Pathways
AGo = +1.8
Triose Phosphate
isomerase

Glycolysis

c
H-C-

Step 5

cH2-o - P-O

OH IrO

O
I

DHAP fqgure 8.4

oo
Glyceraldehyde-3-phosphare

--::his point we have completed phase I of glycolysis. we have converted the 6 -::rons the original glucose molecule into 2 three carbon phosphorylated -of ::ermediates. Because these two phosphorylated intermediates are intercon'-:.]ble, we have a "pool" of triose phospirates. you should remember, we i::;ally have millions and millions of glucoie molecules forming twice as many r:i 'ions and millions.of triose phosphate molecules. up to this point the o.r".uil r-iation states of the carbons has remained constant. There have been no '-:ation or reduction reactions.

Fhase II - rr: now consider Phase II of glycolysis. In step 6 we will see a change in the '-'alion state of the C-1 carbon of glyceraldenyae-e-pnosphate. The oxidation r;;;j of the C-1 carbon on glyceraldehyde-3-phosphate is +t. The enzyme s ''r ;eraldehyde-3-dehydrogenase will convert giyc"tutd"nyde-3-phosphate'into .r-Diphosphoglycerate (abbreviated as 1,3-Dpt). Note that the C-l carbon of
We have a change in oxidation states form *i :: +3. This is a tzao electron oxidation. when you look at 1,3-DpG, note the : t:; ;r.thyd-ride linkage and the monophosphoestei linkage. How did this mixed

:-DPG has an oxidation state of +3.

,;1j6lg linkage get there? we needed a phosphat6 in order to make this and that phosphate comes from inorganic phosphate (p) and not from l1 *rlF, llhis inorganic phosphate and glycerataenyae-e-phosphate have to be put ri';*::.er. There must also be an oxidizing agent th-atwill-oxid-ize the C-l carbon of I :-:aldehyde-3-phosphate. This oxidizing agent is NADo. The AGo'value for r' .* :.acfion is +1.5 kcal/mol (see Figure g-5).

.-g

':-r

Mixed Anhydride

AGo'= +1.5

Linkase
hat e

"\\

ii.'
I

G ly c e r dlde hy de - 3 - p ho sp

dehydrogenase

o. o- P-O \'^,

?
ll

H.C-OH O T lr, - o- !- ot oo
"

*31
H-C-

o
O
I

NAD* NIDH

+Pi

+H+

Step 6
;eraldehyde-3-phosphate
-:

cH2-o-P-O oo
1,3-Diphosphoglycerate

OH |ilO

&5
It. -',ed

l: : ::d a high phosphoryl transfer potential. If we were to carry out


9 by The Berkeley Review

anhydride linkage in 1,3-DPG is relatively unstable, thus giving that


the

la5

The Berkeley Review Specializing in MCAT preparation

Biology

Metabolic Pathways

Glycolysis

hydrolysis of 1,3-DPG, the A Go' would be about -11.8 kcal/mol. However, instead of using water in this reaction we will use ADP. The enzyme involved here, phosphoglycerate kinase, will not operate unless there is ADP present. The product of this reaction, as shown in Step 7 (Figure 8-6), is 3-phosphoglycerab and ATP. We now have a return on our investment in the form of ATP. In fact we have a return of 2 ATP's (recaIl we split our 6-carbon glucose molecule inb two 3-carbon triose phosphate molecules).

o. o-P-o \'-' ll

oo lO

AGo = -4.5
Phosphoglycerate
kinase

Yo H-C- OH
til

A o. t'c' o"
I

O
I

H-CATP

Step 7 oo 1,3-Diphosphoglycerate
Figure 8-6

cH2-o-P-O oo
3-Phosphoglycerate

OH O rilo

In Step 8,3-phosphoglycerate is converted to 2-phosphoglycerate by the phosphoglyceromutase (see Figure 8-7). The phosphate group at the position of 3-phosphoglycerate is not the same phosphate at the C-2 positi 2-phosphoglycerate. This is because the enzyme itself is aphosphorylated During an intermediate state the phosphorous from the enzyme gets the C-2 position of 2-phosphoglycerate while the phosphorous from the position of 3-phosphoglycerate gets transferred to the enzyme. An exchangp taken place. The AGo'for this reaction is +1.1 kcal/mol. [Note: Recall that we discussed hemoglobin, we encountered 2,3-bisphosphoglycerate (2,7 This molecule binds in the middle of the hemoglobin molecule and stabili deoxy state of that molecule so more oxygen is released to the tissue. It is d point in the cell that 2,3-BPG can be made. The enzyme that would cataly reaction would be 2,3-BPG phosphatase (phosphatases catalyze the hydrol;
phosphate esters).1

Figure B-7

In Step 9 we have the loss of water from 2-phosphoglycerate. This inr hydroxyl function at the C-3 carbon and the hydrogen at the C-2
Figure 8-7). The enzyme enolase catalyzes the reaction of 2-phosphogll phosphoenolpyruvate (PEP). PEP has an unstable high energy phosphate

Why? The phosphate moiety attached to the C-2 carbon is arrangement with the rest of the molecule because of the enol
Copyright @ by The Berkeley Review

in

la6

The Berkeley Specializing in MCAT

Biology
':

Metabolic Pathways

Glycolysis

:lectrostatic repulsion is quite pronounced in this situation. The AGo' for this ::action is +0.4 kcal/mol.
v'e hydrolyzePEP, we would get the enol of pyruvate plus pi. The enor form , ould be in equilibrium with the keto form. These are referred to as tautomers. :-iorvever, the equilibrium between the enol and the keto forms is very much in
:..r'or of the keto form. This is shown in Figure 8-8.

::ep 10 is the last reaction in glycolysis (see Figure 8-9). The enzyme pyruvate r,:inase will catalyze the transfer of a phosphoryl group from PEp to ADp and r---'e us pyruvate and ATP. Pyruvic acid may also be called 2-keto-propionic acid : a-keto-propionic acid. This phosphorylation is nonoxidatioe. Recall that when rDP is converted to ATP the AGo'is +7.3 kcal/mol. The high energy bond in IEP has a AGo'of about -14.8 kcal/mol. coupling these two reactions will give ,-- cverall AGo' of -7.5kcal/rnol for Step 10. Note that since we started with a 6,::ron glucose molecule we now have two 3-carbon pyruvate molecules. In this

oo o. t\-zo
cc
C

o \
-r>

o
L:

CH:
enol-Pyruvate

-r il+i

!H

r'=
CH.

keto-Pyruvate

Figure B-B
Enol-keto equilibrium with pyruvate.

::

we have actually made 2 ATP's (net gain).

oo
t\/

AGo'= -7.5
O

c-o*P-o ill
CH,
O^

CO ll I

-z-\ADP

Pyruvate Kinase

A o o-l\ zC
I

C=O
I

ATP

CH:
Pyruvate

?:osphoenolpyruvate

Step 10

''gure 8.9
-',' that we have finished glycolysis, we need to consider the overall ,..,rtption and generation of ATP. Recall that in Step 1 and in Step 3 we invested -TP each. Thus, our total investment is 2 ATP's. in Step 6 we got 2 ATP's back ' :. our investment (remember, we have two 3-carbon compounds at this :-:r. And it is in Step 10 that we have made a net profit of 2 ATP's from our
...,."1 investment.

'- . return to Step'6 in glycolysis (Figure 8-5). In order for this reaction to take r:. \ve used two molecules of NADo and we produced two molecules of i.DH + H@. Remember, we actually have two molecules of glyceraldehyde-3, r:hate at this point. In order to continue with glycolysis we must have some r :o restore the reduced NAD's back to their oxidized form. How do we
'-:.erate NAD@? '

:r:

dr several ways to do this andpyruaate is involved

in all of them. If we

, :'.'ruvate itself to regenerate the NADo, then pyruvate will get reduced.Look -- = C-2 carbon of pyruvate. It is in the keto form and the oxidation state is +2. = lid a 2 eiectron transfer, we would end up with a carbon that has a zero

..:aie as shown in Figure 8-10. The enzyme involved here is lactate


.

-:.:ion state. The bompound that we can pioduce from this reaction ls Llrogenase. This is one of the possible fates of pyruvate. in this way we can ::ze NAD and pay off our debt in Step 6.

: -:'

:ht O by The Berkeley Review

ta7

The Berkeley Review Specializing in MCAT Preparation

Biology

Metabolic Pathways

Glycolysis

I
il
rt*,:

oaa
C
I

o
.ro
Inctate
dehydrogenase

_o ().()
l0 H-CI

&

+2C=O
I

Pyruvate H*
+

CH:

^ZN

OH

CH:
Lactate

Figure 8-lO

The reaction in Figure 8-10 occurs in the cells of an organism when oxr-gmL
becomes a limiting factor. For example, during vigorous exercise actiae sl<el-,:,il tyuscle produces lactate. Lactate turns out to be a dead end in metabolism, and as converted back to pyruvate.

we will later discover, is transported by the blood to the liver where it

ls

Pyruvate will then be converted into glucose by a process calieJ


gluconeogenesis. Lactobacillus casei, a member of a family of bacteria that are u-.e,j in the preparation of yogurt, excrete lactic acid into the surrounding medium a_o a waste product. This is what gives yogurt its slightly acidic taste. The reacticrru

shown in Figure 8-10 therefore allows glycolysis to continue under anaerobis conditions. Note that lactate is still at the same oxidation state as glucose (Lactate: C3H6O3 x2 = C6H72O5,. Glucose: C5H12O5).

o.o o
C

j_

c=o
I

_Z\- i=o-n-i',
decirborylase I

Pyruvate iO,

dehydrogenase oH NADH
+

Alcohol

CH:
Pyruvate

H+

CH:

d i
CH:

Acetaldehyde

NraO*

Ethanol

Figure

B-

II

This is not the only way to regenerate NADo. Yeasts use a slight modificahc,n Instead of doing a direct reduction of pyruvic acid with reduced NADH, yeL.is first decarboxylate the pyruvic acid to acetaldehyde as shown in Figure 8-11. The enzyme that catalyzes this reaction is pyruvate decarboxylase (which contai:m thiamine pyrophosphate (TPP) as a coenzyme). Pyruvate decarboxylase is a lyase. Acetaldehyde will next react with the reduced NADH to form ethanol. enzyme involved in this step is alcohol dehydrogenase. This process, conversion of the sugar glucose into ethanol, is called alcoholic fermentation, ethanol, a waste product, is excreted into the surrounding medium.

-{

-1-

The Big Picture


Let's consider the overall picture of glycolysis. This is shown in Figure 8-12. way to become intimately familiar with this pathway is to follow a radioacti labeled carbon through each series of steps (1-10) that we outlined and see wh that label ends up on the end products. For example, if the C-1 carbon of glu were radioactively labeled (designated as *C-1), where would that label
Copyright @ by The Berkeley Review

la8

The Berkeley Specializing in MCAT

Biology

ll

Metabolic Pathways
will do it for a *C-1

Glycolysis

iJ it does at all) inpyruaate,Iactate, or even ethanol? We ,abeled glucose molecule as shown in Figure 8-19.

CHo rll H-C- oH I


: "

Hexokinase
/ \ nu-

CHo H_c- oH ^i
L-n

* Cuz_gr-r Phosphogluco i= o Isomerase i


=: HU- L-H
H

| H.C- OH I|il_Iil cH"-oH

H-C- OH

t/\t

ATp .^,.

nbp

c- oH H-C- OH O
H

c-

oH

H-C_

OH

O O

cH2_

o_ P_ o

cH,_ o_ P_
Fructose-6-phosphate

Clucose

;l

Clucose-6-phosphare

ATP:

TriosePhosphate,
Isomerase

""'flx:{::"" -:J1:_l_" ^,"1 i''- '- | *"n,-o-!-oi=o


o
I

o.

\.'" I " 1-.pi"'ii"t, ' '..'rl0dehydrogenasell


auinYde- \r

9 "1 o- P- o

GlYcer- tl

//7 I

1",-o" '"J""
* H
.

"

Aldolase

c=O
I

l'2 "
o"

'rv- !--

H-C- OH O | "r ffc"r-o-f-o l o

H-C- OH O /=-== il \ \ *a",-o-p-oekt NADH NAD+ ! """"

"_l-

/ {

H-C-

OH O I il cur-o-e-o|

t,3-Diphosphoglycerate Phospho- lf glycerate ll\

H*

^ Pi

o oGlyceraldehyde- Fructose-1,6-diphosphate
3-phosphate

ll-ooo
lrp

kinase '{l \

o.\ ,o .C
I

oo -a4", I

c",- cH)- oH NAD+ Ethanol \ f irooo, \=-,*


NAD.

NADH

il -"t ' cH-- o- P'l


o

H.C- OH

"tct, ff

H-C-

dehydrogenase \ H_ C=

"O

OH

NADH
*,H'

* ln,
Acetaldehyde

Lactate
Iactate
dehydrogenase mutuse

3-Phospho$ycerate

o,!l#::j;",,

I.
I

,o,

oa\ ,o

Jl
rr

ll

nhospnogryc"ro-

CO til H-C- O- P-

o:\ ,o

\
Pyruvdte Kinase

ott

Enolase
O

CO lil

zo

HO-H2C

l*"

ill *cHz

c-o-P-o
o

ADP

,/\

c=o
ATP

*l , CH,
Pyruvate

2-Phosphoglycerate

Phosphoenolpyruvate

Figure a-

12

.i.

lhere is usually one point of confusion in this process and that comes at Step 4 ::.d Step 5. When fructose-1,6-diphosphate splits to DHAP and glyceraldehyde:-;hosphate, the label is initially on the *C-1 carbon of DHAP (which was the ;beled nC-1 carbon of fructose-1,6-diphosphate). However, since DHAP is in :::"ilibrium with glyceraldehyde-3-phosphate, that labeled carbon at the *C-1
-

rpvright

by The Berkeley Review

la9

The Berkeley Review Specializing in MCAT Preparation

Biology

Metabolic Pathways

Glycolyir

position of DHAP is now the labeled carbon at the *C-3 position of glyceraldehyde-3-phosphate. \44ren you split fructose-1,6-diphosphate the numiering system of the carbon atoms in the subsequent productJ are now based o1 u thrd carbon molecule. In other words, the phosphate attached to the C-l position off DHAP is the same phosphate attached to the C-3 position after DHAP hns isomerized to glyceraldehyde-3-phosphate. Therefore, the labeled *C-l carbon cf DHAP is the same labeled *C-3 carbon of glyceraldehyde-3-phosphate.

regulation of the glycolytic pathway. In particular, regulation ui th" level od phosphofructokinase is the most important. (we will r"e why in future
discussions.)

three places where the reactions are essentially irreveisible. Those reactions a-re catalyzed by_,the enzymes hexokinase (AGo' - -4.0 kcal/mole), phosphofructokinase (AGo'= -3.4kcal/mole), and pyruvatekinase (aco' = -z.s ical/moleil respectively. It turns out that these reactions all involve control points in ttrc

Regulation If you look at the glycolytic pathway presented in Figure g-12, you will notice

lvhen the cell has plenty of glucose it will make a lot of ATp. Times are good fm the cell and so it doesn't want to waste its source of energy (glucose) uy irtltizrng it to make more ATP than it really needs. one thing *," would titu to ao "utt with its supply of glucose is store it. As we will later see, the storage form qd glucose is glycogen.
However, before the cell can begin to store this glucose it must somehow slow down the glycolytic pathway. High levels of ATp tend to allosterically inhiw

phosphofructokinase. Not_only do high levels of ATp inhibit phospdofructokinase, but high levels of H@ inhibit it as well. one place we could get in increa_ce in hydrogen ions is from the conversion of pyrursate to lactate (see F-igure 8-10). If you were to produce too many hydrogen ions, then your blood pH would begio to drop and you would experience acidosis. one other regulatory molecule"of phosphofructokinase is citrate, an intermediate in the Krebs cycle. if the levels of citrate are high, it must mean that glycolysis is functioning at some optimal rahe (because there is plenty of glucose around). once again, why waste the glucose that has been made available to you. High levels of iitrate also inhibit phosphofructokinase.
that when the concentrations of ATP are low the activity of phosphofructokinase is enhanced. what indicates when the levels of ATp are low? If ATp is being utilized, it is most often converted to ADP and even AMp. It turns out that high levels of AMP stimulale phosphofructokinase.

If high concentrations of ATP inhibit phosphofructokinase, then it must

mean

Copyright @ by The Berkeley Review

19()

The Berkeley Review Specializing in MCAT Preparation

Biology

Metabolic Pathways

Disaccharide Metabolism

DisHbdhffi dd' MGtd$6ti$m


:onstituent monosaccharide residues which can then enter into"the glycolytic rathway. In humans we find that maltose can be hydrolyzed by thJ L"ri^u maltase into two molecules of B-D-glucopyranose while iucrose can be hydro]i'zed by sucrase into a-D-glucopy.anose ind B-D-fructofuranose . Lactosecan be rvdroiyzed by lactase to B-D-glucopyranose and B-D-galactopyranose. These :nzymes are located in the epithelial cells that line the smill intestine. In bacteria -actose is hydrolyzed by the enzyme B-galactosidase. ]nce maltose is hydrolyzed, the monosaccharide residues are immediately ready :o enter into the glycolytic pathway. However, the B-D-fructofuranose residue ::om sucrose and the B-D-galactopyranose residue from galactose both need to :e converted to a form which cun ente. into the glycolytic iutn*uy. -he hydrolysis of sucrose into g-D-glucopyranose and B-D-fructofuranose is tlow_n in Figure 8-13. The glucopyranose residue will readily enter into
:lvcolysis.
.-oH Sucrase,
cH2-oH

Disaccharides such as maltose, sucrose and lactose can be hydrolyzed into their

HO

H:O

HzC o
+

-)
OHH
a-D-Glucopyranosyi- B-D-fructofuranose

nU tlcH^

K"J
I-l OHH

oH

HOH
a-D-Glucopyranose
(Enters Glycolysis)

on'

Sucrose

B-D-Fructofuranose

Figure

B- t

i{cwever, the fructofuranose residue must first be converted into a form which --'n enter into the glycolytic pathway. Fructofuranose can either be converted p-D-fructofuranose-6-phosphate by the enzyme hexokinase -- lo or into B-D-:uctoJuranose-1-phosphate by the eniyme fructokinase. Both reactions are :rosphorylation reactions and both require ATp.
CH,.O-PO.2J -

| C=

lrtose
Phosphate

HO-H2C H

K"J V V/
OHH -

OH

oT

tt

CH2-OH

Fructokinase

10"

""W
2-o.Po32-

l"r-on {"'"'" \
DHAP

o.,
C
I

+
} \"
I

ADP

H-C- OH
I

OHH
B-D-Fructofuranose1-phosphate

CH2-O-POj2-

B-D-Fructofuranose

Glyceraldehyde
3-phosphate
I

H-C_ OH
cH2-OH

D-Glyceraldehyde

Glycolysis

iigure B-14
:pyright
@

by The Berkeley Review

l9l

The Berkeley Review Specializing in MCAT preparation

Biology

Metabolic Pathways

Disaccharide Metabolism

The hydrolysis of lactose into B-D-galactopyranose and a-D-glucopyranose is shown in Figure 8-1.5- once again, the grucopyranose residue ivitt reaaity enter into the glycolytic pathway.
cHz-oH
Lactase,

rm

G
H

*+
H'O

OHH
HOH
(Enters Glycolysis)

HOH
Lactose

HOH

HOH

B-D-Galactopyranosyl-cr,-D-glucopyranose

B-D-Gatactopyranose g-D-Glucopyranose

d h h
wn

Figure B-15
The galactopyranose residue must now be converted into a form which can enter into the glycolytic pathway (this sequence is not shown). Note that galactose is an epimer of glucose at t_he C-4 position. It turns out that galactlse can be converted into glucose-6-phosphate in 4 steps. Recall that glucoJe-6-phosphate m

an intermediate in glycolysis.

Copyright @ by The Berkeley Review

192

The Berkeley Specializing in MCAT prepara

Biology
ltr6$$l'i

Metabolic Pathways

Krebs Cycle

=r-entually be converted into carbon dioxide.

lhe citric acid cycle is also called the tricarboxylic acid cycle or the Krebs cycle. ":n eukaryoflc cells the citric acid cycle o"".rrc inside lhe mitochondria. Glvcolysis, however, occurs in the cel|i cytosol. Recall that the end product of :jvcolysis was pyruaafe. Under anaerobic conditions, in the absen." bf o*yg"r,, :r'ruvate was used as an oxidizing agent to reoxidize NADH to NADd so :lvcolysis could continue. For example, in exercising muscle and in certain :acteria pyruvate can be converted into lactic acid wrile in yeast pyruvate can !e converted into carbon dioxide and, ethanoL Under aerobic in the "orrd-iiio.,r, :resence of oxygen, we will find that all of the carbons of pyruvate will

The first reaction to consider is the oxidative decarboxylation of pyruvate to form acetyl CoA. This is a complicated reaction involving three caialytic cofactors 'Siamine pyrophosphate (Tpp), lipoamide, and fla"vin adenine dinucleotide FAD)) and two stoichiometric cofactors (CoA and NADo;. this reaction is by a group of three enz,ylres (pyruvate dehydrogenase component, :1:u1{r"9 iihydrolipoyl transacetylase, and dihydiohpoyl dehydrolenase) which are :ranged into a structure called the pyruvate dehydrogenase iomplex.

TPP adds to the carbonyl function of pyruvate. The addition compound ,lecarboxylates and the remaining portion -oi pyruvate (a hydroxy-ethyl moiety) *r transferred to lipoamide where it is oxidiied to form aletyl-iipoamide. The :cetyl moiety of acetyl-lipoamide is next transferred to CoA to form acetyl-CoA :nd dihydrolipoamide (the reduced form of ripoamide). NADo is converted to \ADH + Ho as it reoxidizes dihydrolipoamide to lipoamide. The essentials of -ris reaction can be seen in Figure g-16.

NADH

cH3.C-C-O -3 +2 +3
Pyruvate

oo ililo

NAD+ |
+ CoA-SH

+H+

\/

Pyruvate
Dehydrogenase Complex

o il CH3'C- S- CoA + O=C=O

-3

+3

+4

Acetyl-CoA

AGo'= -8.0

Figure 8-

l6

The AGo'for this reaction is about -8.0 Kcal/mol and is very much in favor of the :roducts. Not only is entropy increasing because we are converting one molecule, :vruvate, into two molecules, acetyl-CoA and carbon dioxide, but the carbon lioxide iiself is free to leave as a gas. This makes the reverse reaction rather

iifficult.

the :arbonyl carbon +2, and the carboxylic acid carbon +3. The oxidation state of the :arbon in carbon dioxide is +4. what about the acetyl group of acetyl-CoA? The :rethyl carbon still has an oxidation state of -3. The cirbonyl carbon, however, row has an oxidation state of +3. Remember, sulfur is more electronegative than :arbon. \A/hat is the difference in oxidation states between the reactant, pyruvate,

:n g-keto acid" The methyl carbon has an overall oxidation state of -3,

l\re mentioned that the reaction shown in Figure g-16 is an oxidation pyruvate is

Copyright @ by The Berkeley Review

193

The Berkeley Review Specializing in MCAT preparation

Biology

Metabolic Pathways

Krebs Cycle

of the carbon of the carbonyl function and the carbon of the carboxylic acid function of pyruvate is +5. The combined oxidation states of carbon dioxide's carbon and the carbonyl carbon of acetyl-CoA is +7. This is a two oxidation process because we have increased from +5 to +7 as we have gone from reactants to products. If there is an oxidation, there must also be a reductionNADo undergoes a two electron reduction to form NADH + H@
The two carbon unit that we have synthesized in the form of acetyl-CoA unique because it has a high energy thioester bond. The problem that we i faced with is turning the two carbons of acetyl-CoA into torbon dioxide. we w to use the energy of the thioester bond in acetyl-coA to help turn both of the carbons into carbon dioxide. The only way we are going to be able to accompli this is to extend the carbon chain. we can do this by taking this two carbon ai unit and combining it with a four carbon acceptor molecule called oxaloacetic (oAA). where did oAA come from? we will discuss this at a later time.

and the products, carbon dioxide and acetyl-CoA? The oxidation state of the methyl carbon does not change, so we can ignore it. The combinedoxidation states

Figure

B-

l7

to form the intermediate citryl-CoA. Citryl-CoA is then hydrolyzed to fi citrate with the subsequent regeneration of coA. It is the hydrolysis of thioester bond that allows this reaction to be favorably pulled to compietion.
The enzyme that catalyzes this reaction is citrate synthetase. It is in the clas enzymes called a lyase. Note that there is no net gain or loss of hydrogens in reaction. Therefore, it cannot be a dehydrogenation--and it certainly cannot hydrogenation. There is no net change in oxidation state either. Those molq who gained electrons also lost electrons, and those who lost electrons also electrons. Think of it as a reaction in which you are adding to the double the carbonyl group in OAA. Thus, it is alyase.

The alpha hydrogen of acetyl-CoA witl form a bond with the carbonyl oAA as shown in Figure 8-17. The reaction proceeds via an aldol con

Even though citrate is a symmetrical rtolecule, the two -CH2-COOo not react identically. why? It has to do with the asymmetrlc nature of the aconitase in relationship to the symmetrical nature of citrate. In other two -CH2-COOo groups of citrate are not equivalent. Aconitase, which citrate to cis-aconitate, can distinguish between them. Thus, citrate is a molecule and is optically inactive.
There is a hydroxyl function at the beta position of citrate. you might we could oxidize the hydroxyl group to make the beta-keto function so we decarboxylate that carbon. We cannot do that because the hydroxyl
Copyright @ by The Berkeley Review

194

The Berkeley Specializing in MCAT Pre

Biology

Metabolic Pathways

Krebs Cycle

:.rtiary and would require very forceful conditions to oxidize it. What we can do, rough, is a little rearrangement. Tertiary alcohols are known for their ability to .:se their hydroxyl function and form a fairly stable tertiary carbonium ion as ::.own in Figure 8-18. You can think of this carbonium ion as being an -::termediate in the reaction scheme. What can we do with this tertiary :arbonium ion? Note that it has some alpha hydrogers. When alpha hydrogens are :lose to a carbonium ion we may have an elimination reaction. This will give us :is-Aconitate.

rrom. .cetrt coa

coo
I

OOC- C- OH

-oH
Atttnituse

I CH.
r'n) ,9

R \\

ooc-

-Ht
c
@

CH'

t'

OOC-

AGo'= +2.0
Tertiary carbonium ion intermediate

lt

C.H

Coo
Cirrate

-.l" Hydroxyl

coo
cis-Aconitate

lg

Figure B-18

-: ive add water across the double bond in cis-Aconitate, we will produce ,.;ocitrate as shown in Figure 8-L9. Aconitase, which catalyzes the reaction from -,lrate to cis-Aconitate, also catalyzes the reaction of cis-Aconitate to isocitrate.
coo
I

o
Hzo OOC_ C-H
Aconilase
I

OOC_

tt C-H I O
C

CH,

coo

AGo=-0.5

H-C-

coo
Isocitrate

lg

OH

cis-Aconitate

figure B-19 i
5
:*

rcitrate has a secondary hydroxyl group that is capable of being oxidized toa ::rbonyl function. We can oxidize isocitrate with NADo.
.

,:
:ri

coo

coo
N,Ao' \ADH + HIsocitate
dehvdrogenose
I

l"' ol\,/o lrtrL L-n


-

CH.

I o: " fcoo "

CH,

-*
-ra

AGo'= -2.0
B-Keto Acid

Isocitrate

c-Ketoglutarate

c=o loo '

t' t-

F"igure
'
rirl,

B-2O

,dt!.

-l: 'q

l-.e enzyme that carries out that reaction is isocitrate dehydrogenase. Note that :: intermediate that will form is a B-keto acid, which is quite unstable. This will :=-ult in a spontaneous decarboxylation at the C-3 position of isocitrate to give anetoglutarate (o-KG) as shown in Figure 8-20.

il

(n

-'.,pyright @ by The Berkeley Review

195

The Berkeley Review Specializing in MCAT Preparation

Biology
e-i"JT

Metabolic Pathways

Krebs Cycle

n-c.-J f? cH2:c:
B-Keto Acid

If you compare the relative stabilities of an alpha and a beta keto acid, you will find that the more stable form is the alpha keto acid. The general mechanism for the spontaneous decarboxylation of isocitrate can be seen in Figure 8-21. NoE that the enol form is in tautomerlc equilibrium with the keto form.
o

l*co,
R-C=CH2
Enol

f.

?"

We now have an o-keto acid which is reminiscent of where we started back at pyruuate. Pyruvate was also an cr-keto acid. The cr-ketoglutarate will lose ttre carboxyl group that is alpha to the carbonyl. After going though a reaction that b similar to the reaction with pyruvate, we will get succinyl-CoA and carbon dioxide (Figure 8-22). This reaction is catalyzed by the cr-ketoglutarate dehydrogenase complex. Many anaerobic organisms do not have this enzyme and therefore cannot carry out this reaction.

1l

R-C-CH3
Keto

fl

o coo coA

+ ir,
i=; I

1""
n

N$D*

NA,,DH +

H+

coo

1",

Figure 8.2

a-KG Dehydrogenase

i;, 1
i=;
I

co:

Coo'r LGo' = -7.2 cr-Ketoglutarate


Figure 8.22

Complex

i-con
Succinyl-CoA

We can take advantage of the high energy thioester bond in succinyl-CoA to some GTP (guanosine triphosphate) as shown in Figure 8-23. GTP can readily converted into ATP (and we will find out how later).

coo
I

o
lTlil GDP+P1

cHz
tc=o
I

GTP

coo
I

o
coA-sH

Succinyl CoA
Synthetase

fnt +
CHz
IA

S-CoA

Go'= -0.8

coo"
Succinate

Succinyl-CoA

Figure 8-25

If you like, you can think of these high energy compounds (ATP, GTP, etc.) nucleoside triphosphates (NTP)--a more generic name. The enzyme catalyzes the conversion of succinyl-CoA to succinate is succiny
synthetase. This enzyne is in the class of enzymes referred to as ligases. \A/hy? ligase will put two molecules together by using a high energy phosphate

When you use GDP and P1 to make GTP, a water molecule is prod However, the water molecule that hydrolyzes the thioester bond and the w molecule produced in the formation of GTP cancel each other out. Therefore, is actually an anhydrous reaction. Also note that succinate is a symmet molecule and the two ends, -CH2-COOe, are indistinguishable from one ln the reaction shown in Figure 8-23 we have made a GTP. Since we started with two pyruvate molecules (remember, the glucose molecule gets split),
Copyright @ by The Berkeley Review

196

The Berkeley Specializing in MCAT

Biology

Metabolic Pathways

Krebs Cycle

nave a total of 2 GTP's formed in the Krebs cycle. In other words, at this point we have a total of 4 net NTp's since the start of giycolysis.

l.the following set of reactions we will be reconstructing succinate back into OAA so we can repeat the Krebs cycle. The only differenJe between succinate and OAA is the keto group on OAA.
in the regeneration of OAA is an oxidation in which succinate is :onverted to fumarate by the enz)ryne succinate dehydrogenase. Fumarate is in fre cis configuration. If it were in the trans configuratiJn, it would be called -'nleate. since we h_ave a dehydrogenation we need an oxidizing agent. The
The first step

-'ridizing agent is flavin adenine dinucleotide (FAD). Let,s muk" some ieneralizations. Every time you have seen an NAD involved in an oxidation :eaction it has been involved with turning an alcohol into a ketone or an :ldehyde (or vice versa). It does so by a hydride ion transfer. In the reaction rvolving FAD, shown in Figure 8-24, thereis no alcohor. In circumstances where rere is no alcohol, and particularly where you form double bonds, the oxidizing:educing agent will turn out to be FAD. (Note the oxidation states of the c-2 anl
C-3 carbons as we proceed from succinate to fumarate.)

OO i"" FAD FADH2 foo -2 cH) -t c-H \ ,/


coo
.2

Succinate
Figure B-24

CH2 loo "

Succinate -l Dehydrogenase
AGo'=

C-H

loo "

Fumarate

react with water to produce malate. I'Ialic acid was first isolated from apple trees and it is this subitance that gives :oples their tart taste. The enzyme that catalyzes this reaction is fumarase. I-t is a int dration reaction and falls in the Iy ase clasi of enzymes.

-r Figure 8-25 we find that fumarate will

\ote the hydroxyl group on malate. we


"'.-i-11

can convert this hydroxyl group into a lieto group by the use of an oxidizing agent like NAD@. This is the reaction that

oo coo l.-^t c-H Hzo


coo AGo = -0.9 Fumarate
Figure B-25

coo

H-cCH.

oH

return us to

-::r-olved here is malate dehydrogenase. oAA is now ready to react rother acetyl-CoA molecule and repeat the whole procedure again.

oAA and bring us full cycle (Figure g-26). The enzyme

C-H tot'o

Fumara.se

with

coo
Malate

have not made any more NTp's. what has happened to all that energy? It not been liberated yet. we have not used oryg"tr. ar far as this is conce-ied, =-"'erything could have been done anaerobically. we have made a lot of reduced :Lrmpounds such as FADH2 and NADH + Ho which can be classed together as reduced coenzymes. These reduced coenzymes represent a "dry brush" that could :e ignited with oxygen to turn them into their oxidized form, and in doing so 'rerate a lot of energy. Part of this energy will get turned into ATp. Even tholugh ;','e have made only 2 GTP's by going through the Krebs cycle, we have stored"a ::eat deal of potential energy. This potential energy can be used to make a whole :-cck of ATP's as soon as the oxygen is made available to us.
--.as

"ie

NADH O NAD+ *.H* foo foo H-c- oH \ L c= o ,/ ' CH, CH' Malate t-o lon a Dehydrogenase coo Malate AGo = +7.1 OAA O
Figure 8.26

.:id into something that can be oxidized. By the time we went around the Krebs --;c1e, two carbons had come off as carbon dioxide. we had oxidized two rat

-he function of the Krebs cycle is to turn those hard to oxidize carbons of acetic

-:rbons and we regenerated the starting material. Eventually the two carbons we put into the cycle will come off as carbon dioxide, too.

lhe ,Krebs cycle can be thought of as a catabolic process in which the overall 'Go'value for the complete oxidation of an acetyl unit from acetyl CoA is about -9.8 kcalslmol. It would be difficult to reverse this cycle.

-t this point in our story we have turned glucose (or some carbohydrate) into :arbon dioxide via glycolysis and the Krebs cycle. In eukaryotic cells glycolysis
lopyright
@

by The Berkeley Review

197

The Berkeley Review Specializing in MCAT preparation

Biology

Metabolic Pathways

Krebs Cycle

occurs in the cytosol while the Krebs cycle occurs in the matrix of the mitochondria. Recall that the average oxidation state of the carbon atoms in our carbohydrate was 0. The oxidation state of the carbon in carbon dioxide is +4. This process was therefore an oxidation. In order to balance that oxidation \^.e needed to have a corresponding reduction. where did the electrons go? Those electrons ended up in either NADH or FADH2. In order for these processes tc continue we must reoxidize NADH and FADH2 back to NADe and FAD respectively. we can summarize the essentiats of glycolysis and the Krebs cycle as shown in Figure 8-27.
Mitochondrial
membranes

CoA

foruvate
2
ATP

foruvate
Matrix

ti I

Acetyl

Y
{

+NADH

il

2 NADH

g
Glucose
Cytosol

Oxaloacetate
NADH

il

Fumarate

M^ff t

,m

Citrate
Krebs Cycle

r@

Isocitrate

|-FADH2 Succinate I CoA

NADH<.$> ge,
V

fin

6[-ketoglutarate

r{il

@[

crp{
Figure B-27

succinyr '

coA

i:'^^ \co:

rd
+ NADH

im

ffi
fuM

The external oxidizing agent, otherwise known as the external electron accepto:. is oxygen. As oxygen is used as the external electron acceptor, it witl t'e

Un
@0

converted into two water molecules by picking up four hydrogens and fo..r electrons per oxygen molecule (O2). This can be summarized as shown in Figr:re
8-28. Glucose

(CH2O)"

\/CO2 *I-

NAD-. FAD
NADH

\/* 11*. -\

2H2O
02 + 4H+ + 4e-

FADH2

Figure B-28

Copyright @ by The Berkeley Review

l9a

The Berkeley Revier Specializing in MCAT Preparatin

Biology

Metabolic Pathways

Electron Transport

-\ very important feature of the mitochondrion is that it is a double membrane


,.rganelle. The nucleus and the plant chloroplast (site of photosynthesis) are also jouble membrane organelles. As we will learn, much of the ATP that we utilize :n our metabolic processes is synthesized within the mitochondrion. The :iitochondrion is the site of oxidative respiration in the cell.

Outer membrane

''hen we discuss the chemical reactions that take place in the mitochondrion, it important to have a firm grasp of the organelle's anatomy. As we mentioned, :,e mitochondrion is composed of a double membrane system. There is an outer rtembrane and an inner membrane. Between these two membranes is the intermembrane space. The inner membrane is folded into structures called cristae. ltis increases the surface area of the inner membrane. The central cavity of the -:ganelle is called the matrix. These structures are shown in Figure 8-29. Let's :-<e a brief look at these structures.
--r

-'

'.e outer membrane is quite permeable and it contains protein channels called : rrins that allow molecules with molecular weights as high as 10,000 daltons to ::ss through. [One dalton is nearly equal to the mass of a hydrogen atom. In , -:,er words, 10,000 daltons is about 10,000 times the mass of a hydrogen atom.] .i-.iecules such as amino acids, carbohydrates such as glucose, and small : -,r-peptides can pass through these porins. The outer membrane is composed , about half proteins and half lipids.
:,.r'in8 inwards we next come to the intermembrane space. This area of the :::ochondrion contains a variety of enzymes and also has a high proton concen: j::on because of protons which are pumped into this space from the matrix. In ::r words, the concentration of protons in the matrix is lower than the . :entration of protons in the intermembrane space. This leads to an -:trochemical gradient being established between the intermembrane space -- : the matrix. Gradients have the potential to do work. That potential is utilized -he svnthesis of ATP.
'

Ribosome

Figure 8.29
The anatomy of a generalized mitochondrion.

::er surface area means more proteins. In fact, the inner membrane

inner membrane is folded into cristae in order to increase its surface area. A
is

:posed of about 75 percent proteins and about 25 percent lipids. Some of the -::ins in this metnbrane are permeases that allow for the passage of ADP into natrix in exchange for ATP. ATP is synthesized within the matrix of the :,:hondrion from ADP and Pi (inorganic phosphate) at the expense of a ,rn gradient. Because the inner membrane is quite impermeable to hydrogen . protons), there are special permeases that allow for this ion's passage into :.atrix from the intermembrane space (down their electrochemical gradient). : .rample, there are permeases that allow for the simultaneous passage of ::anic phosphate and hydrogen ions into the matrix. Other molecules like ::n (O2) can readily diffuse through the inner membrane and into the matrix - : mitochondrion.

-.r the matrix of the mitochondrion we find hundreds of different types of : .:res. This is a region rich in enzymatic activity. One of the important *-.emical pathways that occurs in the matrix is the Krebs cycle (also called the ".::oxylic acid cycle or the citric acid cycle). The Krebs cycle is the final r rLorr pathway for the oxidation of fuel molecules like glucose. It also -,ies intermediates for biosynthetic reactions and it generates carbon dioxide
:ht
@

by The Berkeley Review

199

The Berkeley Review Specializing in MCAT Preparation

Biology

Metabolic Pathways

Electron Transport

(COz) which can diffuse out of this organelle. ATP is synthesized by an ATP synthase protein which is situated on the matrix side of the inner membrane Also, within the matrix we find mitochondrial DNA.
So far we have turned glucose (or some carbohydrate) into carbon dioxide via glycolysis and the Krebs cycle. In eukaryotic cells glycolysis occurs inthe cytosol

while the Krebs cycle occurs in tlne matilx of the mitochondria. We mentioned that the average oxidation state of the carbon atoms in our carbohydrate was (L The oxidation state of the carbon in carbon dioxide is +4. This process wffi therefore an oxidation In order to balance that oxidation we needed to have a corresponding reduction. Where did the electrons go? Those electrons ended up in either NADH or FADH2. In order for these Processes to continue we must reoxidize NADH and FADH2 back to NADo and FAD, respectively. external oxidizing agent that we will use for this reoxidation is oxygen. We witr see that as oxygen is used as the external electron acceptor, it will be converted into two water molecules by picking up four hydrogens and four electrons per oxygen molecule (O2). This process, which is referred to as electron tra and oxidative phosphorylation, occurs on the inner membrane of mitochondria. Mitochondria are about the same size (1500 nm by 500 nm) as the prokary bacterium Escherichiq coll. Mitochondria have both an outer and an i membrane. Between the two membranes is the intermembrane space. The ou membrane is permeable to many small molecules and ions. The inner however, is essentially impermeable to most polar molecules and ions and it highly folded into structures called cristae. Because prokaryotic cells do not mitochondria, they carry out the reactions of electron transport and oxida phosphorylation on their cytoplasmic membrane (i.e., the inner pl membrane). (Prokaryotic organisms have an outer membrane, followed by a wall, and then an inner membrane.) Mitochondria contain their own DNA. They arise from the growth and di of existing mitochondria within the cell. Human mitochondrial DNA (mt for short) is circular and contains 1,6,569 base pairs. This DNA encodes for tRNAs and 2 rRNAs. It also codes for 7 subunits of the NADH-Q enzyme. This is a complex located on the inner membrane and is responsible proton trans-Iocatio?? across the inner membrane. Cytochrome red cytochrome oxidase, and ATP synthase subunits are also encoded by mitochondrial DNA. We will learn the functions of these complexes in just a bit. How are the reduced NADH and FADH2 coenzymes reoxidized? Recall that Krebs cycle occurs in the matrix of the mitochondria in eukaryotic cells. reduced coenzymes, therefore, are released into the mitochondrial ma Associated with the inner mitochondrial membrane ale sequences of prc complexes that act as electron carriers' Taken collectively, the reiluctase (also called Complex I or Site 7), succinate-Q reductase (also Complex II), cytochrome reductase (also called Complex III or Site 2), cytochrome oxidase (also called Complex IV or Site 3) protein complexes often referred to as the respiratory chain (due to their eventual involvement oxygen) or electton tuansport chain. There are many of these respiratory ch associated with the inner membrane. In turn these protein complexes composed of an array of prosthetic groups (e.g., FMN, Fe-S, FAD, heme, and which can act as electron carriers. As we will see, the electrons from NADH (
Copyright @ by The Berkeley Review

200

The Berkeley Specializing in MCAT

Biology

Metabolic Pathways

Dlectron Transport

into the electron transport chain at the level of the NADH-Q reductase complex rvhile the electrons from FADH2 enter by way of the succinate-Q reductase complex and a molecule called coenzyme Q (also known as ubiquinone). This can be seen in Figure 8-30.
Let's consider each of the enzyme complexes shown in Figure 8-30 in a little more detail. We'll start with the NADH-Q reductase complex first. Reduced NADH + He passes two electrons and two hydrogens to the oxidizedflaain mononucleotide FMN) prosthetic group associated with this enzyme complex. This results in the

NADH
NADH-Q
Reductase
Co mplex

:roduction of reduced FMNH2 and the regeneration of oxidized NAD@. The electrons from FMNH2 are passed to a series of iron-sulfur clusters (Fe-S) in which ]re iron atoms cycle between the reduced ferrous (feZe, and the oxidized ferric Fe3o; states. There is an uncertainty as to how many Fe'S clusters there are in ne NADH-Q reductase complex. The electrons will eventually be passed from a
:educed Fe-S moiety to the oxidized form of coenzyme Q (Figure 8-31).

FADH2
Succinate-Q Reductase
Complex tr

NADHY-

F\,IN

FC]+-S).,- FC3+-S\ZCOAHZ

C Qc=

0
Cytochrome Reductase
NADH-Q Reductase
Complex

Figure 8-3 I
CoQ). The oxidized quinone is just called quinone while the reduced quinone is -il)ed dihydroquinone. Coenzyme Q was first discovered back in the 1930's and it ,.as turned up in every organism which is capable of doing this type of electron

.-t the end of the Fe-S complex there is a quinone. It is called coenzyme Q (or

Cyt c

trI

+
fc".. h--.

:ansport. In the late 1940's the international fraternity of biochemists realized ::,at they were finding CoQ everywhere and so they thought it would be nice to -eII it ubiquitous. Coenzyme Q in the oxidized form is now called ubiquinone ,"-rile coenzyme Q in the reduced form is called ubiquinol. The structure of *riquinone (CoQ) and ubiquinol (CoQH2) and a semiquinone (free-radical) :,:ermediate is shpwn in Figure 8-32. o.
H*+eLE]R

bxioase I

ta

or1

ge n

Complex IV

Figure B'3O

OH
I

-\>wute.
cHl
R

"..n-A'_.".

H,co-14)-

",."\,JOH

Hrco-UJI

o Ubiquinone

OH

Semiquinone

Ubiquinol

Figure B-52

l:ca1l that as succiriate is oxidized to fumarate in the Krebs cycte by the enzyme .::cinate dehydrogenase, FADH2 is generated. This FADH2 needs to be '',:':idized. Succinate dehydrogenase turns out to be part of the succinate-Q -.juctase complex. FADH2 is immediately passes its electrons to an Fe-S ::rtein(s) which then funnels them into the oxidized form of CoQ. FADH2 is -. rridized to FAD as shown in Figure 8-33.
-

,:i'right O by The Berkeley Review

201

The Berkeley Review Specializing in MCAT Preparation

Biology

Metabolic Pathways

Electron Transport

What we have done so far is to collect all of the electrons from all of the various substrates into one pocket which is CoQH2. No matter what is happening we will always have a large turnover of CoQ. our next interest is to reoxidize CoQ.

FAD

FADH2

The electron carriers between CoQ and oxygen are called cytochromes
Cytochromes are electron transporting proteins that contain a heme prosthetb group with an iron atom that alternates between the Fe2o and Fe3e conditioru within the cytochrome reductase complex are two cytochromes, b and c1, and an Fe-s protein. As coQH2 transfers one electron at a time to an Fe-s protein in thc

complex it is converted to CoQH. (semiquinone). The reduced form of cytochrome b (Cyt b2o) reacts with CoQH.to give the oxidized form
cytochrome b (Cyt 63e) and CoQH2.

Figure 8.3

time. However, the Fe-S protein can carry only one electron at a ti Cytochrome b is the go-between that allows this interaction to occur, electrons are eventually passed to cytochrome c. The iron atom of the group of cytochrome c is bonded to a sulfur atom of a Met residue on one and to a nitrogen atom of a His residue on the other side.

Cytochrome b3e can then oxidize another molecule of CoeH. to Coe. This shown in Figure 8-34. Coenzyme Q is a molecule that carries two electrons at

3+ 2+ C1t b Cyt b

Figure 8-54

o
o o a
c3".

The electrons that end up on the reduced form of cytochrome c in Figure next passed to the cytochrome oxidase complex. This complex consists of heme groups, heme a and heme a3, and two copper ntorns, one associated heme a and the other with he'me a3. These copper atoms can alternate between the +1 and +2 oxidation stateselectrons are passed from cytochrome c to the heme a and heme a3

Cyt

a3

o x
0)
(D

the cytochrome oxidase complex and then to oxygen. The transfer of


electrons to molecular oxygen leads to its reduction to two molecules of This is shown in Figure 8-35.

:r{:,o q-4H*
$cm cf weps
WTh.

At this point we have completed electron transport and have reoxidi


reduced coenzymes. This means that there must have been a strong ne AGo' for this process. The question we want to get at now is how A synthesized. So far we have not seen anyway for a high energy phosphaF to be formed (none of these compounds are phosphorylated).

E-

lligm D55

Curyqmm6[r @ @n

Ih

tserkeley Review

202

The Berkeley Specializing in MCAT

Biology

Metabolic Pathways

Oxidative Phosphorylation

oxiildti#$ri:mm:u$HH fition
lhere are three areas in this oxidation reduction scheme where ATp can be ..'-nthesized due to a conservation of energy. It turns out that the passage of two .lectrons from NADH down this chain allows for the synthesis of 3 molecules of \TP. This is described by the P/o ratio, The P/o ratio is the number of high

:nergy phosphate bonds made per atom of oxygen used. The P/o ratio for

F--\DH2 allows for the synthesis of 2 molecules of ATP. The reason that FADH2 :enerates only 2 ATPs is because the electrons from FADH2 enter into the =lectron transport chain at a lower energy level than the electrons from NADH.

rain

:J of the ATP molecules that we will be producing

via the electron transport

is associated with these types of reactions is oxidative nhosphorylation (ox-phos). oxidative phosphorylation will yield a flock of -{TPs. All the other ATPs that we have made (i.e., in glycolysis and the Krebs --' c1e) can be distinguished from ox-phos ATPs because they did not directly :epend on oxygen. Al1 six of the high energy bonds that were formed in botir
--->toric term that
:-,. colysis

depend on the presence of oxygen. By taking ADP and Pi and making ATp ".-e are doing a reaction which is referred to as a phosphorylstion reaction. The

and the Krebs cycle are not referred to as oxidative phosphorylations

: -rt rather substrate level

phosphorylations.

:-lon' are the A T Ps synthesized during oxidative phosphorylation? The :echanism, which was first proposed in 1961 by Peter Mitchell, is called the ;hemiosmotic hypothesis. Mitchell said that ATP synthesis and the electron -:arLsport chain are coupled together by a proton gradient that had established .:-'elf across the inner mitochondrial membrane and not by a high energy ::osphorylated intermediate. The coupling factor for this mechanism is an ::z\/me called the FsFl ATPase. This enzyme allows for the synthesis of ATp at :-e expense of the free energy that is released as a proton (Ho) passes through :.is F6FlATPase from the intermembrane space to the matrix of the :-rtochondrion. How was the proton gradient established?
the following diagrams the membrane that is drawn will be taken to be the '.tner membrane of the mitochondria. Associated with this membrane are the ::rLplex of proteins we collectively call the electron transport system (or the :=:piratory complex). This particular array of enzymes allows for a smooth and : -'ntinuous flow of electrons from one end to the other. As we have mentioned, ::.ese respiratory complexes are repetitive within the inner membrane.
-: there are reduced substrates (e.g., NADH or FADH2) contributing electrons =-fier from within the mitochondrion or from without (e.g., the electrons from ::.e NADH synthesized during glycolysis), then the electrons will move down ::e electron transport chain and eventually come into contact with oxygen. The

,:,\'gen will be reduced to water. Mitchell's hypothesis is that there is a ::arrangement of these enzymes as they pass back and forth through their :::dized and reduced states such that hydrogen ions are removed from the interior :.atrix) of the mitochondrion and pumped out into the exterior (intermembrane ;:ace) of the mitochonidrion. Hydrogen ions are thought to be translocated across -:e rnner mitochondrial membrane at each of the three enzyme complexes (i.e., :.re NADH-Q Reductase, Cytochrome Reductase, and Cytochrome Oxidase --.mplexes). This is schematically shown in Figure 3-36. If this process repeats ,:-.elf, then the pH of the interior will increase (the acidity will decrease) with :=spect to the exterior. That is what constitutes the proton gradient.
-

rpyright O by The Berkeley Review

203

The Berkeley Review Specializing in MCAT Preparation

Biology
Hish [H+l

Metabolic Pathways

Oxidative Phosphorylation

If there is this gradient of hydrogen ions where the [Ho] is greater on the outside than on the inside, then it will allow the flow of i,yd.ogJ' ions back into the matrix through the FoFlATPase complex. This is u.ron-"qiitiurium situation and represents potential worf,If thelrydrogen ions are allowed to enter through this ATPase compler; then this work energy can be converted into useful chemical energy through the synthesis of ATp as shown in Figure g_37.

Inner mitochondrial
membrane

Figure 8-36

Low

[H']

FlFoATPase
Figure 8-57 This ATPase complex is not part of the respiratory unit (electron transport The oxidation takes place in the respiratory unit whiie phosphorylation place at the ATPase complex. These ATpase units occur ali orru, th" iorr". of the inner membrane of the mitochondrion and essentially alternate with units of the respiratory assembly. These two units taken together, the elecl transport chain and the F6FlATpase comprex, constitute thi oxidatiae of ATP.
<

unless there was a specific port to allow these molecules purrug".

The hydrogen ions cannot freely pass across the inner mitochondrial membra If they could flow freely back and forth across the membrane, no work would done. This membrane-must exclude hydrogen ions except at the ATpase port entry. Not only is this membrane impermeable to protons but it is al impermeable to almost all charged molecules unless there are specific in protein ports which allow the molecule in question to come across. For exa the membrane would be impermeable to ATp, ADp, pi, NAD, and N

affect on the oxidative phosphorylation but not on the substrate level phorylation of ATP. Physical damage to the mitochondrial membrane can in

Consider for a moment the integrity of the mitochondrial membrane. If were to happen to the integrity of the mitochondrial membrane, it would have

phosphorus from compounds inside the matrix to ATp, then the removal of membrane should not have made a difference and phosphorylation should
Copyright @ by The Berkeley Review

20,4

The Berkeley Specializing in MCAT prel

Biology
in the mitochondrial

Metabolic Pathways
membrane, oxidative phosphorylations ,,o longu,

Oxidative Phosphorytation

be taking place. But that was not the case because every time a hole was punched

proceeded. substrate level phosphorylations did cbntinle though.

It turns out that there is more than one way to punch a hole in a membrane. one way to do this is to chemically modify the membrane or even modify the passage of the hydrogen ions across that membrane. suppose hydrogen ions could cross the membrane without the membrane realizing it. If yort modify the way in tllt.h the hydrogen ions can enter into the matrix such "orld that they sneak past the ATPase complex without doing any work, then that will wipe out oiidative phosphorylation.

{ iv9 aaa a hydroxyl group to dinitrobenzene, we will make a compound" called 2,4-dinitrophenol (2,4-DNp). This is one of the first compounds discovered that .auses a problem with the integrity of mitochondrial membranes. 2,4-DNp is a sufficiently strong acid. Recall that phenols are acids--they are slightly acidic. If 1,4-DNP were capable of moving across a mitochondriai membiane and then cissociating inside, it would have transported. a hydrogen lon across the nembrane. The method of getting a protonicross the membiane is referred to as a proton transport.
,\'hat will be the effect of the presence of 2,4-DNp on a system such as this where -he hydrogen ion gradient was the coupling aspect of electron transport and ATp

will uncouple it. If we weie to uncouple this hydrogen ion gradient, would be just getting -52 kcals and no work. This rn"ut r thai all the ihemical :rergy will be dispersed as heat.
:ormation? It
;'.'e

\'ery large number of mitochondria, and since the mitochondria have :"'tochromes which contain iron, they give the fat a brownish color. In infants :-ts brown faf is located in_the interscapular space. Adult humans have very little ::orvn fat. Embedded in the mitochondria of the brown fat is a special transport :.'stem that allows protons to move back across the inner membrane. since the -. drogen gradient is dissipated, there is no means to capture the energy of ATp. -:-otead, this energy,is lost as heat.
can calculate the overall generation of energy from glucose. In the process of i--rcose going to pyruvate we found that 2 net ATps were generated directly. In ::s process we also produced 2 NADHs which were extramitochondrial. Tlrese
',-e

-r similar heat production mechanism is utilized by hibernating animals and :ervborn human babies. There are specialized fat ceik in which mitochondrial :-spiration is naturally uncoupled from the synthesis of ATP. These fat cells have

:r =a-mitochondrial NADHs are transported into the mitochondrial matrix by ::j1er one of two shuttle systems--the glycerol phosphate shuttle or the malateispartate shuttle. rf the glycerol phosphate shuttle is utilized, each cytoplasmic 's,\DH yields two ATPs. rf the malate-aspartate shuttle is used, each cyto:,,=mic NADH yields the normal three ATps.

- re Krebs cycle the 2 pyruvates


i'D2s.

,"

: 8 NADHs (four for each pyruvate). These NADHs and FADHfs will be ::a-mitochondrial.
riright
@

lll-P from the Krebs cycle. The NTps that we have produced so far are referred 's substrate leael NTPs because they have not come into contact with oxygen. : " also found that the Krebs cycle produced 2 FADH2s (one for each pyruvite)

we also proddted 2 more high energy phosphate bonds in ihe form of

that we generated from glycolysis produced

by The Berkeley Review

205

The Berkeley Review Specializing in MCAT Preparation

Biology

Metabolic Pathways

Oxidative Phosphorylation

Bacteri
Photons

g+

ATP

Synthetase

ATPs produced is 38. If we had used the glycerol phosphate shuttre, we would have produced 3d ATps. It turns out thit-the thirmoiynamic efficiency for the complete oxidization of glucose is about 40% under stairdard
"oi.raitiorrr.

Recall that we mentioned that the p/o ratio for NAD is 3 ATps while the pio ratio for FAD is 2 ATps. Consequently, if we utilize the marate-aspartate shuttle, the 2 NADHs from glycolyiis wiil give us 6 ATps while the 8 NADH' fr91n the Krebs cycle wilr give us 24 ATps. fhe 2 FADH2, tro*it Krebs cycle will give us 4 ATPs. we have 2 ATps made in glycolysi and we have " 2 GTps (which can be converted into ATp) from the r-rebs cycle. Therefore, the total

AD

Halobacterium halobium (from the Greek hals f or salt) is an archaebacterium

+Pi
Reconstituted Vesicle

Figure B-38

NADH
NADH-Q
Reductase
Comple
Rotenone
G)

chromophore). The retinal found in bacterioihodopsin isjaen'ticat to the retinal in. the rhodopsin of the rod cells of vertebiates. when incident photm loun! flux impinges on this purple membrane in the absen"" or o"yg"r,, it acts like a proton pump and translocates protons from the cytosol of the lell b the outsil environment. This proton gradient can be used to synthesize ATp. If oxygen i prl""t, this organism can also carry out oxidative phosphorylation to ATP.

end of san Francisco Bay. Its membrane can be separated into various fractions, one being a purple-m-embrane protein called baiteriorhodopsin which contains seven transmembrane helices and a light absorbing ptorlh"ti" group called retinal (a

Compare this to the salt concentration in sea water which is about 0.6 M. one place where these arachaebacteria can be found is at the south

that requires an optimal sodium chloride concentration of about 4.3 I\{-

xr

CoQ..

I I

FADH2
Succinate-Q Reductase
Complex

II

experiment (circa 1974) added support to peter Mitchell's chemiosmo hypothesis (circa 196r) involving the coupling of a proton gradient with a
svnthesis.

Cytochrome Reductase
Complex

Antimycin

AG)

trI

{"tAscorbate Oxygen
CN,N3,

',to cl
Water

one experimental means that was employed in elucidating the pathwav electron flow in the electron transport thuitt *u, by th! use-of sp* inhibitors. Rotenone is a toxic plant poison that inhibits electron tranq within the NADH-Q reductase complex. Antimycin A, a toxic antid isolated from a strain of streptomyces, blocks electron flow b cytochromes b and c1 (at the cytochrome reductase complex). Addition ascorbate reduces cytochrome c and allows for electron flow from cytochru c to oxygen. The cytochrome oxidase complex can be inhibitei by molecules as cyanide (cNe), azide (N3e), or carbon monoxide (co). The of these inhibitors can be seen in Figure g-39.

Figure 8.39

Copyright @ by The Berkeley Review

206

The Berkeley Specializing in MCAT pre

Biology

Metabolic Pathways
iiiiiriiiiiiiiiiiiii*i

Pentose Phosphate pathway

PuGffisG,:..ifililo$flnfiffi1li

,,iliii'

',

,;,;,,,,,;:,

Let's consider the pentose phosphate pathway. The purpose of the pentose phosphate pathway is to generate reducing power in the form of trllopH and five carbon sugars such as ribose-5-phosphate. These reactions occur in the cytosol of the cell and, as we will learn later, some of them will be involved in the formation of other sugar molecules from CO2 in photosynthesis.
Recall that during glycolysis, glucose can be phosphorylated to form glucose-5phosphate. The enzyme that cataryzed this reaction is hexokinase. Glucose-6phosphate has many fates and one of them involves the pentose phosphate pathway. rf we dehydrogenate (remove hydrogens) glucose-6-phosphate at the C-1 carbon, we will get a molecule called 6-phosphoglucono-d-lactone. This oxidation reaction yields NADPH and is catalyzed by glucose,6-phosphate dehydrogenase. See Figure 8-40.

o- P- o- cH.
II

il

NADPH O- P- O_CH.
o-H

o9

oo

\-^ ' ctuco,se 6-phosphate no\fl--1\


Qenyarogenase l"
6-Phosphoglucono-

NApP.

,F - "6 /

o,,

Glucose 6-phosphate

Slactone

Figure B-4O

}e predominant form of glucose-6-phosphate in solution is in the ring form. l"ote that the C-1 position is bonded to two oxygen atoms. As shown in Figure & {0, oxidation at the c-l carbon will give 6-phosphoglucono-d-lactone. If we have : carbonyl function that is next to an oxygen atom in a ring structure, we have a ,',;lic ester or lactone.
Oll O o-P-o o- 9Ht
I

oat
C
I

,o
OH

NADPH F 'o
+

o"

H.CI

1I{
- Hzo Lactonase
OH

Ho - c_H NADP- |

H*

H-CI

OH

I ".f-o" H-C- OH

6-Phosphoplutonate H-C- OH " ,,, | cHr.o-Po,) dnatc cHr-o-po.,2


|
I

\'

- ^[-o, 1
B-Keto

c=

cPt coz i= o
I

cH2-oH

"-t-o"
H-CI

OH

cHr-o-Po,2

6-Phosphoglucono6-lactone

6-Phosphogluconate

Ribulose
5-phosphate

Intermediate

Fi'gure B-41

l:.e next reaction is the hydrolysis of that cyclic ester. The enzyme that catalyzes ---: reaction is called lactonase. Lactonase hydrolyzes the lactone and lets the ."-r 3 open to form 6-phosphogluconate. Recall that the best way to release carbon :.--xide is to have a keto function at the beta position. In the structure of 5-

;rright

by The Berkeley Review

207

The Berkeley Review Specializing in MCAT Preparation

Biology

Metabolic Pathways

Pentose Phosphate pathway

phosphogluconate we have an alcohol function at the C-3 position. If we could relieve that hydroxyl group of its hydrogen, we would have an intermediate that is a beta-keto acid. This reaction is catalyzed by NADpo and the enzyme dphosphogluconate dehydrogenase. The transitional state, which is the beta-keto acid, rapidly decatboxylates to form ribulose-5-phosphate. This set of reactions is shown in Figure 8-41. The reactions shown in Figure g-40 and Figure g-41 constifute an oxidatiae way to form S-carbon sugars.

I
il

The enzyme phosphopentose isomerase is able to isomerize ribulose-5phosphate to ribose-5-phosphate. This reaction is shown in Figure g-42 and proceeds through an enediol intermediate. This reaction should look familiar to you. Recall the interconversion between glucose-6-phosphate and fructcse-6phosphate in glycolysis. The intermediate in that reaction was an enediol, too.

il
,il

ilt

fi

o.H
CH.-OH
l'

ilh

CH.OH

\\r

C=O
I

il
I

C
I I

CPhosnhoI

il.

OH

H-C- OH H-C* OH
I

,@

H-C- OH H-C-

H-C- OH
H.C_ OH
I

@il!

pr,,ioro CH2-O-PO32- isomerase

OH

H-C- OH
I

cHz-o-Poj2Enediol Intermediate

cH2-o-PO32-

Ribulose
5-phosphate

Ribose 5-phosphate

Figure 8-42
Formation of Ribose 5-phosphate

What is the value of having something like NADP? It turns out that this molecuc serves a special purpose. AII of the reactions that we have considered so far a:e energy releasing reactions in which ATP was generated. Because they rr*e energy releasing reactions they were also breakdown reactions. The entropy tr:w
increasing. are degradative and energy releasing are referred to am catabolic reactions (catabolism). NAD is the coenzyme that is involved in t}'cmer reactions. In contrast, there are reactions that require energy to decrease Fre entropy by putting things together. These are biosynthetic reactions and ::e collectively characterized by the term nnabollsn. NADP is the coenzyme that m involved in these reactions. There are a variety of things that can happen to the S-carbon sugars that we tu just mentioned. For example, enzymes such as a transketolase, a transaldo or even an epimerase can make use of a specific S-carbon sugar as their Let's examine some of these reactions. The transketolase enzyme can catalyze the synthesis of xylulose-S-phosp

All of the reactions that

and erythrose- -phosphate from glyceraldehyde-3-phosphate and f phosphate. The coenzyme involved in this reaction is thiamine pyrophosi (TPP). This is shown in Figure 8-43. Note that the "boxed" portions of molecules are those portions that are being transferred.

Copyright @ by The Berkeley Review

204

The Berkeley Specializing in MCAT

Biology

Metabolic Pathways

Pentose Phosphate Pathway

o.H
C
I

T&;oH I I i= o I ,,on,- l- cH"oH I |-.r-------------rketolase li HO- C-H


I

o.H
C
I

H-C- OH
I

TPP HO_

C_H
I

H-C- OH
I

H-C_ OH
I

H-C- OH
I

H.CI

OH

H-C- OH
I

cHr-o-Porz
Glyceraldehyde
3-phosphate

CH2-O-PO32-

CH2-O-PO32-

cH2-o-Po32'

Fructose 6-phosphate

Xylulose
5-phosphate

Erythrose
4-phosphate

Figure B-45
Intermediates in the Pentose Phosphate Pathway

Xylulose-5-phosphate can also be converted to ribulose-S-phosphate by the enzyme phosphopentose epimerase. This enzyme is a member of the isomerase class of enzymes. Note that these two molecules are epimers at the C-3 carbon. This is shown in Figure 8-44.
cH2-oH I I c-o Ir I HO- C-H H-C- OH
CHr-OH

NAD-iinked
Phosphopentose epimerase

tC=
I

o
OH OH

it i lt

H.CH.CI

cH2-o-Po32 Xylulose S-phosphate

cH2-o-Po32-

Ribulose
5-phosphate

Figure 8-44

-low could we make this epimer? The hydroxyl function at the C-3 carbon of
,.r-lulose-5-phosphate could be oxidized to the corresponding keto function. This '",-ould eliminate the chirality. Reduction of that keto function could then give the

=pimeric hydroxyl function at the C-3 carbon of ribulose-5-phosphate. The :oenzyme NAD islnvolved in this oxidation-reduction scheme. NAD would be a :atalyst as it is not involved stoichiometrically.
lhrce we have ribulose-S-phosphate it can undergo an isomerization rcaction to jve ribose-S-phosphate as shown in Figure 8-42. Ribose-S-phosphate could

:,,-entually become incorporated into biomolecules such as RNA, DNA, ATP,

,:d NAD.
Irythrose-4-phosphate can react with DHAP via an sldol condensation and the aldolase enzyme to give sedoheptulose-"1.,7-diphosphate (first isolated from .',-ocados, which belong to a family of plants generally known as sedum plants--.ence the prefix). Sedoheptulose-1,7-diphosphate can be converted to sedo":i eptulose-7-phosphate by a hydrolase enzyme known as sedoheptulose-1,7Jiphosphate phosphatase. This is shown in Figure 8-45.

-,,.pyright @ by The Berkeley Review

209

The Berkeley Review Specializing in MCAT Preparation

Biology

Metabolic Pathways

Pentose Phosphate ? athw ay

o..

'c ,H
I

H-C- OH H-C- OH
I

t+

o Ho- i-H s-l- on CH2-o-Po32- u-b- oH


c=

?H2-O-PO32sed-1.7-dP phosphatase-

CH.-OH

c=o
I

t'

+ H2o

HO- C-H
I

H-CI I

OH

- Atd"k* H-C- OH f= o
cH2-oH
CH2-O-PO32-

l-t

H-C- OH H-C- OH
I

CH2-o-Po32-

CH2-o-Po32-

Erythrose
4-phosphate

DHAP

Sedoheptulose 1,7-diphosphate

Sedoheptulose 7-phosphate

Figure 8-45
s-edoheptulose-z-phosphate can react with glyceraldehyde-3-phosphate to form ribose-S-phosphate and xylulose-S-phosphate as shown #rig.rru s-46. Thi$ reaction is catalyzed by the transketolase enzyme.

HO- C-H
I

o.H .'c,
OH

H-CI

H-CI I

OH

o. H ..c, H-l- on

Transketolase
-

H-C- OH
I

H-CI I

OH

H-C- OH
cH2-o-Po32Sedoheptulose 7-phosphate

H-C- OH
cH2-O-PO32-

cH2-o-Po32-

Glyceraldehyde
3-phosphate

Ribose 5-phosphate

Figure 8.46

form fructose-6-phosphate and erythro"e-+-pho"pirate. This reaction


catalyzedby a transaldolase enzyme
as

sedoheptulose-7-phosphate and glyceraldehyde-3-phosphate can also react


shown in figure
A_+2.

H-CI

OH

H-CI

OH

o.. ,H :+
C

Transaldolase

o..,11 .C
I

H-C_ OH
I

H-C- OH
I

u-l- on
I

H.C_ OH
I

CH2-O-PO32-

CH2-O-PO32-

CH2-o-lqzErythrose
4-phosphate

Sedoheptulose 7-phosphate

Glyceraldehyde
3-phosphate

Figure B-47

Copyright @ by The Berkeley Review

2to

The Berkeley Specializing in MCAT prep

Biology

Metabolic Pathways

Pentose Phosphate Pathway

Note the portion of sedoheptulose-7-phosphate that is being transferred to glyceraldehyde-3-phosphate. The reactions shown in Figure 8-42 through Figure 8-47 represent anon-oxidatiae way to generate S-carbon sugars.
The pentose phosphate pathway, which occurs in the cytosol of cells, is one way

to get five carbon sugars and NADPH. There is also a mitochondrial-linked method of obtaining NADPH (which we will examine during fatty acid
metabolism). Most eukaryotic cells have mitochondria. However, one of the most common types of (mature) eukaryotic cells that do not have mitochondria are the erythrocytes (red blood cells). If mature erythrocytes do not have mitochondria, then they must obtain their NADPH from the pentose phosphate pathway.

-\s oxygen becomes reduced it goes through a number of intermediate stages such as the superoxide radical, hydrogen percxide, and hydroxyl free radical oefore it is converted into water. All of these intermediate species are "toxic" in -he sense that any one of them (outside the usual enzymatic channel of becoming '-';ater) can cause cellular damage (which is a means of aging). There are various :ompounds and enzymes which can scavenge these radicals and convert them -nto less toxic substances. For example, compounds like vitamin C, vitamin E iocopherols) and ergothioneine can act as free radical scavengers (or anti:xidants). Superoxide dismutase (SOD) can catalyze the conversion of the 'uperoxide radical into hydrogen peroxide and oxygen. Hydrogen peroxide can :eact with an enzyme called catalase and be converted into water and oxygen.
NADP"

NADPH + H+

Glutathione
reductase Reduced

Glutathione (GSH)

Oxidized Glutathione
(GSSG)

w:'
F-rgure

\/\,/
Glutathutne peroxidase

n:\-,

a.4a

:---.-drogen peroxide can also react with an enzyme called glutathione peroxidase rnl be reduced to water. Reduced glutathione (GSH) reacts with the peroxide to

r::=L water and the oxidized form of glutathione (GSSG). Oxidized glutathione is

::luced back to the sulfhydryl form (GSH) by NADPH. The enzyme that
:-:lvzes this reaction is glutathione reductase. This is shown in Figure
8-48.

lLere are cases in which humans lack one or more of these "housekeeping" s?',"rnes. For example, there are individuals who are catalase negative (they lack f:-. enzyme) and who still manage to survive. Individuals have not been found ,,,':.o are completely lacking in either superoxide dismutase or in glutathione re:.'rxidase. This suggests that these two enzymes are quite essential for normal
lL'::cgical function.

l I

l.;rlright

by The Berkeley Review

2ll

The Berkeley Review Specializing in MCAT Preparation

Biology

Metabolic Pathways
g,enase (see

Pentose Phosphate p athw ay

Figure 8-40), then that person would not 6e abie to synthesize NADPH' This sex-linked trait is the single largest class of all human mutations and_ is quite prevalent in malaria-infested regions of the world like the Mediterranean area, central Africa, the Middle Eist, Lrdia, southeast Asia, and China. It turns out that glucose-6-phosphate deficiency is one means of protection against the malarial p2rasite .pla-smodium falciparum.lbtner means of protection involves the sickle-cell trait.) How is it that a glucose-6-phosphate deficiency
affords protection against this parasiie? Glucose-6-phosphate dehydrogenase deficiency is sex-linked. It is on the Xchromosome. Females are XX while males are Xy. If a male has glucose-G phosphate dehydrogenase deficiency, he is said to be hemizygous for that defecL A female may carry the defect on one of her X-chromosomei 6ut not on the other one. she is said to be heterozygous for that defect. Heterozygotes for this deficiency turn out to have red blood cells that are about ten times more resistant to the malarial parasite than normal wild type individuals who do not have this deficiency. why? The malarial parasite requires products of the pentose phosphate pathway and reduced glutathionJ for thlir survival. If there is a glucose-6-phosphate dehydrogenase deficiency, then these products are limitedNote that the female of the species is the one being protected because she is the one who is heterozygous for the trait. she is the one-who bears the offspringl

If an individual were deficient in- the enzyme glucose-6-phosphate dehydro-

jaundice.

Individuals with glucose-d-phosphate dehydrogenase deficiency are unable to form NADPH and therefore unable to form ieduced glutathione (GSH) as shown in Figure 8-48. If GSH is not formed, then p"ro"*id", are free to lurk within the cell and cause damage. Erythrocytes have a typical lifetime of abogt Jz! days before they are degraded in the spreen. However, the erythrocytes individuals with this deficiency have a relatively short life span (about t{ da1 femglyli9 anemia (lysis of the red blood cells) iesults and hlmogl obrn is retea: into the blood. As the hemoglobin is destroyed the protein portLr, (i.e., globi and the iron can be recycled. However, the iron-free porphyrin portioriof heme is degraded in the liaer (and spreen and bone marrow) to a compo, called bilirubin.If the concentration of bilirubin were to exceed what thl hrr could excrete, a condition of hyperbilirubinemia results in which bilirubi diffuses into the tissues and turns yellow. This condition is referred to

Copyright @ by The Berkeley Review

212

Biology
GlucoheogensiS::

Metabolic Pathways

Gluconeogenesis

Gluconeogenesis is the synthesis of glucose from non-carbohydrate precursors such as lactate, amino acids like alanine, and glyceroL Recall that during vigorous exercise lactic acid accumulates in skeletal muscle. We had mentioned that lactate is a dead end in metabolism, unless is can be converted back to pyruuate.In order to convert lactate back into pyruvate it must first be carried by the blood to the liver. At the liver, lactate is converted to pyruvate, which can be converted into glucose by way of gluconeogenesis. Glucose can leave the liver and return to the skeletal muscle to once again undergo glycolysis. These series of reactions, as shown in Figure 8-49, ate referred to as the Cori cycle.

Figure 8-49
The Cori Cycle.

llloiecules like lactate, alanine, and glycerol can be converted into glucose by :luconeogenesis. Note that all three of these compounds contain three carbon "::oms (C3). It turns out that animals cannot get a net conversion of a two carbon C2)compound like acetyl CoA into glucose. Recall that when acetyl CoA enters -nto the Krebs cycle and combines with OAA, two carbon atoms leave as CO2. In :ther words, the OAA that is regenerated is not synthesized de novo when acetyl CoA is oxidized in the Krebs cycle. Animals lack the necessary enzymes that will .llow the conversion of a two carbon acetyl unit into glucose. However, plants :nd many bacteria can make use of these two carbon precursors because they ::-ioe the enzymes necessary for the proper convetsions. They use the glyoxylate cl'cle and bypass,the two decarboxylation reactions in the Krebs cycle by :onverting the two carbon acetyl units into a four carbon succinate molecule. ,.'-ith this in mind, let's consider gluconeogenesis.
--',-ithin

the cytosol of (eukaryotic) liver cells the ratio of NADH/NAD@ islow, favoring the oxidation of lactate to pyruvate. Once you have pyruvate you -:us right think that you can convert it to phosphoenolpyruvate (PEP) by a simple :er.ersal of the reaction at Step 10 in glycolysis. Recall that the AGo for the -onversion of PEP to pyruvate in glycolysis is -7.5 Kcals/mol. If we were to :everse this reaction, the AGo' would be +7.5 Kcals/mol. This reaction must be :,passed for one with a more favorable standard free energy change.

ihis bypass is catalyzed by the enzyme pyruvate

carboxylase. Pyruvate :arboxylase is located inside the mitochondrial matrix of the liver cells and :-rntains a biotin prosthetic group that carries activated CO2. CO2 was rctivated" at the expense of a molecule of ATP. In other words, ATP facilitated --:re attachment of CO2 to biotin. Pyruvate then diffuses into the mitochondrial
lopyright
@

by The Berkeley Review

2t3

The Berkeley Review Specializing in MCAT Preparation


I

Biology

Metabolic Pathways

Gluconeogenesis

matrix and is carboxylated.to.form oAA. pylu,vate carboxylase is an example of a Iigase enzyme because a-high energy bond is being used. ir *".gv was not used, the enzyme would have been a lyase. This is showi in Figure g_5'0.

ilm:

qpq

OA

:mum

rsrlr,

ngm

@
I
OAA Figure 8,5O
I

Omu dfiilrt
idffie

Pyruvate I

erytor

lCarboxylasel

Ws'r

pyruvate carboxylase is actiaatedby high leaels of acetyr CoA. I: tf9r9 alenigh levels of acetyl coA, then acetyl CoA mult not be condensing with oAA. why? Because the oAA levels are low and. there are not enough to meet the demand' The forma.tion of oAA from pyruvate is called an anaplerotic regctior: (from the Greek, meaning to "fill up"). wiat happens to this oeaz If the cell L. low in ATP, oAA will enter the Krebs cycle und .ondense with acetyl CoA. Thi_. will lead to the eventual synthesis of more ATp. However, if the cell has plenty c: ATP, then OAA will be utilized for gluconeogenesis.
!n1t

It turns o-u!

@
;hdir"
msprT
Mturffi

",lftrmffi

dryft
w@ffi

N.+o-

9o' oL ApP , f *r={ .crP \ NADH pyruvare n^ + H+ potuntial co: ll t-

Lactate 'K"

Glxose

ll

ry,iy Cycle
Cytosol

ADH + H+

l,_ *AD+
Malate

ry\'m'ate

OAA

ff,
Figure B-51

7Z-\

Malare

i|,1 TT:

NIo.

Mitochondrial Matrix

oAA is next reduced to form malate by NADH-linked malate dehydrogenase This enzyme is in the oxidoreductase crass of enzymes. Malate is iran$ortec across the mitochondrial membrane and is reoxidized to oAA in the cytosol t,r an NADo-linked Ttljt" dehydrogenase. oAA is decarboxylated ani phosphorylated by GTp to givephoiphoenolpyruaafe (pEp). This reaction is catalyzed by PEP carboxykinase and can be r"".t itr Figure g-d1.
right @ by The Berkeley Review

214

The Berkeley Review Specializing in MCAT preparation

Biology

Metabolic Pathways

Gluconeogenesis

l{t

rl

In reaction sequences shown in Figure 8-5L we have the potential for a futile cycle. Why? If we go from PEP to pyruvate, we synthesize one ATP. Pyruvate to OAA costs us an GTP. OAA to PEP costs us another ATP. We end up with a net loss of one NTP. If we were to continue around this cycle, we would eventually run out of NTPs and still not have gotten anywhere. It would be a futile effort. I{owever, there are controls that regulate this potential futile cycle. If we were to remove those controls, a lot of heat would be generated.
Once you have PEP the rest of the reactions in glycolysis are reversible and the

equilibrium

will favor moving

back towards glucose until fructose-L,6-

diphosphate is reached. This is true as long as there are high levels of ATP (i.e., the energy charge is high). High leaels of ATP turn out to allosterically inhibit pyruvate kinase, the enzyme which converts PEP to pyruvate'
Recall that the AGo' for the conversion of Fructose-5-phosphate to Fructose-1,6-

diphosphate was about '3.4 kcals/mol. This reaction was catalyzedby phospho:ructokinase.Instead of trying to reverse this reaction it would be much easier to :rydrolyze the phosphate at the C-1 position.
Recall that hydrolysis reactions are always favorable. The enzyme that catalyzes

this reaction is in the hydrolase class of enzymes and is called fructose-1,6diphosphate phosphatase, This is shown in Figure 8-52. Again, there is the rossibility of another potential futile cycle.
ADP

cH?-oH
I

, *l-l
F ructo
se-

ilo CH"-O-P-O
C= lv

C=O
I

O,.,

HO_ C-H
I

1, 6- Dip ho sp

hate

HO- C-H
I

H.C_ OH

Phosphatase

H-C- OH
I

H.C-

OH O o ril cH?-o-P-o
O^ I
I

H-CPi

o cH"-o-P-o -l oo

O r OH ll

Fructose-6-phosphate

Fructose- 1,6-diphosphate

Figure 8-52 iructose-6-phosphate is in equilibrium with Glucose-6-phosphate. Glucose-6rhosphate can be used to make glycogen (a storage form of glucose) or it can be :onverted into glucose. The brain uses about 120 grams of glucose per day as an :nergy source. It would be advantageous, then, to have a means for the :onversion of glucose-6-phosphate into glucose.
R.ecall

that the AGo'value for the reaction of glucose to glucose-6-phosphate is :bout -4.0 kcals/mol. This reaction is catalyzedby hexokinase. It would be easier :o overcome this enerly barrier by simply hydrolyzing the phosphate from the C-6 :osition of glucose-6-phosphate. This reaction is catalyzed by glucose-6phosphate phosphatase. These reactions represent yet another potential futile ,tlcle.This is shown in Figure 8-53.

Copyright @ by The Berkeley Review

2t5

The BerkeleY Review Specializing in MCAT Preparation

Biology

Metabolic pathways

Gluconeogenesis

HO ,{,
I

ATP
Step OH
1

HO
AGo'= -4.0

'i'
I I I I

H-CI

H-CGlucose-6-phosphate

OH

HO- C-H
I

HO_ C.H

H-C_ OH
I

H.C- OH
H-CHzo
lll

H-C- OH
I

OH

cH2-OH

cH2

Glucose
Figure 8-58

- o- p- o" Og

t:t

Glucose_6_phosphate

Recall that glycorysis.(from grucose to pyruvate) will give us a net yield of 2 Glt'coneogenesis (from pyruvate io gtn"or"; +Tlr' *itt".ori,r, a erpr. we cm th-ink of this cycle as one large po"tential futile loop t}rut gi .J.,, u net loss of I
ATPs.

"orra

Copyright @ by The Berkeley Review

216.

The Berkeley Specializing in MCAT prer

Biotogy
F

Metabolic Pathways

Fatty Acid Oxidation

Let's look at fatty acid metabolism. We will start with fats or triglycerides which have the general structure shown in Figure g-54. The carboxyl function of the fatty acid is in an carboxyester linkage iui*, tnu hydroxyl or ine glycerol. The structure shown in Figure g-54 has three carboxyester rinkages. The most .ommon fatty acids are 16 or 1g carbon atoms long and they lre the energy storage form that is most frequently used in animals uid ptur",tr.'

H2C-

is more nergy available per unit weight of triglyceride than of hydrates Tut". -rke glycogen. You can see that i? you think o"f -CH2- u', u"i"j u,r" form in which :arbon exists. If we were to oxidize that unit, then it wou*ld take 1.5 oxygen ::rolecules to bring it to Co2 and H2o. In contrast, if you were to think of a --arbohydrate structure like -CHoH-, then you would fir,d thut it is already :artially oxidized. As a result, only 1 oxygen molecule is required to oxidize that :arbon and hydrogen completely to CO2Ind H2O. -: the amount of energy that had been concentrated in these neutral fats were -'istead stored in the-form of glycogen, then you would be grossry overweight. here are a couple of reasons tot tfrit. Fats are at a lower oxid.ation state. The ;'"-erage oxidation state of the carbons in a fatty acid molecule is -2. The average " ddation state of the carbons in a carbohydrate is 0. In the pro."r, of turning thit ::-rbon into Co2 there is a rot more oxidation taking plaie. The result is more being produced from the buming of a gram oifut to a gram of ":.9-y :*bohydrate. Fats are also fairly hydroplobic"and so when "o-pured thiy are stored there -: not very much water. This is quite different from the stoiage of glycogen :rere we find the molecule to be iull of hl/droxyl groups. There is a lot of water ,;:red along with the glycogen. one disadvantage lrrat iat has is that it cannot be ::tabolized anaerobically. Fats have to be metabolized aerobicaily.

H-9- O- C-

lo I

O ctt

cH2-(cH2),iCH3

lo lil

CH2_(CH2),,_CHr

H2C-

O- c-

CH2-(CH2)n_CH3

A Triacylglycerol

Figure 8.84

-pase' To mobilize triglycerides (or neutral fats), the first step would be the 't,;'clysis of the carboxyester bond. This is accomplished by hydrolysis of the :-:-r-ceride into a molecule of glycerol and 3 fatty acid residue's. This is shown _ f tgure 6-55.

-;t s consider the way in which fat is metabolized. This pathway involves g ::7\'matic steps. The first step that we want to consider involves the enzyme

",fIO lil

c- cH2-(cHrn-cH3
CH2-(CH,)n-CH3

H-q- O- C-

H'C- O- C- CH2-(CH)n-CH3

lo til

Lipase + L_____________z
3 H2O

H2C- OH
I

H-C- OH
I

H2C- OH

Glycerol

o il HO- C- Rl o tl HO- C- R2 o il HO- C- R3


Fatty Acids

ffqnare 8-58

r: : rriefly consider what happens to the fatty acids and, glycerol after the lipase "a:x::ron shown in Figure 8-55 has taken place. The fatty uliar tnut are produced * .L- :e undergo a series of reactions

'

L-

ur,d b" converted to acetyl CoA. Glycerol :e also be converted to acetyl CoA, but by a different series of reactions.
@

ur -rsht

by The Berkeley Review

2t7

The Berkeley Review Specializing in MCAT preparation

Biology

Metabolic Pathways

Fatty Acid Oxidation

Acetyl CoA will then be able to enter into the Krebs cycle and energy, in the form of ATP, will be produced as we have previously discussed. A very general vier,rof this outline is shown in Figure 8-56.

FATS
Glycogen

Glycerol

\l
Figure 8-56

Occurs the

in cytosol ;ytosol

FattY Acids
I

DHAP

r---l il
r Occurs in the I mitochondrion

I I

Pyruva te

tsAcetylCoA

it

ll

Glycerol Metabolism
The activation of glycerol is catalyzed by glycerol kinase and ATP. Glycerol-} phosphate is produced. The coenzyme NAD@ reacts with glycerol-3-phosphate

to form DHAP. This is catalyzed by glycerol phosphate dehydrogenase ani occurs in the cell cytosol. This general reaction sequence is shown in Figure 8-57.
NADH H2C- OH
I

UI

H-C_ OH
I

ArP oPt \_-,/ t--------------+


Glycerol
Kinase

NAD+ H'CI 't\,/ OH ' rH-C-OH

+ H+

H2C- OH

ffi

c=o
I

,i!

H2C- OH

Glycerol

H,l- opo.z- GlYcero! H2C- OPO32phosphate Glycerol delrydrogenase DHAP


3-phosphate

0m

Figure 8.57

Activation of Fatty Acids


The second step that we want to consider is the actiuation of the fatty acids. We

n'il

find that fatty acids are degraded 2 carbons at a time. If we are going to sph: these carbons off two at a time, then we will need some type of "handle" at the ft position of the fatty acid. What we will need is a keto group at the p-position. L we had a p-keto acid, the carboxyl function would decarboxylate and we wouli lose that carbon atom. As a result this is not handled as a p-keto acid but rather a. the thioester. The thioester of a p-keto acid does not decarboxylate. How do rre
form this thioester linkage?

n
h
Wfr1

ffill

r die

Copyright @ by The Berkeley Review

2ta

The Berkeley Revieu Specializing in MCAT Preparation

{i,m

Biology

Metabolic Pathways

Fatty Acid Oxidation

AMp and pyrophosphate (ppJ. pyrophosphate wilr be nvdrolyzed in the presence of water and pyrophorpt two morecules of -'t.hophosphate, giuing a AGo'of another -73Kcal/mol. "t"r"io The activation of the fatty acid is now complete as shown in Figure 8-58. This activation reaction takes placl ;'n the outer mitochondrial membrane and is catalyzed,by the enzyme acyl CoA srnthetase (which is in the ligase enzyme class).
rr O R-C-O
Fatty Acid

rydrolyzed to

The driving force for forming the thioester linkage between the sulfhydryr group of CoA and the carboxyr gtolp ofa fatty acid is erp. If we are going to do this reaction with ATp, then one phosphate grou-p will not be enoughlTheiydrolysis energy of a thioester is about -z.b kcal/mol-. If we allowed thi rrr" of only one -7.3 kcar/mol. 1sh gne.rsr phosphate bond, then we would have a lC"tor clearly this reaction will not proceed very well. what we find ""ry here is that ATp is

ATP
Acyl-CoA
Synthetase

R_ C- AMP Acyl Adenylate

PP;

-. R_C-AMP + HS-CoA :4
Acyl Adenylate

R_ C_

S_CoA

AMP

Acyl-CoA

Figure B-EB lnce,the fatty acid is activated in the cytosol of the cell it needs to be transported - to the mitochondrial matrix where it can be oxidized. Activated fatty acyl CoA : olecules are shuttled across the inner mitochondrial membrane by .u*itir,u. l:ce the activated fatty acid is released in the mitochondrial matrix, carnitine ' j] return to the cytosolic medium and the process will repeat itself. The B-Oxidation pathway l:-lc.e the acyl-CoA molecule is in the mitochondrial matrix, we can begin B_ : ':dation. The first four steps of B-oxidation are highly reminiscent of the steps -:-at we saw in the Krebs cycle. The third step in our breakdown reaction is the : -:mation of a trans double bond between the alpha and the beta carbons in the

ic'l-CoA structure.'This is accomplished by an oxidation of the acyl_CoA '::ucture with FAD. lhe e11zme involved is acyl-CoA dehydrogenase. This . r:dation yields an dnoyl-CoA.
r :ereas

;. lrvn in Figure 8-59.

Acyl-CoA slightry ,eru^bie, succinic acid enoyl-CoA slightly resembles fumnric qcid'n the Krebs cycle. This is

p.- cH2

- cHz-cHz- t- s- coa *
Fatty Acyl-CoA

g il\/

"to

FiDH'?
RcH2-a= |

Ho

Oxidation

l-'J- r-

Acyl-CoA
dehydrogenase

"oo

Enoyl-CoA

ffigure B-59
::ep four we have hydration of the double bond in enoyl-CoA. The two ends of 1'-' molecule are not symmetrical and therefore the hydration of enoyl-CoA is ,::-tospecific. The enzyme that catalyzes this reaction is enoyl-CoA hydratase. we r - only get the L-isomer, which is L-Hydroxyacyl-coA as shown in Figure g-60.

l'right

by The Berkeley Review

219

The Berkeley Review Specializing in MCAT preparation

Biology
cycle.

Metabolic Pathways

Fatty Acid Oxidation

This reaction is analogous to the hydration of fumarate to malate in the Krebs

R-CHr-9=C-c-sR-CHr-c-C- C- s'lHydration'ri CoA -H HH Enoyl-coA Hydratase L-Hydroxyacyl-CoA Enoyl-CoA


Figure 8-6O

Ho"rpHoHo ril\ttil

CoA

,l

lw

r@

In step fiae we have another oxidation reaction where we introduce a keto group at the B-position. This reaction is catalyzed by the enzyme L-3-hydroxyacyl-CoA dehydrogenase. The coenzyme involved is NADe. The reaction is shown h Figure 8-61. This reaction is analogous to oxidation of malate to oxaloacetate in
the Krebs cycle.
NADH

NAD* HOHO,IOHO I I " ,-coa - ^jS41 R-cH2-[-l-[-,-"oo R-cH2-9-f-c-l oxidation

+T*

H H L]-Hydroxyacyl-CoA L-Hydroxyacyl-CoA dehydrogen'ase

Ketoacyl-CoA

Figure 8-61

In step six we have the cleavage of ketoacyl-CoA between the s and B carbon_.. -4, second molecule of CoA-SH is used in the process. The products are an acetvL CoA molecule (which can be utilized in the Krebs cycle) and a fatty acyl-CoA molecule that has been shortenedby 2 carbon atoms. This is shown in Figure 84IThe enzyme that catalyzes this thiolytic cleaaage is B-ketothiolase. The fatty aqll. CoA molecule that we just produced is already actiaated, which means that we not have to repeat the activation step shown in Figure 8-58.

Figure 8-62

The Oxidation of Palmitic Acid (a C16 even-numbered fatty acid) If we were to completely oxidize palmitate using the B-oxidation pathway, would go through the cycle that we just described 7 times to get 8 two-ca acetyl CoA fragments, 7 FADH2 molecules and 7 NADH + He molecules. can be seen in Figure 8-63. In return for these products we will get a wealth ATP's via the Krebs cycle, electron transport and oxidative phosphorylation. point of this process is that the sum of the ATP liberated by the comp oxidation of a f.atty acid makes this a very elegant breakdown process.
Copyright @ by The Berkeley Review

ltg

'@',]i

1m

mfr

220

The Berkeley Specializing in MCAT Prepara

Biology

Metabolic pathways

Fatty Acid Oxidation

cH3-CH2-l-cH2_cH2_l-cH2_CH2_l_cH2_CH2_l_CH2_CH2_l_CH2_CH2-._CH2_""r_'_""r_aoo:

#8

#7

#6

#5

#4

#3

#2

#I

the net production of ATp from the 'v:,ul: of one morecule of palmitic acid. oxidation In the ,t"p we used 1 ATp ar{ we hydrolyzed the pyrophosphate bond. In activatio" other words, we have used 2 u:"rgl-phosphate bonds to aciivate the fatry acid for lql B-o"iautior,. At rhe end rf the B-oxidation process we will have formed the eqrrirrai" nt of 129high energy :hosphate bonds. The rest of the steps in our process are outlined in Table g_1.

now in a position to calculate

Process
Activation Oxidation (7 rounds)

ATP Equivalents
-02

FADH2 (worrh 2 ATp's each) 7 NADH (worrh 3 ATp's each)


Z

+14 +21

Acetyl-CoA (catabolism) 8 Acetyl-CoA's enter Krebs Cycle

8Acetyl-CoAx3NADH 8Acetyl-CoAxlFADH2 8Acetyl-CoAx l GTp


B,l

+72 +16 +08 + 129


H

Table

fgl Fis process is about -2g40 Kcal/mol. =".,19t. :'''ailable (which is -2340

The fraction of energy Kcal/mol) that is actually trapped as ATp is about 40% i29 x 7.3 = 9 4'1 .7, and then g4]^.T l2g40 x 100'/, = +OXS.''

RAI-^ [- s-con T0c


cds-Enoyl-CoA

lH

ii:uation is that you arrive at a double bond in the B,y-position after B-oxidation ;'' :i-le the other situation is that you arrive at a double bond at the a,B_position
a:'ter B-oxidation. -'he

Lnsaturated Fatty nliOs )-rt.all fatty acids are completery saturated with hydrogen atoms. some have ::uble bonds in them. These are-referred to as unsaiuratid yatty acid.s. There are :"'^'o type of situations concerning double bonds in unsaturatei fatty acids. one

ll lro*rroro

n^tf

HO
H

[,

s_coA

lrazs-Enoyl-CoA

double bond is at the B,y-position, we need to convert to a double bond at h: a,B-position. The enzyme tirat catalyzes this reaction is it an isomerase. This is 5r r\-vn in Figure 8-64. onq.q,you have the a,B-position established you are at the k;el of enoyl-CoA in B-oxiaation. The enoyl-C^oA double bond is trans. If it were 'cs, it would need to be converted to trans Ly an isomerase.

li

I
lJ
B-oxidation

Figure 8-64

:ryright

by The Berkeley Review

221

The Berkeley Review Specializing in MCAT preparation

Biology

Metabolic Pathways

Fatty Acid Oxidation

The Oxidation of Pelargonic Acid (a C11 odd-numbered fatty acid) Pelargonic acid is a C-11 odd chained fatty acid. If we were to completely oxidize pelargonic acid using the p-oxidation pathway, we would go through the cvcle that we have been discussing 4 times to get 4 two-carbon acetyl CoA residues and 1. three-carbon propionyl-CoA residue. This is shown in Figure 8-65.
cH 3-cH2-cH2- -cH2-cH2- l-CH2-CH2,|-CH2-CH2- -CH2_COO| |

#5 Figure 8,65

#4

#3

#2

#1

In order to utilize propionyl-CoA we must carboxylate it to

Propionyl-CoA Metabolism in Animals

fora. Methylmalonyl CoA. This reaction is catalyzed by propionyr CoA carbon (a biotin enzyme). This enzyme is a ligase because ATp is simul converted to AMP and PPi. Pyrophosphate is subsequent$ hydrolyzed:,r molecules of orthophosphate. D-Methylmalonyl-CoAis converted to L-\fic malonyl-CoA by a racemase enzyme (which is in the class of enzvmes refm as isomerases). L-Methylmalonyl-CoA is next converted to succinyl-CoA rw enzyme methylmalonyl-CoA mutase. This is a unique enzyme beca& contains vitamin 812 @obalamin) as its coenzyme. [There is only one ;-ffi reaction known in mammals dependent on vitamin B12 and that is the fonr:,mu of methionine by methylation of a homocysteine residue.l once succinyl{cA formed it can then enter into the Krebs cycle.

Can Fatty Acids be converted into Carbohydrates? Would it be possible for animals to convert fatty acids into carbohydrates? tr*ul know that when fatty acids are degraded by the $-oxidation pathway, a wealth n 2-carbon acetyl-CoA units are synthesized. These acetyl-CoA units will enter i:-ru the Krebs cycle and condense with OAA to form citraie. F{owever, as citrate goem around the cycle there are two decarboxylation rcactions, catalyzed by isocif,am dehydrogenase and a-ketoglutarate dehydrogenase. Even though we will sts form oAA as we continue around the cycle, we will have lost 2 carbons in tF.e process, which means that we would be unable to convert fatty acids in:-r carbohydrate material. Plants and many bacteria, though, can utilize acetyl-CoA for energy production and biosyntheses by using a reaction sequence cailed tle glyoxylate cycle. However, animals can degrade carbohydrates to acetyl-coA units and then take those acetyl-CoA units to synthesize fatty acids.

pathways, then the rate at which the Krebs cycle operated would begin to declinre (and might even stall). One of the most importanf a naplerotic reacti6ns (from the Greek, to "fill up") is the synthesis of oAA from pyruvate and Co2. This reacticni is catalyzed by the enzyme pyruvate carboxylase. For example, if a cell r.t,a-. exclusiaely catabolizing fatty acids to obtain a high level of ATp, and if there o,,a-< no carbohydrate catabolism taking place, then the anaplerotic reaction of pyruvate to OAA wouldlot take place. The result is that the Krebs cycle woulo run down. In other words, fatty acid degradation needs the flickering flame ot lalbohydrate degradationto keep it going (i.e., a little conversion of plruvate to oAA is needed to ensure that fatty acid degradation will continue).
Copyright @ by The Berkeley Review

Not only does the Krebs cycle function in the oxidative catabolism of aminLo acids, fatty acids, and carbohydrates, but it also serves as a primary starting poir,l in many biosynthetic reactions for which it is able to provide precursors. fJ tnese precursors were removed from the Krebs cycle for various other metabohl

niinilffifir

{i$rrrrrlm!

222

The Berkeley Review Specializing in MCAT preparation

Biology
UrearrG$dt6
-: ive hydrolyze proteins, we

Metabolic Pathways

Urea Cycle

:,

will be able to obtain a variety of amino acids. The acids that we obtain can then be used in a variety of reactions, one of 'nich is protein synthesis. However, for most amino acidj the amino group can := removed to yield the s-keto acids which can then be used in citric icid cycle -::ermediates to eventually give ATp, Co2 and H2o. This is shown in Figure gLet's focus on the Phase I and Phase II (we have already looked at phlase III "o

':',iro

lhe Krebs cycle).

Proteins

H3\

tT: ffi
u
Protein Synthesis

cr-Keto Acids

Krebs cycle Intermediates

il il

il pnu,. rrr U
ATP, CO2, H2O

Figure 8-66

Phase

-:om an amino acid to c-ketoglutarate as shown in Figure g-67. (The portions -:.at are in boxes will form the cr-keto acid.) o-ketollutarate is the icceptor :-rolecule of the amino group from most amino acids. TLe enzyme that catalyzes ris reaction is an aminotransferase (also called a transaminase). There will be a -:ecific aminotransferase for each amino acid.
-

-:. this phase of amino acid degradation (the major site in mammals being the ':t'er) we are essentially dealing with two reactions. The first is amino transfer

:.etoglutarate, then you would use alanine aminotranslerase. The mechanism of -:-ese reactions involves the coenzyme pyridoxal phosphate (plp) which is

rr example, if you wanted to transfer the amino group from

alanine to a-

rerived from vitamin B5 (pyridoxine).

lcoo ,lrl
tvt

T--;r
I I

(ur H
NH3-

C-

c-o

ilo

Ic=o
CH, -

PI,P

H
c-Amino Acid Figure 8-67

RrAminotransferase

C- C- O"
a-Keto Acid

oo ll ll

o
ri

\7

9oo NH?- C- H 'l


CH"

l'

coo"

9n' .'

CH.r -a
|

COO,J

cr-Ketoglutarate

Glutamate

-'orm of NH+.

-he glutamate that isproduced in Figure g-67 gets oxidatively deaminated as 'hown in Figure 8-68: Glutamate dehydrog"ni"" catalyzes tiris reaction and =rther NAD@ or NADP@ can be utilized. Note that nitrogen is released in the

iopyright

by The Berkeley Review

223

The Berkeley Review Specializing in MCAT preparation

Biology

Metabolic Pathways
o
t Lrzv =-

Urea Cycle

@ too NHl"l

C- H CHT A NAD.
1or NADP+)

r ./t

H+l

coo

o
T

C=O
(or NADPH)

coo

a"" lO

L11, Glutamate | -

NADH +

NHa+

dehydrogenase

coo

?", a

Glutamate

cr-Ketoglutarate

Figure 8-68
The sum of the reactions shown in Figure 8-67 and in Figure 8-68 can be seen in Figure 8-59. In terrestrial vertebrates the excess NH+e produced in Figure 8-68 is converted into urea and then excreted. In most aquatic organisms, NH4@ itself is excreted, while in birds and terrestrial reptiles the NH4+ is converted to uric acid before it is excreted.
+ H2O

g-Amino Acid +

NAD+=-

(or NADP+)

ct-Keto Acid + NADH + H+ + NH4+ (or NADPH)

Figure 8-69

Phase II
Let's consider the degradation of the carbon skeleton of the amino acids. that carbohydrate metabolism will eventually yield pyruvate which can enter Krebs cycle through acetyl-CoA. we also mentioned that fatty acid degradatim generates acetyl-CoA as well. When amino acids are degraded they will metabolic intermediates which can funneled into the Krebs cycle.

Carbohydrate Metabolism

Fatty f(rhG)

@,""1$Acetoacetyr
@t", ,ti)ru*."t"

@*1"#{
Knehs citrate
224

tm

[wJ

\-_lspJ\uyanel.

@'*'r'-.****@
Figure B-7O
Copyright @ by The Berkeley Review

The Berkeley Specializing in MCAT

Biology

Metabolic Pathways

Urea Cycle

Krebs cycle).

glucogenic and they will be degraded io pyruvate, o-ketoglutarate, succinylCoA, fumarate, and oxaloacetate. rnt is shtwn in Figure "g-20. oth", amino acids will be degraded to acetyl-coA and acetoacetyl-c11. These amino acids are termed ketogenic because they will eventually give rise to ketone bodies. Remember, mammals do not have a pathway *rat"witt allow acetyl-CoA (or -co2 acetoacetyl-CoA) to be converted to carbohydrate (due to the loss or in the

Some of these amino acids will provide the carbon skeleton framework for the net synthesis of carbohydrate via gluconeogenesis. These amino acids are termed

free ammonia that we produced in the glutamate dehldrogenase reaction combines with Co2 to form carbamoyl phosphate. This is r'r,o*.r" in Figure g-7L. Thiq reaction is_ catalyzed by carbamoyl phosphate synthetase, requirei 2 ATps, and occurs in the mitochondrial matrix. since carbamoyl phosphatels now a high energy compound, we can think of it as an activated carbamoyl donor.

{s we have seen, amino groups will flow from the various amino acids to glutamate and then to ammonii by the glutamate dehydrogenase reaction. The

oo
CO2 + NH4+ +

Z ATp+ H2CE::+ urN- 3-o -

oo
Figure 8-7 I

i-o" t'

+ 2 ADp + p; + 3 H+

Carbamoyl Phosphate

the carbamoyl group from carbamoyl phospr,ut" to ornithine to produce citrulline as shown in Figure g-72. citrulline is another example of an amino acid that does not occur in proteins. Citrulline can leave the mitochondrial matrix and enter into the cytoplasm.

iransfers

ln the next reaction, carbamoyl phosphate reacts with a molecule of ornithine (an amino acid that does no-t appear in proteins). Carbamoyl transcarbamoylase

NHq

CHr

f"'t H-C-NHr
looo
Ornithine Figure B-72

CHr +. t-

tt'

Ornithine

oo
Carbamoyl Phosphate

H-c-NHl

f"t.
l6-

coo"
Citrulline

-: we take citrulline and let it be aminated by aspartate via the enzyme argino.-:ccinate synthetase, we will form arginosuccinaie, which can be cleaved r! tire :-rzyme arginosuccindse to give arginine and fumarate. This reaction is a way to

',nthesize arginine from ornithine. How do you get ornithine? If you hydroiyze ::ginine with water, you will get ornithine and urea. Urea can then be excreied. lhe overall pathway for the urea cycle is shown in Figure g-73. [once fumarate is ::nerated it can react with water and be converted to malate. Malate reacts with
,rpyright
@

by The Berkeley Review

225

The Berkeley Review Specializing in MCAT preparation

Biology

Metabolic Pathways

Urea Cycle

NADo and is converted to oxaloacetate. oxaloacetate can react with glutamate to give aspartate and o-ketoglutarate.]

2ATp 2ADp+p1
NH+*

COF><:+
phosphate synthetase
H

\/oo

",*Carbamoyt

[- o- i- o
6

H3N-

+l

C-

COo

Ornithine ?"' cH" t'


rl 'NH.l
CH"

Ornithine
transcarbamovlas e

+l HrN"l C- COO
CH"

CHr

r'

Citrulline

t-

CH"

H2N- C- NH2

H.N- C- NH. u' o

t'

Urea
Arginase

o":;:,';:;':""11(
ll +
+l
H COO

ll .-

H3N-

*Y c- CoO
cooAspartate

i:l

H?N-l

+l

CCH"
I

COO

HrNC'l
Arginosuccinase

CHn

CH, t' CHr H-N_

t'

CHr tCH"

t-

C-

il-

NH?

NHz

coo
I

H-N- C- NH.
_l

t'

u'
t'

Arginine

CH

il
I

OOC- C- CH,_ COO


H

CH

coo
Fumarate
Figure 8-73
The Urea Cycle

Arginosuccinate

Copyright @ by The Berkeley Review

226

The Berkeley Review Specializing in MCAT preparatim

Metabolic Pathways
15 Passages

L00 Questions

Passage Titles

Questions

L II. ilI. IV. V. VL VII. V[I. IX. X. XI. XII. X[I. XIV. XV.

Glycolysis and Electron Transport Chain Electron Transport Chain Cholesterol Metabolism and Regulation Very Low-Density Lipoproteins (VLDLs) Calvin Cycle Lactose Intolerance B-Oxidation GlycogenMetabolism Glycolysisand2,3-Bisphosphoglycerate Leucine Catabolism Trehalose Experiment Fuel Oxidation during Exercise Urea Cycle Gluconeogenesis and the Cori Cycle Starch Blockers

12-tl
18-23

1-5 6-11

24-29 30-36 37-43 44-50 51-58 s9-65 66-12


13 -19

80-87 88-94 95 - 100

R.E.tV,,I .b.W@

Speciahzing in MCAT Preparation

Suggestions
The passages that follow are designed to get you to think in a conceptual manner about the processes of molecular biology at the organismal level. If you already have a solid foundation in molecular biology, many of the questions you read here will seem to be very straight forward and easy to answer. But if you are new to the subject or if you have not had a pleasant experience with molecular biology in the past, some of them might appear to come from the void that spreads out beyond the Oort field at the edges of our solar system. Pick a few passage topics at random. For these initial few passages, do not worry about the time. Just focus on what is expected of you. First, read the passage. Second, look at any diagrams, charts, or graphs in it. Third, read each question and the accompanying answers carefully. Fourth, answer the questions the best you can. Check the solutions and see how you did. \A/hether you got the answers right or wrong, it is important to read the explanations and see if you understand (and agree with) what is being explained. Keep a record of your results.

After you feel comfortable with the format of those initial few passages, pick another block of try to do them in one sitting. Be aware that time is going to become important. On average, you have about 1 minute and 15 seconds to complete a question. Be creative in how you approach this next group. If you feel comfortable with the outline presented above, fine. If not, then try different approaches to a passage. For example, you might feel well versed enough to read the questions first and then try to answer some of them, without ever having read the passage. Maybe you can answer some of
Passages and

the questions by just looking at the diagrams, charts, or graphs that are presented in a particular passage. Remember, there are many effective leaming styles. You need to begin to develop a format that works best for you. Keep a record of your results.

The last block of passages might contain at least a few topics that are unfamiliar even to those who know a good deal about molecular biology. Find a place where the level of distraction is at a minimuru Get out your watch and time yourself on these passages, either individually or as a group. It is important to have a feel for time, and an awareness of how much is passing as you try to answer each question. Never let a question get you flustered. If you cannot figure out what the answer is from information given to you in the passage, or from your own knowledge base, dump it and move on to the next question. As you do this, make a note of that pesky question and come back to it when you have more " time. When you are finished, check your answers and make sure you understand the solutions. Be inquisitive. If you do not know the answer to something, look it up. The solution tends to stay with you longer that way. (For example, what is the Oort field, anyway?)

The estifrated score conversions for 100 questions are shown below. At best, these are rough approximations and should be used only to give one a feel for which ballpark they are sitting in.

Section VIII Estimated Score Conversions


Scaled Score

Raw Score

>13

lt-12
9- 10 7 -8 5-6
<4

- 100 70-79 60-69 50-59 40-49


80

0-39

Biotogy
Passage

Glycolysis and Dlectron Ttansport Chain


1.

Passage I

I (Questions l-5)

enters the citric acid cycle, where it is completely oxidized to COz and HzO. During glycolysis anA tnl citric acid cycle, high-energy phosphate bonds are produced in the form of ATp and GTp (collectively
known as nucleoside triphosphates or NTps) by a process called s ubstrate -level p ho spho rylation.
The NADH and FADHz generated from glycolysis and the citric acid cycle transfer their electrons to the electrontransport chain in the inner mitochondrial membrane. As

Glycolysis involves a series of reactions that converts glucose into pyruvate. Under aerobic conditions pyruvate

approximately what factor?

de.creased by a factor of 6, while th6ir glucose utilization increased. Under aerobic conditions, yeasts can synthesize 36 NTps per molecule of glucose oxidized. The rate of glucose consumption under anaerobic conditions would increasi by

Louis Pasteur discovered that by depriving a culture of yeast cells access to oxygen, their giowth rate

4.2 8.3 c.4 D.6


2. If oligomycin A. B.
C.

electrochemical gradient that is used to do work. efp is synthesized from ADP and pi as protons move through an FoFlATPase located in the inner membrane. This process is called oxidative phospho rylation.
Outer Membrane Cytosol

the electrons flow down a series of electron carriers to molecular oxygen, protons are pumped into the intermembrane space from the matrix, eitablishing an

cells, the culture will:

is added to a growing culture of yeast

continue with aerobic respiration, synthesize 36 net NTps, produce COz and HzO, and
increase their growth rate.

i?f,?tgtE4.:?i:?r?[gr,?f??3rf f ?r?f ?r,?f ?r?r?r?r,fi ??rE? 88888888Jd88J888888S888899e8 Jg888g8sig J8J8 J9868


Intermembrane
Space

switch to anaerobic fermentation, synthesize 4 net NTPs, produce COz and CH:CHzOH, and
increase their growth rate.

net NTPs, and produce COz and CH:CHzOH, and decrease their growth rate.
D.

switch to anaerobic fermentation, synthesize 2

net NTPs, produce COz and HzO,


decrease their growth rate.

continue with aerobic respiration, synthesize 2


and

NADH NAD+ FADH2 FAD

+H+
Matrix

ADP H+ ATF
+Pi

3.

The FoFrATPase can be inhibited by oligomycin,


ro allow binding to every one of the Fq subunits, ATp cannot be synthesized by oxidative phosphorylation. Under these circumstances, electron transpoit stops, the electrochemical gradient increases, and the energy associated with the electron-transport chain becomes rnsufficient to pump any more protons from the matrix
rvhich binds specifically to rhe Fo portion of the complex. When oligomycin is pfesent in a concentration sufficient

_oligomycin and 2,4-DNp are simultaneously of growing yeast cells. In this case:
added to an aerobic culture

Both

A.

oxidative phosphorylation is stimulated, a will not be formed, electron transport will increase, CHgCHzOH and COz
proton gradient
are generated, and 36 net NTps are made.

B.

oxidative phosphorylation is stimulated, a proton gradient will be formed, electron transport will increase, HzO and CO2 are
generated, and 2 net NTps are made.

into the intermembrane space.

C.

rhosphorylation by transporting hydrogen ions from the intermembrane space to the matrix of the mitochondrion, lhus bypassing the F6F1-ATPase complex.

DNP) uncouples electron transport from oxidative

production of NTPs as well. For example, arsenic acid H:AsO+) has a chemistry similar to that of phosphorus acid (H:PO+). The comlound 2,4-dinitrophenol (2,4_

Other chemical agents can interfere with the

oxidative phosphorylation is inhibited, a proton gradient will not be formed, electron transport will increase, HzO and CO2 are
generated, and 4 net NTps are made.

D.

oxidative phosphorylation

proton gradient will not be formed, electron transport will increase, HzO and COz are
generated, and 2 net NTps are made.

is inhibited, a

Copyright @ by The Berkeley Review

229

The Berkeley Review Specializing in MCAT preparation

Biology
4.

Glycolysis and Dlectron Transport Chain

Passage I

in place of inorganic phosphate, the enzymatic reaction proceeds quite well as shown below. This linkage between the C-1 carbon and arsenate is unstable and decomposes to 3-phosphoglycerate and inorganic arsenate.

Step 6 in glycolysis is catalyzed by glyceraldehyde 3-phosphate dehydrogenase. If arsenate is supplied

o.- -H C
I

H- C*
I

OH

Enzyme H- cPOr2

oH
O

HrC-

O-

H2C-

PO12-

Glyceraldehyde
3-phosphate

1-Arseno-3phosphoglycerate

Addition of arsenate to a culture of cells growing


anaerobically on glucose:

A. B. C. D.

inhibits glycolysis at Step 6 and prevents further growth of the cells. does not inhibit glycolysis but prevents further
growth of the cells, because ADP cannot
phosphorylated in Step 7.
be

{fi

inhibits glycolysis at Step 7 but prevents further growth of the cells, because ATP
cannot be produced in a net yield.

I
I

does not inhibit glycolysis but induces the culture to switch from aerobic to anaerobic
metabolism. so they can continue to grow.

if

-t.

In terms of its effect on glycolysis, arsenate is BEST


described as:

I.

competitive inhibitor.

II. IIL
A.
B. C. D.

a noncompetitive inhibitor an uncoupling agent.

I only

II only I and III only II and IiI qnly

Copyright @ by The Berkeley Review

230

The Berkeley Specializing in MCAT Pre

Biology

Dlectron-Transport Chain

Passage II

t t
++++++
I
Matrix
NADH

Outer
Membrane o ot

o+
o

I
+H+
NAD+

Inner Membrane

ADP

Y g+
ATPase

+Pi
Figure I

Transport Protein

Passage

II

(Questions

6-ll)

Mitochondria are organelles that have two membranes (Figure 1). The outer membrane contains proteins called porins that permit the passage of molecules with molecular weights of less than 10,000. The inner membrane is selectively permeable to different molecules and is quite impermeable to polar and ionic substances. The matrix is enclosed by the inner membrane and
contains the enzymes necessary for the citric acid cycle,

The FAD-linked dehydrogenase at complex II interacts with coenzyme Q in very much the same
fashion.

Complex III contains a number of cytochromes, hemecontaining proteins involved in one-electron transfers, and

an iron-sulfur protein. This complex, also called the


cytochrome bc t complex, passes the electrons to a small peripheral membrane protein called cytochrome c (Cyt c).

fatty acid oxidation, and even amino acid oxidation.


Between the two membranes is the intermembrane space.

Cytochrome

c diffuses through the membrane and

Most of the electrons that enter the electron-transport :hain come from the action of dehydrogenase enzymes. llany of the dehydrogenases are NADo-specific. These enzymes remove two hydrogen atoms from their substrate; one in the form of a hydride ion (:He) and the
cther leaves as a proton (H@). Dehydrogenases that are FAD-specific can also remove two hydrogen atoms. The electron-transport chain operates within the inner

IV, known as cytochrome oxidase. These electrons are accepted by cytochromes a, a3, and two copper ions before they are passed to molecular oxygen. Electron flow through this complex results in protons being pumped into the intermembrane
space.

passes its electrons to complex

mitochondrial membrane and is composed of a series of electron carriers, many of which are integral membrane proteins that bear prosthetic groups capable of carrying
-rut

Protons from the intermembrane space can return to the matrix though the Fo transmembrane proton channel. Attached to the Fo component on the matrix side is an ATPase designated as Ft. As protons pass into the matrix through the FoFrATPase, ATP is synthesized from ADP
and Pi.

electron-transfer reactions.

The NADo-linked dehydrogenase at complex I not rnly transfers hydride ions to coenzyme Q, but it can also
pump protons into'the intermembrane space. As QHe liffuses through the membrane to complex III, it picks up r proton from the matrix and forms QHz. Electrons are lassed to complex III, and protons are pumped into the
.ntermembrane space.

Inhibitors like dicyclohexylcarbodiimide (DCCD) and oligomycin inhibit proton transport through F6. Amytal and rotenone inhibit electron transport from complex I to coenzyme Q. Finally, electron transport and oxidative phosphorylation can be uncoupled using hydrophobic
agents like 2,4-dinitrophenol (DNP), FCCP, and CCCP.

Jopyright O by The Berkeley Review

231

The Berkeley Review Specializing in MCAT Preparation

Biology
6.

Electron Transport Chain


10.

Passage II

Elecffons from pyruvate enter the electron-transport


chain at:

A. B. C. D.
7.

Complex I Complex II Complex III or IV none of the above

The flow of electrons from succinate to coenzyme Q occurs by way of the FADHz that is produced in the oxidation of succinate to fumarate.
FADH2

FAD Succinate

'-

Fumarate

The chemiosmotic hypothesis states that ATP is synthesized in the mitochondrial matrix because of
the:

A. B. C. D.

passage of electrons from NADH and FADH2 to molecular oxygen.

free energy decreases. This decrease can be quantified if we know the standard reduction potentials (E6') of redox pairs and by using tbe
equation

As electrons flow down the electron-transport chain,

electrochemical gradient that forms across the inner mitochondrial membrane.

increased permeability of ADP into the mitochondrial matrix at a specific protein


antiport.

AGo,_ _ (n)(F)(AEo,) where AGo'is the standard free-energy change, n is the number of electrons transferred, F is the Farada5r

increased permeability of ADP into the mitochondrial matrix at a specific protein


symport.

constant, and AEo' is the change in standarrd reduction potential between the oxidized and
reduced species. The following information is given:

8.

Synthetic phospholipid vesicles were constructed, and purified Fq protein was incorporated into the membrane. High levels of Ko were added to the vesicles. A short time later, valinomycin was added to the exterior medium. H@ translocation was not observed in vesicles missing the Fo protein. After valinomycin was added, however, He translocation occurred, because valinomycin was able to:

Fumarate + 2H@ + 2eo -----+

Succinate

0.03

FAD + 2H@ + 2eo

-+ FADH2

- 0.22

A. B. C. D.
9.

transport H@ from the exterior to the interior of the vesicle. transport Ho from the interior to the exterior of the vesicle.

We can conclude that in the reaction involving conversion of succinate to fumarate:

create a diffusion potential that allowed an efflux of Ke and an influx of Ho.


create a diffusion potential that allowed an influx of K@ and an efflux of H@.
11.

A. B. C. D.

AGo'AGo'-

AGo'_

AGo'_

- (0.50)(F). +(0.s0XF). +(0.10)(F). _ (0.IOXF).

Addition of DCCD to cells inhibits Ho transport


through the mitochondrial Fo protein. Addition of
2,4-DNP would be expected to:

In prokaryotic and eukaryotic cells, the e transport chain and oxidative phosphorylation coupled. Oxidative phosphorylation in pro
cells occurs in the:

A. B. C. D.

matrix, while electron transport occurs on


inner mitochondrial membrane. intermembrane space, uhile electron tr occurs on the inner mitochondrial cytosol, while electron transport occurs oo
outer membrane.

A.
B.

decrease electron transport and decrease ATP synthesis.

increase electron transport and increase ATP

C.

synthesis. r increase electron transport and decrease ATP


synthesis. decrease electron transport and increase ATP synthesis.

cytosol, while electron transport occurs on


plasma membrane.

D.

Copyright @ by The Berkeley Review

232

The Berkeley Specializing in MCAT

Biotogy
Passage

Cholesterol Metabolism and Kegulation

Passage III

III

(Questions 12-17)

Most of the cholesterol synthesized in the liver is exported in the form of a bile salt or a cholesteryl ester.

In

1912 Brown and Goldstein began

to examine

dominant human hereditary condition called familial hypercholesterolemia (FH). Their work on the transport of exogenous cholesterol in the bloodstream and its receptor-mediated endocytosis and metabolism by target tissues (Figure l) won them the Nobel Prize in 1985.
Cholesteryl
esters

Bile salts are stored in the gallbladder until they are needed in the small intestine. Cholesteryl esters are synthesized from cholesterol and a fatty acid from coenzyme A through the action of acyl-CoA-cholesterol acyl transferase (ACAT). High intracellular levels of cholesterol stimulate ACAT, promoting esterification of cholesterol for storage.
Unlike bile salts, cholesterol and cholesteryl esters are rather insoluble in water. These lipids are sequestered in a

lesterol

LDL
Lipoprotein

LD L

receptors

,/l

/l

Extracellular
space

\-

\ W.-ffi
&*t*d qs4ffi1 {-st\ A
{-1>"
\\
Endocytotic

\ ffi\

rffid5p
\ vesicle \

'lj,"'3jfiil''

micelle-like particle called a low-density lipoprotein (LDL) and transported from the liver in the bloodstream to various target tissues. LDL receptors on the surface of a target cell cluster into coated pits, and binding of LDL to these receptors initiates endocytosis. The internalized coated pit becomes an endocytotic vesicle. Several of these vesicles fuse to form an endosome, which in turn fuses with a lysosome. Enzymes within the lysosome hydrolyze the cholesteryl esters and the protein portion of the LDL particle, releasing cholesterol, fatty acids, and amino acids into the cytoplasm. The LDL receptors are
recycled and move back to the plasma membrane to form another coated pit. High levels of intracellular cholesterol

inhibit LDL receptor synthesis.

tffi \ [::"::i:>w
/ -!-R-Zacids ,'-*
CO-

#
I

12.

Based on information in the passage, what is meant by "exogenous cholesterol"?

Endosome I a Amino ,,^ I =-WA /

A. B. C. D.

Cholesterol Cholesterol Cholesterol Cholesterol

in the blood that may enter the cell inside the cell sequestered inside arterial plaques in the nucleus of the cell

13.

Brown and Goldstein studied people with FH. The primary defect of FH is ineffective or missing LDL receptors. How would FH affect the metabolism of
cholesterol?

Figure 1. LDL receptor-mediated endocytosis of


chlosterol and its metabolism.

stages, primarily in the cytosol of ra-te-limiting step in this pathway The nepatocytes. rnvolves HMG-CoA reductase, an enzyme that can be :ilosterically inhibited by high levels of intracellular :holesterol, unidentified cholesterol derivatives, and hormonally regulated by glucagon (inactivation) and Jccurs

Cholesterol biosynthesis begins with acetyl-CoA and

in four

I. il. III. A. B. C. D.

Increased intracellularcholesterol synthesis Decreased activity of ACAT Increased LDL cholesterol in the blood

I only

II only I and III only I, II, and III only

insulin (activation). Copyright @ by The Berkeley Review

235

The BerkeleY Review Specializing in MCAT PreParation

Biology
14. In Canada,

Cholesterol Metabolism and Regulation

Passage III

public health guidelines are given in SI units. For cholesterol (MW - 386.64), the public health guideline is plasma total cholesterol levels below 5 mM. What is this in mg/dl, the units for
this guideline in the U.S.?

B. C. D.
15.

A.77.2mgldL
193.3 mg/dl 1933.0 mg/dl

772,0 mg/dL

All of the following A. B. C. D.

events happen after the

lysosome fuses with the endosome EXCEPT:

lipases degrade cholesteryl ester


cholesterol.

to

free

proteases degrade apolipoproteins


acids. LDL receptor synthesis increases.

to amino

cholesteryl ester
cytoplasm.

is stored in droplets in

16.

What would be the physical results of changing to a no-cholesterol diet?

A. B. C. D.

Intracellularcholesterol synthesisincreases Intracellularcholesterolsynthesisdecreases Extracellularcholesterolsynthesisincreases Extracellular cholesterol synthesis decreases

17. Mevinolin

is the active ingredient in a class of cholesterol-lowering drugs. It works as a potent competitive inhibitor of HMG-CoA reductase and lowers plasma cholesterol levels. In the following Lineweaver-Burk plot, which line BEST indicates mevinolin's action on HMG-CoA reductase?
Without mevinolin

A.
B. C. D.

Line A

Line B
Line C Line D

Copyright @ by The Berkeley Review

234

The Berkeley Revier Specializing in MCAT Preparatic

Biology
Passage

Very Low-Density Lipoproteins (VLDLs)


18.

Passage IV

IV (Questions 18-23)

Phospholipids, triacylglycerols, cholesterol, and


cholesteryl esters are hydrophobic molecules that are essentially insoluble in water. In order to be transported in the bloodstream, these lipids combine with carrier molecules called apolipoproteins to form one of four major classes of plasma lipoprotein particles. Different combinations of lipid and apolipoproteins produce plasma
lipoproteins ofvarying densities and sizes (Table 1).

From the plasma samples she draws, the researcher conducting the three experiments described in the
passage must separate the
this?

VLDL from the other

lipoproteins. What is the BEST way to accomplish

A. B. C. D.

Extraction with organic solvent Centrifugation in a density gradient Thin-layerchromatography Extraction with 5Vo NaCl solution

Particle
High-density lipoprotein (HDL) Low-density lipoprotein (LDL)
Very low-density

Density (g/ml)
1.13 1.04

Diameter (nm)

l0
20
50

19.

Based on the information in Table l, which are rhe largest and MOST buoyant lipoproteins?

lipoprotein (VLDL) Chylomicron

0.98 0.95

500

Table 1. Major classes of plasma lipoproteins.

A. B. C. D.

High-densitylipoproteins
Chylomicrons

Low-densitylipoproteins Very low-density lipoproteins

De novo lipogenesis (DNL) is the synthesis of fatty


acids from acetyl-CoA. DNL occurs in the cytosol of the

of the enzyme complex iatty-acid synthase. After DNL, fatty acids are esterified to glycerol. The product, triglyeride, is packaged into VLDLs and excreted from the liver.
hepatocyte with the assistance

20.

Which lipoprotein contains dietary lipids?

A researcher uses intravenous l-13C acetate to label rhe triglyceride that is synthesized de novo during the iollowing experiments. She then analyzes the .ncorporation of l-l3C acetate using gas chromatographymass spectroscopy (GC/MS).
Etperiment

A. B. C. D.

High-densitylipoproteins Low-densitylipoproteins
Chylomicrons Very low-density lipoproteins

Subjects fast from 3:00 pm until noon the following day. Blood samples are taken every hour from 6:00 am until noon.
Experiment

2I.

Why does the researcher use 1-l3C acetate to label the VLDL triglycerides?

II

Subjects fast from 8:00 pm until 6:00 am. A glucose drink containing 25 grams of glucose is given once per hour from 6:00 am until noon, during which time hourly blood samples are taken -.rperiment
I

I. II. IV.

The acetate is metabolized to acetyl-CoA. 8 acetyl-CoAs are used to make I palmitate


molecule.

UI. l-l3C acetate contains

a stable isotope that is

safe for use in humans at high doses.

ll

l-13C acetate contains a radioactive isotope that is safe for use in humans at high doses.

'.

Subjects fast from 8:00 pm until 6:00 am.

A fructose

drink containing 25 grams of fructose is given once per hour from 6:00 am until noon, during which time
hourly blood samples are taken.

A. B. C. D.

II and III only I and IV only I, II, and IV only I, II, and III only

ilopyright

by The Berkeley Review

235

The Berkeley Review Specializing in MCAT Preparation

Biology
22.

Very Low-Density Lipoproteins (VLDLs)

Passage IV

Analysis on the GC/MS requires transesterification of the triglycerides to form fatty acid methyl esters. Which is the BEST reagent to use for this purpose?

A. B. C. D.

Methanol + HCI Methanol Ethanol + HCI


Ethanol

23.

Subjects participating in which experiment would show the LEAST incorporation of l-l3C into VLDL triglycerides?

A. B. C. D.

Experiment I Experiment II Experiment III Subjects in all three groups would show the same degree of l-13C incorporation.

Copyright @ by The Berkeley Review

236

The Berkeley Revier Specializing in MCAT Preparatia

Biology
Passage V (Questions 24-29)

Calvin Qycte

Passage V

Within the chloroplasts of green plants, COz can be fixed in the form of simple organic compounds. The synthesis of these compounds takes place in a cyclic pathway called the Calvin cycte (Figure l), in which specific metabolites (Table l) are continually regenerated.

ATP and NADPH are generated as products of the light reactions of photosynthesis. These iwo metabolites
photosynthesis. These dark reactions make up the Calvin cycle. All of the reactions of the Calvin cycle, except those catalyzed by enzymes Er, Eto, and btr, can be found in animal tissue. Many of the reactions are common to both glycolysis and the pentose phosphate pathway.

during the light-independent or dark reactions of

are then used to convert COz and HzO into carbohydrates

carbon dioxide ( l4CO2).

Key intermediates in the Calvin cycle, like 3-pG and Ru-1,5-BP (Figure 2), were identified as rhe resulr of experiments using green algae and radioactively labeled Hrc-

o-

po.}

^o u-t^-0
L
I
I

c-- o

H_C-OH
HrCRu-1,5-BP
O

POr2

H- C- OH H* C- OH HrC- OI I

POr2'

-CO" Etl{ + n,o I


3-PG

3-PG

Ru-1,5-BP

Figure 2. Calvin cycle intermediates.

Ni

{P lr tE"
anp
NADPH

Experiment
Step A:

A mixture of green algae is exposed to COz and light for


an extended period of time.

DHAP

1,3-BPG

\ p+\\
\\
G-3-P

Step B:

The light source is removed, and 14COz is added to the


mixture.
Step C:

NADP@

+Ho IrluT. l.The Calvin cycle. These CO2 in


the lorm of carbohydrates. reactions fix atmospheric

Metabolites are analyzed for l4C labeling. Experiment


Step A:

II

Table l. Abbreviation
Ru-1,5-BP
3-PG 1,3-BPG

A mixture of green algae is exposed to tacoz and light


CompoundName
Ribulose- 1,5-bisphosphate 3-Phosphoglycerate

for an extended period of time.


Step B:

The light source is removed, and COz is added to the


mixture.
Step C:

G.3-P

DHAP
F-1,6-8P F-6.P G.6-P Xu-5-P E-4-P
s- 1,7-BP S.7-P

3-Bisphosphoglycerate Glyceraldehyde-3-phosphare Dihydroxyacetone phosphate


1,

Metabolites are analyzed for l4C labeling. Experiment


Step A:

Fructose- 1,6-bisphosphate Fructose-6-phosphate Glucose-6-phosphate

III

Xylulose-5-phosphate Erythrose-4-phosphate
. Sedoheptulose1,7-bi sphosphate

A mixture of green algae is exposed to l4CO2 in darkness for an extended period of time.
Step B:

Sedoheptulose-7-phosphate Ribose-5-phosphate Ribulose-5-phosphare

After all forms of COz


Step C:

Ru-5-P

R.5-P

are removed from contact with the mixture, a light source is introduced.

Metabolites are analyzed for 14C labeling. Jopyright @ by The Berkeley Review

237

The Berkeley Review Specializing in MCAT preparation

Biology
24.
reactions correspond to:

Calvin Cycle
28.

Passage V

The first five steps in the Calvin cycle are catalyzed by enzymes Et,E2, E:, E+, and Es (Figure l). These

Based on the information in Experiment I, which of the following graphs BEST represents the levels of l4C in Ru-1,5-Bp and 3-pG?

A.

dephosphorylation, enolization,
condensation.

carboxylation, phosphorylation, reductive


and

A.
'+

B.

U
Ru-1,5-BP

O .+

B. hydrolysis, phosphorylation,
condensation. C.

reductive o
(6

dephosphorylation, isomerization, and

dephosphorylation, enolizationn
condensation. D.

carboxylation, phosphorylation, oxidative


and

Ru-1,5-BP

&
Time

&
Time D.

hydrolysis, phosphorylation, oxidative dephosphorylation, isomerization, and


condensation.

C. tU
>r

O +
>r

25.

Based on the metabolic pathway in Figure l, what is the overall balanced reaction for the Calvin cycle?

A. B. C. D.
26.

o o
(!

COz + 2ATP + 4HzO + NADPH + H+ -+ Glucose + 4Pi + 2ADP + NADP|

&
Time

Ru-1,5-BP

6COz+ 18ATP + I2HzO + 12NADPH + 12H+ -+ Glucose + l8Pi + 18ADP + l2NADp+ 6COz+ I2ATP + 6Hz0 + I2NADPH + l2H+ -+ Glucose + l2Pi + 12ADP + I2NADP+
COz + ATP + HzO + NADPH + H+ -+

Time

29.

Glucose+Pi+ADP+NADp+ After 3 turns of the Calvin cycle, 6 molecules of glyceraldehyde-3-phosphate are formed. These 6 "molecules will be converted into:

Based on the information in Experiment III, of the following graphs BEST represenrs the of raC in Ru- 1.5-BP and 3-PG?

A.
U .+

B.

O
Ru-1,5-BP

A.

3 molecules of Ru-1,5-BP
c-3-P.
glucose.

and

I molecule of I molecule of
O (!

>' o
Ru-1,5-BP

B.
.D.
-

3 molecules of Ru-1,5-BP and

C. I

molecule of Xu-S-P, I molecule of E-4-P, and I molecule of S-1,7-BP. I molecule of F-1,6-BP and 2 molecules of

&

&

Time

glucose.

c.
U s
>a

D.
$

27. If radioactively

labeled carbon dioxide (l4COz) is used as a substrate for the Calvin cycle, it will appear in the carboxyl group of 3-phosphoglycerate. A short time later, the label will be found in glucose-

o c! !

()
(g

6-phosphate at carbon atoms:

A. C-l and C-d. B. C-l and C-3. C. C-2 and C-5. D. C-3 and C-4.
Copyright @ by The Berkeley Review

&
Time

&

234

The Berkeley Specializing in MCAT

Biology
Passage

Lactose Intolerance 3f.

Passage VI

VI (Questions 30-36)

Human beings are the only animals that are known to

Which biochemical reaction during fermenration changes milk so that yogurt and cheese made from it can be consumed without complication by some
lactose- intolerant individuals
?

drink the milk of other species and to continue to drink milk after weaning. The majority of the adult population of the world cannot digest lactose, the sugar in milk.
Almost all human infants have sufficient lactase activity to break lactose down into glucose and galactose. But with age, the activity of lactase generally declines, except

A. B. C. D.

The lactic acid produced during fermentation


breaks down lactose.

The alcohol produced during fermentation


breaks down lactose. Microorganisms produce disaccharides during fermentation. Microorganisms break down disaccharides during fermentation.

in some people of northern European origin and certain


groups of Africans, such as the Tutsi and the Felani.

The symptoms of lactose intolerance are unpleasant but not often fatal, unless milk is the only available food. After the ingestion of 30-50 grams of lactose as a test dose, the lactose-intolerant person usually experiences diarrhea, abdominal cramps, and intestinal gas. Even if

fluid milk is not tolerated, some lactose-intolerant people


may still be able to consume fermented milk products,
such as yogurt or cheese, without complications.

32. What is the cause of the intestinal


associated with the ingestion lactose-intolerant person?

symptoms
a

of fluid milk by

30. Which of the following


glucose)?

diagrams represents the

chemical structure of lactose (galactose-p(1-+4)A.


cH2oH cH20H

I. il. UI.
A.
B.
C.

Bacteria in the colon ferment the lactose to


produce gases.

Colon contents are hypertonic to surrounding


cells, and water enters the colon by osmosis. Bacteria in the colon produce irritating acids from the lactose.

I only I and II only

D. B.

II and III only I, II, and III

33. C.

Lactase isolated from the mold Aspergillus niger is available commercially for lactose-intolerant people to add to milk products before ingestion. How does this commercial lactase work?

A.

It is carried to the small intestine, where it functions exactly like the missing human
lactase.

B.

D.

It breaks down the lactose in the food and is inactivated in the stomach.

c.
D.

It

induces the production

of lactase by

the

intestinal mucosa cells.

Mold is cultivated in the milk, and it digests


the lactose.

Jopyright @ by The Berkeley Review

269

The Berkeley Review Specializing in MCAT Preparation

Biology

Lactose Intolerance

Passage VI

34. After the ingestion by a test subject of a dose of


lactose to check for lactase deficiency, which of these physiological variables should the clinician
measure?

il.

Breath hydrogen

III.
A.
B. C. D.

Blood glucose Blood lactose

I only I and II only

II and III only


I. II. and

III

35.

Where in the human body is lactose synthesized?

A. B. C. D.

The thymus The small intestine The mammary gland


The adrenal gland

36.

Where

is the enzyme

lactase secreted during

digestion, in lactose-tolerant individuals?

A. B. C. D.

The mouth The stomach

The small intestine The colon

Copyright @ by The Berkeley Review

240

The Berkeley Specializing in MCAT

Biology
Passage

B-Oxidation

Passage VII

VII

(Questions 37 - 43)
o

il

Catalyzed present the

enzymes

by I
I

R-CH,-

C- oH + ATP + CoA-SH
ll
ll

A. Working on a theory proposed by an American biochemist, Albert Lehninger, that fatty acids are

During the early 1950s another German chemist, Feodor Lynen, discovered that one of the end products of B-oxidation was not acetate, but rather acetyl coenzyme
somehow activated before they are degraded, Lynen and his colleagues discovered that coenzyme A (CoA-SH) is esterified to the carboxyl group of a fatty acid in an ATPdependent enzymatic reaction.

outer

in

J
I
I

Cytosolic Fauv Acid

mitochondrial

membrane

t R-CH)-C-S-CoA + AMP + PP1


Fatty acyl-CoA Inner Membrane

oll il

Activation of long-chain fatty acids occurs in the cytosol of a cell, and they are transported across the
mitochondrial inner membrane and into the mitochondrial matrix by a shuttle system involving L-carnitine. Short-

Camitine
transports the fatty acid

u..oi, th"

inner membrane\ V7777VA Fatty acyl-Camitine and into the MATRI' matrix of the

e;::" Wffi

chain fatty acids can cross the inner mitochondrial


membrane as free fatty acids. Once they are in the matrix, they are activated. These fatty acids can be obtained in their free form from the blood or released from cellular triacylglycerols by the enzymatic action of lipase.

mitochondrion

.^:z:Y
FAD
7

R-CH2-CH2-CH2-C

- S - CoA Fatty acyl-CoA \

Oxidation of a fatty acid occurs exclusively in the mitochondrial matrix. This is in contrast to fatty-acid
synthesis, which occurs exclusively in the cytosol

of

\ \\__, \

FADH2

R-CH2-C=C-C-S-CoA
H

Ho lll

cell. In other words, fatty-acid catabolism and fatty-acid anabolism occur in different compartments of the cell.
Even though fatty-acid synthesis utilizes acetyl-CoA as a precursor molecule, the reaction mechanisms are more complex and are not a direct reversal of B-oxidation.

)<

co-

ttll R-cH2- c- c-c - t- .it . NAD@ H H \*oo" \


R-CHr-

HOHO

37. Once a fatty acyl-CoA molecule completes the four

ffrff C- C-C- SH

sequential reactions in B-oxidation, it produces an acetyl-CoA molecule and a fatty acyl-CoA molecule which has been shortened by two carbon atoms. The reactions sequences of p-oxidation is BEST described in terms of:

CoA

A. oxidation; hydrolysis; oxidation; thiolysis.


B. C.

CoA-SH

oxidation; reduction; dehydrogenation; hydrolysis.


dehydrogenation; hydration; oxidation; esterification. oxidation; hydration; reduction; cleavage.

\/ / il ff R-CHr-C-S-CoA + CHr-C-S-CoA
Fatty acyl-CoA (2 carbons shorter) AcetYl-CoA

D.

38. Consider caprylic acid, a C8 saturated fatty acid


found in small amounts in butter:
o

1904), the German chemist Franz Knoop began to feed animals fatty acids with a phenyl group (C6H5) attached to the last ;arbon atom in the fatty acid chain, the omega (to) carbon. .\nalysis of the urine of these animals led him to conclude that fatty acids are degraded in two-carbon fragments by a process that involves ckidation at the p-carbon to give a 3-keto acid. This is followed by cleavage of the p-keto acid to give two products: acetate and a fatty acid derivative (which is two carbon atoms shorter than the p-

At the turn of the 20th century (circa

CH:-CH:-CHu-CH1CH2-CH2-CH2

! -on

Caprylic acid

Complete oxidation of one molecule of this shortchained fatty acid to CO2 and HzO gives a net yield
of:

lieto acid). The pathway for fatty-acid degradation

became known as B-oxidation.

A. B. C. D.
241

63 ATPs.

62ATPs.
61 ATPs. 60 ATPs.

Copyright @ by The Berkeley Review

The BerkeleY Review Specializing in MCAT Preparation

Biology

B-Oxidation

Passage Vtr

39. If we were to label caprylic acid with a phenyl ring at the C-8 position, then which of the following
chemical compounds would be present in the highest concentration in the urine of a rabbit fed with this modified fatty acid?

42. Morc energy is available from the oxidation of fatn acids than from the oxidation of carbohydratei.
because:

I. il.

fatty acids are stored in an anhydrous form.


whereas carbohydrates are not.

A. ll o c-o
o

B.

IIII. f atty acids are more reduced thal

the hydroxyl hydrogens in a carbohydrate are released as bare protons with no electrons.
carbohydrates.

I \n'J-." \_./
D. o

A.
B.

I only

c.
C. D.

II and III only I and III only I, II, and III

ll o c-o

il

/r\ \J
cHr-cH2-cH2-cH2-CH2-CH2-CH2

_)

43. One of the three major biological roles for fatty

acrJs

fats) and used as food molecules. The genera

is that they can be stored as triacylglycerols (neura.

structure for a triacylglycerol is shown below. The R. group represents the hydrocarbon chain of the samr fatty acid:

ll

il

ri0

40. Suppose we were to label the C- l, C-3, C-4, C-6 and C-7 carbon atoms of caprylic acid radioactively with l4C, as shown below (where an * 14C): =

**

Io Irr H-C_ O_ CH,C- O- C-

H2C- O- C-

fi

lm

,il!

fl fi
R

ff

Caprylic

acid

C*

OH

lu Itr

Triacylglycerol

If this molecule were completely metabolized, then in which of the following molecules would the label be
located predominately?

The melting points of four common Cra fatty


are:

A.
o=c=o
*
C.
o
CHr -

B.
o
CH, -

Stearic acid (+69.6 oC) Oleic acid (+13.4 oC)

il

Linoleic acid (-5 oC) Linolenic acid (- I I oC)


S-CoA

C-

A decrease in the melting temperature (Tp) of Crs fatty acids would indicate;

D. il
o
S-CoA

C-

cHr t<

il

C-

S-CoA

I. II. III. A. B. C. D.

an increase in fluidity. a decrease in unsaturation.

a decrease in the number of van der


interactions.

\l

4L. A rabbit completely oxidizes I mole of caprylic acid to COz and H2O. The net yield of water produced is:

I only

I and II only

A. B. C. D.
Copyright

greater greater greater greater

than than than than

0, but less than 20 moles. 20, but less than 40 moles. 40, but less than 60 moles. 60, but less than 80 moles.

II and III only I and III only

by The Berkeley Review

242

The Berkeley Specializing in MCAT

Biology
Passage

Glycogen Metabolism
44-50)

Passage VIII

VIII (Questions

45. Based on Figure I, which of the following


statements is FALSE?

Human beings store excess energy in glycogen or in triglyceride. Glycogen in muscle and liver accounts for about 1200 kcal, while energy stores in triglyceride are virtually limitless. Glycogen is stored in the liver at 44
muscle tissue. The human liver weighs about 1.8 kg. This

A.
B.
C.

Carbohydrate oxidation increases with


carbohydrate overfeeding.

Carbohydrate is used to

fill

glycogen stores

g/kg liver tissue and in the skeletal muscle at l4glkg


D.

organ metabolizes excess dietary carbohydrate into triglyceride and releases it in the form of very low-density lipoproteins (VLDL). Swiss researchers studied the disposal of excess dietary glucose following a glycogen-depleting fast.
Three male subiects were fed a l07o carbohydrate, iat, and l57o protein weight-loss diet for 3 days. Their glycogen stores were also depleted by exercise.
75%o

beforc de novo lipogenesis begins. Glycogen stores cannot be saturated. Net de novolipogenesis (DNL) is used in part to dispose of excess carbohydrate consumed during Days 5-10.

46.

What changes in VLDL should be seen in subjects given a massive overfeeding of carbohydrate for one
week?

Sometime late on Day 2 of this dietary experiment, the

sub.jects entered a respiratory chamber, a room that permits the researchers to sample all the air exhaled by rhe test subjects. The subjects stayed in the chamber for ren full days. Beginning on Day 4, they were switched to : diet consisting of 85Vo carbohydrate,3To fat, and ll7o protein for one week. This diet increased in calories from in excess of 1000 kcal to an excess of 2000 kcal. The :ollowing diagram (Figure 1) shows daily carbohydrate .ntake and its disposal during the 7 days of progressive
;

A. B. C. D.
47.

Increased VLDL production Decreased VLDL production No change in VLDL production

Complete inhibition of VLDL production

The respiratory quotient is defined as the ratio of COz to Oz in expired air. If glucose (CoHtzOe) were the sole fuel for these test subjects, and if it were oxidized completely to COz and HzO, what would
the respiratory quotient be?

arbohydrate overfeeding

A. B.

c. D.
800
O

0.70 0.82 0.95


1.00

48.
600

Shown below is the structure


T?

E
O

residues in glycogen. What are the linkages at

of linked glycosyl I and

E u

+oo

2oo

oI
I

67
Days

cH2oH

Figure

l{.

Which tissue in the human body has the largest net


supply of glycogen?

A. B. C. D.

Liver
Kidney Muscle Heart

A.
B.
C.

D.

is a-1,6, and II is s 1,4, and II is o-1,4, and II is a-1.6. and II

is p-1,4. is p-1,6.
is a,-1,4.

is cx-1,4.

- rpyright @ by The Berkeley Review

243

The Berkeley Review Specializing in MCAT Preparation

Biology

Glycogen Metabolism
the

Passage VItr

49. In an experiment with rats, a researcher labels

Which of the following tracers would NOT


effectively label the glycogen?

glycogen stored in the liver. Animals are first fasted and then refed with a labeled precursor to glycogen.

A. B. C. D.

1-l4C palmitate

l-l3C glucose l-l4C alanine l-l3C pyruvate

50. The experiment described in the passage is


extreme version

of carbohydrate loading. Athletes also carbo-load by eating low-carbohydiate foods


carbo_

an

loading possibly have?

right before an event. What benefits might

and exercising followed by a high carbohydrate feast

A.
B.

c.
D.

Increases glycogen stores to boost endurance Increases fat stores to boost endurance Decreases glycogen stores to boost endurance Decreases fat stores to decrease endurance

Copyright @ by The Berkeley Review

244

Biology
Passage

Glycolysis and

2, 3

-Bisphosphoglycerate

Passage IX

IX (Questions 51-58)

Glycolysis involves ten enzymatic steps, during which slucose is converted to pyruvate. In animals, this is the cnly metabolic pathway that produces ATP in the absence of oxygen.
^ Ll ,l H \at
C

Under aerobic conditions in animal cells, pyruvate diffuses from the cytosol to the mitochondrial matrix, where it is converted to acetyl-CoA before entering the citric acid cycle. Under anaerobic conditions, pyruvate is converted into lactate in animal cells, while in yeasts it is
converted into ethanol and carbon dioxide.

.O

c2 H- C- OH rl cr Ho- c- H .t c4 H- c- oH 'l c5 H- C- OH 'l c6 cH2-oH


Glucose

ADP

H-C=O H- C- OH HO- C- H H- C- OH
t I I I

Nine of the intermediates in glycolysis are phosphorylated to prevent them from leaving the cytosol. This ensures that during specific reactions, phosphate groups are donated to ADP to make ATP.

H- CI

OH

cHr-o-Por2'

Glucose 6-Phosphate

st"e,
cHr-o-Porl

ll
CH,-OH

c=o ADp i- ri t ATP "ol- o" "Step 5 H-C-OH


I

H-c=o
I

t-

51.

In prokaryotic cells, glycolysis occurs in rhe:

":::-:"
H- CI

OH

A. B. C. D.

mitochondrialmatrix.
cytoplasm. lumen of the Golgi complex. lumen of the smooth endoplasmic reticulum

cHr-o-Po.2-

cHr-o-Por2-

Fructose 1,6-Bisphosphate

Fructose 6-Phosphate

"r.o
c=o
I

ll

Pr*

NADH

g NAD+ r o. ttc'\)c
r \t '-

'

+H*

52, Lactic
o.. ,o H- COH

acid is the end producr of glycolysis in:

po,2'

H- C- OH step 6
cHr-o-Por2-

cH?-oH

cHr-o-Por2-

A. B. C. D.

red blood cells. brain tissue cells. adipose tissue cells. heart and muscle tissue cells.

Dihydroxyacetone Phosphate

Glyceraldehyde
3-Phosphate

1,3-Bisphospho-

glycerate
11 ,. step z l[

ADP

oat

.l H- L- r:r-t{-r-- -------------'-'

C 'o

ooo \\,
C
I

ll\
I

erp

53.

Glycolytic enzymes can catalyze all of the following reactions in glycolysis EXCEPT:

!",_o"

step a

H_ C-

OH

cH2-o-Po32'

2-Phosphoglycerate

3-Phosphoglycerate

A. B. C. D.

phosphorylations.
isomerizations.

enolizations. ligations.

step e

ll.,' n r\/
C
I

1\'",o
ATP

o.t ,o
C

il-

c-

o-Po12'

c=o
Step ro
I

54.

The net oxidation state ofcarbon in glucose is:

cHz

CH:

Phosphoenolpyruvate

Pyruvate

A. -1 8.0 C. +l D. +2
245
The Berkeley Keview Specializing in MCAT Preparation

Copyright @ by The Berkeley Review

Biology

Glycolysis and 2,S.Bisphosphoglycerate


intermediate 1,3-

Passage D(

55. In red blood cells, the glycolytic


a mutase enzyme.

bisphosphoglycerate (1,3-BpG) can be converted to 2,3-bisphosphoglycerare (2,3-BpG) by the action of

57,

Pyruvate-kinase deficiency

children will show a deficiency in pyruvate kinase?

one parent has thc disease and the other parent is heterozygous for the disease, what is the probability that one of their

autosomal recessive trait.

If

is inherited as a

rare

o. s/

- Po,l .+
Mutase

H-C-OH
I

o.\ ,o

- C- OPOra
cHr-o-po.2'

cH.-o-Por2-

1,3-Bisphospho-

glycerate

2,3-Bisphosphoglycerate

A. B. C. D.

jVo
25Vo 50Vo 757o

This bifunctional enzyme can also convert 2,3-BpG to 3-phosphoglycerate and Pi, metabolites used in during glycolysis if this 2,3-BpG shunt is operating
in red blood cells?

58.

glycolysis. How many net ATps are produced

enzymatic reactions that includes phosphorylati

Based on the glycolytic pathway, which of following structures is formed after a series
a

dephosphorylation and reduction,


isomerization?

A.0 8.2 c.4 D.6


56.

A.
oat
C
I

B.

'o
o-po.l

- ct"

tc=o
I

cH,-o-PO,r'

Pyruvate kinase is an important regulatory enzyme in glycolysis, and it catalyzes the conversion of pEp

cH2-oH

cH2-oH

to pyruvate. Which of the following oxygen-

C.

D.

hemoglobin dissociation curves (dashed line) BEST represents an individual with a mild deficiency of this enzyme?

o.a,H
C

o. \}/ oo
C
I

H- CI

OH

H_ CI

OH

cHr-o-POr2'

cHr-o-Po.2-

A.

B.
N (,

o
3

=
U)

rn

Poz D.

poz

poz

Poz

Copyright @ by The Berkeley Review

246

The Berkeley Specializing in MCAT

Biology
Passage

Leucine Catabolism

Practice Passage X

X (Questions 59-65)

59. Which of the following types of reactions is catalyzed


by Enzyme l?

In

animals, amino acids can undergo oxidative

degradation. Leucine, an essential ketogenic amino acid,

can be degraded to acetyl-CoA and acetoacetate as outlined in Figure l. Each reaction is catalyzed by a
specific enzyme and involves either an important cofactor or coenzyme. Many of these reactions are analogous to reactions found in other metabolic pathways.

A. B. C. D.

Hydration Transamination Carboxylation Dehydrogenation

foo HlN- C- H
CH,

coo

60. In human beings, the ultimate fate of the c-amino


group in leucine is BEST represented by which of the following metabolic structures?

o=c
I

HrC-

t-

CHr

l-

A.
H^
H_

B.
l(g|
H H2N
I

CH CHr

Enzyme

H1C-l

CH

CHr

NH

Leucine
@

o-Ketoisocaproate

- CUrea

il

NH2

NADH + H.,\l CoA-SH

-.

Ammonium ion
I

^ OA
NAD, CO2

Enzyme 2

--l+
S-CoA

c.
o il

D.

' o=f
s,C-

S-CoA C- H

oo?"t ,.oo \ ,/
Enzyme

o=

!
|

CH.

HN
I

.c

@t H1N-C'I

coo
CH.

o
H

c=o
N H N
H

t"

3 u,c- cn
CHI

otrc-

CHt

CH:

lO coo
Glutamate

B-Methylcrotonyl-CoA
I cotactorA I -.,1 \t/
Enzyme

Isovaleryl-CoA

Uric acid

El-i-rc] L--J :
-/l\

a )( tniotinS-coA

7 ucol
Dependenr)

61. The carboxylation reaction catalyzed by Enzyme 4

is

o=i
f CH, | '6

nro

,9=" nrccoo

S-CoA

o=i
CH,

a biotin-dependent carboxylase, which also requires cofactor A. Cofactor A is:

llnzYme

?*' 5 H,c- c- oH
coo p-HydroxyB-methylglutaryl-CoA

A. B.
C. D.

HzO. ATP.

-/
+ ,/

NADH.
FADH2.

l-9

B-Methylglutaconyl-CoA

62.
Enzyme 6

In the reaction catalyzed by Enzyme 4, the bicarbonate ion (HCO3e ) is converted into
Compound C. Compound C is:

o o ooc- cH. - 'll c- cHl


Acetoacetate

HrC- C-

S-CoA

Acetyl-CoA

Figure

I
247

A. B. C. D.

H2CO3.

NH:.
HzO. COz.

- :pyright @ by The Berkeley Review

The Berkeley Review Specializing in MCAT Preparation

Biology

Leucine Qatabolism

Practice Passage X

63. The enzyme holocarboxylase synthetase appears to be responsible for attaching biotin to at leist four different apocarboxylase enzymes in mammals.

In the

Biotin is involved in specific carboxylation reactions. absence of the holoenzyme, which substrate Aceryl-CoA
o-Ketoisocaproate B-Methylcrotonyl-CoA

begins to accumulate most rapidly?

A. B. C. D.

p-Methylglutaconyl-CoA

64. Which of the following types of reactions is catalyzed


by Enzyme 6?

A. B. C. D.

Thiolysis Aldol condensation


Reverse aldol condensation

Hydrolysis

65. If leucine were radioactively labeled with 14C at the d-carbon atom, we would expect to find that label
entering cellular respiration at which of the following
stages?

A. B. C. D.

Glycolysis Krebs cycle Electron transport Oxidative phosphorylation

Copyright @ by The Berkeley Review

24a

The Berkeley Specializing in MCAT

Biology
Passage

Trehalose Dxperiment

Passage Xl

XI (Questions 66 - 72)

Trehalose is a naturally occurring sugar with the following structure:


cH?oH

The following graph indicates the resulrs of a pilot study performed with four test subjects using the
experimental protocol just described:

Legend

n tr
ffi
Figure 1. Trehalose Trehalase is an enzyme present in the microvilli on the surface of cells in the duodenum, the first twelve inches -rf the small intestine. This enzyme hydrolyzes trehalose, r sugar found in certain beetle larvae and in mushrooms,

0 g fehalose 20 g trehalose 30 g trehalose

I I
tr

40 g trehalose 50 g trehalose

*30
o
bo
CJ

-nto two monosaccharides. Researcher are studying ruman tolerance for different amounts of the sugar, recause it is of interest to people in the food processing
ndustry as a possible food preservative.

:E

820

o
o
E

o
-o

The following experiment was devised to learn more ,bout individual tolerances for trehalose. Each subject .,, as given five incremental doses of trehalose, or fewer if iey felt gastrointestinal distress and had to stop the :rperiment. Gastrointestinal distress was defined for this :rperiment as diarrhea, bloating, or gas pains. Breath ,:mples were collected in leak-proof syringes and ,jected directly into a gas chromatograph for analysis of
-

Subject

Subject

Subjecr

Subject 4

Figure 2. Effects of trehalose ingestion on concentration of hydrogen in the brearh.

i drogen concentration.

Part

l:

Subject drinks 200 mL of distilled water. Breath hydrogen is collected and analyzed
after 2 hours.

66.

What process causes the increase in breath hydrogen


when a trehalose dose is consumed?

PartZ: Subject drinks 200 ml of distilled water containing 20 g of trehalose. Breath


hydrogen-is collected and analyzed after
hours. Part
2

A. B. C.

Digestion of the trehalose in the small intestine leads to the production of hydrogen gas in the intestinal tract. Bacteria in the colon ferment the trehalose and produce hydrogen.

3:

containing 30
hours.

Subject drinks 200 ml

g of

of distilled

warer

The interaction between bicarbonate from the


pancreas and trehalose leads to the production of hydrogen. Hydrogen is produced when trehalose is denatured in the stomach's acid.

trehalose. Breath

hydrogen is collected and analyzed after 2

D.

Part

4:

Subject drinks 200 ml of disrilled water containing 40 g of trehalose. Breath


hydrogen is collected and analyzed after 2
hours.

67. Which

subject exhibited a dose,response effect during all five parts of the experiment?

Part

5:

Subject drinks 200 ml of disrilled warer containing 50 g of trehalose. Breath


hydrogen is collected and analyzed after 2
hours.

A. B. C. D.
249

Subject I
Subject 2 Subject 3 Subject 4

:;'right

@ by The

Berkeley Review

The Berkeley Review Specializing in MCAT Preparation

Biology
68. Which two

Tfehalose Experiment
monosaccharides does the enzyme

Passage

il

72.

trehalase produce when it hydrolyzes trehalose?

A. B. C. D.

Which subject exhibited a threshold effect during tbe experiment? (Exhibiting a threshold effect means that after a certain dose, the subject's responsa
changes dramatically.)

Glucose and galactose Glucose and fructose Galactose and fructose Glucose and glucose

A. B. C. D. if
lactose were

Subject I
Subject 2 Subject 3

None of the subjects experienced a threshof,d


effect.

69. What results


substituted

should be expected,

for trehalose in this experiment, using


both

the same subjects?

A. Since lactose and trehalose are B. C. D.


sould be expected.

disaccharides, the same pattern of intolerance

Lactose would be tolerated at higher doses than trehalose, since it is a commoner


component of the diet. Lactose would be tolerated at lower doses than trehalose, since it is a less common component of the diet.

One cannot determine the answer from the


information presented in this passage.

70,

What can you conclude about Subject


results shown in Figure 2?

from the

A. B. D.

Subject
the diet.

has a high tolerance for trehalose in has a low tolerance for trehalose in

Subject
the diet.

C. Subject I regularly
containing foods. containing foods.

consumes trehalose-

Subject 1 almost never consumes trehalose-

71. The basic function of the gas chromatograph, like the one used in the experiment described in the passage to analyze the concentration of hydrogen in

the breath samples, is to

separate gaseous

compounds in a mixture on the basis of:

A. B. C. D.

their densities. their molecular interactions within a column. the masses of each isotope in the compound. their solubilities in organic solvents.

Copyright

by The Berkeley Review

250

The Berkeley Specializing in MCAT

Biology
Passage

Fuel Oxidation during Dxercise


7

Passage XII

XII

(Questions

3-79)

75.

Which of the following compounds is an important fuel source for resting muscle?

Exercise is a time of dynamic change in fuel oxidation

ior skeletal muscles. During rest, skeletal muscle respires


aerobically. Skeletal muscles respire anaerobically for the tlrst 45-90 seconds of moderately intense exercise. As erercise progresses, the muscles switch to aerobic
respiration.

A. B. C. D.

Oleate

B-hydroxy butyric acid

Lactic acid
Glycogen

Table 1 outlines more specifically the fuels used Juring different phases of exercise. The following :bbreviations are used: FFA (free fatty acids), GMG
slucose from muscle glycogen), BG (blood glucose), Gly glycogenolysis), and Gng (gluconeogenesis).

76.

What is the carrier molecule for blood glucose?

Liver Glucose Output


PhaseT Rest
5- 10

FFA

GMC

BG
+

from Gly from Gng


75Vo 25Va

A. B. C. D.

Albumin
Glucose-bindingprotein Hemoglobin There is no carrier.

+++ +++
+

10-40

++
+

++

40-90 by 240

++

+++
++

I
T
557o

t
77.

+++

457o

of these hormones involved in blood glucose maintenance?


During exercise, which

IS

Time in Minutes

Table

A. B. C. D.

Insulin
Glucagon

Glycogenphosphorylase Oxytocin

73. Why is the first interval of moderately intense


exercise a period of anaerobic respiration?

A. B. C. D.

Oxygen supply is transiently lower than oxygen demand. The Krebs cycle enzymes required for aerobic respiration must be activated. Fatty acids must be metabolized anaerobically. Glycolysis cannot keep pace with the need for
energy.

78.

Which fuel provides the MOST energy per gram?

A. B. C. D.

Fatty acids Blood glucose Glycogen

Amino acids

?{.

How does the blood glucose level change during


moderate exercise?

79.

Which of the following statements is TRUE of


exercise that continues for more than four hours?

A. B.
C. D.

Blood glucose rises rapidly during exercise,


and then plateaus at a higher level. and then plateaut at a lower level.

A. Muscle
B.
C. D.
decrease.

glycogen

is repleted by

gluconeogenesis.

Blood glucose .falls rapidly during exercise,

The rate of gluconeogenesis continues to

Blood glucose is relatively unchanged by


exercise. Blood glucose rise and falls in a cyclic fashion as exercise progresses.

The availability of free fatty acids from

The rate of glycogenolysis continues to


decrease.

adipose tissue begins to decline.

Copyright

by The Berkeley Review

251

The Berkeley Review Specializing in MCAT Preparation

Biology
Passage

Urea Cycle

Passage XIII

XIII

(Questions 80-87)

Every amino acid that is catabolized by the human body must have its o-amino group removed from its carbon skeleton. Amino groups that are not reused in the
synthesis of nitrogenous compounds are converted to urea in the urea cycle (Figure 1):
2ADP +Pi

Glutamate, once it passes into the mitochondrial matrix from the cytosol, can lose its amino nitrogen as the ammonium ion in a reaction involving water and NADe or NADPo, or it can be converted to cr-ketoglutarate, as
indicated by the reverse reaction in Figure 2. The urea cycle involves four enzymatic steps, each of compound necessary for the synthesis of an amino acid (Figure 3).

which catalyzes a unique intermediary

I Ir znrp .-\LHCo3e + NH4o -cg|:nr'] pnospnate E,


I Citrulline ornirhin" I f
<---.
l
I

H3N-CI

o
?oo
H

H1N-l C-

foo
H

H1N-C-H -l
CH,
I

@ too
CHt
I

HrN-C-H -l
CHr

foo
tt-

CH,
CH,
I

CH'

lt' t'
I

cH'
CH" NH

CH'
CHt

Aspartate

U
|

^tt NHt

CH,

CH"

o=c
I

H,N=C -l
I

oT"
NH

t-

tI

Ot
H2N=

NH
C

@ NH:

NH:

l'' "*,+,L^.,]-*J :',"ff). | ",;;'-*"1 oxa,o. )


.fJEHfY{

ooc- c- c- coo HzH o O


Ornithine Citrulline Argininosuccinate

Argininc

Malate Citrate I Krebs lEo cycte + Fumarate cr-ketoI

G*",u,.1

Figure

. succrnare

t'"'r""
coe,'

Succinvl-

80.

Urea is primarily synthesized in the:

Mitochondrial matrix

A. B. C. D.

kidney.
blood.

liver.
muscle.

Figure

by transaminases,"in enzymes that interconvert o-amino and a-keto acids during transamination reactions. The ketoacid acceptor of many amino groups is c-ketoglutarate.

Amino u"ia,

have their c-amino group removed

81. According to the passage, what is


balanced reaction for the urea cycle?

the

A. B.
C.
D.

o coo
c=o
CH,
CH'
I I

o foo H3N-C-H
R

H3N-C

@i*
CHz

o
"ooo c=o
I I

2NH+o + HCO:o + 3ATP + HzO -+ Urea + 2ADP + AMP + PPi + 2Pt 2NH+e + HCQo + 3ATP + Aspartate + Urea + 2ADP + AMP + PPi + 2Pi
NH+@ +

-H

| -o coo

Amixo
acid

to coo
Glutamate

HCqe

+ 3ATP + H2O -+

cr-Ketoglutarate

Keto acid

Urea + Fumarate + 2ADP + AMP + PPi +

Figure 2

NHao + HCqe + 3ATP + Aspartate + H1 Urea + Fumarate + 2ADP + AMP + PP; +

Copyright

by The Berkeley Review

252

Biology
82.
NH4o and:

Urea Cycle
86.

Passage XIII
Argininosuccinicaciduria (ASA) is a rare autosomal recessive disease caused by the absence of argininosuccinase (E4 in Figure l). Its primary

The two nitrogen atoms in urea can be traced back to

A. B. C. D.

citrulline.
arginosuccinate. arginine. glutamate.

the blood. A man whose morhJr had ASA and whose father did not carry the ASA gene marries a woman with the disorder. What is the probability that their child will NOT express the trait?

symptom is the accumulation of argininosuccinate in

83.

directly from the:

The carbon atom in carbamoyl phosphate comes


glycolyric pathway. citric acid cycle.
electron-transportchain. oxidation of fatty acids.

A.

A. B. C. D.
{.

D.
87.

B. c.

0.25

0.s0
0.'7s 1.00

toxic compound?

The ammonium ion is quite toxic to brain tissue. Which of the following amino acids is able to react with the ammonium ion and convert it into a non_

compound easily excreted in the urine.

pathway to reduce the toxic concentrations of a particular metabolite). Administration of sodium benzoate leads to the formation of hippurare, a
o

Hyperammonemia can be caused by a deficiency in arginase (E5 in Figure l). This disorder can be controlled by the restriction of protein in the diet and by diversion therapy (use of an alternative metabolic

A.
coo @t HjN- C- H
I

B.

H3N- CI

ot coo
coo

o-\ -$-fi,-8-oo

CH,

t-

CH, -a

SH

"
individuals is that it:

Hippurate

D.

@t
H3N-

coo

CI

HrN- C-

Orcoo
CH.

o
H

The rationale for the use of sodium benzoate as a therapeutic procedure for hyperammonemic

CH, CH.

coo

t' | "g

t'

H-N--N-g

F:n

A. _ B. _ C. D.

reduces the concentration of ammonia in the blood by increasing the synthesis ofglycine. reduces the concentration of ammJnia in the blood by increasing the synthesis of aspartate. increases the concentration of urea in the blood so it can be eliminated in the urine.

ils

Which of the following types of reactions can result in the formation of ammonia by the removal of the
o-amino group from an amino acid?

increases the concentration of Krebs cycle intermediates in order to produce more


aspartate for the urea cycle.

.{.
B.
C. D.

Transamidation

Oxidative deamination d

Reductive deamination Dehydrogenation

ii,-:i_eht @ by The Berkeley Review

2s3

The Berkeley Review Specializing in MCAT preparation

Biology
Passage

Gluconeogenesis and the Qori Cycle


90. Which of

Passage XfU

XIV (Questions 88-94)

these statements would describe

th

metabolic fate of lactate under anaerobic conditiom! Under normal physiological conditions, the brain uses

about 120 grams of glucose each day as its only fuel source, compared to the roughly 160 grams of glucose
that the body uses each day. Since the body stores less than a day's supply of glucose, this carbohydrate must be synthesized from noncarbohydrate precursor molecules
during times of fasting or intense physical exertion. The synthesis of new glucose occurs during a process called gluconeogenesis.It takes place primarily in the liver and, to a lesser extent, in the cortex of the kidney. Glycerol, lactate, pyruvate, and many amino acids (e.g., alanine) are all important precursors of glucose. Lactate is an end product of anaerobic glycolysis and is produced primarily in erythrocytes and skeletal muscle. When lactate is released into the bloodstream, it travels to the liver where it is taken up by the hepatic portal vein and reoxidized to pyruvate in the cytosol ofhepatocytes.

L n. ilI. A. B. C. D.
9t.

Lactate is converted to pyruvate. Laclate is converted to alanine. Lactate is converted to lactose.

I only

II only I and II only I, II, and III

What metabolic condition could activate the

Cori cycle, in which there is glucose output


muscle tissue and lactate output from the liver?

A.
B.

c.
D.

Pregnancy activates the reverse Cori cycle. Exercise activates the reverse Cori cycle. Fasting activates the reverse Cori cycle.

There is no metabolic condition that


activate the reverse Cori cycle.

The cycle of reactions involving the production of glucose from lactate in liver tissue and the production of lactate from glucose in muscle tissue is called the Corl
cycle (Figure 1).
Glucose Glucose

92.

If
A.
B. C.

lactate dehydrogenase were inhibited

anaerobic conditions, what metabolic effect this have?

4^rP
2

GrP

jll

+
ll

[,o,"
Pyruvate

Glycolysis would eventually stop due to a ofNAD@. Glycolysis would continue, but without
production.

Pyruvate

+
il

Glycolysis would eventually stop due to a

n
Lactate

D.

ofATP.
Glycolysis would continue, using FAD
electron transporter.
as

Lactate In liver
Figure

tissue
Cori cycle.

In muscle tissue

93.

Which of these statements correctly


alanine and its biochemical derivation?

de

1. The

A.
88.

Alanine is a nonessential amino acid


produced by transamination from pyruvarc-

Under what conditions would muscle release the


MOST lactate per minute?

B. C.
D.

Alanine is an essential amino acid


produced by transamination from lactate.

A. B. C. D.
89.

During sleep During studying During vigorous exercise Following a light meal

Alanine

is an essential amino

acid

produced by reduction from lactate.

Alanine is a nonessential amino acid


produced by reduction from pyruvate.

Which of the following enzymes is involved in the first step ofgluibneogenesis in the liver?

94. When glucose produced by gluconeogenesis the liver, it exits through which blood vessel?

A. B. C. D.
Copyright

Pyruvatedehydrogenase
Pyruvate carboxylase Lactatedehydrogenase

Lactatecarboxykinase
@

A. B. C. D.
254

The The The The

hepatic hepatic hepatic hepatic

artery vein portal vein portal artery

by The Berkeley Review

The Berkeley Specializing in MCAT

Biology
Passage

Starch Blockers
95.

Passage XV

XV (Questions 95-100)

One volunteer for the study was not allowed to

developed and marketed

the "starch blocker" tablet. Starch blockers were to inhibit the intestinal

In the early 1980s, a popular weight reduction fad was

carbohydrate-hydrolyzing enzyme, o-amylase. In theory, a dieter could consume unlimited amounts of complex carbohydrates, take a pill containing the starch blocker, and be protected from weight gain, since the starches rr ould not be digested. Instead, they would pass through the small intestine mainly undigested.

participate, because she produced no increase in breath hydrogen following the administration of lactulose in the first phase of the study. Why would this disqualify her from the parriciparion?

A. B. C. D.

She had too many colonic bacteria that ferment nondi gestible carbohydrate. She did not have enough ofthe colonic bacteria

that ferment nondigestible carbohydrate.


She had too many colonic bacteria that ferment

Further claims about starch blockers focused on the ibsorption of glucose and the lowered insulin response to
:ngested glucose after they were used. Several researchers planned the following experiment to examine the effects ,f starch blockers. Breath hydrogen content was measured :-r determine fermentation of undigested starch by colonic ::cteria. Eight men and women participated in each of the 'rree phases ofthe study:

digestible carbohydrate. She did not have enough of the colonic bacteria that ferment digestible carbohydrate.

96.

Figure 1 indicates the breath hydrogen response to all three phases ofthe study:
90

tt:ase I: Lactulose Test


..

15 grams of the nondigestible carbohydrate lactulose ere administered, and breath hydrogen concentration rs measured over 4 hours.

ca

Eto o e f50 3:o


L

10

::'.cse

II:

Test meal with placebo


Figure

0 60 120 180 240 300


Time (minutes)

-:.ms of starch. Subjects took a placebo capsule " ltaining calcium carbonate with the meal. Breath
-..

Subjects ate a test meal ofregular foods, containing 50

l.

Breath H2 response as a function of time.

Jrogen concentration was measured over 4 hours.

Which of these statements could be supported by the data in Figure 1?

I.
"
".":se

III:

test meal with starch blocker

U. III. A. B. C. D.
97.

-*:ns of starch. Subjects took a test capsule containing of phaseolamin, an c-amylase inhibitor, " :rmilligrams the meal. Breath hydrogen concentration was
- t.-rured over 4 hours.

5ubjects ate a testlneal ofregular foods, containing 50

The starch blocker acted in a manner that was similar to the placebo. I actulose promoted a rise in breath hydrogen beginning about an hour after it was given. Intestinal c-amylase was nor inhibited by the starch blocker.

I only
I and III only

II

and

III only

I, II, and III

Intestinal o-amylase is synthesized in the:

A. B. C. D.
'

small intestine. gall bladder.


large intestine.
pancreas.

r-ight @

by The Berkeley Review

255

The Berkeley Review Specializing in MCAT preparation

Biology
98.
study.

Starch Blockers

Passage XY

The following graphs show the glucose and insulin responses of test subjects to Phases II and III of the

100. How does an actual inhibitor of cr-amylase affect th


digestion ofthe simple sugars glucose and fructose?

A.
Placebo Starch blocker

B.
F

r<n

Ae ,o0
do-

o^

v.

roo
5u

D. 0 30 60 90 120 150
Time (minutes)
180

The digestion of simple sugars is unaffected the starch blocker. The digestion of simple sugars is affected the same way as starch digestion. TL^ r rrs ll-^^+!^utEsJrtuil ^f ur ^:--l^ stlrtprg ^--^^-^ suBalrs :^ ts blocked by an inhibitor of o-amylase.

Simple sugars are destroyed by


actions of the cr-amylase inhibitor.

Figure 2. Glucose response.

r40
Placebo

7o F>

100

Starch blocker

60

f20
0 30 60 90
Figure 3. lnsulin response. 120 150
180

Time (minutes)

These data indicate that when phaseolamin was


given to these subjects, it produced:

A. B. C. D.

an increased glucose response and a decreased

insulin response compared to the placebo test.


a decreased glucose response and an increased insulin response compared to the placebo test. no change in glucose or insulin responses compared to the placebo test.

decreases

in both the glucose and

insulin

responses compared to the placebo test.

99.

The test meal contained 50 grams of starch in foods the subjects ordinarily consumed. Other components of the test meal were protein, simple sugars, and fat. Which of the following foods is mostly carbohydrate

in the form of starch?

A. Turkey 'rB. Rice C. Butter D. Orange juice

Copyright

by The Berkeley Review

256

The Berkeley Specializing in MCAT Pre

Biology

Metabolic Pathways

Section VIII Answers

1.

B is correct. The faster the growth rate of an organism, the shorter its doubling time. The guiding principle in problems such as this one is that growth rate is directly proportional to the amount of ATP available to each cell per unit time. In the presence of plenty of 02, prokaryotic organisms (such as E. coll can make 38 NTPs per molecule of glucose oxidized, whereas eukaryotic organisms (such as yeasts) can make either 36 or 38 ATPs. For the purposes of this question, we were to assume 36 NTPs per molecule of glucose oxidized (under aerobic conditions). When oxygen is removed, the yeast cells switch over to anaerobic metabolism, and the net yield of NTP per molecule of glucose falls to just 2. This is called anaerobic fermentation.
Depriving yeast cultures of Oz causes each cell to produce 18 times less NTP per molecule of glucose consumed (from3612 = l8). Pasteur found that yeast cells grew 6 times less rapidly under these conditions, implying that they were forming NTPs at a rate only 6 times less rapidly than before. This means that they must be consuming glucose more rapidly than before (as was noted in the question). In particular, they must be consuming glucose at a rate approximately 3 times faster (from l8/X = 6, or X = 18/6 = 3). The correct choice is B. C is correct. In the presence of oligomycin, the Fs transmembrane protein is blocked. The proton gradient formed by electron transport and proton-pumping can no longer be relieved. The result is that the cells revert to anaerobic metabolism and just do glycolysis, synthesizing only 2 net NTPs per molecule of glucose consumed. The metabolic products will be ethanol (CHaCHzOH) and carbon dioxide lCOz). The overall effect is the same as if you had removed the cells from oxygen, or had added an electron-blocking agent such as cyanide. The correct choice is C.

3.

C is correct. In this case, oxidative phosphorylation is still prevented by the binding of oligomycin ro the

Fo

can and will continue. The end products of glucose metabolism will be COz and HzO. If anything, the rate of electron transport will be a little faster, since no NTPs are being formed during electron transport (i.e., there is no resistance to electron transport). But overall, the growth rate of the culture would be expected to decline. A net of 4 NTPs are synthesized per molecule of glucose metabolized. Why? 2,4-DNP uncouples all of the NTPs that are normally formed by oxidative phosphorylation, but leaves unaffected any NTPs formed by substrate-level phosphorylations. Those substrate-level phosphorylations include the 2 net ATPs formed in glycolysis and the 2 net GTPs formed during the Krebs cycle. Therefore, in the presence of excess 2,4-DNP, the amount of NTP formed per glucose molecule when metabolized under aerobic conditions can be expected to decline from 36 to 4. The correct choice is C.
4.

transmembrane protein; but the presence of 2,4-DNP means that no proton gradient is formed, so electron transport

B is correct. During normal glycolysis, one of the high-energy phosphate compounds, which serves as a source of phosphorylating energy for the synthesis ofATP, is 1,3-bisphosphoglycerate.

-o U ^lO U. P-O - c- O- Il ro H-C-OH


I

ATP

ADP \
StepT

'-

o- no ''cI

H- C- OH
I

H2C-O -

POj2-

Phosphoglycerate kinase

Hrc- o -

Po.2-

1,3-Bisphosphoglycerate

3-Phosphoglycerate

In the presence of excess arsenate ions, the synthesis of ATP in Step 7 does not occur. In Step 6, the compound 1arseno-3-phosphoglycerate is hydrolyzed to inorganic arsenate and 3-phosphoglycerate, the intermediate we would see at the end of Step 7.
The overall result is that glycolysis appears to proceed normally, but there is no net formation of ATP. Two ATPs are invested in Step I of glycolysis; but in the presence of excess arsenate, only two ATPs can be synthesized (in Step l0 of glycolysis when phosphoenolpyruvate is converted to pyruvate). Therefore, there will benonet gain in ATP and no conservation of chemical energy. All of the free energy difference between glucose and pyruvate will

be lost as heat. The cells must stop growing, since they are no longer able to make any ATP under these
circumstances. The correct choice is B.

Copyright @ by The Berkeley Review

2s7

The Berkeley Review Specializing in MCAT Preparation

Biology
5.

Metabolic Pathways

Section VIII Answers

C is correct. Arsenate competes with phosphate for the active site of the dehydrogenase enzyme at Step 6 in glycolysis. Note that arsenate is an analog of phosphate. If we were to flood the system with phosphate, then phosphate would outcompete the arsenate, and the (majority of) cells would continue to thrive. If arsenate were a noncompetitive inhibitor, then: (l) it would not resemble phosphate, and (2) no matter how much phosphate we added to the system, the inhibition would not be overcome.
Arsenate can also be considered an uncoupling agent, because it uncouples the phosphorylation of ADP to ATP in Step 7 but allows glycolysis to proceed. The action of arsenate is analogous to the action of 2,4-DNP in the electrottransport chain, in that they both uncouple a phosphorylation event. The correct choice is C.

6.

A is correct. It is important to know the basics of metabolism. Pyruvate is the end product of glycolysis, occurs in the cytosol. Pyruvate can pass from the cytosol to the mitochondrial matrix through a pyruvate-l symport. Once in the matrix, it is oxidized to acetyl-CoA. In this process hydrogens and electrons are passed NAD@, and NADH + Ho is formed. In Figure 1 of the passage, we see that NADH + Ho enter the electronchain at the level of complex I (also called the NADH-Q reductase complex). The correct choice is A. B is correct. As shown in Figure 1 of the passage, hydrogen ions are vectored across the inner mi membrane from the matrix to the intermembrane space. This not only establishes a difference in the [HCI] the matrix and intermembrane space, but it also establishes a charge difference. Note the net positive charge on surface of the inner membrane facing the intermembrane space and the net negative charge on the surface of
inner membrane facing the matrix. This electrochemical gradient allows the hydrogen ions to come back into matrix through the FoFrATPase. In the process, ATP will be synthesized from ADP and Pi. Choice A says that electrons are passed from NADH and FADHz to Oz. This is true, but it does not answer question. In choices C and D, ADP is indeed transported into the matrix, and it can occur by way of either antiport or a symport. Even though ADP enters the matrix, it does not account for the mechanism behind A synthesis. The correct choice is B.

7.

8.

C is correcL First, we need to draw a picture, so that we can visualize what is happening. In Figure A below, we a phospholipid vesicle with an Fq protein. Inside the vesicle is an elevated concentration of K@.
Fo Protein

inR,r*

\\
Phospholipid

A {H+
Valinomycin

lH+l

Bilayer

\\
efflux
Vesicle Figure B

Vesicle Figure A

Based on information in the passage, we know that the Fe protein facilitates the specific transport of Ho membrane. If the Fo protein is not in the membrane, He cannot be transported across. If the Fo protein is membrane, H@ can be ffansported across if there is some type of gradient. In the case of the inner membrane mitochondrion, H@ is vectored into the intermembrane space. This allows for a chemical and electrical Ho across the inner membrane. In the case of the inner mitochondrial membrane, the F6 protein allows H@ to to the matrirdown its chemical and electrical gradient.

ir

In the case of these synthetic vesicles, the situation is pretty much the same. In order for Ho to enter the there needs to be a gradient. Within the vesicles is a high concentration of K@. Once the antibiotic valinom cyclic peptide) is added to the solution, it diffuses through the membrane and into the vesicle's interior, binds a ion, and then transports that ion to the exterior of the vesicle (Figure B). Since K@ is leaving the vesiclg
Copyright
@

by The Berkeley Review

254

The Berkeley Specializing in MCAT

Biology

Metabolic Pathways

Section VIII Answers

referred to as efflux. Valinomycin can transport up to about 104 K@ ions/second through the membrane. This decreases the [Ko] inside the vesicle, which in turn means there is less positive charge at the interior. The Fo prote in now allows the H@ to pass to the interior of the vesicle. Since H@ is entering the vesicle, it is referred to as influr. The correct choice is C.
9.

C is correct. DCCD is inhibiting the passage of H@ through the Fo portion of the FoFtATPase protein. This is telling us two things: First, since FP cannot get back into the matrix, these ions are accumulating in the intermembrane space. Eventually, there will be such a high [Ho] in the intermembrane space that H@ will no longer be able to be vectored from the matrix to the intermembrane space. In other words, NADH and FADHz will not be able to drop off their electrons and hydrogens. The electrons will not be able to be passed down the electrontransport chain. Second, since He cannot pass thorough the Fo pore, ATP cannot be synthesized. The net result of
DCCD addition is elimination of electron transport and ATP synthesis. 2,4-DNP is an uncoupling agent and has the ability to pick up a H@ in the intermembrane space and transport it across the inner mitochondrial membrane. This transport of H@ through the membrane bypasses the FoFtATPase system. This reduces the [H@] in the intermembrane space. In turn, this allows more H@ to be pumped from the matrix to the intermembrane space. When this happens electrons are dropped off at the electron transport chain and o the rate of electron transport increases. However, since DCCD is blocking passage of H through the FoFtATPase, ATP cannot be synthesized. Even if DCCD were not present, we would still see a decrease in ATP synthesis because of the uncoupling action of 2,4-DNP. The correct choice is C.

10.

B is correct. In the question we are given the equation that relates the standard free energy (AGo') to the standard reduction potential (Eo'). We are told that F is the Faraday constant and that n is the number of electrons involved in the transfer. We do not need the value of the Faraday constant. Even if we did, we could just consider the value as a constant (i.e., unity) to simplify calculations.

\co,-

(nXFXAEo')

We find the value for the number of electrons in the passage (see second paragraph) and in the table in the question. FADH2 can pass 2 electrons to the electron-transport chain. Our equation now looks like the following:

aco'

(n)(F)(AEe') = _ (2XF)(AEo')

In the table in the question, we also find the values for Eo'. Note that the redox pairs are written as reduction halfreactions.

Redox Pair

Eo'

{voltsl

Fumarate + zHo + 2eo

-"-+

Succinate

0.03 - 0.22

FAD + 2H@ +Zea

---

FADH2

The question now becomes one of arriving at the correct value for AEo'. The question it states that AEo' is the change in standard reduction potential between the oxidized and reduced species. What is the oxidized species and what is the reduced species? We need to consider the reaction given in the question.

FAD Succinate \

FADH'

'-

Fumarate

In this reaction, succinate is the electron donor. Succinate gives up its electrons to FAD, the electron acceptor. The electron donor isrcalledthe reductant,while the electron acceptor is called theoxidant. Succinate is oxidizedto fumarate, while FAD is reduced to FADHz. We can expand on our equation as shown below:
AGo'

- (nXFXAEo') = - (2XF)(AEo') = - (2)(F)(Eo' (acceptor) - Eo (donor))

right

by The Berkeley Review

259

The Berkeley Review Specializing in MCAT Preparation

Biology
If

Metabolic Pathways

Section VIII Answerg

we substitute the appropriate values, we get:

AGo'- -(2XFXAE9')=- (2)(F)(Eo'(accepror) -Eo (donor))= - (2XF)t(- 0.22) - (0.03)l=- (2XF)(- 0.25)
We can clean this up a bit:

AGo'- + (0.50XF)
This question tests our knowledge on oxidation-reduction reactions and the manipulation of signs. The
choice is B.
11.

D is correct. It is important to know the differences between prokaryotic and eukaryotic cells. Mitochondria found in prokaryotic cells. They are lound only in eukaryotic cells. Therefore, we can immediately eliminate A and B.

are

cell,

All prokaryotic cells have a plasma membrane that surrounds their cytosol. If the prokaryotic cell is a Gram-

it

also has an outer membrane (outside the peptidoglycan layer).

A Gram-positive bacterium has oniy

the cytosol. Since oxidative phosphorylation and electron transport are coupled, it would mean that elecl! transport occurs on the plasma membrane. And this is what is observed in both Gram-positive and Grambacteria. The correct choice is D.

plasma membrane surrounded by a peptidoglycan layer. Since a prokaryotic cell requires energy (ATp) to survir will want that ATP delivered to reactions in its cytosol as quickly as possible. Oxidative phosphorylation occurs

12.

"extracellular." This rules out answers containing cholesterol inside the cell. So choices B and D are wrong. C is wrong, too. Although cholesterol in arterial plaques is exogenous to the cell, it is not free to move aroundword sequestered means "trapped" or "stored away." The correct choice is A.
13.

A is correct. Exogenous means "from the outside." Its antonym is endogenous. For the cell, exogenous

C is correct. From the diagram, we see that LDL particles deliver cholesterol to cells. They must interact u'ith LDL receptor. If the LDL receptor is damaged or missing, LDL particles and their cholesterol remain in the This makes statement III correct. If LDL can't enter, then the negative feedback from exogenous cholesterol available. Therefore, the cell alters itself to increase intracellular cholesterol. This means levels of reductase are increased. Statementl is correct. Also, ACAT activity increases to hydrolyze stored cholesterll into free cholesterol. Statement II is wrong. The correct choice is C. B is correct. This is a simple change in units. (5 mmol/liter) x (386.64 mg/mmol) x (l liter/10 dL) = 193.3 The other choices are incorrect due to either multiplying or dividing error by a factor of l0 (choice C) or flippi units upside down (choices A and D). The correct choice is B. C is correct. Three of the answers are true, but the answer we want is the false one. We are told in the the lysosolne fuses with the LDL receptor-LDl particle complex. Hydrolytic enzymes degrade both esters to free cholesterol and the apolipoprotein into amino acids. Free cholesterol is released. This is a si
the cell decreases LDl-receptor synthesis. The free cholesterol can be reesterified into cholesteryl ester and the cytoplasm. Choices A, B, and D are all true. The false answer is choice C. The correct choice is C.

14.

15.

16.

A is correct. First of all, there is no extracellular cholesterol synthesis. Choices C and D are therefore inc diet provided no cholesterol, then the cells would need to synthesize more cholesterol to meet their requi Choice B is wrong. The correct choice is A.

17.

B is correct. Mevinolin is a competitive inhibitor of HMG-CoA reductase. On the Lineweaver-Burk competitive inhibitor has the same Vmax as the enzyme acting alone, but the Ku is higher. Only Line B c y-axis at the same point as the line for HMG-CoA reductase. This means choice B is correct. Choice C i noncompetitive inhibitor. Choices A and D are just tricks. The correct choice is B.

18.

B is correct. The names of the lipoproteins comes from their activity during centrifugation. Extraction organic solvent would separate fat-soluble components from water-soluble components, but this is not want. Choice A is thus incorrect. TLC requires organic solvents as well, so the lipids would all mix
26,0

Copyright @ by The Berkeley Review

The Berkeley Specializing in MCAT

Biotogy
means choice

Metabolic Pathways
c

Section VIII Answers

:::,Tl'rffi#l'ff"1'ii1}t"fl:1"';T;.this
19.

is also incorrect' A salt solution.would not help: choice D is incorrect. Take advantage of the table is how ttev sr,oi,iJb" sepa.ated, which is best accomprished by

20.

B is correct' Read this from the table. chylomicrons. have the largest diameters and the lowest densities. Therefore, they are relatively large and more buoyantihan the other lipoproteins. The correct choice is B. c is correct' This is not in the passage. we can eliminate VLDL, because the passage tells us those lipids are made in the liver' This eliminates choice"D.

If

3iiitr'Xk?i;::;H*ffiifii::'i$:"o'
21. Palmitate contains

we remember how ihe lipoproteins transform, we know that vLDL rhe rest comes do*n,o.'"^o.v. HDL does nor conrain
dietary ripid;

D is correct' Acetate is linked to CoA by the enzyme thiokinase, forming acetyl I is correct. l6 carbons, which come from 8 acetyl c"A;, ;; statement II coA. Statement r3c is also correct. is a stable isotope of carbon' Statement III is correct' because_stable isotopes are safer for use with human beings than at high doses. onlv staiement IV is incorrect, so choice D is the bert-answe.. The correct :X1t::1";.tsotopes
A is correct' Eliminate choices c and D, since we are trying to make methyl nucleophile in an acidic environment. The correct choice is A. A is correct' The group that is fasting is not doing much DNL, because
esters. Transesterification requires a

'r',

23.

;ffi:;l?::#::lXil:,::j|f
24.

converted to rat.-rhe sroups

;;i;;

that is a pathway that is most activated when carbohyirate a" oNL than the rasting

-"..

B is correct' In all four answer choices, we find that the steps in the calvin cycle catalyzed by E2(phosphorylation) and Es (condensation) are the same. Therefore, we do not need to concern ourselves with them, only with enzymes Et, Es, and E+. Let's consider these one at a time.
ribulose-1,5-bisphosphate carboxylase. It is also called rubisco.In the reaction, we see that coz and H2o are involved' The reactant Ru-1,5-BP is first carboxylated, which means that the reaction is a carboxylation Enzyme Et is

reaction' However' after the carboxylation event, the transient six-carbon intermediate is immediately hydrolyzed to two molecules of 3-Pc' Thus, this reaction is also a hydrolytic re""ri"". This tells us that the reaction catalyzed by El corresponds to either a carboxylation or (once the Cbz has been added) a hydrolysis. Enzyme E5 is involved in a dephosphorylation reaction, but is it a reductive or oxidative reaction? The coenzyme here is NADPH (+ H@)' This is the reduced form of the coenzyme. It reacts with 1,3-BpG, which is the oxidized form of the three-carbon compound. NADPH is converted to NADpo. This is the oxidized form of the coenzyme. l'3-BPG is converted to 3-PG' In the process inorganic phosphate (Pr) has been lost (i.e., a dephosphorylation) to the medium' 3-PG is the reduced form of the three-carboncompouno. itris sequence constitutes a redox reaction. As written' it is therefore areductive dephosphorylation. we.; coenzyme is reduced and which is oxidized. ";tiit;"swer this by considering which form of the
Oxidized form of the coenzyme (no H_atoms)
Reduced form of the coenzyme (has H-atoms)
________-

NADP+ NADPH +Pi -<


+

r1

Lostaphosohare (dephosphorylarion)

oxidized.l

o--.

I
At this point'

\ "lJ:""'"' u,c-o-eo,]
1,3-BPG

-o

- 'o''

'H* E3

- ":i_"1 |*"0,..0 srl_o_po,r, )


3-PG

we know that enzyme E3 is involved in a reductive dephosphorylation. This allows us to eliminate choices C and D. Once we can determine what enzymeE+ does, we will have the answer.

I . ryright

by The Berkeley Review

26t

The Berkeley Review Specializing in MCAT preparation

Biology

Metabolic Pathways

Section VIII Ans

molecule. This is exactly what we see below. Notice that there has been no loss of atoms.
CH,-OH

Enzyme E+ should look familiar to you (think of glycolysis). It is an isomerase enzyme that makes possible interconversion between G-3-P and DHAP. Isomerases simply rearrange the carbon and hydrogen atoms Lrm
o--

^, (
I I

c=o
I

H-C-OH

H,C-

PO,2

E^

Hrc-

Po,2'

DHAP

G-3-P

An enolase enzyme promotes the reversible removal of water from a molecule. We do not see the loss of u ata the addition of water) between G-3-P and DHAP. The correct choice is B.
25.

B is correct. The function of the Calvin cycle is to convert COz into carbohydrates. As shown in Figure lcarbohydrate that is produced is glucose, and glucose is a six-carbon sugar (i.e., C6H1206). Glucose is an i molecule to remember. If we end up with a six-carbon sugar, we must start with six molecules of COz. \\-e eliminate choices A and D.
The difference between choices B and C involves the number of ATP molecules and the number of water on the reactant's side. Let's consider the ATP molecules to see if we can deduce the correct answer.

In Figure l, there are only two places where ATP is used. If we react a molecule of COz with the five compound ribulose-1,5-bisphosphate, a six-carbon transient intermediate is formed, which is rapidly hydroll yield two molecules of 3-phosphoglycerate. Since six molecules of COz react with six molecules of iibulc', bisphosphate, the result will be twelve molecules of 3-phosphoglycerate. The next step involves the conversion phosphoglycerate into 1,3-bisphosphoglycerate. Twelve molecules of 1,3-bisphosphoglycerate are required ro twelve ATPs. That takes care of the first part of the cycle that generates ATPs. The second part of the c1'cre generates ATPs is in the conversion of ribulose-5-phosphate to ribulose-1,5-bisphosphate. Since six molei ribulose-1,S-bisphosphate are needed to react with six molecules of COz, then six ATps are also required ro the conversion from six molecules of ribulose-5-phosphate to six molecules of ribulose-1,5-bisphosphate. Thu requirement for ATP in a balanced reaction for the Calvin cycle is 18. Without going any furthei, we can ehmi choice C. The correct choice is B,
26.

A is correct. This question involves carbon balancing. Glyceraldehyde-3-phosphate is based on the noc glycerol, which is a three-carbon compound. At some point, glycerol was oxidized to the aldehl-de
(glyceraldehyde) and then phosphorylated, giving glyceraldehyde-3-phosphate. The point is that the mol, three-carbon compound. We have 6 of these molecules for a total of l8 carbons. Note that the Calvin cycle for all of the carbon atoms--not one is lost through decarboxylation.

that the molecule contains 2 phosphate groups, one on the C-l carbon and the other on the C-5 carbon. \[e conclude from this that Ru-1,5-BP contains 5 carbon atoms. We could also arrive at this same idea based name" ribulose" (Ru). In the Calvin cycle, we saw that one place ribulose can come from is ribose. Ribose u carbon sugar that is found in numerous compounds, including nucleic acids (DNA and RNA). If ue molecules of Ru-1,5-BP, it means we have 15 carbon atoms. Since we need only 12 carbon atoms, ttrmc carbon atoms to account for. Eliminate choice B. We can follow the same procedure for choice C. Xucontains 5 carbon atoms, E-4-P has 4 carbon atoms, and S-1,7-BP has 7 carbon atoms. This is a total of l6 atoms. In this case, we have 2 carbon atoms that are not accounted for. Eliminate choice C. Choice D is interesting. Glucose contains 6 carbon atoms. Since there are 2 glucose molecules, we have ll atoms. F-1,6-BP is on the pathway to glucose. It also has 6 carbon atoms. The total between these three mc l8 carbon atonts, which is how many we have with 6 molecules of G-3-P. At first glance, this seems like a

In choice B, we have 3 molecules of Ru-1,5-BP and 1 molecule of glucose. Glucose is a six-carbon this leaves 12 carbons to account for. How many carbons does Ru-1,5-BP contain? Look at the name.

\\e

How do we distinguish between choices D and A? To remove glucose as our product carbohydrate, we musl the Calvin cycle intermediates in order to regenerate 6 molecules of Ru-1,5-BP. If we do not, there will be 1,5-BP to condense with the COzthat is hanging out withrubisco (Er). This is exactly what will h
Copyright @ by The Berkeley Review

262

The Berkeley Specializing in MCAT Pre

Biology

Metabolic Pathways
I

Section VIII Answers


moleculeof F-1,6-BP.However,if weconvert

convertthe6moleculesof G-3-Pinto2moleculesof glucoseand

intermediates waiting in the holding tank can be combined to form a product carbohydrate (like glucose). The correct choice is A.

the 6 molecules of G-3-P into 3 molecules of Ru-1,5-BP and I molecule of G-3-P, then we can put that I molecule of G-3-P in a conceptual holding tank. The 3 molecules of Ru-1,5,-BP can combine with 3 molecules of COz. After the transient intermediate splits, there are again 6 molecules of G-3-P. This is what we started with in the question. If these 6 molecules of G-3-P are converted into 3 molecules of Ru-1,5-BP and I molecule of G-3-P (which we also place in a holding tank), then there are enough Ru-1,5-BP intermediates to continue the Calvin cycle. The two G-3-p

same place. Figure I indicates that G-3-P and DHAP combine to form F-1,6-BP. Since DHAP and G-3-P are in equilibrium with one another through an isomerase enzyme (E+), the label also appears at the C-1 carbon in DHAP. Again, there has been no realrangement ofcarbon atoms (because carbon-carbon bonds have been broken).

D is correct. The structure of 3-PG is given in the passage. The question states that the labeled l4C is at the carboxyl carbon (indicated by the dot (.) in the structures below). This is also the C- I carbon atom. 3-PG is eventually converted to G-3-P. Since there is no rearrangement of that C-l carbon atom, the label remains in the

o-...H E1 E3 i H-C-OH :=:::+ H-C-OH +", + Hrc- o- por2' Hrc-o - por23-PC c-3-p f
carbon atoms, which are the C-3 and C-4 carbons. HrC- OPOr2-

o...ro o

.
cH2-oH

H.c-

f=O

o-

po.2-

DHAp

G-3-P and DHAP combine in an aldol condensation to form F- 1,6-BP. This is achieved by the covalent union of the two C-1 carbons from both metabolites, The six-carbon sugar is formed, and the label appears at the two central

o\c- -H
I

C:o
.l HO- C- H al H- C- OH F"o-

H_C-OH

E" '-

HO-C- H .l H-C-OH H-C-OH


I I

.l

H- CI

OH

Hrc-

Po.2-

HrC-

O-

POr2

F-1,6-8P

G-6-P

F- 1,6-BP is converted to F-6-P and then to G-6-P with no rearrangement of carbon atoms. The labels do not change.

The correct choice is D.

D is correct. From the passage, we know that ATP and NADPH are synthesized during the light reactions and that these metabolitd3 are used during the (lighrindependent) dark reactions. When light is shining on the algae, ATP and NADPH are synthesized; when there is no light, ATP and NADPH are not synthesized.

in Step A of Experiment I, green algae are exposed to light and COz for an extended period of time. A lot of ATP and NADPH are synthesized. In Step B, the light source is removed, and l4COz is added. At this point, the cells are in darkness, so ATP and NADPH cannot be synthesized. This means that the supply of these metabolites to the
Calvin cycle begins to decrease. The reactions that rely on these metabolites are:
NADP+

ATP

ADP

coz
+ H2O

\/

Ru-5-P

\._
E1

ATP OI' /3-PG \


E2

Et3

NADPH +Pt +H+ l /\ 1.3-BPc o-r-,


Ej

ii the supply of ATP decreases, less Ru-5-P is converted to Ru-1,5-BP. Similarly, less 3-PG is converred to 1,3BPG, and less 1,3-BPG is converted to G-3-P. As we start our addition of lacoz, that label is incorporated into the R.u-1,5-BP that is still present and is converted to 3-PG. But 3-PG is not as readily converted to 1,3-BPG. Therefore,
263
The Berkeley Review Specializing in MCAT Preparation

Biology

Metabolic PathwaYs

Section VIII A

to increase. This is e the l4C label accumulates in 3-pG and the level of radioactivity of that metabolite begins what we see in the graph shown in the passage. converted to labeled GSome of this labeled 3-pG is still converted to labeled 1,3-BPG, which in turn is still ever-decreasing. The I are concentrations but their around, NADPH and ATp This is because there is still some into labeled Ruconverted will be metabolite this of some and Ru-5-P, way to its find in G-3-p will eventually is being synthesi: of it not enough Bp. The levels of raOloaciiuitv in this metabolite will be quite low, because The correct choice is D. 29.
a high level of radioac distributed (as we have previously discussed) . At the end of Experiment I, 3-PG had and Ru- 1,5-BP had a low level of radioactivity

B is correct. In Step A of Experiment III, the algae

are exposed the algae

to lacoz in darkness' The labeled

begins to accumulate and there is no COz, Ru-1,5-Bp cannot be converted to 3-PG. Therefore, Ru-1,5-BP of radioactivity. The level a low shows and high level of radioacrivity, while 3-PG begins to disappear choice is B.

the algae, and In Step B, all forms of COz are removed from the environment immediately surrounding in the Calvin cycle reactions The NADPH. ATP.and to synthesize lighh ;re turned on. The light reactions begin to the It is converted 3-PG labeled The rate. increased an at operate again on"" ian ;tffi;il;';;;"ii;r However' Ru-1'5-BP. I,j-SpC, and so on. Evenrually, the labelld Ru-S-P will be converted to the labeled

then

30.

C is correct. This question tests our recognition of the chemical structures of common sugars. Choice A is Choice B is sucrose. Choice C is correct, lactose. Choice D is cellobiose. The correct choice is C.

31.

D is correct. Bacteria are the most common organisms used in the fermentation of milk to produce y
However, cheese may be cultivated by the activity of both bacteria and molds. Alcohol is not produced in amounts by these particular fermentations (yeast is used for alcoholic fermentation). So, choice B is incorrecr acid gives yogurt its sour taste, but it does not break down lactose. Choice A is incorrect. Disaccharides are dimers. Microorganisms break down disaccharides such as lactose during fermentation. This makes the fermented product more tolerable to those lacking lactase. Choice C is incorrect. The correct choice is D.

32.

D is correct. Even if one has forgotten most of the digestive physiology that one learned, it is still possible to reason this out. Lactose is not broken down nor is it absorbed in the small intestine, if lactase is missing. Therefore, the lactose passes intact into the colon, where there are lots of bacteria. They rapidly ferment the lactose, producing gas and acidic compounds. The gas production is mentioned in the passage as a symptom, so statement I is correct Working from there, the bacterial production of excess acids irritates the colonic epithelium, and the bowel contens are passed through faster. Statement III is also correct. This produces diarrhea, as mentioned in the passage. Finally. both the unabsorbed lactose and the acids increase the osmolality of the colon contents, so that more water remains in the feces. This also increases diarrhea. Statement II is correct. The correct choice is D.

33.

B is corrcct. The enzyme is produced by (isolated from) a mold, but only the enzyme is commercially
means that choice D is incorrect. Exogenous lactase does not induce the production of anything in the intestinal mucosa, so choice C is also incorrect. Enzymes are broken down in the stomach, so lactase cannot pass to the small intestine. Choice A is incorrect. The lactase is added to the food before ingestion. It breaks down lactose in the food on the plate, in the mouth, and a bit in the stomach, before it is inactivated by stomach acids. The correct choice is B.

available-not the mold. This

34.

B is correct. This question addresses your test-taking skills, as well as your ability to extract answers from
passage.

the

the blood glucose would rise. This makes statement II correct. There is no mechanism in the digestive tract for the uptake of lactose itseli so the clinician would see no increase in blood lactose. Statement III is incorrect. Thus, eliminate choices C and D as incorrect answers. Finally. we are left with statement I. Even if statement I is true, we can see at this point that choice B is a better answer. since stateme.nt II is also correct. It does not say this in the passage, but if the test subject is lactose-intolerant, then the gas produced by the bacteria in their colon would increase and should be detectable in their breath after lactos ingestion. Hydrogen is a common gas to test for this assay. The correct choice is B.
35.

If the lactose is digested to glucose and galactose,

C is correct. This is a give-away question. Lactose is a sugar found only in milk, which is synthesized only in the mammary gland. The correct choice is C.

Copyright @ by The Berkeley Review

26,4

The Berkeley Revier Specializing in MCAT Preparation

Biology
36'

Metabolic Pathways

Section VIII Answers

C is correct. Lactase is released from the brush border membrane of the duodenum, the first twelve inches of the small intestine. This is the only location in the body where lactase is secreted. The correct choice is C.

37.

to FADHz. Also. if we remove

c is correct. In the first reaction of B-oxidation, we use the oxidized form of FAD to remove two hydrogens from the fatty acyl-CoA molecule.,The more hydrogens a carbon has on it, the more reduced that carbon atom is. The fewer hydrogen atoms the carbon atom has, the more oxidized it is. Therefore, the second molecule (enoyl-CoA) in the p-oxidation pathway is more oxidized than the fatty acyl-CoA molecule. This first reaction could be thought of as an oxidation reaction. Note that if the reaction is an oxidation reaction, then the oxidized form of FAD is reduced
hydrogen atoms from a molecule, then we can call that type of reaction
a
a

dehydrogenation reaction What this means is that we can call this first reaction either an oxidation reaction or dehydrogenation reaction. So far, this supports all four answers.

The second reaction involves water (H2o). Note that we add the water across the double bond. Instead of cleaving the molecule with water (a hydrolysis reaction), we are instead hydrating that double bond. This type of reaction is called a hydration reaction, and from it we get the hydroxyacyl-CoA deriiative. Choices A ancl B are eliminated.

reaction. This reasoning eliminates choice D.

In the third reaction' we use the oxidized form.of the NAD@ coenzyme to remove the hydrogen atoms associated with the B-carbon of the hydroxyacyl-CoA derivative. If we lose hyirogen atoms, the molecule becomes oxidized. This is therefore another oxidation reaction. Since we are losing hydiogens, it is also

.uit"d u dehydrogenation

or cleavage is not mentioned. These are one and the same thing, and they do occur. The molecule breaks apart. ihai much is obvious. But wh.at else is happening at this step? we ire adding a molecule of CoA. When CoA reacrs with the carboxyl group of the fatty acid, which has been shortened by two carbon atoms, there is an esterification reaction (see the second paragraph ofthe passage). The correct choile is c.
38.

What about the last step? we use coenzyme A (i.e., coA-SH) to cleave the p-ketoacyl-CoA molecule at the Bposition' CoA-SH contains. a thiol (SH) group. We cleave the molecule, which ii like lysing the molecule (breaking it apart). The reaction is indeed a cleavage, but more properly it can be called a ilriotlri, reaction.It is not a hydrolysis reaction, because we do not use water to cleavi that uona. However, in the answir for choice C, thiolysis

c is correct. Activation of the Ca fatty acid with CoA will cost I ATP. We thereby generate the fatty acyl-CoA molecule, plus an AMP and pyrophosphate (PPi). The driving force behind this activaiion is the hyclrolysis of ppi to 2 P1, which is equivalent to the energy contained in a high-energy phosphate bond (or another ATp equivalent). Hence, activation of the fatty acid costs us 2 ATp equivalents.
The activated fatty acyl-CoA proceeds through 4 rounds of B-oxidation and is converted into 4 acetyl-CoA molecules. Each acetyl-coA then enters the Krebs cycle. From the Krebs cycle, we get t Crp, I FADHz, and 3 NADH molecules. The NADHs drop off their electrons and hydrogens at the elect.on-tlnsport chain and produce 3 ATPs per NADH. The FADHz molecule does the same but prod-uces only 2 ATps per rinHz. The GTp can be
=

converted to'Rtp in a separate reaction. Therefore, we find that 1 acetyl-CoA molecuie is the potentially equivalent of l2 ATPs' Since we have 4 acetyl-CoA molecules, we can in theory syhthesize + x tZ +S nrps.

the3FADHzmoleculesproduce3x2=6ATPs,andthe3NADHmolecilesproduce3x5=gATps.Fromthese
two reduced coenzymes, we get a total of l5 potential ATps.

Complete oxidation of the Cg fatty acid yields 3 FADHz and 3 NADH molecules from the p_oxidation pathway itself. Note that cleavage of the last 4-carbon compound gives 2 molecules of acetyl-CoA. Both of these aceiyl-CoA molecules enter the Krebs cycle, but neither one undergoes B-oxidation again. Thus, from p-eli6ution we find that

Let's complete our ATP calculation. We add all the ATPs we can generate from acetyl-CoA, FADFb, and NADH, which is 48 + 15 = 63 ATPs' However, we invested 2 ATP equivalents to activate our fatty acid. Therefore, we produce a net total of 63 - 2 = 6l ATPs for the complete oxidation of the Cg saturated futty u.ij. The correct choice

isC.
39.

B is correct. when Franz Knoop first ran a similar experiment in Germany in about 1904, he attached a phenyl ring to the omega (rr1 or the last) position of various fatty acids and then fed those fatty acids to Jogr. If rhe fatty acid fed to the animals had an even number of carbon atoms, he discovered large quuniiti., of pheiylacetic in their urine (choice B).

Copyright @ by The Berkeley Review

265

The Berkeley Review Specializing in MCAT preparation

Biology

Metabolic Pathways

Section VIII Answers

in the urine, but this could still be metabolized to phenylacetic acid. If the fatty acid fed to the animals had an i,a.r
number of carbon atoms, their urine would have contained benzoic acid (choice A). The urine would also
h".-,e

Note that caprylic acid is an even-chained, saturated fatty acid. There would be some phenylbutanoic acid (choice

i,

contained some phenylpropanoic acid (choice C), but this compound could still be metabolized to benzoic acid. Tfo correct choice is B.

40.

carbons, as in choice D. One residue has label just at the C-2 carbon, as in choice C. Based on this alone, you rrug:r be tempted to pick choice B, because that species would predominate. But, what about the carbon dioxide? Acen CoA is completely oxidized to COz and HzO in the Krebs cycle, which means that all the carbon atoms of a f.:3 acid can potentially be lost as COz. Since there are five labeled carbons in caprylic acid, there will be five molecr.rrru, of laCOz being produced by the Krebs cycle. This makes choice A the best answer. The correct choice is A.

A is correct. If we degrade caprylic acid through B-oxidation, we get four units of acetyl-CoA, all having labals. Two of those residues have the label at the C- I carbon, as in choice B. One residue has label at both the C- I and C-l

41.

A is correct. The molecular formula of caprylic acid can be written


CsH16O2

as CsHtOOz. We are told that this compoun; completely oxidized to COz and HzO, a reaction we can write as follows (remembering to balance it):

ll 02 -------)

8CO2

+ 8 Hzo

The 8 moles of water shown in this equation are produced at the end of the electron-transport chain when ox\ l:rl (Oz) combines with the electrons and the protons. Then more water is produced when Abp combines with F- u yield ATP. If the complete oxidation of one molecule of caprylic acid has a net yield of 61 molecules of ATp. rf;em maybe the answer we want is 61 molecules of water:

6IADP+61P1+61H+

61ATP+61 Hzo

But the overall equation for the complete oxidation of one molecule of caprylic acid is given by:
CgH16O2

+ 1102 + 61 ADP +

61 P1 +61

H+

8CO2 + 8 H2O

r6lATP+6 1H2O

Looking at this equation you might be inclined to choose 69 moles of HzO (which would lead you to select choice l as the best answer). But be careful. To regenerate the ADP and Pithat is needed to resynthesize the ATp, ware: $ needed to hyftolyze that phosphoanhydride bond between the beta and gamma phosphates of ATP. What this meanw is that 6l of those 69 moles of water are continuously being recycled, so that new ATP molecules can be made. T c only water that is produced in a net quantity is the 8 moles from the complete oxidation of caprylic acid at the le',sL of the electron-transport chain. The correct choice is A.
42.

D is correct. Fatty acids are rather nonpolar; and because they are nonpolar, they can be stored in an anhydrous slajur (without water). The storage form of carbohydrates in animal tissue is glycogen. Recall that glycogen is compc,re,u of glucose residues. Glucose is a very polar molecule. It has many hydroxyl groups associated with it. Glucose end glycogen are both stored in a hydrated form. It turns out that on a gram-for-gram basis anhydrous fatty acids u 6n, metabolized yield approximately six times more energy than hydrated glycogen.
When molecules like fatty acids or carbohydrates are oxidized, energy is abstracted in the form of protons (Ho i tild electrons. Recall that these protons and electrons are dropped off at the electron-transport chain from redu;C coenzymes like FADHz and NADH (+ Ho). In other words, if a carbon atom has more hydrogens attached to it, rhrsn it has more electrons and protons to give to these reduced coenzymes, which in turn will have more to give to rn electron-transport chain. Note that the methylene carbons 1-CHz-) of the saturated fatty acids have t*o hydro:o atoms per carbon, whereas the majority of carbon atoms in a carbohydrate (like glucose) have just one hydrosm atom attached to each carbon atom. This tells us that the fatty acid is more reduced or, if you wish, the carbohydremr is more oxidized.

Copyright @ by The Berkeley Review

266

The Berkeley Revieu Specializing in MCAT Preparatior

Biology

Metabolic Pathways

Section VIII Answers

What about the hydroxyl hydrogens of a carbohydrate? If the hydroxyl oxygen were to lose a hydrogen, it would do so as a bare proton. The electrons stay with the oxygen atom of the functional group and give it a negative charge. Putting all of this together, it is clear that choice D is the best answer. The correct choice is D.
43.

D is correct. A decrease in the melting temperature of a fatty acid means that less heat is needed to disrupt the interaction between any two fatty acid constituents of that type when they are next to one another in a triacylglycerol molecule. In other words, the hydrocarbon chains of the fatty acids are not well ordered or well packed together. If this is the case, there are a lot fewer van der Waals interactions between the individual hydrocarbon chains. Fewer van der Waals interactions means that the carbon atoms in the hydrocarbon chain are farther apart. This could be because of cis double bonds in the hydrocarbon chain itself. Cis double bonds cause kinks to form in those hydrocarbon chains and tend to keep them apart from one another. The more kinks, the less chance there is for stronger van der Waals interactions. Adding kinks through cis bonds means that the carbon atoms participating in those cis bonds are unsaturated (i.e., they have fewer hydrogen atoms). What about unsaturation with trans double bonds? Fatty acid hydrocarbon chains can have trans double bonds, but they tend to allow the hydrocarbon chains to come closer to one another than cis double bonds. The van der Waals interactions are greater and the fluidity decreases (compared to the cis conformation). The conect choice is D.

44.

C is correct. There is much more skeletal muscle than liver tissue in a anyone's body. A 70-kg man has about 4O7o of his weight in the form of skeletal muscle and a liver weighing 1.8 kg. This means the muscle contains 0.40 x 70 x 14 = 4OO g glucose, while the liver has 1.8 x 44 - 80 g. The correct choice is C.

45.

C is correct. We can see that carbohydrate (CHO) oxidation increases from Day 3 to Day 4. The diet changes on Day 4 to high CHO intake. Choice A is true. We can see from the chart that the glycogen stores are beginning to fill before DNL kicks in. Choice B is correct. From the diagram, we see that net DNL occurs on Days 5-10. Choice D is correct. Since we are looking for the false choice, choose C. We can see from Figure 1 in the passage that glycogen stores are eventually saturated. The correct choice is C.

#.
17.

A is correct. Net DNL is occurring during the overfeeding of CHO. Since the liver packages the fat that it makes into VLDLs, we would see an increase in that class of compounds in the blood of test subjects on this high-CHO diet. The comect choice is A.
D is correct. In the question we are given the fact that glucose has a molecular formula of GHtzOe, and that it is completely oxidized to COz and HzO. If a biological molecule like glucose is being oxidized, molecular oxygen
(Oz) is being used. This allows us to set up
C6H12O6 C6H12O6
a

rough equation that needs to be balanced:


CO2

+ 02

H2O

Unbalanced equation Balanced equation

+ 602

6CO2

+ 6H2O

In the batanced equation we see that there are 6 COz molecules produced for every 6 Oz molecules consumed. Therefore, the ratio of COz to Oz is 6 to 6, or 1.00. The correct choice is D.
{8.

D is correct. If the hydroxyl (OH) group at the anomeric carbon is down (i.e., below the plane of the ring), the
position is alpha (cr). Note that in the diagram every anomeric carbon has the OH group below the plane of the ring. Therefore, the linkages for I and II are both in the o-position. This allows us to eliminate choices A and B. Linkage II begins at the C-1 carbon and ends at the C-4 carbon. This is an s-1,4 linkage. Linkage I begins at the C-1 carbon and ends at the C-6 carbon. This is an G-1,6 linkage. We can eliminate choice C. The correct choice is D.

{9.

A is correct. Both the stable and tadioactive forms of carbon can be used as tracers, so ignore the 13C and 14C tricks. Glucose can proceed to glycogen by the direct pathway. Alanine and pyruvate proceed to glycogen by
gluconeogenesis. Palmitate is a fatty acid, and it is not made into glucose. The correct choice is A.
1, we can see that the first few days of excess CHO feeding mainly increase the glycogen stores and not the fat stores. This eliminates choices B and D. The focus is on glycogen. Depletion of glycogen by exercise and a low-CHO diet, followed by a high-CHO diet might increase glycogen stores and thus increase the endurance of the athlete. The correct choice is A.

50.

A is correct. From Figure

Copyright @ by The Berkeley Review

267

The Berkeley Review Specializing in MCAT Preparation

Biology
51.

Metabolic Pathways

Section VtrI Answerc

B is correct. The important differences between eukaryotic and prokaryotic cells cannot be emphasized enoughEukaryotic cells have mitochondria, so they have a mitochondrial matrix. They also have cytoplasm, in which Sc mitochondria and Golgi complex reside. And they have an endoplasmic reticulum, extending from the cell's nucleus. The only item here common to the prokaryotic cell is the cytoplasm. Prokaryotic cells do not have mitochondril" Golgi, or an endoplasmic reticulum (because they do not have a nucleus). Therefore, in prokaryotes, glycolysir occurs in the cytoplasm. The correct choice is B.

52.

A is correct. Besides knowing the general differences between prokaryotes and eukaryotes, it is also good to k
some specific fact about each type of cell. Lactic acid is the end product of glycolysis under anaerobic conditi Under aerobic conditions, the end product of glycolysis is pyruvate (which can then enter the citric acid cycle). our cells were deprived of oxygen, then lactic acid would be the end product in each one of the four choi However, there is only one best answer here, not four. Therefore, we must assume that the question asks about cr that are utilizing oxygen. If oxygen is present, then glucose is completely oxidized to CO2 and HzO (by way glycolysis, the citric acid cycle, electron transport, and oxidative phosphorylation). But if lactate is being prod in a cell and is not being oxidized to COz and HzO in the presence of Oz, then either the citric acid cycle, ele, transport, and oxidative phosphorylation are not being utilized or those pathways are missing from the cell. It out that mature red blood cells lack not only a nucleus, but they lack membrane-bound organelles like mitoch< as well. The correct choice is A.

53.

D is correct. All we need to do is look at each step in the glycolytic pathway and see what general type of reactim catalyzed. There are two phosphorylation reactions, one at Step 7 and one at Step 10. Eliminate choice A. Therc a isomerization reactions at Step 2 and at Step 5. The only enolization reaction occurs at Step 9. Only ligaril reactions are left. Ligation reactions are catalyzed by enzymes called ligases. The word ligate means "to bind." h ligation reaction two molecules are joined together, at the expense of hydrolyzing ATP to ADP and Pi. The choice is D.

54.

Biscorrect. If weassignhydrogenavalueof +l andoxygenavalueof -2,theoxidationstateof carbonmusil whatever it takes to achieve charge neutrality. Consider the C-l (aldehyde) carbon. There is no change electronegativity between the carbon atoms C-l andC-2. However, the C-l carbon bears a hydrogen (+l) and oxygen (-2). This adds up to -1. Therefore, in order to be neuffal, the C-l carbon must have an oxidation state of
Consider theC-2 carbon. The hydrogen directly attached to the carbon brings a value of +1. The oxygen of hydroxyl group has a value of-2, and the hydrogen ofthe hydroxyl group has a value of +1. This adds up n Therefore, the C-2 carbon must have an oxidation state of 0. In fact, the oxidation state of the C-3, C-4, anO
carbons are each 0.

Consider the C-6 carbon atom. The two hydrogen atoms directly attached to the C-6 carbon bring a value of +2hydroxyl oxygen is again -2 and the hydroxyl hydrogen is again +1. This adds up to +1. In order to be neutral, 6 carbon must have an oxidation state of -1. Adding up all the oxidation states of the six carbon atoms gives a oxidation state of 0. The correct choice is B.
55.

A is correct. We need to consider the diagram outlining glycolysis. In particular, we need to focus our atte Step 7. The 2,3-BPG shunt bypasses the enzyme phosphoglycerate kinase, which at Step 7 in glycolysis 1,3-bisphosphoglycerate to 3-phosphoglycerate. In this reaction, ADP is converted to ATP; and since this
occurs twice per glucose molecule oxidized, we should generate 2 ATPs at this step. But we are bypassing this so we do not get those 2 ATPs. In fact, the only ATPs that are synthesized are those at Step 10 in the phosphoenolpyruvate to pyruvate. The 2 ATPs made here repay the 2 ATPs that we used as investments at and Step 3 in the first part of glycolysis. Therefore, the net production of ATP is zero (0). [You may wonder red blood cell can survive, if glycolysis yields zero net ATP. It turns out (which was not important for this q that about 257o of the glucose metabolized in the glycolytic pathways of the red blood cells is converted to 2, Recall that this molecule is used to stabilize the deoxy state of hemoglobin. The remainingT5Vo of the utilized by the glycolytic pathway can generate those much needed 2 net ATPs.l The correct choice is A.

56.

B is correct. If there is a mild deficiency in this enzyme, PEP cannot be converted to pyruvate as readilybegins to increase in concentration as do other glycolytic intermediates behind it. An increase in 3-phosphogly leads to an increase in both 1,3-bisphosphoglycerate and 2,3-bisphosphoglycerate. Recall from our discussi hemoglobin that 2,3-BPG stabilizes the deoxy form of hemoglobin and results in a decrease in the a hemoglobin for oxygen. A decrease in its affinity for oxygen means that more oxygen has been released

Copyright @ by The Berkeley Review

264

The Berkeley Specializing in MCAT

Biology

Metabolic Pathways

Section VIII Answers

tissue. The oxygen-hemoglobin dissociation curve would then shift to the right. Since the deficiency is mild (rather than extreme), the shift is only slightly to the right. Also, note that a deficiency in this enzyme leads to a decreased synthesis of ATP in red blood cells. The correct choice is B. 57.

C is correct. This can be worked out by using the Punnett square shown below. We can use the notation large p for the good gene and small p for the defective gene. If the trait is expressed, the genotype is pp. If it is not e>riressed the genotype is either PP, Pp, or pP.

P
p

Pp Pp
will

pp pp

p
There is a 50Vo chance that a child of this couple 58.

express the trait. The correct choice is C.

ordinary.

t*o ui St"p 6]two at Step 7, and rwo at Step 9. At Step 7, ADP is phosphorylated to make ATP. At rhe iame time, t,3-bisphosphoglycerate is dephosphorylated. But no reduction is involved in this step. This is the clue rhat something is a bil lut of the

B is-correct. The glycolytic pathway is not a one-way street. There are reversible reactions, as indicated by the equilibrium arrows in Figure 1 of the passage. The entire reaction series follows a sequence. The first event is a phosphorylation, followed by a dephosphorylation. If we study the glycolytic pathway from glucose to pyruvate, we
can,count eight 8 phosphorylation events. There is one at Step 1, one at Step 3,

What we are looking for is a phosphorylation step that is immediately followed by a dephosphorylation-andreduction step. The only oxidation-reduction in glycolysis occurs where we see the .o"nry*" NAD. If we examine glycolysis in the reverse direction starting with 3-phosphoglycerate, we will see that in order to form 1,3bisphosphoglycerate a phosphorylation event must occur. Even though no ATP is indicated to be going to ADp at the equilibrium arrows of Step 7 for this reverse reaction, it must be happening-otherwise, 3-pf,ospioglycerate could not be phosphorylaje.d to 1,3-bisphosphoglycerate. Notice that onci 1,3-bisphosphoglycerite is pro-duced, it can be dephosphorylated it (which means the loss of Pi) and reduced to glyciraldlnyle-:-pnosphate. This is indicated by the reverse of Step 6. The reverse of Step 5 is an isomerization reaction that converts glyceraldehyde-3phosphate to DHAP (see the structure below).
cH,-g-p9.:-

Dihydroxyacetone
phosphate

eo
I I

cH2-oH

These three teversed steps in glycolysis do occur, and they are part of the pathway called gluconeogenesis, the synthesis of $ucose from specific precursor molecules. The correct choice is B.

59.

B is correct. Many of the L-amino acids can lose their s-amino group in a transamination reaction. This reaction is carried out by enzymes called transaminases or aminotransferases.In this reaction, the g-amino group of leucine is transferred to o-ketoglutarate (a component of the Krebs Cycle), an o(-keto acid. This generatei L-glutamate and another cr-keto acid, which in this case is cr-ketoisocaproate.
coo

o
H3N-C+
I

o=c

?oo
H

coo

CH: tCH, l-6

..

H1N-C-H -l
Aminotransferase
CH, CH,

@t

coo

coo

o=c

cH-

HjC-

t'

CH
I

CH,

coo
Glutamate

t' t-g

*
HlC-l

cH,

t-

CH

CH:

(I-Ketoglutarate

Leucine

0-Ketoisocaproate

r;ight

by The Berkeley Review

269

The Berkeley Review Specializing in MCAT Preparation

Biology

Metabolic Pathways

Section VIII Answers

Note that there is no net deamination in this reaction sequence. Transferring the amino group to glutamate allows glutamate either to use that amino group in biosynthetic reactions or to eliminate that amino group (via the urea cycle) as a waste product. We can eliminate a hydration reaction as a possible answer choice, because we are not adding water to leucine. Thc

water molecule HzO has not been added (does not appear) on s-ketoisocaporate. Also, we are not adding a CQ residue by carboxylation. nor are we removing hydrogens by dehydrogenation in this reaction. The correct choic
is B. 60.

B is correct. This question asks you to remember how the human body disposes of nitrogen as a waste produaAmmonia is quite toxic to animals. Since the pKs of ammonia is about 9.5, we would, at physiological pH, expcr m find this molecule present as the ammonium ion. The amino nitrogen is excreted as the ammonium ion by aquatic organisms. These animals are referred to as being ammonotelic. Reptiles and birds excrete the ami nitrogen as uric acid, and are therefore referred to as being uricotelic. Human beings and many other terrestri animals excrete the amino nitrogen as urea, and because of this are referred to as being ureotelic. Recall that urea produced in the urea cycle. Glutamate is simply an intermediate in the pathway for nitrogen elimination in The correct choice is B. B is correct. Adding HzO to the reaction would either hydrate the reactant or hydrolyze it. Neither of those

6t.

is occurring. NADH and FADHz are both reduced coenzymes. Adding either to the reactant means
hydrogens across a carbon-oxygen double bond or across a carbon-carbon double bond, respectively. Again, we not see either case here. This leaves, by the process of elimination, ATP as our choice.

S-CoA

o=c
I

HCOj

S-CoA

o=c
I

C_H
il HlCI

C-H ll
H3C'l
C

(A biotin-dependent carboxylase enzyme)

CH.

CH:

1"6 coo

p-Methylcrotonyl-CoA

p-Methylglutaconyl-CoA

COz is a gas that tends to diffuse away. Trapping that COz and adding it to the reactant (via the biotin-dependem enzyme), requires energy in the form of ATP. Carboxylation reactions involve energy. The correct choice is B-

62,

C is correct. If HCO3o reacts with the reactant to add COz, then an OH moiety is left over. A hydrogen (Ht one of the methyl groups of the reactant can combine with this OH group to produce HzO. Since there is no group on the reactant, none can be formed in the products. Similarly, since there is no COz group on the decarboxylation reaction will not yield COz in the products. Finally, the addition of a carboxyl group to the
(via HCOf9), does not form H2CO3 (carbonic acid). The correct choice is C.

63.

C is correct. The enzymes mentioned in the question are proteins. The prefix apo- rcfers to the protein the enzyme, while the prefix holo- refers to the enzyme having all of its necessary parts (cofactors, prosthetic groups, etc.) in order to function. Holocarboxylase synthetase attaches biotin to a specific enzyme. This makes the apocarboxylase enzyme complete (i.e., we could now call it a holoenzyme itselfl and to carry out a carboxylation reaction. If the holocarboxylase synthetase enzyme is deficient, biotin attached to the apoenzyme, and the reaction does not occur. This means that the substrate that accumulatcs rapidly is the reactant for this reaction, which is p-methylcrotonyl-CoA.
Neither acetyl-CoA nor p-methylglutaconyl-CoA can be formed, since we cannot get past the reaction Enzyme 4. Therefore, they do not accumulate. cr-Ketoisocaproate does begin to accumulate, but not as r methylcrotonyl-CoA. The correct choice is C.

64.

C is correct. If this reaction were a thiolysis, we would need to add the element of sulfur (S) across the was broken. Similarly, if it were a hydrolysis, we would need to add a molecule of water (HzO) across the was broken. We do not see this in either case, even though we had a lysis in both situations.

Copyright @ by The Berkeley Review

270

Biology

Metabolic Pathways

Section VIII Answers

The product of an aldol condensation is a molecule with both an alcohol and an aldehyde functional group. These two groups are the source of the term -aldol, and these are exactly what we would have if we removed the CoA-S portion of B-hydroxy-p-methylglutaryl-coA and added a hydrogen atom. The mechanism for this is shown below:

ENZ-Base:

H2c-

fl

c-

S-CoA

o Oil H2C-C-

() o S-CoA

H)C-C-

S-CoA

ENZ-Base

Acetvl-CoA

Enolate ion

( o /'----- -o fl oOOC- CH2 xtt)/ C CHr H,C- CAcetoacetate

S-CoA

alil

o oo
cH,

"ooc-

-c -

t-

cH,

- C-

S-coA

CH,

An alkoxide ion

o oOC-

CH2

-l- a"r-C- S-CoA H:


cH,

"/---\ fi ?

\.,

Base-ENZ

otil -

oHO
CH2

OOC-

- 9I

CHz

-C - S_CoA + :Base-ENZ

CH..

An alkoxide ion

B-Hydroxy-B-methylgluraryt-CoA

However' it is evident the reaction outlined in Figure aldol condensation. The correct choice is C.

I in the passage is proceeding

in the direction of a reverse

65'

B is correct' Glucose. enters the glycolytic pathway and is converted to pyruvate in a series of ten reactions. Pyruvate can react with coenzyme A (coA) in a decirboxylation reaction to glve u""tt-coe, which is one of the end products of the degradation of leucine outlined in Figure 1. once acetyl-ioA is fo'rmed, it can enter the Krebs cycle and be completely oxidized to coz and Hzo. But acetyl-CoA cinnot d;ur;i enrer glycolysis, electron transport, or oxidative phosphorylation. we do find that the carbon atoms of acetyl-coA can end up in the glycolytic intermediates through the process of gluconeogenesis. Be aware,th_at acetyl-CoA cannot be used as.a pr"cu.sbr to yield a net synthesis ofglucose in mammals' Recall that acetyl-CoA is a two-carbon compound. Whln it enters the Krebs cycle,2 carbon atoms are lost as Co2; yet those 2 carbon atoms are not necessariiy the same 2 carbon atoms derived fiom acetyl-CoA. This means that if we were to label acetyl-CoA, we would find some of.it in the glycolytic intermediates. s:re just would not be able to get a net conversion ofa two-carbon compound to a six-carborisugar.
The energy tied up in the bonds of acetyl-CoA can be transferred (via the Krebs cycle) to the coenzymes NADH and FADHz' These coenzymes can then transfer this energy, in the form of electrons and 'hydrogen ions, to tr,e eLcirontransport etrain and eventually to the process of oxidative phosphorylation, where eip is generated. The correct

choice is B.

66.

B is correct. The normal digestion of trehalose would not lead to the production of hydrogen, so choice A is incorrect' No hydrogen is given off from interactions with bicarbonate or with gastric .ont"ntr. Choices C and D are therefore incorrect. The reason for.increased hydrogen is the passage ofundigeJed trehalose from the small intestine into the colon, where many bacteria begin to metabolize it. ttrey iroduce thE hydrogen uy i".menting the trehalose. The correct choice is B.

67.

c is correct' Exhibiting a dose-response effect means that every time the dose is increased by a certain increment, the subject's response follows a similar patterns. ofincreases. suUlect 3 has an increase of 10 ppm of hydrogen for every l0 g iribrease in trehalose, so choici C looks like the best answer. Subject 2 shows no dose-response effect, so choice B is incorrect. Subject I shows no particular pattern, either. Choic" e i, in"orr."i SrU:*t 4 did not complete the study. Choice D is incorrect. The correct choice is C.
f)
is correct.Figure

68.

choice is D.

shows us that trehalose is made of two glucose residues joined

in cr-l-l

linkage. The correct

--rpyright

by The Berkeley Review

271

The Berkeley Review Specializing in MCAT preparation

Biology
69.

Metabolic Pathways

Section VIII Answers

D is correct. This is one of the few times that "I can't tell from this passage" is the correct answer. Although is a disaccharide, this does not mean it behaves in the body exactly as trehalose does. Choice A is incorrect. may or may not be a commoner component of the diet, depending on what culture you are examining. But you can't predict someone's lactose tolerance based solely on their degree of tolerance for another sugar. Choices C B are incorrect. The correct choice is D.

70.

A is correct. The enzyme trehalase is found in the brush border of the human duodenal mucosa. It may be present large or small quantities, depending on one's genetics. It is not induced by eating trehalose like the lac operon induced by lactose in bacteria. A person's tolerance for trehalose is not the result of their habitual consumption trehalose-containing foods. Choices C and D are therefore incorrect. Subject I has almost no increase in h
upon ingesting increasing doses of trehalose. This means the digestion of this sugar by trehalase is more effici The subject's tolerance for doses of trehalose is high. The correct choice is A.

7t-

separates the component parts of a mixture by density, so choice A is incorrect. A spectrometer characterizes compounds based on masses of isotopes, so choice C is incorrect. Extraction compounds based on their solubility in organic solvents, making choice D incorrect. The correct choice is B.

B is correct. A centrifuge

72.

B is correct. Up to a certain dose, Subject 2 does little bacterial fermentation of trehalose. But after that point, a threshold, their enzymes can handle no more, and there is a suddenly big increase in the amount of h in their breath. The correct choice is B.

73.

A is correct. In the initial phases of exercise, the oxygen consumption is not high sufficient to provide oxygen to the body's muscle tissues for the oxidation of fuel molecules through the TCA cycle and ox phosphorylation. During this brief period of lowered oxygen relative to demand, only glycolysis is carried out TCA cycle enzymes are constitutively on; they do not need to be activated by exercise. Thus, choice B is i Fatty acids are always metabolized aerobically. They cannot enter glycolysis, which is the only anaerobic oxidizing pathway. Choice C is also incorrect. Glycolysis can keep pace with the energy demands of intense exercise, since this is what occurs. Choice D is incorrect. The correct choice is A.
C is correct. The maintenance of glucose homeostasis has many complex and interacting safety checks. Sina brain and nervous system are both almost entirely dependent on glucose as their fuel source, blood glucose maintained within a fairly narrow range for all times, during a variety of activities. This may be accompli through the intake of dietary glucose, through the use of glycogen stores, and through gluconeogenesis. Choi B, and D are incorrect. The correct choice is C.

74.

75,

A is correct. Oleate is a fatty acid, which makes up abofi


see

40Vo of the circulating fatty acids. Resting muscle, in Table 1, uses FFAs as fuel primarily. Glycogen is not used at rest, so choice D is incorrect. p-hydroxy acid is a ketone, used during fasting and very strenuous, sustained execise. Choice B is also incorrect. Lactic a waste product of metabolism in the muscle. It is produced during exercise and exported to the liver for gluconeogenesis. This ,means choice C is incorrect, too. The correct choice is A.

76.

D is correct. Since glucose is a small, water-soluble molecule, it does not need a carrier molecule. Usually, proteins such as albumin serve as carriers for large, insoluble molecules. Choice A is incorrect. There is no
binding protein, so choice B is incorrect. And hemoglobin carries oxygen. Choice C is also incorrect. The choice is D.

77.

B is correct. Insulin synthesis is a response to dietary carbohydrate entering the blood. Its function is to glucose transfer from the blood into cells and also into the glycogen stores. Insulin levels in the blood are low exercise, so choice A is incorrect. Glycogen phosphorylase is an enzyme involved in glucose maintenanc" hormone. Choice C is incorrect. Oxytocin is a hormone involved in labor and childbirth, so choice D is i too. Glucagon promotes glycogenolysis, so that muscle can use the available glucose for fuel and the liver can down and release stored glucose (glycogen) to the blood supply. The correct choice is B. provide about half the energy per gram that lipids do, so choices B and C incorrect. Proteins, or amino acids provide about the same amount of energy value per gram as ca Choice D is incorrect. The correct choice is A.

78. A is correctlCarbohydrates
79.

ac

D is correct. Muscle glycogen contributes none of the energy demanded by the body during the first four
exercise. Choice

A is false. Gluconeogenesis increases during exercise, as you can see from Table

1. Therc

Copyright

by The Berkeley Review

272

The Berkeley Specializing in MCAT

Biology

Metabolic Pathways

Section VIII Answers

reason to expect it to decrease after four hours. Choice B is incorrect. Adipose tissue has about 100,000 kcal of stored energy in the form triglyceride. It is released as free fatty acids and glycerol into the blood. There would be no significant change in available fat stores for exercise that continues after four hours. Choice C is incorrect. Liver glycogen's contribution to blood glucose does drop over the four-hour exercise period, however, and that decrease would probably continue. The conect choice is D.

80.

C is correct. Glutamine is the major form of ammonia transport and is carried in the blood to the liver. The anitrogen is then removed as ammonia in the liver mitochondria. At a neutral pH, most of the ammonia that is released is in the form of the ammonium ion (NH+@). This toxic ion is converted to urea in the urea cycle and later exported to the blood from the liver. From there, it travels to the kidneys and eventually eliminated in the urine. The correct choice is C.

81.

A is correct. Urea contains two nitrogen atoms. One comes from the condensation of NFI+CI with HCOre and the other comes from aspartate. In order to place urea on the product side of the equation, there must be a balance of 2 nitrogens on the reactant side. This allows us immediately to eliminate choices C and D. We can eliminate choice B also, because if we add aspartate on the reactant's side of the equation, would have 3 nitrogen atoms on that side of the equation and only 2 nitrogen atoms on the product's side of the equation. The correct choice is A.
D is correct. We know one of the nitrogen atoms comes from the ammonium ion. As shown in the diagram of the urea cycle, the ammonium ion nitrogen is found in the structures of carbamoyl phosphate, citrulline, argininosuccinate, and then arginine before it gets to urea. Where does the other nitrogen atom come from? The only other reaction entering the urea cycle comes from aspartate, which is formed from the reaction of oxaloacetate and glutamate. Glutamate passes its nitrogen to oxaloacetate and in the process is converted to a-ketoglutarate and
aspartate. There are no nitrogen atoms in oxaloacetate or in the citric acid cycle. The correct choice is D.

82.

E3.

B is correct. Carbamoyl phosphate is formed from the reaction of the ammonium ion with bicarbonate (HCO3o).
The bicarbonate ion is carbon dioxide (COz) in disguise. The vast majority of metabolic COz comes from the citric acid cycle (Krebs cycle). Carbon dioxide is not produced in glycolysis, from the electron-transport chain. or in the poxidation of fatty acids. The correct choice is B.

&{.

C is correct. The only two choices we have that allow a stable reaction with the ammonium ion are choice B (aspartate) and choice C (glutamate). The ammonium ion reacts at the side chain carboxyl group to form the amide. The more favorable reaction is the one with glutamate, because its side chain carboxyl group is farther away from the positively charged cr-amino group. There is less steric repulsion. Once the ammonium ion reacts with glutamate, the amino acid glutamine is formed. Glutamine is a neutral, nontoxic compound. It can readily pass through cell membranes and eventually into the blood, where it is carried to the liver for removal of the nitrogens. The correct
choice is C.

Li.

A is correct. We need to be careful of words here. The word amidationrcfers to the introduction of an amino group into an organic compound. However, we want to remove an amino group from an amino acid. Therefore, the reaction is a deamination. This points to choice A and choice C. A transamination reaction involves the transfer of the amide nit?ogen (from glutamine) to another compound, so eliminate choice B. A dehydrogenation reaction involves the removal of hydrogens (not nitrogens) from an organic compound. Eliminate choice D. Is the removal of the amide nitrogen an oxidative or reductive deamination? In the passage, we see the conversion of glutamate to oketoglutarate. The removal of the amide there results in a carbonyl group at the s-carbon. This is an oxidation
reaction. The correct choice is A.

ffi.

B is comect. Draw a pedigree as shown below. This indicates how a gene is transmitted from one generation to the next. Let's represent the disorder by aa.

-II
III

""ii:'ii';,",:';il:;""'D "l "" |


273

^FI;] *
|

The Berkeley Review Specializing in MCAT Preparation

Biology

Metabolic Pathways

Section VIII Answers

Individuals who do not express the trait are represented by either AA, Aa, or aA. The man's mother had ASA, so trer genotype must be aa. His father did not carry the gene, so his genotype is AA. This means that the man's genotype ir either Aa ot aA.It does not matter which he is, because he still doli not express the trait. Now, the man mai.i., e woman who has the disorder, so her genotype must be aa.What is the probability that their child will be affected! Since we do not know the sex of the child, we represent it by a diamond in the pedigree. A simple punnett squarg tells us that the probability the child will express the trait is 507o. The correct ctroice is B.

87.

and a reduced production

to urea. However, in the reaction drawn in the question, glycine is reacting with sodium benzoate, not asparmn Based on this mechanism, we would not have a way to increase the synthesis of aspartate. Eliminate choice B. \||l cannot increase the concentration ofurea, because th^ere is a deficiency in the arginaie enzyme that converts argi to ornithine and urea. Eliminate choice C. Again, if we were to increase the Krebs cycle intermediates to prl more aspartate, and if aspartate were to drop off its nitrogens at the urea cycle, there w-ould still be a lack of argi

A is correct. By increasing the synthesis of glycine, we are pulling more of the ammonium ion out of solutim* thereby decreasing its concentration. If we increase the syntheiis of aspartate (by some mechanism), thc concentration of ammonia will begin to fall, because we are passing those nitrogens to tire urea cycle and eveniua$p

increase (hyperammonemia). The best way to eliminate nitrogen is by some other route. The correct choice is A

of urea. The intermediates would back up, and the concentration of ammonia *,

88.

C is correct. Think of

in the.electron-transport chain. However, this metabolic detour through lactate is necessary during anae conditions. The purpose of gluconeogenesis is to maintain constant blood glucose levels, especially duriig vig exercise and during a fast. Following a meal, gluco_se is provided by the diet, so choice D is incorrect. D;ing or studying, little anaerobic activity is happening. Choices A and B are inconect. Exercise involves rapid gl to produce energy. vigorous exercise is anaerobic. The correct choice is c.
89.

regeneration of NAD in the muscle tissue. Lactate is an end product. When lactate is formEd from pyruvate, N-.: is oxidized to NAD@. This NADo is free to reenter glycolysis. under aerobic condirions, NADH would be oxi

gluconeogenesis as having two roles: maintenance

of blood glucose by the liver

Finally, there is no lactate carboxykinase, so choice D is false. The correct choice is B


90.

B is correct. Pyruvate is first carboxylated to make oxaloacetate (OAA). This is the first step in glucone Pyruvate dehydrogenase is involved in the conversion of pyruvate to acetyl-CoA and COz. ihoice .q is i Lactate dehydrogenase is the enzyme the uses NAD(H) to interconvert pyruvate and lactate. Choice C is i

D is correct. Pyruvate is converted to lactate, not the reverse, during anaerobic conditions, so statemen[ incorrect. Lactate is not converted to alanine, but pyruvate is, during anaerobic conditions. Statement II is inc orrec L ac rare i#;", ;" i,T :i : lffi;TJiJl"in"# " III is inconect. The correct choice alphabetically. Statement "iJT"}:::# 5 is D.

i'

d;';:;".;;#;;;il:

#i'

91.

D is correct. So, you've never heard of the reverse Cori cycle? Pay attention to your intuition, because in this you to thecorrect answer..Ther"_o las go1lt1ng lo reverse Cori cycle. Muscle tissue neverreleases glucose i blood. It does not have the enzymes to do so. Choices A, B, and C are incorrect. The correct choice is D.

92.

A is comect. NAD@ is required in glycolysis by the enzyme glyceraldehyde-3-phosphate

this cofactor, the enzyme could not function, and glycolysis would stop. Ctycotysis produces a net lain of 1 so it would not stop due to a lack of ATP. Choice C is incorrect. Since glycolysis is a fixed pathway of enzy cannot change its ATP output or its cofactor requirements. Choices B and D are incorrect. The correct choicr
93.

dehydrogenase. q.'
,

A is correct. Since the body produces alanine, by definition it must be nonessential. Eliminate choices B

Since the amino group is what distinguishes pyruvate from alanine, reduction is not the metabolic ormr interconverts them. It is transamination. Choice D is incorrect. The correct choice is A.

94.

B is correct. This is an anatomy question. Usually an artery flows into an organ, and a vein flows out of the However, the hepatic portal vein (choice C), is a specialized venous system that carries nutrients absorbed fr
. is inro rhe liver for rhis particutar system. This was mentioned in the passage so choice C could be easily eliminated as a possible correcr There is no hepatic portal artery, so choice D is wrong. Choice A is aiso incorrect, because the hepatic a bringing blood to the liver. The hepatic vein drains the liver and carries glucose and other nutrients releasod liver to the rest of the body, making choice B the best answer. The correct choice is B.

i;:;ilJir*d;;;;d;*

Copyright

by The Berkeley Review

274

The Berkeley Specializing in MCAT

Biology
95. B is correct. Carbohydrate

Metabolic Pathways

Section VIII Answers

generated by bacteria in the colon as they break down otherwise nondigestible carbohydrates, like lactulose. Since her breath hydrogen did not rise, her bacteria were not of the type that ferments lactulose. A change in breath hydrogen was the measured variable in the study, so she could not participate. Choice A is incorrect. The correct choice is B.

that can be digested by the body's own enzymes (i.e., digestible carbohydrate) is digested in the small intestine and does not reach the bacteria in the colon. Choices C and D are incorrect. Hydrogen

in the breath during the lactulose test is

96.

B is correct. The points for the breath hydrogen produced following the starch blocker are not statistically different from the placebo points. Statement I is correct. The rise in breath hydrogen following lactulose administration was after the l2O-minute mark. Statement II is incorrect. Starch in the small intestine is digested by o-amylase. If this enzyme does not work, then the starch passes to the colon for fermentation by bacteria. This fermentation gives off hydrogen, which is measured in this experiment. In people who have enough bacteria in their colon to ferment lactulose, there are only two possible outcomes: either c-amylase worked, and no extra hydrogen was produced, or c-amylase did not work, and the colon bacteria produced measurable hydrogen. Since there is no difference in the
amount of hydrogen gas subjects produced in the breath between a meal with a placebo and a meal with a starch blocker, we can assume that the amylase was not inhibited. Statement III is correct. The correct choice is B.

97. 98. 99.

D is correct. The digestive enzymes trypsin, chymotrypsin, elastin, amylase, and lipase (among others) are produced by the exocrine portion of the pancreas. They are secreted into the pancreatic duct and then empty into the small intestine. Choices A, B, and C are incorrect. The correct choice is D. C is correct. The figures show no significant differences between the placebo and starch blocker tests for insulin and glucose levels. There was essentially no change due to the administration of the starch blocker. It did nothing to inhibit starch digestion. Choices A, B, and D are incorrect. The correct choice is C.

B is correct. Starch is a plant's storage form of carbohydrate. Turkey is not a plant, and its storage form of carbohydrate is glycogen, just like other animals. Choice A is incorrect. Butter is mainly fat, not carbohydrate. Choice C is incorrect. Orange juice contains carbohydrate, but predominantly in the form of simple sugars. Choice D is incorrect. The correct choice is B.
sugars are the monosaccharides, and they are absorbed by the body intact, without any digestion. Amylase is not needed for their digestion. The inhibitor has no secondary effects that destroy simple sugars. Even if a real starch blocker existed and blocked amylase in vivo, simple sugars would not be affected by it. Eliminate choices B, C, and D. The correct choice is A.

100. A is correct. Simple

i.:pyright

by The Berkeley Review

275

The Berkeley Review Specializing in MCAT Preparation

$t$r6
Sctid,n',,,,,IX
Genetic
',1ntu*mation 155
l51.11

A.

Classical Genetics

l.

Patterns of Inheritance
:t

. :i , , '5.r

:,Gene.tiC LOei

&,,j{1191*,$.,,,,,..1,',.1

B. C.

4. l. 2. t. 2.

The Pedigree

Genetic Information
Central Dogma of Molecular Biology

Function of Nucleic Acids

'"'**
l51q l4p

DNA Synthesis
DNA Polymerase DNA Replication Er
..

,'*-'

:5

/lil{ ;;:lJ',, /:il| (' il ;i*;"; ill lil


14111

l{1q,1,19, .,

,,

Practice Passages

Answers
:. .
:r: r1:r:: i::r: I r r:j,i.r:t:r
i. I

,, I' 14N,14N,.

':

1419,, 1419"

;::,,r[,[.

:'.

"

'',' ,taN,,15

iri,

r,

,'1[

14Nlr4X I l5Nll4N ' :';',lgqo ,' , Savo ,

Direction of Sedimentatioa'

r:*

Specializing'in' MCAT Preparation

PEffi L)n.n.v.r.-ilff

tfre

Gentic Information
Top lO Section Goals
Ee famitiar,r,yith the transmission of genetic traits. Understanf how genetic traits are transmitted from one generation to the next. Think of meiosis in this context, especially genetic recombination.

the'work of Mondel. uw Be familiarwitlr Gregor Mendel^and his garden Pioneering wolk


Trace the

lld
1lt

square, and De able to obtarn srmple ratros of genetic events.

9f

peas. Understand how to

,rr"l

prrn.r"tt

tS

ttfi

r{m

rffiil

frh

Don't get lost in the details of difficult pedigrees. At most,_be able to take a pedigree back to three r o generations. Be able to follow the lineages oi the individuals.

mr

ffit

]WM

ih

tffiun
r@U'

k
Eq familiar 4th the gener{_Irrocess of DNA replication. Don,tget1ostinalltheenzymesinvolvedin.DNAreplication.J"' the overall process, and feel comfortable with some of the names and structurbs.

\me

TWW

tu
fomr

Mru,u

Mm

UB
@B

Understand,how tlre

chain reaction works.


ter\axr \obe on \heMCN\.

iffilum'

The discovery of the polymerase chain reaction ranks among the greatest scientific air.ouetler of

'uffiii$

\\e1$\\trn\tny.\\ert\astbrtlriurrl ar\rt\esun\h\spru,e{s,an{\\rs
t-

Wuil'\l ,lwlitl
,,&i,rtlru

Be able to relate the discussions of this section with genetic expression. Besides DNA replicatiory there is also DNA transcription and mRNA translation. Once a cell divides, it must synthesize proteins in order to survive. You will need to understand this process.

{i!*'-

Mrru'

,&:r
flfF*

Biology

Genetic Information

Patterns of Inheritance

P',

#iiiiufriffiffi

6e

cross breeding different species. For example, the minotaur from Cretan mythology was a creature with the torso and head of a human and the body of a bull. The giraffe of the African plains was thought to be a cross between a leopard and a camel. This is even reflected in the scientific classification testablished by Carl Linnaeus) of the giraffe; Giraffa camelopardalls. By the time the Middle Ages arrived it was believed that crosses were viable only within a particular species.

Let's begin a discussion on patterns of inheritance. There were many early ideas about heredity. It was widely believed that strange creatures could be breed by

cne of the early models proposed for heredity was that of pangenesis. This -leory stated that each part of the body produced tiny particles called pangenes or gemmules. Pangenes were though to be miniature replicas of each organ or ].sue of the body. These pangenes were carried to the reproductive organs by he circulatory system where they were packaged into the sperm or the egg. During fertilization the male and female pangenes united in the femal"'s rro-b. Er-entually a new organism would result. If a pangene was healthy or defective, :: l'ould be passed on to the offspring. The combining of pangenes led to the idea :f "blending inheritance." This simply meant that the individual was a mixture of
--t'le

ris notion of blending. This was the prevailing


1S68.

two parents. The male pangenes were thought to be the major contributors to theory up to the end of the last :entury and it is interesting to note that it was proposed by Charles Darwin in

in the future all of the offspring should resemble one another. We r.ow that this is not the case. Flowever, it seemed to present a paradox for the :,eory proposed by Darwin. In 1760 Josef Koelreuter, a German botanist, was
'"-me point

-: the male and female contributions to the offspring resulted in blending, then at

:ossing different species of the tobacco plant. The offspring that were produced ;,' ere fertile and they were able to produce a new generation of plant which was ::*tly variable. A few members of this new generation resembled the original srecies of tobaccb plant. Even though these results were not in agreement with ::,e ideas of blending inheritance they did provide clues for the mechanism of :e:edity.
i+":elreuter's work was taken seriously by many investigators over the next :rundred years and in the 1790s an investigator named T. A. Knight crossed two :rrae-breeding lines of the garden pea (Pisum satiaum). Within the pea plant

:[rrver are the male (anther--produces pollen) and female (stigma-produces *:gs) sexual organs. A true-breeding plant is one that if left to itself will self;q:llinate and always produce the same kind of plant. One true-breeding plant ll,':-.r-ld always produce purple flowers while the other true-breeding line would ilL'-,q dvs produce wlute flowers. These plants were designated as the parental (p) #.:,eration. When the pollen from a purple flower was sprinkled on the eggs r::1a white flower, only purple flowers were produced in the first filial (F1) Fneration (Figure 9-1).

:;rq\nght

by The Berkeley Review

279

The Berkeley Review Specializing in MCAT Preparation

Biology
P

Genetic Information
Purple Flowers

Patterns of Inheritance

l
{
G

Fl F2
Figure 9- I

I Purple

X +

White Flowers

Flowers

Purple

Flowers

:.:

White Flowers

-. -6

The purple flowers of the F1 generation were allowed to self-pollinate. In the next generation, the second filial (F2) generation, both purple and *hite flo.,r,ex appeared. This says that some characteristic trait was hidden (masked) in the Flt generation but reappears in the F2 generation. Knight's comments on tl:_o phenomenon were that there were more purple flowers in the F2 generation tha:, while flowers. He left it at that.

+t

;-_,
-.:?

-Ar

pl,li:
l--,i-

s-glgi

!L

$[=rat uE:;

nu:-:ch

fu'{r,de able

:--en I

:(ffi

-^ )

"- J^l :--qC_[

ro 'J

+i-

::l oi tl

iener
ro .r tlL +-l

iital,
:.:ifera

n-atir

each c :--5 tha

:i
Copyright @ by The Berkeley Review

lreq

2AO

The Berkeley Specializing in MCAT

right

Biology

Genetic Information

Gregor Mendel

fu', ildl
have given him a teaching certificate. while at the monastery he carried out examinations on the common garden pea. His worked marked the beginning of modern genetics.
\A/hy did Mendel choose the garden pea? There were a number of reasons. (L)

Gregor Mendel was born in Austria tr.1g22. He entered a monastery in Brtinn and received a formal education. He later attended the University of Vienna and after two years returned to the monastery because he failed the exams that would

available for use in experiments. (3) Mendel choose 32 of the mariy different true_ breeding pea plants of which to work with. From these 32 varieties he choose lines that could be distinguishedby z different paired traits (e.g., alternative characteristics like purple versus white, smooth ,rets.r, wrinkled-, etc.) (4) pea plants were rather easy_ to grow, they have a short generation time, and they are small and would not take up much space in the garJen.

A with the garden pea. Many e_arlier investigators (e.g., Knight) had produced hybrid peas by crossing different true-breeding lines. (2) Many different true-breeding varieties weri
great deal of work had already been carried out

As we have mentioned, both the male and female sex organs are contained. within the flower of the pea plant. If the pea plant is left undiiturbed, it will selfpollinate. Flowever, if the anthers are removed before pollination and pollen is introduced from another pea plant, then cross-fertiliiation will result. The fertilized eggs in the stigma develop into the embryo (the seeds). Each is the product of a separate fertilization. The pod that contains the seeds has the characteristics of the parents while the seeds themselves belong to the next
generation.

Dominant vs

Trait
Flower Color Seed Color
Seed Shape Pod Color Pod Shape

Recessive
Purple vs White

Yellow vs Green
Round vs Wrinkled Green vs Yellow Round vs Constrictec

Flower Position Plant Height

Axial vs Top Tall vs Dwarf

Table 9.1 Mendel choose 7 traits that were alternatively expressed for 7 characteristics of the pea plant. Those 7 traits are shown in Table g-t. Note which is dominant and
rvhich is recessive.

Mendel choose parent plants that had breed true for many generations. He then performed crosses by cross-fertilization for each of the 7 pairs of traits shown in Table 9-1. For example, from the purple flower he used the pollen and from the ''vhite flower he used eggs. Mendel also performed the reciprocal crosses (e.g., rollen from the whTte flower and eggs from the purple flower). These pian"ts :epresented the parental or P generation. The offspring of each of these z crosses, :eferred to as the first filial or FL generation, expressed one of the two parental
:haracteristics.

\{endel allowed the F1 pea plants to self-pollinate for one generation. He then .cored the characteristics of the second filial or F2 generation. Mendel found that .ach of the 7 characteristics reappeared in the F2 generation. For example, in the F2 generation round and wrinkled seeds were observed as were purple and .rhite flowers. This observation did not agree with the notion of biending rheritance that we previously mentioned. In contrast to the earlier experimental :,bservations that Knight made on the garden peas, Mendel counted the number :f different types of plants in the F2 generation.
- or each of the 7 traits in the F2 generation, Mendel found close to a 3 to 1 ratio of :lants that expressed one parental trait compared to the other parental trait. The :rost frequently expressed trait was always the one that was exclusively

-opyright @ by The Berkeley Review

281

The Berkeley Review Specializing in MCAT Preparation

Biology

Genetic Information
expressed

Gregor

in the F1 generation. Mendel proposed that the trait that

expressed in the F1 generation was dominant while the other unexpressed was recessive. In other words, if a purple flower was crossed with a white and the F1 generation gave all purple flowers, then those purple flowers dominant over the white flowers. Even though the white flower trait is in the F1 generation it reappears in the F2 generation.

Mendel proposed that the parent plants do not transmit their physiological transmit "hereditary factors" which act later in the offspring to produce the Today we call these hereditary factors "genes."

or forms directly to their offspring (i.e., there are no pangenes) but

ra

Purple P"gP White Purple White White


Figure 9.2

Each individual possesses 2 factors with respect to each trait. For example, can be one factor for purple color and another factor for purple color, one

for purple color and another factor for white color, or one factor for white and another factor for white color (see Figure 9-2). One of these two factors contributed by each parent to the offspring.
The alternative forms of the factors determining a given trait are called The flower color trait has a purple allele and a white allele. An individua-l have two different alleles or two identical alleles. If the alleles are different, the individual offspring is referred to as being heterozygous. If the two are identical, the individual offspring is referred to as being homozygous-

example,

in Figure 9-2 the Purple/Purple alleles would be


as

con

homozygous

would the White/\4/hite alleles. The Purple/\Alhite alleles

be heterozygous.

The two alleles contributed by the parents to an offspring do not influence other. For example, if an individual pea plant has a purple and a while then those alleles will stay purple and white. They will not form in alleles as expected from the blending inheritance hypothesis.

Let's consider one of Mendel's experiments on flower color. In this ex two true-breeding pea plants are crossed. The plant with the purple (dominant) is represented by an upper case W while the plant with the flowers (recessive) is represented by a lower case zo. [The letter "W" is chosen from the recessive allele.] The purple flower can only produce W while the white flower can only produce zo gametes. We can express this a Punnett square as shown in Figure 9-3. Note that four heterozygotes produced. However, since W is dominant oyer u) | we see that the F1 yields all purple flowers.

In the next step Mendel allowed the Fl generation to self-pollinate. In Punnett square in Figure 9-3 we find two homozygous individuals (WW ww) and two heterozygous individuals (Ww and Wzo). However, since allele is dominant over the w aIlele,3 of the offspring from this cross will purple flowers while just 1 will have white flowers. The color of the flower
observed is referred to as the phenotype. The total number of alleles individual contains is referred to as the genotype. Therefore, the phenotypic of the F2 generation is 3:1. However, the genotypic ratio of the F2 L:2:L (because there is one WW, two Ww, and one ww). Similar expeti be performed for the other traits that Mendel studied.

Copyright @ by The Berkeley Review

242

The Berkeley Specializing in MCAT

Biology
Parental Generation

Genetic Information
Whire

Gregor Mendel

(P)

*w
@@
@F;[it]ll

(Wn)

0
Purple

@lryilEl ffi tr

ww ww
F1
(A11

(Wn)

Purple)

(wr)

First Filial
Generation

(Fr)

*w
Puryle

(ww)W
F2
(3 Purple:

o
Purple

ffi tr
figure 9.5
fr

@@ @lEl,t;l

@''"'
I White)

clt{.'y;l

(nn

i'ased on experiments like these, Mendel proposed that:

s
ts

t
s
M

1,. The traits that were expressed could either be dominant or recessive. For example, purple flower color in the pea plants was dominant over the
recessive white flower color. The parental generation does not transmit their

E
ml

traits or th6ir form to their offspring but rather they transmit hereditary "factors" while include information about the traits to be expressed. Each individual possessed two of these hereditary factors for each trait (one coming from each of the two parents).

The alternative form of a hereditary factor, which leads to an alternative form

of a given trait, is called an allele. For example, the hereditary factor for a purple flower would be one allele while the hereditary factor for the
corresponding white flower would be the other allele.

i"

An individual who possesses two identical alleles is said to be homozygous (e.g, WW for ihe puiple flower color or ww for the white flower color) while an individual who possesses two different alleles is said to be heterozygous (e.g., Ww for the purple flower color).
The two alleles contributed by the parents to an offspring do not influence in any way. In other words, the two alleles come into a cross and
@

',

each other

-,:rvright

by The Berkeley Review

243

The Berkeley Review Specializing in MCAT Preparation

Biology

Genetic Information

Gregor Mendel
t',e

leave a cross unchanged. They do not form intermediate alleles as would expected from the blending hypothesis.

5.
Pollen

The presence of hereditary characters in an

will be expressed. For example, in the F1 generation the white

individual does not ensure that it

ll2W
L/2
h

ll2w
(PrifbXe)

allele is present in the purple and white hybrid but it is not expressed (it is recessirel,lL

1/4wW. tIl,4,'Ww

(le

b!
bo

:1/,4,,I&W

l/4ww
(White)

ll2w tP.urplc)
Figure 9.4

and genotype of an individual. we said that the phenotype is the ou appearance of a given trait. It is what you can actually see (e.g., flower color genotype is the genetic makeup of an individual (e.g., purple flower color rrbe WW or Ww while white flower color would be ww). when Mendel crossed the two true-breeding lines of pea plants he found rn F2 generation a 3:1 ratio of purple flowered plants to white flowered ph Another way to express this (other than Figure 9-3) is shown in Figure 9-4. Ir two alleles are alternate alleles and equally contributing, then haH of tt should be w (upper case) and half should be w (lower case). This would be for both the pollen and the eggs. [when Mendel did this work the punnett sq had not been invented. Lrstead, he used algebraic expressions.] Mendel rcalized that not all the purple flowered plants should have the genotype. He said that some of the purple flowered plants would har-e He predicted that 1/3 of the purple flowered plants should 9tlfgt"i genotype. ww while 2ig ahould be ww. (we are yust coniidering the prrrpl" florve:sd plants and not the white flowered plants. The genotypes of the p"tpt" flou-erEil plants in the F2 generation are w.w, ww, and ww. see Figure s-s.) He tested I prediction in two ways.

Recall that in relationship to what was expressed we mentioned the

His first test was to allow the F2 plants to self-pollinate. If a purple flowered H! plant with the genotype ww self-pollinates, then the offspring will onlr ncr purple flowered plants. If both purple flowered F2 plants with ihe genotrlm ww self-pollinate, then both purple and white flowered plants are produced., This confirms the prediction that 1/3 of the purple flowered F2 plants will be ltw while 2/3 of the purple flowered F2 plants wlli be ww. [a similar test could be
done for just the white flowered pea plants.l

His second test involved a test cross. In this cross Mendel used a purple floweredi plant from the Fl generation in which the genotype was now knbwn (i.e., it rt-m either WW or Ww). In order to determine the genotype of this unknown purple flowered plant, he crossed it with a white flowered plant which rr-as homozygous recessive (i.e., ww) as shown in Figure 9-5.

If the unknown allele distribution in the purple flowered plant is ww, then all of the offspring in the test cross with the homozygous ww white flowered plard will be ww. The offspring will be heterozygous and they will all have purple flowers. F{owever, if the unknown allele distribution in the purple flowered plant is ww, then the results of the test cross will give half heterozygous ww purple flowered plants and half homozygous ww white flowered plants (Figure 9-5).

Copyright @ by The Berkeley Review

2A4

The Berkeley Kevieu Specializing in MCAT Preparation

,I

Biology
White

Genetic Information

Gregor Mendel

(rn,o)

8{*Wry
Since all purple flowers, unknown flower was homozygous (WW ) domi nant.

Allele distribution is unknown. It is either WWor Ww.

Since half purple flowers and

half white flowers, unknown flower was heterozygous (Ww).

Figure 9-5

\'Iendel's experiments therefore confirmed his hypothesis. Alternatiae


segregate

alleles

from each other in heterozygous indiuiduals and retain their identity. This is i<rrown as Mendel's First Law of Heredity (also called the Law of segregation). Even though Mendel showed this to be true for garden peas, it has been shown io be applicable to all eukaryotic organisms (including humans).

RrGg (Fl
Pollen (Male)
1/4

Generation)

RG

1/4

Rg

ll4 rG
1n6
1.116

F2 Generation
Resulted from parents

1ll6

RRGG

RRGg

o o o o o @ o @
1t16

ltr6 RRGg

RrGG

RrGg

with two different


characteristics.

U16

/16

t16

RRgg

RrGg

Rrgg

Round, Yellow

U16

t/t6

1/

16

U16

RrGG

RrG$

o o o o o @ o @
1n6
1t16 1/16
1

RrGg

rrGG

rrGg

Round, Green

t16

Wrinkled, Yellow Wrinkled, Green

Rrgg

rrGg

rrgg

Copyright @ by The Berkeley Review

245

The Berkeley Review Specializing in MCAT Preparation

Biology

Genetic Information

Gregor Mendel

Recall that Mendel identified 7 different pairs of traits. He next examined two different traits segregating in the same plant. The two different traits he considered were round versus wrinkled seeds and yellow versus green seeds. The round seeds are dominant over the recessive wrinkled seeds. The yellow seeds are dominant over the recessive green seeds. [Mendel had determined which seeds were dominant and recessive in a previous crossing experiment involving only one of the pairs.] After establishing pure breeding lines of pea planis with these traits, Mendel crossed a true-breeding pea plant with round seeds which were yellow with a true-breeding pea plant that has wrinkled seeds which were green. He wanted to know if a particular allele for one trait (such as seed color) would influence which allele the gamete had for the other trait (such as the seed shape). The type of cross Mendel constructed is referred to as a dihybrid cross (Figure 9-5). lNote that in this example we will let the uPPer cas R represent round seeds, the upper case G represent yellow seeds, the lower case r represent wrinkled seeds, and the lower case g represent green seeds. The reason behind this is because we already used the letter W in the previous
example with flower color.]

Mendel crossed true-breeding RRGG pea plants with true breeding rrgg Pea plants. This gave an F1 generation in which all the pea plants had round yellow seeds (i.e., RrGg). These dihybrid individuals were next allowed to self-fertilize "If the segregation of alleles affecting seed shape were independent of the segregation of those affecting seed color, then the probability that a particuJar puir of seed shape alleles would occur together with a particular pair of seed color alleles would be simply the product of the individual probabilities that each pair would occur separately. Thus, the probability that an individual n'ith wrinkled, green seeds would aPpeal in the F2 generation would be equal to ttn probabilit| of observing an indlvidual with wrinkled seeds (1/a) limss 'ho probability of observing in individual with green seeds (714), or116J
The composition of the dihybrid cross shown in Figure 9-6 can be predicted mechanism behind placing an R allele or an r allele into a gamete is i

iI

of the mechanism behind placing a G allele and a g allele into another (separate) from each othn [Recail Mendel's First Law: Alternatiue alleles segregate heterozygous indiaiduals and retain their identity. Another way to put this is "two members of a gene pair segregate from each other into the gametes, so 1/2 of the gametes ott" member of the pair and the other 1/2 of the ga "itty carry the other member of the gene pair." A gene pair could be a dominant (allele) and recessive gene (allele).1 Notice that in the dihybrid individuals genes involved in the shape of the seed and the color or the seed can each iepresented by a pair of alternative alleles. This means that one would R6, Rg, rG, and rg gametes. The probabitity that a gamete would be Rg is on Mendel's First Law. In other words, the probability that a gamete would RG is simpty Ll2 x ll2 ot,L14. The same would hold for the Rg, rG, gametes. fi-tii ir where thellacomes from in the dihybrid cross shown in 6-0. rnt would also explain why the probabi]ity of finding an individual wrinkled., green seeds in the F2 generation isLl6.It is simply the-probabil 1/4) times the probability of obsenir observing lhe male gamete rg (w_hich is t|u). tr, other words, the probability of findi female gimete rg (whifh is also rrgg zyfote-onld be L I 4 x\ I 4 or L I 15.
I ,

The results of Mendel's dihybrid cross are shown in Figure 9-7. These v obtained from the F2 generation. Mendel examined 556 seed types and that 315 were round and yellow, 108 were round and green, 101 were u
Copyright @ by The BerkeleY Review 28,6.

The Berkeley Specializing in MCAT

Biology
respectively.

Genetic Information
9:3:3:'1,,

Gregor Mendel

and green, and 32 were wrinkled and green. This gave a ratio of

If you look at the Punnet leuare in Figure 9-6, you will notice that there arc 9116 round and yellow seeds, 3/15 round and green seeds, 3/15 wrinkled and yellow seeds, and116 wrinkled and green seed. This is a ratio of 9:3:3:1. There are 4 phenotypes in the F2 generation. How many genotypes are there for each of the 4
phenotypes?
does this dihybrid cross mean? It simply means that the hereditary factors lgenes) for color (yellow and green) and shape (round and wrinkled) assort independently of one another. This is referred to as Mendel's second Law of Hereditary (or the Law of Independent Assortment). Another way to put this rvould be that the segregation of one gene pair is independent of other gene pairs during the formation of the gametes. one addition to this statement (which \'{endel did not postulate) is that independent assortment of the genes will occur rt they are located on different chromosomes or are far apart on the same

F2 Generation
Round & Yellow Round & Green

Seed #
315
108

Ratio
9 J 3

Wrinkled & Yellow Wrinkled & Green

10r 32

556

t6

\vhat

Figure 9-7

;hromosome.

\Iendel published his findings in 1866 but did not receive much attention until
about 1900, some 16 years after his death. \A/hen Mendel published his papers he lid not know about meiosis, chromosomes, DNA, or even genes. today we i,now his hereditary factor to be genes and his law of segregation to be meiosis.

Flower Color

Seed

Color

:le same gene.

"gene" was first coined in 1911 by wilhelm Johannsen, a Danish geneticist. Genes, which are the basic units of hereditary (DNA), are located at .recific locations along the chromosomes. In fact, they are arranged linearly {ong the chromosomes. Recall that we mentioned that alleles are one of two or =ore alternative states of a given gene. For example, the white and purple flower :olor of the pea plants that Mendel examined is simply due to alternate forms of

lhe word

23-

're
tl *
Flower

chromosome

"t

the genes are on different chromosomes, they will assort independently :ecause the chromosomes themselves assort independently. However, if two

4ffi

renes are on the same chromosome, then they may assort independently if :ossing-over occurs between them. As we will see, the order and spacing of renes on a chromosome can be determined from the frequency with which the :--fferent gene pairs tecombine. Genes that are far apart will recombine more :equently than genes that are closer together.

Positio^ / Pod Pod color Shape . l. Y

,l' / ,l

Plant

Height

lhe frequency with which crossing-over occurs allows one to map the relative
:,:sitions of the various genes on chromosomes. For example, consider the seven :aits which Mendel studied. These traits can be located on the chromosomes of f,e pea plant as shown in Figure 9-8. Seed color and flower color can be found on :"romosome #1 while seed shape can be found on chromosome #7.
1',-ou studied the segregation of seed color and seed shape, you would find that :i:ev wouid assort independently (because they are on different chromosomes). It.rrver color and seed color should assort independently because they are rather m: apart on chromosome #1. Note the location of the genes for plant height and ;,:d shape. Those two genes are quite close to one another and therefore should

5ffi

67ffi
WR

*ta
I

Seed Shape

Figure 9-B

rrt

assort independently.

-.:nyright @ by The Berkeley Review

287

The Berkeley Review Specializing in MCAT Preparation

Biology

Genetic Information

Gregor Mendcl

Let's place the traits for seed color and seed shape on the chromosomes as shown in Figure 9-9. We will let G stand for the seed color and wR stand for seed shape In the gonial cells we will just use chromosome #1 and chromosome #7.

ffi, o',''',,ffi

M ir, WV
ffifl \\
Gonial Cells

Rt/ ffe

/)

"w" ,v,

&A

%f)
wRd
Figure 9.9

&1/ HR

b*

After replication we get homologous pairing and then crossing-over as shown Figure 9-10.

Flgure 9-lO Note the arrangement of the crossover events as they proceed through Te I. This is shown in Figure 9-11. Two possibilities for Telophase I are shown

Copyright @ by The Berkeley Review

2Aa

The Berkeley Specializing in MCAT Prep

Biology

Genetic Information

Gregor Mendel

Figure 9- I I

lValter Sutton r 1902 the American geneticist walter sutton made a logical argument that
.:ated that chromosomes were the places where Mendel's hereditary iactors were . rcated. Sutton postulated:

If Mendel is right, then the sperm and the egg must contribute equally. Also, since the sperm has very little cytoplasm and is mainly composed of nuclear material, the hereditary factors must be contained within the nucleus.

The meiotic behavior of chromosomes is the same as that of Mendel's hereditary factors.
cametes from meiosis have one copy of genetic material for each hereditary factor. Mendel's hereditary factors are distributed in the same way. Chromosomes assort independently in meiosis as do Mendel's hereditary
factors.

with this because it was mentioned that there ::e many more independently segregating pairs of genes (hereditary factors) .: there were chromosomes.
'-"iomas Hunt Morgan 1910 Thomas Hunt Morgan performed a crucial experiment that ,,:'onstrated that sutton's theory was correct. Morgan showed that a gene . :olling eye color in the fruit fly Drosophila melanogaster was located on the X -:-:omosome. As we will see, the results from Morgan's experiments led to the :.,ceptance of sutton's theory. In 1909 F. A. Janssens suggested that homologous -:1romosomes exchanged material during meiosis. At first Janssens theory was - 'ot widely accepted but later experiments would prove him correct. One of those :rperiments was performed in 1931 by the geneticist Curt stem who proved that :rossing-over in the X chromosomes of Drosophila melanogaster involved the rhysical exchange of chromosomal material. with independent segregation of :hromosomes as well as crossing-over along the chromosomes, there canbe more -ndependently segregating units than chromosomes.
Copyright O by The Berkeley Review

-'\'ever, there was problem a

249

The Berkeley Review Specializing in MCAT Preparation

BiologSy

Genetic Information

Gregor

Let's consider Morgan's experiment for a moment. The fruit fly D melanogaster normally has red eyes (referred to as the wild type because it normal). During one of his experiments he had noticed a mutant male fruit i with white eyes. Morgan crossed this mutant white eyed fly with a wild type eyed female fly. All the progeny (offspring) in the Fl generation had wild ! eyes (i.e., all the offspring had red eyes). Next, a male and a female fruit fly v red eyes from this F1 generation were allowed to mate. In the resulting generation all the female fruit flies were of the wild type while only half of male fruit flies were of the wild type. The remaining male fruit flies had white eyed mutation. After examining all of the Progeny in the F2 generab Morgan concluded that eye color segregated among the progeny as predicted Mendel. However, the white-eyed characteristic only expressed itself in the male fruit flies. This is a very unusual result.

After looking at the chromosomes of both sexes of Drosophila

melanol

Morgan discovered that the female had two copies of the X chromosome the male had only one coPy of the X chromosome and a copy of a Y In other words, females flies have an XX genotype while males have an genotype. It tumed out that the trait for white eyes lies on the X chromosome ii missing from the Y chromosome. If a given trait is determined by a gene on X chromosome, it is said to be sex linked (or X linked).

White-eved

Red-eYed

LWd;bWi
Lffi@X@Wl

\\il

Figure 9-12

Copyright @ by The BerkeleY Review

29o

The Berkeley Specializing in MCAT

Biology

Genetic Information

Gregor Mendel

As shown in Figure 9-12,.the F1 generation all have red eyes. This means that the gene for red eyes is dominant. Notice that the female fly itt *," Er generation has a mutant X chromosome. Her eyes are red because her one good X chromosome is stillable-to express red eye coior. However, when this F1 flmale mates with an F1 male who has red eyes, she passes her good X chromosome and her defective X chromosome to her progeny in the F2 generation.

The Fl male passes his y chromosome and. his good X chromosome to the orogeny-in the F2 generation as well. If a female rty"in ure F2 generation were to receive the defective X chromosome, her eye color will still be red because she rvould have received a wild-type x chromosome from the male of the F1 generation. However, if a male received the defective X chromosome from the :emale oJ the Fl generation, then he would have white b".u.rre there would :e no other X chromosome with the gene for-the red "y", .oto, urrurtuble. why? 3ecause the genotype of the male is Xi while "fu that of the female is XX.

Curt Stern 1931 Curt stern did,an experiment that proved that crossing over invorved the ;hysical exchange of chromosomal material. stern examined i*o g"r,", on the X chromosomes of the female ny Drosophila *rtonogo,itii. ii" ,*o genes that {Ttl :e was interested (Figure 9-13) in were the recessive gene for carnation

car) and the dominant gene for the bar_shaped eye (B).

eye color

Fr Femare

'ilfr|

o,o_) |ii t,y'

.rm
..orr,T!-ou", !

L;"'-""'
car
B

tr

firriil /fiD

+-

.o:l',i:?i-:r'
and B

H
t*Fl 'tA
.

''B

u U )\ ./ \ ,/\
H''
Bar Parental

\/[

----" I

tr
'TB

?.T#i' 'If,
t/\ ,l

to.'' ro.,,"rromFl , from Fr MaieGametes Mare ru,. Gameres cun'.,.r-

ja )1r'

f,fi

Carnation

Normal

tr

"TH
Carnation

fi=

rH
Bar

Combinations

Recombinant Combinations

Figure

9-I5
@

rpyright

by The Berkeley Review

291

The Berkeley Review Specializing in MCAT prCparation

Biology

Genetic Information

Gregor Mendd

One of the X chromosomes could be identified because it had a "gap" in its structure (due to inability to stain properly) while the other X chromosome could be identified because it had a piece of a Y chromosome attached to it. These two chromosomes could be clearly distinguished from one another under the liglt
microscope.

As shown in Figure 9-13 the experiment starts out with the female's FL I chromosomes. Each has the respective genetic locus (location of a gene on r
chromosome) for the carnation eye color and the bar-shaped eye. Stern crossed" these F1 females with the sperm from a male fruit fly who had carnation co eyes. He then separated out the cross-over events in the F2 progeny' He found normal shaped eyes that were carnation colored and normal eyes that were bar-shaped in the progeny that had experienced crossing-over. the progeny that had not experienced crossing over, he observed carnati colored eyes that had the bar-shape and normal colored eyes without the shape. Stern concluded that the genetic exchanges of various traits on a gi chromosome (such as eye color or eye shape) involve the actual exchange portions of those chromosomes in a event referred to as crossing-over. In words, whenever genes recombine, chromosomes recombine. [Note that in diagram in Figure 9-13 the "+" sign indicates the wild type individual or normal individual.l

Alfred Sturtevant Alfred Sturtevant (a student of Morgan's) constructed the first genetic map L93L. Sturtevant examined three traits in the female fruit fly Droso
melanogaster. Th"y were yellow body color (designated as y), white eye color and miniature wings (min). These traits resided on the X chromosome and sex-linked. These lower cases refer to the recessive alleles' The normal color is gray (+), the normal eye color is red (+), and the normal wing (+) is

50% longer than the miniature wing. [Note that here we are using the terminology which means the dominant allele. We could have used upper Y instead of the +, an upper case W instead of the +, and upper case MIN i of the +, respectively. The meaning is the same.]

Figure 9-14 Sturtevant crossed a female fruit fty that was homozygous recessive for the traits with a male fruit fly that was normal for these traits. All of the progeny were heterozy1ous as shown in Figure 9-14. The important point lhaf the progeny females are heterozygotes. This means that if crossir
Copyright @ by The BerkeleY Review

292

The Berkeley Specializing in MCAT

Biology

Genetic Information
will result with different combinations of

Gregor Mendel

occurs between any two traits, gametes the alleles.

In order for sturtevant to see all the possible recombinant types, he crossed the F1 females shown in Figure 9-14 with males that were tecessirre to all three traits. This cross is shown in Figure 9-15.
Female
X Chromosome
X Chromosbme

Male

+..+-4-

ywmin
+

min
X Chromosome Y Chromosome

Progeny Fruit Flies


Figure 9.

l5

The progeny were next scored for the phenotype expressing these three traits. sturtevant's results are shown in Table 9-2. when you look ut this table, keep in mind the cross shown in Figure 9-15. Let's consider the body, uyu, #ir,g phenotype (1st row) which reads + + +. How is this phenotype ottaineaz mo[ at the female's X chromosomes in Figure g-l.s. rf the femlleX chromosome rvhich reads + + + combines with the male X chromosome which reads y ta min, henthe resultant progeny will be a female with the phenotype +++. Similarly, if

;hromosome, then the resultant progeny will be male with the phenotype + + +. This is one parental phenotype (i.e., + + +) that is observed in ribte g-1. we can lo a similar analysis for the y za min phenotype (2nd row) in Table 9-2 as well. This is also a parental phenotype. The number of progeny observed for the larental phenotype + + + is 758 while the number of progeny observed for the rarental phenotype y m min is 700. Because these are random events we do not
*et the same numbers.

lhe female X chromosome which reads + + + combinei with the malL y

PHENOTYPES
Body
Number of

CROSSOVBR TYPES
Body/Wing

-Eye Wing Progeny Body/Eye EyeAiling


+

Single Crossover

Double crossover

++ yw ++ yw +w y+ +w y+
(o)

758
700 401 401
317

min
mrn
+

40i
317
16

3t7
16

min
+ +

t2
1

1; t2
1
1

t2
0

min

TOTAL
Recombination Frequency

2205

29

719
32.608

746 33.832

Iable 9-2
;\'here did the other six classes of phenotypes come from? Consider the 3rd row :f phenotypes in which we have the + + min phenotype. How did we get this :henotype? There must have been a crossover as shown in Figure 9-16. \A/hen
Jopyright @ by The Berkeley Review

293

The Berkeley Review Specializing in MCAT Preparation

Biology

Genetic Information

Gregor

get the phenotype y w +. The number of progeny observed for the + + phenotype is 401 while for the y w + phenotype it is 317.

the resulting male progeny would have the phenotype + + min. A si analysis can be done for the 4th row of phenotypes in Table 9-2. In this case

the two female X chromosomes separated we would have y tn + and + + Suppose the female + + min combined with the male X chromosome which y w min. The resulting female progeny would have the phenotype + + mi [Remember, a genotype of (+ + min) / (y w min) would give a phenotype of + min because the dominant + + genes would be expressed over the recessive y genes. The min / min genotype is still recessive and so the phenotype would min.l Similarly, if. the female + + min combined with the male y ch

Progeny Fruit Flies


Figure 9.16
Consider the 5th row of phenotypes in Table 9-2 which reads + w min. We obtain this type of phenotype by doing a cross as shown in Figure 9-12. Aftu

female

X chromosomes crossover and pull apart we will have a y +


If the female
+ ut min

chromosome and a + zu min chromosome.

progeny will be female with the phenotype + za min. There are 16 observed with this phenotype. Similarly, if the female + zn min combines with the male Y chromosome, the resulting male progeny will have phenotype + w min. We can do a similar analysis if the female y chromosome combines with either the male X chromosome which rcads y m or the male Y chromosome. In both cases the progeny will have the w min. The number of progeny observed in this case is 12. Some will be and some will be males.

combines with the X chromosome of the male which rcads y za min, the

+s
Progeny Fruit Flies
Figure 9- l7
Copyright @ by The Berkeley Review

294

The Berkeley Specializing in MCAT

Biotogy

Genetic Information

Gregor Irlendel

Notice that the cross in-Figure 9-r7 yields 16 progeny with the phenotype + zo y:" 11112 progeny with the phenotype y + +. Notice that the cross in Figure 915 yields 401 progeny with the phenotype + + min and 317 progeny ,"ith th" phenotype y w +. why the difference in the number of progenyi Ii has to do with the distance the genetic markers (i.e., the genes) uru u*ulifrom one another. The further they are away from one another, the more chancl there will be for a cross. The closer they are to one another, the less chance there will be for a cross. Irr.other words, the y and zu genetic markers are closer together than the zp and. min genetic markers. Therefore, we can modify our chroirosomes to represent this as shown in Figure 9-18.
Female

Male

X-as

ywmin

x+

++

+ft

x-1

ywmin

Progeny Fruit Flies


Figure g- l8 Consider the 7th rowinTable 9-2. we can explain the phenotype+ 70 + by a double crossover as shown in Figure 9-19. The number oi ptog".y observed for

:ecause it is explained in the previous examples.] In the gth row we see the :henotype y + min. we can obtain this phenotype from the double crossover ;hown in Figure 9-19 as well. The number o-f observed progeny for the :henotype y + min is 0. Female

crossover is 1. [I have left out the details of expTaining how the :hromosomes combine to get the genotype and then the ob^served"phenotype

:his

Progeny
Figure g,-r,e

l9

total number of progeny observed. in this experiment is 2205. The total -rmber of progeny observed to have crossover events between the y and w ;:netic markers is 16."+ 12 +'1, = 29. Therefore, this amount of cross-ing over :=presents (2912205) x 100 or 't .g'l,s% of the total number of crossover events.

:-nilarly, the total number of progeny observed to have crossover events

::tween the zu and min genetic markers is 401 + i17 + I = 719. This represents '7912205 or 32.6080/" of the total number of crossover events. Finally, the total ' :rnber of crossover events between the y and min geneticmarkers is 401 + 317 +
:ryright
@

by The Berkeley Review

295

The Berkeley Review Specializing in MCAT preparation

Biology

Genetic Information

Gregor Mendel

16 + 12 = 746. This represents 74612205 or 33.g32% of the total number of crossover events. What we have just defined in this discussion is the map distance between a gene pair. This distance is usually expressed in terms of i map unit. In other words, if the frequency between two genes (e.g., y andmin) is 33.832'/", then those genes are said to be separated by about 33.g map units. A map unit is sometimes referred to as a centimorgan (cM) in honor of r. H. Morgan. Knowing the map units between genetic markers allows us to draw a genetic map. we can do this for the analysis in Table 9-2. This is shown in Figure 9-20.

min

Figure 9-2O

,n

ir

t:.

ta

'ffiri,

:,i:

:E:

::
-!e

!r

Copyright @ by The Berkeley Review

296

The Berkeley Review Specializing in MCAT Preparation

Biology
ffiGHGffi$i:::ruUffi

Genetic Information

Genetic Loci & Alleles

lliffilliffilil

iG

;iiii

So far we have only considered two alleles at each genetic locus. For example, in Mendel's garden peas we had a flower color locus in which one allele was able to express a purple flower while the other allele was able to express a white flower. Recall that the purple flower allele was dominant over the recessive white flower allele. It tums out that a given genetic locus can have many alleles. One example is the ABO blood group.

We have considered only one gene controlling a particular phenotype (e.g., flower color, seed shape, etc.). However, as we will see, many genes may control a particular phenotype. For example, in the biosynthesis of many compounds within the cell there can be many sequential steps. Each step is controlled by a separate enzyme which in turn is controlled by a particular gene. One such biochemical pathway of interest involves the synthesis of the tryptophan. This amino acid is considered an essential amino acid. An essential amino acid is an amino acid that an organism cannot synthesize itself and therefore must obtain that amino acid in its diet.

Tryptophan
One class of mutants that we should consider are the auxotrophic mutants. An auxotroph is a mutant that will grow only when its medium is supplemented

rvith a particular compound which is not required by the normal wild type
organism. The wild type organism is referred to as a prototroph. An auxotroph rvill not grow on a minimal medium. For example, an auxotroph might require certain vitamins, amino acids, purines, or even pyrimidines for its growth. A prototroph will grow on a minimal medium.

will consider is an auxotroph for the amino acid tryptophan. Tryptophan auxotrophs will grow on a complete medium. They will not grow on a minimal medium unless that minimal medium has been supplemented with tryptophan.
One specific type of auxotroph that we
IrL

itill

order to study these tryptophan auxotrophs we need to isolate them. As we see, they are not all the same. How could we isolate such an auxotroph? L:r -]re case of a penicillir mutant all we needed to do was plate bacteria on a agar plate, add penicillin, and find which bacteria survived. In the case of the sugar utiJization mutallts all we needed to do was look at the array of petri plates that ive generated and find the white bacterial colonies. F{owever, in the case of a :yptophan auxotroph we need to find a colony of bacteria that does not grow. l{ow can we isolate a bacterium that will grow in a complete medium but will :.ot grow in a minimal medium?

In 7952 Esther and Joshua Lederberg devised an experimental procedure called teplica ptating. This procedure was designed to isolate auxotrophs. The :rocedure behind replica plating is straight forward. Bacteria are incubated for a :eriod of time on a master plate which contains a complete medium. Both the will grow on this master plate. These bacteria 'uxotrophs and the prototrophs that they will form individual colonies. Next, you -e distributed in Such a way master plate with a velvet stamp. The velvet of this lently touch the colonies of the bacterial cells of the master pick up some :tamp has fine hairs which can to a new plate so that the bacterial touched :late. The velvet stamp is then they were in on the master that orientation :clonies are set down in the same (see Figure 9-21). If our plate replica referred to as a :late. This new plate is
Jopyright @ by The Berkeley Review

297

The Berkeley Review Specializing in MCAT Preparation

Biology

Genetic Infonnation

Genetic Loci & Alleles

replica plate contained just a minimal medium, then only the prototrophs would grow. The auxotrophs would not grow because they need to have a particula_r supplement in their medium. The replica plate can be compared with the master plate to determine which colonies are auxotrophic. once you have determined which colonies on the master plate are auxotrophic colonies, you can then test those colonies to see exactly what compound it is that they need for their growth-

Master Plate

Replica Plate

Figure 9.21
Replica Plating Technique.

Let's consider some tryptophan (abbreviated as Trp) auxotrophs. A bacterium" which is a tryptophan prototroph is designated as Trp+. This bacterium is the wild type. It is normal. It does not need tryptophan in order to grow because rt can make this amino acid from the compounds given to it in lts medium. -{ mutant bacterium which requires tryptophan for its growth is designated as Try-.

It turns out that the study of tryptophan


e-22).

auxotrophs was important in th;

elucidation of the biochemical pathway for the synthesis of tryptophan (Figure

Gene

1t
Anthranilate

Gene 2

Anthranilate
phosphoribosyl lransferase

Chorismate'->
trpE
Gene 5 Enzyme 5

synthase

Anthranilate

+ trp"t

N-(5'-Phosphoribosyl) anthranilate
-

Gene 4 Enzyme 4

ry

C1

{,

fl

Enzyme 3

(f

Gene

-r

Indole

Enzvze 6

iI

\;t
(-'
\

Indole-3-glycerol
phosphate
+
NHq
tI

r-J
trp
C2

Enol- 1-o-carboxy-

phenylamino1-deoxyribulose phosphate

Hnc- C-COO
H

"-*u

e+
H

Tryptophan

,ffi

Figure 9-22
The Tryptophan Biosynthetic Pathway.

Lffi

In Figure 9-22 we see part of the pathway that leads to the synthesis od tryptophan. We start with chorismate because it has a number of differeni
Copyright @ by The Berkeley Review

ffi

ru

294

The Berkeley Reviw Specializing in MCAT Preparatiol

Biology
,

Genetic Information

Genetic Loci & Alleles

,nemical routes that it can take. one is towards the synthesis of tryptophan. -:ismate is converted to anthranil ate by the enzyme anthranilati iyninase. -: gene that codes for this enzyme is referred to as trpE.Anthraiitate is - erted to a compound called PRA by a transferase enzyme. The gene that *es for this enzyme is designated as frpD. pRA is converted to a molecule r--=d CDRP by an isomerase enzyme. The gene that codes for this enzyme is :..-gnated as trpC1. CDRP is converted to a molecule called iGp by a synthase '* -'.'me. This gene that codes for this enzyme is designate d as trpC2.IGp can be ::-','erted to indole by the alpha subunit of the tryptophan synthase enzyme. -r-: gene that codes for this enzyme is designated as trpA. Finally, indole cin be ::-,,'erted to tryptophan by the beta subunit of the tryptophan synthase enzyme. - = gene that codes for this enzyme is designate d as trpB.
'n orE: Do not get lost in the names which are involved in this pathway. The : , -rt is to understand what happens to the synthesis of tryptophan if a mutation

.:e to occur in a gene coding for one of the

enzymes

: t:nrVay.]
,

in this biochemical

- -: three mutants that we want to consider in this pathway involve the trpE, -:'-{, and trpB genes.If there are mutations in these genes, then the designation ; lryE-' trpA-, and trpB'. Consider the entries in Table 9-3. what we would iike , .Tlo\4/ is whether or not certain types of bacteria will grow if their minimal :.=dium is supplemented with (a) nothing, (b) anthraniGte, (c) indole, or (d) :-. ntophan. when we consider this table, we will look at the type or condition of :.: bacterium (e.g., trp+), and then follow the row that the bacterium is in, across ':^d towards the right, as we supplement its medium with the four items that we *-.t mentioned.

Tryptophan Auxotrophs of E. coli

k
TrP+

Growth on Minimal Medium Plus:

Nothing Anthranilate Indole


+
+ + + + +

Tryptophan

TrpETrpATrpBTable 9.5
:+'ptophan Auxotrophs.

+ + + +

\ote that the trp+ prototroph will grow if nothing is added to the minimal nedium. This is simply by definition (as it is the wild type bacterium). This 'racterium can utilize tire components of that minimal medium to synthesize iryptophan. The trp+ prototroph will still grow if you add either anthranilate, ndole, or even tryptophan itself. This is why we see all pluses (+) in that row.
Consider the hpB'*rturlt. This bacterium will not grow if nothing, anthranilate, or indole is added to the minimal medium. This auxotroph will only gro.vr. is tryptophan is added. Thus, as we go from left to right in this row we see three minus signs (-) and one plus sign (+). What is the explanation for this behavior? It must mean that the mutation in the trpB gene leads to a defective enzyme that
Copyright @ by The Berkeley Review

299

The Berkeley Review Specializing in MCAT preparation

Biology

Genetic Information

Genetic Loci &

will not allow indole to be converted to tryptophan. In other words, there is block at the level of the trpB gene. Note that anything preceding indole (even the enzymes are of the wild type) cannot be converted to tryptophan eith \A/hy? Because once the products get to indole it is indole that uitimately must converted to tryptophan.
Consider the tupE- mutant. This mutant will not grow if nothing is added to medium. Frowever, this auxotroph wilt grow if either anthranilate, indole, tryptophan is added. Therefore, as we go from left to right in the table, we minus sign t) and three plus signs (+). we can use the same analysis as abr The mutation n the trpE gene leads to a defective enz).me which cannot con
chorismate acid to anthranilate.
,

Finally, consider the trpA'mutant. This mutant will not grow is nothing anthranilate is added. However, this auxotroph will grow if either indole tryptophan are added. Thus, as we go from left to right across the row in table we find two minus signs (-) and two plus signs (+). Again, we can use same analysis as above. The defective enzyme is a result of a mutation in trpA gene. This enzyme cannot convert the precursor of indole to indole. the pathway must be blocked at the level of the trpA enzyme.

defective (possibly due to a mutation), then that gene will not be able to prodt the enzyme which converts a particular reactant to a particular product. means that tryptophan is not synthesized. In order for the cell to survive it be supplemented with this amino acid. Even if two or more of the genes in biosynthetic pathway for tryptophan are defective, then the cell willitill req the presence of tryptophan to grow. Therefore, the genes involved in biosynthetic pathway of tryptophan ale said to act in an epistatic fashi Epistasis occurs between different pairs of genes. It does not occur between members of an allelic pair. [In other words, two different genes which are alleles of one another may affect the same outcome, which, in our example, is inability to synthesize tryptophan.l In order for tryptophan to be syntheiized, dominant allele of all the genes involved in this biosynthetic pathway must present (in the absence of tryptophan).
There are other sets of genes which may act in an additive fashion. For in yeast there are 7 genes which control the synthesis of the enzyrrle i

If any one of these genes that is involved in the synthesis of tryptophan

Epistasis and Pleiotropy

This enzyme hydrolyzes the disaccharide sucrose into its two m constituents, glucose and fructose. If the yeast cell possesses the dominant of any one of these 7 genes, it will be able to ferment sucrose. If the cell more than one dominant allele, it ferments sucrose at a quicker rate. It is that many human traits (e.g., height) are controlled by such gene families.
We mentioned that the dominant gene in Mendel's pea plants expressed a color while the recessive gene expressed a white flower color. As far as it known, the gene for white flowers in the pea plant only causes the flowers to white. Flowever, there are examples in mice in which a dominant gene ca yellow coat color. If the mice had one copy of this gene, they had yellow coats. the mice had two copies of this gene, it would be lethal for the mice. In this not only does the gene affect coat color but another copy of it affects via This is an example of pleiotropy. It is where an individual allele has more
Copyright @ by The Berkeley Review

60()

The Berkeley Specializing in MCAT

Biotogy
dead).

Genetic Information

Genetic Loci & Alleles

one effect on the phenotype (e.9., a mouse with a yellow coat or a mouse that is

Chromosomes We have mentioned that humans have 23 pairs of chromosomes. Of those

23

pairs of chromosomes 22 pairs appear nearly identical and are called autosomes. These autosomes are numbered from I to 22 and are arranged in order of decreasing size. This is simply a convention. The size of the chromosomes ranges from about 200 mb (the largest chromosome) to about 50 mb (the smallest chromosome). The other pair of chromosomes is the sex chromosomes. The sex chromosomes are either designated as XX (female) or XY (male).
Let's consider an individual human chromosome such as chromosome 19. This chromosome contains about 60 x 106 base pairs in length. This is a fairly small

chromosome as far as chromosomes are concerned. The largest human chromosome is chromosome 1, then chromosome 2, and so on. Chromosomes can be stained to reveal a banding pattern as shown in Figure 9-23. These banding patterns are quite specific and they are constant from chromosome to chromosome. Within these different banding patterns we can assign genetic markers. The long arm of the chromosome is assigned the letter q while the short arm is assigned the letter p. Each arm is further divided into regions such as 1, 2, 3, and so on. Each region is divided into band numbers.

tr F
63

r a

r
p
1

band

number
13.3 13.2

Insulin receptor gene and

LDLR gene are located in this area of the arm

region

I 3'

F
50

t
q
L

centromere

Excision repair gene is located in this area

L
Chromosome 19
Figure 9-23

of the arm

.i.ere are a variety of known genes on this chromosome. For example, the gene :"ne for excision repair (ERCCI) is located in the q arm. The gene for the insulin :r,.eptor is also located in the p arm.

:: low density lipoprotein receptors (LDIR) is located in the P arm while

the

:. one haploid set of 23 chromosomes there are about 3 x 109 base pairs of DNA. .rce we are diploid there should be about 6 x'L09 base pairs of DNA in every :=-1. It has been estimated that there are about 100,000 genes per haploid set of
-:pyright
@

by The Berkeley Review

50t

The Berkeley Review Specializing in MCAT Preparation

Biotogy

Genetic Infonnation

Genetic Loci &

chromosomes. The size. of can range from about 1,000 base pairs about L,000,000 base pairs. 1g"1" The function of only about a thousand. out of hundred thousand genes is known. In the years to colne you probably witr hearing a lot about the human genome project. This project'is a irast underbali designed to map the. entire human genome. one fr ihe goals of the Genome Project is to find out the function of all of those genIs.

Copyright @ by The Berkeley Review

302

The Berkeley Specializing in MCAT

Biology

Genetic Information

The Pedigree

\Alhen one established a pedigree a number of different symbols are utilized. For example, a square represents a male while a circle represents a female. If this male and female matry, the square and circle are joined by a line. If they have children, an inverted "T" is dropped to their offspring. Their offspring are

by short vertical lines. The birth order of the offspring is arranged from oldest to youngest and runs from left to right. Separate generations will reside on separate horizontal lines. The most ancestral generation is always at the top. Each generation is numbered with Roman numerals. The oldest generation is given the Roman numeral I, the next oldest r,leneration the Roman numeral II, and so on. If a square or a circle is filled in ,r'ith a dark color, then that individual is affected with a given defect. If there is a siash through the square or circle, that individual is heterozygous for a given jefect. An example of this symbolism is shown in Figure 9-24.

attached to this inverted "T"

Heterozygote

Yigure 9-24

::a11that when Mendel crossed the ww geno,lype from the F1 generation with in the F2 generation the gienotypes WW, Ww, Ww, and ww. -.:.e flowers with the WW and Ww genotypes were purple while the flowers --*r the ww genotype were white. We mentioned that this was the characteristic : i ratio. In human hereditary we are able to observe an individual similar to the
r

:-=U, he obtained

;=rotypes in the F2 generation. We do not know what has happen in the ::-','ious generations (i.e., the F1 generation). What took place in the previous - -rerations of a human lineage is something that must be deduced. This : .iuction is based on the rules for pedigree analysis.
a hundred trillion cells and in each cell we have about - --'-rndred thousand genes. In most humans all of those genes in each of those :-; rvorks properly from day to day. It is known that one defective gene in one ::.ese cells may be enough to cause a cancer. If an individual inherits a I :.:.tive gene from one of both of his/her parents, then this may cause a genetic t'.iease. Today, about 4,000 genetic diseases are recognized.
::

ft'enetic Diseases: -- :ur body we have about

i --:r- chromosome 4ssortment during meiosis may lead to genetic ,:. ,rmalities. For exafirple, if there is only one copy of any one of the ',: -.:rosomes

'

::'

(called monosomy), the individual will not survive development. arly, three copies (trisomy) of all but chromosom es L3, L5, 78, 21, and 22 is are trisomic for these chromosomes are severely 1,8 causes severe developmental

, r=-ied. Trisomy for chromosomes 13, 15, and


:rght O by The Berkeley Review

--:i and individuals who

303

The Berkeley Review Specializing in MCAT Preparation

Biology

Genetic Information

The Pedigree

problems and such individuals die a few months after birth. Trisomy for chromosomes 21 or 22 survive to adulthood but are severely affected. The mos; common from of trisomy is for chromosome 21. This is Down's syndrome. It is an example of aneuploidy (that condition in which nuclei have an unbalanceo set of chromosomes--that is, they do not contain an exact multiple of the haploii number of chromosomes). If the nuclei have a normal complement o: chromosomes, it is referred to as being euploid.
The incidence of Down's syndrome increases dramatically with the age of the female. For example, the incidence of having a chitd with tiisomy 21 for mothes who are between the ages of 20 and 30 is about 1 in 1,400. For mothers who or-s 45 years of age the incidence is about 1 in 16 births. All of the eggs that a woma:: will ever produced have developed to the level of prophase t uy tne time she rs born. some of these eggs then complete meiosis every menstruil cycle. In othe words, the older the woman is, the older are her eggs that complete Meiosis I ani Meiosis II. It is thought that as time passes there is damage to the eggs and tha: interferes with the normal disjunction of chromosome 21.

I
I

{ I

,l

ni

,{

Nondisjunction of the sex chromosomes can also occur during meiosis. F;r example, nondisjunction of the two X chromosomes in females can lead tc a

referred to

o). If the female XX gamete combines with a male y gamete, fr* resulting offspring will be XXY male. This condition is refeired to as Klinefelter's syndrome. If the o gamete from the female combines with ar.l 1. gamete from the male, the resulting offspring will be an Xo female. Thi_e r
as simply
as

gametes that are XX or gametes that do not contain an X chromosome (referred. :r,

Turnerts syndrome.

The many thousands of genetic diseases resulting from single gene defects can be classified into one of the four following traits:

\ '

(1) Autosomal recessive (2) Autosomal dominant (3) Sex-linkedrecessive (4) Sex{inked dominant

Pedigree analysis allows one to determine which of these four classification-c a given genetic disease falls into. Let's consider some examples of the more common genetic diseases.

Hemophilia, a blood clotting condition due to a lack of a protein called Factor VIII, is an example of a sex-linked recessive disease involving the L chromosome. If a male inherits an allele that has the mutation or Factor VIII, ru will develop hemophilia. Perhaps the most well documented case invoh-ei Queen Victoria of England and her offspring. Queen Victoria had 9 childrrTwo of her daughters were carriers while one son was a hemophiliac. Of her $0 children, grandchildren, and great grandchildren, 19 were males and 21. n e:e females. Of those 19 males, 10 (i.e., 10140 - 25%) developed hemophilia. Note th,-d: those 10 individuals who developed hemophilia were all males. None of the 11 female descendants were affected with hemophilia. This is because it L. n recessive condition.
Consider a sex-iinked disease versus an autosomal recessive disease. If a grl mr disease effects both sexes equally, then it is likely to be autosomal. If males as affected almost exclusively, the disease is said to be sex-linked. Roughli' c'-:e
Copyright @ by The Berkeley Review

3o'4

The Berkeley Revicn Specializing in MCAT Preparatic

Biology
appear normal.

Genetic Infonnation

The Pedigree

quarter of the children should be affected in either case. Also, both parents may

is possible with a recessive trait to have one parent who is also affected parent is homozygous for the defect.

Consider dominant versus recessive diseases. An individual who is affected with a dominant genetic trait will have one parent who is also affected. Individuals affected with a recessive trait may have parents who both appear to be normal. It

if that

An example of a hereditary disease that is autosomal dominant is Huntington's disease' This disease leads to the progressive deterioration of brain celli and eventual death. The onset of the symptoms of this disease do not usually manifest themselves until the individual is in mid-life (i.e., 30 to 50 years of age). A test is available to determine if an individual will get the disease. As you -an imagine, this has created quite an issue for those individuals who think they might be afflicted with the disease. Is it better to know or not to know?

\Ve

shown in Figure 9-25. In this pedigree there are two couples. This is generatiJn I. will number these individuals in these two couples r,2, g, and 4lfrom left to ngh0. Individuals 1 and 2 are a couple and individuals 3 and 4 are a couple. The ;ouple on the left have 4 children while the couple on the right have 3 children. This is generation IL Again, the individuals in generation II can be numbered 1 though 7 (from left to right). one member of each family mates (i.e., II-4 and II-5) end have 4 children. This is generation III. once again, the children are rumbered 1 though 4 (from left to right).

Autosomal Recessive s-uppose you are a genetic counselor and you are presented with the pedigree

Il

ilI
rigure 9.25
I-

rte that the female atII-2 has a genetic disease while the female at III-1 and the III-3 have a genetic disease. Is this disease sex-linked or autosomal and is : lominant or is it lecessive? Even though our sample is small we can say that ::*. disease is autosomal. \,vhy? The disease is affecting both males and females. l::s disease is also recessive. \Arhy? The parents of child ll-2 are both normal as
:-a,le at

;,:e the parents


r*cessive.

of children III-1 and III-3. Thus, the disease is autosomal

:r'right

by The Berkeley Review

305

The Berkeley Review Specializing in MCAT Preparation

Biology

Genetic Information

The Pedigree

It is either one or the other. However, with the information given you do not know which one is the carrier. [They could both be heterozyfotes and still not have any diseased children just by chance!] Those individuaf who are carriers are indicated in Figure 9-25.
should the couple I-3 and I-4 have any more children? If one of them is heterozygous, then there should be no chance of them having a child with the disease. However, the couple I-1 and r-2 and the couple II-a and II-5 should te advised that if they have more children, they run the risk of having children ryith

as are the parents of siblings II-1 through II-4. wh; about the parents of siblings II-5 though rr-7? rn this case either parent I-3 or parent t-+ wouta be the carrier.

for the disease. Thus, the parents of the siblings in generation

The next step in analy-zing this pedigree would be to figure out the heterozygotes. since the disease is autosomal and it is recessive, then the onhway that you could get affected individuars would be if both pur"rrt, ur"

iIt

";;; are both carriers

the disease.

in generation IIL \44rat is the chance that siblings III-2 anc disease? They have a 2/3 chance of beiig carrier: Suppose that sibling III-2 matures to adulthood and marries. \Alhat is the chance that her children will be carriers? It would be ll2.
C_onsider the siblings

III-4 are carriers of this

rl

For many diseases there are tests that can determine if an individual is a carris of a given disease. For example, genetic analysis for sickle-cell anemia can easih: determine if a person has the gene or not ior that disease. There is a tesf t.", Huntington's disease that is about 95% accurate. A new development that rras just approved this year by the National Institute of Health is gene therapy. T|,Js ?nu oltherapy is accepted for somatic cell lines but not fot ger"m cell lines. \,\'h.,,: Consider the sickle-cell gene for a moment. That gene is noipresent just to be a:: annoyance for individuals. It has had a tremendous selective advantage over tne years in malarial infected environments by giving resistance to malaria to thcr"e

fl

i0

ffi

ffi

for-sickle-cell anemia, and that would begin to decrease.

introdr,rce a normal gene into the germ line of individuals who are heterozygotes

individuals who are heterozygotes for sickle-cell anemia. If you were :r

g"."

p.opugated, then resistance to malar:a

'@

lm

,M

ffi

Copyright @ by The Berkeley Review

306

The Berkeley Specializing in MCAT prep

Biology

Genetic Information

Central Dogma of Molecular Biology

Up until

1'944

chromosomal proteins. During the late 1940s and early tesos it was proven that DNA was the carrier of genetic information (and not proteins as had otherwise been assumed). Once it was realized that DNA was involved in the genetic of an organism it was suggested that the flow of information went from DNA to protein (DNA -+ Protein). Is the flow of information from DNA to proteins a direct process? Could information flow in the reverse direction--from proteins to DNA?

it was assumed that the carrier of genetic information

was

the cell. Because proteins are being synthesized in a different compartment, it was a direct argument that there was not a direct template of DNA to protein. What is the intermediate step in this process?
Within the cytoplasm are structures called ribosomes. This was the site where protein synthesis was occurring. Analysis of the ribosome revealed that it was composed of two different types of biomolecules--ribosomal proteins (several iypes) and ribosomal RNA (rRNA). Maybe there was an RNA intermediate between the flow of information from DNA to protein. where was rRNA synthesized? Inside the cell's nucleus (and then lateiexported to the cytoplasm). Thus, it was imagined that DNA could give rise to a paiticular type oi nNa ana then that RNA could be exported from the nucleus where it would eventually give rise to proteins. This is almost entirely true but nevertheless somewhat

resides the cell's DNA. synthesis of proteins, however, occurs in the cytoplasm of -cellular

Within a eukaryotic cell there is a defined nucleus and within that nucleus

confusing.

If we look at the analysis of certain proteins-ik-e silk, we wilt find that it is composed almost entirely of three amino acids--Ser, Gly, and Ala. other proteins h\: l"f have a very high content of Cys. There are a variety of different proteins have quite a variety of different amino acids. However, when we analyze ryhich the base content of the rRNA in ribosomes from a variety of different organisms, rve find that they all have an rRNA base content which is practically identical. This did not seem right. If we had an organism that synthesized pioteins of a particular type, we would expect to find an intermediate instructi,onal process that would be different in its base content than we would for an organism with a r-ery different set of proteins.
Francis Crick suggested that there must be at least two or three different types of RNA of which rRNA would be only one. His suggestion was taken to helrt in 196lby Francois Jacob and Jacques Monod. They postulated that DNA can also form a type of RNA called messenger RNA (mRNA) which had the instructions that resulted in the actual protein. They suggested that mRNA is the actual iemplate for synthegzing proteins and that rRNA is simply a portion of the architectural framework of the ribosome.

-{ bacterium does not utilize all of its DNA all the time because it has more nJormation than it could possibly need at any one given moment. If the food 'upply for this bacterium was changed from glucose to lactose, and back and
Copyright @ by The Berkeley Review

307

The Berkeley Review Specializing in MCAT preparation

Biology

Genetic Information

Central Dogma of Molecular Biotogi

forth a few times, that bacterium would adapt and change the proteins it was synthesizing. The mRNA made under one set of conditionJ,"iff U" different from the nRNA made under a different set of conditions.
For example, when a bacterium is infected by a phage, most of the operations ot the host bacterium cease. In 1962Jacob, Meselson,"and sydney Brenner were able to isolate *RN+ a T2 viral phage. No new ribosomes are synthesized" {rom Thus, no new rRNA is being made by the infected host. However, all of a sudden the phage DNA will give rise to many proteins that were never seen before the infection. These proteins must have been synthesized from mRNA that lt,a_. m1f9 specifically by the T2 phage DNA. In other words, there is T2 viral specific mRNA and bacterial specific mRNA--and both of those mRNAs can be translated on the same ribosome.
IA/hat physical properties should mRNA have? one property is for the mRNA tc, have a rapid turnover. If the conditions change in tire cell, you do not want the old mRNA making the same old protein anymore. However, rRNA has a rather long half-life. This is why nobody had discovered up to this point. It had 'RNA a very short half-life and was rapidly turning over in the ce[. In fac't, only about 4% of the total RNA in the cell is ever mRNA. Almost all of the RNA in a cell i_.

rl

rRNA.

il

Informational flow from DNA to RNA is called transcription wh"i-le informational flow from RNA to protein is called translation. Because DNA car replicate itself we can draw an arrow on DNA as shown in Figure 9-26.

At the time there was not enough evidence to prove this hypothesis of mRNA being a template for protein s;mthesis. Crick siw this and tus"d this article ot faith on what he called a dogma. This lineage of informational flow from DNA -+ RNA -+ Protein soon became the central dogma of molecular biologr,_

fr

ffi

,d

,t

DNA :

RNA

Protein

-->

im

il m

Figure 9-26

fim

iln

m
@[

,S

ffi"i

ilh
@ @
GX

Copyright @ by The Berkeley Review

30a

The Berkeley Revier Specializing in MCAT preparation

Biology
nilffiftt$fi

Genetic Information
$.::::df rffi Uol$ild,i::fi,Uidi

Functions of Nucleic Acids

What are the functions of these nucleic acids? Fred Griffith (circa 1928) was looking at infections caused by the pneumococcus bacterium. This bacterium can cause pneumonia in mice. The pneumococcus comes in two varieties--the normal house and garden variety that is surrounded by a smooth (S) polysaccharide capsule and the mutant variety that does not have this capsule and appears rather rough (R). The smooth pneumococcus kills mice while the rough pneumococcus do not kill mice. If Griffith heat-killed the S form of the pneumococci and then injected the mice with it, the mice lived. However, if he made a solution of the heat-killed S form mixed with the live R form, and then rnjected that into the mice, they would die. Somehow the heat-killed S form of lhe pneumococci transformed the live R form into the lethal S form that is able to kill mice.

it was not until the mid-1940's that this phenomenon was understood. Oswald
-{very, Colin Macleod, and Maclyn McCarty published a landmark paper based rn Griffith's findings that stated that DNA was the transforming factor (genetic material) and not protein as others had believed.
Bacteriophage

.r

the intervening years it was suggested by Roger Herriot that a bacteriophage -rke the T2 virus is actually a core of DNA surrounded by a protein coat and that

somehow injects its DNA into a host cell causing that cell to be ::ansformed to produce more phage progeny. This hypothesis was tested by \lfred Hershey and Martha Chase in 1952. Their findings tumed out to be yet confirmation that DNA is indeed the genetic material. 'r-rother

re virus

Figure 9.27

lhe T2 bacteriophage consists of a head which carries its DNA. Surrounding the :ead is a protein coat. Stemming off of the head is a tail with fibers that have the

lljs l\A

::ility

to recognize specific markers on a given host, such as the bacterium E. coli. is shown in Figure 9-27. Once the phage attaches to its host it then injects its into that host via its tail mechanism (which acts like a hypodermic needle).

,:: the time that Hershey and Chase were doing this experiment they were not .;re which portion of the phage was being-inlected into the host cell. Was it the
::otein from the protein coat or was it really the DNA? Hershey and Chase had .::ess to various radioactive isotopes, including 32P and 35S. Other radioactive j-rtopes include 14C and 3H. Another non-radioactive isotope is 15N. They :=asoned that the phage DNA could be easily labeled with 32P is because of the ::'osphate atom in the DNA backbone. DNA does not contain any measurable .::rount of sulfur. They also knew that proteins were composed of amino acids .:.1 two amino acids had sulfur atoms (Cys and Met). Amino acids do not :: rtain any measurable amount of phosphate in their structures. Therefore, they 35S. - - rld have proteins labeled with

llP and 35S" After a brief period of time the infected solution was agitated in a
in order to separate the phage from the bacterium. The solution :s centrifuged into a supernatant and a pellet. The bacteria were collected from :: pellet and analyzed for either 32P or 35S. It was discovered that the bacteria : :riained a high percentage of 32P while the phage protein in the supernatant ..rtained a high percentage of 35S. This was a conformation of the Avery , ,periment that DNA was the genetic material.
',-arrng Blendor

-. :u1ture of E. coli bacteria were infected

with phage that had been labeled with

:yright

by The Berkeley Review

5()9

The Berkeley Review Specializing in MCAT Preparation

Biology

Genetic Infonnation

Flrnctions of Nucleic Acids


x_

fames watson and Francis Crick deduced the structure of DNA based on an
was a double helical molecule. (Linus pauling, who was utro

in all of the organisms that he examined.

*ortir,g on DNA at that it was a triple heiix.) The base pairing scheme that watson and Crick proposed, adenine (a purine) pairs witf, thymine (a pyrimidine) while guanine (a purine) parrs with (a pyrimidine), war "yiorirr" slpported by evidence from earlier studies by Erwin Chargaff in which he found thatadenine and thymine ratios and guanine and cytosine"ratios were almostl:t
th-e time, proposed

ray diffraction photograph taken of DNA fibers by Rosalind Franklin and Maurice wilkins during the early 1950's. watson and Crick proposed that DNA

electron clouds between juxtaposed bases.

ba;e_ pairing of adenine with thymine and guanine with cytosine Jhe through hydrogen bonding. Adenine and thymiie have two hydrogen between-them while guanine and cytosine have three hydrogen bonds. Note adenine hydrogen bonds l and 5 positions while thynirne hydrogen br "iat via. its 3 and 4 positions. This is shown in Figure s-iaa. similarly) guanin hydrogen bonds via its 2,1, and 6 positions whiL cytosine hydrogenboLas vl its 2,-3, and 4 positions. This is shown in Figure g-2gb. Not ojy do"the hydn bonds between the base pairs help to hold the DNA double helix togeiher, the stacking of the bases also plays a major role due to the interaction of the

f\Sure 9.2a
These bases are said to be complementary to one another. Not only is complementarity between specific bases but the two DNA strands thai cor the double helix run antiparallel to one another. In other words, one DNA s

will run in the 5' -+ 3 direction while the other strand will run in the 3' direction. As the DNA double helix winds around an imaginary axis, grooves are formed. These two grooves are called the majoigroove and minor groove. These grooves (as we will later see) make it much easier certain proteins to bind specific sequences in the DNA duplex. This can in the simplified DNA double helix shown in Figure g-29.
I

Figure 9-29
Copyright @ by The Berkeley Review

5lo

Biotogy

Qenetic Information

Functions of Nucleic Acids

How can we denature a double stranded (duplex) molecule of DNA? one way would be to add a denaturing agent. Recail that when urea was added to proteins, it denatured those proteins. one way to denature DNA would be to change the pH of the solution or even heat the solution. These procedures will first denature the hydrogen bonds in the AT rich regions of the duplex DNA anJ then in the GC rich-regions. why? Because adenine and thymine are held together try only two hydrogen bonds whereas guanine and cytosine are held by three hydrogen bonds. If we just denitured the AT rich region of a l"-g-"lh".t DNA double helix, it would be a partial denaturation. If we continue the denaturation process, we can get complete denaturation of the DNA duplex.
adjust the pH back to normal. \Atrat will happen? one finds that the DNA duplex joins back together--a process cailed renaturition or annealin g. rn

suppose we have a partially denatured piece of DNA and remove the heat or

other words, DNA is a dynamic structure aid not a rigid crystal.

fact, it out that DNA is constantly denaturing a few hydrogen"bonds and turns then reforming those same hydrogen bonds igain--a process"called breathing. in

If we were to heat DNA srowry, we wourd be able to look for a melting iemperature (Trn). The Tp is that temperature at which half of the helica"l structure of the DNA is rost. As we begin to slowly heat DNA there will be a gradual dissociation from double strand-ed DNA to single stranded If the )NA double helix is rich in GC base pairs, it will have"a higher TpDNA. value than fNA with an abundance of AT base pairs. This can be seen iriFigure r-so.
ilow_is this phenomenon detected? one way would be by viscosity separation. -\nother.way would,be by-measuring the reritive absorbance of the"DNi. if you ..ave a structure such as DNA that is held together by hydrogen bonds between :he base pairs, it should be possible to break thor" r,yarog"r-,", by increasing the 'emperature and allow the two strands to separate. This is ,".r"rJibl" nelting and can be observed by folowing the absorbance properties "ullua(at 260 nm) rf the solution in which the meit is taking"place. we can foro# the absorbance of lis melt because of the different absorptlon properties between double stranded DNA_ and single stranded DNA. Dowpk stranded DNA will have a rower =bsorbance (by about 40 to 50%) than sifgle stranded DNA.

(J

(.)

-o !

a
O

&

o
Tm

Figure 9.3O

lopyright O by The Berkeley Review

3ll

The Berkeley Review Specializing in MCAT preparation

Biology

Genetic Information

Functions of Nucleic Acids

why is there a change in the absorbance between double stranded DNA ani
single stranded DNA? The stacking of the bases in the DNA double helix is quite important. This close association of the bases brings about a phenomenon called hypochromicity. Hypochromicity simply means that the absorption of the total molecule is less than the sum of its parts. In other words, each of the base pairs has its own particular absorption spectra, and when we allow all these base pairs to form the double helix we do not rcalize their full absorptive capability.

When light impinges on these stacked base pairs in the DNA double helix there is a light induced electronic transition. In other words, certain electrons will be sent to higher energy levels. Whenever there is an electronic transition, there i.s polarization. Once one of the base pairs becomes polarized, the ability for the a_r: adjacent base pair to become polarized is diminished. Thus, when the bases are tightly stacked, as in the double helix, there is a decreased absorption. This is the basis for this phenomenon of hypochromicity. If you heat the DNA double helir and break the hydrogen bonds between the bases, you will get an increase i:r absorption--an effect called hyper-chromicity.
The Tp of DNA naturally depends on its base composition. We can talk about that base composition in terms of the percentage of GC base pairs in the Dl\A duplex of interest. Most mammalian DNAs are about 50% GC. The GC conteu of the phage T2 is about 35% while the GC content of the bacterium E. coli is about 50%. If this phage were to infect an E. colibacterium, one could determine which DNA in the bacterium is actually the bacteria's own DNA and which is tlre phage's DNA simply by the difference in GC content. Suppose you were to take a piece of DNA and denature it for awhile by addin* heat. At some point you stop this process and begin to reduce the temperatura Would the DNA rehybridize? The answer is yes. If you cool the DNA quicklr you will find that some portions of the single stranded DNA will find each oths and reanneal. However, there will still be a great deal of single stranded DNA that did not renature. If you cool the DNA slowl/, then there will be more ti_u'.e for the single strands of DNA to find their partners and join back together agairu This is shown in Figure 9-32. The more complex the DNA, the less perfect will b'e the renaturation process.
Single Stranded

[n

til

tu

Gm

qW

rw

dro

,&m

o
C)

DNA
Cool

Lr
U)

Quickly
Double
Stranded

Cool

&

DNA lncrease

Slowly

Stop

Decrease T

Figure 9-52

Once DNA was proposed to be double helical in nature, the question off replication immediately surfaced. How is it that DNA can replicate itself? There
Copyright @ by The Berkeley Review

512

The Berkeley Revic Specializing in MCAT Preparatior

Biology

Genetic lnformation

Functions of Nucleic Acids

are several possible modes. Two of them are the conservative mode and the semiconservative mode as shown in Figure 9-33.

Parental

DNA

Daughter

v-- v

DNA

!=
(a)
Figure 9-33

GG
\\ /: --:
(b)

\\

,/-:
=-=

Conservative

Semiconservative

in the conservative mode of DNA replication the parental strands

remain

Crick) the two parental strands separate and two new daughter strands are svnthesized off of the respective parental template. The parental strands do not :ejoin but rather remain with the newly synthesized daughter strands. This is shown in Figure 9-33b. ./

together after replication of two new daughter strands. This is shown in Figure 933a. In the semiconservative mode of replication (proposed by watson and

medium with heavy nitrogen, 15N. When new DNA is synthesized these nitrogen atoms (14N o.11551; will be incorporated into the structure of that molecule. The light DNA (containing 1aN) could be separated from the heavy DNA (containlng 15N) by the use of density gradient sedimentation. "qnilibti.r-\ gradient ranging from 1.66 to'1,.76 g/cm3 was established in a concentration of cesium chloride (CsCl). The density of the house and garden variety DNA is about 1.7 g/ cm3. Two bands were clearly visible as shown in Figure 9-35a. This ivas the starting point{or their experiment.
\4eselson and Stahl grew some E. coli a medium with 15N for many generations. The DNA in these bacteria thus contained 15N in their DNA. Once they had a batch of E. coli will all 15N in their DNA they started the experiment. At time zero they switched the 15N to 14N in the growth medium (see Flgure 9-34).

It turns out that DNA is rcplicat{semiconservatively, as watson and crick had hvpothesized. This was proven by Matthew Meselson and Franklin stahl in 1957. Meselson an{ stahl grew the bacteriurn E. coli in an ordinary medium containing laNH+Cl. The nitrogen in this compound, 14N, is the regular house and garden variety nitrogen. They also grew a different batch of E. coli tn a

Copyright @ by The Berkeley Review

3r3

The Berkeley Review Specializing in MCAT Preparation

Biology

Genetic Infororation

Flrnctions of Nucleic Acids

ttN

'o*/''*O
/\

O l'

E. coli

o."?i"',,""

toVtt* toN toN toVtt*


Figure 9.54

OOO

Now, in each subsequent generation the E. coli would be utilizing laN instead r5N. Analysis of the DNA afte-r one generation of growth (E. coli replicatic revealed a hybrid DNA band(lap1lsp; between thaiof the 15N DNA band a the 1aN ONA band. This is shown in Figure 9-35b and implies semiconservative mode of DNA replication. If the mode of DNA replication been conservative you would still expect to see two bands just like that shown i Figure 9-35a. Why? Because the heavy band would be from the parental s while the light band would be from the daughter strands (see Figure 9-33a).

toN

toN

toVtt"
ttN

toVtt^

@
(a)
Figure 9-55

E
(b)

What would the bands look like after two generations of E. coli Again, if this were a conservative mechanism, you would still see two bi One would be heavy 115N; and the other would be light (14N), except that there would be three times as much 14N as 15N. tf the mode of replication semiconservative, then you would find one all 1aN band and one hybrid 151 band as shown in Figure 9-35c. Thus, we find that the mode of DNA repli
is indeed semiconservative.

Copyright @ by The Berkeley Review

3t4

Biology

Genetic Information

DNA Polymerase

ul$

iffiul#ffic$fip$

DNA replicates semiconservatively. rn 7955 Arthur Komberg and his colleagues incubated extracts of E, coli with radioactively labeled deoxyribonucleoside triphosphates at the a-phosphate position. They discovered that a very small amount of nascent DNA was synthesized. The enzyme that catalyzes this :eaction was called DNA polymerase I (since it was the first such enzyme liscovered). -\'hat is a deoxyribonucleoside triphosphate? Let's consider the structure of J.eoxythymidine as shown in Figure 9-36a. There are a few important features to ::ote about this molecule, so we will consider them one at a time. The sugar in lNA is deoxyribose, and in particular it is B-D-2'-Deoxyribose. The "deoxy" :erm simply means that the ring structure lacks an oxygen atom at the 2'-carbon. lhe C-1 carbon of the ribose ring is bonded to either the N-9 nitrogen of a purine iase or the N-1 nitrogen of a pyrimidine base by an N'glycosidic linkage. This -:r'ukage was formed via a dehydration reaction which means that it can be :';drolyzed. In the case of deoxythymidine it is bonded to the N-1 nitrogen of the ::trogenous base thymine. This structural unit, the ribose ring bonded to either a ::.rrine or a pyrimidine, is called a nucleoside (note the "side" ending). In DNA :rere are four nucleosides: deoxyadenosine, deoxyguanosine, deoxythymidine, urd deoxycytidine.
(b)

Thvmine ll "C

-- \c -cH. "-"til o:c \ ..c-H N


4',

O-P-O. ,O tot' -.! U ./\

il

Acid Anhydride ^ Linkage


H-N
I

o/

"!a
o o

,,,

ftD-2'-Deoxyribose

Phosphomonoester

;v"
HOH

o-c -*-"-n
$
I'

-" -"-"t, il
N-glycosidic
linkage

Linkage

HOH
Deoxythymidine
(a nucleoside)

Deoxythymidine- 5'-triphosphate (a nucleotide which can be abbreviated as dTTP)

ffiEure 9-56

;",-e

add a phosphate group to the Slhydroxyl function of the ribose ring of any
i

,: :,.!r nucleosides, we will form a phosphate monoester bond (Figure 9-36b).


lhe name of our molecule now changes from a nucleoside to a nucleolic[g (note :e 'fide" ending). Be careful of what you mean when you use the "side" and the ':lie'endings. For example, we could name one of these molecules as a ,lr*,rlwribonucleoside$'-monophosphate (or as a deoxyribonucleotide). There m';l also be two or three phosphate groups attached to the S'-carbon of the
'T:5,:se

ring. Recall from organic chemistry that two phosphate groups are to each other by a phosphoanhydride linkage. If we added two :'!:ti:sphates to our molecule, we would call it a deoxyribonucleoside-5'ru,nt'-osphate. If we added three phosphates to our molecule, we would call it a
unfra:hed
i iru'-ight @ by The Berkeley Review

315

The Berkeley Review Specializing in MCAT Preparation

Biology

Genetic Information

DNA Polymerasc

deoxyribonucleoside-5'-triphosphate. This can be abbreviated as dNTp. The four deoxyribonucleoside triphosphates (dNTps) of interest to us are dATp, dGTB dTTP, and dCTP.

I adds about 20 deoxyribonucleotide residues (e.g., dATp, dGTP, dTTP, or dCTP to the 3lhydroxyl function of a pre-existing DNA strand. what this means is that a primer is needed in order for DNA polymerase I to function. The dNTPs are called for by a DNA template and aie added to the newly synthesized DNA chain in 5' -+ 3' direction (which means the DNA template is read in the 3' -+ 5'direction). The incoming dNTp will hydrogcr bond with its complementary base and then there will be a nucleophilic attack by the 3'-hydroxyl of the primer strand on the o-phosphate of the incoming dNTp. A phosphodiester bridge is formed between the a-phosphate of the incoming
DNA Polymerase
dNTP and the 3'-hydroxyl function of the ribose ring of the primer strand phosphodiester bond
Pyrophosphate is released and subsequently hydrolyzed in order to help drive this reaction to completion. This is shown in Figure 9-37. Note that the

complementary to its respective base on the template strand.

will only be formed if the incoming dNTp is

Primer

I HuCr
T E

Bo*"

l,r"{

M
P

L
A
T E

OH O-P=O
I

H
Bare

T
E

M
P

L
A T E

-H
o
I

HOH
Figure 9-57

Copyright @ by The Berkeley Review

3t6'

The Berkeley Specializing in MCAT

Biotogy

Genetic Information
,,.1r ,,..._.,,1:u.....,.

DNA Reptication

a right-handed helix with the iurine and pyrimidine bases arranged on the inside of the heli* while the deoxyribose and phosphate moieties are arranged on the outside of the helix. The DNA double helix can be represented as shown in Figure 9-3ga or in Figure 9-3gb. Both representations will give us similar information

...,.,,,.,'.iffilflllilitiiffi DNA is-_ composed polydeoxynucleotide strands arranged in an :j_ l-o antiparallel fashion. DNAis uzually

D'N

c##"
trH:::
Single

bp = base pairs (A=T or G=C)

(a)

(b)

Figure

9.iB

containsa major and a minor groove (see Figure 9-3ga). proteins that recognize specific DNA sequences can gain access to the DNA double helix via the riajor
groove.

-\ great deal of structural information about the DNA double helix can be gained rrom X-ray analysis. For example, there are two forms of DNA which can Le in a right-handed helix. They are the A-DNA and the B-DNA (which was proposed bv watson and Crick in 1953). Recall that we have mentioned that the B:DNA

There are a number of bonds in the DNA polymer about which rotation can For example, the backbone of the DNA polymer can be rotated about 6 :rccu_r. bonds in each monomeric unit. Many of the differences between the A-DNA and the B-DNA arise from the different conformations of the ribose ring.
Recall that the ribose ring in DNA can also be called B-D-2-Deoxyribofuranose. These furanose rings are not planar. They can pucker. In the A-DNA the minor

Sroo-ve essentially vanishes due to s specific puckering of the furanose ring. This results in the base pairs of the A-DNA helix being tilted away froin the perpendicular axis by about 19 degrees. However, becalse of a different type of puckering, the B-DNA has its bases arranged perpendicular to the axis of the relix, thus allowing foq. the distinct characteristics of the major and minor grooves as shown inlig;re 9-3ga.

The type of helix that is found in A-DNA is also found in regions of double stranded RNA (e.g., involving hairpins) and in RNA-DNA hybrids.

Copyright @ by The Berkeley Review

317

The Berkeley Review Specializing in MCAT preparation

Biology

Genetic Information
NHz

DNA Replication

Adenine#i
N-Glycosidic

o-cH2

Bond

Conformation Left Handed Helix, Z-DNA Purine is Syn, Pyrimidine is Anti


Syn

Anti Conformation
Right Handed Helix Either A-DNA or B-DNA

(a)
Figure 9.39

(b)

A third type of DNA helix involves rotation about the N-glycosidic bond that connects the furanose ring to the base. This type of DNA is called Z-DNA
(because the phosphates in the backbone zigzag due to a repeating dinucleotid.e

unit rather than a mononucleotide unit). Z-DNA is a left-handed helix and when the pyrimidine bases and the ribose units are far apart it is in an antil conformation while it is in a syn conformation when the purine bases and the ribose units are close together. This is shown in Figure 9-39a. The N-glycosidrc bonds in both A-DNA and B-DNA are in the anti conformation. This is shown in
Figure 9-39b.

DNA does not have to be linear, rather it can be covalently joined at its ends forming a circular structure. For example, the DNA found in mitochondria an'j. in the chloroplasts of plant cells is circular. The topology of both linear and circular DNA is rather interesting. DNA is typically right handed. If we were to twist this DNA molecule around its own axis in the right-handed direction, tha:
we would introduce into that double helical structure a phenomenon known a-" positive supercoiling. Conversely, if we were to twist this DNA molecule around its axis in the left-handed direction, then we would introduce negative supercoiling. Supercoiling greatly changes the overall form of the DNA not onkby making it more compact but also by altering accessibility to the major and

,1

il

"t

(!

minor grooves.
The number of times that one DNA strand can be wound around another DNA strand is referred to as its linking number (L). Topoisomers are DNA molecule:

ffi

s!

that differ only in their linking number. The degree of the linking number in DNA can be altered by enzymes called topoisomerases. Type I topoisomerases reversibly cleave one strand of DNA and relaxes negatively supercoiled DN-{ while Type II topoisomerases (e.g., DNA gyrase) reversibly cleaves both strands of DNA and adds negative supercoils.

t
a

A ff

tu

In order to introduce a supercoil into a DNA double helix it costs energy. For example, DNA gyrase is a topoisomerase (Type II) found in E. coli which converts the potential energy of ATP into the torsional energy of a negativeh supercoiled structure. It turns out that negatively supercoiled DNA prepares the
Copyright @ by The Berkeley Review

318

The Berkeley Review Specializing in MCAT Preparation

Biology

Genetic Information

DNA Reptication

DNA duplex for processes such as replication, recombination, and transcription where the separation of the two helical strands is required.
In 1958 Arthur Kornberg discovered the enzyme DNA polymerase I which plays in the replication and repair of DNA. This enz)rme adds about 20 deoxyribonucleotide-residues 1e.g., dRTt, dGTp, dTTp, or dcrp (which collectively can be called dNTps)) to the 3'-hydroxyl function of a pre-existing DNA strand.
a crucial role
Primer

".J

'2"{

primer
Base

----- Brse

2Pi
T E

('on _ \ro (]-P-o

H
r

M
P

L
A
T E

)'H
o-P-O
o
I

"rJ

"'i"!

Base

T
E

M
P

L
A T E

I HII

Base

---..

Base

t/"!

-T" "ro
HzC

Base

HOH

Figure g-4O

ivhat this means is that a primer is needed in order for DNA polymerase I to runction. The dNTP's are called for by a DNA template and are added to the newly synthesized DNA chain in 5'-+ 3' direction (which means that the DNA template is read in the 3' -+ 5'direction).

:ing of the primer strand. Pyrophosphate is released and subsequently

will hydrogen bond with its complementary base and then lhere will be a nucleophilic attack by the 3'-hydroxyl of the primer strand on the n-phosphate of theljncoming dNTPi phosphodieiter bridge is formed between :he cr-phosphate of the incoming dNTp and ihe 3'-hydroxyllunction of the ribose

The incoming dNTP

hydrolyzed in order to help drive this reaction to completion. This is shtwn in Figure 4-40. Note that the phosphodiester bond wifl only be formed if the ncoming dNTP is complementary to its respective base on the template strand.

\ laboratory procedure called the polymerase chain reaction (pcR) is having a significant impact on what can be done with DNA. suppose we have a relativ6ly iong piece of double stranded DNA as shown in Figure 9-41. within this DNA is a portion that we wogld like to examine. However, we do not have very much of fiis DNA available to us. The idea is to amplify the segment of pNA we are nterested in so we can studv it.

Polymerase Chain Reaction

Copyright @ by The Berkeley Review

319

The Berkeley Review Specializing in MCAT preparation

Biology

Genetic Information
We want to examine this area of the DNA.

DNA Replication

Figure 9-41 The concept of the polymerase chain reaction would only have been possible after the discovery of one of the organisms that grows in hot springs. These organisms (e.g., bacteria, fungi, algae) are thermophilic--they love heat. ln particular, there is a bacterium called Thermoaquaticus (Taq) which has a DNA polymerase referred to as the Taq DNA polymerase. This polymerase is stable at temperatures up to 90 "C and does not denature at that temperature. With this polymerase in mind, let's amplify the DNA in Figure 9-41.
Suppose we know some base sequences as outlined in Figure 9-42a.If we were tc somehow denature that area of the DNA, we could manufacture complementan-

base sequences with a free 3'-OH function at the end. These complementansequences are made such that their 3'-OH functions point inward. In other words, we can synthesize a primer as shown in Figure 9-42b.

How can we do this? If we heat this DNA to about 90 'C, the ends will separate first. We can then add the short chain DNA primers that are complementary to the base sequences of that portion of the DNA that we already know. If we cool the solution to about 50 'C, our primers will hydrogen bond to their complement areas as shown in Figure 9-42b.
We want to examine this area of the DNA.

.C.T.
.G.A.
(a)
I H"ot on,l cool in the pr.r"n". of excess artifical nrimers

T.A-C.T.
ArT-G-A. I

lt

ll|
Primer

3' -C-T-T.C
Primer

I
s'

.i-i-i.C-on

T-A-C-T.5'
3'HO-.T-A-C-T. llrl A-T-c-Ar

5' .G-A-A
(b)

J'

Figure 9-42
Once we have our primers in place we can then add dNTP's and the Taq DNA

polymerase. This enzyme


Copyright @ by The Berkeley Review

will add

the dNTP's as shown

in Figure

9-43

32o

The Berkeley Revien Speciatizing in MCAT Preparation

Biology

Genetic Information

DNA Replication

Remember, the Taq DNA polymerase is stable at high temperatures (around 70 'C). By this time we have twice as much DNA as we did before.

3' Primer

TrA-CrT3'HO_-T-A-C-Ttrrl A- T-G-AI

5t

Primer

.G-A.
t"o o*o
Polymerase

dNrP's

I
'' :E:i:l:
Primer

A-C-T-

5,

.G.AFigure 9-45

3'

But we still want to amplify our DNA in order to get a better yield. How do we amplify the DNA that we have produced in Figure 9-43? we heat the DNA to 90 'c in the presence of our primers (which we have in excess). The DNA strands will separate and the primers will hydrogen bond as before. we now have four single strands of DNA with their respective primers. This is shown in Figure 944. These four strands correspond to their respective strands from Figure 9-43.

3' 5'

-c- T- T-C-i- [- A-C-o" r'


Primer

Original DNA

TrA{rTr

5,

5, -G- A- A-c

***rr*-**o*rrJ-*ti$ll{*li*ti{-

c* A { 3'HO_.T-A-C_TAa.

tttt

T.\

3'
5'

Primer

3'

\-

c $ T s 1 *' g,-r-rr*rrrrr***P*Sitili{{l\Tili{ Primer

A{-T.
A-T.G.A. tltt
Primer

5'

5'

-i-tr-A-C-oHs'
3' 5'

S'.G-A.A-G
3' HO-

-T-A{-T.

Figure 9-44 Taq DNA polymerase in the presence of the dNTP's will once again synthesize new DNA. This is shown in Figure 9-45. At this point we have now made four
Copyright @ by The Berkeley Review

321

The Berkeley Review Specializing in MCAT Preparation

Biology

Genetic Inforrnation

DNA Replication

times as much DNA. This process can be continued indefinitely to amplify the DNA.

3'-c-T-T-C tt 5' -G-Ar


5' 3'

\\\\t\\\\\\\\\a\'
,,111111t,,,,,,,,,t,,,,,,,,r!rrrrrr*arrrrrrrrrrrtrrrrrrrrrrrrrrrrrrrrr,,,,,,,,,,,,,,,,,,,,,,,,,

f Tt - A-i

tlf : i,
5' 3'

3' *-Qr.

5t -G- A- d-Q trttttttrtrtrrrrrrrrrrrrurluunrnrr'lrrrrrrrr

lllr

frJxQsrmnrrrrurrrrrrsrllrrnlrrrru

T-A-C-T.
,,.,.,,,,,,,,.,,,,..........l|trr,.-

5'-G-A-Arc-

3, .rllllll,,rlrrrr,,,,,,,,,,rtttttrtrrrrllrrrrrrrtt*t.trrrrrrrrrrr"rrrr.rrrrrrrr,,,,,,,,,,,r,,,,,,,,,,,,,{-l-J-{Figure 9-45

A-T-G-A.

3,

t'

one of the problems with this process is that Taq DNA polymerase does not have 100% fidelity. Once in awhile an incorrect baies will be added. Flowever, the number of errors will be quite small compared to the quantity of DNA tha: can be obtained. A general temperafure cycle for the polymerase chain reaction is shown in Figure 9-46.
Condition for denaturation

o 920
o
a
d H

il

o.

F
Time (minutes)

il

Figure 9-46
--wnu'---..-.-..-t

are some of the applications of the polymerase chain reaction? one application involves analysis of the DNA from organisms which have been deceased for a long time. For example, the tissue of ancient Egyptian mum:nies can be analyzed for their DNA content as well as ancient bones from various burial mounds throughout the world. DNA from these organisms can then be compared with the DNA from modern organisms can to see if there has been a significant change (evolution).

Unwinding of Supercoiled Parental DNA The synthesis of new daughter DNA is coupled to the unwinding of parentarn. DNA at sites called replication forks. DNA replication does not begin random-E but rather at sites referred to as an origin (or ori for short). For example, in E. ;:rfu DNA replication will begin at a locus called ori C. In order for the DNA doubre
Copyright @ by The Berkeley Review

322

The Berkeley Revier Specializing in MCAT Preparatin

Biology

Genetic Information

DNA Keplication

helix to unwind, DNA gyrase negative supercoils ahead of the advancing replication fork. This is shown 1d{s in Figuie S-aZa.itNhy? As the replication fork i! initially unwound, positive supercoils are introduced thus making DNA separation rather difficult.)
(a)
3',

DNA Gyrase

DNA
adds

negative supercoils

Gyrase

<-o"oJl..iion--1>

Origin of

Helicase unwinds

DNA (ATP-driven)
Template

\)1
Figure 9.47

DNA sinele-srranded

bindin! proreins

Once the negative supercoiling is introduced, a protein complex called a helicase binds to the ori site and catalyzes the ATp-driven unwinding of the duplex DNA. single-stranded binding proteins then stabilize the unwound portion of the parental DNA. This is shown in Figure g-47b. \Ahen a replication fork has been created, both strands of the parental DNA serve as templates for the synthesis daughter DNA. Becausl DNA is replicated ?f.tl"ry semiconservatively (proposed by watson and Crick in 1953 and proved by Meselson and stahl inr9s7), and because the DNA strands are antiparalel, thl synthesis of new daughter DNA must be in the 5' --+ 3' direction for one strand but in the 3' -+ 5' direction for the other strand. Synthesis of one daughter strand Ln the 3'-+ 5'direction seems to be a paradorbecause of the faci that DNA polymerase will only synthesize new DNA in the 5'+ 3'direction. This is shown
Ln

Figure 9-48.

"Appears" to grow

3'-r

5'

Copyright @ by The Berkeley Review

323

The Berkeley Review Specializing in MCAT preparation

Biology

Genetic lnformation

DNA Replication

Continuous and Discontinuous DNA Synthesis This problem of one daughter strand of DNA paradoxically growing in the 3' -+
5' direction was tesolved by Reiji Okazaki (ckca 1969).

{-

Parental

As the replication fork proceeds in one direction, one daughter strand

is

nN.q,

synthesized continuously in the 5'-+ 3'direction. This daughter strand is called the leading strand. The other daughter strand is called the lagging strand because it is synthesized discontinuously. Even though the lagging strand is synthesized discontinuously, it is synthesized in the 5' -+ 3' direction (as shown in Figure 9-49).

11-

Primases How is the lagging strand synthesized? when the DNA template has been exposed at the replication fork, nascent DNA cannot be made until a primer is
synthesized.

Figure 9-49

Pri

Figure 9-5O

An enzyme called a primase (complexed to other polypeptides to form arL aggt"gut" called a primosome), is a specific RNA polymerase that synthesizes about 5 nucleotides of RNA which are complementary to one of the DNA rvifl templates. DNA polymerase cannot start the synthesis of nascent DNA (we
come back to this point later)' This is shown in Figure 9-50'

DNA PolYmerase Itr lloloenzYme Once the primer has been created, the enzyme DNA polymerase III holoenzymc enters the scur,e and catalyzes the synthesis of about a thousand phosphodiester bonds (i.e., about one thousand dNTP's have been used) before it dissociates from the DNA temPlate.

.W}yisDNApolymeraselllusedinsteadofDNApolymerasel?D\A D\A " -{olymerase III is quicker and it can add more dNTP's to the growing
itrina.
Recall that we mentioned that DNA polymerase dNTP's to a pre-existing DNA chain'

I only

adds about lO

This segment of RNA primer and newly synthesized DNA is called an OkazaE fragmeit. As more Okazaki fragments are synthesized in the 5' --> 3'direction flt
Copyright @ by The BerkeleY Review

324

The Berkeley Specializing in MCAT Pre

Biology

Genetic Information

DNA Reptication

eventually leads to overall growth of the daughter DNA in the 3' -+ 5' direction. This is shown in Figure 9-51.

Growing
Replication Fork Hel

3',

DNA
Gyrase

DNA Polymerase
Holoenzyme

III
Fragment

Primosome (Primase)

ok
Fragment

Parental

3', \5'

Figure 9.51

DNA Polymerase I and DNA Ligase DNA polymerase I has a 5' -) 3' exonuclease activity that allows it to remove the short segments of RNA primer (starting from the free end). \A/hen the primer has been removed, DNA polymerase I then adds deoxyribonucleotides to the free 3'hydroxyl function of the chain undergoing elongation.

Growing
Replication Fork

DNA
Gyrase
DNA Polymerase I uses its 5'ro 3' exonuclease activity to remove RNA primer

DNA Polymerase I next


Parental

DNA
Nascent
3',

adds dNTP's to where RNA primer once was

located.

DNA

nue ligase.ioins/
the fragments.

Figure 9-52 once the gap that was once occupied by the RNA primer is filled with dNTps,

DNA ligase will join the free 3'-hydroxyl group of new daughter DNA just

synthesized by DNA polymerase I with the S'-phosphate group of the okazaki fragment just ahead of it. Remember, DNA ligase only joins chains of DNA that
Copyright @ by The Berkeley Review

325

The Berkeley Review Specializing in MCAT preparation

Biology

Genetic Information

DNA Replication

are double-stranded and not two molecules of single-stranded DNA. This is shown in Figure 9-52. In bacterial cells the source of energy for this reaction comes from NAD+ while in animal cells it comes from ATP.

Replication Forks (revisited) The main chromosome in the prokaryotic bacterium E. coli is circular (there are also minor chromosomes called plasmids). Recall that we have mentioned that in the E. coli replication starts at a unique site called ori C (an origin of replication)In Figure 9-47a we drew a segment of DNA and located the origin of the replication fork. In Figure 9-47b we proceeded to draw a typical replication fork Flowever, what we did not show was that replication can be bidirectional. In other words, as the parental DNA is opened, replication will proceed at two replication forks simultaneously. This can be seen in Figure 9-53.
Parental DNA

6-\ o*r, V'


I t
/il
+

Origin of
Replication

DNA Gyrase adds negative


supercoils

Fork Origin

{-

Growing
Replication

Fork

Replication{
Fork

Growing

Growing Forks

Replication

Helicase

\**y'

6;.r"N

Figure 9.53

)
1

Origin

/
5'

Eukaryotic chromosomal DNA is a linear polymer and undergoes active Dl replication during the S-phase of the cell's life cycle. Because the DNA eukaryotic cells is quite long compared to E. coli's DNA, it is repli bidirectionally from many origin sites as shown in Figure 9-54. This ensures there will be rapid replication of the genome.

3',

Figure 9-54

Copyright @ by The

Ber\eview

326

The Berkeley Specializing in MCAT

Information
L5 Passages

Genetic

100 Questions

Passage Titles

Questions

il. ilI.

I.

IV.

VIU. IX.

vI. vII.
X.

V.

XIII.
XIV.

xII.

XI.

HbA/HbS Gel Electrophoresis Gregor Mendel and Inheritance Incomplete Dominance and Codominance DNA Replication Color Blindness Alleles and Coat Color DNA Calculations Semiconservative DNA Replication DNA Structure Mitochondrial DNA (mtDNA) Viral Complementation Atrial Natiuretic Peptide Experiment

1-5 6-11 12-11 18-23 24-29 30-36


31 -43

Griffith's Pneumococcus
Drosophilia eyeless Gene Meiotic Nondisj unction

xv.

44-50 51-58 59-65 66-72 73-79 80-87 88-94 95 - 100

REKI{EIEY

l)n.n,.v.r.ilw'

Speciahztng in MCAT Preparation

Suggestions
The passages that follow are designed to get you to think in a conceptual manner about the processes of molecular biology at the organismal level. If you already have a solid foundation in molecular biology, many of the questions you read here will seem to be very straight forward and easy to answer. But if you are new to the subject or if you have not had a pleasant experience with molecular biology in the past, some of them might appear to come from the void that spreads out beyond the Oort field at the edges of our solar system.

Pick a few passage topics at random. For these initial few passages, do not worry about the time. just focus on what is expected of you. First, read the passage. Second, look at any diagrams, charts, or graphs in it. Third, read each question and the accompanying answers carefully. Fourth, answer the questions the best you can. Check the solutions and see how you did. \A4rether you got the answers right or wrong, it is important to read the explanations and see if you understand (and agree with) what is being
explained. Keep a record of your results.

passages and

After you feel comfortable with the format of those initial few passages, pick another block of try to do them in one sitting. Be aware that time is going to become important. On average,

you have about 1 minute and 15 seconds to complete a question. Be creative in how you approach this next group. If you feel comfortable with the outline presented above, fine. If not, then try different approaches to a passage. For example, you might feel well versed enough to read the questions first and then try to answer some of them, without ever having read the passage. Maybe you can answer some of the questions by just looking at the diagrams, charts, or graphs that are presented in a particular passage. Remember, there are many effective learning styles. You need to begin to develop a format that works best for you. Keep a record of your results.
The last block of passages might contain at least a few topics that are unfamiliar even to those who know a good deal about molecular biology. Find a place where the level of distraction is at a minimum. Get out your watch and time yourself on these passages, either individually or as a group. It is important to have a feel for time, and an awareness of how much is passing as you try to answer each question. Never let a question get you flustered. If you cannot figure out what the answer is from information given to you in the passage, or from your own knowledge base, dump it and move on to the next question. As you do this, make a note of that pesky question and come back to it when you have more time. When you are finished, check your answers and make sure you understand the solutions. Be inquisitive. If you do not know the answer to something, look it up. The solution tends to stay with you longer that way. (For example, what ls the Oort field, anyway?)

The estimated score conversions for 100 questions are shown below. At best, these are rough approximations and should be used only to give one a feel for which ballpark they are sitting in.

Section lX Dstimated Score Conversions


Scaled Score

Raw Score
80
'70

>13

100

n-12
9- l0 7 -8 5-6
<4

- 79

60-69 50-59 40-49

0-39

Biology
Passage

nbAlllbs Gel Electrophoresis


Protei n
sample

Practice Passage I

(Questions 1-5)

Protei n
s ample

Human adult hemoglobin (HbA) is a tetrameric


protein, which has a molecular weight of approximately 65,000 amu. This protein consists of two (identical) aglobin polypeptide chains containing 141 amino acids each, and two (identical) p-globin polypeptide chains

H
tffii W

E
U{
W
w

Cathode C

*w

containing 146 amino acid residues. Each globin


polypeptide contains one iron atom (55.8 amu), and each iron atom can complex with one molecule of molecular oxygen (32.0 amu). Large proteins like hemoglobin can have many acidic and basic groups, giving the molecule both negative and positive charge. These types of molecules are called polyampholytes, and they will always have an isoelectric point (pI) where the net average charge is zero. The pI of HbA is 6.9 An ampholyte that has a net charge of zero is called a zwitterion.
Separated

NU tt
tto Ilo tt

ll I
5

components

Anode O
Lane

JL

ll3 |# ll s
o

Lane2

Figure 2. Separation of protein samples by PAGE.

Sickle-cell hemoglobin (HbS) and the wild-type HbA

differ from one another by the substitution of a single amino acid residue at position 6 in the B-globin chain. Individuals with normal adult hemoglobin have the
hemoglobin composition ozAFzA. Individuals with sickle-

cell trait have both HbA and HbS and a hemoglobin


with sickle-cell disease have just HbS and a hemoglobin composition of ozABzs. The relevant portions of the two beta chains for both HbA and HbS are shown in Figure 1:

1.

What is the approximate percentage


hemoglobin?

of iron rn

composition

of a2Ap2A and cr2AB2s. Individuals

A. B.

0.09

7o vo vo
7o

c.

D.
HbA HbS
H3N-Val-His-Leu-Thr-Pro-Glu-Glu-LysH3N-Val-His-Leu-Thr-Pro-Val-Glu-Lyso

0.17 0.26
O.34

Bl 2 3 4 s 6 1
Figure
1. The amino acid sequence

HbS contains a valine residue at the p6 position, while HbA contains a glutamate residue.

ofthe B-chains for both

HbA and HbS.

This difference makes it easy for different proteins to be separated from one another by a process called polyacrylamide gel electrophoresis (PAGE), diagrammed in Figure 2. The gel is formed from the polymerization of acrylamide and N,N'-methylenebisacrylamide and then suspended between an upper and lower buffer solution. The buffer solution usually has a pH of about 9 and is the same in both reservoirs, as well as in the gel itself. Different protein samples are placed in designated wells in the gel, and a direct currnt of 300 volts is applied for about 60 minutes. After separation, the protein bands in the gel can be visualized by a variety of techniques (e.g., staining or UV inadiation). Electrophoretic separation is quite important in clinical laboratories, as it can be used to identify abnormal hemoglobin patterns.
Copyright
@

@Yfl
HlN- C- CH3CCH

H3N-C-C-O

oifl
I I

o
O

CH,
CH,

CH:

O=C-O
Glutamate (Glu) pKa= 4.4

l-o

Valine (Val)

In HbS, the pI is expected to be:

A. B. C. D.

equal to the pI for HbA. slightly greater than the pI for HbA. slightly less than the pI for HbA. representative of a nonzwitterionic species.

by The Berkeley Review

329

The Berkeley Review Specializing in MCAT Preparation

Biology
3.
Lane 2 of Figure 2 represents:

IlbA/HbS Gel Electrophoresis

Practice Passage I

If HbA and HbS undergo gel electrophoresis at a buffered pH of 9, then the protein band shown in

A. B. C. D.

HbA, because of its movement towards the


negatively charged anode.

HbS, because of its movement towards the positively charged anode. HbA, because of its movement towards the positively charged anode. HbS, because of its movement towards the
negatively charged anode.

4.

Changes

in the nucleotide

sequence

If each amino acid in a protein is coded for by three bases in DNA, then the difference between HbA and HbS must be the
represent mutation.

of

DNA

result of a:

I.

u.
A.
B. C. D.

III.

base-substitution mutation. base-addition mutation. base-deletion mutation.

I only II only III only

II and III only

5.

Sickle-cell trait is a genetic disorder inherited as an autosomal recessive disease. If a man has sicklecell trait and marries a woman who is heterozygous for a type of hemoglobinopathy referred to as HbC, what is the probabiliry thar their first child will nor inherit any genetic disorder related to hemoglobin?

!
rN

A.
B.
C.

0.25

0.s0
0.75
1.00

D.

ffi

il il d

,fl

&

Copyright @ by The Berkeley Review

330

The Berkeley Specializing in MCAT

Biology
Passage

Gregor Mendel and lnheritance 6.

Passage Il

II

(Questions 6-11)

Mendel first allowed plants of a given variety to


produce progeny
generations in order to:

The first quantitative studies of inheritance involved the garden pea and were carried out by Gregor Mendel. His choice of the pea was fortuitous, because a large number of true-breeding varieties (uniform from one generation to the next) was available. Mendel selected seven easily distinguishable traits of these plants (e.g., purple versus white flowers, and round versus wrinkled
seeds).

by self-fertilization over many

A. B. C. D.

carry out an experimental cross. assure himself the plants were of the truebreeding variety. observe segregation of alternative traits among
progeny. carry out a reciprocal test-cross.

Pea plants contain both male and female sex organs. This was also advantageous to Mendel, because he could either let self-fertilization take place within an individual flower or perform an experimental cross. A cross involves removal of a flower's male parts before fertilization can occur and introducing pollen from a strain with alternative characteristics.

7.

Mendel first allowed plants of a given variety to produce progeny by self-fertilization for many
generations. He then conducted crosses. Mendel took a pea plant producing white flowers and introduced pollen

Mendel discovered that out of the Fz generation for any trait, l/2 of the individuals were not true breeders. This class of plants, if allowed to self-

pollinate, should produce Fr individuals that exhibit dominant and recessive traits in a ratio of:

from a purple-flowered plant. He allowed the hybrid offspring produced by these crosses to self-pollinate for many generations. Mendel kept a record of the number of
offspring ofeach type and in each generation.
The hybrid offspring from the experimental crosses are termed the first filial (Ft) generation. These offspring always resemble one of their parents. For example, a cross between wrinkled versus round seeds always produce F1 progeny with round seeds. Mendel referred to the Ft trait as being "dominant" over the invisible, "recessive" trait. Mendel allowed individual F1 progeny plants to self-pollinate, and he observed the offspring in this second filial (Fz) generation. He found that some

A. B. C. D.

1:1

2:l
2.5:1.5

3:l

8.

What is the ratio of homozygous to heterozygous


individuals in an Fz generation?

individuals in this generation exhibited the recessive trait.

Out of 7,324 Fz individuals resulting from the roundseeded F1 self-pollinufion, 5,414 had round seeds and 1,850 had wrinkled seeds. The ratio was essentially 3:1.

A. B. C. D.

l:1 3:l
9:3

16:l

Mendel examined all seven traits with alternative forms and in every case obtained the same ratio results: 3/4 of the Fz individuals exhibited the dominant form of the trait, and ll4 displayed the recessive form. He accounted for his data by forming a central assumption that alternative forms of a trait are specified by alternative
alleles, which are discrete.

9.

The ploidy number (n) of a cell refers to how many sets of chromosomes are found in its nucleus. For a human cell in prophase I, the cell is said to be:

In other words, the fgrctors specified by a pair of alternating alleles are separate. This is known as the lqw
ofallele segregation.

A. B. C. D.

haploid (n = 1). diploid (n = 2). triploid (n = 3). tetraploid (n = 4).

Copyright

by The Berkeley Review

331

The Berkeley Review Specializing in MCAT Preparation

Biology
10.

Gregor Mendel and Inheritance

Passage Il

In an effort to determine whether a purple-flowered pea plant is heterozygous or homozygous, a test cross is performed with a white-flowered pea plant. Based on information in the passage, the MOST
individual is:

likely conclusion is that the purple-flowered

A.
B. C. D.

heterozygous,
flowers.

if all

offspring have purple


have

heterozygous,
purple flowers.

if half of the offspring if all offspring have

homozygous,
flowers.

white

homozygous, if half of the offspring have purple flowers and the other half have white
flowers.

11.

The following diagram, illustrating a cross of wrinkled and round seeded peas, is inaccurate
because the:
P generation

Round
seeds

Wrinkled
seeds

0
H
0.)

6,)

bo

F!

0
f) oa

o o 4
FD

Round

seeds

Wrinkled

seeds

A.
B.
C. D.

ratio of rcitrnd to wrinkled seeds in Fz


incorrect.

P generation is not made up oftrue breeders.

is

segregation

is incorrectly shown as being

between pods. number of progeny in Fz is incorrect.

Copyright @ by The Berkeley Review

332

The Berkeley Revieu Specializing in MCAT Preparation

Biology
Passage

Incomplete Dominance and Qodominance


12-17)

Passage III

III (Questions

12, Suppose two pink Japanese four-o'clocks

are

classical dominant/recessive pattern, this is not true of every genetic trait. Certain genes follow the trends of
either codominance or incomplete dominance.

Although many genetic traits are inherited

in

crossed. What colors would the offspring, the Fz generation, exhibit?

phenotype that is intermediate between those

When a heterozygote for a particular trait has a of its two parents, the genes exhibit incomplete dominance. Although many examples are known in both plants and
animals, a well-studied example of incomplete dominance

A. B. C. D.

1:2:1,pink:white:red 1:2:l,red:pink:white All flowers either pink or red 2:2 pink:white

in color occurs in a flower called the Japanese fouro'clock. When a homozygous white flower (n) and a homozygous red flower (RR) are crossed, the Ft
generation is entirely pink (Rr). Both the red color and the white color are expressed together in the pink offspring.

13. Which of the following terms BEST fits


definition?

this

Genes that occupy corresponding loci on homologous chromosomes and govern variations of the same characteristic.
A.
B. C. D. Homologous pairs
Pheromones
Segregants

.F'f*-l

.[*l*l

Alleles

Figure 1. Cross of red and white


flowers, producing all pink offspring.

t4. Which type of genetic cross is represented in


Another type of genetic expression is codominance, in Figure I ?

which both alleles are expressed independently and


simultaneously in the heterozygote. For example, a horse with a red coat (RR) is crossed with a white horse (rr). The fbal (Rr) has a mixed red and white coat, in which red hairs and white hairs are interspersed. Horse breeders refer to these crossed breeds as being "roan-colored."

A. B. C. D.

A monohybrid

cross.

A test cross. A dihybrid cross. An inbreeding cross.

A classic example of codominance in humans occurs in the ABO blood group patterns. A and B refer to two
and B glycoproteins are is present, it is expressed. The lack of either protein results in the O blood type,

of red blood cells. The A codominant. If either protein


which is recessive to A and B.

specific glycoproteins'that can be present on the surface

15.

Suppose a gene independent of the one governing the color of a Japanese four-o'clock determined its leaf form in a classical dominant/recessive pattern, with straight leaves (L) being dominant over cudy leaves (l). Given the following cross: RR Ll (Parent

1) X

n Ll (Parent 2)

What is the probability of producing a plant with pink flowers and curly leaves?

A. B.

0.25

c. D.

0.50
0.75 0.00

Copyright @ by The BerkeleY Review

333

The BerkeleY Review Specializing in MCAT PreParation

Biology
16.

lncomplete Dominance and Codominance

Passage III

The paternity of a royal heir is in dispute. The queen

transmitted as a classical Mendelian dominant/ recessive trait, with + being dominant. Is the child
the offspring ofthe king and the queen?

blood. The positive or negative Rh factor

(known to be the baby's mother) has type ABblood, while the king has B-. The baby has A+
is

A. B. C. D.

Absolutely Possibly No Unable to tell from this passage

17. Two people with type O- blood have a child


together. What blood types could their child have?

I.

oA-

II. m.
A.
B.
C.

B.
I only I and II only I and III only I, II. and III

D.

Copyright @ by The Berkeley Review

334

The Berkeley Ke Specializing in MCAT Prepar

Biology
Passage

DNA Replication 18. The replication


a

Passage IV

IV (Questions 18-23)

of the entire E. coll genome requires relatively short time, approximately:


160 minutes. 100 minutes.

4.7

The entire Escherichia coli (E. coli) genome contains x 106 nucleotide base pairs. At special sequences known as replication origins, two replication forks move in opposite directions, with polymerization occurring at a rate of 500 nucleotides per second.

A. B. C. D.
19.

80 minutes. 60 minutes.

The average human chromosome is a single DNA molecule containing about 150 million nucleotide base pairs of DNA. In an experiment to determine the pattern of eukaryotic chromosome replication, human cells are grown in a culture labeled for a short time with 3Hthymidine. The cells are lysed and placed on the surface glass slide. The glass slide

Which of the following patterns BEST represents


DNA replication of a eukaryotic chromosome? A.

coated with a of a photographic emulsion so the pattern of labeled DNA can be determined by autoradiography. Results of the

is

experiment determine that polymerization occurs at a rate

of 50 nucleotides per second.


B.

Autoradiography results of two experiments on identical pieces of DNA from the same organism are shown in Figure l. The patterns of replication are
demonstrated by these results:

C.

\
Experiment
Replicati onongin

D.

I
20.
The slower rate of DNA replication on a eukaryotic chromosome could be explained by:

Experiment B

+
ds

A. B.
DNA

C. D.

the presence of multiple chromosomes. the presence of exons and introns within the DNA molecule. the difficulty in replicating DNA packaged as

chromatin.

the difficulty in replicating DNA packaged


inside the nucleus.

Figure

21.

Replication forks are MOST likely to be activated


during which ofthese phases ofthe cell cycle?

A. Gr B.S C. Gz D.M
Copyright @ by The Berkeley Review

J'D

The BerkeleY Keview Specializing in MCAT Preparation

Biology
22.

DNA Keplication

Passage [Y

In a modification of the experiment discussed in the passage, a eukaryotic cell was pulse-labeled with 3H-uridine. The autoradiograph results would reveal the presence of a label:

A. B. C. D.
23.

only in the nucleus.


in both the nucleus and the cytoplasm. only in the cytoplasm. only in the nuclear membrane.

Which of the following statements could be TRUE regarding the autoradiography results shown in
Figure I ?

I. II. III. A. B. C. D.

The organism is most likely eukaryotic. The polymerases used in Experiment slower than those used in Experiment B.

A of

are an

Further incubation after the addition I only I and II only I and III only

unlabeled medium occurred in Experiment B.

II and III only

Copyright

by The Berkeley Review

336

The Berkeley Specializing in MCAT

Biology
Passage

Color Blindness
24. Which

Fassage V

V (Questions 24'29\

of the following statements is LEAST likely to result from gene duplication?

In human beings, the ability to perceive color depends

on genes that are both autosomal and X-linked. These genes initiate the production of visual proteins that are
iensitive to red (565 nm), green (535 nm), and blue (420 nm) wavelengths of light. Each visual pigment is found separately in a unique cell called a cone. These cells are concentrated in an area called the fovea, located in the
retina of each eye.

A. B. C. D.

Synthesis of Synthesis of Independent Synthesis of

similar amino acid sequences similar tRNA sequences mutation of the DNA sequences longer mRNA sequences

The genes that code for the red and green pigments are

close to one another on the

chromosome. The X

chromosome contains one gene for the red pigment and up to three genes for the green pigment. The gene for the biue pigment is found on chromosome 7. Genes which are

25.

What percentage of females would be expected to exhibit red-green color blindness, if one genetic
locus were involved?

closely related to one another belong to gene families. The X-linked genes appear to have evolved through gene duplication during the past 30 million years.

Red-green color blindness is a sex-linked recessive trait and is found in about 8Vo of the male population and in less than l7o of female population, even though about l57o of the female population are carriers. A defect in the synthesis of the green-sensitive pigment is referred to as tie deutan type of color blindness. Approximately 6Vo of the 87o of all males who are color-blind have a deutan

A. B. C. D.

O.04Vo

O.l2Vo
0.36Vo
O.64Vo

defect. The remainin g 2Vo have a protan defect, a type of color blindness resulting from a defect in the synthesis of the red-sensitive pigment. Both defects lead to poor recognition ofvarious shades ofred and green colors. The frequency for X-linked genes in males is the same as the phenotypic frequency, simply because the phenotype of both the recessive and the dominant alleles is expressed. The frequency of normal and mutant alleles can 6e calculated using the concepts of Hardy-Weinberg

26.

Since two genetic loci are involved in red-green color blindness, the incidence of color blindness among females in the United States is 0.47o The percentage of females that would be deutan colorblind is:

equilibrium and the.equation

A. B. C. D.

O.04Vo

O.l27o
0.367o 0.647o

P2+ZPq+q2=l

where p is the frequency of the dominant allele, and q is the frequency of the recessive allele. Color blindness is not a deleterious trait and therefore does not affect either the viability or the reproductive success of the individual.

27.

The percentage of women who are heterozygous


carriers ofprotan color blindness is about:

A. B. C. D.

27o 4Vo 87o

llVo

Copyright @ by The BerkeleY Review

scl

The BerkeleY Review Specializing in MCAT PreParation

Biology
28. A woman with normal
have:

Color Blindness

Passage Y

vision marries a man who is affected with deutan color blindness. Her sons will
deutan color blindness. homozygosity for the trait. heterozygosity for the trait.

A.
B. C.

D.

normal color vision.

29.

development

A dominant autosomal allele C is required for proper

of the cones in the retina. The

complete color blindness. A woman who has a deutan defect is homozygous for the autosomal C allele. She marries a man who is completely colorblind but carries the dominant deutan allele. What types of visual problems will their children have?

recessive allele c in the homozygous form results in

A. B. C. D.

Daughters

will

have normal vision; sons will

will be completely color-blind. Daughters will have normal vision; sons will be red-green color-blind. Daughters will be completely color-blind; sons will be red-green color-blind.

have normal vision. Daughters will be red-green color-blind; sons

t I

,[

Copyright @ by The Berkeley Review

3;'4

The Berkeley Specializing in MCAT

Biology
Passage

Alleles and Coat Qolor

Passage VI

VI

(Questions 30-36)

Neither the black nor the brown coat color is expressed when the hamster is homozygous for allele a, found at a

A gene is a unit ofinheritance located at a specific site (locus) on a chromosome (DNA). Genes exist in pairs in diploid organisms, and the individual genes of a gene pair are refened to as alleles. If two alleles of a gene pair are identical, the organism is homozygoas for that gene pair.

different locus on the chromosome. The suppressive influence of allele d on an entirely different allele is an
example of epistasis.

If the two alleles are different, the organism is heterozygous for that gene pair. The complete set of genes in an organism determines its genotype, while the expression of those genes as a collection of detectable
characteristics determines its phenotype.

m m ba
A
hamster that

Ba

is

homozygous

In a heterozygous individual, a dominant trait is due to a gene that expresses itself in the presence of its allele. A recessive trait in a heterozygote is due to a gene that does not express itself in the presence of its allele. Incomplete dominance in a heterozygote involves the expression of both alleles, with the effect of one allele appearing grcater than the other allele. In a heterozygote, a codominant trait involves both alleles producing a combined effect. In this case, two gene products can be detected. Each product is associated with one of the allelic forms.

genotype that produces a white (albino) coat.


has the

for allele a has a If a hamster

+ (wild-type or normal) allele at one of its a loci, its coat color will be normal.

Two experiments were conducted genetics of coat color in hamsters.


Experiment

to investigate

the

A group of true-breeding hamsters with the genotype B/B;

Alleles on separate chromosomes can segregate from one another at the time of gamete formation. This is the
basis of Mendel's second law (independent assortmen$ ard

the genotype b/b; a/a to give the first filial


generation.

+./+ is crossed with a group of true-breeding hamsters with

1nt)

results in interchromosomal recombination, which always produces a recombination frequency of 507o in the second filial (Fz) generation.

Experiment

II

However, two or more pairs of alleles can be associated

One hamster from the F1 generation of Experiment I was back-crossed to the parent with the double recessive

with the same chromosome. If this is the case, the genes are said to be linked. If two dominant alleles are linked,
those alleles are in coupling. If a dominant allele is linked with a recessive allele, those alleles are in repulsion. If there is crossing over between any two non-sister chromatids, intrachromosomal recombination results, which always produces a recombination frequency of less

genotype (blb: a/a). The progeny


generation are shown below:

of the resulting

F2

than 50Vo in the Fz generation. The recombination frequency (percent crossing over or ToCO) can be determined fronithe following equation:
Number of Cross over Individuals in F2 Total Number of Individuals in F2

offspring: 66 Brown offspring: 34 White offspring: 100


Black
Total

offspring:

200

IOOVa

The black coat color

in

hamsters

is due to a dominant
3

chromosome. The recessive allele, b, at this same locus results in a brown coat color when it is in the homozygous condition: gene, i-

B, at a particular locus on a

0.

Based

on the information given

in the passage,

which of the following statements is correct?

ffi

m
b

. B. C. D.

The a allele is dominant over the + allele. The a allele is recessive to the + allele. The a allele and the + allele are codominant. The + allele shows incomplete dominance.

Copyright @ by The BerkeleY Review

539

The Berkeley Review Specializing in MCAT Preparation

Biology
31

AIIeles and Coat Color


3

Passage VI

A true-breeding strain of black hamsters would show which of the following genotypes?

5.

Based on the information in Experiment


passage, which

II and in

the

of the following BEST

describes the

A. B. C. D.

two genetic loci?

B/b: +/+
B/b; +/a B/B: +/+ B/B: +la

A. B. C. D.

They They They They

are linked. are unlinked. are epistatic. assort independently.

6.

32. A different true-breeding strain of

The genetics of coat color example of:

in albino hamsters is m

hamsters has the

genotype b/b; a/a at both loci. The phenotype coat of these hamsters would be:

of

the

I. II. III.

black.

A. B. C. D.

dominance. recessiveepistasis.

incompletedominance.
codominance.

brown. white.

A. I only B. II only C . III only D. I and III only

(
3

ff

3.

on the information in Experiment I, what is the MOST likely phenotype of the progeny from the
Based

fr

o
fl 0

first filial (F1) generation?

A. All black B. All brown C. All white D. Both black and brown

4.

Based on the information in Experiment II, estimate the minimum genetic map distance between the two

genetic

loci. (Note: One genetic map unit (m.u.)

gives a recombinant frequency (RF) of 1 percent.)

A. B. C. D.

1? m.u.

34 m.u. 52 m.u. 68 m.u.

Copyright @ by The Berkeley Review

34o^

The Berkeley Reviw Specializing in MCAT Preparatic

Biology
Passage

DNA Calculations

Passage VII

VII

(Questions 37-43)

In

1953 James Watson and Francis Crick submitted a

However, the separation of cellular constituents and subsequent cytokinesis requires a constant time of 20 minutes after the completion of the corresponding round

landmark one-page paper to the British science journal Nature proposing a three-dimensional model for the

structure
obtained

of DNA. Using X-ray diffraction patterns by Rosalind Franklin and Maurice Wilkins,

of DNA replication. Therefore, bacterial cells with doubling times that are shorter than 60 minutes must begin DNA replication before the end of cytokinesis of
the preceding cell cycle.

Watson and Crick suggested that native DNA consisted of two antiparallel polynucleotide strands that wind about each other in a helical fashion along a common axis. The helix is righrhanded with a diameier of about 204. This is due to adenine-thymine (A-T) and guanine-cytosine base-pairings (G-C). Each base pair has an average molecule mass of 660 Daltons (D). In an ideal situation, the DNA helix has 10 base pairs (bp) per turn of the helix, resulting in a helical twist of 36' per bp. The rise of the helix per turn (pitch) is 34A, while the rise per base pair is 3.4A. Shortly after Watson and Crick proposed their model for the structure

Replication of the DNA duplex in E. coli is an ordered

process and

is promoted by at least 20 enzymes

and

proteins. A helicase enzyme uses the energy of adenosine triphosphate (ATP) to unwind the DNA at the orlC locus to give two replication forks, from which the parental strands serve as templates for the synthesis of new daughter DNA. Since all known DNA polymerases require a primer before new DNA can be synthesized, a short RNA primer of about 5 nucleotides is synthesized by a DNA-directed RNA polymerase called a primase.

of DNA, they postulated that DNA replicates


semiconserva-tively. The result

polymerase

is two molecules of

Once the primer has been established, DNA III begins to synthesize new DNA in the 5' to 3' direction (at a rate of about 1000 nucleotides per
fashion.

double-stranded DNA (a duplex), each containing the original parental strand of DNA and a newly synthesized daughter strand of DNA. This hypothesis was verified in 1958 by Matthew Meselson and Franklin Stahl.

Discontinuous DNA synthesis occurs in the form of lkb to 2kb segments of DNA calledOkazakifragments.DNA

on the lagging strand in a discontinuous

second) on the leading strand in a continuous fashion and

1957 Arthur Kornberg, using the bacterium Escherichia coli, discovered the enzyme DNA polymerase I, which is involved in the DNA-directed synthesis of DNA. In 1969 Paula Delucia and John
Cairns discovered a mutant of E. coli that did not display

In

polymerase I removes the RNA primers using its exonuclease activity and then slowly adds new nucleotides at a rate of about l0 per second. The
fragmented stretches of newly synthesized DNA are finally joined together by DNA ligase.

DNA polymerase I activity. This mutant was named the

polAl
l%o

of the normal

mutant. Even though the

polAl

mutant had about

polymerizing function

of

DNA

polymerase I, it reproduced at the same rate as its parental

37. If the average molecular


residue is I

strain, thus suggesting that there might be polymerase activity by an enzyme or enzymes other than DNA
polymerase I.

weight of an amino acid l0 D, and if each amino acid is specified by three contiguous bases on a single strand of DNA, then for a 40-kD protein the contour length and molecular weight of a given section of DNA
would be about:

This observation -led to the discovery of DNA II and DNA polymerase III. DNA polymerase III is also involved in the DNA-directed
polymerase

synthegis of DNA. However, the physiological function of DNA polymerase II is still unknown.

A. B. C.

D.

3'7oL,7.0 x 104 D 37004, 7.0 x 104 D 310L,1.0 x 105 D 37ooA.7.o x los D

chromosome is initiated at a single locus referred to as oriC. [One kilobase (kb) is equal to 1000 nucleotide base

4000 kilobase pairs found in the circular DNA


pairs (bp) of duplex (double-stranded) DNA or 1000 nucleotides of single-stranded DNAI. The 245 bp segment of the orlC locus is highly conserved among Gramnegative bacteria. Even though the doubling time of this bacterium can range from less than 20 minutes to about
10 hours (depending upon the growth medium), it takes about 40 minutes for the chromosome to replicate itself completely. Copyright @ by The Berkeley Review

ln E. coli, the bidirectional replication of the roughly

38.

The rate of nucleotide incorporation at each growing

replication fork would be roughly:

A. B. C. D.

225 nucleotides per second.

425 nucleotides per second.


850 nucleotides per second. 1700 nucleotides per second.

34t

The Berkeley Review Specializing in MCAT Preparation

Biology
39.

DNA Calculations

Passage VII

Unlike the circular chromosome of a prokaryotic organism like E. coli, the linear chromosomes of

42.

During DNA replication, the base sequence 5'pApTpApGpApC-3' would give rise to which of
these complementary base sequences?

eukaryotic organisms have many origins of


replication. These sites of replication are roughly
spaced 30 to 100 kilobases apart from one another. Suppose that a given tissue culture of mammalian DNA has L2 meters of duplex DNA in every single cell and that the synthesis phase (i.e., that period in the cell when DNA is synthesized) for these cells is 5 hours. How many replication forks are there, if the growth rate in these cells is l6 pm/minute?

A. B. C. D.

3'-UpApUpCpUpGp-5' 5'-pTpApTpCpTpG-3' 5'-pGpTpCpTpApT-3' 3'-TpApTpGpUpGp-5'

A. B. C. D.

125 replication forks

250 replication forks 375 replication forks 500 replication forks

43,

Which of the following statements about the action of DNA polymerases must be true?

DNA polymerase I also has a 5' to 3' exonuclease activity that excises RNA
primers.

In addition to its 5'to 3'polymerizing activitl'.

il. 40.
In a rich growth medium, bacterial cells can divide every 20 minutes. Which of the following BEST
describes the cell's bidirectional replication in this
case?

DNA polymerase III can add nucleotides at the 3' end of a growing polynucleotide chain. while DNA polymerase I can add nucleotides at the 5' end of a growing polynucleotide
chain.

nI. DNA
At division,
each daughter cell receives each daughter cell receives
a
a

polymerase

III

adds new deoxyribo-

I. III. IV.

U. At division,

chromosome that is half replicated.

nucleotides to a growing polynucleotide chain roughly 100 times faster than DNA polymrase I.

chromosome that is one-quarter replicated. There is a total of four replication forks. There is a total of six relication forks.

A. III only B. I and III only C. II and IV only D. I and IV only

A. B. C. D.

II only II and III only I and III only I and II only

41-

During the replication of an E. coli chromosome, roughly how many Okazaki fragments are
synthesized?

A.
B. C. D.

100 to 200 1000 to 2000

2000 to 4000 4000 to 6000

lr

Copyright @ by The Berkeley Review

342

The Berkeley Reviev Specializing in MCAT Preparatiom

Biology
Passage

Semiconservative DNA Replication


44-50)

Passage VtrI

VIII (Questions

44. Watson

and Crick's proposal for the structure of DNA does not allow for which of the following base

After Watson and Crick announced their discovery of the structure of DNA in the early 1950s, they proposed that DNA could reproduce itself autocatalytically. They hypothesized that the two parental strands of a DNA duplex would separate, and each would serve as a template for the synthesis of a new daughter strand of DNA. Once DNA replication was complete, each new

pairings?

I. I I. ilI.

Cylosine-guanine
Adenine-guan ine

Cytosine-thymine

duplex would contain one parental strand and one daughter strand, linked by hydrogen bonds between complementary purine (adenine and guanine) and pyrimidine (thymine and cytosine) bases. Watson and Crick called this type of replication semiconservative.
Conservative replication, in contrast, would conserve the integrity of the parental strands in the DNA duplex after replication.

A. I only B. II only C. II and III only D. III only

45. After Watson and Crick

proposed the structure of

In the late 1950s, Meselson and Stahl were able to confirm Watson and Crick's hypothesis that DNA
replicated semiconservatively by performing a densitygradient equilibrium sedimentation experiment. In this experiment, a 6M cesium chloride (CsCl) solution with a density of about 1.7 glcm3 is placed in a test tube, which is spun at high speed in an ultracentrifuge. A smoothly
changing concentration gradient is thus established in the test tube (Figure 1).

DNA, it was postulated that DNA could replicate in a conservative fashion. After two rounds of conservative DNA replication, the original duplex
parental strands would give rise to:

A. B. C.

two duplexes of DNA, each containing


daughter strand and one parental strand.

one one

four duplexes of DNA, each containing


daughter strand and one parental strand.

r_____

Increasing density of solution

two daughter strands and one


containing two parental strands.

two duplexes of DNA, one duplex containing


duplex duplexes each

r)

D. four duplexes of DNA, two

containing two daughter strands and two


duplexes each containing two parental strands.

CsCl spinning in uitracentrifuge

Figure I

46. The structural

component

of DNA is

called

nucleotide (see below):

E. coli were grown in a complete medium containing l5NH+C1 for numerous generations, ensuring that the DNA in these cells i.vould contain the heavy, stable isotope of nitrogen, 15N. An excess of l4NH+Cl was then added to the culture of l5N-labeled bacterial cells. Immediately after this addition of laNHaCl, the DNA from a sample of the cultured cells was extracted and sedimented to equilibrium in the CsCl density gradient. Ultraviolet absorption photographs (at 260 nm) indicated the presence of just one band of DNA. This band corresponded to the all heavy lsN-labeled DNA.

A nucleotide
containing
the base guanine

o.R Jt''^ n=J-n ,'Y ";; trA"A**,

'[J'
I","'^
o-R'

The cells that remained in culture were allowed to


double and complete one generation of growth. Analysis of the DNA from a sample of these cells indicated the presence of a new band, that o1 u 1411-15p hybrid. On the

The stable, heavy isotope of nitrogen, l5N, can be found in all of the following components of DNA
EXCEPT:

basis

of this

concluded that DNA replicated


fashion.

density-gradient experiment, it was in a semiconservative

A. B. C. D.
345

the the the the

pyrimidine rings. purine rings. ribose rings.


acetal linkages.

Copyright @ by The Berkeley Review

The Berkeley Review Speciatizing in MCAT Preparation

Biology
47. Ultraviolet

Semiconservative DNA Keplication


50.

Passage Vtrl

absorption photographs (at 260 nm) of DNA in a CsCl density-gradient solution indicate a particular DNA banding pattern. The distribution of l4N and l5N in a CsCl density gradient after two rounds of semiconservative DNA replication is represented by which of the following DNA banding
patterns?

Macromolecules that are not bound to ions have densities close to 1.3 g/ml. Which of the following single-stranded nucleic acid polymers would show a
greater density in a CsCl density gradient?

5'-pGpCpApApCpCpGpGpCpC-3' DNA 5'-pCpCpGpGpCpCpApApCpG-3' RNA

A.

B.

ll
I

l -l

tt tt
C.

FF -tt tt
Htt
@jj m
Control

(C = cytosine; G = guanine; p = phosphate)

A.

DNA, because the Cso ions can bind to the negatively charged phosphate groups and the free hydroxyl groups of DNA. RNA, because the Cs@ ions can bind to the negatively charged phosphate groups and the free hydroxyl groups of RNA. DNA, because the Cle ions can bind to RNA, because the Cle ions can bind to
the the

".:.1
r,i.1,'.:$

B.

\.ry
Control

C.

D. D.

positively charged nitrogenous bases of DNA. positively charged nitrogenous bases of RNA.

)f'\B*.

t)i):::i

Control

Control

48. After two rounds of

replication, the ratio of laN to l5N will be:

semiconservative DNA

A. B. C. D.

Ir:

3:l 2:l l:2 l:3

:u

trx;

[5

llll,

tr,"

T,,I,

49. DNA

synthesized from the 1sN isotope is denser than DNA synthesized from the l4N isotope, because of the additional:

f;;
illf

A. B. C. D.

neutron in l5N. proton in 15N. electron in l5N.


water molecules surrounding the 15N DNA.

mi

j
m

im r

,ilE

Copyright @ by The Berkeley Review

344

The Berkeley Revier Specializing in MCAT Preparatiol

Biology
Passage

DNA Structure
Replication

Passage IX

IX (Questions 51-58)

of the DNA helix requires

local

The molecules that store and transmit information in

cells are deoxyribonucleic acid (DNA) and ribonucleic acid (RNA). The fundamental unit of each of these
nucleic acids is the nucleotide. Each nucleotide contains a nitrogenous base, a five-carbon sugar called ribose, and a phosphate. The bases adenine (A), guanine (G), and cytosine (C) are found in both DNA and RNA. Thymine

denaturation (melting) of hydrogen bonds between base pairs. Each single-stranded segment of old (parental) DNA then serves as a template to which new (daughter)

DNA is attached. After one round of replication is complete, two new DNA duplexes are formed, each containing one strand of parental DNA and one strand of daughter DNA. This is referred to as semiconservative
replication.

(T) is found in DNA and uracil (U) is found in RNA. Figure 1 shows the structural components of these two
nucleic acids (Note: dR = deoxyribose; R = ribose): O
NH.

Three other forms


examined.

If

of DNA replication have been of replication one of the DNA

the DNA helix replicates conservatively,

rt "1,,tr
dR

""4at"' *)"
dR

"I)
Cytosine

HN}

A
I

then after one round

.L*)
R

Thymine
NH"

Uracil

replication is dispersive, then after one round of replication the DNA in each strand of each duplex is composed of alternating segments of parental and daughter DNA. Finally, if replication is end-to-end conservative, then after one round of DNA replication
each strand in each duplex contains one end that is all
parental DNA and one end that is all daughter DNA.

duplexes still contains both parental strands, while the other DNA duplex contains two new daughter strands. If

r3)
I

"*a-"
"."1_-L)
dR

il

dR

Adenine

o- P- on -o
I

o_ tt

Guanine
5

51.

Which of the following bases is LEAST likely to


exist in the enol form?

cH.

A.
B. C. D.

Adenine Cytosine
Guanine

Ribose phosphate

OH (or H)

Uracil

Figure I

DNA and RNA can exist in a double-stranded (duplex)

or a single-stranded form, and they can be linear or


circular. Both nucleic acids can also hybridize to each other. In the nucleus of eukaryotic cells, DNA is associated with positively charged proteins called
histones.

52. DNA can be classified by all of the following


characteristics EXCEPT:

A. B. C. D.
53.

it exhibits

polynucleotide arrangement.

it is negatively charged at neutral pH. it is deoxy at the C-3' position. it contains acid anhydride linkages.

The DNA double helix consists of two polynucleotide strands that wind about a common axis, making a complete turn every 3.4 nm. Each strand is joined by hydrogen bonding between the nitrogenous bases. The base pairs are partially stacked on each other and are
separated by 0.34 nm.

The following sequence of a short segment of single strand of DNA is given:


5'-A-T-G-C-C-G-A-T-3'

Each strand in the DNA double helix has a specific

chemical orientation. Attached to the 5' carbon of the


ribose ring is a hydroxyl gr6up that bears a phosphate; the

The complementary single strand of DNA is:

3' carbon atom of the ribose ring has a free hydroxyl group. Polynucleotide sequences are conventionally written in the 5' -+ 3' direction, and in DNA which is double-stranded the two polynucleotide sequences are
antiparallel to one another. Copyright
@

A. B. C. D.
345

3'-pTpApCpGpGpCpTpA-3' 3'-pTpApCpGpGpCpTpAp-5' 5'-ApTpCpGpGpCpApT-3' 5'-pApTpCpGpGpCpApT-3'

by The Berkeley Review

The Berkeley Review Specializing in MCAT Preparation

Biotogy
54.

DNA Structure

Passage

On average, how many bases are there per turn of the DNA double helix?

58. An unknown organism is incubated in a complelt

A.5 B. 10 c. 15 D. 20
55.
Which of the following base pairs is NOT usually
associated with DNA?

new growth medium containing l4NH+Cl. After tr*: generations in the new growth medium, its DNA ri analyzed using CsCl equilibrium density-gradien: ultracentrifugation. Next, ultraviolet absorpticr,

growth medium containing l5NH+Cl for manl generations. The organism is next transferred to I

photographs
results.

of the ultracentrifuge test tubes ar;

taken, and a microdensitometer trace is made of tle

A.
B.
C.

Uracil hydrogen bonding to adenine


Adenine hydrogen bonding to thymine Guanine hydrogen bonding to cytosine Thymine hydrogen bonding to uracil

o !J:E qN .Ed

D.

50.

DNA is the carrier of genetic information during cell


characteristics
accurate transmission of this information?

/.
Direction of sedimentation
--------->

growth and division. Which of the following of DNA is NOT essential for the

A. B. C. D.
5/.

A A A A

genetic code that is degenerate mechanism for self-replication low mutation rate

Two peaks arre observed, and the amount of DNA :r each peak is recorded. Based on the information n the graph presenting these data, we can say thc DNA replication in this organism is:

conformationally variable molecule

Denatured DNA shows an increase in ultraviolet absorbance. The temperature at which one-half the maximum absorbance is reached is called the
melting temperature,
T

A typical DNA melting curve is

shown below.

A. B. C. D.

conservative. semiconservative. dispersive.

end-to-endconservative.

6.

1.4
C)

X
.Ed

r.r

8r.2 o c]

fi &

r.r
1.0

50

60

'70 80

90

Temperature ("C)

All of the following factors in the Tm of DNA EXCEPT:

will lead to

an increase

A. B. C. D.

n.-

a lower mole fraction of A.T base pairs. a higher concentration of Mg2o.


a a

lower concentration of histones. higher mole fraction of G'C base pairs.

Copyright @ by The Berkeley Review

346,

The Berkeley Revier Specializing in MCAT Preparatiou

Biologv
Passage

Mitochondrial DNA (mtDNA)


61.
the

Passage X

X (Questions 59-65)

The biosynthesis of mitochondria involves

The codon AUG is used as a start codon in the genetic code of the mitochondria. This codon codes
for which of the following amino acids?

contribution of two separate genetic systems. Most of the proteins are encoded by nuclear DNA and imported into the organelle. However, some proteins are encoded by
organelle DNA and synthesized on ribosomes within the organelle. The protein traffic between the cytosol and the mitochondrion appears to be unidirectional.

A. B. C. D.
62.

Leucine Histidine Methionine N-formyl methionine

Mitochondrial DNA (mtDNA) molecules are small

All mitochondria contain multiple copies of their DNA molecules, and the genome is likely to
and circular.

resemble bacterial chromatin. The human mitochondrial genome has several surprising characteristics. One is that every nucleotide appears to be part of a coding sequence for a protein, rRNA, or tRNA. Second, only 22 tRNAs are needed for mitochondrial protein synthesis versus the 31

As stated in the passage, every nucleotide in the mitochondrial genome is part of a coding sequence. A probable result of this is thar compared to the
nuclear genome, there would be:

tRNAs required in the cytosol. Third, the genetic code


used in the mitochondrion differs from the universal code used in other genomes. It has been shown that four out of the sixty-four codons in mtDNA have different meanings.

A. B. C. D.
63.

fewer mRNA molecules ffanscribed. more mRNA molecules transcribed. more room for regulatory DNA sequences.
less room for regulatory

DNA

sequences.

Further investigation has shown that the mitochondrial genetic code is different in different organisms. For example, the codon UGA codes for tryptophan in the mitochondria of mammals, fungi, and protozoans. But in plant mitochondria, UGA is a stop codon.
are transcribed from a single promoter region on each strand.

Which of the following statements offers


found in mitochondria?

reasonable explanation for the different genetic code

A. B. C. D.

Both strands of human mitochondrial DNA

Two giant RNA molecules are thus created. The two molecules are known as the heavy and light strands. The heavy strand is processed to produce many RNAs, including ten polyadenine-containing RNAs. Roughly 90Vo of the light strand contains no discernibly useful
information.

The drift responsible for changes in the genetic code occurs only in mitochondria. The small number of proteins encoded in the mitochondrial genome makes changes in the meaning of the code tolerable.

The mitochondria has unique tRNAs, which correct for the different genetic code. In mitochondria, the primary structure of a protein has no effect on the function of the
molecule.

64. The ten polyadenine-containing RNAs noted in the

59. In a human cell, * protein transport


from cytosol to:

passage are MOST likely to:

does NOT move

A. B. C. D.

mitochondria.
peroxisome. nucleus.

A. B. C. D.
65.

make their way to the cytosol to be translated. have an attachment site for amino acids. be involved with the structure of ribosomes. code for proteins.

chloroplast.

The DNA that codes for the light strand is also


known as the:

60. Which of the following is LEAST likely to


associated with the mitochondrial genome?

be

A. B. C. D.

antisense strand, because the


complementary.

light chain light chain

is is is

A. B. C. D.

Distinct prombter regions RNA polymerase protein DNA polymerase protein


Histone protein complex

antisense strand, because the nearly identical.

sense strand, because


complementary.

the heavy chain

sense strand, because the heavy chain is nearly

identical.

-opyright

by The Berkeley Review

347

The Berkeley Review Specializing in MCAT Preparation

Biotogy
Passage

Viral Complementation
66.

Passage )il

XI (Questions 66-72)

The bacteriophage T4 is a DNA virus which infects E.

The DNA of the bacteriophage T4 encodes all of the components necessary for the replication of its viral genome. Which of the following might NOT be
encoded?

coli cells. When a wild-type T4 phage is placed on a lawn, or thin layer, of E. coli cells growing on an agar
plate, a local clearing develops where bacterial cells have been lysed by the original phage and its progeny. This clear region on the otherwise opaque lawn is termed a plaque.

A. B. C. D.

tRNA
Primase

Ligase

DNA polymerase

Two mutant forms of phage T4 have been isolated and named rllA and rlIB, respectively. Both mutants cause larger-than-normal plaques on plates containing a strain of bacteria known as E. coli B. However, both of these T4 mutants fail to form plaques at all on a second bacterial strain, E. coli K. Wild-type T4, however, is capable of lysing cells of E. coli K.

An experiment is conducted in which rllA and rIIB mutants are allowed to infect E. coli K cells simultaneously. Individual bacterial cells are therefore coinfected by both mutant forms of T4. While infection by either mutant alone causes no plaque formation, the
simultaneous coinfection by both mutants causes cell lysis and the formation of small plaques. This phenomenon is an example of genetic complementation.

67.

E. coli K cells are coinfected with rIIA and a neu T4 mutant called X1. No plaques form. What can be concluded from this experiment?

A.
B.

The rIIA and different genes.


The rIIA and
same gene. The rIIA and

Xl
X

mutations each aff*tn


mutations both affect rtu

C. D.

XI

mutations complement e,rin

In this example, complementation occurs when protein that is defective in rIIA is provided by rIIB,

the and

other.

vice versa. This complementation results in the restoration of the wild-type, lysis phenotype (Figure 1). In order for two mutants to complement each other, they must each affect different genes. If they both were to affect the same

The X1 mutant phage could lyseE. coli K ceilin if it infected them alone.

gene, they would be incapable

of

complementation,

because neither mutant could provide a functional protein product.

Genes (phage rIlB)

(+)

rIlA

C)

rIIB
68.
Lysis

Pathway (l

)r------)

Based on information given in the passage, whi;h

the following would explain why rII mutanl:

(2)r----)

incapable of forming plaques on lawns of E. cal;


cells?

,o,,lr"Jii,o,
Figure 1. Genetic complementation in phage T4.

I. U.
IIII.

r11

mutant DNA has sequences that

recognized and cleaved


produced by E. coli K.

by

endonucle
pro*X

r11 mutants

lack functional gene

necessary for lysis of E. coli K cells. E. coli K lacks specific surface receptor$

which r11 mutant phages bind.

A. I only B. II only C. I and III only D. I.II. and III

Copyright @ by The Berkeley Review

34A

The Berkeley Specializing in MCAT

Biology
69. a curly-wing
In Drosophila, recessive mutants

Viral Complementation
72.

Passage Xf

A and B each cause phenotype when homozygous. However, transheterozygous flies (i.e., flies that are NB)have curly wings, even though neither mutation is homozygous. Which of the following conclusions
is MOST consistent with these results?

The T4 bacteriophage can BEST be described as:

A. B. C. D.

an obligate parasite. an obligate heterotroph.

auxotrophic. prototrophic.

A.
B. C. D.

Mutations A and B complement each other; therefore they affect different genes.

Mutations

and

B fail to complement

each

other; therefore they affect different genes. Mutations A and B complement each other; therefore they affect the same gene.

Mutations

and

B fail to complement

each

other; therefore they affect the same gene.

70.

A large number of the T4 mutants rIIA and rIIB are added separately to two plates of E. coli K cells. The plate infected with rIIA forms one revertant plaque, while the plate infected with rIIB forms no plaques at all. Based on these results, which of the following assumptions may be valid?

A. rlIA B. D.

C. rlIA is caused by a point mutation, while rIIB


is caused by a deletion of DNA. rIIA and rllB are caused by frameshift mutations.

rlIB is caused by a point mutation.

is caused by a deletion of DNA, while rIIB is caused by a silent mutation. rIIA is caused by an deletion of DNA, while

Both

71.

Which of the following would be incapable of


exhibiting the type of complementation described in
the passage?

A. B. C. D.

Recessive mutations Sex-linked recessive mutations

Dominant mu-tations Deletion mutations

Copyright @ by The Berkeley Review

349

The Berkeley Review Specializing in MCAT Preparation

Biology
Passage

Atrial Natriuretic Peptide Experiment


74. Atrial
tissue from the animals

Passage XII

XII

(Questions 73-79)

About lOVo of the US population has sodium-sensitive hypertension. As dietary sodium increases, blood pressure increases in these people. Blood pressure is regulated by blood volume, peripheral resistance, and the cardiac rate. One molecular mechanism of control involves a 28amino acid peptide produced primarily in the atria, called atrial natriuretic peptide (ANP). It is stored in dense granules in the cytoplasm of atrial cells as a larger precursor, pro-ANP. In response to atrial distention or stretch, pro-ANP is processed, and ANP is secreted into the bloodstream. It promotes sodium excretion into the urine and the movement of fluid out of the intravascular
space.

homogenized. The amount of ANP (pg/mg wet tissue) in the atria is then measured. The results are given in the following table:

is

isolated

and

Animal
Homozygous mutant Heterozygous mutant

Right atria
Undetected

Left atria
Undetected

54.3

t'.t.1

53.7

!7.1
r7.5

Wild-type

114.7 + 5.9

112.2

to study the interrelationships

Researchers use a genetically engineered mouse model

What would be the BEST type of measuremenr r:


detect and quantify ANP, which is a small pepride?

between ANP, dietary


and

sodium, and blood pressure. Mouse embryos are treated

so that the gene coding for ANP is mutated

nonfunctional. The mice are then interbred to produce


homozygous mutants and heterozygous mutants. Control, wild-type animals are also used. All mice are fed one of three diets: either standard chow at 0.5Vo sodium, an intermediate diet at 2Vo sodltm, or a high-sodium diet at
87o sodium.

A. B. C. D.

Lowry assay
Peptide digestion Southern blot Radioimmunoassay

Diet
7o

sodium
0.5

Homorygous mutants
124 + 3.7*

Heterozygous

Wild-type
animals
116 + 2.6

t5. Based on the observed effects of ANP om intravascular compartments, explain the follourrnE
hematocrit values taken from the study animals:

animals
115

r !

1.8

(standard

Animal
Homozygous
156 + 9.0*

Hematocrit
45.80/o

chow)
2
8

132

6.7

t34 + 6.1

mutant Heterozygous mutant


48.6Vo

145 + 7.8*

11816.2

* p < 0.05 versus wild-type.

Table 1. Blood pressure (mmHg) in experimental animals on various sodium-containing diets.

Wild-type

49.9Vo

A.

73.

Which of the following statements is TRUE about


the data in Table 1?

The homozygous mutants have a loqtnhematocrit due to more fluid in inc


intravascular space.

B.

A. B. C. D.

Homozygous mutant animals were normotensive on the standard chow diet. The 27o sodium diet increased blood pressure in all groups. Heterozygous animals mimicked the responses C.

The wild-type animals have a highcu" hematocrit due to more fluid in ftc
intravascular space.

ANP directly affects the blood cells,


animals, raising the hematocrit.

so

more of them are produced in the wild-rlpcr


D.

of wild-type
increased.

animals as dietary sodium

fewer of them are produced


hematocrit.

ANP directly affects the blood cells,

so

Blood pressure increases in the heterozygous animals were inversely proportional to dietary
sodium.

in homozygous mutant animals, lowering

Copyright

by The Berkeley Review

550

The Berkeley Specializing in MCAT

Biology
76.

Atrial Natriuretic Peptide Experiment


79.

Passage XII

Individuals processing a newly discovered human mutation are found to be lacking the protease that liberates ANP from the pro-ANP precursor. What would be the probable fate of pro-ANP in these
people?

Applying these ANP gene data to human beings, if

child has one parent who is not salt-sensitive (AA) and one parent who is heterozygous (Aa) and saltsensitive, what is the likelihood that the child is also salt-sensitive (either Aa or aa), assuming Mendelian
inheritance?

A. It would probably be degraded inside the cell B. D.


by lysosomal mechanisms.

It would probably be released

as pro-ANP and

C. It would

activated by proteases in the bloodstream.

probably be activated by lysosomal

A. B. C. D.

O% 257o 50Vo 757o

mechanisms and released as ANP. It would probably be released as pro-ANP and excreted by the kidney.

77. Which other theoretical

experimental approaches

could provide information similar to what was found in the experiment described in the passage?

I. II. ilI.

Administer to wild-type animals the same anti-

ANP antibodies and compare to untreated


animals.

Administer to wild-type animals a compound that irreversibly binds the ANP receptor and
compare to untreated animals.

Administer to wild-type animals a compound that binds and inhibits processing of pro-ANP and compare to untreated animals.

A.
B. C. D.

I only

I and II only

II and III only I, II, and III

78.

Based on information in the passage, what is the postulated role of ANP?

A. B. C. D.

To To To To

lower blood pressure raise arterial blood pressure raise atrial piessure lower sodium output in the urine

Jopyright @ by The Berkeley Review

351

The Berkeley Review Specializing in MCAT Preparation

Biology
Passage

Griffith's Pneumococcus
80-87)

Passage tr(il

XIII (Questions

81.

Based on information in the passage, which of following conclusions is NOT valid?

fu

In 1928, Fred Griffith carried out the following set of experiments using mice and the bacterium Streptococcus pneumoniae, a Gram-positive, facultative anaerobe.
Experiment

A.

The DNA of pathogenic strains codes for rte

polysaccharide coat seen

on

transforrnod

B. R C. D.
nonencapsulated strain

strains.

mutants lack enzymes needed

for

thc
inmo

A live, encapsulated strain of virulent pneumococcus is injected into a mouse. The mouse dies.
Experiment 2

synthesis of the polysaccharide coat.

Injection of only the polysaccharide coat


mice will result in their death.

The transformation of the R-strain to the


strain is permanent.

A live,
Experiment 3

of

nonvirulent

pneumococcus is injected into a mouse. The mouse lives.

A heat-killed strain of virulent pneumococcus


injected into a mouse. The mouse lives. Experiment 4

is

82.

Pneumococcus strains used in the Griffifrr experiments that were treated with heat were killlsd,
because of damage to:

A mixture of hearkilled virulent and live nonvirulent pneumococcus is injected into a mouse. The mouse dies

and is found to contain the live

pathogenic

pneumococcus.

A. B. C. D.

lipids.
proteins. carbohydrates.

nucleic acids.

Griffith concluded that the heat-killed virulent cells contained a transforming factor that converted the live, nonvirulent cells, to live virulent cells.
categorized by the presence or absence of an outer polysaccharide capsule. Strains with the capsule are pathogenic and are denoted as the S-form of the bacteria.
Nonpathogenic strains are denoted as the R-form.

The pathogenicity of

pneumococcus

can

be

83. All of the following


conclusion EXCEPT:

statements support Ave4r'b

A.
B.

there was no loss of transforming activity the addition of trypsin and chymotrypsin.

In 1944, a series coworkers characterized this transforming principle as nucleic acid of the ddbxyribose type.

of experiments by Avery and his


a

C. D.

the addition of ribonuclease had no effect the transforming principle. there was no loss of transforming activiq' the extraction of protein or lipid. there was no loss of transforming activiq the addition of deoxyribonuclease.

80.

The stain used to distinguish between Gram-positive (+) and Gram-negative (-) bacteria takes advantage of differences in which of the following structures?

84. Which of the following BEST


A.
B.
C.

A. B. C. D.

describes metabolic requirements of pneumococcus? The organism requires the presence of oxy-g The organism may function in the presence absence of oxygen. The organism requires the absence of oxygur. The organism requires the absence of ni

Plasma membrane

Nuclear region
Ribosomes

Cell wall

D.

Copyright @ by The Berkeley Review

352

The Berkeley Specializing in MCAT

Biology
85. All of the following

Griffi th's Pneumococcus


statements are FALSE except:

Passage XIII

A. B. C.

D. the pneumococcus

pneumococcus translation takes place in the mitochondria. pneumococcus does not contain organelles. pneumococcus cells are larger in diameter than most eukaryotic cells.

nucleus

is a

double

membrane-bound structure.

86.

In an experiment that provided further evidence for Avery's conclusion, radiolabeled elements were used to distinguish between protein and nucleic acids.
Which of the following elements were MOST likely
used?

A.
B. C. D.

Oxygen and nitrogen Phosphorus and nitrogen Sulfur and oxygen Phosphorus and sulfur

87.

Pathogenic pneumococcus coated with antibodies can be rendered avirulent. Therefore, it can MOST likely be concluded that the polysaccharide capsule normally inhibits:

A. B. C. D.

pinocytosis of the bacterium by B-cells. phagocytosis ofthe bacterium by p-cells. pinocytosis of the bacterium by macrophage. phagocytosis of the bacterium by macrophage.

Copyright @ by The Berkeley Review

ccc

The Berkeley Review Specializing in MCAT Preparation

Biology
Passage

Drosophila eyeless Gene


88.
Assume that ey

Passage XIV

XIV (Questions 88-94)

is X-linked.

If

ey/+ females

are

The eyeless (ey) mutation in Drosophila melanogaster was first described in 1915 on the basis of its distinct phenotype, the partial or complete absence ofthe insect's

crossed to wild-type males, what are the phenotypes of the progeny?

gene are recessive hypomorphs, meaning that the mutant gene product has reduced function relative to wild-type. Heterozygous (ey/+) flies have smaller eyes, while homozygotes (ey/ey) have almost no eyes at all. The eyeless gene has been cloned and appears to be a transcription factor. It shares sequence homology to a gene in mice called small eyes.In situ hybridization shows that ey is normally expressed only in the
embryonic tissues that eventually form the eyes. Mice and flies are separated evolutionarily by 500 million years.

compound eyes. Known alleles

of the e)

A. B. C. D.

of males have small eyes, 50Vo of females have small eyes; all others are normal. 5OVo of males have no eyes, 507a of females have small eyes; all others are normal. 50Vo of males have no eyes, 50Vo of males are normal, l00%o of females are normal. IOO% of males have no eyes, l00vo of females are normal.
50Vo

In

Drosophila,

technique has recently been

developed in which a gene of interest can be ectopically expressed in specitic tissues in which it is normally not active. GAL4, a transcriptional activator protein originally

isolated

in

yeast, can bind

to an upstream activating

89.

In

Experiment 1, some fertilized eggs died

as

sequence (UAS) on DNA, triggering transcription of the gene directly downstream from the UAS. "Downstream" means in the 3'direction, while "upstream" means in the 5' direction.

embryos. In situ hybridization, a technique wherebl fluorescently-labeled antisense ey mRNA is allowed to hybridize with sense mRNA in vivo, showed e-r mRNA was expressed throughout the embryo. Whar would MOST likely have caused this? A. B.
C.

ft,-*'#\

ftf!)
uAS

p,ot.in

-"':T:::::*Eennancer
ofGAL1
Figure I
Wing specific expression

Jf, era
wing tissue only

substitution mutation at the UAS binding

site

Transcription of ey in

D.

A constitutive mutation in the GAL4 promoter Expression of the normal genomic ey mRNA An amber mutation in the ey coding sequence

By ligating the cDNA sequence encoding the CAL4 protein to a weak promoter and a Drosophila tissuespecific enhancer (a sequence that boosts transcription of

nearby genes), GAL4

will be transcribed only in the specific tissue where the enhancer is active, thereby activating only the UAS-controlled gene in that particular
tissue.

90. Which of the following

theories CANNOT

be

supported by evidence given in the passage?

Experiment

A transgenic line of flies is engineered with a DNA construct consisting of wild-type ey cDNA directly
downstream from the UAS site. These flies are crossed to

I.

'The ey gene is necessary and sufficient fcr


normal eye development in
D

II. The III. A. B. C. D.


in

rosophila.

another line that has a wing tissue-specific enhancer in wings) adjacent to a GAL4 sequence. Remarkably, most of the resulting progeny have complete

ey protein acts as a developmente, "master switch," activating transcription r:


insects and mammals.

(active only

genes needed for eye formation. Evolutionarily, eyesoriginated independenril

compound eyes growing from their wings. Experiment 2

I only

The wing-enhancer/dAl4 line from Experiment 1 is


crossed to flies containing a construct that links the mouse small eyes gene sequence to the UAS site. The results are identical to those in Experiment 1.

II only
I and III only I, II, and III

Copyright @ by The Berkeley Review

354

The Berkeley Keviex Specializing in MCAT Preparation

(--

Biology
91.
Which
mentioned in the passage?

Drosophila eyeless Gene

Passage XIV

of the following is MOST likely be the cause of the mutant phenotype shown in the ey alleles

A. B. C. D.

A point mutation in a non-conserved region of the ey sequence A two base-pair deletion near the beginning of the ey sequence An inversion of a segment of the ey sequence A point mutation in a highly conserved region of the ey sequence

92. Which of the following findings would


inconsistent with the results of Experiment 2?

be

A. B. C. D.

Humans have a gene, Aniridja, which is


homologous to ey.

9OVo

Ectopic expression of ey in the mouse results


in ectopic compound eyes.

ey mRNA forms hybrids with small eyes


antisense mRNA.

Several other organisms with visual organs


have genes homologous to ey.

93. A population of Drosophila

is in Hardy-Weinberg

equilibrium with respect to the eyeles.r gene. Which of the following conditions must apply?

A. B. C. D.

There can be no selective pressure on the


population.
Genetic drift must be present in the population. Mutati-ons must occur in the population. There -can be no random mating between members of the population.

94. A group of 1000 flies is in Hardy-Weinberg equilibrium. If the frequency of an eyeless mutant
allele is l07o,how many flies in the population have small eyes?

will

A.0r B. I

c. D.

180

810

Copyright

by The Berkeley Review

5bb

The Berkeley Review Specializing in MCAT Preparation

Biology
Passage XV (Questions 95-100)

Meiotic Nondisjunction
96. The
commonest form

Passage XY

of aneuploidy in liveborr

gametes requires the separation of the paired chromosomes at the first meiotic division and the separation of the sister chromatids at the second meiotic
division. The failure to disjoin properly at either division

Maintaining of the normal chromosome number in the

infants is Down's syndrome (trisomy 21). It follo$. an incidence curve that is indicated in the diagrar: shown below:

chromosomes and offspring with an abnormal number of chromosomes in each cell.

will

produce gametes with abnormal numbers of


l0
8

Aneuploidy (having an abnormal number of chromosomes) is the most frequently observed chromosomal abnormality in human infants and miscarried fetuses (abortuses). The following table
indicates the incidence of this condition in both live and miscarried fetuses:

-o

4
2

a)

tr.
O
1

0.8

c)

0.6 0.4
0.2 0.1

Incidence in

Incidence in

Aneuploidy

Live Births
1/700 - l/1000 1/6000 - 1/9000

Abortuses
8/100 - 9/100 5/100 6/100
Rare Rare Rare

Trisomy

21

18 13

25 30 3-5 40
Matemal age

Ul2,O00 , 1/24,000 U975 1t930 U975


1/2500

XXX
XXY

What can be determined from this graph regardiig


the incidence of trisomy 21?

XYY
Monosomy X

- 1/5000

20/r00

Thble l. Incidence of aneuplody in livebom children and in recovered abortuses.

I. II. III.
A. B. C. D.

The incidence is lowest when a womrtt conceives a child in the latter half of tire
second decade of her life.

The incidence is approximately the same


1

:l

3-year-olds and 35-year-olds.

The incidence is highest in the age r3rgt


closest to menarche.

I only

II only I and II only I, II, and III

95.

Karyotypes are pictures of chromosomes that are

prepared

by arresting cell

development with

97. How does the karyotype of a person with D;'mn


syndrome differ from the karyotype of a perscn affected by this genetic disorder?

What information CANNOT be learned from the karyotype of fetal chromosomes obtained from an amniotic fluid sample of a fetus?

colchicine. Paired chromosomes are spread out on a slide and photographed to produce a karyotype.

A. B.

The Down individual has three copiei.


chromosome 21.

A. B. C. D.

The Down individual has three copiei


chromosome
18.

The sex cif the individual The incidence of trisomy The presence ofdevelopmental defects The incidence of monosomy

C. The Down individual has one D. The Down individual has one
chromosome 18.
chromosome 21.

Copyright @ by The Berkeley Review

556

The Berkeley Rer Specializing in MCAT

Biology
98.

Meiotic Nondisjunction

Passage XV

Does an individual with trisomy XXY (Kleinfelter's syndrome) have a definite genotypic sex?

A. Yes, the

person

is a

female, based on

B. C. No, the person

genotype. Yes, the person is a male, based on genotype.

has clearly differentiated

D. No, the XX chromosomes


for either sex.

genitalia of both sexes.

cancel out the effects of the Y chromosome, producing a person with genitalia not clearly differentiated

99.

What is the commonest form of aneuploidy among


abortuses?

A.
B. C. D.

Trisomy l8 Trisomy 21 Trisomy X


Monosomy X

100.

It is possible through an examination of restriction fragment length polymorphisms (RFLPs) to identify the parent from whom a nondisjoined chromosome originated. Given the following gel electrophoresis pattern for a child with trisomy 21, which parent or parents contributed the extra chromosomal material?

Mother

Child

Father

-IlA.
B. C. D.

-l-

| -

"9 EE
!o

Both the mother and the father The mother only The father only Neither the mother nor the father

Copyright

by The Berkeley Review

357

The Berkeley Review Speciatizing in MCAT Preparation

Biology

Genetic Information

Section IX Answers

1.

D is correct. In order to find the

percentage of iron in hemoglobin, we must first find the mass of iron in hemoglobin and then divide that value by the mass of hemoglobin itself. Information needed to do this is given in the passage. The quotient is multiplied by 1007o to give the desired answer. Since each globin protein has one iron atom, and each iron atom has a molecular weight of 55.8 amu, we multiply 4 iron atoms by 55.8 amu to get an overall iron mass of about 223 amu. We next divide this value by the molecular weight of hemoglobin, which is about 65,000 amu. This gives a value of 0.0034. Multiplying this value by IOOVo gives an overall percentage of iror in hemoglobin of 0.347o. The correct choice is D.

2-

B is correct. A zwitterion is a molecule (e.g., amino acid or protein) that has an overall net charge of zero. Er e n though the overall net chatge is zero, the molecule can still have a charge. A change from HbA to HbS involves the
substitution of a negatively charged amino acid (Glu) for a neutral amino acid (Val). HbS is now a protein with one less negative charge. But having one less negative charge on HbS does not mean that the molecule no longer has " zwitterionic form. It still does, but the zwitterionic form of HbA is slightly different from the zwitterionic form r: HbS. This allows us to eliminate choice D.
Since we are changing the zwitterionic form of the protein when we go from HbA to HbS, we must also be changin_.

the isoelectric point (pI). Remember, the pI is defined as the pH at which the zwitterion is at its maximun,
concentration. This allows us to eliminate choice A. What happens to the pI if we change HbA to HbS? It's the same as what happens to the pI of a protein when one :: its negative charges is removed. The pI increases (becomes a larger value). As you have learned in gener", chemistry, we can determine the value of an isoelectric point through titration. Suppose we start off with tu: beakers, one beaker having a fully protonated HbA protein, the other beaker having a fully protonated HbS protei:. We begin to add a strong base (e.g., NaOH) to each of these beakers. As the base is added, we begin to titrate off tht hydrogen atoms on those amino acids in each protein that have ionizable side chains. Note that HbA and HbS .rr: identical, except for the presence of valine at the p6 position in HbS. In other words, HbS has one less ionizable si.i* chain. Valine does not have a side chain pKn value and therefbre has no ionizable hydrogens on its side chain. n: HbS has one less ionizable side chain, then the pI (i.e., the zwitterionic species) for HbS will be reached sligh:-; later than the pI for HbA. The pI for HbS is a little higher than the pI for HbA. The correct choice is B.
3.

C is correct. In the second paragraph of the passage it states the pI of HbA is 6.9. A buffered solution of pH 9 ". about 2 pKa units above the pI of6.9. This tells us that HbA is negatively charged (and behaves as an anion) at pi
values greater than its pI. This can be verified by using he Henderson-Hasselbalch equation. Negatively

charll;

anions migrate toward a positively charged electrode. Thus, the positively charged electrode is the anode. This mu;r information eliminates choices A and D. Because HbS has a valine residue instead of a glutamate residue at the p6 position, it is missing one negative char::

We know that the pI for HbS is slightly less than the pI for HbA; the exact pI for HbS is irrelevant. Since r: :; slightly less than the pI for HbA, we find that HbS, at a buffered pH of 9, is also negatively charged and migra::r toward the positively charged anode. This is exactly what we see in choices B and C. Since we cannot pick b:il
answers, there must be something we have overlooked.

What we have overlooked is that both HbA and HbS, at a buffered pH of 9, contain a different amount of nege:,, * charge. HbA has one more negative charge than HbS. Since HbS has one less negative charge, it moves more slo'n l toward the anode. There is less force pulling on HbS than on HbA. This is why we are able to see a separs:--m between HbS and HbA in the two lanes in the gel. Since HbA moves toward the anode at a faster rate than HbS. r: * f'arther down in the gel. In other words, Lane 2 represents HbA, while Lane 1 represents HbS. The correct choice h
C.
4.

A is correct. The

passage clearly states that the difference between HbA and HbS is one amino acid. In the quesr::,rrwe read that each amino acid is coded for by three bases in DNA. Those three bases represent a reading frame .r** we will see later, an enzyme "reads" the bases in a particular strand of the DNA duplex and then makes a : -,li calledmessengbr RNA (mRNA). Then ribosomes next "reads" those copied bases three at a time. Each set of ti::m bases is referred to as a codon, and each codon codes for a particular amino acid. As the ribosome joins :lnn individual amino acids together in sequence, a protein is formed.

A base-substitution mutation is the substitution of one base in a DNA sequence for another base. Recall that D\*r contains the bases adenine (A), thymine (T), guanine (G), and cytosine (C). For example, suppose we examin: :'nu
by The Berkeley Review

Copyright

558

The BerkeleY Kelicmrr Specializing in MCAT Preparatitm

-,1r.

Biology

Genetic Information

Section IX Answers

base sequence in the DNA coding strand for the sixth position in both HbA and HbS. In HbA, that sequence is XXX-CTC-XXX-, where "X" is a base we are not interested in at the moment. In HbS, that sequence is -XXXCAC-XXX. If we were to look at a table of the genetic code, we would find that CTC codes for glutamate, while CAC codes for valine. In this case, there has been a base substitution (A for T) that distinguishes HbA from HbS. Note that this single base substitution does not affect the rest of the DNA sequence (i.e., it does not alter the reading

frame) and therefore does not alter the rest of the amino acids in the HbS protein. Let's consider what happens in a base-addition mutation. These types of mutations do alter the reading frame. For example, suppose we add an "A" between the "CT" bases in -XXX-CTC-XXX. The new sequence, read three bases at a time, becomes -XXX-CAT-CXX-X-. It turns out that CAT codes for valine. However, the rest of the reading frame beyond that point has changed as well. This type of mutation does not generate the HbS molecule. A basedeletion mutation has similar consequences. If we were to remove one of the bases in the "CTC" portion of -XXXCTC-XXX-, then we would again alter the reading frame. This type of mutation also does not generate the HbS molecule. The correct choice is A.
5.

A is correct. If the father has sickle-cell trait, then his hemoglobin composition is HbA and HbS. The mother, who is heterozygous for HbC, has the hemoglobin composition HbA and HbC. We can set up a Punnett square as shown
below. We find that one quarter (0.25) of the children should not have any hemoglobin disorder.
Father

AS

^[;T;_I Mother.ffi

Only l/4 of the children are free of any


hemoglobin disorder.

HbC turns out to be the second hemoglobin disorder identified. This disorder is caused by a change in amino acids glutamate to lysine at the p6 position. Individuals who carry the gene for HbC have a milder form of the disease, compared to those who carry the gene for HbS. The correct choice is A.

6.

'

B is correct. This was the first part of Mendel's experiment, by which he was able to assure himself that the forms of the traits he was studying were indeed constant, transmitted regularly from generation to generation. With this constancy in mind, he felt safe to carry out the experimental crosses. In hindsight, we must consider the entire experiment to appreciate the significance of choice B, the correct answer. From this step, Mendel could go on to carry out crosses to get the Ft generation. If the P generation had not been true breeders (if they were heterozygotes), he might have seen the recessive trait in the Fr generation and not realize that it is indeed recessive. Remember, he saw only one trait in the F1 generation of true-breeding varieties of peas, and this is considered the
dominant forrn of the trait. The correct choice is B.

7.

"
8.

D is correct. lf ll2 of the individuals in the Fz generation are not true breeders, they are heterozygotes. If heterozygotes are allowed to self-pollinate, they produce progeny that exhibit a dominant trait in a ratio of 3:l over a

recessive trait. Let's take white-flowered versus purple-flowered individuals. A heterozygous individual is Ww. If we cross Ww x Ww, we get WW, 2Ww, and ww. This shows that the expression of the dominant to recessive trait is in a ratio of 3:1. The correct choice is D.

A is correct. The second filial

F2 generation comes from the self-pollination of the F1 generation. Recall that all Ft individuals are heterozygotes, because they are the product of two different, but true-breeding parents. Therefore, if we cross Ww x Ww, we get WW, 2Ww, and ww. From this cross, it becomes clear that the ratio of homozygotes to heterozygotes is l:1. This is different from asking for the ratio ofphenotypic expression. Since the question is asking about the genotypic makeup, 1: I is correct. The correct choice is A.

9.

B is correct. A human cell that is in prophase of meiosis I has a replicated set of chromosomes, but homologous pairs have not yet separated. They still have 46 chromosomes. Even though the amount of genetic material in the
cell has been doubled, the number of chromosomes remains the same. One could call the chromatin ploidy number 4, but still only two sets of chromosomes remain in the cell. The choice becomes diploid, with n = 2. The correct choice is B.

Copyright

by The Berkeley Review

559

The Berkeley Review Specializing in MCAT Preparation

Biology
10.

Genetic Information

Section IX Answers

is ts.
11.

B is correct. We are performing a test cross to determine the genotype of the flower. We can do this by crossing it with a plant of known genotype (the white flower must have the genotype ww). If the plant in questioriis heterozygous, we have the following cross: Ww x ww. The possibilities are 2**, and 2 ww. Therefbre, if the plant in question is heterozygous, we should expect to see that 1/2 of the plants have purple flowers. The correct choice

C is correct. The pods themselves do not segregate and contain uniform types of peas as shown. Each pea within a pod is an individual case. Let us consider the other choices. The ratio of wrinkled to round seeds is ionect; but again, not all the wrinkled seeds would be fbund in one pocl. The number of progeny in the Fz generation is correct. as only two of the pods from F1 are used to create the F2 generation. Finally, we have no reason to believe that the p generation are not true breeders. In fact, we have evidence to the contrary. That evi<lence is that the peas in the F1 generation are all uniform. Again, each pea, not an entire pod, is an individual case where segregation of alleles is occurring. The correct choice is C.

12.

B is correct. The following Punnett square shows why. Each pink parent is Rr. This gives 2 homozygous whites (rr), and 2 heterozygous pinks (Rr). The correct choice is B.

homozygous red (RRr.

H l*l"l
13.

[;-EI
Thar

Rr

means choice A is incorrect. Pheromones are substances secreted by one member of a species that affect other members of the same species, so choice B is incorrect. Choice C is a nonsense answer and is incorrect. Alleles fit the detlnition given. The correct choice is D.

D is correct. Homologous pairs are chromosomes paired with each other; they are not specific gene loci.

14.

A is correct. A monohybrid cross involves individuals that differ only with respect to the alleles at a single locus The flower example assumes only one trait, color, distinguishes the flowers. Choice A is correct. A test cross
of the unknown parent. Choice B is incorrect. A dihybrid cross involves inaivlduits that differ at two alleles. Thr: would yield a 16-box Punnett square, not a 4-box Punnett square, so choice C is incorrect. Inbreeding mear, crossing closelyrelated individuals. The parents ofthe Ft generation are notclosely related. Choice D is als.
involves a homozygous parent and an unknown parent. By examining the offspring, one can determine the genolpe

incorrect. The correct choice is A.


15.

A is correct To get a prnk tlower with curly leaves, the genotype must be Rr 11. The Rr yields a pink color due i_ incomplete dominance. The ll yields a curly leaf due to classical dominance. Make a l6-box Punnett square to prc.,: the answer to yourself. Only 4 out of the 16 offspring would be pink flowers with curly leaves. This is a l_{i probability, or 0.25 since probabilities are expressed as a decimal fiaction of l. The correct choice is A.
C is correct. The passage tells us that the A and B proteins are codominant, and that O is recessive to A and B. T:t: text of the question tells us that the Rh factor is classically dominant/recessive, with the + being dominant. So rh: queen contributes A and B alleles, and the king contributes B and X (either B or O) alleles. The offspring of tl,i king and queen must be either AB, AO, BB, or BO. Since the presumed heir is A, this is compatible witlithe _tchoice. So far, so good. However, the answer lies in the Rh-factor part. Both the king and the queen a.e homozysi;; recessive, or negative, for the Rh factor. Since the presumed heir is Rh positive, this allele hadto come from anorhr f'ather. Neither the queen nor the king could have provided it. The presumed heir is not the child of the king. Choi-;, A and B are incorrect. The passage and the text ofthe question did provide what is needed to answer the qiestion. r choice D is also incorrect. The correct choice is C.
are expressed on the RBC of an individual, then that person is ty,pe Type O blood is recessive in the ABO blood group system. The person with type O blood is homozygous recessr. : A child born of two people who are homozygous recessive for the same traiiwill also be homozygous recessr,: Since neither parent expresses A or B, the child cannot express them, either. Statements II and III are incorr:r Incidentally, the same reasoning holds true for the Rh factor. The correct choice is A.

16.

17.

A is correct. If neither the A nor B proteins

Copyright O by The Berkeley Review

360

The Berkeley Revim Specializing in MCAT Preparatim

Biology

Genetic lnformation

Section IX Answers

18.

C is correct. This can be calculated from the information given inthe passage.The genome of E. coli is said to x 106 nucleotide base pairs. It is also stated in the passage that each replication fork polymerizes at a rate of 500 nucleotides per second. Since there are two replication forks (the replication is bidirectional), the total rate is
contain 4.7 about 1000 nucleotides per second or 60,000 nucleotides per minute. Simple division gives one an answer closest to choice C, 80 minutes. If one did not take into account the two replication forks, one would come up with 160 minutes, or choice A. Choices B and D can be eliminated with the above information. The comect choice is C.

19.

needed to have any sort of timely, complete replication. Let us consider the other possibilities: Choice B is incorrect, because the chromosome shown is circular. Circular chromosomes are found in prokaryotes; the question specifically asks for a eukaryotic chromosome. Consider choice C. While this chromosomes is linear, there are no replication bubbles, and the question asks for what a chromosome will most likely look like during DNA replication. Consider

A is correct. Remember, the eukaryotic chromosome has so much DNA that multiple origins of replication

are

choice D. This answer can be tempting, but one must remember that eukaryotic chromosomes have multiple transcription bubbles, and this shows only one. Yes, this may happen and does happen; but the question asks for the best representation, which is the chromosome with multiple replication bubbles. The correct choice is A.
20. C is correct. Chromatin is DNA wrapped around histone proteins. The appearance is that of beads on a string. One speculative model claims that the histone is split in half while the DNA is replicating. The two halves stay on one parent strand as the new DNA is being synthesized. After the DNA is completely replicated, the two halves recombine to form the functional histone. Consider ttp other possibilities: Choice A is incorrect, because we are talking about the replication of a single chromosome, and the presence of other chromosomes will have no effect on replication. Choice B is incorrect, because the enzymes involved in polymerizing are not prejudiced towards whether the bases are coding (exons) or are noncoding (introns). They read the template and insert the correct base pair in the growing strand. Choice D is incorrect, because the mere location of the DNA does not affect the rate of polymerization. We need to be looking at the actual structure of eukaryotic DNA compared to prokaryotic DNA. When we do this, we remember that eukaryotic DNA is packaged with proteins to make chromatin. While choice D is not the correct answer, the principle behind the claim is extremely important and represents one of the fundamental differences between -ukaryotes and prokaryotes. Eukaryotes have their DNA enclosed in a nuclear envelope, while prokaryotes do not have a nuclear envelope. The correct choice is C. 21.

B is correct. This question is very straightforward and calls upon your knowledge of the cell cycle. The replication forks will become activated when DNA is undergoing replication. This occurs in the S division of the cell cycle. Gt, S, and Gz are known collectively as interphase. Interphase usually takes up about9OVo of the total cell cycle time. Gt and G2 are stages where biosynthetic activities of the cell take place, preparing the cell for DNA replication and division. S, as already stated, is where the DNA replication takes place. Finally, M is the mitotic stage where we see both a nuclear and a cellular division. Based on this information, we are most likely to see the replication forks activated during the S phase ofthe cell cycle. The correct choice is B.
3H-uridine, which will be incorporated not into DNA, but B is correct. The question tells us we have the addition of eventually be removed from the nucleus and enter the will but into RNA. RNA ia initially made in the nucleus, to make a functional protein. Therefore, we are most likely translation it will undergo In the cytoplasm, cytoplasm. to sei the label in both the nucleus and the cytoplasm. Based on this information, we can easily eliminate choices A and C. They are both true but incomplete, and therefore not the best answers. Choice D is incorrect, because the membrane will not have nucleotides incorporated into its structure. The nuclear membrane is composed of primarily phospholipids and large protein granules making up the nuclear pores. The correct choice is B.

22.

tl

I is correct, because we see the presence of multiple replication bubbles, which should organism. Statement II is incorrect, because there is no evidence for this claim. a eukaryotic immediately suggest Yes, we see more label in B, but half of the label is less dense, indicating some sort of change in the level of radioactivity. The level of radioactivity is not an indication of the rate of polymerization. If we look carefully, the dense regions a/6 the same in both A and B, indicating that the polymerases are not different, but,exactly the same. Therefoie, statement II is false. Statement III is correct, because in B we see more silver grains from the
C is correct. Statement

radioactivity. That means that more incubation occurred. However, we can definitely recognize the low level of radioactivity. That must mean that there was a pool of unlabeled nucleotides added to the medium. When this happens, the labeled and unlabeled are competing for the polymerase, and we should see a smaller level of radibactivity. This is what we see by the lesser density of silver grains. The correct choice is C.
@

J,rpyright

by The Berkeley Review

361

The Berkeley Review Specializing in MCAT Preparation

Biology

Genetic Information

Section IX Answers

24.

D is correct. Gene duplication involves copying the gene or genes in the DNA that are already there. If gene duplication occurs, then replication of those genes into new duplexes of DNA will give the same DNA sequences Similarly, the transcription of DNA into mRNA will be similar, as will the translation of the mRNA into a potypepiiAe chain (composed of amino acids). If similar mRNA sequences can be synthesized, then similar tRl{A RNA) and even similar rRNA (ribosomal RNA) sequences can be synthesized. Therefore, choices A and B

tt.u*tir

both occur. We can eliminate them. Duplication of genes results in more of the same types of genes. And this means that more (of the same) genes can begin to diverge. Each of the newly duplicated genes can experience its own forn of mutation, independerit from the oiher duplicated genes. This is how natural selection begins to take hold. Sincc choice C is correct, we can eliminate it. tf we duplicate a gene, we will have two genes of equal size and not om hfl gene which is twice as long. Therefore, when a transcript of mRNA is made, it will still be as long as the gene is D. choice correct The was duplicated and not any longer.

25.

26.

(arl D is correct. A genetic locus refers to a given gene location on a chromosome. Males have one X chromosome foi male in the blindness for color The frequency one Y chromosome), while females have two X chromosomes. for th homozygous must be they color-blind, to be females for passage. In order (0.08) in the given as 8% Iondition. The third pu.ug.upi of the passage says that red-green color blindness is sex-linked and recessive. Whn b (0 rhe chance of a female U"eing cotor-Uiind [.e., ito*orygooi for the allele)? It would just be q2, or (0.08) x D. which is 0.0064 or 0.64Vo. The correct choice is C is correct. Paragraph 3 of the passage says that about 67o of all males who are color-blind have a deutan defo which results in abnoimal syntheais of the green-sensitive pigment. The percentage of females that would be derr color-blindis(0.06)x(0.06)=0.0036 =0.36Ta.Notethatthepercentageof femaleswhoareprotan(red-sensi pigmenr defect) color-blind is (0.02) x (0.02) = 0.0004 = O.04Vo.If we add 0.36Vo and 0.04Vo, we get 0.40%" inIia"n." of red-green color blindness among females in the United States. The correct choice is C.
B is correct. In the passage we learn that about 6Vo of all males who are color-blind have the deutan defect while of all males who are coloi-blind have the protan defect. In a heterozygote, the dominant allele is p and the reces allele is q. We see that q = 27o = 0.02 and that p = 987o = 0.98. From the Hardy-Weinberg equation, we find thm heterozygote women u'ho = 2(0.98j(0.02) = 0.O3gZ x lOOTo = 3.927o or about 4Vo. Note that the percentage of ll%o. lf we add 1l7c + ll.287o or about 1007o = .u.ri.r, of deutan color blindness is 2(0.94X0.06) = 0. 1128 x defect. The blindness carry the color who women of percentage then we get 157o, which is roughly the B. choice is

27.

28.

D is correct. If we let the dominant allele for deutan color blindness


by d. If the husband is deutan color-blind,

be represented by D, then the recessive recessive allele, d, on his one and he has the can be represented chromosome. His wife has normal color vision. She has two X chromosomes, and they both carry a dorn allele (color

Father blind)

Mother (normal)

II
xtxd
Daughter

XDY
Son

(carrier)

(normal)

If they have a daughter, then she will receive one X chromosome with the D allele from her mother and chromosome:with the d allele from her father. The daughter will be heterozygous for the deutan trait. In other she will be a carrier. If they have a son, the mother donates an X chromosome with the D allele but the falhu donates a Y chromosome, which has no defect for color blindness. The son will thus have normal color visimcorrect choice is D.
29.

C is correct.The easiest way to follow the alleles is to draw a pedigree as shown below. The symbol D rep the dominant form of the allele for the deutan trait, while the allele d represents the recessive form. In order
@

Copyright

by The Berkeley Review

362

The Berkeley Specializing in MCAT

Biology

Genetic lnformation

Section IX Answers

trait to be expressed, the genotype must be dd in the female or d- in the male. The symbol C represents the dominant autosomal allele required for proper development of the cones. If the genotype cc is present, the individual will have complete color blindness.

Father
(does not have deutan trait) (does not have color vision)

Mother

(will express deutan trait) (will have color vision)

It
Daughter (carrier tbr deutan (will have color

vision)

trait)

XDXd
a

xdv

Son

.
I

"ll'

(will express deutan trait) (will have color vision)

By examining the pedigree, we

see that both children will have color vision. However, only the son green color blindness by expressing the deutan trait. The correct choice is C.

will show red-

30.

B is correct. The last paragraph of the passage says that if a hamster is homozygous for the a allele (i.e., a/a),then its coat will be white. In other words, the hamster will be albino. The same paragraph also states that the normal (wild-type) allele is usually designated as +. If a hamster has a + allele at one of the two loci, then the coat color will be normal. This must mean that the + allele is dominant over the a allele, or that the a allele is recessive to the + allele. If the a allele and the + allele were codominant, we would see both types of coat color. If the + allele showed incomplete dominance, we would also expect to see both types of coat color. Also, the a allele, as written in the answer choices, is written in the lower case. This is the nomenclature that has been agreed upon by geneticists to indicate a recessive allele. The correct choice is B. C is correct. A true-breeding strain of black hamsters will always produce black hamsters (unless there is a mutation of some type). In order for progeny hamsters to be purely black, they must always receive a B and a + allele from each parent. Therefore, each parent and the progeny hamsters must have the genotype B/B +/+. The correct choice
is C.

31.

7''

C is correct. In the third paragraph of the passage, we see that the recessive allele Z' results in brown coat color when it is present in the homozygous (b/b) condition. The fourth paragraph says that if the hamster has the allele a present in the homozygous (a/a) condition, then the coat color will be white. The genotype a/ameans that these hamsters do not have any pigmentation (that is why they are white). The correct choice is C.

33.

A is correct. We can use the outline shown below to help


generatbn progeny.
Black coat

us determine the coat colors

of the first filial (Fr)

color
Gamete

Parents (P1)

Gamete

lormation

formation

X
Cross

ffi

ba

0
Firstfilial(Ft) generation

re
a=:r---:-:

:--Onr'---.-T:

Blackcoar color

Copyright @ by The Berkeley Review

363

The Berkeley Review Specializing in MCAT Preparation

Biology

Genetic Information

Section IX Answers

We see that in the F1 offspring, the genotype is B/b +/a. Since B is dominant over b, the coat color will be black Since the wild-type allele + is dominant over the a allele, the coat color will be normal, which in this case is black. If there had been two a alleles (e.g., a/a), the coat color would have been white. The correct choice is A.

34.

B is correct. In order to estimate the genetic map distance between the two genetic loci, we must use the equation
given in the passage, which is:
Vo

CO =

Number of Crossover Indidivuals in Total Number of Individuals in F2

F2

lOOTo

We need to determine the number of crossover individuals in Fz, and we need to determine the total number of
individuals in Fz before we can estimate the genetic map distance. Refer to Experiment II. In Experiment II, we learned that one hamster from the Ft generation of Experiment I was back-crossed to the pareil with the double recessive genotype (b/b a/a). We determined that the genotype of the hamster from the Fl generation is B/b +/a. The result of this cross will give the Fz progeny, namely 66 black hamsters, 34 broc,r hamsters, and 100 white hamsters. The total number of hamsters is 200. At this point we know the total number d individuals in Fz. Our equation now becomes:
vo

CO =

Number of Crossover Indidivuals in 200

F, x

loovo

This alone does not tell us the number of crossover individuals we have. Let us next assume that the two loci linked, and let us also suppose that no crossover events took place. What would we expect? Let's look at this cross as outlined by Experiment II. We can set up the chromosomes as shown below and generate the Fz progeny'.
First filial (Fr)
generatlon

Blackcoat

color

ffi

B+
ba

<==-a.-"llr:l.-:rlr--;=-

ttlr:rl-*irirCl (...=f........ -T-.. a

ba

Whitecoat color

Gamete

Gamete

formation

formation

ba

ba ffifir,'fir.rrre

X
Cross

--_--\\ \ \
Gamete

reba
rTr
@
(:ir:l_lriTl*.Iillll'l!:

Gamete

(1.:rrlx'|:=l*.E1

B+
ba

!
b

Black coat color

Gamete

White coat
color progeny from test cross

F2

Copyright @ by The Berkeley Review

36.4

The Berkeley Specializing in MCAT

Biology

Genetic lnformation

Section [X Answers

If no crossover took place, we would expect to find half of the progeny B/b +/a and half the progeny b/b a/a. In other words, we would expect that 507o of the hamsters would be black and 5OVo would be white. We would not expect to find any brown hamsters. But this is not what is observed in the results of Experiment II, which was 34 brown hamsters. This tells us that the brown hamsters are the crossover or recombinant hamsters. If we use this
value in our equation, we get:
7o

CO

= 34 *
200

l00Vo

lTVo

Since one genetic map unit (m.u.) gives a recombinant frequency (RF) of I percent, we have a recombination frequency of l7% or 17 m.u., which is choice A. But choice A is not the correct answer.

if a crossover event does take place. In paragraph 4 of the passage, we read: "If there is crossing over between any two non-sister chromatids, intrachromosomal recombination results." Crossing over must take place between non-sister chromatids. One possibility of how this could happen is shown below in Crossover 1;
Let's see what happens

<-:-r:;.=:-:.o-lrl;;r=Crossover I
b
b

a
Crossover

ba
:

ba ba <rl--r---:;r;Ct :: re

<=.--lr:'-.-.

cr. :;r=='

Blackcoat color

Whitecoat color

There is also another possible crossover event, as shown below in Crossover 2. Again, this crossover is taking place
between non-sister chromatids

@
Crossover 2

ba
B b
+
Crossover

<:l:r.Tl:ll..--r::. ffi::i:-:tr-ll.n

Ba

Whitecoat color

rcba

(::lirl-:]..:..TrJii:l==rrili,, Browncoat

b+f

.re

color

In Crossover l, we produced black and white hamsters in equal numbers. In Crossover 2, we now produce brown and white hamsters in equal numbers. We can clearly tell from the phenotype which are the hamsters are brown, and we know that these brown hamsters can arise only from crossing over. What about the white hamsters? We can get white hanrsters from non-crossover events (see above). How can we tell which white hamsters came from noncrossover events and which came from crossover events? We can't! Here is where the insight comes in. In order to get a more accurate number of recombinants, we must assume that there are also 34 white hamster recombinants. This is because when we produce one brown hamster recombinant, we also produce one white hamster recombinant. And since we have 34 brown hamster recombinants, we must add an additional 34 white hamster recombinants to the total number of crossover individuals in the Fz generation. (Why don't we need to do this for Crossover I ?)
We can now use our equation to calculate the genetic map distance between the two loci:

ToCO

= 2 x 34 x
200

IOOVo

347o

In other words, there are 34 m.u. between the genetic loci. fNote: Since there can also be double crossover events, the actual grhetic distance between the loci is probably greater than 34 map units. What we have calculated is the
minimum map distance.l The correct choice is B.
35.

A is correct. If the genetic loci were not linked, we would expect to see a recombination frequency in the Fz generation of 5O7o (from the third paragraph of the passage), which means that there was interchromosomal recombination (i.e., between the chromosomes). If the recombination frequency is less than 5OVo, it means that there
@

Copyright

by The Berkeley Review

365

The Berkeley Review Specializing in MCAT Preparation

Biology
then genetic loci must be linked.

Genetic lnformation
If

Section IX Answers
there is intrachromosomal recombination.

was intrachromosomal recombination (i.e., within the chromosome).

There is another way to look at this: We would expect half of the hamster offspring to have white coats, which thel' do. What about the other half of the hamster population? If the two genetic loci were not linked, we would expect to see this half of the hamster population having 507o black coats and 50% brown coats. But when we look at the values given in Experiment II we do not see this. Instead, we see that there are almost twice as many hamsters with black coats as there are with brown coats. In order for this to happen, the genetic loci must be linked. If they were not linked, they could be called unlinked (i.e., they assort independently). The genetic loci are not entirely epistatic" because we do see black and brown hamster coats. This means that the a allele is not homozygous (a/a) but rather heterozygous (a/+).The a allele has to be homozygous before it can influence a different genetic locus. Besides, the B and b genetic loci are not epistatic to the a allele. The comect choice is A. 36.

B is correct. In the sixth paragraph of the passage, we see that neither the black nor the brown coat color of a hamster will be expressed, if the hamster is homozygous for the a allele. For example, consider these chromosomes:
B .-TIr1Ir'T:T.Efl::=t.-'It)

rerF

If the a allele were not present, the coat color of the hamster would be black. This is because the B allele is domin*m over the b allele. However, the presence of the a allele, in the homozygous form, suppresses the activity of the alleh for the black coat color. Because the a allele is recessive (note that it is written in a lower case), we can cali rhrn recessive epistasis . As stated in paragraph 2 of the passage, incomplete dominance and codominance are concerned with heterozygotes. Both of these characteristics involve expression of either one of the same allele (e.g., B or b). [o the case of epistasis, allele a influences some other allele, such as allele B or b. The correct choice is B.

37.

D is correct. Since the average molecular mass of an amino acid residue is I l0 D, the protein has 40,000/110 = 360 residues. This is specified by 3 x 360 nucleotides. Each base pair has an average molecular mass of 660 D. Hera,;cthe molecular mass of the DNA is 660 x 1080 = 700,000 D (or 7 x 105^ D;. The length of this molecule, considening that B-DNA has a rise along its helix of 3.4 A per base pair, is 3.4 A x 1080 = IIOO it = 0.37 pm. The "o-"i choice is D.
C is correct. There are 4.0 x 106 bp in the E coli chromosome, and it takes 40 minutes to replicate the chromosom Thus, (4.0 x 106 bp)/(40 min) x (1 min/60sec) = 1,666 bp/sec replicated. But since there aie two replication fodq there are about (1,666 bplsec)l2 or about 833 nucleotides per second added. The closest answer is 850 nucleoridse" The correct choice is C.

38.

39.

B is correct. At 16 pm/min, each replication fork travels 4800 pm or 4.8 x l0-3 m in 5 hours (300 minutest replicate'theentirecontentof DNA(1.2m)inthisinterval,theremustbe(1.2m)/(4.8x 10-3m/replicationfoikr*
250 replication forks. The correct choice is B.

40.

D is correct. In a rich medium, a second round of bidirectional replication begins at the origin when the first is only half-completed. This second initiation results in four new replication forks, making a total of six.

6 replication forks

Thus, one round of replication is competed every twenty minutes, and each daughter cell at division receives chromosome that is already half-replicated. The correct choice is D.
Copyright by The Berkeley Review

366,

The Berkeley Reviev Specializing in MCAT Preparation

Biology
41. 42. about 4 x 106 bp. Thus,

Genetic lnformation

Section IX Answers

choice is C. 43.

C is correct. okazaki fragments are about 1000 to 2000 nucleotides in length. The E coli chromosome consists of E coll must produce about 2000 to 4000 okazaki fragments. The correct choice is C. C is correct. Since DNA is antiparallel, the complementary strand must be 5'-pGpTpCpTpApT-3'. The correct

C is correct. In the next-to-last paragraph in the passage, it we read that "...all known DNA polymerases require a primer before new DNA can be synthesized...." And in the last paragraph of the passag", *" u'r" told that .,Once the primer has been established,_DNA polymerase III begins to syntheiize DNA in the 5' to 3' direction.,' From this information, we can assume that DNA polymerase I must also iaa itr nucleotides in the 5' to the 3' direction (i.e., at the 3' end) of a growing polypeptide chain and not in the 3'to the 5' direction (i.e., at ttre s; ena;. This allows us to eliminate statement II, which in turn allows us to eliminate choices A, B, and D. The correct choice is C.

44.

C is correct. There were two ways to answer this question. First, you could have remembered that the base-pairing arrangement in a DNA double helix (a duplex) follows the rules that adenine (A) base pairs with thymine 1rj through 2 hydrogen bonds and that guanine (G) base pairs with cytosine (C) through : nydrogen bonds. fne onty base-pairing arrangement in the answers that shows this is the bonding between cytosine and guanine. The adenineguanine and cytosine-thymine base-pairing is not allowed. This allowi us to pick choice C u, ih, unr".r ff you Jia not remember this' then.you needed to rely on information in the first paragraph of the purrul". It was stated that the purine bases were adenine and guanine, while the pyrimidine bases were-thy-in" uni .i;;.. Ir was also stated that.complementary purine and pyrimidine bases ai" tint"o together through hydrogen bonding. In the answer we need to find that base pair that depicts a purine bonding with a-pyrimloine. itre onry""rroi.. ir the cytosine-guanine pair' The adenine-guanine pair representi two purines,lhile thi cytosine-thymine pair represents two pyrimidines. The correct choice is C. D is correct' The second paragraph of the passage says that "conservative replication would conserve the integrity of the parental strands in the DNA duplex urt"r ..ptl"ition." In other words, uitr. on" round oIDNA replication, we would see one DNA duplex that contained both parental strands, and another DNA
duplex that contained the two

45.

new daughter strands. This is shown schematically below:

One round
6 o

of

replication

.--_----_------./

3R

Effi^,,Fg]

Dupld{ DNA

RlJ

duplexes contain all daughter DNA.

contain all parental DNA, and two

strands each giving rise to duplex <iaughter strands of DNA. Thus, in the'end we"haue four duplexes of DNA. Two of the duplexes

both duplexes are now treated as parjntai

When the second round of replication begins,

bf

the

As the second round of DNA replication begins, we now treat both duplexes as being parental strands. After the ,second round of replication, we would find two DNA duplex that contained all pa"rental DNA and two DNA duplexes that contained all new daughter DNA. The correct choice is D.
46.

47.

c is correct. This question is designed to get you to think about the fundamental components of DNA. The answer cannot be obtained from the passage. Instead, it must be deduced from your fundamental knowledge of DNA. As mentioned in the question,-DNA is composed of nucleotides. Each nucleotide contains a base (either thymine or cytosine, which are pyrimidines; or adenine or guanine, which are purines). Eactr nlt-genoni, uur. is attached to a ribose ring,(i."', u pentose sugar). Each ribose is attached to a phosphate. Each oithose ring systems could incorporate 15N. The ribose ring does not contain any nitrogen, und it seems like a good answer choice. But what about the acetal dnkages? The linkage between the sugar -a tn. nitrogenous base is referred to as a glycosidic bond'rn particular, it is.an N-glycosidic bond becauie of its attachment to a nit.olen uio- in the base. This particular type oflinkage is also considered an acetal linkage. And sincethe acetal linkate contains a nitrogen atom that could be replaced by l5N, we can eliminate it as a possilble answer. The correct choiie is C. C is corect. The passage mentions that the CsCl solution is less dense near the top of the test tube and denser near the bottom of the test tube. If we were to centrifuge the DNA rhar was labeled .^.iuriu"iy *iir.llN, *";;;;il;
@

Copyright

by The Berkeley Review

367

The Berkeley Review Specializing in MCAT preparation

Biology

Genetic Information

Section IX Answers

that its band would appear at a lower position in the test tube compared to DNA that was exclusively labeled with t4N. This is what is represented by the control test tube (see below).

AII

14N

DNA

fr

\'l-_

uuuna l+N7tsp

pp4

AII15N DNA

Control

one generation following the incorporation of "14N into the growth medium, then that DNA would be neirher exclusivel/all heavy 11sN) nor exclusively all light 1laN;. Instead, the DNA would represent

If we analyze the DNA after

a hybrid of laN and 15N DNA. The banding pattern (after analysis with ultraviolet absorption) would
choices.

be

intermediate between the two bands shown in the control. Note that we do not see this pattern in any of the answer

15N; and duplexes After two rounds of DNA replication, we find duplexes that contain the hybrid DNA (14N and that contain exclusively

expect to find two bands: One intermediate between the all-l4N as the all-laN band on the is C. choice control. There is only one answer choice that gives this pattern. The correct

tigntbNe (llN). W" wouid

[""J""J

irr" all-lsN buni l"hu.u"teristic of the hybrid DNA), and one that is the same

48.

A is correct. As described in the passage, one round of semiconservative replication would give two duplexes of l4N. After the second round of replication' each DNA, each containing one strand with-l5N and one strand with hybrid duplex would-generate.two duplexes. One duplex would still be a hybrid 1t+p-tstri;, but the other duplex would coniain DNA that is all 14N. This is outlined below.
l4N
I

15N
I

rN@\@\@ lt {
l''t
Ir
Parental

l5N

u@\@\@ -tlttN
I

duplex DNA

LN@\@\@
'oir
1st Generation

i N@\@tu4
2nd Generation

lsN

In the second generation, note that there are four duplexes of DNA, for a total of eight strand,s of DNA. Out of these l5N of 3:1. The correct eight DNA striands we see that two are l5N and six are l4N. This gives a ratio of laN to
choice is A.
49-

A is correct. The characteristic that distinguishes one atom from the next is the number of protons contained within an atom of a particular element. The atomic mass unit (amu) of a proton is about 1.0073. If the number of protons changes, the atom changes, and therefore its elemental name changes as well. Any mass difference between atoms
of the same element is due to the difference in the number of neutrons. These atoms are referred to as isotopes. The amu of a neutron is about 1.0087. The natural isotopes found in the nitrogenous bases are hydrogen (atomic mass of 1.01), 12C (12.00), t+N 1t+.Ot;, and 160 (16.00). The corresponding heavy isotopes would be deuterium (2.01), l3C (13.01), tslt (IS.OO), and 18O (18.00). Heavy isotopes have a greater density, because they have one or more neutrons in their nuclei. Electrons have an amu of about 0.0006 and do not significantly effect the mass of an atorru The same number of water molecules would surround l5N DNA as would surround laN DNA. The correct choic is A.

Copyright O by The Berkeley Review

368

The Berkeley Review Specializing in MCAT Preparation

Biology
50.

Genetic Information

Section IX Answers

B is correct. First, note that the polymers of DNA and RNA in the question have the same base composition. Therefore, we must look elsewhere to find differences in the density of these two nucleic acids. It is important to know the basic differences between DNA and RNA. We know that DNA contains adenine (A), guanine (G), cytosine (C), and thymine (T). RNA contains the same bases, except it replaces thymine with uracil (U). The question avoided the differences in the structures of thymine and uracil by leaving them out of the question. One other important difference is the presence of a hydroxyl (OH) group at the C-2' position of the ribose ring of RNA. DNA does not have this C-2' hydroxyl group (hence the name "deoxyribonucleic acid" for DNA). This tells us that each nucleotide of the DNA polymer is missing an oxygen atom. In other words, the RNA polymer has ten more oxygen atoms than the DNA polymer. The RNA polymer is heavier and shows a greater density. This allows us to eliminate choices A and C. Consider choice D for a moment. Nucleic acids do not have positively (or negatively) charged nitrogenous bases at physiological pH. The only charge they show is on their negatively phosphate groups. We can eliminate choice D. The positively charged cesium ion (Cs@) can bind to the negatively charged phosphates of both DNA and RNA. It can also bind to those free hydroxyl groups at the C-2' position of RNA as well. The correct choice is B.

51.

A is correct. Examine the

bases shown in Figure I of the passage. The purine and pyrimidine bases can exist in alternate forms called tautomers. Keto groups can be converted to enol groups, and enol groups can be converted to keto groups. Similarly, amino groups can be converted to imino groups, and imino groups can be converted to amino groups. Selected examples are shown below:
OH

",ffup
dR

r\-ti
Guanine (enol)

l-A
{\-)
I

f^o

OH

*"/ n.4. .-)

tfoi+ "-A'(^Ai
H

dR

dR

Cuanine

(keto)

Uracil

(keto)

Uracil

(enol)

Adenine

(amino)

Adenine (imino)

In order to form the enol in one of the bases, we must first have a keto group in the molecule. Not only that, but we must have a hydrogen atom on a neighboring nitrogen atom that can leave and participate in the formation of the enol. All of the bases except adenine have keto groups. Even though cytosine has a keto group, there is no hydrogen atom on a neighboring nitrogen atom that can participate in enol formation. Therefore, the bases adenine and cytosine cannot form enol derivatives. The only bases that can form enol derivatives are guanine, thymine, and uracil. What about adenine? This base has an amino group at the C-6 position of the purine ring. Tautomerism leads to the imino form. The correct choice is A.
52.

C is correct. DNA and RNA are polynucleotides that contain repeating nucleotide units linked together through
phosphodiester linkages. Remember, a nucleotide is composed of a base, a ribose sugar, and a phosphate group. The phosphodiester linkages are formed between the 5' carbon of one ribose ring and the 3' carbon of the adiacent ribose ring (see below). In order for the acidic phosphate group to be positioned between the two ribose rings, there was a loss of water during the reaction. Anhydrous refers to the loss of water. Therefore, this phosphodiester linkage can also be called an acid-anhydride linkage. At neutral pH, each nucleotide contains a negatively charged phosphate group.

\o

cH^

-\2"v
I

Base

-'y+
OH
o
I I

Phosphodiester

Ol o-P=o

linkage

'hil^"
3' end of the polymer +HO
H

Copyright

by The Berkeley Review

569

The Berkeley Review Specializing in MCAT Preparation

Biology
If

Genetic lnformation

Section IX Answers

the nucleotides were deoxy at the C-3' position, the DNA polymer would not be able to form, because the oxygen atom of the hydroxyl group at the C-3' position attacks the nexi incoming nucleotide's phosphate group in order ro form phosphodiester linkage. The correct choice is C. 53.

following sequence:

D is correct. Selection of the correct answer depends on things. First, we must obtain the correct sequence ot .two the desired DNA strand in the 3' --> 5' direction. Following the fatson and Crick base-pairing rules, we get rh.
3'-T-A-C_c_c_C_T_A_5'

Notice that in all four answer choices the DNA bases read the same in the 3' -+ 5' direction. Be aware of rhe sequence from both ends of the polymer. Just because we are looking for a sequence that reads from the 3' -+ _i direction does not necessarily mean that it will be written with the 3' eni on the leti and the 5'end on the right.
" DNA polymer represents the C-3'iydroxyl group on the ribose ring. The 5' end of the DNA polymer represents rhr p\osp\ate group attac\red t9 r\e a-5 \r'ydrox'y\ group o\ t\e rrbosi rrng.\l e are \ookng ior' oo ooa*., chcl,:: -:r,irii beans ap\r.ospha\e (P) at.the 5' end, anil ahydroxy\ !o\\) ar the 3 end.'when writrngiout u DNA sequen;e. .r so-metimes customary to include the phosphate group at the 5' ends of the bases. rn, iTnyiri-yl groupsare lerr --,1r of the picture. They are understood to be at the 3' end. In choices A and B, we find a phosphate at the 3, posiri which is not allowed" Therefore, we can eliminate the first two answers. In choice C, tie phosphate at the 5' en: ;
.

Once we have determined the sequence, the second thing we neecl to consider are the 3' and 5' ends. The 3' end of

mu

missing. We can eliminate this answer, too. In choice D, we see that the phosphate group ii at the 5, end. The 3' e r: shows no phosphate group. It does not show a hydroxyl group either, but the hydioxyi group is understood to b.r there. The correct choice is D.

54.

per turn of the DNA double helix. The correct choice is D.

D is correct. The distance between base pairs is 0.34 nm. The helix undergoes one complete turn (twist) even i -r nm' This tells us that there are l0 base pairs (3.4 nml0.34 nm = 10) per turn of the DNA double heljx. Wl mi_sh: :,: inclined to pick choice B This would be wrong. Remember, it is 10 base pairs per turn. A base pair is compoieJ :, two bases held together by hydrogen bonds. The question specifically aski for ihe number of baies (not base p: :.

D is correct.The classical hydrogen bonding between base pairs in DNA occurs between adenine and thymine :.. : hydrogen bonds) and between guanine and cytosine (three hydrogen bonds) as shown in the following drawing. T:,.r allows us to immediately eliminate choices B and C.

Cytosine

Hydrogen

Thymine
CH,
I

aY"'
Riuo,e-

,,,,,,,('ono
H _

)f o,,,,.

N"',,,,,,,

H. ..T ./
H

I jr\ ---"-\
Guanine

"'il

niuo..t

aYu"',,,,,,, Nf N'
",,,,,,,,,,,"

X] *.. (.JL J
f,,0o..

Adenine

What about choices A and D? Both answers contain bases (adenine and thymine) found in the DNA double h.';r, Both answers also contain uracil. a base found in RNA.

Transcription Bubble

A-G-T-6-A-G_^
5'
J

i;;-;-'l
mRNA

t_r*tr. (Y^-Mt

Hybrid
37o

+ DNA-RNA

DNA

3'

Copyright

by The Berkeley Review

The Berkeley Revier Specializing in MCAT preparation

Biology

Genetic Information

Section IX Answers

Consider choice A. Can uracil hydrogen bond to adenine? If it can, that base pair associated with DNA? The answer to both of these questions is, yes. As the DNA double helix unwinds and forms a transcription bubble, two single strands of DNA are exposed. One of those strands is used as a template to make -".r"ng.. RNA (mRNA;. As ihe mRNA is synthesized, it is hydrogen-bonded to the DNA. During transcription, a ONe-RXa hybrid is temporarily formed (see above).

If these is a cytosine (C) in the DNA template, then in the mRNA a guanine (G) is incorporated. If there is a thymine (T) in the DNA template, then in the mRNA an adenine (A) is incorporated. If there is an A in the DNA template,
then a uracil (U) is called for in the mRNA.

Uracil

Ribose-

Z\ao",,,. tt .N- ,N-

Ht,,,,,,t,,,,i,I)
t*,0o."

H-N-H

Adenine Remember, in RNA the base U replaces the base T. A hydrogen-bonding arrangement between adenine (in DNA) and uracil (in mRNA) is shown above. The correct choice is D.
56.

cytosine (C), and thymine (T). A triplet of these bases represents a codon, and a codon specifies a singlJ amino acid. There are 64 different codons (from 43 = 64), and among these 64 coclons there is reduniancy. In othlr words, more than one codon can code for the same amino acid" Because of this, the genetic code is referre<l to as being degenerate. If there were no mechanism that would allow DNA to replicate itself, then the information contained in these codons would not be passed to the next generation. Thus, a means of self-replication is crucial. As the information is replicated, the fidelity must be maintained. If a mutation were to be incorporated into the next generation of DNA, then it could have deleterious consequences. Therefore, a very low mutation rate is essential. DNA can exist as either a single strand or a double strand. In the doubled-stranded form, it can exist in a number of different states (e.9., A-DNA, B-DNA, or Z-DNA). DNA is said to have variable conformations. The information passed on to the next generation is still contained in these different forms of DNA. It is just a matter of having the appropriate mechanism to transmit that information. DNA, as a conformationally variable molecule, woulcl not-pose a problem for the transmission of genetic information. The correct choice is D. 57.

D is correct. In order to transmit information accurately from one cell to the next, there must be an alphabet that makes the communication possible. In DNA, this alphabet is composed of the bases adenine (A), guanine (G),

C is correct. If there is an increase in Tm, it means that it takes a higher temperature to melt (denature) the DNA. This must gean that the DNA is stabler. What would make the DNA stabler? If there were more hydrogen bonds between the base pairs, then the stability of the double helix would increase slightly. Since there are three hydrogen bonds between G'C base pairs and two hydrogen bonds between A.T base pairs, we would want an increaie in the amount of G'C base pairs and/or a decrease in the amount of A.T base pairs. Both of these factors lead to an increased stability of the DNA duplex. We can eliminate choices A and D.
What about the divalent Mg2+ ion? DNA is quite negatively charged. If these negative charges were not shielcled from one another, they would tend to blow the duplex apart (i.e., make it less stable). The result is a decrease in the Tm. However, if there is a higher concentration of Mg2+ ions in the medium, there is also a greater chance of these magnesium ions associating with the negatively charged phosphate groups and shielding those negative charges from one another. The DNA double helix becomes slightly stabler. We can eliminate choice B.
Histones are proteins that bind to DNA. These proteins have a large proportion of the amino acid residues arginine (Arg) and lysine (Lys). Both of these amino acids have side chains that are positively charged at physiologi.it pH. These positively charged residues ionically bind DNA's negatively charged phosphates and heip stabilize the molecule. Histones can be dissociated from their interaction with DNA by using a sufficiently concentrated salt solution that interferes with these ionic interactions. The DNA becomes slightly less stable (due to the deshiel<ling of the negatively charged phosphates) and easier to melt. The correct choice is C.

Copyright @ by The Berkeley Review

371

The Berkeley Review Specializing in MCAT Preparation

Biology
58.

Genetic Information

Section IX Answers

a modest understanding of the Meselson-Stahl experiment performecl in 1957. Recall that Meselson and Stahl were able-to prove that DNA replicated semiconservatively. They grew the bacteria E. coli in a growth medium containing 15NH+CI for many generations. At a particulur *o*.ni in time, they then transferred the bacteria to a growth medium containing laNH+Cl. At the time of transfer, they analyzed the bacterial DNA and found it to contain the heavy isotop_e of nitrogen, l5N. After one generation, they founcl that the DNA in one strand of the double helix contained l5N, while the DNA in the other strand containe<l laN. After two generations, the DNA was diluted even more with laN.

A is correct. The question is based on

We would therefore expect lo fild two peaks in the graph. One peak would correspond to two duplexes of DNA, both of which were hybrid 155- lay DNA. The other peak would Corresponcl to two d-uplexes of DNA, both of which -50Vo were all l+N pNA. ln other words, each peak would correspond to of a part'icular type of DNA. We can eliminate semiconservative DNA replication as_ a possible answer choice. Note that in semiconservative replication, the DNA strands that contain the l5N and 14N strands become increasingly rare as the number of bacterial
generations increase.

15y 15y

layTlay
50Va

1sp71451
5O7o

'-TiJ,"j"X."7
15111

la1g

[1

a,
l4ir1

-\O oN

1411

l5p

cg

c*:|:n""y'/
ls1..1

o"#r,."

l[ [[ [[

la11

lIf

lay 14111

l4p

/Wr

&
Direction ol Sedimentation

rrl

lary

15y

ill

What would the replication process look like for conservative replic_ation? In conservative replication, the original parental strands of the DNA double helix (which are labeled with 15N) serve as templates for new daughter DNA. However, after the first round of replication, we find that the parental strands recombine. Two duplexes result. One is completely labeled with lsN, and the other is completely labeled with laN. This type ofreplication continues_through each succeeding generation. Note that after two generations, we have one duplex completely labeled with l5N, and three duplexes completely labeled withlaN. The co.."rponding graph shows two peaks. One peak represents that DNA which is completely labeled with lsN (25%). The other peak

represents that DNA which is completely labeled with laN (157o).

o
o
LF

l5y
G"":::n",

151..1

14p

14

9:
'd
e.

d\O

lsy 1511

y'/

llf

lay 14111 l4p l4p [[[[

/an \lay l4y


[[

Direcl ion ol' Sedimentation

r)

""#1,'*'ll What
Copyright
@

about dispersive and end-to-end DNA replication? After one generation, we see that both types of replication

followthesemiconservativemodeinthateach
by The Berkeley Review

duplexishalfheavyll5N)andhalf light(l4N).

372

The Berkeley Review Specializing in MCAT Preparation

Biology

Genetic Information
l51g 15y
15111

Section IX Answers

l5p

'-'TiJ,"ilKo',/
rst

llX
1s5714P
Dispersive Both

DNA

ll

Generatio"

II

Both are

IIare

it
Both are 1sP7l411 End-to-end
conservative

tt
Both are
1sp7141g

lspTlay

if we were to denature these duplexes after the first generation, we would find that every single strand would be composed of half heavy (lsN) and half tigtrt 1l4N; DNA. This is quite different from what we would find if we denatured a DNA duplex after the first round of replication of either semiconservative or conservative DNA. What do you think the graphs would look like for these two proposed types of replication? The correct choice is A.
However,

59.

as organelles. Therefore, there should be no protein transport from the cytosol into the chloroplast. There is protein transport from the cytosol to the mitochondria, nucleus, and peroxisome. A peroxisome is an organelle that carries out oxidative reactions. It has no genome and is surrounded only by a single membrane. Human beings are not sessile organisms. Therefore, we do not need to photosynthesize in order to obtain energy. The correct choice is D.

D is correct. Human cells do not have chloroplasts

60.

D is correct. From the passage, we learn that the mitochondrial genome is very similar to that of the bacterial genome. Therefore, one must realize that a bacterial genome does not have its DNA packaged with histone proteins, as is the case with human DNA. The mitochondrial genome has distinct promoters and must have both DNA polymerase and RNA polymerase in order to replicate and transcribe its genome. Thus, the mitochondrial genome
lacks histone proteins. The correct choice is D.

61.

D is correct. This problem requires the we know which amino acid is called for by a start codon. It is neither histidine or leucine. Therefore, choices A and B can easily be eliminated. Now the question becomes, is it methionine or N-formylmethionine, the modified amino acid used in prokaryotes? There is our big clue. The mitochondrial genome is very similar to the prokaryotic genome. Thus, mitochondria use N-formylmethionine
instead of methionine. The nuclear genome uses methionine. The correct choice is D.
sequences in the mitochortdrial genome. Let's consider the other possibilities. Choice A is a true statement. There will be fewer RNA molecules coming from the mitochondrial genome. However, the reason is not because every nucleotide in the mitochondrial genome is a coding nucleotide. The reason is simply the size of the respective genomes. Therefore, choice A can be eliminated. Choice B is a false statement and can be eliminated. It now becomes only a matter of discriminating between choices C and D. The correct choice is D.

62.

D is correct. If every nucleotide is used for coding purposes, this leaves little room for any regulatory

63.

B is correct. The question is asking why we see a different genetic code in the mitochondria. The answer lies in the volume of proteins that are produced by the genome. Relative to the nuclear genome, there is a small number of proteins produced by the mitochondrial genome. Thus, a change in the genetic code is not very far-reaching. In other words, the change is tolerable because more likely than not, a small number of proteins would be affected. Furthermore, the changes that do occur in the protein as a result of the change in genetic code may be harmless (they may not affect function). Because of the increased number of proteins that are coded for by the nuclear genome, there is a higha probability that a change in the genetic code would be very far-reaching, affecting a good many proteins. Therefore, the understanding comes from thinking about the probabilities and how that relates to the size of
the genome.

Consider the other choices: There is no evidence that genetic drift occurs only in mitochondria. Furthermore, it would be difficult to rationalize how such a force would affect only one particular genome. The reason we do not by The Berkeley Review

Copyright

373

The Berkeley Review Specializing in MCAT Preparation

Biology

Genetic lnformation

Section IX Answers

see the drift in other genomes probably has to do with the elaborate proofreading systems that maintain their fidelity. Therefore, we can eliminate choice A. For choice C, we have no reason to believe that the tRNAs in the mitochondria have any special correcting function. They are bound by the same physical laws that all other molecules abide by. In other words, we have codon-to-anticodon base-pairings, and the mitochondrial tRNAs do not change those base-pairing rules to accommodate a different genetic code. Finally, like proteins anywhere else, the primiry structure definitely affects the function by dictating the final shape of the molecule. Therefore, we can eliminate choice D. The correct choice is B.

64.

D is correct. The question is asking us to identify the function of the ten polyadenine-containing RNAs noted in the tail is the clue to this question. Recall that mRNA has a polyadenine tail. We can assume iUr nNn ntoleCute to be 6RNA. Choice A assumes that the mRNA is translated in the cytosol, but this is happening in the mitochondria. The mRNA that is created is translated in the mitochondria and not in the cytosol. EliminatJchoice A. Choice B indicates that the RNA is a transfer RNA. This is not the case, because IRNA does not have a polyadenine tail. Eliminate choice B. Choice C clearly indicates that the RNA is rRNA. Again, the polyadenine tail indicates that the RNA is mRNA, not rRNA. The mRNA is functionally responsible for coding for ribosomal proteins. The correct choice is D.
passage. The polyadenine

65.

D is correct. We are told from the passage that the heavy strand of RNA is responsible for encoding many proteins. while the light chain of RNA is over 907o nonsense. One should realize that the heavy chain of RNA is nearll' identical to ihe strand of DNA that gave rise to the light chain of RNA. The only difference is that the thymines in the DNA have been replaced by uracils. Since the heavy chain of RNA contains all of these functional codons, the strand that is nearly identical to it is called the sense strand. The piece of DNA that actually gave rise to the hear'1' strand of DNA is called the antisense strand. The correct choice is D.

66.

phage codes for all of the proteins necessary for its own DNA problem by choosing the answer that has nothing to do with DNA this can approach replication. Therefore, we replication. Transfer RNA, or tRNA, is a ribonucleic acid compound that is important during translation, rather than DNA replication. Therefore, the phage genome need not contain instructions for making tRNA; the virus exploits the hostiell's tRNA. This problem can also be answered by eliminating choices that actually do play a role in DNA replication. Recall that primase is an enzyme which lays down RNA primers during DNA replication. Eliminate choice B. Ligase seals nicks introduced on the lagging strand during DNA replication; rule out choice C. And finally, DNA polymerase is the enzyme responsible for extending newly replicated DNA. Eliminate choice D. All of these choices affect replication of DNA. The correct choice is A.

A is correct. The question tells us that the T4

67.

B is correct. The experiment described in the question is basically identical to the complementation experiment detailed in the passage. In this case, however, the results are different; no plaques form, suggesting that the tu'o coinfecting mutants could not complement each other. Recall that complementation occurs when one mutanl provides Jfunctional protein product that the other one lacks, and vice versa. Therefore. the net effect would be a wild-type phenotype, as all of the protein components of the wild-type pathway would be present (see Figure I in ttre pashgit. In the case of the experiment described in the passage, however, there are no wild-type plaques formed as tlhe reiult of coinfection of the rIIA and X1 phages. A logical conclusion to be drawn from this failure ot complementation would be that both mutants affect the same gene. In other words, neither mutant could provide the othei with a functional copy of the defective protein. As a result, a wild-type phenotype can't be achieved. With this knowledge, we can eliminaie the other answer choices. Choice A is inconect; if the two mutations affected different genes, th;y would complement each other by each providing a good copy of the protein that the other lacks. This ivould result in wild-type plaque formation, which is not observed. By the same reasoning, we can eliminate choict C; the two mutationJ do ,rt-"o*plement each other, which is why they are incapable of lysing E. coli K cellsChoice D can be eliminated, because the experiment does not support this statement in any way (even though it ma3 not directly contradict i0. Recall, the question asks, "What can be concluded from this experiment?" Definitely ntrt answer choice D! The correct choice is B.
r11 mutants of phage T4 are incapable of lysing E. coli K cell-q" (resulting in plaque formation). Additionally, the rll mutants cells Recall that wild-type T4 is capable of lysing these they form plaques on lawns of E. coli K? Let's approach this Why can't B cells. E. coli are capable of lyJing problem first by eliminating unlikely answers: Statement I postulates that the mutant viral DNA sequences contaim new restriction sites that are recognized and cleaved by bacterial endonucleases (or restriction enzymes). Recall thar such enzymes recognize palindromic sequences and cleave them. Normally, this would be a perfectly plausible

68.

B is correct. The question essentially asks us why

Copyright

by The Berkeley Review

374

The Berkeley Kevieu Specializing in MCAT Preparatio'n

Biology

Genetic Information

Section IX Answers

explanation for why r11 mutants can't lyse E. coli K cells. But we must remember that the question is prefaced, "Based on information given in the passage...". From the passage we learn that when a single E. cotik cell is coinfected with both mutants at once, the result is lysis of the bacterial cell and the formation ofi plaque. Could this occur if both mutants' DNA were chopped up by restriction enzymes? Most likely not, so we must eliminate statement I. Statement III hypothesizes that since E coliK cells lack specific receptors for mutant phage T4, the phages could never bind or inject their DNA in the first place (recall the sequence of viral infection: adJorption to the cell surface, injection of DNA, etc.). Could this be the case, considering that the successful complementation experiment described in the passage required the injection of mutant DNA into E. coli K cells? Probably not, so eliminate statement III. This leaves us with statement II, which states that r11 mutants lack functional gene products (i.e., enzymes) necessary for the lysis of E. coli K cells. Referring to Figure I in the passage, we see i pathway for virally-induced cell lysis, which directly supports statement II's hypothesis. Statement II is our winner. ihe coirect choice is B. 69.

D is correct. This question basically proposes an experiment similar to the one described in the passage. In this case, however, we are using a diploid organism, Drosophila melanogaster (the fruit fly). The question states that mutants A and B are each recessive, meaning that they normally express their curly wing phenotype only when they are homozygous (i.e., A/A or B/B). However, transheterozygous flies were made by crossing the two mutant lines to each other, resulting in some progeny which were A/8. Each individual mutant is still heterozygous, but for some reason A/B flies have the curly wing phenotype. What is going on? It turns out that thii experiment is a complementation test. We are trying to learn whether mutant A and mutant B affect the same gene. Hypothetically, if they affected different genes (as in Figure I from the passage), each would provide a functi,onal profein which-the other lacks, resulting in completion of a wild-type pathway leading to normal wing formation. On the other hand, if the two mutants both affected the same gene, flies that are AiB would lack a furctional protein encoded by the mutually-affected gene. Therefore, wild-type wing formation would not occur and a curly wing phenotype *ouid b" observed. This is the case in the experiment described in the question. Therefore, mutanis A ind B fail to complement each other and must therefore affect the same gene. The correct choice is D.
C is correct. The experiment described in the question is basically testing whether each of the T4 mutants can revert to the wild type and form plaques on a lawn of E. coli K cells. Recall that normally, rllA and, rIIB can'tform plaques

70.

t
71.

(i.e., lyse cells) on this bacterial strain. The question informs us that r11A forms one single revertant ptuqu., however. The mutation that prevents rlIA from lysing E. coli K cells has spontaneously revertid. For example, if the mutation were caused by a single base-pair substitution, another spontaneous mutation that switches the suLstitution back to the wild type would cause a reversion of the mutant phenotype (i.e., plaques could now form). This is most likely what has happened to rIIA. What about r11B? No revertant plaques form at all. We can infer from this that the type of mutation that causes the rIIB defect is difficult or impossible to revert. The bottom line is that this question tests your understanding of the different types of mutations that can occur in DNA. Let's go over the ones mentioned in the answer choices: A point mutation occurs when a single base pair is altered (i.e., G is changed to A, etc.). point mutations revert relatively easily, because it only takes another mutation in that base pair to switch it back to the wild type. Since r11A is capable of reverting, we can fairly assume thatIIIA is caused by a point mutation. A deletion mutation occurs when a segment of DNA is removed. Deletions hardly ever revert, mainly because it is nearly impossible to?eplace a segment of DNA spontaneously (eliminate choices A and B). Since r11B doesn't revert, wL can fairly assume that it may be caused by a deletion mutation. Therefore, choice C is the correct answer. Frameshift mutations result from deletion or insertion of base pairs, resulting in the altering of the normal reading frame of a !eno. Frameshift mutations are difficult or impossible to revert (eliminate choice D). The correct choice is C.

C is correct. Answering this question requires a solid.understanding of what complementation is all about. Complementation in diploid organisms occurs when two mutants which affect different genes each provide a functional protein which the other lacks. A dominant mutation is one which shows a mutant phenotype even when heterozygous, or present in one copy. In other words, there is one wild-type chromosome present in organisms that are heterozygous for a dominant mutation. This wild-type chromosome, although providing a full complement of wild-type proteins, does not complement the dominant mutation; in other words, the mutant phenotype is still expressed. This problem can be approached by eliminating answers that are incorrect. For example, choices A and B are both recessive mutations. Recessive mutations don't express their phenotype when they are heterozygous, because the homologous wild-type chromosome provides enough functional protein to make up for that *irictt ls defective in the mutant chromosome. In other words, the wild-type chromosome complements t6e defect present in the mutant chromosome. We can eliminate choices A and B. Choice D can likewise be ruled out; a deletion mutation would most probably be a recessive mutation, because the wild-type chromosome could provide a normal protein product. The correct choice is C.

Copyright @ by The Berkeley Review

5/b

The Berkeley Review Specializing in MCAT Preparation

Biology
72.

Genetic lnformation

Section IX Answers

A is correct. The key to this question is knowing our terms. An obligate parasite is an organism that must rely on other organisms solely to survive. A virus can't reproduce without a host cell; therefore, it is an obligate parasite.
This BEST describes the T4 bacteriophage. Let's eliminate the other answer choices: An obligate heterotroph is an organism that must feed on others to survive. More specifically, it must consume complex carbon molecules. Animals are obligate heterotrophs, while plants are autotrophs; they can synthesis their own complex carbon molecules (i.e., carbohydrates, proteins, etc.). Therefore, we can eliminate choice B, because the virus does not feed on anything directly. It simply uses its host to reproduce. Prototrophic organisms, such as certain bacteria, can survive on a minimal media, while auxotrophs need supplementation of certain nutrients to survive. These are terms most often used to describe bacteria, and therefore they wouldn't apply to bacteriophageT4. Eliminate choices C and D. The correct choice is A.

73.

B is correct. The homozygous mutant animals were hypertensive on the standard chow diet. Eliminate choice A. The response of the heterozygous animals differed from that of the wild-type animals at the 8Vo dietary sodium
level. Eliminate choice C. Blood pressure increases in the heterozygous animals were directly proportional to dietary sodium. Choice D is incorrect. The correct choice is B.

74.

D is correct. The Lowry assay is for protein concentration, and it cannot discriminate among proteins. Choice A is incorrect. To digest the peptides in a tissue would create a bunch of amino acids, but no real information on ANP as an intact molecule. Choice B is incorrect. Southern blotting is for DNA fragments, not proteins. Radioimmunoassay uses antibodies to the compound of interest to separate it from a mix of compounds. Also, it is sensitive to at very low levels (picograms). The correct choice is D.

75.

A is correct. They do not have any detectable ANP, so the action of ANP to direct fluid out of the blood is not available. The intravascular fluid level tends to be higher in these animals, as shown in the hematocrit. The hematocrit measures the Vo of the blood that is RBCs. A lower hematocrit indicates more fluid in the blood relative to the RBCs. Choice B is incorrect. ANP does not directly affect the RBCs, and this is not srared in the passage.
Choices C and D are incorrect. The correct choice is A.

76.

A is correct. The pro-ANP is stored in granules, cleaved by a specific protease, and then released from the cell as ANP. A person missing the enzyme would not secrete ANP. The question is whether pro-ANP is released intact. Compare this to what you know about glycogen. Glycogen is made of glucose polymers stored in granules, but only the subunit, glucose, is released from the cell. It is unlikely that the larger pro-ANP precursor is released directly. Also, there are no proteases in the bloodstream to activate the pro-ANP, since that would be disastrous to cells. Choice B is incorrect. Choice D is incorrect. The correct choice is A.
D is correct. Instead of genetically altering animals, other approaches are to somehow remove ANP from regular
wild-type animals. These are all theoretical answers. If all the ANP is bound by antibody in the blood, then the effect is that no ANP is present. Statement I is correct. If the receptor is bound by something else and cannot interact with ANP, then the effect is the same. Statement II is correct. If pro-ANP were bound and not processed to ANP, the effect is the same. Statement III is correct. The correct choice is D.

77.

78.

A is correct. The atria would be stretched in the case of too much pressure. ANP lowers fluid volume in the blood and promotes sodium excretion (and water follows sodium). The lowering of fluid volume promotes a lower blood volume and lower pressure. No pressures are raised, therefore choices B and C are incorrect. Sodium output in the urine is increased by ANP ("natriuretic" means promotes sodium in the urine). Choice D is incorrect. The correct
choice is A.

79.

C is correct. Use a Punnett square to figure this out. We cross AA x Aa. AA means having 2 good copies of the ANP gene, and Aa is the heterozygote. The offspring will be 5OVo AA and 507o Aa. The Aa are salt-sensitive, just like the Aa parent. The correct choice is C.

80.

D is correit. We must come up with this answer based on previous knowledge, and not something stated in the passage. The gram stain takes advantage of differences in the cell walls of bacteria. The cell wall of a Gram-positive
bacteria contains peptidoglycan, assorted polysaccharides, and teichoic acids. The cell wall of Gram-negative bacteria contains peptidoglycan, phospholipids, lipopolysaccharides, and assorted proteins. The cell wall of a Grampositive cell does not have an outer membrane, and the peptidoglycan layer is very thick when compared to the layer in the Gram-negative bacterium. The correct choice is D.

Copyright @ by The Berkeley Review

376

The Berkeley Review Specializing in MCAT Preparation

Biology
81.

Genetic lnformation

Section IX Answers

C is correct. This is not the case, and can be determined from one of Griffith's experiments. Let's look at the injection of a heat-killed, pathogenic bacteria. The mice lived. If the polysaccharide itself was the cause of death, then this would not have been the case. The mice would have died. The death must result from something a live
bacterium can produce; and furthermore, the polysaccharide coat must serve another function besides causing death. The correct choice is C.

82.

B is correct. Recall that proteins are very sensitive to temperature. When the temperature becomes too high or too
low, the protein shape is altered. That protein shape is the key to protein function, and helps a protein carry out
cell's everyday function. If the proteins are non-functional, then nothing can continue. There can be no synthesis or metabolic processes. If no processes can be carried out (even the replication of nucleic acids), the cell will not be able to produce energy and it will die. The correct choice is B.
a

83.

D is correct. This question is very straightforward. Avery's conclusion was that the transforming principle was DNA. Recall that deoxyribonuclease destroys DNA. If Avery's conclusion held true, the addition of deoxyribonuclease should eliminate transforming activity. We are looking for a statement that does not support
Avery's conclusion. The correct choice is D.

84.

B is correct. This question is straightforward. We are told from the passage that the bacterium is a facultative anaerobe. This means that the bacteria can function either in the presence or absence of oxygen. An obligate anaerobe cannot function in the presence ofoxygen, while aerobes can function only in the presence ofoxygen. the correct choice is B.

85.

B is correct. Remember that ribosomes are considered to be organelles. They are just not membrane-bound organelles. The bacterium certainly requires the presence of ribosomes to carry out the translation of mRNA to make protein. The bacterium does contain organelles. The correct choice is B.
D is correct. We are looking to use two elements that will enable us to discriminate between proteins and nucleic acids. This question is based on the experiments of Hershey and Chase. The backbone of nucleic acids contains phosphate groups. Nucleic acids do not contain sulfur. Proteins contain sulfur, but do not contain phosphorous. Therefore, phosphorus and sulfur are the two elements we can use that will enable us to discriminate between
nucleic acids and proteins. The correct choice D. D is correct. This question comes in two parts. The first requires one to think whether a cell taken up by another cell is pinocytosis or phagocytosis. The answer is phagocytosis. Recall that solid particles endocytosed by a cell is referred to as phagocytosis, while endocytosis of dissolved particles is termed pinocytosis. The bacterium can be classified as a solid particle, enabling one to eliminate two of the answers. Next, is phagocytosis carried out by Bcells or macrophage? p-cells produce antibodies, while macrophage are considered the body's professional phagocytes. The correct choice is D.

86.

87.

88.

B is correct. Since we are assuming that eyeless is on the X chromosome (X-linked), we can infer that the genotypes usedin the cross are: ey/+ females x +,/Y males. Fruit flies have the same sex chromosome setup as
humans (two X chromosomes for females and an X and Y for males). The real trick to this question is remembering from the passage that flies heterozygous for ey have small eyes, while those homozygous for it have no eyes. After that, it's just a matter for a Punnett square. The correct choice is B.

89.

B is correct. As you recall, a promoter is an untranscribed sequence of DNA upstream (towards the 5' end) of the DNA sequence that is actually transcribed (and that codes for the gene's protein product). The relative "strength" of a promoter (i.e., the degree to which it promotes transcription) is dependent on the promoter's sequence. In Experiment I, GAL4 has a very weak promoter. The is the reason that GAL4 is not normally transcribed or expressed. The wing-specific enhancer can "enhance" transcription of GAL4 only in wing tissue, overcoming the weakness of the promoter. In this question, this system has failed and GAL4 is transcribed everywhere, thereby activating the UAS-controlled eyeless gene everywhere. This is why the in situ hybridization experiment showed thatey mRNA wqs present throughout the embryo when we should expect it to be present only in certain cells. A
constitutive (meaning "always active") mutation in the GAL4 weak promoter could have made it "stronger," thereby increasing transcription. Answer choice A is wrong, because a mutation in the UAS site could only prevent GAL4 from binding, thereby reducing ey expression. Answer choice C is wrong, because the normal genomic copy of ey is expressed only in cells that will become eyes (from the passage). Answer choice D is wrong, because an amber mutation (a stop codon) would cause an abnormally short ey protein but would not affect where it is transcribed. The correct choice is B.

Copyright

by The Berkeley Review

cl I

The Berkeley Review Specializing in MCAT Preparation

Biology
90.

Genetic Information
I is an unsupported

Section IX Answers

statement, because from the passage we know that while the ey gene may be necessary, it is not sfficient for normal eye development. We know this because in heterozygotes, there is one good copy of the gene, but the phenotype is still abnormal (small eyes). Moreover, no evidence in the passage ruppo.tr theiiaim tharey is sufficient for normal eye development (there are in fact hundreds ofother genes that are n.i-"srury to make an eye). We can infer only that ey is a developmental "master switch," possibly acting to turn on other genes needed for eye formation. From the passage, we see that ey is a transcription factor, which serves to back this claim. Transcription factors bind to DNA and promote transcription of certain genes. This means statement II can be supported by the passage. Statement III says that eyes arose independently in insects and mammals. This means they ivolved r"pu.uGly from different ancestors and have nothing in common evolutionarily. The results of Experiment 2 as well is the homology between Drosophila eyeless and mouse small eyes shows this to be false. This evidence suggests that eyes evolved from a common ancestor. The correct choice is C.

C is correct. Statement

9t.

D is correct. The mutant eyeless allele is a recessive hypomorph, meaning that its function is reduced, or weaker. A point mutation in a nonconserved region of the sequence would probably not have much effect on protein function, t".uur" nonconserved sequences vary throughout evolution without altering protein function to any great degree. A two base-pair deletion near the beginning of the sequence would cause a frameshift mutation that would completely wipe out protein function, and this can't be the case. An inversion would cause the same effect, so that leaves choice D.'A muiation in a sequence of eyeless that has been highly conserved throughout evolution would cause a reduction in the function of the protein. These sequences are conserved, because they are crucial to the proper functioning of the protein. The correct choice is D. B is correct. Ectopic means "not in the normal place." Since Experiment 2 showed that the mouse small eyes gene in Drosophila, a finding that the reciprocal experiment failed to make normal mice eyes made normal fruit fly eyes -Such a finding is shown in choice B. Answer choices A and D both confirm that eyeless is would be inconsistent. evolutionarily conserved, which agreis with the results of Experiment 2. Answer choice C confirms that eyeless is highly homoiogous to small eye,s, a case that supports the results of Experiment 2. The correct choice is B.

92.

93.

A is correct. In order for a population to be in Hardy-Weinberg equilibrium, answer choices B, C' and D must not there be true, while choice A musi be true. This is a memorization question; but if you remember that at equilibrium The right answer' with the up you come pool, could then is no net change in the frequency of alleles in the gene
correct choice is A.

94.

C is correct. This type of question was given o^n a previous MCAT. You must know how to use the HardlWeinberg equarion, *ti.h ,tut., thar p2 + ipq + g2 = l.In this equation, p is the frequency of the wild-type allele\\'ill while q ii thi frequency of the recessive alleie in the population. The questions asks for the number of flies that term 2pq The eyeless. for those heterozygous means that this know we have small eyes, and irom the passage, population represents heierozygotes. Thus, Zpq=Z1O.S1(0. 1) = 0.18. This is the frequency of heterozygotes in the ilh"n *" multiplythis by the populition of 1000, we come up with 180 flies. The correct choice is C.

95.

is C is correct. The karyotype gives a picture of the chromosomes, which means that the sex of the fetus chromosomes-m missing or of extra The incidence answer. best the not immediately discernibli. itoi.e A is thus

defects may or may not be also immediately visible, so choices B and D are poor selections as well. Developmental it the best answer' The making false, due to geneti. .rtorr. Some have environmental causes. Choice C is therefore correct choice is C.

96.

first decade when we turnalx C is correct. The second decade of one's life spans the years fiom 10- 19. we finish the 15-19' or the later porti;m' years from in the women for ir lowest syndrome Down ten. The chance of bearing a "ilta in both 13-1'earof the second decade, so itatement I is correct. Reading from the graph, we see that the incidence beginnine cf is the Menarche too. iI is olds and 35-year-olds is about 0.9 per 1000 births, sJsmtement "orre.t, of Down births is higher in \r'o{TEa incidence The menstruation. of menstruation, while menopause is the conclusion III is therefore inconect. 11: near melopause (age 40-50+) than in women close to menarche (9-15+). Statement is C. correct ihoice

97.

(in this A is correct. Down syndrome is also called trisomy 21. Tri means "three," somy refers to body

case"

refer to chromosome I 8' bodies of the chromosomes), and 2 I refers to chromosome 21. Choices B and D, which A' is choice The correct incorrect. is 2l) (monosomy C both incorrect. Also, choice

fu m

Copyright

by The BerkeleY Review

374

The Berkeley Revirr Specializing in MCAT

Biology
98' 99'

Genetic Information

Section IX Answers

B is correct. The presence of the Y chromosome means the person is a genotypic male, so choice A is incorrect. This question is about th,e genotype' not the phenotype. Choices C andb are'thus incorrect, since they refer to phenotype. The correct choice is B.

I provides us with the_answer: Monosomy X is the highest, at 20/100. There is no trisomy X listed in the table, so choice C is incorrect. Both trisomy 21 and trisomy 18 Jccur less frequently than monosomy X, so choices A and B are incorrect. The correct choice isD.
D is correct. Table
RFLPS can be used for this kind of work because restriction enzymes cleave different DNA strands into fragments of different lengths, based on the location of specific restriction sites. The child has ttr"" of chromosome 21. One clearly came from the father and two from the mother. The mother "opi"s is the source of the nondisjunction in this instance. Choices A, C, and D are incorrect. The correct choice is B.

100' B is correct'

Copyright @ by The Berkeley Review

379

The Berkeley Review Specializing in MCAT preparation

sgction'
Genetic
' .i. .2rd.Position .

rcr6$y x
A.:,, I;INA

Pfocssing

Expr$$idn,
.

rOf

,', 2,. "RNA$linthesis &Modifiiationsl , 1 ', f;; ' ProteinSynthesis ' ' 4.. The Lactose Operon '
.

'[.

RhJA,Polymrase & Promoters

5.,,

The Tryptophan Operon

Information
lst
:

8,,,,

Psiqltlon

{5'.End}

UCA
Plrq i

r'

llutations l. Mutations & Ploofreading ' , 2. ' The Ames Test ,, l. Cenetic Enginqerin$,lsequencing, ..&Cloning..
r

G
,

;3rd,
{3l.rEndl

C.,Genetic: Engineering

U
C

Leu .. : .Ser,,.,,

.' Ser: . Tyr . . -: Cys Phe ..Ser. 'tyr .rCys.. Leu , Ser, STOP ' ST&p
..S.TOP.

U
C

A
G U
C

r ..:,...TrP
fu'g

.,

Practice Passages & Ansryers

Leu Leu Leu Leu

Pro

Pro ' Pro Pro

His His r Cln Cln


A,!n Asn Lvq

'A{g
Arg Arg Ssr
Ser

A
G

A
G

Ile Ile IIe Mff

Thi
Thr Thr Thr

U
C

Arg

A
G U C

Lys

Arg Gly:

val , Ala r Val * Ala val Ala Val Ala

A$F, Asp Glu . , Clu

Cll
Gly Gly

A
G

Specialiiing in MCAT Preparation

Expression of Genetic Information


Top
1O

Section Goals

o?

Understand the differences between RNA & DNA polvmerase.


RNA polymerase is used to transcribe DNA inio RNA language. DNA polymerase replicates DNA before cellular division. Understand these functions and how they apply to the cell.

Be familiar with the concept of a transcription promoter. on the DNA helix. There are specific RNA polvmerase does not beqin transcription just anyplace -Understand the basics of these funitions. addresse's that fcl/ the polyderase where to bind.

Iftrow the differences between eukaryotic & prokarJrotic BNA Processing. Eukaryotic RNA must undergo extensive processing in the nucleus before it can be used in the cytoscil for protein synthesG. Prokaryotic RNA ii used immediately and is not processed.

@?

Be able to read the genetic code. DNA codons on ihe coding strand are the same as the mRNA codons. DNA codons on the template strand are the same as theinticodons on IRNA (except for replacement of Us and Ts)

tlave a feel for transcription & translation in eukaryotes

&-

prokaryotes.

In prokaryotes, transcription and translation are tightly coupled. Il eukaryotes, transcription and translation are separat-ed both in space and in time. Understand this important distinction.

t?

Be familiar with the actual process of transcription. You should have a firm grasp of IRNA activation and stibsequent binding of the activated IRNA. to the P-site and A-sitd on the ribosome. Understand translocation-and protein synthesis.

Understand the lactose operon and how regulation is achieved.


The lactose operon ties in all of these aspects

is an exceilent example of how a bactbrial cell regulafes its need for a particular metabolite.

of

transcription and translation disc-ussed so_far. It

Understand the basic tools involved in genetic engineeringmemo.ire their recdgnitiorisequencds. Instead, know the basics of how they function
Probably the most important genetic engineering
to_ols are the

restriction endonucleases. Do.nc:

-:

Jltl

flt

Be able to read autoradiographs from polyacrylamide gels. Understand the process of gel eiectrophoresis_and why it-is-that bands are read f1om. the bottom oi the gel to the top of the gellBe famili'ar with the Sang6r dideoxy sequencing method.
'a-

al

fl'

pr
rmd

um

r-'

Be familiar with tfte

pto."tt of .lo"it

T
,iun

This is ihe core of genetic engineering. Understand how to remove a seqment of forergn DNA, ope: up a plasmid, inse"rt that forEign DN"A into the plasmid, and then express the gene oFinterest.

m ]fi

fmr

ffm

Biology

Expression of Genetic Information

RNA Polymerase

& Promoters

||Fol# a$Hliffi1
RNA Polymerase

o't-brs

The enzyme RNA polymerase transcribes DNA into RNA. Let's compare the

differences between replication and DNA-directed DNA polymerase with transcription and DNA-directed RNA polymerase.

'1,. Both polymerases proceed in the 5' -+ 3' direction. 2. DNA polymerase uses dNTP's (releasing PP1) while RNA polymerase
uses NTP's (also releasing PPi).

3. 4.

Both polymerases require a DNA bubble for operation.

With DNA polymerase we find that DNA replication is semiconservative while in the case of RNA polymerase the two DNA strands will eventually rewind, thus conserving the process.
polymerase requires a primer with a free 3'-OH group to initiate chain elongation. RNA polymerase does not require a primer.

5. DNA

6. The DNA directed DNA

polymerase is almost always involved in

synthesizing DNA bidirectionally. Both DNA strands are being copied. In the case of DNA directed RNA polymerase only one strand of the DNA is being copied. one way to demonstrate that RNA polymerase does not need a primer is to use labeled substrate such as ATP labeled at the B andyphosphates (Figure 10-1).
a

ooo ll* ll* ll -o-P-o-P-O-P-O-CFI? trt\ o_o_o4'


Ho
Adenosine triphosphate (A ribonucleoqile triphosphate)

Adenine

I'

3'2'T

oH

Figure lO-l
Are any of the p or yphosphates incorporated into the nucleic acid? If we try this using dATP and DNA polymerase, the answer would be no. Why? Because we are starting with a primer and when we add dATP to the free 3'-OH group, the p

and y phosphates of dATP are lost

in the reaction mechanism

(as pyro-

phosphate). However, with RNA polymerase we do get some p and y labeled phosphates in the RNA polymer. This is because the first nucleotide (ATP) incorporated into the growing RNA polymer has at its 5'end a labeled B and y

phosphate.

r.

The time course of incorporation of the B-y labels and also of cr labels in a growing RNA polymer can be measured. It takes about 1 or 2 minutes to make an average mRNA molecule in a test-tube using RNA polymerase and the DNA from a phage such as T2 or T 4. If we measure the incorporation of the p-y label,
Copyright @ by The Berkeley Review

343

The Berkeley Review Specializing in MCAT Preparation

Biology

Expression of Genetic Information

BNA Polymerase & promoters

we will find that within a few seconds that label is in the RNA polymer and. then no more is incorporated. However, if we measure the incorporation of the a label, we will find that the time course is more even up to about two minutes when RNA synthesis stops. This can be seen in Figure ro-2. rnis graph tells us that the p-y goes in first while the a label goes in second.
o

Promoters
rn E. coli there is only one RNA polymerase and this RNA polymerase has to make transfer RNA linrvA;, ribosornal RNA (rRNA), and miny different types of messenger RNA (mRNA). It prefers to start transcription at a particular siie c,n the DNA duplex. what tells an RNA polymerase whele to staritranscription? I:

N
0

Time (minutes)

Figure to.2

turns out that there are very specific starting sites for transcription *hi.h utt different in both prokaryotic and eukaryotic organisms. consider the doub;e stranded DNA in Figure 10-3.
Upstream
Start Site

Downstream

5'

-35 TTGACA

-10 TATAAT

+1
Coding Stiand

3' Duplex

Template Strand

DNA 5'

mRNA

Figure IO.3 Note the 5'and 3' ends of the duplex. Let's suppose that this small segment cd DNA is about a thousand or so base pairs and that we wish to copy the lowm strand. The direction of RNA synthesis will be in the 5'-+ 3' dlirection as previously mentioned. The very first base pair of the transcription process rr referred to as the +1 base pair. Immediately to the left (upstream) of +i we hane the -1 base pair, -2 base pafu, -3 base pair, etc. Immediatery to the rigfit (downstream) of +1 we have the +2 base pair, +3 base pair, etc. There is no base
designated as "0".

upstream from the +1 site. In a prokaryotic organism these areas are usua_Ilr centered around the -10 region (TATAAT) and the -35 region (TTGACA). The -10 region is referred to as the Pribnow box while the -35 region is simply the -3F region. As we will see, there is some flexibility in these numbers from gene !o gene or organism to organism. Therefore, we can speak of these regions as be,rng consensus sequences. As the RNA polymerase moves along the DNA dupler, d is looking for the proper signals that will tell it where to promote transcripticmHence, these regions are called promoter sites.

The signals that tell the RNA polymerase where to begin transcription

a_re

what are the consensus sequences for eukaryotic organisms? In the case d' eukaryotic organisms the promoter areas are centered around a -25 regi,m (TATA box) and a -75 region (CAAT box), upstream from the +1 site. This m shown in Figure 10-4. The -25 region is referred to as the Hogness box. The CAAT box may or may not sometimes be present. It might even be moved
further upstream.

Copyright O by The Berkeley Review

3A4

The Berkeley Revien Specializing in MCAT Preparatfun

Biology
Upstream

Expression of Genetic Information


Staft Site

RNA Potymerase

& promoters

Downstream

5'
3

-75
GGNCAATCT

-25

+1
Coding Strand Template Strand

TATA

3' Duplex

DNA
5'

RNA

Figure lO-4
There are also signals in the DNA that tell the RNA polymerase when to stop transcription. These occur in specific areas downstream from the +1 site. Elongation of the RNA chain occurs by a nucleophilic attack of the 3'-oH group at the end of the growing RNA chain with the alpha phosphate of the neit incoming ribonucleotide triphosphate. A phosphodiester bond is formed. This reaction is similar to that of DNA svnthesis.
DNA Template

v
3'-GCGI TCGGGCGI GATTACTI CGCCCGA IAAAAAAACTTGTTT-s' s'-CCC 4GCCCGC CTAATGA TGCGGGCT TTTTTTTGAACAAA_3'
I

kRNA Transcript

GC-Rich region

---*l*

AT-Rich region

I
CA
C-G G.C C.G C-G C-G G-C

5'-CCCAGCCCGCCUAAUGAGCGGGCUUUUUUUU-OH-3'

Figure lO.5
The RNA polymerase will proceed down the DNA template until it runs into the terminator sequences. These stop signals are composed of a GC-rich region followed by an AT-rich region on the DNA template (Figure 10-5).

5'-CCCA UUUUUUUU-OH.3'

rn E. coli the terminator sequence for the end of protein synthesis is a basedpaired hairpin sequence on the newly synthesized RNA strand (Figure 10-6).
The hairpin structure is formed by the complementary base-pairing of a region of

Termination Hairpin

Figure lO.6

the RNA sequence that is rich in the bases G and C. Following this hairpin structure is a sequence of four or more u residues (referred to as the poly-U tail). Once this hairpin pairing occurs in the RNA molecule the RNA polymerase pauses (stalls). The polyribonucleotide uracil and polydeoxyribonucleotide adenine that are still annealed to each other (the RNA-DNA hybrid) are rather unstable. The result is that the RNA chain will dissociate from RNA polymerase and the DNA dupleN This process is referred to as rho-independent termination.
The second way in which termination can be accomplished involves the rho (p) protein and is referred to as rho-dependent termination. The rho complex is a hexameric protein consisting of 46 kd subunits and has an ATPase activity that

allows

it to specifically bind newly synthesized single-stranded RNA and pull


345

Copyright @ by The Berkeley Review

The Berkeley Review Specializing in MCAT Preparation

Biology

Expression of Genetic Information


itself

BNA Polymerase 6( promoters

towards the replication bubble where it wilt dislodge RNA polymerase from the DNA template. The end result is a fully transcribed"transcript of RNA, a free rho protein, and a Jree RNA polymerase enzyme. In higher organisms it is not entirely clear what the termination signals are.
The overall process which we have just described happens at a variety of places on the DNA duplex and the result is that a lot of specific RNA molecules are synthesized. The three phases of RNA synthesis are called initintion, elongation, and terminstion.

Copyright @ by The Berkeley Review

386

The Berkeley Review Specializing in MCAT preparation

Biology

Expression of Genetic Information

RNA Synthesis

& Modifications

l$.

tffiCi'lffi .iffidiffidAtf'UHS

RNA, like DNA, is synthesized in the 5' -+ 3' direction. one immediate difference between DNA synthesis and RNA synthesis is that the enzyme RNA polymerase incorporates ribonucleoside triphosphates into the growing RNA chain. In DNA synthesis deoxyribonucleotide triphosphates were used. Another difference is that RNA polymerase does not need a primer in order to start transcription of the RNA polymer.

while local rewinding takes place to the rear of the polymerase. Note that towards the 3' end of the RNA transcript we have an RNA-DNA hybrid.

polymerase moves along the DNA duplex it unwinds about i 17 base pair section of the DNA duplex and reads the template strand in order to synthesiie the RNA transcript. Local unwinding of the DNA duplex occurs ahead of the polymerase

Y-1:" the RNA polymer is synthesized during transcription, only one of the DNA template strands is utilized. This is shown in Figure 10-7. As RNA

5',

l-/

Figure lO-7

Types of RNA Recall that there are three major classes of RNA. They are messenger RNA (mRNA), transfer RNA (IRNA), and ribosomar RNA (rRNA). In the case of prokaryotic bacterial cells there is one type of RNA polymerase that is able to transcribe all three of these types of RNA. How"rr"t, i.r-the case of eukaryotic cells there are three separate types of RNA polymerases. RNA polymerase i will transcribe rRNA, RNA polymerase II will transcribe mRNA, and RNA polymerase III will transcribe tRNA.

polymerase. These promoter sites determine where transcription is to begin. In bacteria such as E. coli there are two sequences on the DNA template which are rmportant to the RNA polymerase, the Pribnow box and the - 35 region. The Pribnow box is located upstream (towards the 5' end) by 10 nucleotides (i.e., at lranscribed (denoted as +1). The -35 region is located upstream by 35 nucleotides i.e., at -35). RNA polymerase will move along the DNA template and :ranscription of one strand will take place until a signal to stop (a terminator region) is reached. This can be seen in Figure 10-8.

Iranscription Promoter Sites lvithin the DNA molecule are regions called promoter sites that bind RNA

-10) from the regiory of the DNA template where the first nucleotide is

Copyright @ by The Berkeley Review

347

The Berkeley Review Specializing in MCAT preparation

BioIo sv

Expression of Genetic Information

RNA Synthesis

& Modifications

Promoter
r0

+l

TTGACA
-35 Region

TATAAT
Pribnow

Gene

DNA

Box

DNA
Temolate

^-Start of RNA

.-/--.,-/
RNA

RNA

End of

Figure lO-B
Note that the -10 region and the -35 regions of the promoter are not palindrome. They are not like the CATC box that we will discuss when we look at t:.= mismatch repair enzyme system or even like the EcoRI sites that we will discu-"= when we look at restriction enzymes. If the promoter region were palindrorn . RNA polymerase would not know what to do.

Promoter Strengths Vary


The promoters for eukaryotic rRNA genes are simply not recognized by R\ d polymerase II or RNA polymerase III. Within a class of genes the promoi.: strengths can also vary. Consider the gene for a glycolytic enzyme. Since enei:-,, metabolism is a major activity of the cell, these enzymes need to be transcribe; :: a high rate. Thus, the enzymes for glycolysis are always quite prominent in ce_.. For example, there might be in excess of 100 transcripts per cell for any of i:* glycolytic enzymes. If you were to assume that these transcripts were degraJeat the same rate as their synthesis, then there must be a lot of transcriptior. :r glycolytic genes. \A/hat this means is that glycolytic genes have strong promoters

In contrast, consider the coenzymes that we have been discussing. Coenzr-r:.s are present in cells in catalytic amounts. In other words, the enzymes for ::r* coenzyme biosynthetic pathways are present in very small amounts. You mi*'L:
only need one copy of an enzyme molecule per cell for each step in the pathr,, =, Thus, the number of transcripts in the cell might be on the order of one. \\hr this means is that the genes in the coenzyme biosynthetic pathway have weall promoters.
This is telling us that you will not find the same promoter sequence in front of agenes. The -10 region (TATAAT) and the-35 region (TTGACA) of the promote: sequence are simply two common motifs which are consensus sequences base; on the analyses of literally hundreds of promoter regions.

Viral Promoters
The promoter region in front of the gene that codes for the RNA polymerase i'i the T7 viral bacteriophage is very strong. Once this phage's DNA is injected int: the host cell, its strong promoter region attracts the RNA polymerase of the hcs; cell and forces it to transcribe the viral RNA polymerase. Once the viral RN-{ polymerase is synthesized it will recognize only its own genes and not the genc: of the bacterial host cell.

Copyright @ by The Berkeley Review

3aa

The Berkeley Revieu Specializing in MCAT Preparation

Biology
E. coli

Expression of Genetic Information

RNA Synthesis

& Modifications

BNA Polymerase lloloenzyrme RNA polymerase is composed of 4 subunits, rt2, B, B', and o. Together these four subunits are-calred the RNA porymerase horoenzyme. The sigira (o) lbun{_1s responsible. for.locating the piomoter site that iniiiates transcription. once RNA synthesis begins, the;igm; subunit wilr dissociate from ttre nNe polymerase holoenzyme. RNA poly-etuse, which now has three su units (cxz00'), is called the core enzyme. The core enzyme can still initiate transcription even though it has lost its sigma subunit, but it does so poorly and anywirere. P:"uyr: the core enzyme has rost one of its subunits lsrgm;;, it is an apoenzyme. The holoenzyrne and the apoenzyme are represented"it, rigrrru 10-9. Note that we are giving directionality to this enzyme.

crrFFb
(A complete

0zF0'
Apoenzyme

.. Holoenzyme enzyme)

(An incompleie enzyme)

Figure lO-9 sigma.factors are quite important in bacterial transcription. For example, they have the ability to (1) Iower the affinity of the_RNA polymerase for gur,"rul regions in the DNA duplex by a factor of about 104, and til tt",uy raise the t'ir,airrg constant (affinity) for one class of promoters. Thus, sigma would be an examplE of a regulatory subunit. It regulites the specificity-of the RNA polymerase

enzyme.

How does RfA polyqerase find its promoter region? The RNA polymerase will bind to the DNA duplex and then sftde along it" looking for the -38 region and the -1.0 region. In this search, transient hydiogen bonJs are formed with the hydrogen acceptor and donor groups of base puir, in the grooves of the DNA duplex' This one dimensionil search for the "*po."d promoter site is much faster than a three dimensional search in which the RNA polymerase would have to repeatedly bind and dissociate from the DNA duple" (se" rig.r.e 10-10).

--+
-35

Promoter
-10

+l

TTGACA

TATAAT

Gene

Polymerase
it

Box

RNA Polymerase slides along the DNA duplex


looking for the promoter site. It does not repeatedly bind and then dissociate in its qwst for this site.

Figure tO-lO
Copyright @ by The Berkeley Review

349

The Berkeley Review Specializing in MCAT preparation

Biology

Dxpression of Genetic Information

RNA Synthesis

& Modifications

initiation, elongation, and termination, shown in Figure 10-11. Initiation

Tfanscription RNA polymerase will synthesize RNA from a DNA duplex in three
as

stages,

indicated in the sequence of diagrims

Promoter Region

Elongation

l'---*o t

-35

Termination

I l--

Rho

i
i

f*g.h"."
@RNA
Figure

gr

it'

|n

b,i

itr

sr

NE

h
lo-l I
when the RNA polymerase binds to the promoter region of the DNA duplex ar initiation complex is formed. As soon as the initiation complex is formld, the sigma subunit dissociates from the polymerase leaving the core enzyme bound
Copyright @ by The Berkeley Review ;g
ca

1r: :1-

5{ ar

39O

The Berkeley Keview Specializing in MCAT preparation

Cq

Biology
to the

Expression of Genetic lnformation

RNA Synthesis

& Modifications

DNA duplex at the promoter site. with sigma missing, the core enzyme is able to bind more strongly to the DNA template. As the DNA tempiate is unwound and read in the 3'-+ 5'direction by the RNA polymerase core enzyrrre, newly synthesized RNA is made in the 5' -+ 3' direction. This region of activity is referred to as a transcription bubble. Nascent RNA is synthesized until a termination signal is reached.

Accuracy of Replication and Tfanscription


We can consider the accuracy of replication and transcription by looking at Table 10-1. Recall that during DNA replication there were two process that ensured the accuracy of the final product. These were (1) editing by the DNA polymerase

enzymes and (2) the repair enzymes. As a result, the probability of an error replicated before an error is made that is not caught by either the editing aspect of DNA polymerase or the repair enzymes. Therefore, the mutation rate is very
1ow.

occurring during DNA replication is quite low. Roughly 1010 bases will be

In contrast, the RNA polymerase do not have a polymerase editing function and they do not have an enzyme repair system. As a result, the probability of an error occurring is about one in every 104 or 105 transcribed bases.

Coding Process

Polymerase Repair Editing Enzymes

probability
of Enor
10-10

Replication + Transcription
Table
I O.

10-4.10-5

why, then, does DNA replication begin with an RNA primer? The editing system of the DNA polymerase system will not work when replication is just starting from the first base. The DNA polymerase enzymes will only work properly i} there is a complementary double stranded structure to start working on. 'it " RNA primer that begins the okazaki fragments has a high probability of error-and they will eventually need to be removed. since there is now a double stranded structure to work with, the DNA polymerase enzyme can use its editing function to remove the RNA primer (and filt in the gap with DNA
nucleotides).

Inhibitors of RNA Polymerase


RNA polymerase can be inhibited. For example, the antibiotic actinomycin D binds specifically to double stranded DNA and prevents RNA polymerase from using it as a template for transcription. Actinomycin D has a phenoxazone ring (similar to the anthracene ring) which can act as an intercalator and slip between neighboring G-C base pairs in the DNA duplex.

Promoters and Dnh4ncers we mentioned that in the prokaryote E. coli, the promoter sites can be found at about the -10 and -35 regions. In eukaryotic DNA the promoter regions are centered around -25 (called the TATA box or Hogness box) and around -75 (called the CAAT box and ranges between -40 and -110). There are also enhancer sequences which also help to stimulate transcription. These enhancer sequences are usually quite far away from the promoter region (e.g., several kilobases
Copyright @ by The Berkeley Review

391

The Berkeley Review Specializing in MCAT Preparation

Biology

Expression of Genetic Information

RNA Synthesis

& Modifications

away) and can be either upstream or downstream from the promoter region.
They can even be within the transcribed gene itself. The process of whether or not a gene gets transcribed not only depends on the correct promoter region but also on the interaction with specific enhancers. ThL. can be seen in Figure 10-12. For example, consider the gene that codes for the synthesis of hemoglobin. Hemoglobin will only be expressed in one cell type, even though you have a great variety of different cell types in your body with the same hemoglobin gene in every one of them. It might be that cell or tissue specific proteins act as passwords which are capable of binding specific enhancer

regions. This interaction, in turn, will allow RNA polymerase to sit down on given promoter region and begin transcription of a specific gene.
Promoter Resion

Several kilobases aw

-75

-25

Gtr}Str
CAAT
TATA
so distant

DNA loops

sites are juxtaposed

Multiprotein
Complex

Figure lO-12
The use of enhancers can also be seen in the hormonal action of glucocorticoidr, Glucocorticoids can bind to a soluble receptor protein that is specific for a certaiun enhancer region. Once this hormone-receptor complex binds to glucocorticri enhancer region, transcription is stimulated.

will not be able to transcribe the genes that are important function unless you have the hormone as well. Males and mammary both have mammary glands. However, females have certain hormones which bind specific proteins. Interaction of this hormone-protein complex with enhancer regions allows for the transcription of mammary genes speci-tic females. Since males do not have enough of this hormone, transcription of mammary genes does not take place.
For example, you

Modification of the Transcripts A variety of modifications to RNA transcripts can occur. There can be
trimming and splicing (in which the ends of the RNA transcript can be excised or
Copyright @ by The Berkeley Review

392

The Berkeley Revier Specializing in MCAT Preparatior

Biology

Expression of Genetic Information

KNA Synthesis Er Modifications

certain intervening sequences removed). There can be (2) addition of bases to the primary transcript (such as the addition of the sequence CCA to the 3' end of IRNA molecules). There can also be (3) modification of bases, a feature that is particularly evident with IRNA molecules. (Processes (2) and (3) are quite common in prokaryotic cells.) For example, once you have a IRNA that has already been synthesized, its

A
HO- CH:n/
H

bases can be modified (post-transcriptionally). Once such case would a modification from uridine to pseudouridine as shown in Figure 10-13. Note the difference in attachment of the glycosidic bond.
The most extreme modifications are those seen in eukaryotic transcripts. Recall

HO

fi"
OH

"1..x

Uridine

that within the nucleus of a eukaryotic cell we find DNA replication,


transcription, and processing taking place before the transcripts are sent out into the cytosol. Once the transcript is in the cytosol, translation occurs at the
ribosomes.

Consider the processing of the primary RNA transcript. This primary RNA transcript can be modified by (1) capping at its 5' end, (2) addition of a polyadenine (poly-A) tail at its 3'end, and (3) splicing which involves the excision of introns.

5' Capping the of the Nascent RNA If the first residue at the 5' end of our nascent RNA were not modified, then you would expect to find three phosphate groups attached to the ribose ring. These "bare" 5'ends can easily be degraded by phosphatases and nucleases lurking within the cell. However, the 5' end is modified in a reaction with GTP.
First, the gamma phosphate is hydrolyzed at the 5' end of the RNA. Next, the beta phosphate at the 5' end of the RNA attacks the alpha phosphate of GTP, releasing pyrophosphate in the process (which is subsequently hydrolyzed to two molecules of orthophosphate). An unusual triphosphoanhydride linkage is formed between the 5' end of the RNA and the 5'end of the guanylate residue as shown in Figure 10-14,

HO

OH

Pseudouridine

Figure lO-15

-"ffc-o-i;'-i;'-i;:H" o-r- "- r- "- i "'* ",.1-.r)^


H,c-

Ji .*,':
HO
cap

\/

OH

o Ol o-P=o
I

ocHl

n- o Figure lO-14

"H
393

ocHj

i]opyright O by The Berkeley Review

The Berkeley Review Specializing in MCAT Preparation

Biology

Expression of Genetic Information

RNA Synthesis

& Modifications

s-adenosylmethionine (sAM) is the methyl donor that next adds a methyl group to the N-7 position of the purine ring system of guanine. Also, the z'-hyd"roxyl groups of the adjacent ribose rings can be methylaled by sAM as well. Capping protects the 5' end of the polymer from degradation and thereby tn! "rrnur,.", stability of the RNA molecule.

5' cAPr
Nascent

3'

3' Polyadenylation of the Nascent RNA As eukaryotic RNA polymerase nears the end of its transcription process, a sequence of bases is recognized in the primary transcrip t by 'a specific endonuclease. The primary transcript is cleaved and about 250 adenine ,"iidrru, (from ATP) are added to the free 3' end of the RNA polymer to form the polyadenylated tail as shown in Figure 10-15. The poly-A tait hetps to proteci
the 3' end of the RNA polymer from nucleases and phorphuturur.

A-A-A-(A)n-OH

RNA

Figure IO-lE

between regions of the gene that are translated orlre expressed (exonsj, The introns are spliced out-of the primary RNA in a process .uil"d splicing and th: exons are joined together. The exons are then used as the coding dorirains for proteins. The 5' non-coding region of the RNA is important for ribosome recognition. This is shown in Figure 10-16.
Primary RNA

Spticing (an overview) within many genes in eukaryotic DNA are sequences which are untranslatei. These untranslated sequences, called intervening sequences (introns), lie

-3'

t
Non-c.oding

l-

Inrron]rntron
I

f
Non-coding reglon

reglon

Processrng Events:

I
mRNA

Spticing: Capping; Polyadenylotion

t'.*

poly-A
Non-coding region Non-coding feglon

3,

Figure

I O-

l6

It is still a mystery as to why the exons are separated by the introns. one possibility might deal with recombination. A relaled possibility is that it might allow enhancers to come and go. This would allow for genetic variation within and among species with regards to the signals that say juit how turned on a gene should be. Recall that we mentioned that enhanc"r ,"qtr"r-r."s can often lie within a gene--in particular, the enhancers reside within these introns.
snRNPs can form Spliceosomes Within the cytosol and nucleus of many eukaryotic cells are RNA molecules that are associated with specific proteins. When this situation exists in the nucleus these complexes are referred to as small nuclear ribonucleoprotein particles (snRNPs), or more fondly referred to as "snurps". snRNps are iivolved with the
Copyright @ by The Berkeley Review

394

The Berkeley Review Specializing in MCAT preparation

Biology

Expression of Qenetic Information


If this

RNA Synthesis

& Modifications

base pairing reactions at the spliced junctions of introns and exons. snRNPs can

recognize the G-U and A-G. frame shift error might result.

processing is not done accurately, then

Self'Splicing RNA
The details of how snRNPs react are still not clear. However, it is quite possible that the actual chemistry is done by the RNAs and not the associated proteins. This came into consideration because of the discovery in ciliated protozoans that the self-splicing reaction involves rRNA (not mRNA) and not the associated proteins. In this case the reaction can be catalyzed by the attack of a 3'-hydroxyl of a guanosine cofactor residue on the phosphodiester bridge between exon A and the intron. This is shown in Figure 10-17.

Exon A ends up with a free 3'-hydroxyl that can next attack the phosphodiester bridge between exon B and the intron. The result is the splicing of exon A and exon B and the excision of the intron with the guanosine residue at its 5' end and a free 3'-hydroxyl. Further reactions (not shown) will ultimately release the guanosine residue, meaning that guanosine has a catalytic function. This type of intron self-splicing is referred to as being in Group I.
G-OH
I enack

uo-ltiTl-s'
o1-.'

f=ft--""
t-J

lrn,ron
t

L.@'''->
5'- Exon AlExon B

:3'

no-lp-oills'
Figure lO-17 This was the first clear example in which RNA could be involved in its own chemistry without the help of any enzymes. Up until a few years ago it was one of the holy laws of bioc\emistry that only proteins and their coenzymes could be catalysts. The clearest case in which RNA can calalyze reactions by itself are from the protozoan rRNAs.

Copyright @ by The Berkeley Review

395

The Berkeley Review Specializing in MCAT Preparation

Biol ogy

Expression of Genetic Information

BNA Synthesis & Modifications

so, under the category of self-splicing there can be Group I introns or Group II introns. Group I introns are spliced out by guanosine at the 3lhydroxyl. Group II introns are spliced out by adenosine at the 3'-hydroxyl.

There can be mutations in DNA which will affect splicing. For example, you could mutate the 5'splice site so that the G-u sequence is changed to some other sequence that is not recognized by the snRNP. Similarly, you could mutate the 3' splice site to something that is not recognizedby the snRNp. Also, the region that contains the adenosine moiety involved in the 2', S'-phosphodiester bond could also be mutated as well.

New splice signals could also be created. If a new A-G splice sequence came too soon in the intron, the exon at the 5' end could attack the wrong site. The resulting functional message would have a piece of the 3rend of the intron which would not be a coding sequence. This would lead to a defective protein. Mutations like these can turn up in the hemoglobin genes, and one of the more common types of mutations are those that cause thalassemia (defective synthesis of a hemoglobin chain(s)).

Copyright @ by The Berkeley Review

396

The Berkeley Review Specializing in MCAT Preparation

Biology

Expression of Genetic Information

Protein Synthesis

ffiftefifiilii:ffiffi

r$

Let's take a look at protein synthesis. Protein synthesis occurs on the ribosomes in three steps: initiation, elongation, and termination. However, before we can begin to consider these processes, we first need to consider how it is that amino acids are brought to the site of protein synthesis.

Amino acids are brought to the ribosome by an adapter molecule called transfer RNA (tRNA). This molecule contains a template recognition site for the mRNA codon called the anticodon and an amino acid attachment site. An amino acid (like Phe) would be attached to the 3'-OH function at the 3' end of the IRNA molecule. This can be seen in Figure 10-18.
The base sequence at the 3' end of the tRNAs is 5'-CCA-3'. The 5' end of the IRNA has a free phosphate group. Hydrogen bonds between U-A and G-C ribonucleotides help to hold the tRNA together at the stems. About half of the base pairs in IRNA can be found in these stems. Certain groups of bases within the tRNA molecule are not base-paired. Some of these groups are found in the loops of the molecule. For example, there is the dihydrouracil (DHU) loop, the ribothymine-pseudouracil-cytosine (TyC) loop, and the anticodon loop. These features define the secondary structure of IRNA.

o=

OO

eo.nin. ! o_.g":.Zo:.1 i_

OJOH

TJ
An amino
acid

o=c
I

H-C_R

t 1",

Figure rO-18

The Genetic Code: How was the genetic code deciphered? In 1961 Marshall Nirenberg took a solution of E. coli, lysed the cells, and removed the cellmembrane and cell-wall fragments. He was left with an extract that contained
various other constituents of the cell, including DNA, mRNA, IRNA, enzymes, and ribosomes. When amino acids and an energy source, such as ATP or GTP, was added to this extract and then incubated at 37 'C for a brief period of time, protein synthesis occurred on the ribosomes. Nirenberg was able to stop this protein synthesis by degrading the DNA template used for nRNA synthesis. It turned out that mRNA had a rather short half life (because it is degraded by specific cellular enzymes). Protein synthesis could be started once again if mRNA was added back to the extract. Nirenberg had access to an enz)rmatic protein called ribonucleotide phosphorylase. This enzyme was an undirected enzyme that allowed for the synthesis of synthetic chains of ribomrcleic acids as long as the right ribonucleotides were added to the medium. R&aI that DNA polyherase at d RNA polymerase were both directed enzymes. Ribonucleotide phosphorylase uses ribonucleoside diphosphates (NDPs) and catalyzes the synthesis of polyribonucleotides (RNA) with the subsequent release or orthophosphate (P). This can be seen in Figure
10-19.

nNDP + (RNA)n

1l

(RNA)'*1 +

nP1

Figure lO-19

Note that in the case of RNA polymerase, pyrophosphate (PP) is formed which can then be hydrolyzed by inorganic pyrophosphatase in order to help drive the reaction (of RNA synthesis) to completion. \A/hat this means is that in vivo the reaction involving ribonucleoside phosphorylase cannot be driven to the right (completion) by the subsequent hydrolysis of pyrophosphate. Why? Because there is no pyrophosphate produced. Why? Because in this case we are using ribonucleoside diphosphates (NDPs) instead of ribonucleoside triphosphates (NTPs). In other words, the equilibrium for the reaction shown in Figure 10-19 is to the left (degradation of RNA). In fact, ribonucleotide phosphorylase actually takes partially degraded mRNA and phosphorylates it to form ribonucleoside diphosphate substrates. So, how does ribonucleotide phosphorylase catalyze the
Copyright @ by The Berkeley Review

397

The Berkeley Review Specializing in MCAT Preparation

Biology

Expression of Genetic Information

Protein Synthesis

synthesis of mRNA? It depends on the concentration of the ribonucleotides added to the reaction.mixture. If high concentrations of a given ribonucleotide are incubated with ribonucleotide phosphorylase, then th! reaction shown in Figure 10-19 will be driven to the right.

lst
Position

2nd Position

(5'End)

U
Phe Phe Leu Leu
Leu Leu Leu Leu Ile Ile Ile Met Val Val Val Val

C
Ser Ser Ser Ser
Pro Pro Pro Pro Thr Thr Thr Thr Ala Ala Ala Ala

A
Tyr Tyr STOP STOP
His His Gln Gln
Asn Asn

G
Cys Cys STOP

3rd

(3'End)

U
C

A
G

Trp

Arg Arg Arg Arg


Ser Ser

U
C

A
G

Lyt
Lys

Arg

A
U
C

A.g
Gly Gly Gly Gly

Arp Asp Glu Glu

A
G

Table lO-2
The Genetic Code.

translated on the ribosome, the protein that was synthesized contained all phe (i.e., poly-Phe). Nirenberg was able to postulate that the nucleotide triptet UUL coded for the amino acid phe.

ten-fold more magnesium than normally found in cells. once pory-u

when Nirenberg used high concentrations of the ribonucleotide uDp in the presence of ribonucleotide phosphorylase, the polyribonucleotide uuuuuL(i.e., poly-u) was synthesized. It turns out that iiboro*", are not designed to take substrates like poly-U' In order to get protein synthesis, artificial conditions had to be employed. For example, 0.01 M Mg2e ions were used, which is at leas;
.w,as

How does the ribosome read the mRNA in order to make proteins? we knoir that proteins are composed of amino acids. In 1961 various ieatures of the base sequence in DNA and the amino acid sequence in proteins led to the establishment of a genetic code. There are foui different Lases in DNA. If the codervere a simple single-base code, then only 4 different types of amino acids could be specified. And since we know that we have at teasi zo different aminc acid this does not seem too likely. If we had a two-base code, then 16 differerri types of amino acids could be specified. Again, this is still less than the 20 different amino acids that we know we have. F{owever, if we had a three_base
Copyright @ by The Berkeley Review

59a

The Berkeley Review Specializing in MCAT preparation

Biology

Expression of Genetic Information

Protein Synthesis

code, then we could specify 64 amino acids. Indeed, the 20 different amino acids molecule. These groups of three bases are referred to as codons (see Table 10-2)

that we have been exposed to are coded by a three-base code on the DNA

which way are the proteins synthesized? Is it from the amino terminal to the carboxyl terminal or is it from the carboxyl terminal to the amino terminal? For proteins it was postulated that they were synthesized from the amino end to the
carboxyl end.

In order to determine this, hemoglobin synthesis from the red blood cells (RBCs) of rabbits was analyzed. There are immature RBCs that make almost nothing but hemoglobin. If you inject a rabbit with hydrazine, it will destroy the rabbit's RBCs. To compensate for this the rabbit will begin to synthesize new RBCs. If you had a start site for hemoglobin at one end of the protein or the other, then you would make a longer and longer chain until you reached the end. It should be possible to label chains that are almost complete just by adding radioactively labeled amino acids to your reaction mixture for just a few seconds. The ends of the polypeptide chains that was synthesized last would be labeled.
The rabbit RBCs were lysed and amino acids with a label were added to the solution for a few seconds. The mature hemoglobin was isolated in order to find out where the label was. The label was found to be at the carboxyl end of the polypeptide. Therefore, the carboxyl end was synthesized last while the amino end was synthesized first. You must also rcalize that as the protein is being synthesized from the N-terminus to the C-terminus, that region near the Nterminus can begin to fold up on itself to give tertiary structure even before the rest of the protein has been synthesized.

5'-AAAAAAAACAAA-3' fl Ribon.r"l""." v 5'-AAAAAAAAC-3'

5'-AAA-3'
$Transtation

N-Lys-Lys-Asn-C

fl 1'

Carboxypeptidase

N-Lys-Lys-C

N-Asn-C

Figure lO-2O

In what direction is the mRNA read? Polynucleotide phosphorylase

was

incubated with a lot of ADP and a little bit of cDP in order to make a mixed polymer. This polymer mostly consisted of A residues. Occasionally a C residue would be incorporated. The polymer was then treated with pancreatic ribonuclease which cleaves on the 3' side of C residues. What is observed are polymers with long runs of As ending in a C residue at the 3'end. This polymer can then be used as a translation system. What you will get is a polymer with a lot of Lys in it and a very little bit of Asn. When this polymer is treated with carboxypeptidase, Asn will be released from the poly Lys polymer. This is shown in Figure 10-20. Carboxypeptidase is a digestive enzyme that hydrolyzes the carboxyl-terminal peptide bond in polypeptide chains, particularly if that residue has a bulky aliphatic side chain or an aromatic ring. Thus, Asn must be at the carboxyl end of the polymer (the end that is synthesized last). This means that the coding triplet for Asn is AAC. In other words, the coding triplet for Asn is at the 3'end of the polymer and also at the carboxyl end of the protein. The direction of translation is from 5'-+ 3'. When RNA polymerase is transcribing DNA in order to

synthesize RNA,

{ About half of the codons were determined by a triplet binding assay. Suppose that ribosomes were able to bind to trinucleotides. The example that we will use is 5'-AUU-3'. Once this binds to the ribosomes, then IRNA can bind. In this case the tRNA for Ile would be the only IRNA that would bind to the ribosomes.
Copyright
@

translation.

it

makes RNA

in the

same direction

in which you

get

by The Berkeley Review

399

The Berkeley Review Specializing in MCAT Preparation

Biology

Expression of Genetic lnformation


codons could be worked out.

Protein S5rnthesis

Thus, AUU was said to be the coding unit for lle. In this way various other

The remainder of the genetic code was filled out by Gobind Khorana. Khorana was able to synthesize polyribonucleotides with a repeating sequence that was well defined from one of the two strands of a double strarided DNA molecule that had the repeating deoxyribonucleotide residues 5,-TAc-3' and 5'-GTA-3'. In order to be selective about which of the two DNA strands that were to be transcribed, only three of the four complementary ribonucleoside triphosphates were added to the reaction mixture. This technique allowed one strand of the duplex DNA to be transcribed but not the other itrand. when Khorana added ATP, urP, and CTP to the mixture containing the double stranded DNA, he got the repeating RNA sequence 5'-UACUACUACUACUACUAC-3'. when he added GTP, urP, and ATP to the mixture containing the double stranded DNA, he got the repeating RNA sequence S'-CATCATCAT-ATCATCAT_3'.

once the defined RNA polymers were synthesized they were used as templates for protein synthesis on the ribosome. For example, if ihe mRNA polymer were poly -UAC, there would be three ways to read it, we could either rlad it as polyUAC, poly-ACU, or poly-CUA. This will give us three different polymers. t? ltis read as poly-UAC, then we will get poly-Tyr. If it is read as poty-lcu, then we will get poly-Thr. If it is read as poly-CUA, then we will geipoiy-Leu. This can be seen in Figure 10-2'1,a.

5'-U-A-C-U-A-C-U.A-C-U-A-C-3' 5'-G-U-A-G-U-A-G-U-A-G-U-A-3' gatJHlt---Jt-'_r

Tyr Tyr Tyr Tyr

Val Val Val


\---

Val

5'-U-A-C-U-A-C-U-A-C-U-A-C-3' 5'-G-U-A-G-U-A-G-U-A-G-U.A-3'
--J

Thr Thr Thr


-/J

Stop Stop

Stop

5'-U-A-C-U-A-C-U-A-C-U-A-C-3' 5'-G-U-A-G-U-A-G-U-A-G-U-A-3'
\--l -r#

Leu Leu Leu (a)

Ser Ser
(b)

Ser

Figure IO.2l However, if we read the complement of the poly-uAC strand, which is poir-GUA, then we will get only two polymers. Why? We could read poly-GUA a_. either poly-GUA, poly-UAG, or poly-AGU. If we read it as poly-GUA, then u-e will get poly-Val. If we read it as poly-AGU, we will get poly-Ser. However, jj we read it as poly-uAG, then nothing ever appears. It turns out that 5'-uAG'-3' L. a stop codon. The other termination codons are 5'-UAA-3'and St-uGA-3'. ThL. can be seen in Figure 10-21b.

Not only are there stop signals but there are also start signals. The start signaXcould not be determined by using Nirenberg's system because his translati.on system would start anywhere (due to the high magnesium levels). In order t;
Copyright @ by The Berkeley Review

40o

The Berkeley Reviel Specializing in MCAT Preparation

Biology
mRNA).

Expression of Genetic Information

Protein Synthesis

find the start signals, a natural mRNA molecule was needed (not a synthetic
The RNA from RNA bacteriophages can be recognized from regular DNA DNA (due to the presence of uracil and the extra oxygen atom at the 2'position of the ribose ring). The RNA from these phages, when first isolated, was thought to have about 3,000 nucleotides (or about 3 genes). A start site on the RNA from one of these RNA phages was determined in the following manner.

because RNA is denser than

Sticking out of this length of phage RNA is a small protrusion that is the start signal for the coat protein of the phage. Ribosomes can be denatured into the 30S and 50S subunits in low magnesium concentrations. The 30S subunits of the ribosome will bind to the start signal for the coat protein. If ribonuclease is added to the solution, then the rest of the RNA will be degraded. The only RNA that is left will be that stretch of RNA "protected" by the 30S subunit of the ribosome. In other words, the start signal is not denatured. Within this protected piece of RNA is the sequence 5'-AUG-3' followed by a few more coding residues. This triplet codes for the amino acid Met. If you now take the phage coat protein and analyze the amino terminus, you will find that it starts with Met, followed by the corresponding coding residues. The start codons for the maturation protein (only one copy at the vertex of the phage) and the replicase protein (copies RNA to make more RNA) were discovered in a similar fashion. The RNA was sheared in a Waring Blendor to obtain smaller fragments. The same experiment was repeated by binding the 30S subunit of the ribosome to all the fragments and then looking for protected segments. Both of these genes also start with 5'-AUG-3'.

It turns out that the most common start codon is 5'-AUG-3'which codes for Met. There are some rare cases in which the start codon is 5'-GUG-3'. This triplet codes for Val.
Eukaryotic cells also start their proteins by using the codon 5'-AUG-3'. Instead of

CH.

fMet

first amino acid we find that it is simply Met-but with a slight variation. Eukaryotic messages get processed before they even reach the ribosome. For example, if the first residue at the 5'end of our nascent RNA were
as the

Gr
5'

Nascent
RNA

A-A-A-(A)n-OH
3'

not modified, then you-would expect to find three phosphate groups attached to the ribose ring, These "bare" 5' ends can easily be degraded by phosphatases and
nucleases lurking within the cell.

Figure lO-22

However, the 5' end is modified in a reaction with GTP. First, the gamma phosphate is hydrolyzed at the 5' end of the RNA. Next, the beta phosphate at the 5' end of the RNA attacks the alpha phosphate of GTP, releasing pyrophosphate in the process (which is subsequently hydrolyzed to two molecules of orthophosphate). An unusual triphosphoanhydride linkage is formed between the 5'end of the RNA and the 5' end of the guanylate residue.
A methyl donor (S-adenosylmethionine (SAM)) adds a methyl group to the N-7 position of the purine ring system of guanine. This process is called capping' Also, the 2'-hydroxyl groups of the adjacent ribose rings can be methylated by SAM as well. Remember, capping not only protects the 5' end of the polymer from degradation but it also enhances the stability of the RNA molecule as well. This can be shown in Figure 10-22.
Copyright
@

by The Berkeley Review

40t

The Berkeley Review Specializing in MCAT Preparation

Biology

Expression of Genetic Information


sequence

Protein Synthesis

As eukaryotic RNA porymerase nears the end of its transcription process, a of bases is recognized in the primary transcripi by i specific endonuclease. The primary transcript is cleaved and about 250 adenine residues (from ATP) are added to the free 3' end of the RNA polymer to form the poiyadenylated tail as shown in Figure 10-22. The poly-a tait hetps to protect the 3' end of the RNA polymer from nucleases and phorihutur"r.
are sequences which are untranslated. These untranslated sequences, called intervening sequences (introns), rie between regions of the gene that are translated or-are (exorrsj. The introns are spliced out-of the primary RNA in a process ""pr"rrud splicing and the "ull"d exons are joined together. The exons are then uied as the coding doirains for

within many genes in eukaryotic DNA

proteins. The 5' non-coding region of the RNA is important


recognition. This is shown in Figure 10-23.

lor

ribosome

Primary RNA

- --^--r1

Trntron=rntron? t Non-c.oding t
reglon

"*on

^.r.r.nffi6

-.1

-'-a"

mRNA

I o.o..rr'n, Evenrs: I Spticing; Capping;


Polyadenylation

Non-coding regibn

it23t
Non-c.oding regron

poly-A
Non-c.oding

3,

reglon

Figure 1O.23

It is still a mystery as to why the exons are separated by the introns. one possibility might deal with recombination. A relaied possibility is that it might allow enhancers to come and go. This would allow for genetic variation within and among species with regards to the signals that say just how turned on a gene should be.
Amino Acid Activation The activation of amino acids is similar to the process in which fatty acids are activated and to the process in which DNA ligase joins to segments or brua.
unfavorable. This unfavorable situation is overcome by activation of tire carbox;,I group of incoming amino acids. In the first step of the reaction ATp reacts with an amino acid to form an activated mixed anhydride linkage in the form of aminoacyl-AMP. In the second step the aminoaryl group of a-minoacyl-AMp is transferred to a IRNA molecule to form aminoacyl-tRNA. rnis transfer can take
Copyright @ by The Berkeley Review

Peptide bond formation between two amino acids is thermodynamicallr-

402

The Berkeley Review Specializing in MCAT preparation

Biology

Expression of Genetic Information

Protein Synthesis

place at either the 2'-hydroxyl or the 3'-hydroxyl of the ribose moiety at the 3' end of the tRNA molecule. The amino group, however, can migrate between either of these two hydroxyl functions. Aminoacyl-tRNA is the activated intermediate in

protein synthesis and the enzyme that catalyzes this reaction is an activating enzyme (i.e., a specific aminoacyl-tRNA synthetase). This is shown in Figure 1024. Step

HO O I ll H1N-C-C-O 'l
R

ATP

HOO t il ll H3N- C-C- O- P- O- Ribose + lrl R oo Adenine o


Aminoacyl-AMP
Transfer RNA
I

ppi

ol
Step 2

o
I

O-P=O
O

Aminoacyl-AMP + IRNA

- CH2 ,.,

)/"v H-\J-"
HN

Adenine

oll

Adenine
I

O-

P-O-Ribose

lo

/\

o AMP

OH

o=c
I

H_C_R
@t
NH:

Aminoacyl-tRNA

Figure 10.24
You shouid be aware that there are many different IRNA molecules within the cell, and they might average about 80 bases long. This means that there has to be great specificity as to which amino acid is placed on which IRNA molecule. There is a minimum of 20 different activating enzymes, one for each amino acid. Each activating enzyme is quite selective. For example, it will only place valine on the IRNA specifyilg valine and not on the tRNA specifying isoleucine. Valine and isoleucine are rather similar to one another and differ only by the fact that :soleucine carries an extra methylene GCHz-) group. How does the activating .nzyme distinguish between the two amino acids?

.{minoacyl-tKNA Synthetase Specificity '\'e need to consider the accuracy of the aminoacyl-tRNA synthetase enzyme
:.e., the activating enzyme). Roughly 1 mistake is made for every 3000 catalytic -r-ents. One way to assure this accuracy is to have a proofreading system aaalogous to that in DNA replication). In many IRNA synthetases there are two :atalytic sites--a hydrolytic site and an acylation site. The hydrolytic site is

:cncerned with the aminoacyl-AMP molecule while the acylation site is , rncerned with the aminoacyl-tRNA.
:uppose we want to attach isoleucine to a IRNA molecule but in the initial step . e mistakenly activate a valine molecule (i.e., valine-AMP). Since valine is

,naller than isoleucine it can easily enter the hydrolytic site where it
,

is

rpyright O by The Berkeley Review

403

The Berkeley Review Specializing in MCAT Preparation

Biology

Expression of Genetic lnformation


the hydrolytic site and therefore is not

Protein Synthesis

zubsequently hydtolyzed. Since isoleucine is larger than valine it cannot fit into hydrolyid. However, isoleucine can fit into the acylation site where it activated and attached to an isoleucine specific IRNA.

error prone comp_ared to replication? Most transcripts are rather short (less than 104 or 105 baseJlong). if ybu have a probability oi ot tt111'a or t/10u,;h"" "rro, most of the molecules that are made that are less than 104 or 105 units long will not have any errors. In other words, you do not need to have an error ra"te of Ll10tu if you are making a molecule that is only 11104 or 1/105 nucleotides lons. The same holds true for protein synthesis. Error rates of lllr1a are fine b".a.rr"" you are not making proteins that are 104 amino acids long.

During replication we mentioned that the error rate is about 1'l101o. During transcription the error rate is about 1/104 or 1/105. Why is transcription more

ffi

mRNA

**;X
3',

3'

define the secondary structure of tRNA.

tRNA Design Atl IRNA molecules are capable of forming a cloverleaf structure that is about g0 ribonucleotides long. An amino acid like phenylalanine would be attached to the terminal adenosine residue at the 3'end of the IRNA. The 5' end of the tRNA has a free phosphate group. Hydrogen bonds between U-A and G-C ribonucleotides help to hold the IRNA together at the stems. About half of the base pairs in IRNA can be found in the stems. One will occasionally see G-U base pairing (we will come back to this point later). Certain groups of bases within tne tnNCmolecule are not base paired. Some of these groups are found in the loops of the molecule. There is the dihydrouracil (DHU) loop, the ribothymine-pseldouracil-cytosine (TyC) loop, and.the anticodon.loop. The featurei that we have just mention

Figure lO-25

The IRNA molecule is not flat and planar but rather has tertiary structure irwhich the molecule is L-shaped and contains two segments of doutle helix, eaC containing about 10 base pairs. Base pairing in the nonhelical regions of thE molecule are rather unusual in that there can be base-base, base-b-ackbone, ol backbone-backbone interactions. The backbone interactions involve the 2'hydroxyl function of the ribose ring.

Codon

Codon

Codon-Anticodon Interactions A triplet of three bases in an mRNA polymer represent a codon that calls for " particular amino acid. The amino acid that is called for should be attached ic : tRNA molecule that has an anticodon that is complementary to the codon of -,:.: nRNA. This can be seen in Figure 10-2s. Note that the codon-anticod.i_
relationship is anti-parallel. The rules for base pairing of the codon-anticodon junction are like those ,fi:: operate in DNA. In this case A will pair with U and G will pair with C. Hou.e-,,e: there is an additional possibility in which G can pair with U. Recall that there .:: only 20 different u*ino acids but there are o+- triplets (3 stop codons anc. dl. informational codons). on the average there are 3 codons per amino acid. Thr:r are not 61 tRNA molecules, which means that a given IRNA can recognize r:,::* than one codon. For example, 5'-uUU-3'and 5'-uuc-3'are codons tiat spe_-:, phenylalanine. Both of these codons are recognized by the same IRNA which" :ro the anticodon sequence 5'-GAA-3'. Codon-anticodon pairing can be see:. rm Figure 10-26. How can one IRNA base pair with two different codons? The b::.r*i for this phenomenon has to do with wobble in the base pairing.

JI

YVU\./ UUC
AAG
Anticodon

I I I 'T;'J"'

AAG
Anticodon

Figure 1o'.26

;n

Copyright @ by The Berkeley Review

4o4

The Berkeley Rericm Specializing in MCAT preparatiun

{m,r

Biology

Expression of Genetic lnformation

Protein Synthesis

These two codons, 5'-UUU-3' and 5'-UUC-3', are both recognizedby the same tRNA as shown in Figure 10-27. The IRNA that is recognized by the activating
a specific activating synthetase. whenthis specific activating enzyme transfers the aminoacyl group of Phe to tRNAPhe, we can designate that activated aminoacyl-tRNA as Phe-tRN4Phe. Note that a charged IRNA would have the amino acid attached to it while the uncharged tRNA does not have the amino acid attached to it.

enzyme is tRNAPhe, and in particular

it is recognizedby

enzyme for Phe called phenylalanyl-tRNA

3'-Phe 5'

3'-Phe 5'

'*^1^L
5'-y-g (a)
Figure lO-27
Note that in Figure l0-27a we have a non-standard base pair between U and G while in Figure l0-27b we have a standard base pair between C and G. We can represent these pairings as shown in Figure 10-28. We can line the structures up such that positions 2,3, and 4 in the pyrimidines and positions2,l, and 6 in the purines are facing each other.

3'

mRNA

5'-

A-A U-U _C(b)

3'

ln the case of tlte non-standardbase pairing between U and G, the pyrimidine base is displaced with respect to the purine base. This is shown in Figure 10-28a. If this base pairing were to occur in a DNA double helix, it would distort the
backbone. However, in IRNA this does not matter too much because we are not

making that large of a double helical structure. We simply have three bases of nRNA pairing with three bases of IRNA. In other words, there is a great deal more flexibility in this type of interaction in IRNA (compared to DNA). Standard hydrogen bonding between the C and G base pairs will occur as shown in Figure
10-28b.

N
I

)
Cvtosine (b)

Guanine

Backbone

Guanine
(a)

Figure lO-28
Copyright @ by The BerkeleY Review

405

The BerkeleY Review Specializing in MCAT Preparation

Biology

Expression of Genetic Information


Wobble Rules in Base Pairing

Protein Synthesis

Wobble is simply steric freedom in the pairing of the third base of the codon. The wobble rules apply at the first base of the anticodon and the third base of the codon. These rules are:

1. 2. 3.

U can pair with A (standard) or G (non-standard). G can pair with C (standard) or U (non-standard). Inosine (I) can pair with either A, U, or C"

We could have a four-fold degenerate codon such as 5'-GGG-3', GGA, GGU, GGC, where they would all specify glycine. Three of these codons could be read by one IRNA if the IRNA had hypoxanthine at the first position of its anticodon. The other codon (GGG) would need to be read by a different tRNA. Recall that hypoxanthine is the purine base of inosinate (IMP), an intermediate in nucleotide
biosyrrthesis.

Ribosome Structure About two-thirds of the weight of a ribosome is IRNA. Ribosomes in E. coli are Iarge molecules, about 200 A in diameter and having a mass of about 2700 kd. The ribosome is described in terms of a sedimentation coefficient called the Svedberg unit (S). A ribosome (70S) can be dissociated into a small subunit (30S) and a large subunit (50S). This is shown in Figure 10-29. Note that it is a nonlinear relationship between size and S-value. The dissociation of the 70S ribosome is controlled by the concentration of Mg2o. Low Mg2e causes
dissociation of the ribosome.

Ribosome

TOSA [ ..',,,,,.
\ /\

50s subunit

qry
I
I
Yo

30s
subunit

-+?
23S

rRNA

55 rRNA

165 rRNA

+ 34 proteins

21 proteins

Figure lO-29
The 50S and the 30S subunits can be further dissociated in the presence of urea. Dissociation of the 50S subunit yields about 34 proteins and 23S rRNA and 55

Copyright @ by The Berkeley Review

406

The Berkeley Review Specializing in MCAT Preparation

Biology

Expression of Genetic lnformation

Protein Synthesis

rRNA. Dissociation of the 30s subunit yields about 21 proteins and 165 rRNA. rRNA has a well defined folding pattern containing many short duplex regions.

Ingredients for Protein Synthesis


In order to make
a

protein we would need the following items:

1,

20 aminoacyl-tRNA's, each attached to the correct tRNA (more may be needed).

2. nRNA
desired.

that contains the codons that specify the protein sequence that is

3. Ribosomes. 4. Additional proteins 5.

called initiation factors that are not part of the

ribosome. There are initiation, elongation, and termination factors. GTP as an energy source.

Initiation of Protein Synthesis


Consider a fairly late stage in initiation when the 50S and the 30S subunits have joined together to form the 70S ribosome. The nRNA is appropriately placed with its 5'end in the 30S subunit. The mRNA sequence which has recently been synthesized from our DNA duplex has signals which are important. For example, the mRNA molecule has polarity-it has a 5'end as well as a 3' end. The initiating codon in mRNA is the base triplet 5'-AUG-3'. About 10 nucleotides upstream from the initiating codon is a purine-rich sequence of bases (e.g., AGGAGGU) called the Shine-Dalgarno sequence. The Shine-Dalgarno sequence is responsible for binding the mRNA to the 165 subunit--an architectural constituent of the 30S ribosomal subunit. Binding of the ShineDalgarno sequence to the 3' end of the 165 ribosomal subunit forms the initiation complex as shown in Figure 10-30.

mRNA

3'-OH AU UC

+-

3'end of 165
ribosomal RNA

UCCUCCA

s'-GAUUCCU AGGAGGU UUGACCU AUG CGAGCU UUU AGU-3'

fMet-Arg-Ala-Phe-SerShine-Dalgarno
Sequence

Polypeptide
Sequence

Figure f O-3O
The codon in mRNA that starts protein synthesis is 5'-AUG-3'. This codon is :a1ling for a special modified amino acid called formylmethionine (fMet). How is :his modified amino acid brought to the ribosome for protein synthesis? It is rrought to the ribosome by an adapter molecule called IRNA. We can abbreviate --nis IRNA adapter molecule with fMet on it as fMet-tRNAf. Remember, a IRNA rolecule contains a template recognition site (for the nRNA codon) called the anticodon and an amino acid attachment site.

,-.rpyright @ by The Berkeley Review

speciarizin"

ffir3i'f Ft:L }.:HH

Biology

Expression of Genetic Information

Protein Synthesis

flVlet

fM",

,RNA.l

i'

l"l
30S

After the 30S subunit has formed a complex with the initiation factors that we mentioned as part of the ingredients in protein synthesis, GTP will bind to a particular initiation factor called IF2. This allows the mRNA and the fMettRNAf initiator signal to join the 30S subunit as well. The subsequent release of these initiation factors allows the 50S complex to attach to the 30S complex, thus forming the 70S initiation complex. The 70S initiation complex is now primed for protein synthesis. This can be seen in Figure 10-31.
,,'f,i",,
mRNA

Initiation Complex

It turns out that there are two tRNAs for methionine. One form of IRNA, called tRNAf, has the initiation role we just mentioned, while the other IRNA, we'll call tRNAm, will simply bind to AUG codons elsewhere in the mRNA.
Let's consider the process by which methionine is attached to its respective tRNA. We would expect that an activating enzyme will attach methionine to tRNAf and form Met-tRNAf. We would also expect the same activating enzyme to attach methionine to tRNAm.

A-site \
3',

li n
70S

fMet

In the next step a formyl group attached to THF will react with Met-tRNAf to form formyl-methionine-tRNAf (abbreviated as fMet-tRNAf) and THF without the formyl group. The formyl group has been attached to the methionine residue as shown in Figure 10-32. Because the amino group on methionine is formylated, it is called N-formylmethionine-tRNA.

Initiation Complex

Figure I O-3 t

Elongation Once the 70S initiation complex is primed for synthesis, what happens? Suppose the next codon in our mRNA is 5'-CGA-3' as shown in Figure 10-30
and Figure 10-31. Looking at our genetic code we find that this codon codes for the amino acid arginine (Arg).

oHo ll t il H-C-N-C-CH

tt

O-rRNAi

Arginine is attached to the 3' end a specific IRNA by a specific aminoacyltRNA synthetase and is then carried by this arginine-specific aminoacyl-tRNA to the empty A-site on the 70S complex. Next to the A-site is the P-site. Note that the P-site is already occupied by fMet-tRNAf. This is shown in Figure 10-33.

CH]

tI

CH,
S
I

P-site
3',

A-site

cHr

fMet
1

Formylmethioninyl-tRNA (fMet-rRNAf)

Figure lO-32

JI
70S

Initiation Complex

Figure lO-33

Ti

Copyright @ by The Berkeley Review

4oa

The Berkeley Revieu Specializing in MCAT Preparation

C,.

Biology

Expression of Genetic Information

Protein Synthesis

once we have amino acids in both the p-site and A-site, the enzyme peptidyl transferase wlrl catalyze the formation of a peptide bond. The amino nltrogen tr Arg attacks (nucleophilic) the carbonyl carbon of the ester linkage in itvtettRNAf. The result is cleavage of that ester bond and the formation Jf a peptide bond between fMet and Arg. In other words, the fMet moiety of the p-site will be transferred to the amino group of the Arg residue of the A-site. The formation of the dipeptidyl-tRNA at the A-site is a favorable reaction.
The empty tRNAf will leave the p-site and the mRNA will move a distance of three nucleotides thus bringing the dipeptidyl-tRNA (5'-fMet-Arg-3') to the psite. This is called translocation and can be seen in Figure 10-34. we are now left tfe A-site empty and ready for the next incoming amino acid (specified by rylth the mRNA codon) during the second round of elongation.

ff:'

J
5'

AUG

I
A-site

mRNA

70S

Initiation Complex

Figure lO-34

Energetics of Protein Synthesis


Consider the ATP equivalence involved in protein synthesis. Recall that each of the amino acids has to be activated by binding to tRirlA. This process involves 2 ATP equivalences (one for the hydrolysis to get pyrophosphate and one for the

hydrolysis of pyrophosphate itself). rhe blnding"of in aminoacyi-tRNA to its respective codon costs an ATP equivalence. In the translocation step we find that no ATP equivalendOs are required for the first amino acid because i1 is already at the P-site. Flowever, each additional amino acid will bind at the A-site first and then translocate over to the P-site. This process will require l ATp equivalence eachl If we add up the sum in terms of ATp equivalences, we will find that g are required for the first amino acid and then 4 for each subsequent amino acid. This is summarized in Table 10-3.

fMet
Activation Binding
Translocation

aa2

aa3

Aan

)))', 111t 0111

ATP Equivalence

Table lO-3

Copyright @ by The Berkeley Review

409

The Berkeley Review Specializing in MCAT preparation

Biology

Expression of Genetic Information


Termination

Protein Synthesis

suppose we are close. to the end of the 3' end of the mRNA. In Figure 10-35 is our mRNA with some nth codon. Attached to the nth codon is a po-lypeptide chain with some nth amino acid. This is all at the p-site. \Arhat we will find is that a termination signal (e.g., UAG, uAA, or UGA) will come into view. Let's use 5'UAG-3'as our stop signal. These stop signals are not recognized by a IRNA but instead are recognized by a protein release factor that will bind to the signal when it comes into view. This release factor has an effect on peptidyl transferase, the enzyme that forms peptide bonds between two amino acids. peptidyl transferase will now use water instead of an amino group to attack the ester linkage of the nth amino acid at the P-site. This wilirelease the polypeptide chain.

Y'' Arg
aa- aa-aa-aa
I

P-site

A-site

5' - XXX
70S

XXX_

UAG

mRNA

Initiation Complex
P

Figure lO-35
HN

: ;

gl

Puromycin
OH

o=c

;-r-.",/\o..n, | \_,/
NH^

Figure f 0-36
Puromycin

You should be aware of the fact that inhibitors were useful (and continue to be) in working out the pathways of protein synthesis. one inhibitor that we can mention is puromycin, which is shown in Figure 10-36. Puromycin binds at the A-site and acts as an analog of an aminoacyl-tRNA, thus preventing other aminoacyl-tRNAs from entering the A-site. The q-amino group of puromycin forms a peptide bond with the carboxyl group of the amino acid at the p-site. once this happens peptidyl puromycin leaves the 70s ribosome, thus causing premature termination of protein synthesis.

Ft

iorl

::
b".r

:c:

:-

ra
_:l

.imc

:i1: Copyright @ by The Berkeley Review

4to

The Berkeley Revieu Specializing in MCAT Preparatiom

Biology

Dxpression of Genetic Information

The Lactose Operon

The Lactose Operon


In bacteria the majority of regulation occurs at the level of transcription (i.e., to make or not to make a particular mRNA). we will now consider the lactose operon. what exactly is an operon? An operon is simply a transcription unit which is involved in the expression of multiple genes. Another way to word this would be that an operon is an arrangement of genes on the DNA which are
cotranscribed.

rn 1967 Francois jacob and jacques Monod proposed the operon model for the regulation of protein synthesis from work they were doing on the metabolism of the disaccharide lactose (glucose and galactose). Even though the bacterium E. coli pteferc glucose as a substrate it can survive on alternate carbohydrates such as lactose, galactose, and even arabinose. However, in order for E. coli to utilize these other sugars it must synthesize a whole new series of enzymes. E. coli d,oes not want to be synthesizing these enzymes unless a particular substrate, like lactose, is in the medium. Thus, E. coli can induce the expression of certain genes only when they are needed. A priori one can imagine an inducible system working in two ways: (1) If you have a system that is always on and then you tum it off or (2) if you have a system that is always off and then you turn it on. This "turning on" and "tuming off" can be mediated by a variety of different mechanisms.
We mentioned that an operon is simply a transcription unit which is involved in the expression of multiple genes. For example, the lactose operon includes a promoter (P), an operator (O), and structural genes (2,y, and a). Upstream from the lactose promoter (on the 5' side of the lactose promoter) is the regulatory gene (i) with its own promoter (P1). These elements of the lactose operon model are shown in Figure 1,0-37. we will modify this diagram as the discussion continues.

Regulatory I gene I

Contlol

sltes

StructuratGenesJ

Pr

+
Figule tO.57

Lactoseoperon

---+l

When the enzyme RNA polymerase binds to the lactose promoter it can begin to transcribe the structural genes of the lactose operon. Structural gene z codes for

b-galactosidase whose major function is to hydrolyze lactose to glucose and galactose (and whose minor function is to convert lactose to allolactose).

Structural gene y codes for galactoside permease, a carrier molecule that transports lactose into the cell. Structural gene a codes for thiogalactoside transacetylase whosdtole is still uncertain.
The regulatory gene (i) codes for a repressor protein that can bind to the operator and prevent the transcription of genes z, y, and a when lactose (the inducer) is not present in the cell's medium. In other words, the products of those genes act on lactose in order to metabolize the disaccharide. If lactose is not
Copyright @ by The Berkeley Review

4tt

The Berkeley Review Specializing in MCAT Preparation

Biology

Expression of Genetic Information

The Lactose Operon

why waste the cell's energy to synthesize those gene products? [An present, -operator is simply a DNA sequence that binds a repressor.]
The conclusion that Jacob and Monod came to was that there is a diffusible component in the cell that is involved in turning down the exp,ression of a whole series of genes which are all involved in the metabolism of lactose. The set of genes thal was need.ed to metabolize lactose would only be turned on and then Jynthesized when lactose was present in the medium. Those particular genes are the structural genes (mentioned above).
Let's consider what happens when lactose is not present in the medium' When lactose is not present, you do not want RNA polymerase to transcribe the structural gurr", y, uttd a. The expression of these three genes are controlled by ", the regulaiory gene (i). The regulatory gene codes for a repressor protein as showti in Figure L0-38. This gene is transcribed separately from the structural
genes of the lac operon.

Pr

mRNA

.-A

s
o

Repressor

protein

ffi

Figure lO-34
Once the repressor protein is synthesized it can do one of two things' If lactose, is

not in the medium, the repressor protein will bind to a region in the DNA
sequence called the operatoi (O) and prevent RNA polymerase from transcribing as the structural genes. Th" t"ptestor protein binds tightly to the operatol region

shown in Figure L0-39.

RNA polymerase cannot


transcribe the structural genes

Repressor protein binds to the operatir

I
Figure lO-59 Even though the RNA polymerase can bind to the plomoter site while the repressor piotein is bound to the operator site, it seems that the rePressor protein pievents transcription by RNA-polymerase because it interferes with the in the formation of the transcripiion bubbie. It turns out that there is an overlap The BerkeleY Review Specializing in MCAT PreParation
I

tr

Copyright @ by The BerkeleY Review

412

Biology
molecule.

Expression of Genetic Information

The Lactose Operon

binding of RNA polymerase and the repressor protein to the duplex DNA
However, if lactose is present in the medium, then a small amount of it will be converted to allolactose by a few molecules of B-galactosidase that are present in the cell. This is shown in Figure 10-40.
HO-H2C
HO_H)C

Ho-H2c

"9/-o,
q"
Figure lO-4O

"^ry

^htrf'

-p-""r"""'id"\Y
B-caractosidase

"1- oyo-'L
Arolactose

o,

au.,o..o"

Q""

OH

Allolactose will bind to the repressor and decrease the repressor's affinity for the operator site. Thus, allolactose is the inducer of the lactose operon. once the inducer binds to the repressor, the repressor (and inducer) dissociates from the operator and RNA polymerase can begin to transcribe the lactose operon genes. The polycistronic mRNA that is transcribed (mRNA which codei for two or more polypeptide chains) will eventually be translated into the desired enzymes. This is shown in Figure 10-41.

z
o

Pi

+5efl\ /\
/ ,/ @

z
1

lnducer-Repressor complex no longer has affiniiy Ibr rhe operaror regio-n

Allolactose

RNA Polymerase can transcribe


the lactose structural genes

E:+

Pr

rnn\-t
(Polycistonic) ruonc./
p-calactosidme

-ilNA

I
n g

n U
+
n

\5)z I transcrirrion
,,** ffi Q!y'acetyrre
nT Translation ,[

ffi

f{F.h \iiii,t,

Figure I O-4 I

It turns out that transcription of the E. coli lactose operon also depends on the relative concentrations of glucose within the cell's medium. E. coli will utilize
glucose as the preferred eneigy source over lactose. Therefore, when glucose is present in the medium there is reduction in the synthesis of the enzymes needed to utilize lactose. This is called catabolite repression. However, when glucose is

absent from the medium, the rate of transcription of the enzymes needed to utilize lactose is increased dramatically.
Copyright @ by The Berkeley Review

413

The Berkeley Review Specializing in MCAT Preparation

Biology

Expression of Genetic Information

The Lactose Operon

z o
A

CAPcAMP

crp
gene

decrease in [Glucosel means an increase in [cAMP].

I i i i

@
CAP

I t

t
i

Figure 10.42
genes of the lactose operon is a gene [you do not need to know the name but if you are interested it is the crp gene) that codes for a protein called the catabolite activating protein (cAP). CAP is sometimes called the cAMP receptor protein (cRp) as well. CAp is a DNA binding protein that has the ability to mediate catabolite repression of many inducible operons, such as the lactose operon. CAp can only bind to DNA when it is complexed with cyclic adenosine monophosphate (cAMp). cAMp is synthesized from ATP and increases in concentration when glucose levels are low. when cAMP binds to cAP it forms the cAMP-cAP complex which can then bind to the DNA at the CAP site, located right next to the lactose promoter on the 5' side (upstream). This is shown in Figure 10-42.
RNA Polymerase can transcribe
the laciose structural

tl
C

A considerable distance downstream from the structural

R
-l L]
SN

c;

it

e)

fc

C, P

AMP

\
I

.,\,

genes

,<<\\\\
a
crp
gene

z o

Pr

CAP. cAMP

v
+
n

U
(r-orycrstronrc)

:\\5>z
$

Trrnscription

p-Galactosidase

ffi@@
Permease
Transacetylase

$Translation

tl(
Su

Figure lO-43
When the cAMP-CAP complex is formed it acts as an activator and binds to the CAP-cAMP site. Transcription of the lactose operon is enhanced by somehow converting a weak lactose promoter into a stronger lactose promoter. This is shown in Figure 10-43.

t'2 bir tfc gii rril


opf b"

hai

ifr
chr

Mutants A normal wild type E. colibacterium would have a genotype i+z+y+a+. One of
the mutants that Jacob and Monod characterized had the ability to synthesize the three proteins shown in Figure \0-43 at normal levels in the absence of the

dip

diP dor rep

gaL

inducer. This type of mutation was called a constitutive mutant (meaning that the genes {or those three proteins are always expressed and unregulated). The mutant cells that they examined had a regulatory gene (the i gene) that had been altered. Instead of this regulatory gene having an i+ genotype it now had an iCopyright

chr,

ind
cop

3 by The Berkeley

Review

414

The Berkeley Review Specializing in MCAT Preparation

Biology

Expression of Genetic Information

The Lactos'e Operon

genotype. In other words, this mutant E. colibacterium would have the genotype i'z+y+a+. These i-mutants can synthesize the three proteins either inthe presence or the absence of the inducer whereas the i+ cells can only synthesize the three proteins rn the presence of the inducer. Jacob and Monod were able to show that the i gene mapped close to the structural genes.

A normal i+z+y+a+ cell can make the repressor protein. If lactose is present, then the repressor protein will bind with the inducer. This repressor-inducer complex will not bind to the operator and thus RNA polymerase will be able to transcribe the structural genes. A cell which is i-z+y+a+ cannot make a functional repressor protein. Therefore, whether the inducer is present or not does not matter because there is no functional repressor to bind to the operator to prevent transcription. Thus, the normal levels of the structural genes are always expressed. Hence, the constitutive mutant.
Recall that we mentioned that bacterial cells such as E. coli have a large circular

chromosome. Besides this large circular chromosome they can also have a smaller circular chromosome called the fertility factor (or F factor). The F factor can exist in two states. It can either be free in the cytoplasm of the bacterial cell or it can be integrated into the larger circular chromosome. Since this F factor can exist in either of these two states it is referred to as an episome (a word coined for precisely this characteristic). See Figure 10-44.

S--* Host
chromosome

F factor
Integrated F factor

E. coli
Figure lO-44
Suppose we have a situation on the host chromosome in which the genotype is i+)n+u+.In this situation a normal replessor would be produced which would bind inducer. The operator would no longer be occupied and the structural genes would be transcribed. However, because there is a mutation in the lac z gene, giving th z- genotype, the enzyme b-galactosidase will not be synthesized (i.e., it will be defective). Suppose we have a situation on the F factor in which the genotype is i-z+y+a+. In this case the repressor protein is non-functional, as we have mentioned, the structural genes are continuously transcribed.

If we have both of these genotypes in the cell simultaneously (one on the host chromosome and one on the F factor), then the cell acts as if it were a partial diploid. This is indicated as i+z-/i-z+ as shown in Figure 10-45' When this partial aiplola was analyzed it rras found to be inducible. In other words, the protein bgalactosidase was being synthesized. This meant that the normal i+ allele is dominant to the recessive i- allele. If there is no inducer in the cell, then the repressor protein from the i+ gene binds to the operator regions of both chromosomes. Synthesis of p-galactosidase does not occur. However, if the inducer is present, then the repressor protein produced by the i+ gene and
Copyright
@

by The BerkeleY Review

415

The BerkeleY Review Specializing in MCAT Preparation

Biology

Expression of Qenetic Information

The Lactose Operon

inducer bind and dissociate from the operator. In this case p-galactosidase can be made. It was this type of genetic analysis that led ]acob and Monod to hypothesize the presence of a repressor protein.

i+ z

\-$
F
factor

.$\\

l- z+

Host
chromosome

Figure lO-45
Partial diploid

Mutations were also discovered in the operator region of the lac operon. These mutants were referred to as operator constitutive mutants (or Oc mutants) and they allow for the continual transcription of the lac structural genes. Why? If there is no inducer present, then the repressor would be free to bind to the operator region. However, since there is a mutation in the operator region the repressor cannot bind. Therefore, RNA polymerase can transcribe the structurai
genes.

Consider a partial diploid which has the genotype i+O%+/i+O+z+ (which we can make by using an F factor). V\4:rat happens in the absence of the inducer? In the absence of inducer the repressor protein that is synthesized cannot bind io the Oc site of the first chromosome because it has been mutated. Thus, RNA polymerase can transcribe the structural genes. However, the repressor protein can bind the O+ site of the other chromosome and thus prevent transcription. This says that the Oc mutation is dominant over the wild type O+ situation. In other words, in the absence of the inducer there will be a constitutive synthesis of the gene products in a cell which is Oc/O+. The Oc mutants are said to be cis-acting mutants while the i- mutants are said tc be trans-acting mutants. A trans-acting situation refers to a product of a gene that is diffusible. For example, in the case of the i- mutants we saw that the gene product from the i+ gene of the partial diploid i+z- /i'z+ was able to diffuse or-er to the mutant chromosome and bind to the operator in the absence of inducerThe structural genes were not synthesized. A cis-acting situation is characteristic of a gene that affects only the genes juxtaposed to it. In other words, the Oc mutation is felt only on that chromosome and not on the chromosome with the
gene. This says that for a cis-acting situation there is no diffusible product. This makes sense because the operator region of the lac operon does nst produce any mRNA or protein that can diffuse anywhere.

wild type O+

Copyright @ by The Berkeley Review

416

The Berkeley Revier Specializing in MCAT Preparatiom

Biology

Expression of Genetic Information

The Tryptophan Operon

ffi*ii

fi

6.$$ffih
are

terminators? There are two types of termination that occur tn E. coti. The first is a spontaneous termination, referred to as rho (r) independent termination; the second is a factor dependent termination, called rho dependent termination.

How can transcription be regulated at the level of termination? what

f
I

Anti-Sense

I
coo;ng Strand

DNA I =:! OUDlexl ===


l===

3' .N -N-N .N-N-N-N-N.N

s"n."
Nis

il:iP
Hairoin Loon

lr.-o,"lir,.o,o

j
(\
NC
tl

ft \

fottowed by

series of U's.

any nucleotide specific to DNA or RNA

Figure 1o,46
Rho Independent Termination.

Rho independent termination usually occurs when RNA polymerase recognizes a looped hairpin structure followed by a run of U's (Figuri ro-ee;. Note th! high GC content in the stem of the loop. When this hairpin structure forms it appeirs to jam the transcriptional process and causes the RNA polymerase to staillor a few seconds. The result is dissociation between the deoxy-A residues in the templaie strand of the DNA and the ribo,u residues in the mRNA. These two bases form a relatively weak hybrid and are therefore not too stable. referred to as the rho;factor. This enzyme is actually a helicase than can iatalyze the unwinding of RNA-RNA and RNA-DNA double helices. This enzyme recognizes an 80 to 100 nucleotide sequence on the nascent nRNA thit is upstrea;n from the termination site. Rho factor binds to this recognition site and moves along the mRNA (5'-+ 3') until it finds an RNA polymerase that has paused. It then begins to unwind the RNA-DNA duplex and terminates transcription.

Rho dependent termination occurs in the presence of a hexameric protein

type of mutant involved structural gene mutations. These types of mutants were auxotrophic for tryptophan. These mutants required trypiophan for growth as they were unable to synthesize this amino acid. yanofsky developed a map for these genes as shown in Figure 10-47. The trpE, trpD, trpC, trpB, and trpA genes coded for a polycistronic message which gave rise to the enzymes needed to convert the precursor molecule chorismate to tryptophan.
Copyright @ by The Berkeley Review

During the early 1950's Charles Yanofsky began to study the regulation of tryptophan synthesis by looking at the genes of the tryptophan operon (abbreviated as trp operon). He began by isolating two types of mutanti. one

417

The Berkeley Keview Specializing in MCAT Preparation

Biology

Expression of Genetic Information

The Tryptophan Operon

Attenuator

z
z '*

rrpR

trpPrO

rrpl ffi

trpE

trpD

trpC

trpB

trpA

0
<t

Transcription
J

n \z

u
@
Repressor

Translation

I
//
Tryptophan

Anthranilate Phosphoribosyl 3-glycerol Anthranilate phosphoribosyl anthranilate phosphate

Chorismut.
Figure lO-47

J%-%
biosynthesis of tryptophan. somehow they were altered regulate the expression of the tryptophan structural genes.
The other type of mutant Yanofsky obtained was a regulatory mutant. These mutants were able to constitutively synthesize the enzymes necessary for the

in their ability to

In other words, the frpR gene, which codes for the tryptophan repressor, is not effective in regulating the synthesis of tryptophan. The gene for frpR mapped in another quadrant of the E. coll chromosome, at about 100 map minutes, while the trp operon mapped at about 28 minutes. Yanofsky purified the dimeric frp rePressor protein and discovered that it does not function alone. In order to regulate the synthesis of tryptophan the repressor must bind the end product of that metabolic pathway. The end product is tryptophan.
Therefore, tryptophan acts as a corepressor for its own biosynthesis. When the concentrations of tryptophan are high, the repressor binds to tryptophan and this repressor-tryptophan complex binds to the operator region of the trp operon.

When this complex is bound to the operator it prevents RNA polymerase from initiating transcription of the structural genes. This is an example of feedback repression at the transcriptional level.

In contrast, if the concentration of tryptophan is low in the cell, the repressortryptophan complex wili not form and therefore will not bind to the operator. RNA polymerase is able to transcribe the structural genes and tryptophan will be
synthesized.

Around 1975 Yanofsky isolated more mutants in the trp operon and came up with some results that could not immediately be explained. Upstream from the start of the structural genes and downstream from the operator is a 15 2 nucleotide sequence of DNA called the leader sequence. This sequence is coded for by the trpL gene. Yanofsky had isolated trp deletion mutants in the leader region that resulted in a 8 to 10'fold increase in the expression of the structuragenes of the try operon. How could this be explained?
Towards the 5' end of the 162 nucleotide leader sequence is a region of 14 amino acids called the leader peptide. The 1Oth and 11th amino acids of this leade:
Copyright @ by The Berkeley Review

4ta

The Berkeley Reviex' Specializing in MCAT Preparation

Biology

Dxpression of Genetic Information

The ftyptophan Operon

peptide are tryptophan residues. These two amino acids are also part of a nucleotide sequence that is GC rich. There are four GC rich regions in the leader sequence. They are sometimes called region 1, region 2, regio"n 3, and region 4. This is shown in Figure 10-48. Becaur" th"r" GC iegions ar"e complemenlary to each other, we find that two alternative ,".orlury structures resembiing hairpins can be formed.
one_secondary structure involves base pairing between region 1 and region 2 and between region 3 and region 4. Note that ihere ur" u r"iiu, of ribo-u'i after the hairpin formed regions 3 and 4. This represents a rho-independent _by termination site. The other secondary structure involves base pairing just between region 2 and region 3. This base pairing sequence will only oc"cur if region 1 is not available for base pairing with region 2. Note that G=C base pairing is rather stable (as opposed to A=T base pairing).

we mentioned that when the concentration of tryptophan is high the repressor protein binds to tryptophan and forms the represior-tryptopha-n compl&. This complex binds to the operator and prevents trinscriptio.t. f in" concentration of tryptophan is low, then the repressor-tryptophan comprex does not form, the operator is left open, RNA polymerase binds to the promoter, and transcription of the structural genes occurs.

Leaderpeptide //

,,

U.U-U.U-3'

U-U-U-U-3'

Figure lO.4A
Let's examine transcription of the leader region in a little more detail. Suppose that the cell has a sufficient level of tryptophan. As RNA porymerase transiiibes region 1, and region 2 these two regions begin to form a hairpin structure that causes the polymerase to pause. By this time the shine-Dilgarno sequence (towards the 5' end oJ the mRNA) has been mad.e and a riboJome binds and begins translation (which we will discuss in more detail in a future lecture). As the ribosome translates the leader peptide region it disrupts the hairpin created between regions '1, and 2"

RNA polymerase then begins to transcribe region 3. Meanwhile, the ribosome reaches the tryptophan codons and inserts the required tryptophan amino acid.s
Copyright @ by The Berkeley Review

419

The Berkeley Review Specializing in MCAT preparation

Biology

Expression of Genetic Information

The Tryptophan Operon

(supplied by a charged tryptophanyl-tRNATTP molecule) into the growing polypeptide chain. As the ribosome moves downstream a few more codons it encounters a termination codon and stops translation. The ribosome dissociates and the hairpin between regions 1 and 2 reform. As RNA polymerase finishes transcription of region 4 a hairpin structure forms between region 3 and region 4. This halrpin is a termination signal because of the ribo-Us that immediately follow region 4. Once this termination signal is formed the transcription complex dissociates from the duplex DNA and the structural genes are not transcribed. Tryptophan will not be synthesized.
suppose that the levels of tryptophan in the cell are quite low. It should be obvious that the cell needs to synthesize more tryptophan. Initially the RNA polymerase and the ribosome follow the same sequence of events we just mentioned but with one important difference. When the ribosome reaches the tandem tryptophan codons it pauses because the concentrations of tryptophan in the cell ui" q"it" low. There are not enough charged tryptophanyl-tRNArrP molecules to bring tryptophan to the site of protein synthesis. This results in the ribosome sitting on region '1, and coaering it up. Meanwhile, RNA polymerase has transcribed region 2 and region 3. Since region l is not available for hairpin formation with region 2 it turns out that region 2 will form a hairpin structure with region 3. By this time RNA polymerase has transcribed region 4 and the string of ribo-U's. Region 3 will not base pair with region 4 because the hairpin structure formed beiareen region 2 and region 3 is more stable than the hairpin structure that could be formed between region 3 and region 4. This means that the termination hairpin is not formed. The transcription complex is not disturbed and RNA polymerase transcribes the structural genes of the trp opeton. This type of iegulation of the synthesis of mRNA is referred to as attenuation and the control element which is responsible for this phenomenon is called an attenuator.

With this information in mind we can return to the question we posed earlier. Why would mutants in the leader region have 8 to L0 fold levels increased tryptophan biosynthesis? The deletion mutants that were made had a deleted terminator sequence. If this occurs, tryptophan will be synthesized.
\n/hy bother with two regulatory systems (i,e., the repressor-tryptophan complex

and attenuation)? The repressor system controls the level of the tryptophan enzymes some 70 fold. Attenuation gives an 8 to 1,0 fold range of expression. The gives a 600 to 700 fold range of control. In other words, the control is tongnty "TIie operon began to explain other data that researchers could tryptophan Uroia. -f a repressor could not be found for the histidine example, oi not interpr"f. the leucine, threonine or valine operons. After operon, the phenylalanine operon, and its regulation the leader regions of operon tryptophan ine of the discovery It was discovered that all these leader s"q,tenced. *ete operons these other they controlled' which acids amino in the rich regions were The histidine operon had a leader region that contained 7 tandem histidine codons. The phenylalanine oPeron had a leader region that contained 7 phenylalanine codons. These leader regions also contained alternative hairpin ittrr.in."t which could act as termination sequences. In some cases there was multivalent repression. There were several end products from some of these pathways und *or" than one amino acid could feedback to regulate
transcription.

Copyright @ by The BerkeleY Review

420

The BerkeleY Review Speciatizing in MCAT PreParation

Biology

Expression of Genetic lnformation

Mutations & Proofteading

Mutations &

Proofreading

::

::: ,

The most common type of mutation is where there is substitution of one base pair for another base pair. A transition mutation occurs when one purine is replaced by another purine or one pyrimidine is replaced by another pyti-iai.,". A transversion mutation is simply the replacement of a pyrimidin" ty a purine or a purine by a pyrimidine. These mutations are summarized in Figure 10-49.

A-T

T-A

hydrogen atoms on each of the four bases can change their positions thus giving rise to tautomeric structures. For example, consider the pyrimidine base thlmini (in the lactam form). Under normal conditions thymine will bond with adenine as shown in Figure 10-50.
H \N rH tttttttttttttrtttO.

Tautomerism How can transition mutations occur? Watson and Crick noticed that certain

c-G
A-T
A

c-c
T-A

Transitions

<->

(,
Backbone

rtrrrrrrrrrrrrfl

N.

/q -N-l
O

\\ \
Backbone

Fr,

c C-c<-> c-c
\t
Transversions

Adenine
Figure lO-5O

Figure 1o,49

Thymine (lactam form)

However, if the hydrogen atom on the nitrogen at position 3 in the thymine ring moves to the oxygen atom at position 4, then we have a rare form of thymine (ttre

lactim form) that is capable of hydrogen bonding with guanine


Figure 10-51. The fraction of thymine in this rare form is about
10-4.

as shown

in

(z
Bac kbone

\r
Gu

.t anrne H

Thymine (lactim form)

Figure lO-51
Suppose we have a duplex of parental DNA, as shown in Figure \0-52a, in which we focus on a particulaf A-T base pair. Each strand is labeled '1, and2so we can follow them during replication. The DNA duplex goes through the first round of replication as shown in Figure 10-52b. Parental strand 1, successfully gives rise to a daughter strand (labeled 3). However, as parental strand 2 is being replicated our thymine base tautomerizes into the rare lactim form of thymine (indicated by T*). In the nascent daughter strand (labeled 4) a guanine will now be inserted

instead of an adenine.
Copyright @ by The Berkeley Review

42t

The Berkeley Review Specializing in MCAT Preparation

Biotogy

Expression of Qenetic lnformation

Mutations & Proofteading

(a)ll A-T
I lm* 12

(b),-#
A=T

During the second round of replication, as shown in Figure '1,0-52c, we will get two normal DNA duplex polymers from the duplex with strands 1 and 3. Since our rare tautomeric form of thymine does not last that long we find that during the second round of replication it returns to the normal lactam form. After the DNA duplex with strands 4 and 2 go through the second round of replication, we will get one normal DNA duplex polymer, indicaied by strands 2 and 8, and one mutant DNA duplex polymer, indicated by strands 4 and7. What we have done is gone from a normal situation in which the A-T base pairs in strands '1. and 2 have spontaneously mutated (transition mutation) to a G-C base pair as shown in strands 4 and7.

lt
13

il

G:T

42

il

Proofreading via 5'-+ 5'Exonuclease Activity fidelity of the genetic message needs to be preserved in order to proliferate the organism, there are methods of detecting mutations like the ones that we have just mentioned. Both DNA polymerase t and D N A polymerase III possess a 3' -+ 5' exonuclease activity.
Since the
Suppose we have a hypothetical template strand composed of adenine bases as shown in Figure 10-53. During our replication process we are incorporating

c,AReericatio{ A-T A-T


1563

lrl I il llll I I
tt
47

thymine bases into the growing nascent daughter strand. A mistake occurs and a rare tautomeric form of cytosine is brought in and becomes part of the nacent DNA chain (because it looks like thymine).

G=C A=T

82 tt

Normal

Mutation Normal

DNA polymerase is continually checking the segment of new DNA that it has just synthesized. If it detects an error, such as a cytosine where there should be a thymine, then that error will be removed by the 3'-> 5' exonuclease activity of the enzyme. After the correction is made the polymerase will continue with the replication of the DNA template. This proofreading mechanism reduces the eror rate by about a thousand times. Cells with high mutation rates have a defective DNA polymerase (III) exonuclease activity.
Cytosine will be removed

Figure 1o,52

with 3'to 5' exonuclease activity.


5'

-T-T-T1 T.T. {",


ilt

3'

-A-A-AJ

il il il( ilil A. A. A-

+
A -A-A5'

DNA Polymerase III

Figure f O-53

Deletion
Suppose in our hypothetical template strand composed of adenine bases one of those bases slips out of register as shown in Figure L0-54. As the nascent DNA strand is being synthesized a thymine base will not be incorporated at the poini of slippage (or tooping out) of the adenine base. The result is that there will be one less thymine base in the nascent DNA strand. in other words, there has been a deletion of a base.

Copyright @ by The Berkeley Review

422

The Berkeley Review Specializing in MCAT Preparation

Biology

Expression of Genetic Information

Mutations & proofreading

s'-T-r-T+T-T-T_3,
3'

.,

Deletion of base

- A-A- A\
of

il il il (il A-.A,il il AStippage--|

t^
./{:

A-,r5',

)*

base

\_/

DNA polymerase

III

Figure l0-la'4

Insertion
suppose, once agaia, in our hypothetical template strand of adenine bases there a slippage of a thymine on the nascent DNA strand as shown in Figure tt_!"9 10-55. This will result in an additional thymine base being added to the gro#ing DNA strand. In other words, we have an insertior oiu base. sequerices liki these, where you have a run of the same base, are hot spots for length mutations (e.g., either deletions or insertions).
Insertion of base

H,N

^,
O

\
Backbone

tshl

5'

Cytosine

---+
3'

o.u'inution

l|
DNA Polymerase

III

Figure lO-55

Deamination
Recall that the bases cytosine, adenine and guanine have amino groups on them that can be hydrolyzed. Roughly 5,000 amino groups are lost fiom ihese bases per cell per day. The example that we will consider is cytosine being deaminated to form uracil as shown in Figure 10-56.

\ r/ "_:h l- *,
O
Uracil

Backbone

Figure 10.56

to be deaminated to form uracil in the template strand of DNA, then the polymerase would put in an adenine at the corresponding position on the nascent DNA strand instead of a guanine. This is another way in which
transition mutations can arise.
Howevet, the cell has a repair system that recognizes these uracils and removes them. Consider the DN*{ duplex shown in Figure 10-57 and the cytosine that has spontaneously deaminated to form uracil. The enzyme uracil-DNA glycosidase hydrolyzes the N-glycosidic bond between the deoxyribose ring und tn" uracil base. The uracil base is removed.

uracil is an analog of thymine. It base pairs just like thymine. If a cytosine were

Copyright @ by The Berkeley Review

423

The Berkeley Review Specializing in MCAT preparation

Biology

Expression of Genetic Information

Mutations & Proofteading

.,H
--)

5'

Ucrt

Figure lO'57
This site on the DNA duplex is called an AP site (either apurinic or apyrimidinic) because it is without either purine base or a pyrimidine base. This baie defect is recognized by an enzyme called AP endonuclease which cleaves the bond on the 3' side of the phosphodiester bond of the nucleotide with the missing base. DNA polymerase I cleaves the phosphodiester bond at the 3'end on the next nucleotide unit via its 5'-+ 3' exonuclease activity. This is shown in Figure 10-5g.

-5'

CutbyAP CutbyDNA endonuclease Polymerasel


Figure lO-54
Once the defective ribose-phosphate unit is removed, DNA polymerase I will read the complementary strand and find the base that corresponds to the AP site. In this case it turns out to be a guanine. This is how DNA polymerase I knows to insert a cytosine at the AP site on the damaged DNA strand. DNA ligase will then seal the inserted cytosine into the damaged strand in the usual fashion.

Mismatch Repair
The most common types of mutations are caused by tautomerism and slippage

during replication. suppose we already have a DNA duplex that has already been replicated. During this replication we had a tautomerism in which a thymine momentarily looked like a cytosine. The result was that a guanine was incorporated into the duplex rather than an adenine. Let's also say that our proofreading system failed to catch this error. DNA polymerase might leave about one mistake per 108 replicated base pairs. Because or trur impeifect base
Copyright @ by The Berkeley Review

424

The Berkeley Review Specializing in MCAT Preparation

Biology
shown in Figure 10-59.

Expression of Genetic Information

Mutations & Proofreading

pairing (G-T) there is a slight bulge in the DNA duplex at that point. This is

Newly synthesi zed

,l a
Template DNA
Strand

DNA strand

Mismatch Repair Enzyme finds the bulge and then searches for the GATC box

CTAG_ GATCI

CH:

Figure lO-59

DNA duplex feeling for bulges like the one we just mentioned. Once this enzyme finds one of these bulges it does not know which base was incorporated into the replicating DNA in error. Is it the guanine or is it the thymine?
The mismatch repair enzyme slides along the

The enzyme needs to make a decision. It scans the duplex DNA for help. What the enzyme is searching for is the palindromic sequence of bases G-A-T-C. This is referred to as a GATC box. This box will tell the er.zyrr.e which is the newly synthesized DNA strand. How?

The template strand (the parent strand) turns out to have the adenine base methylated in each of these GATC boxes. The methyl groups are attached to adenine by SAM (S-Adenosylmethionine). This process of methylation is a signal that identifies the old DNA strand from the new DNA strand. The newly synthesized DNA strand is not methylated until a few seconds or minutes after the replication process. This gives the mismatch repair enzyme plenty of time to
locate these errors. Once the mismatchJepair enzyme finds a bulge in the DNA duplex, how far does it have to travel before it will find a GATC box? The probability is (ll4)4. The probability that a particular base at a particular position is, say, a guanine is

1/4. The probability that a guanine


(1,14\(1.14). Therefore, the probability (114)(1t4)('t 14)(1/4) or (114)a or 't 1256.

will have an adenine right next to it that there will be a sequence GATC

is

is

This means that the enzyme will need to more along the DNA duplex about 256 residues before it will randomly encounter a GATC box. (Note that this is not quite correct. What we have done is to calculate how far apart the GATC boxes are from one another on a duplex of DNA.)
We now have a *uy oi,"lllr-rg which is the correct base and which is the incorrect base. Once the incorrect base is located, the mismatch repair enzyme binds to the unmodified GATC box (with the non-methylated adenine) and to the incorrectly inserted base as shown in Figure 10-60a.

Copyright O by The Berkeley Review

425

The Berkeley Review Specializing in MCAT Preparation

Biology

Expression of Qenetic Information


(b)

Mutations & proofreading

CTAGGATCI

CH:

CH:

Figure l0.60

The mismatch repair enzyme then removes the mispaired base and the intervening sequence of DNA up to the point of the unmeihylated GATC box as shown in Figure 10-60b.

formation of a phosphodiester bond between the r"triuini.,g 3'-hydroxyf at one end,of the replaced DNA strand and the 5'-phosphate"of the previously synthesized DNA strand. This is shown in figurJfO-ei.
Template DNA
Strand

DNA polymerase (I) will then resynthesize the DNA that has been removed by the mismatch repair enzyme and will replace the mispaired base with the correct base, which in this case would be a adenine. DNA ligase will then catalyze the

DNA

Itil
DNA Ligase will lorm

--+

CTACF__.
GAT
I

phosphodiester bond

C:--'

CH:

Figure lO.6l

Mutagens Certain types of external agents can not only cause mutations but can also increase the possibility of mutations. These agents are called mutagens. Three types of mutagens that we will briefly consider are base analog, chemical, and ultraviolet mutagens.
Base Analog A base analog mutagen can easily substitute for a naturally occurring base in DNA. For example, the compound S-bromouracil is a base analog of thymine.
Copyright @ by The Berkeley Review

426

The Berkeley Review Specializing in MCAT preparation

Biology

Expression of Genetic Information

Mutations & Proofreading

This can be seen in Figure '1.0-62. Even though the bromine group of 5bromouracil is more electronegative than the methyl group of thymine, the methyl group and the bromine group have about the same rran d"r waals radius. One of the possible base pairs that results after replication from the substitution of S-bromouracil in DNA is 5-bromouracil-adenine. After a subsequent round of replication occurs, a transition mutation would result.
O

-Nt-t O

)<
' )

CHi

OBr

l-*,

"_.H
O

Backbone

>'.(

Beckbone

Thymine

5-Bromouracil

Figure 1o,62

Chemical (Nitrous Acid (HNO2)) Roughly 5,000 amino groups are lost from the bases cytosine, adenine and guanine per day. Nitrous acid can cause the deamination of cytosine to uracil as shown in Figure 10-63. Cytosine wants to base pair with guanine. However, if cytosine is deaminated to uracil, then uracil will have a tendency to base pair with adenine. The result is that a transition mutation would occur.

Cytosine
Figure 10-63

Uracil

Ultraviolet (UV) Radiation If two pyrimidine ltsidues such as thymine are adjacent to one another, then the energy stored in uv radiation (at a wavelength of about 260 nm) can link them together as shown in Figure 10-64. Thymine dimers inhibit DNA replication.

Sugar

\!r
o
O

- N ii
Thymine Dimer

Phosphate

Sugar

\>
N

Figure 10-64

Copyright @ by The Berkeley Review

427

The Berkeley Review Specializing in MCAT Preparation

Biology

Expression of Genetic Information

Mutations & proofreading

remove a segment of about 12 nucleotides as shown in Figure 10-65.


Thymine

Pyrimidine dimers can be corrected by a repair mechanism. A specific endonuclease enzyme (called uvrABC) wlil cut on both sides of the dimer and

oamageo

ow A is removed by enzyme

--TRT-

rttw ll
DNA Polymerase I fills in the
missing nucleotides and is sealed by DNA Ligase

Figure l0-65
The resulting gap is filled in by DNA ligase.

DNA polymerase I and eventually sealed by

Intercalators
ring systems which can intercalate between adjacent base pairs in the DNA molecule. These mutagens are about the size of a base pair, making them quite capable of slipping in between ad.jacent base pairs in DNA and causing frame shift mutations (e.g., either the insertion or deletion of one or more base pairs) during the replication process. If it were not for the repair systems that we have been discussing, an orginism would soon die due to the various lesions that can be introduced into DNA.
The tricyclic benzenoid hydrocarbon called anthracene, which is shown in Figure 'l'0-66, is the basic planar structure for many

Anthracene

Figure 10-66

Copyright O by The Berkeley Review

42a

The Berkeley Review Specializing in MCAT Preparation

Biol ogy

Expression of Genetic Information

The Ames Test

The biosynthetic pathway for histidine requires ten enzymatic steps. Even though histidine is an essential amino acid for humans, bacteria like E. coli and Salmonella typhimurium are quite capable of synthesizing it. In these bacteria each of these ten enzymes is encoded for by a gene and all of these genes are clustered together in an operon, which is essentially a set of functionally related genes. we can simplify the histidine pathway as shown in Figure 10-67.
Possible point

of mutation

Enz I Enz 2 Enz 3 Enz4 r----) B ----1C ----1D r---)

Enz Enz r---1 F -aG


5
6

Histidine

Enz I0 (::-

::-

Enzg

==H

EnzS U

ll

nnz z

Figure 1o,.67 If there is a mutation in any one of these genes, then these bacteria will no longer be able to make histidine. If they cannot make histidine, their growth will stop

because of the requirement of histidine in many metabolic pathways. For example, histidine is a five-carbon amino acid which can be degraded to o,ketoglutarate which can then enter into the Krebs cycle. If histidine is required for'growth, then those organisms are referred to as being His-. Such an orginism is auxotrophic (it needs something for its growth). If the organism has no problem in synthesizing histidine, it is referred to as being His+. Such an organism is prototrophic (it can make what it needs). Ames started with a gram negative bacterium that was a histidine auxotroph (His-). These mutations are very often transition point mutations. The transitibn mutation not only has the possibility of being created from a normal gene, but it also has the possibility of being reverted from a mutant form back to a normal or wild type form (e.g., Sir*). The reversion of a point mutation back to a normal base pair can occur by way of a transition mutation. A transition mutation is simply the replacement of one pyrimidine by another pyrimidine or one purine by another purine (e.g., the base pair A-T for G-C). A transversion mutation, however, involves the replacement of a pyrimidine by a purine or a purine by a pyrimidine (e.g., the base pair A-T for C-G). This is the basis of the Ames Test. It examines the efficiency of conversion of a His- cell (an auxotroph) to a His+ cell (a prototroph) that can grow on a minimal plate without histidine.

A lawn of bacteria that are His- are spread out to confluence on a petri plate that lacks histidine. The His- bacteria settle down and may go through one or two cell divisions before they s{op growing. They need histidine to grow. A test spot containing a chemical that you suspect is a mutagen is then applied in the middle of the plate. If this chemical causes mutations, then with some frequency it will revert the His- auxotrophic mutations back to His+ prototrophs. If this happens, then colonies of bacteria will emerge in the vicinity of the test spot. This can be seen in Figure 10-68. The potency of the mutagen is roughly proportional to the
Copyright @ by The Berkeley Review

429

The Berkeley Review Specializing in MCAT Preparation

Biology

Expression of Genetic tnformation

The Ames Test

size of the ring of colonies that forms around the test spot. Ames has been able to show that chemical mutagens have a high probability of also being carcinogenic.
Test Spot (suspected mutagen)

Revertant colonies

in vicinity of

$@
Plate with minimal medium (no Histidine)

mutagen
f

His'

Figure l0-64

If we

are providing so much mutagen that there might be a chance that new

mutations will be introduced into the same bacterium that has sustained a reversion event, then we might lose that bacterium if the new mutations also inactivate one of the 10 enzymes required for, say, histidine biosynthesis. This capacity is diminished by the fact that (1) mutations that are used to measure reversion frequencies are particularly easily mutable and revertible. In other words, they are hot spots (i.e., the context in which the mutation is located readily recruits mutagens). The effective dose of mutagen detected by this test is such that (2) the reversion event is one of very few mutations that would be caused by a mutagen. We don't want to place so much mutagen in your test spot such that we create lots of mutations in each bacterium. We only use a little mutagen so that we are detecting relatively small numbers of mutations in each bacterium.
Dilute region Q.:
Area of high mutagenic

His'

il
Plate

,::ttra:t .::

:o',

with minimal medium


(no Histidine)

Figure fO-69

If we were to use a high concentration of the mutagen, then that

area

immediately around the test spot would be void of any revertant colonies. This is because we are creating other mutations that prevent the revertants from

growing. As the mutagenic agent moves out and away from the point of application, its concentration decreases (i.e., it becomes more dilute) as shown in Figure 10-59. If we compare Figure 10-68 with Figure 10-69, we will find that the
concentration of the mutagenic agent in Figure 10-58 is not as concentrated as the mutagenic agent in Figure 10'69.

Copyright O by The Berkeley Review

43o

The Berkeley Review Specializing in MCAT Preparation

Biology

Dxpression of Genetic Information

"

Efigiildb

Cloning

the enzymatic processes in specific proteins that are designed to -inherent synthesize and process the DNA and RNA found in both eukaryofes and prokaryotes.

Let's consider some of the technology involved in analyzing and altering genes. Recombinant DNA technology (or genetic engineering)not"o'ty dependi In the restriction enzymes and the modificatiorl ryrL*r fou"na in bacteria but also on

Recall that we have mentioned that proteins could be cut into smaller fragments by using enzymes such as or chymotrypsin. It wasn't until the earti, tozos that this application could-trypsin be appried to nucleic acids. Restriction enzymes restriction endonucleases) recognize specific sequences in a DNA polymer 1or and then cleaves that DNA at those sites. For example, bacterial cells have a variety of restriction enzymes which can degrade the foreign DNA of an attacking virus. The bacteria's restriction enzymes do not degrade-its own DNA because its DNA h1.b::. methylated at specific locations b"y modification enzymes. when the viral DNA is injected into the bacterial cell its DNA has not been methylated yet " and so the bacteria's restriction enzymes recognize that DNA as being foreign.

Phage particles create a clear plaque

in lawn

:.ii:::.iiiii...,i

grown on a sorid surface of nutrient agar in a petri dish. This solid medium is called a plate. If you add about 108 E. colibacteria to your plate, they will eventually form a "lawn" of bacteria that will cover the surface of the nutrient agar._ if a phage infects a bacteria in this assay, then that infected bacterium will lyse and release more phage progeny which in turn will infect other bacterial cells. After several hoursLf ^rep"eated infection by the phage progeny/ a clear, circular region will form in the lawn of bacteria. ThiJ cleir region, as shown in Figure 10-70, is called a plaque. The efficiency of plating (EoP) for most bacterial viruses is about 1. The Eop i, that fraction of pnagl progeny that can form a plaque.
l,-Phage RI Plasm id

Ba.cteria can be

Petri dish with agar and a confluent lawn of E. coli

Figure lO.7O

aLawn of E.

e-{ ::,,\-_/
Lawn of E coli with RI Plasmid

coli

Figure lO-71 In 1953 it was discoveled that if you try to grow a bacterial virus on a new strain of bacteria, that virus will run into trouble. The bacteria virus called lambda (l) p-hage had been grown on a standard E. coli strain. The Eop on these plates was about 1. If you took phage from a plaque and tried to grow it on an E. coli strain carrying an RI plasmid that was known to have drug resistance genes (the "R" stands for drug resistance), then the Eop decreased to 2 x 10-3. In other words,
Copyright @ by The Berkeley Review

431

The Berkeley Review Specializing in MCAT preparation

Biology

Expression of Genetic Information


1bo,ut
71.

Genetic Engineering & Cloning

1in 500 virus particles would make a praque. This is shown in Figure l0It could be shown that most of the l-phage DNA had been degraded.

If_you take a phage from one of those 1in 500 plaques that could be formed and plate it on the same plate, then the Eop would be 1. If you plated a phage from that plaque back on the original strain of E. coli, then the Ebp would alio be 1. Frowever, if you now take one oJ these plaques and replate it on the plate with the E. coll stain carrying the RI plasmid, you will once again get an Eop of about 2x'!,0-r. This can be seen in Figure 10-72.

l.-Phage

EOp =

Lawn of E. coli

EOP=1in500
Figure 1o,72

EcoRI

5'-pG-pA-pA-pT-pT-pC-- 3' 3'--Cp-Tp-Tp-Ap-Ap-Gp--5'

Cut

Initially the phage were restricted in their growth but later became modified in order to grow on the E. coli strain with the RI plasmid. If these modified phage grow for a number of generations on the E. coli strain without the RI plasmii, they will eventually lose their modification and once again become restricted. Thus, it was thought that the RI plasmid codes for rortre gerr"s that have the ability to restrict and modify phage DNA.
Herbert Boyer (at UCSF) purified the RI plasmid restriction endonuclease. The DNA that is recognized by the Eco RI restriction endonuclease is shown in Figure 10-73. This enzyme cuts the DNA at the sites indicated by the arrows. once the phosphodiester bonds in the duplex DNA are nicked, ii is a simple task to denature the hydrogen bonds holding that portion of the duplex DNA together. Note that this particular cleavage site his twofold rotational symmetry. It is palindromic. A palindrome is a word (e.g., radar and rotator), and,
sequence of expression (e.g., nurses run), or a sentence (e.g., a man, a plan, a canal, panama) that reads the same from right to left as it does from left to right. once the hydrogen bonds break we will be left with two ends which are cohesive (i.e., "sticky end"s,'), meaning that they can join back together again given the right conditions.

o
Sticky End

Cut

-.!.r-a

pA-pA-pT-pT-pC--3'
Gp-5'

5'-pG

How often does this enzyme cut DNA? There are 6 nucleotides in the Eco RI
sequence. There are also 4 possible nucleotides at the first position in the Eco RI sequence/ 4 possible nucleotides at the second position, and so on. If we multiply these possibilities together (4x4x4x4x4x4), it turns out that (on the average) every 4,096 nucleotides (or about every 4 kb) there will be an Eco RI restriction site.

3'--Cp-Tp-Tp-Ap-Ap
Sticky End

Figure 10.73

Remember, though, that this is only in theory.

when you look for restriction sites in DNA you want to look for two-fold rotational symmetry. For example, consider the DNA sequence shown in Figure 'l'0-74. only one strand has been shown because the complementary strand is quite easy to write. However, you should still be able to find regions of two-fold
Copyright @ by The Berkeley Review

432

The Berkeley Keview Specializing in MCAT Preparation

Biology

Expression of Qenetic Information

Genetic Engineering & Cloning

rotational symmetry if presented with only one strand of DNA. In this example there are two possible regions of two-fold rotational symmetry. In Figure 10-74 we have one region of two-fold rotational symmetry that is 6 nucleotides in length. we mentioned that (on average) we would expect to find one cut made by such a restriction enzyme every 4,096 base pairs. This particular restriction enzyme is called Bgl II (Irom Bacillus globigii). Also shown in Figure '10'74 is a region of two-fold rotational symmetry that is 4 nucleotides in length. We would expect this restriction enzyme to make a cut in the DNA (on average) every 256 base pairs. This particular restriction enzyme is Hae III (from
Ha emophilus
a e

Regions of twofold rotational symmetry.

-A-A-G-A-T-C-T-G-G-C-CFigure lO-74
--t

gy p tiu s).

II and Hae III in a little more detail. The restriction sites for Bgl II are shown in Figure 10-75a. Note that if we make our cuts at the arrows/ we will have a 4-base overhang that we call the sticky end(s). These
Let's take a look at Bgl overhangs are also referred to as being S'-overhangs because of the 5' phosphate group that is still attached to that particular end. This is similar to the restriction enzyme Eco RI. Hae III does not give sticky ends when it makes its cut but rather gives blunt ends as shown in Figure l0-75b. There are also restriction endonucleases which will give 3'-overhangs when they make their cut in the DNA. An example of such an enzyme is Hae IL This is shown in Figure 10-76.In this particular example, "Pu" stands for a purine while "Py" stands for a pyrimidine.
Cut (a) Cut

5'-pA-pG-pA-pT-pC-pT- 3' 3'-Tp-Cp-Tp-Ap-Gp-Ap-5'


Bgr

rr

tl {,|

o
cur

5'-Pu-pG-pC-pG-pC-pPy-3' 3'- Pyp{R-Gp-Cp-Gp-Pup-5'

S'-pG-pA-pr nc-p[n1s, 5'-pA 3'-Tp-Cp-Tp-Ap-Gp-5'

Cur I I
3'-Pyp
Figure lo-76
3' - Cp

\Z

Haerr
(b)

5'-Pu-pG-pC-pG-pC-3'
pPy-3'
-Gp -Cp- Gp - Pup- 5'

5'-pG-pG-pC-pC-3' 3'-cn-cn7flp-Gp-5'

Cut

The modification enzyme for EcoRI (a DNA methylase enzyme) recognizes the same sequence as the restriction enzyme and transfers a methyl group from a molecule called S-Adenosylmethionine (SAM) to the C-6 position of the purine adenine to make 6-methyladenine. This process is a signal that identifies this DNA as being the DNA of the host cell. The l,-phage that survived the transfer to the E. coli plate with the RI plasmid did so because that phage's DNA was methylated before the restriction enzymes had a chance to degrade it.

5'-pG-pG-3' 5'-pC-pC-3' 3'-Cp-Cp-5' 3'-Gp-Gp-5'


Figure lO-75

Cut

Hae

III

Let's consider how to make a map of restriction sites on a duplex of DNA. In order to separate large fragments of DNA an agarose gel is used whereas small fragments are separated on an acrylamide gel. The smaller fragments (in both gels) have an easier time of getting through the gel matrix and therefore migrate faster. The larger fragments migrate slower. If ),-phage is digested with the
Copyright @ by The Berkeley Review

433

The Berkeley Review Specializing in MCAT Preparation

Biology

Expression of Genetic Information

Genetic Engineering & Cloning

fragments. These fragments can be separated on the basis of size.


DNA is digested wirh
restriction enzyme(s)

restriction endonuclease Eco RI, then you should (in theory) get about 10

Add fragments
at top
Nitrocellulose

I
7.(J ,-,. (
)
Restriction fragments are separated by gel
electrophoresis

\ Paper \ Paper towels

-+lF:l -r-T cel


Electrophoresis

r@Gr

ntltin.
Nitrocellulose
paper

solution

Figure 10.77

How would you know the order in which these fragments went together? one method to determine the order of the fragments wou'id be by south"L utottir,g. The DNA sample is digested with a particular restriction endonuclearu ur",-d separated into fragments using gel electrophoresis. The DNA is then denatured (because later you will want to hybridize to tne ONa; with alkali and transferred from the gel onto a piece of nitrocelrulose paper. paper towels can be used to absorb the buffer solution. The DNA binds to the nitrbceilulose paper and gives you a permanent record of the DNA fragments that you had on the get.inis procedure is shown in Figure 10-77.
suppose we were to cut our DNA sample with Eco RI and Hind III. what you would like to know are the restriction sites for Eco RI and for Hind III. Fiow would the Hind III fragments relate to the Eco RI fragments? The chances are the Hind III fragments and the Eco RI fragments wilioverlap. A possibility is
Cut

shown in Figure 10-78.


Eco RI Fragment

Eco RI Fragment

DNA
Hind

Itr

Fragment

Figure lO.7A

Eco RI restriction enzyme and a gel for the Hind III restriction enzyme. we follow the procedure outlined in Figure 10-76 for making a nitrocelluloie record 9f th" fragments from our two restriction enzymes. we ian pick a fragment cut by the Hind III enzyme and radioactively label it and use that ridioactive sequence as a probe. If we apply this probe to the nitrocellulose paper with the fragments cut by Eco RI, then it will hybridize to those sequences oinNA ir-, the
Copyright @ by The Berkeley Review

each restriction enzyme in separate testtubes and each set of restriction fragments (in each testtube) is separated by gel electrophoresis. we will have a gellor the

How do we analyze this possibility? The DNA to be analyzed is cleaved with

434

The Berkeley Review Specializing in MCAT preparation

Biology

Expression of Genetic Information

Genetic Engineering & Cloning

Eco RI fragments which are complementary. If the labeled Hind III fragment overlaps two Eco RI fragments (as shown in Figure 10-78), then we should see two dark bands on our developed x-ray film indicating which Eco RI fragments contain the complementary sequences to the Hind III fragment. Repeating this procedure with other fragments will eventually lead to a map of your DNA sample.

have been used successfully for sequencing DNA are the Maxam-Gilbert method, involving specific chemical cleavages, and the sanger dideoxy method, involving a controlled interruption of DNA replication. Since the Maxam-Gilbert method is not used that much anymore, we will turn our attention to the Sanger dideoxy method.

FIow can the sequence of a piece of DNA be determined? Two techniques which

Sanger Dideoxy Sequencing The sanger dideoxy technique makes use of the fact that DNA polymerase I can be inhibited by a 2',3'-dideox;mucleoside triphosphate (ddNTp) analog of one of the four bases A, T, G, or C. The example shown in Figure 10-29 is ddATp.

-o-P

-o -P -o -P -O -CHr ttt _O _O _O 4'

TBO
Figure 1o,79

3',2'

1. Select the DNA strand to be sequenced. Attach a short radioactively labeled section of primer to the DNA strand being analyzed. The label of choice is 35S instead of 32P begruse 35S has a longer friU tUe than 32p, Also, 35S has such a weak emission that it doesn't require shielding. The 35s atom replaces the oxygen atom that is double bonded to the o-phosphate of dNTP.
To each of four solutions add DNA polymerase I, the four deoxynucleotides triphosphates (dNTP)-which are dATP, dTTP, dGTP, and dCTP-and one of the ddNTP's (ddATP, ddTTP, ddGTP, or ddCTP)in a controlled ratio.

2. DNA polymerase I will take the dNTPs and add them to the primer. The 2',3'-dideoxy analog can be incorporated into the growing DNA chain by reaction with its 5' triphosphate function. However, when the next deoxynucleotide triphosphate is use{ in the reaction sequence it cannot be added to the 2',3rdideoxy analog because the analog does not contain a 3'-OH function to form a phosphodiester bond. Termination of the DNA chain occurs specifically where the nucleotide analog was incorporated. This yields DNA fragments of different length containing the analog at the 3' end.

Copyright @ by The Berkeley Review

435

The Berkeley Review Specializing in MCAT Preparation

Biology

Expression of Genetic Information

Genetic Engineering & Cloning

DNA backbone

TTAGACCCGATAA

CCCGCA GCGT
Labeled 5t
Primer

DNA backbone

TTAGACCCGATAAGb.iG6i
ATTCGGGCGT
+

H
H H

ATCTGGG

9T4TTCGGGCGT

++TgTggggT4TTCGGGCGr

Figure lO-aO
fragments.shown_in Figure 10-g0 are then separated by size on a . polyacrylamide gel by electrophoresis. The pattern of fiuj*e'ts from the autoradiogram shown in Figure 10-g1 yieldi tt u pNa ,"irr"r,." that was synthesized using the radioactively rabeled complementary DN,i primer strand__ not the original DNA strand you wished to anaiyze. since this "primer,,strand is complementary to the DNA sequence of the original strand, there should be no

3. The labeled

problem in obtaining the base sequence of that or:iginal strand.

ddATP ddTTP I
L

ddCTP AT AT TA CG TA GC CG TA AT TA TA CC

a.

!)_ qa

Ebo q
AF

II

II I

II II

'9

I
I

I-r

r
-

oc) boc

436

E -I -l

: : cc cc
TA

DNA sequence of
complementarv strand './

,/

Figure lO-81
Copyright @ by The Berkeley Review

The Berkeley Review Specializing in MCAT preparation

Biology
Cloning

Expression of Genetic Information

Genetic Engineering & Cloning

Recall that we mentioned that there were restriction enzymes that cut DNA at specific sequences. Those sequences have a unique characteristic in that they have twofold rotational symmetry. Some restriction enzymes leave a 5' overhang while others leave a 3' overhang. There are also restriction enzymes that leave

blunt ends.

will allow you to make a map of restriction sites and restriction fragments in your DNA. bne method that we mentioned was Southern blotting. If you wanted to determine the sequence of the DNA fragment, then you could use the Sanger dideoxy technique (or the MaxamGilbert technique).
There are several methods that
VVhen the DNA that you are going to analyze is labeled, you want to make sure that it is labeled on the end of your fragment of DNA. why? one reason involves sensitivity. In the case of the Maxam-Gilbert technique, labeled 32P at the end of the DNA fragment might give you about 1,09 radioactive counts per minute. Another reason to use end labeling involves the simplicity of the pattern that you see on your gel.

can use recombinant DNA technology to create new combinations of genes which can then be cloned. For example, a piece of foreign DNA cut with a specific restriction enzyme, such as Eco RI, can be inserted into a DNA vector which has also been cut with Eco RI. Replication of this recombinant DNA molecule takes place in the host cell. A suitable vector for E. coli would either be l,-phage or a plasmid.

with the use of restriction enzymes, one

It turns out that there are a few problems involved in the cloning procedure. one problem concerns the copy number. In the case of the first plasmid used, there were only three per E. coli chromosome. Another problem is the size of the plasmid. The plasmid that was first used in a cloning experiment was about 100 kb, which is a lot of DNA to have to work with.
There is also a problem with reclosure of the DNA. For example, if you cut your DNA with Eco RI, then that same piece of DNA can close right back up again without ever taking in the foreign piece of DNA that you want to clone. One way to get around this problem is to add an excess of the foreign DNA so that you get more collisions between"the foreign DNA that you want to insert and the plasmid.

Another way to get around this reclosure problem is to use phosphatase on the plasmid DNA. If you want to join a nick between two nucleotide residues in a segment of double stranded DNA, then the enzyme DNA ligase is used. The energy source for this procedure comes from either ATP or NADo, depending on the organism in use. (In bacterial cells the energy source is NADo while in animal cells the energy source is ATP. NAD is an abbreviation for nicotinamide adenine dinucleotide.) Figure 10-82 we see a seginent of double stranded DNA with a nick. Note the iree 3' hydroxyl and the free 5' phosphate at the nick site. DNA ligase will eventually form a phosphodiester bond at that position in order to seal the nick.
Ii-r

The 5'phosphate on the DNA duplex is absolutely required in order for DNA -rgase to work. If we were to treat the DNA segment shown in Figure 10-82 with
Copyright @ by The Berkeley Review

457

The Berkeley Keview Specializing in MCAT Preparation

Biology

Expression of Genetic Information


the nick with DNA ligase.

Genetic Dngineering & Cloning

phosphatase and remove that 5' phosphate residue, we would not be able to close

3'_A_T_G_T_5, *Holll
5

ll

ll V,rC
OHP

A-

ll

3'

-T-A..
Figure lO-82

3'

5'

Suppose we have 5'phosphates hanging out at the ends of our cleaved DNA plasmid. If we remove those 5' phosphates with phosphatase, then we will not be able to join these ends of the plasmid. This is shown in Figure 10-83.

Phosphatase

3'

OH

A restriction enzyme
cuts the duplex DNA

The ends will not rejoin

Figure lo-a3 If a foreign piece of DNA that has 5' phosphates at two of its ends is now inserted into our plasmid (from Figure 10-83), then only two strands of the DNA can be joined. The other two strands of DNA will be left with nicks as shown in Figure 10-84. However, there will be some integrity to this plasmid.

3'HO
J

OH

5'P

P5'

Foreign DNA to be inserted

DNA without 5'


phosphates

DNA with two nicks

Figure rO-84
Suppose we have cut our plasmid with the restriction endonuclease Eco RI and we would like to insert a piece of foreign DNA that has also been cut with Eco
Copyright @ by The Berkeley Review

434

The Berkeley Review Specializing in MCAT Preparation

Biology

Expression of Genetic lnformation

Genetic Dngineering & Cloning

RI. One way this could be done is shown in Figure 10-85. Note the arrow above the sequence 5'-TAC-3' in the fragment of foreign DNA. Let this represent a direction for the insertion of the foreign piece of DNA. This direction is "in phase" with the direction of the arrow for the plasmid. The "8" in our example simply represents any base.

5'
1

B_G

A_A-T -T
+

lil B-C-T-T_A-A

{ -B lil G+

DNA from plasmid cur with EcoRI

5'A-A-T -T-C-T -A-C-T -G-G-C


3' G

lllllt _A-T -G-A-C.C-C-T


----)

cur with EcoRI

-T -A-A

Annealing fragments with

DNA ligase

r
5 B
J

-C-A-A-T-T-C-T -A-C-T -c-c-CFA-A-T -T -C -B lil B -C-T -T -A*TG -A-T-G-A-C-C-C-T -T -A-TG -B

,)

llr

ill

lll

Figure lO-85
Our foreign piece of DNA could also be inserted into our plasmid in the opposite direction as shown in Figure 10-86. The two arrows are "out of phase." This is an important aspect of cloning because if you are inserting a gene that you want to clone into a plasmid, then in order for that gene to be properly expressed it must go into the plasmid in the right orientation. Why? Because if the foreign DNA goes into the plasmid in the wrong orientation, then when RNA polymerase transcribes it, the protein that will eventually be produced (if it is produced at all) most likely will be non-functional.

5'

3',

-G-A_A_T -T -C -C-C -A_G_T _A-C -A_A-T_T{-B ilt il ilt B _C _T -T-A*A -G .G.G-T -C-A-T -C_T -T -A-A.C +
B

ilt

Figure f O-Ao
Let's consider the plasmid pBR322, which has a high copy number of about 30 per chromosome. This plasmid has an ampicillin resistance gene (Amp) and a tetracycline resistance"gene (Tetr). Both of these genes have promoters and the direction of transcription in each case is indicated by the arrows. Within the tetracycline gene is a restriction site for the restriction endonuclease Bam HI. This
is shown in Figure 10-87.

Copyright O by The Berkeley Review

439

The Berkeley Review Specializing in MCAT Preparation

Biology

Expression of Genetic Information


Promoter

Genetic Dngineering & Cloning

Tet'

pBR322
Promoter

A*p'

Figure 10.87

If we open the plasmid with the restriction endonuclease Bam HI, then we can insert a foreign DNA fragment in two possible orientations. If this fragment contains a gene without a promoter, then it is important as to which orientation the foreign DNA is inserted. If the gene is inserted in the direction given by the arrow for the gene, then the tetracycline promoter will not work with that gene. This is shown in Figure L0-88a. However, if the gene is oriented in the same direction as transcription, then the tetracycline promoter will work with the
gene. This is shown in Figure 10-88b.

Td'
Promotel

DNA

Foreign

DNA

FR322

FRr2
Atttpt

A*p'

(a)

(b)

Figure f O'aa

How would you know which way the foreign piece of DNA has inserted itself into the plasmid? Suppose we have an Eco RI site in our plasmid as shown in Figure L0-89. Also suppose we have an Ssp site in our piece of foreign DNA. If our piece of foreign DNA inserts as shown in Figure L0-89a, and we cut at the Eco RI and Ssp sites, then we will get two fragments--one large and one oI
moderate length.
Copyright @ by The Berkeley Review

440

The Berkeley Review Specializing in MCAT Preparation

Biology

Expression of Genetic Information

Genetic Engineering & Cloning

FRe2

frR322

A*p'

Amp'

Figure lO.ag
Flowever, if our piece of foreign DNA inserts in the opposite direction as shown in Figure 't'0-89b, and we cut it the Eco RI and the ssp sites, then we *iiig"i ; large fragment and a smalr fragment. If you take your ilor,", and do a restriction digest, you will see different pattems. you can just save one sample of each fw9 until you are sure which one you want.

A plasmid carrying genes_for drug resistance is usefur because if you were to insert a piece of foreign DNA into one of those genes that codes for a drug resistance, you would lose the activity of that particrilar gene. If a piece of foreigf; DNA has inserted into, say, the tetracycline resistan." gIr," of pBk322,then you would get a host cell that has tetracyciine sensitivity. If"the piel of roreign dla did not properly insert into the tetricycline gene, then the host cell would still be resistant to tetracycline. This would. t"ll yo.tlhut your insertion reaction failed.
To remove the problem of orientation of the insertion fragment, two restriction endonucleases were used that gave different cleaved ui-rdr. tro, exampre, in pBR322 there is an Eco RI site and a Hind III site not too far away.If you cut the pBR322 plasmid with these two enzymes and purify the large fragment, you will have two diff"l"", truis..1"aqs. if you have a piece oT foreign DNA that is -Now, cut with Eco RI and Hind IlI, then this insertion fiagment will insert into the pBR322 plasmid in only one orientation.

5'-A-C-G-C-G_T_3' 3'-T-G-C-G-C.A-5'

Mla

restriction enz).rne that gives that type of cut.

The pB_R322 plasmid also has a pvu rr (proteus aulgaris) site which gives blunt ends. These blunt ends can anneal with the blunt Jnds produced b| any other

5'-G-C-G-C-G-C-3' 3'-C-G-C-G-C-G-5'

has been inserted (because you lose the restiiction site).

You could also join together two half restriction sites that have the same cohesive ends but different recognitions. Two examples are Mla I and Bss HII. The sequences that these endonucleases recognize are shown in Figure 10-90. This is good for insertion of the ioreign pNa but you cannot remove that DNA once it

Bss

HII

Figure lO-9O

T"t"_11" certain types of plasmids that have been engineered at UC Davis calred the puC plasmids. These plasmids are about 2 kb ind have a very high copy number, about 90 per chromosome. These plasmids have a gene for a.ipicitiii-,
Copyright @ by The Berkeley Review

44t

The Berkeley Review Specializing in MCAT preparation

Biology

Expression of Genetic Information

Genetic Engineering & Ctoning

resistance and in one small region, called a restriction sites.

polytinker, there are about

17

Another conunon vector is lambda (1,) phage. This phage infects the bacterium E. coli and injects its linear DNA into the host. The viral DNA soon becomes circular and is quickly presented with two choices--to become part of the host (the lysogenic pathway) or to produce more progeny and destroy the host in the process (the lytic pathway). It turns out that the genome of this phage is about 4g kb, and not all of it is essential for the phage's life cycle. What this means is that portions of the phage's genome can be removed and foreign DNA inserted. once the bacterium has been infected, the fragments of foreign DNA can be amplified many fold, thus giving rise to a genomic library. This library is then analyzed to find the gene of interest. A particular gene of interest can be located by using a radioactively labeled probe.

insulin. within a given plasmid is a region of inserted DNA that is complementary (called cDNA) to the mRNA for pancreatic proinsulin. In order to make this cDNA from the mRNA, the enzyme reverse transcriptase was utilized. In order to ensure maximum transcription of the cDNA, it is inserted
into the plasmid in the correct reading frame and next to a strong promoter.

One useful application of cloning is the production of proinsulin, a precursor to

1970s the complete sequence was determined for the filamentous phage M13. This 6.4kb phage has nine known genes. The area between genes vII and IX is capable of coding for a small peptide. Mutants were made for this region. A mismatched oligonucleotide was made in which a C residue was made to be opposite a T residue. This is shown in Figure 10-91. when this region is transcribed there will be a 5'-uAG-3' stop codon in the middle of the gene. This inactivates the gene (which is necessary for growth of the phage), thus causing a mutation. However, the mutation rate was only about lo/o because within the E. coli bacterium is a mismatch repair enz)rrne that is constantly checking the DNA and correcting errors in the base pairing.

In the late

3'-T-A-c-T-c-T -A-T-T-C-T-T-T-C-G-C-C _5'

5'-C
3'-G-A-U-5'
Figure lO-91

at

DNA

mRNA

Copyright @ by The Berkeley Review

442

The Berkeley Review Specializing in MCAT Preparation

Expression of Genetic Information


15 Passages

100 Questions

Passage Tifles

I. Southern Blotting II. Restriction Endo*nucleases ilI. Ribozymes and HIV IV. The Genetic Cocle Chain Reacrion (pCR) _y. Polymerase VI. Lactose Operon VII. Enhancers^andGeneExpression VI1f. Replication Block C;li cycte ""J;fi; IX. Northern and Southern Blottins X. Restriction Enzvmes ---o XI. Arabinose Opeion _XII. Gene Therapy Strategies Tryprophan'Operon' {.!U, Xry. Translation XV. Cancerous Gene

Questions

1-5

24-29 30-36 31 -43 44-50


51 -58 59-65 66-72 73-19 80-87 88-94 95 - 100

t8-23

6-11 12-11

speci altzing in MCAT Preparation

Suggestions
The passages that follow are designed to get you to think in a conceptual manner about the processes of molecular biology at the organismal level. If you already have a solid foundation in molecular biology, many of the questions you read here will seem to be very straight forward and easy to answer. But if you are new to the subject or if you have not had a pleasant experience with molecular biology in the past, some of them might appear to come from the void that spreads out beyond the Oort field at the edges of our solar system.

Pick a few passage topics at random. For these initial few passages, do not worry about the time. Just focus on what is expected of you. First, read the passage. Second, look at any diagrams, charts, or graphs in it. Third, read each question and the accompanying answers carefully. Fourth, answer the questions the best you can. Check the solutions and see how you did. \Alhether you got the answers right or wrong, it is important to read the explanations and see if you understand (and agree with) what is being explained. Keep a record of your results.

After you feel comfortable with the format of those initial few passages, pick another block of
passages and try to do them in one sitting. Be aware that time is going to become important. On average, you have about 1 minute and 15 seconds to complete a question. Be creative in how you approach this

next group. If you feel comfortable with the outline presented above, fine. If not, then try different approaches to a passage. For example, you might feel well versed enough to read the questions first and then try to answer some of them, without ever having read the passage. Maybe you can answer some of the questions by just looking at the diagrams, charts, or graphs that are presented in a particular passage. Remember, there are many effective learning styles. You need to begin to develop a format that works best for you. Keep a record ofyour results.
The last block of passages might contain at least a few topics that are unfamiliar even to those who know a good deal about molecular biology. Find a place where the level of distraction is at a minimum. Get out your watch and time yourself on these passages, either individually or as a group. It is important to have a feel for time, and an awareness of how much is passing as you try to answer each question. Never let a question get you flustered. If you cannot figure out what the answer is from information given to you in the passage, or from your own knowledge base, dump it and move on to the next question. As you do this, make a note of that pesky question and come back to it when you have more time. When you are finished, check your answers and make sure you understand the solutions. Be inquisitive. If you do not know the answer to something, look it up. The solution tends to stay with you longer that way. (For example, what ls the Oort field, anyway?)

The estiniated score conversions for 100 questions are shown below. At best, these are rough approximations and should be used only to give one a feel for which ballpark they are sitting in.

Section X Estimated Score Conversions


Scaled Score Raw Score

>13

rt-12
9- 10 1-8 5-6
<4

- 100 10-19 60-69 50-59 40-49 0-39


80

Biology
Passage

Southern Blotting
3.

Passage I

I (Questions l-5)

this technique are:

fragment may be isolated with the Southern transfer technique, also known as Southern btotting. The basics of

From a mixture of DNA sequences, a particular

Gel electrophoresis is_ commonly used to separate molecules and molecular fragments. What does the separation of DNA into fragmints depend on when a Southern blot is done?

l)

DNA is fragmented into lengths that are appropriate for.the.agarose gel. This miy be accomptisteO
Convenrionally, abort 100,000 base pairs (bp) is the upper limit on size for agarose gel ellctrophoresis of DNA.
sonication, mechanical shearing, or enzymatic action.
Uy

A. B. C. D.

Charge Size

Radioactivity Linking number

2)

gel.

The DNA on rhe gel is denatured with NaOH. A sheet of nitrocellulose paper is placed over the gel, 99ver9d with paper towels, and pressed down. T:his blots liquid and any single_stranded DNA from the

4.

If you were probing for

the

DNA fragment:

5'CGATTACCCG3, which of the following probes would you use?

3)

heat, permanently fixing the adhered DNA.

The nitrocellulose paper is dried under vacuum with

4) A DNA

probe, labeled either radioactively or enzymatically, that is complementary to the sequence of interest is applied to the nitrocelluiose paper.

A. B. C. D.

5'GCTAATGGGC3,

3'CGGGTAATCGs' 3'GCTAATGGGCs' 5'CGATTACCCG3'

5) Heat is applied over several hours to


only unbound probe.

allow renaturation. Then the paper is washed to remove

5.

6)

film or by quantifying

The paper is read either by autoradiography on x_ray the bnzymatic riaciion.

certain inherited diseases. for example.

blotting is particularly useful for identifying ., Southern the presence of a specific_ DNA fragment ln a largE mixture of fragments. It can be used to iientify carriers of

statements is TRUE?

D are rhe. offspring. Which of tn" following


Parents
Offspring

gel electrophoresis done totest for a DNA fragment cortaining a region common to people who have HC, assume that A and B are the pu."ntr, and C and

Huntington's chorea (HC) is a fatal, degenerative brain disease that is transmitted in an autosomal dominant fashion. In the results shown below for a

1.

Which types of enzymes are commonly fragment DNA for Southern blotting?

used

(EII
0)

AB''CDf

(-

((I) (rD ((rD (-

A. B. C. D. 2.

Resiriction enzymes
Proteases

fiIV
A.

F" rrr

rD (D (-

Nucleotidesynthases

(r(-

DNA

gyrases

When DNA is denarured by NaOH in Srep structural change takes place?

2, what

B.
C.

A. B. C. D.

The phosphates of DNA become positively


charged.

D.

The children show identical patterns of


inheritance.

If Fragment II is the HC fragment, then both children are affected. B^oth parents carry the HC Fragment, V. If Fragment III is the HC fralment. then both children are affected.

join to form a quad structure. The base-pairing of DNA is altered. DNA is separated into single strands.
strands

DNA

Copyright @ by The Berkeley Review

44s

The Berkeley Review Specializing in MCAT preparation

Biology
Passage

Kestriction Endonucleases

Passage tr

II

(Questions 6-11)

When viral DNA enters a bacterial cell, a defense


mechanism in the bacterial cell degrades the DNA of the

Restriction endonucleases can cut DNA into fragments of varying length, which can then be separated from one another by gel electrophoresis and identified using autoradiography. Larger fragments are located near the top of the gel, while smaller fragments are located near the bottom.

virus. The activity of the viral DNA is restricted from operating within the bacterial cell by the action of
bacterial enzymes called
re st

ric tion endonucle as e s. These

restriction enzymes recognize a particular sequence of bases in double-stranded DNA and then cleave the DNA
at specific sites within those sequences.

If

a single restriction endonuclease is used to cut

Recognition sequences are written in the 5' -+ 3' direction and, by convention, only one DNA strand is given. A vertical arrow that represents the cleavage site is placed within the written DNA sequence to indicate where the restriction endonuclease cuts the DNA. For

sample of DNA, the procedure is called a single digest. A double digest involves the use of two different restriction endonucleases. DNA fragments are described in terms of length. One kilobase (kb) is equal to 1000 bases (or base pairs). A restriction map can be established by comparing the lengths of the DNA fragments from different digests.

example, the DNA sequence AJ AGCTT


abbreviation for:
Cut
+

is

an

6.

Which of the following restriction endonucleases


would produce 5' sticky ends?

5'-AAGCTT-3' 3'-TTCGAA.5'
Cut

L il. III.

PstI SalI TaqI

The restriction endonuclease Hindlll recognizes this sequence and cleaves the DNA at the cut sites. Once the two phosphodiester bonds holding the DNA backbone together are cleaved, the hydrogen bonds between complementary bases break apart. If the cut sites are staggered, then complementary sticky ends are produced. If the cut sites are not staggered, then blunt ends are produced. Most restriction endonucleases recognize DNA sequences that are palindromic (the sequence is the same

A. I only B. II only C. III only D. II and III only 7.


The plasmid pBR322 is a circular duplex of DNA that can replicate autonomously in the bacterium Escherichia col/. This plasmid contains 4.3 kb of DNA. In a random sequence of DNA, how many cuts would be expected, if the restriction enzyme
A/u I is used?

if

read forwards

or

backwards). Examples

of

some

restriction endonucleases and their recognition sequences are shown in Table 1.

A.8 B. ll

Recognition

Sequence

Recognition
:Enzyme
Sequence

Enzyme
Sac

D.

c.

t4
t',l

AGJCT

Alul

GAGCTJC

GJGATCC BamHI AJGATCT BgtIl

GJTcGAC
CCCJGGG

I Sal I
Smal
TaqI

8. A polylinker is a segment of DNA that contains


several restriction endonuclease sites. Based on the information in Table l, how many unique restriction endonucleases can cut the following segment of duplex DNA?
5'-CGGATCCCGGGTCGACG-3' 3'.GCCTAGGGCCCAGCTGC-5'

GJAATTC
CTGCAJG

EcoRI
PsT

TJCGA
CCCJGGG

Xmal

Table 1. Restriction Endonucleases

Bacterial DNA il protected from the cell's own restriction enzymes by the action of modification
endonucleases. These enzymes catalyze the methylation of either an adenine or a cytosine base within a particular

A.2 B.3 c.4 D.5

recognition sequence. Copyright by The Berkeley Review

446

The Berkeley Review Specializing in MCAT Preparation

Biology
9.

Restriction Endonucleases
11.

Passage II

estab.lish hvdrogen bonds at the sticky ends formed by which of the tbllou ine restriction endonucleases,l

A segmenr of bacrenal D\{ is cut by the restriction BamHI Tuo complementary sticky ends are formed. These sricky ends are uUt. to
endonuclease

by agarose gel electrophoresis and compared to a known standard (S), as ihown below:
analyzed-

are used to digest a length of bacterial DNA. The fragments from these single and double digests are

Various combinations of restriction endonucleases

A.
B.
C.

HindlII BglIl
Psr I Sac I

J
7kb 6kb

A8l

II

6co

Rt

psr

Eco
Bgt

Rl pst I U Bgl lt

Eco Rl psr I

D.

II
I I I I II I

r
I I I I II III

10.

experiments, and their-fragments

Two linear DNA molecules are digested with the same restriction enzyme (,EcoRIJ in different

5kb 4kb
3kb

size using agarose gel electrophoresis (see below;:

ui" ,"pu.u,"O

Uy

DNA
Lane
1

DNA
Lane 2

r
II

I I
C

2kh
A

r r
I I

lkb

I I

Based on the DNA fragment patterns in the gel shown above, which of the following ,estriction maps BEST represents the length of bacierial DNA? A.
Psr

Y
E

Bgl

B.

Il

Cco Rl

Psr

DNA

two DNA molecules seeri to be similar. Ttie only d_ifference is that Fragment D in Lane I does not show up at the same position in Lane 2. Instead, two new fragments, X and y, have appeared. The presence of Fragments X and y is due tb the:

Basedo_n the separation of fragments in the gel, the

pst

Pst I

Eco

Rl

Bgl

lI

DNA

c.
Bsl
Pst I

Il
Pst

A. B. C. D.

Eco RI

addition of DNA that contains


to Fragment D.

at

Eco

Rl

site

DNA

deletion of DNA from Fragment D. addition of DNA to Fragment D.

D.
Eco Rl
Pst

site.

deletion of DNA from"Fragment D, resulting in the subsequent formation of a new Eco RI

Bgl

tl

Pst I

DNA

lopyright @ by The Berkeley Review

447

The Berkeley Review Specializing in MCAT preparation

Biology
Passage

Ribozymes and IIIV


13. HIV is

Passage III

III

(Questions 12-17)

Although it is well known that enzymes are catalysts (molecules having the ability to increase reaction rates), it is less well known that RNA molecules can have catalytic activity as well. The term ribozyme refers to an RNA

notorious for mutating in response to treatment. For example, giving a particular experimental drug led to a two-week decline in the number of HIV particles, followed by a dramatic mutation resulting in a95Vo new population of HIV particles. How could the ribozyme approach BEST
fight the mutation problem?

University of Colorado and Sidney Altman of Yale University won the 1989 Nobel Prize in chemistry for their work that indicated RNA was more than a passive carrier of genetic information. RNA can act to catalyze
itself or other molecules of RNA.

molecule with catalytic activity. Thomas Cech

of

the

A. B. C. D.

Increase the concentration of the ribozyme by


a more efficient transfection process

Follow one ribozyme with another in two


weeks

Ribozymes were probably the first catalytic agents


formed during the evolution of life. Later, RNAs began to

Create a specific ribozyme for the strain of

code for protein synthesis, and proteins are superior


enzymes. Proteins became the dominant form of catalysts.

HIV the patient has Link multiple ribozymes to attack HIV mRNA
at several locations at once

Cech's work in 1982 indicated that some of the RNA found in Tetrahymena is autocatalytic. It has the ability to splice out introns and edit its own mRNA sequence. One year later, Altman discovered some RNA molecules that could cleave other molecules as well.

14. Why do Hampel

and Wong-Staal target a highly

Ribozymes are currently being studied to combat human immunodeficiency virus (HIV). Researchers Arnold Hampel and Flossie Wong-Staal are modifying a
tobacco virus ribozyme to recognize and cleave a highly

conserved region of the HIV mRNA that is common among many different strains of HIV?

A. B. C.

The highly conserved regions are known for

conserved region common among

HIV strains.

their wide range of variability, and


approach targets more mutants.

this

test-tube experiments. In culture, this modified ribozyme reduced HIV transcripts by 10,000-fold.
Cech and Bruce Sullenger are focusing on packaging the ribozyme gene so it co-locates with the virus into the virus packaging center of the cell. This means that the ribozyme against HIV will be present inside the "HIV factory" and will destroy copies of the viral mRNA. One clinical approach is to remove T cells from the patient, infect them with a ribozyme that cleaves HIV mRNA, and return it to the patieot. Although this scheme will not be tested ln vivo for several years, these are the big plans some researchers are exploring right now.

in

specially designed ribozyme can chop up the HIV mRNA

Usually, highly conserved regions code for


well-designed functions critical for survival,
and this approach targets more mutants.

D. The highly conserved regions attract


ribozymes
mRNA.

The HIV particle resists attack by ribozymes except at these highly conserved sites.

for more efficient

cleavage of

15.

12. If the right ribozymes attacked


structures could be damaged?

What is the immediate effect (prior to mutarional changes) of decreasing with a ribozyme the number of available mRNAs for HIV in a T cell?

an animal cell, which

A. B. C. D.

I. II. ilI. A. B. C. D.
Copyright

The number of virus particles does not change. because each mRNA becomes more efficient
at translation.

Ribosome

Transcript copy of DNA Amino acid carrier for protein translation

:
I only

I and II only

II and III only I, II, and III

The number of virus particles decreases, due to fewer copies of the transcript. The number of virus particles increases, due to more copies of the transcript. The number of virus particles does not change. due to competing HIV mRNAs.

by The Berkeley Review

444

The Berkeley Keview Specializing in MCAT Preparation

C"

Biology
16. In
whv?

Riborymes and HIV

Passage III

the course of evolutionary history, RNA_based catalysts were replaced by protein-bised catalysts.

IL

protein-based catalysts. Fnzymes outcompeted ribozymes and chopped

I wider variety of subunits provides greater flexibility in the structure and function of

III.
A.
B.

them up. Enzymes are more advantageous, because they change the equilibria of reactions.

I only
I and Ii only II and III only
I, II, and III

c.
D.

17.

Which of the following statements would be TRUE about catalysts?

I. il. III.

Catalysts remain unchanged after completing a reaction. Catalysts increase the rate of a reaction. The body does not synthesize catalysts.

A. I only B. I and II only C. II and III only D. I,II. and III

Copyright @ by The Berkeley Review

449

The Berkeley Review Specializing in MCAT preparation

Biotogy
Passage

The Genetic Code


18-23)

Passage IV

IV (Questions

18.
experiment, Marshall

The difference in the number of possible kinds of


amino acids between a single-base code and a threebase code is:

In a pioneering biological

precipitate was washed, and the number of l4C counts per minute (cpm) was measured.

synthesizing system. To begin, E. coli bacteria were broken by agitation and subsequently centrifuged. The resulting supernatant contained DNA, tRNA, mRNA, ribosomes, enzymes, and other cytoplasmic components. Protein synthesis could be obtained by the addition of GTP, ATP, and amino acids. The mixture was incubated at physiological temperature for some time, after which trichloroacetic acid was added. The protein containing

Nirenberg showed how to produce polyphenylalanine from the addition of polyuridylate to a cell-free protein

A.
B. C. D.

4
16

60 64

19.

The following base sequence is given:

ATCGGTATA
If the genetic code is overlapping,
a mutation in the base cytosine (C) may alter how many amino acids?

t23456789

An essential component of this system is the addition of deoxyribonuclease (DNAse). The mRNA present when
DNAse is added is short-lived, and protein synthesis soon ceases. But synthesis resumes with the introduction of additional mRNA, making possible a protein-synthesizing system that is entirely cell-free.

The synthetic polyribonucleotide that is added (polyuridylate) is created using polynucleotide


phosphorylase, an enzyme that catalyzes the following reaction:

A. I 8.2 c.3 D.4

(RNA)n + Ribonucleoside diphosphsls = (RNA)1.,.1 + Pi This enzyme does not use a template, and the relative composition of ribonucleotides present determines the RNA to be synthesized. A polynucleotide consisting of two different bases is created by using two types of

20.

The following base sequence is given:

ACTGTTACATTG

1 2 3 4 5 6 7 8 9 l0ll

12

ribonucleoside diphosphates in the presence of polynucleotide phosphorylase. Table I shows the amino
acid incorporation created by a random copolymer of l67o uracil (radiolabeled) and 24Vo guanine.

Insertion of the base thymine (T) between the 7rh and 8th base may aft-ect how many amino acids?

A.0 B. I c.2 D.3

AminoAcid
Phenylalanine Valine

Relative amount incorporated

r00

21.

To obtain measurable results in this experiment,


which of the following cellular components MUST
be used?

3t
36

Leucine Tryptophan Glycine

t4 t2

A.
B.
C.

Radiolabeled mRNA Radiolabeled amino acids


Radiolabeled ATP Radiolabled ribosomes

Tatrle l. Amino acid incorporation resulting from a random copolymer of U* (76Vo) and G (247o). (Not& *Radiolabeled)

D.

Copyright @ by The Berkeley Review

450

The Berkeley Review Specializing in MCAT Preparation

C.

Biology
22. Which of the following
experiment?

The Genetic Code


graphs BEST represents
the

Passage IV

protein synthesis as measured throughout


A.
B.

E*I'
Time
C.
a)

a*l / lgl /

;EI \
Time D.
o
tr

*=\ lEl \

!o o.N Oo Zq

V
Time

!6 ao

b.s

Y4

\
Time

23.

Based on the information in Table l, the triplet that codes for glycine MOST likely contains:

A. B. C. D.

more guanine than uracil. more uracil than guanine. equal amounts of guanine and uracil. all guanine and no uracil.

Copyright @ by The Berkeley Review

45t

The Berkeley Review Specializing in MCAT preparation

Biotogy
Passage V (Questions 24-29\

Polymerase Chain Reaction (PCR)

Passage V

The polymerase chain reaction is a process that amplifies DNA within a certain region of a chromosome. The first step (Figure 1) is to denature the targeted DNA by heating it. Next, an excess of two DNA primers, about 15 to 20 oligonucleotides in length, is added to the denatured DNA. For proper hybridization, the two oligonucleotide primers must be complementary to the opposite strands of DNA.

A DNA polymerase called Taq, from the thermophilic bacterium Thermus aquaticus, is added to the mixture. The polymerase extends the primers and completes the first cycle of DNA amplification. The result is two new DNA double helices where there was once just one.
Each cycle, which takes approximately five minutes,
doubles the number of DNA molecules present. The cycle

is usually

repeated 20

to 40 more times. The DNA

synthesized in subsequent cycles extends to just the ends

of the primers.

dsDNA
Targeted

rr sequence \,2
. Denature

One application of PCR involves the screening of particular steps in cloning. For example, one must screen
transformants after the step of inserting a DNA fragment

cYcle

I ll

.enneut primers

into a plasmid. The reason is to check to see whether the transformants contain the inserted fragment and to verify that the clone is suitable for further work. In this screening process, transformant colonies are picked and placed into tubes used for PCR. The cells are lysed, and primers (indicated by I and 2 in Figure 2) are used that flank the site into which the fragment is to be cloned. The product of the PCR amplification is run on a gel.

. *,-., [l \7
tMttlttltt +

extension Cloning site


Inserted

DNA

Cvcle

'

[l . Denature 2 I I l.AnnealDnmers (7
I

rrrrIrrrrrr

i r r r r r r r r r I I I I I I ll=!]lt I I | | I I
t r rr

O O
Products of PCR

27

rr r rrll tffit

rrrrlrrr rr rr rrr
Figure 2

T'1

tl .Primerextension ll
\.7 tttrtttt +
ttttlttttttltlffi
@

24.

The MOST likely reason that the DNA polymerase fromThermus aquaticus is used preferentially over the polymerase from E. coli is that the:

A. B.
rttrlttt

polymerase from Thermus aquaticus has a


much faster rate of polymerization. heat used to denature the DNA would destroy the polymerase from E. coli. polymerase from Thermus aquaticus is known to be much easier to isolate.

rrlri,tttttttttt + a-*

C. D.

polymerase from cell-free systems.

coli is not functional in

Figure

Copyright @ by The Berkeley Review

452

The Berkeley Review Specializing in MCAT Preparation

a*

Biology
25.

Pol5rmerase ehain Reaction (pCR)

Passage V

A brief heat treatment is used to separate the double_ stranded DNA into single_strandedDNA. The result of this heat treatment should result in:

28.

After seven cycles of the pCR (as depicted in Figure would expect to see how many strands of }-gi" DNA?

A. B. C. D.

increased absorbance of light at effect krown as hyperchroiism. increased absorbance of light at effect known as hyperanneaTing. decreased absorbance of lighl at effect known as hypochromiim. decreased absorbance of light at effect known as hypoannealing.

260 nm, 260 nm,

an an

260 nm, an 260 nm,


an

A. B. c. D.

64 128 256

612

29,
26.

The primers used in the polymerase chain reaction:

result in the product of the:

Separation of the pCR products in Figure 2 by gel filtration chromatography would Ue U6Sf tit"ry to
suitable clone emerging first, based on its size.

4. B. C. D.

A.
B.
C.

amplified.

surrounding the region

are complementary to each other. contain the same nucleotide sequence. should contain uracil. are designed using portions of the sequence

of DNA to

be

charge. slze.

suitable clone emerging first, based on its

unsuitable clone emerging first, based on its

D.

charge.

unsuitable clone emerging first, based on its

7.

duplex DNA:

Shown in the diagram below are four different DNA melting curves for four different molecules of

al

\o

o
-o
c!

!6 o

&
Temperature

C)

(.C)

rv

an abundance of

Which of the four curves would be expected to have

A-T

base pairs?

A.
B.
C.

n
IV

I II

D.

ryright @ by The Berkeley Review

The Berkeley Review Speciatizing in MCAT preparation

Biology
Passage

Lactose Operon

Passage VI

VI (Questions 30-36)

Lactose is synthesized in the mammary gland by lactose synthase, an enzyme composed of two subunits, one with catalytic activity (galactosyl transferase) and one without catalytic activity (a-lactalbumin). In human

Wild-type (normal) structural genes of the lac operon are written as Z+Y+A+. This designation indicates that normal levels of the three proteins coded for by these genes are synthesized. A mutation in any one of these genes (e.g., Z-Y+ A+) could lead to an inactive form of the
corresponding protein.

infants, this milk sugar is hydrolyzed by lactase, an enzyme located in the epithelial cells of the small intestine. In some adult humans, lactase synthesis is

inhibited, leading to a condition known as lactose


intolerance.

Independent mutations can also occur in the transacting (capable of activity on another chromosome) regulatory gene and in the cis-acting (capable of activity only on the same chromosome) operator gene. Regulatory gene mutations are designated as I- and are called constitutive mutants. This type of mutation leads to an increase in the synthesis of the lac operon proteins in the
absence of an inducer. In contrast, normal inducible bacteria synthesize the proteins of the lac operon only in the presence of lactose. A constitutive mutation in the
operator gene is designated as Oc.

especially

coli car also utilize lactose, glucose, the preferred carbon source, is not available. Metabolism of lactose requires galactoside permease, a cotransporter of protons and lactose across the bacterial plasma membrane, and p-galactosidase, an
The bacterium E.

if

enzyme that catalyzes the hydrolysis


glucose and galactose.

of lactose into

The enzymes required for the catabolism of lactose in E. coli are derived from genes organized into an operon. The lactose (lac) operon contains three structural genes designated as lac Z (B-galactosidase), lac Y (galactoside
permease), and lac A (thiogalactoside transacetylase). The

30.

control region of the lac operon is adjacent to the lac Z

Lactose intolerance can lead to all of the following EXCEPT:

gene and contains

a promoter (P) that binds RNA

polymerase, and an operator (O) that binds a repressor protein transcribed from the (regulatory) Iac I gene. A genetic map of these sites is shown in Figure 1:

A. diarrhea. B. CO2 production. C. epithelial cell D.


intestinal lumen.

uptake

of water from

the

intestinalbacterial fermentation.

DNA

mRNA

c c

\c$c

PlOllacZllacYllacA

CRP-cAMP v site

Protein

otla
Figure I

+s+
.

---\-r'--\-

31.

Which of the following transport systems is MOST likely to be associated with galactoside permease?

If

lactose is absent from the medium, the lac repressor

binds to the operator, and RNA polymerase is unable to transcribe the structural genes. When lactose is present in the medium, it enters the cell and is converted to the inducer, allolactose. The inducer binds to the repressor and reduces the repressor's affinity for the operator. RNA polymerase can now initiate transcription. When glucose is in the medium, cAMP concentrations in the cell are low and transcription of the lac operon is reduced by 50-fold. F*owever, in the absence of glucose the cAMP levels in the cell are high, allowing cAMP to bind to the cAMP receptor protein (CRP). The CRPcAMP complex then binds upstream from the lac promoter and increases the rate of transcription of the lac structural genes by RNA polymerase. Copyright @ by The Berkeley Review

A. B. C. D.

Active Uniport
Symport

Antiport

32. The process of synthesizing mRNA from DN-\


occurs with the use of a:

A. B. C. D.

RNA-directed DNA-directed RNA-directed DNA-directed

DNA DNA RNA RNA

polymerase. polymerase. polymerase. polymerase.

454

The Berkeley Revien Specializing in MCAT Preparatio'm

Biotogy
33.
initiated?

Lactose Operon
36.

Passage VI

Which of the followin_g arrangements is necessary !9f9re transcription of the strictural genes can be
Lactose must bind to the repressor before RNA polymerase can bind to the operator. before the inducer can bind to the promoter. and the inducer must bind to the repressor.

A. B. C. D.

crossing is shown below:

recipient bacteria to produce partial diploid cells called m.erozygotes. The genotype of one such

Wild-type donor bacteria can be crossed with mutant

DNA polymerase must bind to the repressor

ocz+y-Al-O+Z-y+A+
Based on this genotype, all of the following genes code for enzymes that are produced constiiuiively Lac Z Lac Y Lac A

RNA polymerase must bind to the promoter, inducer-repressor complex must bind the
Allolactose must bind to the repressor, and the

EXCEPT:

promoter.

I. il. III.

34. A

observed after the transfer is made?

culture of E. coli is transferred from a growth medium containing lactose to a growth medium conraining glucose. Which of the f6llowing will be The repressor
levels

A.

and B-galactosidase will be synthesized.

will increase, CAp will become active,

will

become inactive, cAMp

A. I only B. II only C. III onty D. II and III only

B.

synthesized.

The repressor will become active. cAMp levels will decrease. CAp will become inactive, and p-galactosidase will not be

C.

D.

synthesized.

The repressor will become active, cAMp levels will increase, CAp will become active, -' and B-galactosidase will not be synthesized. The repressor will become inactive, cAMp levels will decrease, CAp will become inactive, and p-galactosidase will not be

35.

presence and in the absence ofan inducer?

The following haploid gnome is found jn E. coli: Lac I-O+Z+y+A+. which of the following enzyme levels BEST represents this genome, boih in- the

A.

lacZ lacy low low lacZ lacy high low lacZ lacY high low

lac A low lac A


high

B.

C.

lacA
low

D.

lacZ lacY lac A high high high


Copyright @ by The Berkeley Review

455

The Berkeley Review Specializing in MCAT preparation

Biology
Passage

Enhancers and Gene Expression

Passage VII

VII

(Questions 37-43)

The effbct of each mutation on transcription is tested


(Figure 3):

Experiments have shown DNA sequences near the

RNA start site are required for gene transcription. These sequences are known as promoters and usually contain
the consensus sequence TATA. Interestingly, sequences further upstream known as enhancer elements arre required to give the same level of transcription as a
normal gene.

The activity of each enhancer and promoter can generally be focused on a segment of DNA 100 to 200 nucleotide pairs long. The function of both regulatory elements requires binding of specific gene-regulatory proteins, which are often tissue-specific. In other words, most regulatory elements function best in the particular cell types that express the gene with which they are
normally associated. One example is the p-globin gene in
chickens.

@o& 0
''T'"liii;T'"
mutant enhancer

t hour incubation
(3'7 "C)

Production of CAT activity is measured

To identify gene-regulatory proteins that bind to the pglobin enhancer, one must first determine the exact nucleotide sequence necessary for function (Figure l):
t

Figure

o o
-o
bo

z
3

Results of the mutant enhancer activity (Figure 3) are then correlated with data showing the location of protein binding sites for proteins in the normal enhancer. These binding sites have been determined by gel-mobility shift

assays and

DNA footprinting. The results of


Regions in enhancer DNA covered by proteins

the

4
5

comelation are shown in Figure 4:

6
7 *a

24
cd

25

26 27

l--k--

E
108 Nucleotide pairs

100

----*-+l

o
L

o{n

JV

Figure I

l!o
sequence

tr

A series of mutated versions of the enhancer

10
Figure 4

15

20

25

can be joined (Figure 2) to a reporter gene whose product is easily measured. Jhis reporter gene is a bacterial enzyme called chloramphenicol acetyl transferase (CAT):
B-globin promoter mid

Mutant Number

These and other studies have led to the following general conclusions regarding enhancers and promoter
elements:

l. Each regulatory element consists of a specific


nucleotide sequence, which binds a corresponding series of gene-regulatory proteins. Some of these proteins are found only in specific tissues, while some are found in all cell types.
2.

Poly-A addition
signal

u,

B-globin enhancer (mutant)


-).

Some gene-regulatory proteins activate transcription when they bind, while others inhibit transcription.

Figure 2 Copyright
@

Enhancer and promoter elements seem many of the same proteins.

to bind

to

by The Berkeley Review

4s6

The Berkeley Review Specializing in MCAT Preparation

Biology

Enhancers and Qene Expression


41. A newly

Passage VII

37, The chicken p-globin enhancer lies downstream


from the p-globin transcription unit.
nuclease-hypersensitive site only in erythrocytes and no other cells. The MOST likely explanation for this statement is:

It

forms

discovered regulatory element from a vertebrate gene is analyzed, and it is found that many of the proteins that bind to the region have been previously found regulating other genes. This finding suggests:

A. the enhancer B. C.
erythrocytes.

sequence

is found only in

A. B. C.

gene-regulatory proteins found only in


erythrocytes bind to the enhancer, displacing
nucleosome complexes.

on the combination of regulatory proteins


bound.

promoters and enhancers affect transcription in similar manners. the net effect of a regulatory element depends

red blood cells are nuclease-hypersensitive,


because loss of their nucleus is inevitable.

D. in other cell types, gene regulatory

proteins bind only to promoters and not to enhancers.

D. promoters and enhancers often


synergistically in promoting transcription.

eukaryotes.

of gene-regulatory proteins control transcription in higher


act

a relatively small number

38.

proportion of which of the following amino acids?

Histone octamers serve as a protein core around which double-stranded DNA can loop twice. The histone proteins are MOST likely to have a high

42. Which of the following molecular complexes

MOST likely to bind to a gene-regulatory element?


Steroid hormone Steroidhormone-receptorcomplex Protein hormone Proteinhormone-receptorcomplex

is

A. B. C. D.

Valine and serine Glutamine and cysteine Lysine and arginine Histidine and isoleucine

A. B. C. D.

43. According to Figure 4, which


39. According to the protocol in the passage, how many nucleotide pairs are changed in each enhancer
mutant?

enhancer's stimulation of transcription?

proteins make the largest contribution to the

gene-regulatory

A.2 8.4 c. 21 D. 108

A. B. C. D.

AP2-like, NFl-like. CACCC factor. Eryfl, CACCC factor, NF1-like. API-like, AP2-tike, Eryfl. AP2-like, CACCC factor, Eryft.

40. Figure 3 illustrates the testing of mutant


transcription is measured:

enhancers

for their effects on transcription. In this experiment,

A. directly by quantifyirlg levels of RNA. B. directly by recording rates of nucleotide C. D.


incorporation into RNA.
protein.

indirectly by moniroring levels of B-globin

indirectly by monitoring CAT enzyme activity.

Copyright @ by The Berkeley Review

457

The Berkeley Review Specializing in MCAT preparation

Biology
Passage

Rereplication Block and the Cell Cycle


44-50)

Passage VIII

VIII (Questions

44.

middle of a eukaryotic S phase, some parts of a chromosome will not yet have started replication, while other regions will be completed. This presents an enormous problem during the middle and late stages of
the S phase. Some replication origins and their respective

In the normal S phase of a cell cycle, the genome must be replicated exactly once and no more. However, in the

When an S phase cell is fused with a Gl phase cell, DNA synthesis is induced in the Gl phase nucleus. This information suggesrs rhar MOST likely the transition from:

A.

S to

Gl is mediated by a diffusible acrivaror of


mediated by

B. S to Gl is

DNA synthesis.

a nondiffusible

DNA

sequences already used and duplicated are

presumably identical to other replication origins not yet used. Since each replication origin may be used only once in each S phase, there must exist some form of recordkeeping and regulation.
lnitiator proteins

C. Gl to S phase is mediared by a diffusible D. Gl to S phase is mediated by a nondiffusible


activator of DNA synthesis. activator of DNA synthesis.

activator of DNA synthesis.

45. The S phase of a eukaryotic cell


DNA
described as:

can BEST

be

A.

synchronous, with one origin of replication.

Ji

B.
C.

synchronous,
replication.

with multiple origins of with multiple origins of

urry
_l

asynchronous, with one origin of replication.

D.

asynchronous,
replication.

\.7

Iniriaror protein (inactivated)

Fragments
G2 G2

of bacterial DNA that replicate when

affected by the rereplication block. This


information: A.

injected into a fertilized frog egg can be shown to be

indicates the mechanism providing for


indicates the mechanism providing

rereplication block requires a highly specific origin of replication.


B.

for

c.
DNA can leplicate once again

rereplication block does not require a highly specific origin of replication. provides evidence for the activator-removal
model.

D.

provides evidence against the activatorremoval model.

Figure

Cell fusion experiments have provided an important clue to the answer to this problem. When an S phase cell is fused with a Gl phase cell, DNA synthesis is induced in the Gl phase nucleus. In contrast, when the S phase

47. It can be inferred

from information in the

passage

that a G2 nucleus fused with a S phase nucleus is:

cell is fused with a G2 phase cell (a cell that

has

A. B. C. D.

proteins, which are inactivated by the passage of a replication fork. This model, known as the activatorremoval model, is illustrated in Figure
1.

synthesis. One model which could explain this rereplication block is b'hsed on tightly bound initiator

completed the S phase), the G2 nucleus is not stimulated to replicate DNA, while the S phase cell continues DNA

stimulated to begin DNA replication by some nondiffusible molecule bound to its DNA. stimulated to begin DNA replication by some diffusible molecule found in the nucleoplasm.

prevented from DNA replication

by

some

nondiffusible molecule bound to its DNA.

prevented from DNA replication by some diffusible molecule found in the nucleoplasm.

Copyright @ by The Berkeley Review

4sa

The Berkeley Review Specializing in MCAT Preparation

Biology

Rereplication Block and the eell Cycle

Passage VIII

48. According to the activator-removal model, the


addition of new initiator molecules occurs during:

A. B. C. D.
49. In

GI.
S.

G2. mitosis.

several types of secretory cells found in fly all the homologous chromosome copiei remain side by side, creating a giant polytene chromosome. In the salivary glands of Drosophila larvae, each of the four chromosomes has been replicated through ten cycles without separation of the daughter chromosomes. How many identical

larvae,

strands of chromatin are lined up side by side?

c. D.
50.

A. B.

128 256

5t2
1024

The initiator proteins used in the activator removal


model:

A.
B. C. D.

are translated in the nucleoplasm

and

transported into the cytosol. are translated in the cytosol and diffuse across the nuclear membrane.

contain a significant hydrophobic region of


amino acids. contain a nuclear import signal.

opyright @ by The Berkeley

Biology
Passage

Northern and Southern Blotting


53.

Passage IX

IX (Questions 51-58)

Based on the information in the passage, detergents


are MOST likely used to inactivate:

To determine the nature of the defect in mutant mice that produce abnormally low amounts of the protein albumin, the following protocol can be followed: First, one collects tissue samples from defective and normal mice. Using strong detergents, the cells are disrupted to inactivate cellular molecules. After removing proteins by phenol extraction, one can precipitate the nucleic acids with alcohol. To separate the DNA from the RNA further,
one can make use of their different solubilities in alcohol.

A. B. C. D.

proteases. nucleases. lipases.

cellulases.

54. The RNA harvested from the normal mouse cells is MOST likely used to:

A technique known as Northern blotting can be used to analyze the albumin-encoding RNAs. Intact RNA molecules from defective and normal liver cells are
fractionated by gel electrophoresis. A replica of the gel is blotted onto a sheet of nitrocellulose paper. Then, radiolabeled DNA probes are incubated along with the paper so that the RNA encoding albumin hybridizes with the correct DNA probe. The sheet is then washed

A. B. C.

establish the size


molecules. molecules present.

test the equipment. establish an experimental control.

of the mutant

RNA

D. establish the amount of mutant RNA


Washing the nitrocellulose paper after incubation with the probes removes all:

thoroughly afterward, and the hybridized probe is detected by autoradiography. Using this technique, one

55.

can gather information about the size of the albumin RNA

in both the defective and normal mice.

By an analogous method, DNA instead of RNA can be


examined. This technique is known as Southern blotting. DNA fragments are separated according to their size by

A. B. C. D.
56.

radiolabelbeforeautoradiography. radiolabelafterautoradiography. background radiolabel before autoradiography. background radiolabel afterautoradiography.

gel electrophoresis, and complementary probes

are

identified by blotting and hybridization. By repeating this procedure with several different restriction endonucleases, a restriction map of a particular portion of the genome can
be constructed.

Shown below is a Northern blot indicating both


defective and normal RNAs encoding for albumin. Band A corresponds to the wild-type RNA, while band B corresponds to the defective RNA.

51.

Band A

The protein albumin is MOST likely to be found in:


Band B

A.
B. C.

D.

the cytosol-of white blood cells. cerebral spinal fluid. blood plasma. the cytosol ofred blood cells.

These results indicate that the:

A. defective protein is smaller, and B.


52.
The cells that produce albumin can be found in the:

A. B. C. D.

spleen.

gall bladder. pancreas.

C. D.

liver.

the differences can be explained by a mutation in the defective cell's RNA. defective protein is larger, and the differences can be explained by a mutation in the defective cell's DNA. wild-type protein is smaller, and the differences can be explained by a mutation in the defective cell's RNA. wild-type protein is larger, and the differences can be explained by a mutation in the defective cell's DNA.

Copyright @ by The Berkeley Review

460^

The Berkeley Review Specializing in MCAT Preparation

Biology
57. In the following

Northern and Southern Blotting


experiment, two different restriction

Passage IX

endonucleases, ResEnd A and ResEnd B, are used to create a restriction map of a lOkb DNA molecule:

58.

10

kB DNA Molecule

the mutant gene is known. DNA probes can be constructed for both the normal and defective genes and used in prenatal diagnosis. A child's mother is homozygous dominant, while the child's father is heterozygous for the sickle-cell defective gene. What is the chance that both normal and defJctive DNA probes will react with the child's genome?

In sickle-cell anemia, the exact nucleotide change in

A. B. C. D.

25Vo 50Vo 75Vo


IOOVo

Which of the following conclusions is consistent with the data? [Note: The numbers indicated the length of the DNA in kb, while the capital letters
indicate the restriction endonuclease cut sites.l

A.

rcAB
ffi
BA

B.

C.

ffi
D.

. AB
7

21 BA

ffi21

Copyright @ by The Berkeley Review

Biology
Passage X (Questions 59-65)

Restriction EnzSrmes
60.

Passage X

Which of the following starements is TRUE of DNA conservation? A.


B.

About 99Vo of human DNA is identical between any

two individuals. The other lVo contains regions of


hypervariability that differ dramatically from person to person. By examining these differences through DNA fragmentation, it is possible to produce a unique DNA
identification for each individual.
One of the most important biochemical tools for DNA manipulation, Typ" II restriction enzymes are often used in producing fragments for analysis of DNA, to generate a DNA "fingerprint." These enzymes recognize and cleave double-stranded DNA at specific sequences of base pairs, usually 4-8 bases long and palindromic (reading the same forwards and backwards).

DNA is highly conserved among


beings. beings.

human

DNA is not highly conserved among human DNA is completely conserved among human
beings.

C.

D. DNA differs dramatically from


person.

person to

61.

By treating DNA with a series of restriction enzymes, a researcher produces precisely defined DNA fragments.
Since genetic differences between individuals create or

Although bacteria are the source of restriction enzymes, their own DNA is protected against cleavage by specific methylations. What is the probable purpose of the bacterial restriction
enzymes?

eliminate restrictions sites, each person has a unique pattern of fragments. Differences also exist between corresponding segments of homologous chromosomes in the same person. The fragments from chromosomes treated with restriction enzymes are of different lengths
and exhibit "restriction fragment length polymorphisms" (RFLPS). The fragments can be separated according to
size by gel electrophoresis.

A. B. C. D.

To activate foreign DNA To inactivate foreign DNA To incorporate foreign DNA into the bacterial
genome

To promote cell lysis in response to foreign DNA

To screen for an inherited disease, probes can be used to check the fragmented DNA for the warning marker, indicated by a unique, predicted cleavage pattern. Screening of fetal tissue for sickle-cell anemia and other inherited diseases can now be carried out. In the case of
sickle-cell anemia, the substitution of valine for glutamate at one site in the beta chain leads to the elimination of a restriction site. This changes the pattern of fragments and can be used as an identifier of inheritance of mutated
genes.

62. This diagram of gel electrophoresis results indicates DNA fragments corresponding to the gene for the beta chain of hemoglobin. The DNA fragments were produced by the restriction enzyme MstII. The parents are A and B, and their children are C and D. Hb A refers to normal adult hemoglobin, and Hb S

refers to mutated hemoglobin that causes the

sickling ofred blood cells.

ABCD

In

some crimiaal investigations, small samples of Hbs HbA

tissue taken from the crime scene can be amplified with the polymerase chain reaction (PCR), fragmented with restriction enzymes, and compared with samples taken

--I

from suspects.

--I
Figure I

59.

Assuming DNA has an equal proportion of all bases, how many fragments would a restriction enzyme that recognized 4 base pairs produce from a DNA fragment that is.10,000 base pairs long?

Which of the following statemenrs are FALSE, based on the fragmentation pattern depicted in the
gel shown above?

c. D.

A. B.

625
63

390
39

A. B. C. D.
462

Child C does not have sickle-cell anemia.


Both parents are heterozygotes.

Child D does have sickle-cell anemia.


The parents have a 50/50 chance of having child with sickle-cell anemia.
a

Copyright @ by The Berkeley Review

The Berkeley Review Specializing in MCAT Preparation

Biotogy
63. In the gel clepicted
parents?

Kestriction

EnzSrmes

Passage X

in Figure l, why do the children have only one band, instead of two like their
Each of the children carries only one copy of

I. II. III.
A.
B.
C.

The two fragments mfurated to the same


location on the cell.

the gene for the beta chain of hemoglobin. The children are homozygotes.

I only

Ii

I and II only
and

III only

D.

I, II, and III

64. How can fetal tissue be collected in utero for analysis ofrisk
of inherited genetic diseases,/

A. Collect fetal cells in amniotic fluid by amniocentesis. B. Collect fetal cells from the maternal blood C. D.
supply.

Perform a therapeutic abortion. Perlorm a Cesarean section.

65.

For which of the following projects would resrriction enzymes be used advantageously?

L I! III.

Isolating and sequencing DNA Solving a paternity dispite Genetic counseling foipotential parents

A. I only B. I and II only C. Il and Ill only D. I, II, and III

)opyright @ by The Berkeley Review

463

The Berkeley Review Speciatizing in MCAT freparation

Biology
Passage

Arabinose Operon
68.

Passage Xl

XI (Questions 66-72)

The arabinose operon is found in bacteria and is


responsible for the bacteria's ability to use arabinose as a source of energy. The operon (Figure 1) consists of a regulator gene, three control sites, and three structural genes. Production of the C protein (from the araC gene) is under control of the araOq operator. Transcription of the araC gene occurs when the level of C protein and cAMPCAP complex is low. CAP is the catabolite gene-activator protein found in many inducible operon systems. If the level of cAMP-CAP complex is low while the level of C protein is high, a C protein binds to the araOl site and inhibits transcription of the araC gene.
The transcription of the structural genes B, A, and D is dependent on the formation of a DNA loop. If cAMP levels are low and C protein levels are high, a C protein

The following reacrion is carried out by rhe product of the araA gene:
CHO
I

H- C- OH
I

dU-L-n
I

.H

tc=o HO-C- H
I I

CH,OH

HO- CI

HO_CI

CH,OH

cHroH

The enzyme that catalyzes this reaction is:

A. B. C. D.

an epimerase. an isomerase. a dehydrogenase. a kinase.

sites by forming a hairpin turn within the DNA. AraI is adjacenr to the promoter for the structural genes, and a bound C protein blocks transcription of those genes. In the situation where both arabinose and cAMP are abundant, the hairpin loop is not formed. The reason is that the C protein with bound arabinose has an altered conformation and binds to both araOl and aral, but not to araO2. In addition, the cAMPCAP complex binds between the araOl and aral sites. The
result of these changes is a polymerase that is now able to

binds to both the araO2 and ara

69. Which of the following conceprs is NOT illustrated by the arabinose operon?

A. A B. C. D.

transcription

bind to the araBAD promoter site and promote of these genes. The products of these
structural genes promotes the uptake of arabinose and

protein can alter its own production by altering the transcription of its gene. Structural genes are regulated only by DNA sites found in contact with those genes. Changes in transcription brought about by a signal molecule are reversible. Proteins can act as inhibitors or activators of transcription, depending on the presence of a

signal molecule.

70. High levels of cellular cAMP are associated with:

converts it to a usable form ofenergy.

Li-_
Regulator
gene

araC I arao2 | araOl

I aral I araB I araA I araD


Structural
genes

Control
sites

l-L___f__j

A. B. C. D.

high levels of arabinose. low levels of arabinose. high levels of glucose. low levels of glucose.

71. The polymerase responsible for the transcription of the structural genes in the arabinose operon is most likely:

66.

The synthesis of the C protein is BEST described as:

A. B. C. D.
67.

unregulated. deregulated. regulated by arabinose. autoregulated.

A. B. C. D.
72.

RNA RNA RNA RNA

polymerase. polymerase L polymerase II. polymerase IIL

An organism lacking the araC gene would exhibit:

It has been discovered that the genes responsible for the uptake of arabinose are not in the operon shown in Figure 1. This may have selective value, because:

A. B. C. D.

increased use of glucose as a source of cellular energy.


a lack of cAIvF production. inefficient transcription of structural genes ara B, A, and D. increased use of arabinose as a source of

A. B. C. D.
46,4

there is an increase in the amount of arabinose transported into the cell. there is a preferential use of glucose over arabinose as a cellular energy source. the design leads to a more efficient metabolism

cellular energy.

of arabinose. a mutation in one area would not affect the integrity of the other region.

Copyright @ by The Berkeley Review

The Berkeley Review Specializing in MCAT Preparation

Biology
Passage

Gene Therapy Strategies

Passage XII

XII

(Questions 73-79)

74.

A plasmid is transfected into

of blocking the A. a mutant Xl B. a mutant Xl C. D.


promoter. a wild-type promoter.

a cell line

expression

Our modern understanding of the molecular nature of genetic expression has paved the way for potential breakthroughs in the field of gene therapy. It is now possible for us, at least in vitro, to prevent the expression

chromosomal gene called Xl. In order for this to be accomplished, the plasmid would have to contain:

of a

with the aim

wild-type

disease-causing genes. Additionally, the same techniques can prevent the expression of viral genes, thereby preventing cellular production of new viruses.
Two major approaches have been at the forefront of gene therapy research in recent years.

of

gene sequence with

a strong

gene sequence with

weak

The first takes advantage of the mRNA molecule's

Xl gene sequence that is inverted with respect to its promoter. a wild-type Xl gene sequence that is separated from its promoter.

ability to form double-stranded RNA-DNA hybrids.


Artificial DNA can be made that is complementary to the "sense" mRNA of a disease-causing or viral gene. This complementary sequence is termed antisense DNA. The antisense and sense strands can hybridize to each other, preventing translation of the disease-causing or viral protein product. DNA, rather than RNA, is used to make antisense strands, because it is inherently stabler. Additionally, to protect it from enzymatic degradation, the DNA's phosphate backbone is replaced with a
synthetic compound called methylphos phonate.

75. There is much debate over the therapeutic potential of ribozymal versus antisense gene therapy. Which of the following would represent advantages of the
ribozymal approach?

I. II. III. A. B. C. D.

Base-paired RNA is less easily degraded than synthetic single-stranded RNA. Ribozymes act catalytically. Individual ribozymes are very broad in their substrate specificity.

The second approach relies on the ability of certain RNA molecules to act enzymatically and cleave mRNAs
at specific sequences. These RNA enzymes are referred to as ribozymes. Ribozymes, like IRNA molecules and

ribosomes, have a tertiary structure resulting from basepairing of part of the RNA strand to itself. Artificial ribozymes can be designed to cleave HIV viral RNA at specific sites, preventing it from being translated and
thereby slowing the reproduction of the virus.

I only II only I and II only I and III only

76. Which of the following would NOT affect the Ku of a ribozyme in vitro?

A. A change in the concentration of substrate B. A large change in the pH of the medium C. A decrease in the salt concentration of the D.
73. Which of the following could NOT be treated effecrively by either of the gene therapy techniques discussed in the passage?
77.

medium

An increase in the temperature of the medium

Of the two gene therapy techniques described in the passage, which would be the MOST successful in

combating a rapidly mutating strain


virus?

of the HIV

A. B. C. D.

Tumors caused
oncogene

by

overexpression

of

an

Sickle-cell anemia caused by a homozygous


mutation in the hemoglobin gene Liver disease caused by the hepatitis B virus Herpes caused by the herpes simplex retrovirus

A. B. C. D.

The antisense approach, because mutation in

the viral DNA would not affect antisensesense binding.

The antisense approach, because DNA-RNA hybrids would preferentially form with HIV
mutants.

The ribozyme approach, because ribozymes


cleave all mRNA.

The ribozyme approach, because ribozymes can target conserved regions of HIV mRNA.

Copyright

by The Berkeley Review

46s

The Berkeley Review Specializing in MCAT Preparation

Biology
78.

Gene Therapy Strategies

Passage XII

Which of the following could be a problem with


antisense gene therapy that would occur in human
patients, but not in human tissue culture?

A.
B. C. D.

Lysosomal digestion of synthetic antisense DNA molecules Immune rejection of synthetic antisense DNA
molecules

Failure of DNA-RNA hybridization due to


cellular pH levels Degradation of antisense DNA by restriction
enzymes

79.

What might be a normal function occurring ribozymes?

of

naturally-

A. B. C. D.

Splicing of unprocessed mRNA Proofreading of single-stranded DNA Catalysis of protein reacrions Cleavage of viral DNA

Copyright @ by The Berkeley Review

466

The Berkeley Keview Specializing in MCAT Preparation

Biology
Passage

Tryptophan Operon
80.

Passage XItr

XIII

(Questions 80-87)

leader sequence (L), and the DNA encoding the enzymes (A-E) needed to convert chorismate into tryptophan (trp), is shown in Figure l. These five enzymes are synthesized by the translation of polycistronic trp mRNA.

The tryptophan operon, including the promoter (p), operator (o), attenuator (a) control sites, genes for the

The fact that the mRNA in the tryptophan operon is polycistronic indicates one:

A. B. C. D.

protein is coded for by multiple genes. protein is coded for by a single gene. mRNA molecule codes for more than one protein. mRNA molecule codes for only one protein.

la

trpE trpD trpC trpB trpA


Structural genes

L
Figure
1

81. As it interacts with the trp operon, tryptophan


BEST described as:

is

One way to regulate the Trp operon is through a specific repressor protein. The repressor protein is encoded by the trpR gene, and it binds to the operator region of the trp operon only when tryptophan itself is bound to the repressor molecule. The repre ssortryptophan complex binds specifically to a region on the operator that overlaps with the promoter region. The result is that no mRNA is produced.

A. B. C. D.

an inducible repressor. an inducer. a corepressor. an attenuator.

82. In

an experiment with the Trp operon, deletions

It

was discovered that

162-nucleotide leader

sequence preceded the initiation codon of trpE, the first enzyme encoded in the operon. In nonmutants with high tryptophan levels, a transcript of only 130 nucleotides is found. In nonmutants with scarce trp levels, the mRNA

increase

first enzyme (trp$, resulting in a significant in the production of trp mRNA. This

were made between the operator and the gene for the

experiment provides evidence for:

transcript product is 7000 nucleotides long. Investigators

concluded that the transcription of the trp operon is regulated by a controlled termination site called an

A. B. C. D.

signaltransduction.
attenuation.

tryptophan-repressorcomplex repression. tryptophaninduction.

GC-rich sbquence followed by an AT-rich sequence. Each region exhibits a twofold axis of symmetry.

attenuator. The site is located between the operator and the gene for the first enzyme. The terminator site has a

83.

The binding of the tryptophan-repressor protein to


the operator prevents interaction between:

is plentiful, initiation of transcription is blocked by binding of the trp-repressor complex to the operator. When trp levels fall, some polymerase molecules

The attenuator site works with the generator site in regulating the transcription of trp genes. When tryptophan

dissociate from the template at the attenuation site, while others proceed to synthesize the entire trp message.

A. B. C. D.

DNA RNA RNA RNA

polymerase and DNA. polymerase and DNA. polymerase II and DNA. polymerase III and DNA.

The presence of tryptophan residues at positions l0 and 1 1 of the leader polypeptide is significanr in the
attenuator site's ability to deieCt the level of tryptophan in

84. According to the passage, it is important


attenuation that:

for

the cell. When tryptophan is abundant, this complete polypeptide is synthesized, which will terminate transcription. When tryptoplan is scarce, the ribosome stalls at tandem UGG codons because of a lack of tryptophanyl tRNA. The stalled ribosome alters the

structure

of the mRNA so that the

polymerase

A. B. C. D.

replication be coupled to transcription. replication be coupled to translation. transcription be coupled to replication. transcription be coupled to translation.

transcribing it proceeds beyond the attenuator site.

Copyright @ by The Berkeley Review

467

The Berkeley Review Specializing in MCAT Preparation

Biology
85.
be described as:

Tryptophan Operon

Passage XItr

The melting of the trp attenuator site on DNA could

A.
B.

continuous, as the nucleotide population is not


varied.

continuous, as the nucleotide population is


unimportant.

C.

discontinuous, discontinuous,

with the A-T rich with the A-T rich

region region

melting after the G-C rich region. D. melting before the G-C rich region.

86.

Several other operons for the biosynthesis of amino acids in E. coli are known to have an attenuator site. In each case, the leader peptide should contain:

A. B. C. D.

tandem pairs of the codon UGG. relatively small number of the codon UGG. an abundance of amino acid residues of the kind synthesizedby the operon. a relatively small number of residues of the kind synthesized by the operon.
a

87.

Increasing the number of residues in the leader peptide of the kind synthesized by the operon MOST likely:

A. B. C. D.

decreases
system.

the sensitivity of the detection

increases the sensitivity of the detection


system.

eliminates the need for attenuationserves only to alter the primary structure of the protein.

Copyright @ by The Berkeley Review

46a

The Berkeley Review Specializing in MCAT Preparation

Biology
Passage

Ttanslation
90.

Passage XIV

XIV (Questions 88-94)

The BEST experimental method to

The expression of genetic information often results in the production of protein molecules from the translation of messenger RNA (mRNA). To initiate protein synthesis,

separate polyribosomes from single ribosomes is through:

initiating factors (IF) bind ro the small ribosomal unit, along with a special initiator transfer RNA molecule. This complex recognizes the mRNA start codon. The recognition of the start codon causes the release of the initiating factors. This is followed by the binding of the large ribosomal subunit and results in a functional
ribosome.

A. B. C. D.

addition of ribonucleases. centrifugation. acidic wash. addition of radiolabeled amino acids.

91. In a prokaryotic cell, the special initiator aminoacyl transfer RNA molecule that binds to the start codon
transports which amino acid?

Multiple initiating events, involving multiple


ribosomes, can occur on a single mRNA transcript. The resulting polyribosome/polysome structure gives multiple, simultaneous translations of a single mRNA transcript. An important eukaryotic IF is eIF-2, which is instrumental in binding the initiator transfer RNA onto the small ribosomal subunit during the initiating phase. In

A. B. C. D.
92.

Glycine Glutamine
Asparagine

Formylmethionine

A polycistronic mRNA molecule is MOST likely to


be found in a cell that is:

reticulocytes, eIF-2 is believed to be involved in controlling the overall rate of protein synthesis, with phosphorylation of one of its subunits reducing eIF-2
activity. Translation is not an error-proofprocess. Roughly 1 in every 104 amino acids that are incoiporated into proteins

A. B. C. D.
93.

eukaryotic and contains many start codons, translated separately into multiple proteins.
.eukaryotic ard gives rise to one polypeptide, cleaved into many functional proteins.

is inserted incorrectly. The fidelity of this process is


the transfer RNA. Cells have evolved a proofreading mechanism for this pairing process involving the elongation factor, a protein that binds to an incoming aminoacyl-transfer RNA and to a molecule of GTp. This factor has the ability to separate in time codon/anticodon base-pairing and peptide elongation. This short delay between the two events allows for a bound-transfer RNA
molecule to be discharged from the ribosome.

contingent on several processes, one being the correct pairing of bases in the codons in mRNA and anticodons in

prokaryotic and contains many start codons, translated separately into multiple proteins. prokaryotic and gives rise to one polypeptide, cleaved into multiple functional proteins.

The delay in elongation, caused by the elongation


factor, acts as a proofreading mechanism, because:

A. B. C. D.
94.

the delay allows the time needed to insert the correct amino acid. the delay allows for hydrolysis of GTp, which attaches the correct amino acid to the correct
transfer RNA.

incorrectly bound transfer RNA molecules


form weaker codon/anticodon interactions and

88. In a hypothetical

elperiment, eukaryotic cells

are

are more likely to dissociate from


ribosome.

the

starved. Based on information in the passage, one would expect increased activity from:

A. B. C. D.
89.

the delay allows for hydrolysis of GTp, which drives the peptide-forming reaction at the
ribosome.

'a mutase.
a kinase. a dehydrogenase.

an epimerase.

Which of the following sratements is FALSE, based on information in the passage?

Given that the average size of a protein molecule is


400 amino acids, one can conclude that there should be a translation error in one out of how many protein

A. B. C.

The initiator transfer RNA binds to the start codon in the P site of the ribosome. Initiation factors are not needed for secreted
proteins, because these proteins are translated and transported in the endoplasmic reticulum. The anticodon found on the initiator transfer RNA is CAU. The elongation factor is reversibly bound to the aminoacyl transfer RNA molecule.

molecules?

c. D.
Copyright

A. B.

10

25

D.

50
100
@

by The Berkeley Review

469

The Berkeley Review Specializing in MCAT Preparation

Biology
Passage XV (Questions 95-100)

Cancerous Gene
96.
The tag sequence used
should:

Passage XV

in the above

experiment

In theory, one could compare the base sequence of all chromosomes in a normal cell to those in a cancerous cell, in order isolate a mutation suspected of causing cancer. However, such a method is entirely impractical. To isolate the mutation more efficiently, one could exploit the process of expressing genetic information. This is
accomplished by using a particular line of mouse cells, which stop proliferation when the population of cells on the medium is one cell-layer thick. This is a normal process, as these cells are sensitive to their population density. However, the cells can be induced, through the addition of DNA, to continue multiplying and to enter a state of uncontrolled growth.

A. B. C. D. 97.

normally be fbund in the mouse cell line. be an unknown, random region of DNA. not normally be found in E. coli. not normally be found in human DNA.

because

The Alu sequence is MOST likely used as a tag it is:

cancerous state is extracted from human tumors and is tagged before it is added to the mouse cell line. Individual cells transformed by human DNA into cancerous cells form a pile of cells known as a focus. Once a focus has appeared, DNA from these focus cells is extracted and

The added DNA responsible for inducing

A. rare within the human genome. B. easily mutated. C. likely to be near a cancer-causing, D.
gene.

mutated

the

easily inserted into the mouse cell line.

cloned in E. coli. The colonies of E. coli can then be screened for the presence of the tag. The screen involves the addition of radiolabeled DNA complementary to the
tag sequence. One particular method used to tag the human tumor DNA takes advantage of the A/a sequence found throughout the entire human genome. Human DNA is cleaved using A/r; restriction sites and is added to the mouse cell line. After
a

98. Mutated gene A has been shown to

induce uncontrolled growth. In vivo studies of the gene have demonstrated normal levels of mRNA and protein product from gene A. From this information, it can be concluded that:

A. B. D.

the mutation in gene A has resulted in a depressed RNA polymerase binding


frequency.

the appearance of a focus and cloning in E. coli,

protein product
mitosis.

radiolabeled probe complementary to the restriction site is used to isolate a DNA sequence containing not only the tag, but the mutated gene responsible fbr inducing a
cancerous state.

A is necessary for

entering

C. the mutation in gene A

has resulted

in

significant structural change in protein A. the cell line used possesses a retarded cellular ribosome level.

99.

Hybridization between complementary DNA


sequences takes advantage of:

A.

95.

B. Comparing the entire genome of a wild-type cell to a

cancerous cell

in order to isolate a mutation

C.

covalent bonds. hydrogen bonds. hydrophobic base stacking interactions.

is

D.

ionic bonds.

impractical, because:

A. it is theoretically B. C. D.

impossible to sequence an

entire genome. it is more important to characterize the faulty proteins and nbt the DNA mutations. naturally occurring mutations would mask
cancer-causing mutations. the human genome varies among individuals.

100. To screen for the

presence of the DNA tag, which of the following experimental methods should be used?

A. B. C. D.
470

Southern transfer Northern transfer Western transfer Gel electrophoresis

Copyright @ by The Berkeley Review

The Berkeley Review Specializing in MCAT Preparation

Biology
A is correct.

Expression of Genetic Information

Section X Answers

structure is changed. The base-pairing is not altered, but the hydrogen bonds holding the base pairs together break and the two DNA strands separate. choice c is incorrect. The correct choice is D.
3.

we don't know the answer, we can eliminate the three wrong answers in this question. DNA. Choice B is incorrect. Nucleotide synthases are involved in the synihesis of nucleotides. Choice C is incorrect. DNA gyrases are involved in nicking and reattaching DNA so it does not tangle. Choice D is inconect. Restriction enzymes cleave DNA at specific, often palindromic-, sequences. Their cleavige points are predictable, and they therefore are often used to cleave DNA for further study. The correct choice is A. D is correct. DNA does not form a quad structure. Choice B is incorrect. Phosphates bear many negative charges, and they do not become positively charged. Choice A is incorrect. Denaturing means that somehow the DNA
Proteases degrade proteins, not

Even

if

B is correct. In many types of gels, the electrophoretic mobility of a fragment is inversely proportional to the number of base pairs in it (i.e., its size). The buffers used often produce a similarity in charge between molecules, so choice A is incorrect. The radioactive molecule used in the Southern blot is employed afterihe gel, in Step 4. Choice C is thus incorrect. The linking number is the number of times one strand of DNA *rupt *ouid anothei in a righthanded direction, so choice D is also incorrect. The correct choice is B.
C is correct. Choices A and B are the same answer, just reversed. Both are incorrect. Choice D is a repeat of the original strand and is incorrect. A probe is the complementary base sequence to the fragment of intirest. The

4.

correct choice is C.
5.

Fragment

A is correct. This is really an easy, read-the-figure type of question. Although you are not told what the fragments mean, you can still test each statement against the figure. One of the fragments corresponds to a region that iniicates the inheritance of the HC gene. Neither parent cirries Fragment V, so choice B is incorrect. Neither child carries

III,

patterns of inheritance. Choice D is incorrect. Finally, both children have Fragment this is the HC fragment. The correct choice is A.

so choice C is incorrect. The children have different fragmentation patterns, indicating different II, so they would be affected if

6.

D is correct. The three restriction sequences for the three restriction endonucleases are given in Table l. Let's place each of those restriction sites in a section of DNA. Consider the Psr I site first. Aiter the enzyme cuts ai the appropriate site, we are left with two sticky ends. Both of those sticky ends show a piece of single-stranded DNA, which bears a 3' end. This is sometimes referred to as a 3'-overhang. Based on this observation, we can eliminate choice A.

DNA

5
3'

[----rcTGCAGr------t 3'
r--------l GACGTC

-t

t-----t

5''--------v'

_\

5'r------r
3'.1-

-----l c-s'

CTGCA-3'

5'-Gr-----r
3'-ACGTC
Sticky end

3'

t-----r

5'

Psr

I site

Consider the Sal I site next. After the endonuclease makes its cut we are again left with two sticky ends. However, this time the sticky ends can be found at the 5' of each DNA strand. Choice B is a correct answer.

DNA

5'r-----r

c-3'
CAGCT-5' \-a-J
Sticky end

s'-TCGAC
3'-G

t-------t

3'

3' t-----_1 CACCTC I-----__-l

5'

3'r------l

t-----r

5'

Sal

I site

However, before wetiush into picking choice B, let's take a look at choice C. If we use the same type of reasoning for the Taq I restriction endonuclease, we also find 5' sticky ends. Therefore, both Sal I and Taq I pioduce 5' stick! ends. The correct choice is D.
7.

D is correct. If we assume that the DNA sequence is random, then any given base (A, T, G, or C) has a l/4 probability of occurring at any position in the polynucleotide chain. Table I tells us that Alu I contains 4 bases. We
@

Copyright

by The Berkeley Review

Biology

Dxpression of Genetic Information

Section X Answers

would expect to find an Alu I restriction site every 256 bases along the length of the DNA. This is calculated as follows: (l/4)n=0/4)4=l/256,wherenisthelengthoftherecognitionsite(i.e.,thenumberofbasepairs).The ength of pBR322 is 4.3 kb or 4,300 base pairs. Remember, I kb is 1,000 base pairs. Dividing 4,300 by 256 gives the
f

expected number of sites that Alu I would cut in pBR322. The correct choice is D. 8.

D is correct. We are asked to determine the number of unique restriction endonucleases that can cut a particular segment of DNA. All we need to do is compare those restriction sites in Table I with the DNA sequence in the
question.

X*or

SmaI

sorl

5'-CGGATCCCGGGTCGACG-3' 3'-GCCTAGGGCCCAGCTGC-5'

BamHl

TaqI

various restriction sites. The correct choice is D.


9.

Be aware of restriction sites that can be cut by two different enzymes, and be aware of smaller restriction sites within larger restriction sites. In the segment of DNA shown above, the heavy lines indicate the location of the

B is correct. First, let's consider the sticky ends created by the restriction endonuclease BamHL This is shown
below:

DNA

3'l---__-lccTAGG
1

t----t 5' -

5'r------rG-3'
3'r------___lccTAG-s'
\-a-J

5'.-GATCC

r------t

3',

3'-cr--------l

5'

BamHI site

Sticky end

We want to use a restriction endonuclease that can cut a segment of DNA and give us sticky ends complementary to those created by Bam HI. Let's immediately examine the answer. The sticky ends created by BgtII are sho*n below.

W DNA
5

3'wrcr;;;*;,+
1 Bgi II site

AGATCTW!

5'WA-3'
3'WTCTAG-s' \-.--'
Sticlry end

s'-GATCT

W3' 3'-AW5'

Next, we can combine one of the sticky ends from BamHI with one of the sticky ends from Bgl II as shown below. An interesting thing happens when we do this: After hydrogen bonding has taken place and the phosphodiester
backbone hasJeen reestablished, we lose the restriction sites for both Bam HI and Bgl IL

Bgl II sticky end

DNA

r-----rc-3'

3',t-------__l ccTAG-5'

5'.GATCTW3' 3',-AWs',

\ 5't-----.l GGATCT M
" 3':-ICCTAGA W

3',

5'

-.,J Bamlil, sticky end

uoE#ur,r,
sites are lost

The other three choices (IlindlIl, PstI, and SacI) do not give DNA with sticky ends that are complementary to those given by B: HI. The correct choice is B.
10.

A is correct. We are digesting two linear DNA molecules. Let's call those DNA molecules DNA #1 (for Lane 1) and DNA #2 (for Lane 2). Note that if we cut DNA #1 with EcoP.I (see below), we get six fragmenrs, each a differenr
size. This must mean that we have five Eco RI restriction sites in DNA #1.

Copyright @ by The Berkeley Review

472

The Berkeley Review Specializing in MCAT Preparation

Biology
DNA#1

Expression of Genetic lnformation


EcoRl
Eco RI

Section X Answers

EcoRl
......

DNA#2
Eco RI Eco RI

Y'..
EcoRI EcoRl

F'
EcoRl

New

acd Rr

.Eco P.r, we get seven fragments, each a different size (see above). This means that there are six Eco RI restriction sites in DNA #2. Note thaiFragment X from DNA #2 ii a little larger than Fragment D from DNA #l' Also note that Fragment Y is a little smalGr than Fragment D, but a little larger than Fragment E from DNA #l' Fragments c and E in DNA #2.are the same as Fragments C and E in DNA #1. we can conclude from this that more DNA and an additional resffiction site were added it Fragment D of DNA #1.

If we cut DNA #2.with

If we were to delete DNA from Fragment D of DNA #1 (without adding a new restriction site), we would still get six fragments' This is not what we see in DNA #2. Eliminate choice g. *Ttre same would be true if we were to add DNA only to Fragment D of DNA #l (without adding a new restriction site). Btiminate ctroice
c.
we delete DNA from Fragment D of DNA #1, and as a result a new Eco RI site forms, then we get seven fragments' This is what we are looking for. However, once we delete DNA fr;. p*g.*n, D, the new fragment that forms (call it x'Y') is smaller than thJoriginal Fragment D. Afrer electrophoresit, irii, n"* Fragmenr X,y, is below the level of Fragment D on the gel. Furthirmor", i? *" now cut at the new rco Rt restriction site within Fragment x'Y" the resulting two fragments still end up below the level of Fragment D ufd"i;;;.ophoresis. This is not what we observe in Lane 2 of the gel for the fragmintation of DNA #2. Thecorrect choice is A.

If

11'

B is correct' The first thing to do when creating a restriction map is to determine the size of the DNA. If we add the fragment sizes in the first lane (created by a sin-gle digest with Bgilr), we get 7 kb + 2 kb = g kb. If we follow this same procedure for the remaining fragmentation tanei, we get s ku for of ttem. rnis tells us our fragmented bacterial DNA was originally o tlu in tength. Look at all of"the restriction "uih maps in rhe answer choices. Do all the fragment sizes add up to 9 kb? yes, they do.
The next step is to look at the fragmentation sizes in each of the six digest lanes of the gel to see whether they match up with the answer choices..A single digest with_ just BglII gives frag-ment ri"", oii [u una 2 kb. we observe rhis fragmentation pattem in choices B and C. A single aigeJt witln just Eco RI gives fragment sizes of 5 kb and 4 kb. we observe this fragmentation pattern in choices i unin. At this poinr in ;r.;;;6*, oi"" B looks like a good answer. But let's make sure. "t

A single digest with just Ps/ I gives fragment sizes of 6 kb, 2 kb, and I kb. we observe this fragmentation pattern in choices A' B' and D. A double digest uiing bo$ Ec9 RI and ngi[gives fragment sizes of 4 kb, 3 kb, and 2 kb. we observe this fragmentation pattern in choices B and D. A douEle dlgest usii'g il#;; i and BguI gives fragmenr sizes of 6kb,2 kb, and I kb. we observe this fragmentation pattern in cholces A and B. Finally, a double digest using both EcoP.r h'nd PslI_gives fragment sizes oi3 kb (there are two of them, since the band is twice as thick), 2 kb' and 1 kb' we observe this fragmintation pattern in ciroices A, B, and c. irt.r.t"tidering all the choices, we find that each digest lane gives thehagmentatibn pattern shown in choice B. The correct choice is B.

t2.

D is correct' This is a question designed to make sure we understand the roles of the three RNA molecules in the cell' A ribosome is composed of amino acids and rRNA, and is essentially a surface for assembling proteins. statement I is correct' .lN{ is the transcript or copy of DNA. statement II is correct. IRNA is the amino acid carrier for protein translation. Statement III is correct. The correct choice is D. D is correct' The focus of this question is the mutation of HIV. To combat this research problem most effectively, one should try attac.ks- simultaneously by linking several ribozymes together. This would cause the most -multiplb damage in the first strike and also would attack any mutaiions that survived and tried to reproduce. Following one enzyme with another two weeks later could still permit mutations to occur, so choice B is inconect. Increasing the concentration of the original ribozyme does noi combat mutation, because the survivors would still be able to recolonize the patient' choice A is incorrect. A specific ribozyme for each patient would not aif'ect further
so choice C is incorrect. The correct choice is D.

13.

mutation,

Copyright @ by The Berkeley Review

473

The Berkeley Review Specializing in MCAT preparation

Biology
14.

Expression of Genetic lnformation

Section X Answers

B is correct. Choice A is incorrect, because highly conserved regions do not have much variability from strain to strain. HIV can be attacked by ribozymes at any number of sites, not just these conserved regions, meaning that
choice C is also incorrect. And eliminate choice D: These regions cannot attract ribozymes. The action leading to catalysis is a chance meeting of two molecules that fit together. The correct choice is B.

15.

B is correct. Prior to mutation, there are fewer copies of a virus if its mRNA is decreased. Each mRNA does not become more efficient at translation. Choice A is incorrect. The number of transcripts is not increased, so viral particles cannot increase. Choice C is incorrect. Choice D refers to mutant (competing) HIV mRNAs, and the question specifically asks about changes prior to mutation, so choice D is incorrect. The correct choice is B.

16.

A is correct. There are only four different nucleic acids used to build every RNA molecule. An enzyme has access to twenty different amino acids. This increase in the number of potential subunits gives much more variety in structure and function to protein-based catalysts. Statement I is correct. Enzymes did not outcompete ribozymes by chopping them up. They merely worked better and were therefore selected by evolutionary pressure. Statement II is incorrect. Enzymes do not change the equilibria of reactions, so statement III is incorrect. The correct choice is A.
B is correct. Catalysts can participate in many reactions, because they remain unchanged themselves while effecting change in other molecules. Statement I is correct. Catalysts do increase the rate of reactions, but not the equilibrium, meaning that statement II is correct. And since we know that the body does synthesize catalysts, statement III is incorrect. The correct choice is B.

17.

18.

C is correct. A single-base code can code for four different amino acids. One can arrive at this answer by knowing that there are four different bases used in DNA and that ifthe code had only one base, then only four different amino acids could be coded. Accordingly, a three-base system can code for a total of sixty-four amino acids (4 x 4 x 4 = 64). The question asks for the difference between the two, making 64 - 4 = 60 the correct answer. The correct choice is C.

19.

C is correct. To answer this question, one has to know what is meant by an overlapping genetic code. As in the question, let us assume the base sequence is ATCGGTATA. If this genetic code is overlapping, the first three amino acids come from ATC, TCC, and CGG respectively. If the code is nonoverlapping, the first three amino acids come from ATC, GGT, and ATA respectively. Since the question asks about the products of an overlapping code (the code is, of course, non-overlapping), as many as three amino acids may be affected. The correct choice is C.
C is correct.Without the extra base inserted, the base sequence can generate four amino acids. Remember, the code is nonoverlapping. Let us look at the amino acids produced with the extra base inserted. The first two amino acids are not affected by the insertion. However, the third amino acid is. Without the insertion, the triplet is ACA. With the insertion, the code is ATC. The same logic applies to the fourth amino acid. Thus two amino acids are affected. The correct choice is C.

20.

21.

B is correct. The passage states that part of the protocol of the experiment is to wash the protein-containing precipihteand measure the level of carbon 14. One can assume we are measuring the radiolabel in the protein. If
this is the case, we must use radiolabeled amino acids as part of the experiment, because this is the only way we can incorporate radiolabel into the protein. Do not lose track of the big picture. We are controlling the mRNA added and asking what amino acids are called for. This means we need to keep track of amino acids, so radiolabeling them is essential. The correct choice is B.

))

C is correct. The passage states that a component of the experiment is the addition of DNAse, which results in a cessation of protein synthesis. This implies that protein synthesis was occurring and then ceased, and it should be indicated on a graph by a line with a positive slope followed by a line with zero slope. This alone should eliminate all choices except C. Looking at the graph in C, the zero-slope line is followed by another rise in protein synthesis. This occurs because we added our own synthesized mRNA, which created the new protein. Remember, that is the point of the experiment--the ability to synthesize protein in a cell-free system. The correct choice is C.

23.

A is correctl Let us look at the data in Table 1. With phenylalanine, we see a l00Vo relative amount incorporated. Remember, the label is on the uracil, and therefore we can conclude that phenylalanine is coded by a triplet consisting entirely of uracil. The question asks about glycine. Glycine has the lowest count, according to the data. The fact that it has some count invalidates the idea that the triplet is all guanine. Furthermore, there cannot be an
equal amount of the two bases in a triplet design. Since the count is low, the most logical conclusion is that there is more guanine than uracil in the code for glycine. The correct choice is A.

Copyright @ by The Berkeley Review

474

The Berkeley Review Specializing in MCAT Preparation

Biotogy

Dxpression of Genetic Information

Section X Answers

24.

hybridized primers and the template' This is so because a temperature of 45" cit n.""rru.y ror the hybridization. At this temperature, the polymerase is inactive. Therefore, *" ,uir" the temperature to 75' C, at wtrich the polymerase becomes active. By this point, those primers that are not hybridized correctly will dissociate from the incorrect sites. Fven though some of the correctly hybridized primers fali off, a much gr.ut.. fr";;i;;;i;.;mers on incorrecr sires fall off' The reaction becomes more specific. Again, the polymerase fr6m E. coli.unnotiuk" the heat used in the denaturation process. The correct choice is B. 25.

E coli could be used in the pcR reaction, the scheme would be inefficient, because each round of denaturation of the double-stranded DNA helix to form the individual single strands would result in destruction of the polym erase. Thermus aquaticus is a thermophl", ana because of its normal living conditions, it contains a temperature-resistant polymerase. This polymerar. *ithrtund, temperatures of 95" c, which is necessary to denature the DNA. The use oi thir polymerase aliows for increased specificity between the

B is^correct' Although DNA polymerase I from

wavelength' This effect is known as hypeichromism. tie spontaneous reassociation of separated complementary strands of DNA is known as annealing. Therefore, this eliminates choices B and D. To discriminate between choices A and c, one must draw on their knowledge of biochemistry. As stated above, the increased absorption due to basepair unstacking is known as hyperchromism. The correct choice is A. 26.

A is correct' The melting of DNA is readily monitored by measuring its absorbance of light at 260 nm. The unstacking of base pairs.th.at occurs during the melting pro."rr results in an increased absorbance of light at this

A is correct' First, we must realize that a suitable clone would contain the inserted fragment. This is why we are using PCR' we are screening clones to see if a piece of DNA was in-serted into the plasmid. The larger piece of DNA thus be represents the suitable clone, becauie the foreign piece of DNA was draw on our knowledge of gel filtration chromatograptry.The apparatus used td"dyl;r"rred. Next, we must in tlis setup includes a column consisting of porous beads made of an insolubte uut trig-trty hydraied polymer. Small moiecules can enter these beads, but larger ones cannot. In the beads, the movement of the small moiecul"r ir tt"i;;d. The result of this is that large molecules flow more rapidly through this column and emerge first (because a smaller volume is accessible to them)' Therefore, the product of the suitible clone, alarge mole"cule, emerges first because of its size, not its charge.
The correct choice is A.

27.

A is correct' we are looking for the DNA molecule that is expected to have an abundance of A-T base pairs. The melting temperature ofa.DNA molecule depends heavily on its base composition. DNA molecules rich in G-C base pairs have a higher melting temperature (defined as hali of helical structure lost) relative to those DNA molecules with a high A-T base pair content. The reason for this lies in the number of hydrogen bonds between the bases, with three between G-c and only two between A-T. In addition, adjacent G-C base paiis interact more strongly with one another than do adjacent A-T base pairs. The DNA melting Ju.u", show absorbance as a function of temperature. Remember that the absorbance riie is indicative of a d'ouble-stranded molecule becoming a single-stranded molecule' We are looking for the molecule that is going_to have the highest A-T bur;-p;; In other words, it is going to have the lowest melting point temperature. ihis correspond"s to the curve on the"oni"",. far left, as its absorbance rise occurs at the l6west te-pe.utuie. The correct choice is A.
C is coffect. According to Figure l, one cycle of PCR uses one molecule of DNA to make two molecules of DNA. Another round of PCR uses two molecules of DNA to make four molecules of DNA. In other words, the number of molecules of DNA made during n cycles is 2n. Therefore, after seven cycles of the pCR reaction, we should see l2g molecules of DNA. However, the question asks how many strands of DNA exist after seven cycles. Each DNA molecule is made up of two strands. So 128 x 2 256. The correct choice is c. =

28.

29.

D is correct' The primers used in the PCR scheme are complementary to the template DNA, which contains the region of DNA to be amplified. They are not complementaryio each other. This eliminates choice A. Furthermore, th-ese primers probably are not going to contain the same nucleotides. Their sequen".r d;;;J;n the sequence of the DNA region with-which they are hybridizing. This eliminates choice g. Rs for choice c, the primers are DNA oligonucleotides. Thdrefore, they do not contain uracil. only RNA contains uracil in place #ttry.in". choice c can be.elimjnated. The only logical choice is D. The primers aie complementary to the iegions of DNA that flank both sides of the piece of DNA to be amplified. one p.i*". is complementary to one side,-while the other DNA primer hybridizes with the other. only then do we get ihe amplification of DNA that we are interested in. Therefore, the primers are designed to conform to the regioni of DNA ihat surround the DNA to be amplified. The correct choice is D.
@

Copyright

by The Berkeley Review

475

The Berkeley Review Specializing in MCAT preparation

Biology
30.

Expression of Genetic Information

Section X Answers

C is correct. If lactose cannot be utilized by the epithelial cells, then stays in the intestinal lumen. Lactose acts as a solute in the lumen and begins to draw water out of the lumenal epithelial cells (the reverse of choice C). Too much water in the lumen can leid to diarrhea. Also, bacterial cells have the ability to take up lactose and metabolize it anaerobically (fermentation). One of the products of fermentation is COz, which is a gas. The correct choice is C.

31.

C is correct. The second paragraph of the passage says, "Metabolism of lactose requires galactoside permease, a cotransporter of protons and lactose...." If protons and lactose are cotransported, it means that they are allowed to pass thiough the iell's membrane together. The transport of lactose into the cell is coupled to the energy stored in the
irunr*.1nd.une potential that residei across the plasma membrane. Recall from metabolism that protons are pumped out of the cell is the electrons move down the electron transport chain. Not only is a large membrane potential established, but a high concentration of protons now resides in the medium outside the cell. These gradients of stored energy can beirsed to import sugars like lactose into the cytosol. When molecules are transported into a cell together (oi-out of a cell together), the process is referred to as a symport.If molecules move in opposite directions as"they are transported through the ceil's membrane, the process is referred to as an antiport.If one molecule is transpbrted through the cell's membrane, the process is referred to as a uniport. Active transport requires the hydrolysis of ATP. There was no mention of ATP hydrolysis in the passage. The correct choice is C.

32.

D is correct. In the third paragraph of the passage, we learn that RNA polymerase binds to the promoter (P). The promoter, as shown in Figure 1 of the passage, is a section of DNA. In the fourth paragraph, we learn that RNA polymerase can transcribe the structural genes (i.e., can make mRNA). The structural genes also involve a section of bNA. Sin"" we know that RNA polymerase synthesizes mRNA, we can eliminate choices A and B. Is the RNA polymerase directed to synthesize mRNA by RNA or DNA? The polymerase is binding DNA. Therefore, it is the

33.

repressor binds to the operaior, and RNA polymerase is unable to transcribe the structural genes." This tells us that in order to make a tranicript, we must remove the repressor from the operator. How do we remove the repressor? We need an inducer (see the fourth paragraph again). Lactose can enter the cell and be converted into allolactose, which is the inducer. Lactose itself is not the inducer. It is allolactose (an isomer of lactose) that is the true inducer. Once the inducer binds to the repressor, the inducer-repressor complex is no longer active, and it no longer binds to the operator. With this informaiion alone, we can eliminate choices A and B. RNA polymerase can bind to the p.o-bt... The inducer-repressor complex (choice D) does not bind to the promoter. If it could, RNA polymerase would not have a place to occupy in order to begin transcription. Once RNA polymerase binds to the promoter and

bNA that is directing the RNA polymerase as it synthesizes the mRNA. The correct choice is D. C is correct. In the fourth paragraph of the passage, we learn that "if lactose is absent from the medium, the lac

the repressor has been removed from the operator (by complexing with the inducer), the DNA is free of

obstructions, and transcription can begin. The correct choice is C. 34.

B is correct. We learned what happens to lactose in the first four paragraphs of the passage' What happens to glucose is contained in the fifth paiagraph. When glucose is in the cell, the cAMP concentrations start to decrease. F.ight u*uy, we can eliminate choices A and C. We next learn that transcription of the lac operon is reduced some SO-tota. Inother words, the synthesis of B-galactosidase is greatly reduced. This is what choices B and D state. We need something else. If the levels of p-galactosidase decrease, it must mean that there is a repressor on the operator. The repressor-must be active in order to be occupying the operator. This allows us to eliminate choice D. Let's continue (even though we now know the answer). If the cAMP levels in the cell are low, there is not enough cAMP to bind the CRp. Tlie cAMp-CRp complex does not form. Therefore, the CRP cannot be activated. It will remain
inactive until the levels of cAMP once again begin to increase. The correct choice is B.

35.

the passage, we find ihat th"re types of mutations are called constitutive. They lead to an increase in the synthesis of lac ope.oi proteins in the absenie of inducer. If there is no inducer around, we would expect the repressor to bind to the operator. However, the repressor is defective due to a mutation. It cannot bind to the operator. Thus, in the absenie of inducer, we would expect to find a lot of the lac Z, lac Y, and lac A proteins being synthesized.

D is correct. The key to this question involves the I- mutation of the FO+Z+Y+A+ genome. In the last paragraph of

lacZ lacA lacY high high high


We would also expect to see high levels of these gene-product proteins being synthesized in the presence of inducer as well. The induier does not bind to the operator, so in this case it does not matter whether it is present. The correct choice is D.
Copyright @ by The Berkeley Review

476

The BerkeleY Keview Specializing in MCAT Preparation

Biology
36,

Expression of Genetic Information

Section X Answers

structural genes are transcribed. However, there is a mutation in the operitor, Oc, that allows the structural genes to be transcribed all the time., The lac Z+ gene is normal and produces a good protein. The lac y and lac A- glnes are mutants. The proteins produced from these genes are defective. Therefore, lhe top genotype permits onlyihe lac Z gene to be constitutively produced. Consider the bottom genotype next (I-O+Z-Y*A*). The f gene does not produce a normal repressor. The mutant repressor is not able to bind to the normal operator. In the presence of an inducer, this is not a problem. The inducer does not bind to the operator anyway. Since the repressor is a mutant, and since nothing is binding the operatoi, the structural genes can be transcribed. However, the lac Z- gene bears a mutation, and the protein is defective. The lac Y+ and lac A+ genes are normal and give good proteins. In the absence of an inducer, the operator region is still wide open. we get a defective lac z- protein and good lac y+ and lac A+ proteins.
Based on this, we might conclude that all three gene products are made constitutively. However, we are dealing with a partial diploid genotype and not individual haploid genotypes. Let's take a look at the combination of th-e two genotypes in the partial diploid:

D is correct. Based on the genotype, which enzymes are produced all the time? Let's break the partial diploid into two haploid segments. Consider the top genotype first (I+OCZ+Y-A-). The I+ gene produces a normal ."p..rro., which in turn binds to the operator, O. In the absence of an inducer, the repreisor bind, to the operator, and the structural genes are not transcribed. In the presence of an inducer, the represior does not bind the operatol and the

lo+z-Y+A+
The last paragraph of the passage states that "independent mutations can also occur in the trans-acting (capable of activity on another chromosome) regulatory gene and in the cis-acting (capable of activity only -on the same chromosome) operator gene." This tells us that the product of the lac I gJne is Aiffusible. In oiher words, once the repressor is synthesized by. the normal lac I+ gene, it can diffuse thoughihe cytosol and bind to the operator, which is O+. In the absence of an inducer,, the repressor remains bound to thaioperator, and transcription cannot proceed. If an inducer becomes available, it binds to the repressor and removes it from the operator. This is refirred to as induction' and this is what we see occurring in the case of the lac Y+ and lac A+ genes. The only gene that is synthesized constitutively is the lac Z+ gene. The correct choice is D.

37.

displacement of the nucleosome.

B correct. Some regions of DNA that lack a nucleosome (histone protein complex) can be detected by treating -is cell nuclei with trace amounts of DNase I. At low concentrations, the nuclease digeits long stretches of nucleosomefree DNA, but not the short stretches of linker DNA between nucleosomes. Manf of the SNase-hypersensitive sites in cell chromatin are located in the regulatory regions of a gene, and more of these sites are present in cells where the gene is active than in other cells. Most such sites are believed to represent regions from which a nucleosome has be_en displaced bgrr sequence-specific DNA-binding proteins involved ln eukary6tic gene regulation. Knowing this information, it is most likely that gene-regulatory proteins found only in erythrocytet-UinA toitr" enhancer andiause

Cbnsider the other answers. Alt cells have the same genome, so the argument that only the RBC has the enhancer does not work. Therefore, we can eliminate choice A. Choice C is simply a false statement. Just because the cell will eventually shed its nucleus does not mean that the biology of an intact nucleus can be ignored. Finally, it is not true that gene regulatory proteins in other cells bind only to promoters and not to enhancers.-Most likely, ih.." *" g"n"regulatory proteins that bind to both regulatory elements. The point here is that only in red blood cells do we fin-d the particular gene-regulatory protein that binds to the enhancer and causes displacement of the nucleosome. The correct choice is B.
38.

C is correct. If welare looking for proteins that can bind to DNA, the amino acids that make up those proteins should carry a positive charge to create an electrostatic attraction. We are therefore looking for baiic aminb acids, because they carry positive charges at physiological pH. The two basic amino acids in ou-r choice of answers are lysine and arginine. Therefore, we should see a high proportion of these amino acids in histone proteins. The correct choice is C.

Copyright @ by The Berkeley Review

477

The Berkeley Review Specializing in MCAT Preparation

Biology
39.

Dxpression of Genetic Information

Section X Answers

B is correct. This problem requires a proper interpretation of Figure 1. We are looking at a sequence of 108 nucleotide pairs. The figure tells us that we make 2J mutant B-globin enhancers. Therefore, a block of four
nucleotide pairs is changed in each mutant. The correct choice is B.

40.

D is correct. Looking at the figures, one should be able to deduce the following: We are inserting a mutant B-globin enhancer in a test plasmid. The oligonucleotide and the cloning site are joined by the enzyme DNA ligase. The enzyme produced after placing the plasmid inside a leaky erythrocyte is the bacterial enzyme chloramphenicol acetyl transferase, which is easily assayed. One should realize that we are not measuring the rates of RNA synthesis directly. We are measuring transcription in an indirect way. Therefore, we can eliminate choices A and B. Also, we are not producing the p-globin protein; we are using the mutated p-globin enhancer and producing another protein (CAT), which is assayed. We can thus eliminate choice C. We are indirectly testing the mutant enhancers for their effect on RNA transcription by measuring the amount of protein that the recombinant gene produces (as CAT enzyme activity). The correct choice is D. C is correct. We are told that a newly defined regulatory element from a vertebrate gene is analyzed, and that many of the proteins that bind to it (gene-regulatory proteins) turn out to be previously described. This can suggest only that there is a relatively small number of gene-regulatory proteins, and that they may control transcription in higher eukaryotes. Some of these gene-regulatory proteins are AP2, ATF, SPl, and NFl. None of the other conclusions can be derived from the claim made in the question. All of the other choices are general conclusions believed to be true. Nonetheless, the information in the question would support only the inference drawn in choice C. The correct
choice is C.

41.

42.

B is correct. The major difference between protein and steroid hormones is that protein hormones bind to receptors on the surface of the cell and bring about cellular change via secondary messengers. Therefore, we can eliminate choices C and D. Steroid hormones, which are quite lipid-soluble, have the ability to traverse the lipid bilayer. When they do, they bind to a receptor molecule located in the cell. The receptor molecule has a region for binding the steroid itself and a region for binding a DNA molecule. When the steroid binds to the receptor to make a steroid receptor complex, the complex becomes active and binds to a region of the DNA known as a regulatory element. The steroid itself cannot bind DNA, because the DNA-binding region is on the receptor. The correct choice is B.
C is correct. One must look at Figure 4 and interpret the data carefully. The y-axis is percent enhancer activity. Let's look at the protein NFl-like. If we mutate the region where NF1-like proteins bind, we see 100 percent enhancer activity. This means that NF1-like proteins do not play a part in this enhancer's activity. In other words, we took away NF1-like activity and nothing changed. We can conclude that this protein does not make a large contribution to the enhancer's activity. Therefore, we are looking for mutations that resulted in a low percentage of enhancer activity. This would indicate that these proteins make a large contribution to the enhancer's activity. From Figure 4, we see that AP1-like, AP2-like, and Eryfl proteins do make a large contribution. We know this because when we mutate the region where these proteins normally bind, the enhancer activity goes way down. The correct choice is C.

43.

44.

C is correct. We fused the two cells together, and we found that the Gl phase cell nucleus began to replicate DNA. Therefore, we are talking about a transition from G1 to S, and not from S to Gl. The cell cycle goes from Gl to S. Based on this information, we can eliminate two of the choices. Next, we ask whether this activator is going to be diffusible or not. If the molecule is not diffusible, it would most likely be bound to the DNA of the S phase cell. In that case, it could not act upon the DNA of the Gl nucleus. The activator must be diffusible, because it can set the G2 nucleus into an S-phase activity, namely DNA replication. The correct choice is C. D is correct. Let us first decide whether the replication is synchronous or not. The passage tells us that during the S phase, some parts of a chromosome will not yet have been replicated, while others are finished. This describes something asynchronous. Furthermore, we are talking about a eukaryotic cell that is known to contain multiple origins of repl,ication because of the volume of genetic material found in its chromosomes. The correct choice is D.
B is correct. One can assume that the replication origin for a frog looks different than one for a bacterium. However, the bacterial DNA is still subject to a rereplication block. We know nothing of how this block is achieved, so choices C and D can be eliminated. Nevertheless, since the rereplication block affects the DNA of the bacterium, one can assume that the mechanism does not require a specific origin of replication. The correct choice is B.

45.

46.

Copyright

by The Berkeley Review

47A

The Berkeley Keview Specializing in MCAT Preparation

Biology
47.

Dxpression of Genetic lnformation

Section X Answers

C is correct. First, we have to decide whether the G2 nucleus is stimulated or prevented from performing DNA replication. From the passage, we are told that this fusion does not result in the rereplication of the G2 nucleus. Therefore, we can eliminate answers claiming that the G2 cell is stimulated to begin DNA replication. The next question becomes whether the molecule responsible for the prevention is bound or diffusible. If the molecule were diffusible, it would seem logical that the S phase cell would cease DNA replication. The passage tells us differently, however, so we can infer that the molecule is bound to the DNA. The correct choice is C. D is correct.Look at Figure

48.

l.

In the G2 phase, the proteins are not on the genetic material. In the Gl phase, they

DNA synthesis again only after mitosis is complete. The two new daughter cells will go on to divide. The rereplication block must be removed at or near the time of mitosis. The correct choice is D.
49.

are. The only phase in between those two is mitosis. This makes sense. We want to be able to initiate

D is correct. This question is very straightforward. We are told that one chromosome has been replicated through ten cycles without separation, so how many strands of chromatin are now side by side? One chromosome replicating makes two strands of chromatin. Two strands of chromatin replicating makes four strands, and so forth. Two to the tenth power is 1024. The correct choice is D. D is correct. The protein bound to the DNA in the model must get there somehow. Recall that in eukaryotic cells, transcription and translation are separated by a nuclear membrane. The mRNA transcript is translated in the cytosol. In order for the protein to reenter the nucleus, it must cross back over the nuclear membrane. This is not accomplished by simple diffusion. Recall that the nuclear membrane contains nuclear pores that allow proteins to enter the nucleus. The pore recognizes the protein as belonging in the nucleus by means of the protein's nuclear import signal, and the protein is then transported into the nucleus. The correct choice is D.

50.

51.

C is correct. Albumin is a plasma protein found in an average concentration of 4 gm/dl. Albumin provides the critical osmotic pressure that regulates the passage of water and diffusible solutes through the capillaries. When the concentration of albumin is severely reduced, excess extracellular fluid may accumulate in the extravascular tissues. This condition is known as edema. Albumin can also act as a carrier protein for things like bilirubin and fatty acids. Based on this information, one can eliminate the other possible answers. Albumin is not likely to be found in the
cytosol of white or red blood cells. Furthermore, there is no evidence that the protein albumin would not be found in the cerebral spinal fluid. The correct choice is C.

<)

D is correct. The cells that produce albumin are the hepatic parenchymal cells. Thus, the cells that produce albumin are found inside the liver. This is a very straightforward question that tests previously acquired knowledge not found in the passage. The correct choice is D.

53.

B is correct. This answer comes from an understanding of the goal of our experiment. We want to amplify the nucleic acids of these liver cells so we can do further experimentation. Therefore, it is important for us to protect
those molecules we want to use later. We can protect the nucleic acids by inactivating the nucleases. Consider the other answers. Would we want to inactivate the proteases? It probably does not matter. We are going to do phenol extractions to rerffive protein. Furthermore, we are not interested in the protein, just the nucleic aiids.lherefore, we can eliminate choice A. Again, we are not interested in inactivating lipases, as long as the nucleic acids are not

threatened. Therefore, we can eliminate choice C. Finally, human cells do not have cellulases (enzymes able to
ddgrade the molecule cellulose), so choice D cannot be right. Therefore, the protection of the nucleic acids is central to our experiment, and thus inactivation of the nucleases is fundamental. The correct choice is B.

54.

B is correct. The RNA from the normal mouse cells is used as an experimental control. For example, when we run a gel of the two RNAs (one from defective cells and one from wild-type cells) we have a means of comparison, or a control. Let's consider the other possibilities. Choice A is silly. We do not need the RNA to test the equipment. Both choices C and D hinge on the word "directly." We cannot use this normal RNA to measure the size of the mutant RNA directly. We will be able to say whether one is bigger or smaller from the results of the gel, but the actual size determination comes from running standards (pieces of RNA whose size is known already) on the gel as well. The same holds true for ihe amount. The normal amount cannot be used to measure the amount of mutant RNA directly. (Although, if we so desired,.we could measure the amount of mutant RNA directly.) The correct choice is B. C is correct. From the passage, it can be determined that the wash is done before the autoradiography. From this alone, one can eliminate choices B and D, because they claim the wash is done after autoradiography. This ofcourse

55.

Copyright @ by The Berkeley Review

479

The Berkeley Review Specializing in MCAT Preparation

Biology
47.

Expression of Genetic Information

Section X Answers

C is correct. First, we have to decide whether the G2 nucleus is stimulated or prevented from performing DNA replication. From the passage, we are told that this fusion does not result in the rereplication of the G2 nucleus. Therefore, we can eliminate answers claiming that the G2 cell is stimulated to begin DNA replication. The next question becomes whether the molecule responsible for the prevention is bound or difTusible. If the molecule were diffusible, it would seem logical that the S phase cell would cease DNA replication. The passage tells us differently, however, so we can infer that the molecule is bound to the DNA. The correct choice is C. D is correct.Look at Figure
1. In the G2 phase, the proteins are not on the genetic material. In the are. The only phase in between those two is mitosis. This makes sense. We want to be able to initiate

48.

Gl

phase, they

DNA synthesis

again only after mitosis is complete. The two new daughter cells 49.

will go on to divide. The rereplication block must

be removed at or near the time of mitosis. The correct choice is D.

D is correct. This question is very straightforward. We are told that one chromosome has been replicated through ten cycles without separation, so how many strands of chromatin are now side by side? One chromosome replicating makes two strands of chromatin. Two strands of chromatin replicating makes four strands, and so forth. Two to the tenth power is 1024. The correct choice is D. D is correct. The protein bound to the DNA in the model must get there somehow. Recall that in eukaryotic cells,
transcription and translation are separated by a nuclear membrane. The mRNA transcript is translated in the cytosol. In order for the protein to reenter the nucleus, it must cross back over the nuclear membrane. This is not accomplished by simple diffusion. Recall that the nuclear membrane contains nuclear pores that allow proteins to enter the nucleus. The pore recognizes the protein as belonging in the nucleus by means of the protein's nuclear import signal, and the protein is then transported into the nucleus. The correct choice is D.

50.

51.

C is correct. Albumin is a plasma protein found in an average concentration of 4 gm/dl. Albumin provides the critical osmotic pressure that regulates the passage of water and diffusible solutes through the capillaries. When the concentration of albumin is severely reduced, excess extracellular fluid may accumulate in the extravascular tissues. This condition is known as edema. Albumin can also act as a carrier protein for things like bilirubin and fatty acids. Based on this information, one can eliminate the other possible answers. Albumin is not likely to be found in the cytosol of white or red blood cells. Furthermore, there is no evidence that the protein albumin would not be found in the cerebral spinal fluid. The correct choice is C.
D is correct. The cells that produce albumin are the hepatic parenchymal cells. Thus, the cells that produce albumin are found inside the liver. This is a very straightforward question that tests previously acquired knowledge not found in the passage. The correct choice is D.

<)

53.

B is correct. This answer comes from an understanding of the goal of our experiment. We want to amplify the nucleic acids of these liver cells so we can do further experimentation. Therefore, it is important for us to protect
those molecules we want to use later. We can protect the nucleic acids by inactivating the nucleases. Consider the other answers. Would we want to inactivate the proteases? It probably does not matter. We are going to do phenol extractions to ren,6ve protein. Furthermore, we are not interested in the protein, just the nucleic aiids.lherefore, we can eliminate choice A. Again, we are not interested in inactivating lipases, as long as the nucleic acids are not threatened. Therefore, we can eliminate choice C. Finally, human cells do not have cellulases (enzymes able to ddlrade the molecule cellulose), so choice D cannot be right. Therefore, the protection of the nucleic acids is central to our experiment, and thus inactivation of the nucleases is fundamental. The correct choice is B.

54.

B is correct. The RNA from the normal mouse cells is used as an experimental control. For example, when we run a gel of the two RNAs (one from defective cells and one from wild-type cells) we have a means of comparison, or a control. Let's consider the other possibilities. Choice A is silly. We do not need the RNA to test rhe equipmenr. Both choices C and D hinge on the word "directly." We cannot use this normal RNA to measure the size of the mutant RNA directly. We will be able to say whether one is bigger or smaller from the results of the gel, but the actual size determination comes from running standards (pieces of RNA whose size is known already) on the gel as well. The same holds true for ihe amount. The normal amount cannot be used to measure the amount of mutant RNA directly. (Although, if we so desired, we could measure the amount of mutant RNA directly.) The correct choice is B. C is comect. From the passage, it can be determined that the wash is done before the autoradiography. From this alone, one can eliminate choices B and D, because they claim the wash is done after autoradiography. This of course

55.

Copyright @ by The Berkeley Review

479

The Berkeley Review Specializing in MCAT Preparation

Biology

Dxpression of Genetic Information

Section X Answers

would make no sense, because it is the radiolabel that makes the autoradiography process possible. If we washed away all the radiolabel after the autoradiography, then we would see no autoradiography results. If we washed the background away after the autoradiography, our results would not be as accurate as they could be. This leaves us with choices A and C. Do we wash all the radiolabel away? Of course not. This would give us no results. We wash the background away only to get better results. Those radiolabeled pieces of DNA that do not really belong (i.e., they are not truly hybridizing to the RNA) but they still manage to bind to something anyway should be washed away. The correct choice is C.
56.
see that the RNA from the defective cell ran farther down on the gel. This indicates that the protein is indeed smaller. Therefore, based on this information alone, one can eliminate choices B and C, because they claim the opposite case. In choice A, we are told the defective protein is smaller, but the defect is explained as arising from a mutation in the RNA. This does not hold. Mutations in RNA come from sloppy transcription and should not be permanent. It is not right to say that mutant DNA gave rise to mutant RNA. The faulty RNA is just the transcript of the faulty DNA. In other words, the mutation is in the DNA, the information molecule. The correct choice is D.

D is correct. From the picture, one can

57.

B is correct. The map should show that the restriction nuclease B cuts 2kb to the right of the left end of

the

molecule. In addition, the restriction nuclease A cuts 3kb to the left of the right end of the molecule. This is the only combination that fits with the given data. If we switch the sites (meaning A is where B is), the data would not fit, because cutting with A gives a piece of DNA that is 2kb long and one that is 8kb long. The correct choice is B.
58.

B is correct. In order for both probes to react, the child must be a heterozygous. The heterozygous child will be in possession of a normal and a defective copy of the gene. So understanding this, we are really only looking for the probability of the child to be heterozygous for the trait. In running the cross, one would find that there is a 50Vo chance that this is true. The correct choice is B.

59.

are 44 = 256 base pairs long. Divide 10,000 by 256 to get approximately 39. Choice C is off by a factor of 10 and is incorrect. Choices A and B are other math mistakes, and they are incorrect. The correct choice is D.

D is correct. Restriction fragments, in general,

60.

A is correct. Approximately 997o of human DNA is identical among all human individuals. This is highly conserved DNA, but not completely conserved. Choices B and C are thus incorrect. Only l%o of DNA differs dramatically from
person to person, so choice D is incorrect, too. The correct choice is A.

61.

B is correct. The probable function of restriction enzymes is defense. The bacterial cell can inactivate foreign, invading DNA (perhaps from viruses) before the invader has a chance to grow and lyse the bacterial cell or to incorporate itself into the bacterial genome. (Neither of these possibilities is particularly desirable for the host.)
Choices A, C, and D are incorrect. The correct choice is B.

62.

D is corred?. Child C

has two copies of normal HbA, so this child does not have sickle-cell anemia. It is a homozygote for HbA, thus choice A is true. Both parents carry one copy of the gene for HbA and one copy of the gene for HbS. This means they areheterczygotes, so choice B is also true. Child D has 2 copies of the sickle cell anemia gene, and therefore has sickle-cell anemia: Choice C is correct. Confirming that choice D is the false (and therefore, the best) answer requires applying our knowledge of standard genetics to the heterozygous parents. AS x AS yields 25Vo AA, 50Vo AS, and 25Vo SS. This gives a 25Va chance of these parents having a child with sickle-cell anemia. Choice D is false. The correct choice is D.

63.

C is correct. The children have to carry two copies each of the gene, one on each of their

homologous

chromosomes. Statement I is thus incorrect. Unlike the parents, the children are homozygotes, having two identical copies of the gene. C is homozygous for HbA, and D is homozygous for HbS. Therefore, statement II is correct. There is only one spot in the lane on the gel, since the two identical copies migrated to the same spot. On a real gel, the spots for the homozygous child should be twice as dense as those single ones for the homozygous parents. Statement III is correct. The correct choice is C.

64.

A is correct. Amniocentesis is a common procedure performed on some pregnant women. A very long needle is inserted through the abdomen and the uterus to collect samples of amniotic fluid. Cells from the fetus slough off into the fluid, and their DNA can be studied after cleavage with restriction enzymes. Fetal cells are not routinely moving
4AO

Copyright O by The Berkeley Review

The Berkeley Review Specializing in MCAT Preparation

Biology

Expression of Genetic lnformation

Section X Answers

around in the mother's blood supply, and it would be difficult to separate the two for analysis if this were the case, so choice B is incorrect. An abortion would not help an in utero diagnosis. Choice C is incorrect. A Cesarean section is used for some deliveries, not fbr tissue collection. Choice D is incorrect. The correct choice is A.
65.

children may face. The correct choice is D.

D is correct. Since DNA is long and fragile, it is usually chopped into small fragments for study and sequencing with restriction enzymes, so statement I is correct. Although parents and offspring have differeni DNA, tirere arJ plenty of similarities in the variable regions that could be used to establish piternlty along with other techniques. That means statement II is also corect. So is statement III: Parents who havi a familial history of genetic disease can be screened for major, identified genetic diseases to provide a better understanding of what -problems their

66.

D is correct. The passage tells us that the production of the C protein is under the control of the araOt operator. Furthermore, the passage states that if the level of C protein is high, the protein binds to the araOt operator site and inhibits transcription of the ara C gene. In other words, protein C is regulating its own production. Therefore, the synthesis is autoregulated. The correct choice is D.
C is correct. According to the passage, transcription of the structural genes takes place when the hairpin loop is not formed. The passage also explains that the loop is not formed because arabinose bound to the C protein aliers its conformation and the C protein binds to both the aral and araOl site. This, along with changes in the cAMp-CAp

67.

complex, brings about transcription of structural genes. It is clear from this information that the C protein is necessary for transcription. Therefore, a organi.sm lack the araC gene would exhibit inefficient transciiption of structural genes ara B, A, and D. The correct choice is C.
68.

position to another within the same molecule. This is describing the work of an isomerase. ttre correct choice is B. 69.

B is correct. One can see from the drawings that accompany the question that the enzyme catalyzed a reaction in which the structure but not the atomic composition of the substrate has been altered by moving a group from one

the structural genes. The hairpin loop intimately involved the ara Oz site, which is certainly not touching the structural genes on either side of it. Therefore, DNA-regulatory sites need not be directly in contact with the gines
that they regulate. The correct choice is B.

B is correct. We know from the passage that a hairpin loop of DNA can be formed that prevents the transcription of

70.

D is correct. This answer must come from one's previous knowledge of operons. The key is to realize that bacteria prefer to use glucose as their fuel source, but they can use a whole host of other energy molecules when glucose is not around. Therefore, the bacteria need some signal to denote when glucose is abundant or scarce. Thai signal is cAMP. Glucose lowers the concentration of cAMP in E. coli. Therefore, we can assume that high levels of J"llulu. cAMP are associated with low levels of glucose. The correct choice is D.

71.

in bacterial systems. Bacteria have only one molecule introlved in transcription, RNA polymerase. Eukaryotic organisms, on the other hand, have three types of RNA polymerases: RNA pol I (rRNA), RNA pol II (mRNA and hnRNA), and RNA pol III (tRNA ana inNn;. Therefore, the polymerase is RNA polymerase. The correct choice is A.
C is correct. If the genes for the uptake and usage of arabinose were all part of the same operon, the entire operon would have to be transcribed before the uptake could occur. However, if the genes were separate, transcription bf the uptake and structural genes could occur simultaneously. In this way, as soon as the arabinose entered thi organism, there would be enzymes already present to begin its metabolism. This is certainly a more efficient means of metabolism. The comect choice is C.

A is correct. The passage tells us that the arabinose operon is found

72.

73.

B is correct. The question asks which problem could not be treated using either of the two gene therapy strategies described in the passage. The first paragraph ofthe passage provides us with an important clue: Gene therapy is u"sed to prevent the expression of both disease-causing human genes and viral genes. The aim of both the gene therapy strategies is somehow to inactivate a dangerous gene. These approaches would be successful in preventing t^G overexpression of a tumor-causing oncogene (eliminating choice A). They would also be effective at preventin! the
translation of viral RNAs into DNA (thereby eliminating choices C and D). We are left with answer choice B. In

Copyright O by The Berkeley Review

4at

The Berkeley Review Specializing in MCAT Preparation

Biology

Dxpression of Genetic Information

Section X Answers

sickle-cell anemia, both copies of the vital gene coding for a hemoglobin subunit are mutated. The result is a defect passage could not help this condition, because inactivation of the mutant hemoglobin gene would only make matters worse. Instead of defbctive hemoglobin, there would be no hemoglobin! This would not bode well for a sickle-cell patient. The correct choice is B.

in erythrocyte function. The types of gene therapy mentioned in the

74.

gene, because mutant RNA does not form hybrids with the wild-type transcripi. Answer choice D iin be eliminated, because a gene sequence separated from its promoter is not transcribed. Recall, a promoter sequence lies just upstream of the transcribed portion of the gene. It attracts RNA polymerase and helpi initiate tranicription. Ifthe gene sequence is separated from the promoter.sequence, the gene might not be tranicribed. Even if the wild-type g_ene sequence on the plasmid were transcribed, this would not knock out the chromosomal gene's RNA transciipt. We are left with answer choice C. Remember that RNA polymerase can transcribe only in the S' -+ 3'direction. If we inveit the gene sequence on the plasmid relative to iti promoter sequence, the RNi polymerase can no longer read the correct strand of DNA, because this strand is now facing in the 3' -+5'direciion. Instead, it reads ite opposite strand, resulting in the production of antisense mRNA. ThiJantisense transcript can then hybridize with the sense transcript made by the chromosomal gene, thereby preventing its expression. The correct choice is C.

into a cell line, how could it inactivate expression of a gene that is present in the cellis chromosomes? From the passage, we learn about the antisense approach to inactivating genes. This question, in essence, is asking how one might design a plasmid that would encode an antisense RNR that could bind to the sense RNA prJduced by transcription of the chromosomal gene. Answer choices A and B both mention plasmids that contain mutant gene sequences. This would be detrimental if we were trying to knock out the expreision of a wild-type chromosJmal

C is correct. The hardest part about this question is understanding what it is really asking. If a plasmid is introduced

75.

B is correct. The question asks which statement would not represent an advantage that ribozyme therapy would hold over antisense therapy. We are asked to compare the two techniques. Statement i may be a tiue statem;nt in general, but it does not represent an advantage of ribozyme therapy, because the antisense approach doesn,t use sinthetic RNA--it uses synthetic DNA. Ribozymes are base-paired RNA, but the statement ^db".n't say that they are any stabler than the synthetic single-stranded DNA used by the antisense approach. Statement IiI is also in.or.""i because if individual ribozymes were very broad in their substrate specificity, they would cleave a lot of mRNAs in the cell' not just the disease-causing ones. This could be harmful ind certiinly ioesn't represent an advantage of ribozyme therapy. Statement II is correct. Ribozymes act catalytically. We know this because the passage statei that ribozymes act "enzymatically." Enzymes are catalysts; they Can be-recycled after speeding up a reaction. In what regard is this an advantage that ribozymes hold over antisense DNA?Antisense DNe is-not a catalyst. Once it hybridizes to its target, it is stuck there for good. It might take many antisense DNA molecules to inactivate a viral gene, for example, while it might take only a few catalytic ribozymes to accomplish the same task. This is an advantage for ribozymal therapy. The correct choice is B.

76.

protein's tertiary structure is dictated by interactions between amino acid residues and the water environment. What factors could affect base-pairing in the RNA ribozyme? A drastic change in pH could alter hydrogen-bonding between base-pairs. A decrease in the salt concentration of the medium *ould mean that the phosphate -backbone oT the base-paired RNA would be less stabilized by ionic interactions, leading to denatuiation. An increase in temperature could melt the RNA hybrid, thereby denaturing it. These possibilities prompt us to eliminate choices B, C, and D. The correct choice is A.
77.

substrate. Recall from the Michaelis-Menten curve that changing the substrate concentration has no affect on the Ku of an etzyme. Therefore answer choice A is correct. What factors would affect the affinity of an enzyme for its substrate? One possibility is the partial or total denaturation of the enzyme. An enzyme can be denatured when its tertiary (folded) structure is disrupted. In this question, the enzyme we are dealing with is a ribozyme. The tertiary structure of a ribozyme is dictated fundamentally by the way it base-pairs with itsell much in the same *uy u, u

A is correct. The Kv of an enzyme is a measure of that enzyme's affinity for its substrate. It is equal to the amount of substrate necessary for the catalyzed reaction to proceed at half of its maximal rate, or l/2 ymax. The question asks for the condition that would not affect the Krt. Answer choice A mentions altering the concentration of

D is correct. From the passage, we learn that ribozymes cleave mRNAs at specific sequences. If a ribozyme could cleave an HIV mRNA at a specific, conserved_sequence, such a ribozyme would be capable of chopping up most mutant forms of the viral RNA as well. This is because a conserved sequence by its definition doesn'i ctra-nge much from mutant to mutant. It may encode a very essential part of a protein product ihut .an't be altered. TherefJre, even if the HIV virus mutates, the ribozyme could still recognize the-conserved sequence and cleave the *RNA, thereby preventing production of mature viral particles. Antisense DNA, on the othei hand, must be complementary to the mRNA to which it hybridizes. If HIV mutates rapidly, a predesigned antisense strand would not be able to hybridize
@

Copyright

by The Berkeley Review

4A2

The Berkeley Review Specializing in MCAT Preparation

Biology
*irh nerl
78.

Expression of Genetic lnformation


murant mRNA. This eliminates choices A and B. Choice C is incorrect, because u ouldn't be much use as a therapy at all. The correct choice is D.

Section X Answers
if ribozymes cleaved all

mRli-{s. thel'

B is correcl This is the only answer that mentions a problem that could occur only in human patients and not in a tissue culture of human cells. Human immune response to the synthetic DNA (containing methylphosphonate) could be a tatal side effect to some kinds of antisense gene therapy. Answer choice A is incorrect, because it describes something that could occur both in the human patient and in the tissue culture. Answer choices C and D are incorrect for the siune reason. The correct choice is B.

79.

A is correcL This question can be answered through the process of elimination. We must bear in mind what exactly ribozymes are capable of, and with this knowledge we can rule out functions that ribozymes could never have in normal cells. Answer choice B is such an example. From the passage we learn that ribozymes cleave RNA at specific sites. Nowhere are we told that it can proofread DNA. This is the function of DNA polymerase. Answer choice C is likewise incorrect: Ribozymes do not catalyze protein reactions. Answer choice D is wrong, because ribozymes can't cleave DNA at all; they can only cleave RNA. This leaves us with answer choice A. In order to
splice mRNA and remove introns, the mRNA strand must be cleaved at a specific site. Ribozymes are ideal for this function, and in fact, this is how they were initially discovered. The correct choice is A.

80.

C is correct. Answering this question correctly just requires one to be familiar with the concept of a mRNA that is polycistronic. The answer cannot be gathered from information in the passage. An mRNA molecule coding for more than one protein is known as a polycistronic transcripl. Let's consider the other possibilities: Since one protein is not encoded for by multiple genes, choice A can easily be eliminated. Choice B wants us to believe that one protein is coded for by a single gene. This statement is true, but it is does not pertain to polycistronic transcripts. The definition is the relationship between mRNA and proteins, not DNA and proteins. Therefore, choice B can be eliminated. Finally, choice D tells us that one mRNA molecule codes for only one protein. While this is not the definition of polycistronic, this method of translation occurs in all eukaryotic organisms. The polycistronic transcript is not seen in eukaryotic organisms. The correct choice is C.

81.

C is correct. From the passage, we learn that the repressor protein alone does not cause the repression of the tryptophan operon. The presence of tryptophan in a complex with the repressor protein brings about the true
repression. Therefore, tryptophan is best described as a corepressor. Let's consider the other possibilities: The term

inducible repressor is nonsense. It certainly has no application to the tryptophan operon and should be easily recognized as an incorrect answer. The only example that comes to mind that resembles an inducible repressor is the situation where glucose in a medium causes a reduction in the synthesis of the enzymes needed to utilize lactose. This occurs in the lactose operon, and the process is termed "catabolite repression." Nevertheless, choice A does not best describe tryptophan, and therefore choice A can be eliminated. Eliminating choice B should be easy, because it is very clear that the presence of tryptophan causes a reduction in the expression of the operon. Therefore, tryptophan is not acting as an inducer. In considering choice D, one needs only to consider what an attenuator is. It is a seque-nce of DNA that offers a means of regulation through differential transcription. This is not the role of tryptophan, and therefore choice D can be eliminated. The correct choice is C.
82.

B is correct. This answer can be arrived at by looking at the diagram. It is clear from the question that

the attenuator site along with the leader sequence has been deleted from the operon. The passage tells us that these DNA

sites are involved in attenuation. Attenuation (along with the repressor-tryptophan complex) regulates tryptophan synthesis in such a way that the operon is expressed only when tryptophan is needed. If one sees a dramatic increase in the production of tryptophan mRNA, one can assume that this experiment provides evidence for the role of attenuation in regulating the synthesis of tryptophan mRNA. Let's consider the other possibilities: The experiment tells us nothing about signal transduction (the process of extracellular signals resulting in intracellular changes). Therefore, we can eliminate choice A. Furthermore, this experiment provides no evidence for the tryptophanrepressor complex repression. The operator/promoter is still intact, as nothing is stated about the repressor protein. Therefore, otroice C can be eliminated. Finally, tryptophan induction is not a term defined in the passage, so the experiment probably does not have anything to do with it. In fact, the term has no set definition, and this answer choice can be easily eliminated. The correct choice is B.
83.

B is correct. The tryptophan-repressor protein binds to the operator and prevents transcription. Therefore, it prevents the interaction not between DNA polymerase and DNA (choice A), but between RNA polymerase and
@

Copyright

by The Berkeley Review

4A3

The Berkeley Review Specializing in MCAT Preparation

Biology

Dxpression of Genetic Infonnation

Section X Answers

DNA. So the question becomes: Which RNA polymerase? The tryptophan operon is found in E. coli, a bacterium. Bacteria have only one type of RNA polymerase, known simply as RNA polymerase. Choice B is the best answer. Eukaryotes, on the other hand, have more than one type of RNA polymerase. In fact, they have three types. The
correct choice is B.
84.

D is correct. During attenuation, it is important for the leader peptide to be translated. That is how the system detects the level of tryptophan in the cell. If the codon UGG is there, but there is a lack of tryptophanyl IRNA, this
causes a pause and a conformational change in the RNA polymerase. That change results in the RNA polymerase continuing to transcribe the operon. If there is no pause, the RNA polymerase falls off the DNA. Therefore, one can see that it is important that tianscription and translation be closely coupled. Based on this information, the other

answers can be easily eliminated. One can see that replication of DNA has attenuation regulates the cellular levels of tryptophan. The correct choice is D.

little to do with attenuation, but

85.

D is correct. The attenuator site, according to the passage, contains a G-C rich region and an A-T rich region. Therefore, melting the piece of DNA (turning double-stranded DNA into single-stranded DNA) does not occur abruptly at one ternperature, because there is a varied nucleotide composition. The melting can be described as discontinuous. This eliminates choices A and B. The next decision comes from knowing that it takes less energy to melt an A-T rich region when compared to a G-C rich region. This, of course, stems from knowing that there are
two hydrogen bonds between adenine and thymine, and three hydrogen bonds between guanine and cytosine. The correct choice is D.

86.

contains Therefore, amino acid. level of the to measure the is designed system tryptophan residues, because that is how the thile;der peptide of any operon should contain a relatively large number of residues of the kind synthesized by the operon. The operon should not contain tandem pairs of the codon UGG, unless it is the tryptophan operon. Tierefore, choiie A is incorrect. If the operon has a relatively small number of the UGG codons, it is most likely by chance and not design. In other words, a leader peptide does not have to contain a small number of the UGG codons. Therefore, choice B can be eliminated. Again, to detect the level of a particular amino acid in the cell better, the leader peptide of an operon should contain a relatively large number of the residues of the kind synthesized by the operon. The conect choice is C.

C is correct. Let us look at the example in the passage. The leader peptide of the tryptophan operon

87.

B is correct. Remember, the residues in the leader peptide are the detection system of the operon. Increasing the number of those residues (the kind synthesized by the operon) would detect the amino acid level in the cell better. This automatically eliminates choice A. There is no evidence for either choices C or D. Attenuation is clearly not eliminated by incieasing the number of residues in the leader peptide. In addition, the change in residues results in
more thanjust a change in structure, but also a change in function. The correct choice is B.

88.

B is corqgct. The passage informs us that in reticulocytes (eukaryotic cell), eIF-2 is involved in controlling the overall rate of prot.in synthesis. In addition, we are told that phosphorylation of this protein reduces its activity and thus most likeiy reduces the overall rate of protein synthesis. Under starving conditions, one can conclude that
protein synthesis in the cell decreases. Therefore, phosphorylation of eIF-2 would occur. This requires the work of a kinut". One assumption should be made here: Yes, the passage does specifically state "reticulocytes," while the question asks abouichanges in the more general term "eukaryotic cell." However, we must use what information the purrug" gives us, as thi; type of questioning can appear on the MCAT. Therefore, we would expect increased activity from a kinase. The correct choice is B.

89.

B is correct. We are told in the passage that roughly I out of every 10,000 amino acids incorporated into proteins is inserted incorrectly. Then the question says that the average size of a protein is 400 amino acids. Therefore, 10,000/400 = 25. One out of every 25 protein molecules should contain an error. The correct choice is B. B is correcL According to the passage, a polyribosome is a single mRNA transcript that has multiple ribosomes attached. Comparing a single ribosome to a polyribosome, we can clearly see a difference in size, with the polyribosome beinglarger. We know that centrifugation takes advantage of this situation, based on the fact that iarge molecules move faster than smaller ones in a gravitational field. Therefore, centrifugation would allow us to ."udily separate these two kinds of ribosomes in solution readily. The correct choice is B.

90.

Copyright @ by The Berkeley Review

4A4

The Berkeley Review Specializing in MCAT Preparation

Biology
91.

Expression of Genetic Information

Section X Answers

D is correct. This question must be answered from a previous knowledge base. It is asking what amino acid is called for by the start codon in a prokaryotic cell. The answer is a modified methionine. In particular, a formyl group is attached to make a formylmethionine. In a eukaryotic cell, the start codon calls for methionine. Since the question asks for the prokaryotic cell, formylmethionine is correct. The correct choice is D.
Each start codon results in the translation of a particular protein, each with a unique structure and function. This is in contrast to a monocistronic transcript, which contains only one recognized start codon, which gives rise to only one peptide chain. Polycistronic transcripts are often found in prokaryotic cells, while monocistronic transcripts are often

ot

C is correct. This question requires previous knowledge. A polycistronic transcript contains multiple start codons.

found in eukaryotic cells. The correct choice is C. 93.

C is correct. A proofreading mechanism implies preventing and/or removing an incorrect unit so that a correct unit can be inserted. Therefore, our answer should address this issue. The only choice that satisfies this criterion is choice C. Choice A is not the best answer, as it implies that time alone assures that the correct aminoacyl transfer RNA eventually will be inserted. There is no explanation about any proofreading function. Indeed, there are hydrogen-bond interactions between the bases in the codon and the anticodon. If there is an incorrect bonding, the dllay makes it very difficult for the incorrect pairing to hold, as such a situation is inherently unstable. The delay

allows for the diisociation of this unstable complex, and the correct aminoacyl transfer RNA can then
incorporated. The correct choice is C.

be

94.

B is correct. Choice B claims that no initiating factors are needed for secreted proteins. Recall that under the signal peptide hypothesis, a protein that is to be secreted begins its translation in the cytosol and then is attached to the endoplasmic reticulum to continue and finish its translation. The key point here is that translation begins in the cytosol, just like a protein that is not to be secreted. This requires formation of a competent ribosome, which according to the passage, involves initiating factors. Therefore, we cannot make the claim that initiating factors are not needed, because they clearly are. The correct choice is B.

9s.

C is correct. The passage states that in theory, one could compare the base sequences of a normal cell and a cancerous cell. In thiory, the differences one would encounter could be attributed to cancer. However, this is not the case. Even if it were practical to sequence both genomes, we could not ignore the fact that mutations naturally occur. Therefore, there would be more than just one difference, and one probably could not determine which mutation was the cause of cancer. Therefore, naturally occurring mutations would mask the cancer-causing gene. The correct choice is C. C is correct. In the passage, it is clearly stated that the screening process involves using a sequence of DNA complementary to the tagging sequence. We want this probe to hybridize only with the tagging region (which is tropetutly uttuih"d to our mutated cancer gene), and not with other pieces of DNA. If we used a tag sequence that was normally found in E. coti, the probe would hybridize to regions of DNA other than the one we are interested in. This would iho* up as many false positives, and our isolation technique would be unsuccessful. The correct choice is C.
C is correct. The tagging sequence is hopefully near the cancer-causing mutated gene we are interested in isolating. From the passage, *e i." told that the Alu site is found throughout the entire human genome. This is a very
attractive tiait as a tag, because we are betting that because the site is so widespread throughout the chromosomes, it has a solid chance of Ueing near our mutated gene of interest. Therefore, when we clone and screen, we will hopefully include this mutated gene. The correct choice is C.

96.

9-7.

98.

C is correct. This question is straightforward. We know from the question that gene A is known to induce cancer. Also, from the information in the question, we can conclude that there is nothing wrong with the transcription of the gene or the efficiency of translating the messenger RNA. Therefore, we should conclude that even though the frotein is syhthesized, it is useless. Recall that protein function is tied into its structure. Therefore, a nonfunctional protein probably suffers from a significant structural change that resulted from the gene mutation. The correct
choice is C.

Copyright O by The Berkeley Review

4a5

The BerkeleY Review Specializing in MCAT Preparation

Biology
99.

Expression of Genetic Information

Section X Answers

B is correct. Two strands of DNA hybridize because they have complementary bases. Recall, that bases bind to each other because of hydrogen bonds. Adenine and thymine share two hydrogen bonds, while guanine and cytosine share three hydrogen bonds. Therefore, the hybridization occurs because base pairs do form hydrogen bonds with each other and create a stable structure. Without the hydrogen-bond interaction, the two strands would not anneal. The correct choice is B.
be answered from a previous knowledge base. A Southern transfer is as follows: Using electrophoresis, DNA fragments are distinguished on the basis of their size. The fragments are transferred to some sort of membrane and are immobilized. Under hybridizing conditions, labeled oligonucleotides are added and complementary bases form pairs. The label can then be found with autoradiography. Northern transfers involve RNA, while Western transfers involve proteins. Although the molecules are different for these experimental methods, the concepts are very much the same. The correct choice is A.

100. A is correct. This question must

Copyright @ by The Berkeley Review

4a6

The Berkeley Review Specializing in MCAT Preparation

Biotogy

Glossary

MCAT Words and Phrases

Glossary
Most of the items listed in this glossary are important words or phrases that have appeared in one of the following publications of the Association of American Medical Colleges (AAMC):

2. 3. 4. 5.

1,. MCAT Practice Test I MCAT Practice Test II MCAT Practice Test III
MCAT Practice Test IV MCAT Practice Items (Biological Sciences)

These words or phrases are specific to the Biological Sciences portion of the MCAT. We have chosen them based on student response, and on how important we feel they are in building a good conceptual understanding of the

information basic to all biological sciences.

A
o-ketoglutaric acid ABO blood system

Alanine (Ala)

Absorption Acetaldehyde
Acetic acid

Acetylcholine
Acetylcholinesterase

Acetyl-CoA Acid, acidic Acidophil, acidophilic Acquired immune deficiency syndrome (AIDS)
Acrosome

Albumin Alcohol Alcohol dehydrogenase Aldehyde Aldosterone Allele Allergy, allergic reaction Alveolus, alveoli (p1.)
Ames test Amino acid

Ammonia
Amoeba, amoebae or amoebas (pl.)

Actin Action potential Active transport Adenine Adenosine Adenosine diphosphate (ADP) Adenosine triphosphate (ATP) Adenylate cyclase Adipose cells Adrenal cortex Adrenal gland Adrenaline Adrenocorticotropic hormone (ACTH) Adsorption
Aerobe, aerobic

Amphoteric

Ampicillin
Anabolism, anabolic Anabolize
Anaerobe, anaerobic

Androgen

Afferent fiber Afferent vessel

!'

African sleeping sickness Agar Agglutination Agonist


Copyright @ by The Berkeley Review

Aneuploidy Angiotensin I Angiotensin iI Angiotensin-converting en4,"rne (ACE) Angiotensinogen Aniline Antagonist Antibiotic Antibody Antidiuretic hormone (ADH) Antigen, antigenic Antigenic shift Antihistamine Antitoxin Aorta
4A7

The Berkeley Review Specializing in MCAT Preparation

Biology
Apocarboxylase Aqueous base

GIossar5r
Camphoric acid

MCAT Words and Phrases

Arginine (Arg) Artery, arterial Asparagine (Asn) Aspartic acid (Asp) ATP synthetase
ATPase

Capillary Capillary bed


Capsule

Carbaminohemoglobin Carbohydrate
Carbonic anhydrase

Carboxyl
Carboxylase Carboxylate, carboxylation Carboxylic acid Carcinogen, carcinogenic Cardiac output Cardiac stroke volume

Atrium, atria (pl.) Atrophy Atropine Autoimmune disorder


Autonomic nervous system Autosomal Autosomal dominant Autosomal recessive

Auxotroph Axial Axon Azidothymidine (AZT) B


p-galactosidase
B lymphocyte

Cardiopulmonary circulation Carrier Cartilaginous Haversian canal


Casein Catabolism, catabolic Catabolize Catalyze Cell membrane Cell wall Cellulose Central nervous system (CNS)

Back-mutate Bacterium, bacteria (p1.)


Base Base

pair

Basement membrane Basic

Centrifuge Centriole Cerebellum Cervix, cervical Channel blocker


Cholesterol Cholic acid Chordate

Basophili, basophilic Bicarbonate ion (HCO3-)

Binary fission Biocytin

Biotin Biotinidase Blood pH Blood platelet Bone deposition Bone marrow Bone remodeling Bone resorption Brain Bronchiole Buffer

Chromatid Chromatin Chromatography


Chromosome

Chylomicron Chymotrypsin Chymotrypsin mechan ism Cilium, cilia (pl.) Circular DNA Circular muscle
Cisterna, cisternae (p1.) Citric acid cycle
Class

ci!
Calcitonin Calcium carbonate Calcium caseinate

Clostridium tetani

Codominant
Color blindness Committed stem cell

Congenital
Conjugate, conjugation Connective tissue

Calmodulin Calvin cycle


Copyright @ by The BerkeleY Review

The BerkeleY Review Specializing in MCAT PreParation

Biology
Constitutive Contractile
Conus arteriosis

Glossary
D r osophila mel ano gaster

MCAT Words and Phrases

Ductus arteriosus

Duodenum
E

Coprophagy Cortex, cortical Cortisol


Creatine phosphate Cross-bridge Crossover

Early gene Ectopic pregnancy Efferent fiber


Efferent vessel Electrical potential Electron transport

Cryoprotection
Cutaneous Cyclic AMP (cAMP) Cysteine (Cys) Cystic fibrosis (CF) Cytochrome b Cytochrome oxidase

Electron-withdrawing group Electrophile, electrophilic

Elution Embryo
Embryogenesis

Cytokine Cytoplasm
Cytosine

Embryonic
Encode Endocrine system Endocytosis Endocytotic vesicle Endogenous Endoplasmic reticulum

Cytoskeleton Cytosol

D
Dalton Dark reactions (Calvin cycle) Daughter cell Daughter strand Decarboxylation Dehydration
Dehydrogenase

Endothelium, endothelia Endotoxin Enol


En4zme

(p1.)

Deletion mutation Dendrite Deoxygenation Deoxyribonucleic acid (DNA) Deoxyribonucleoside Dephosphorylation Depolarization * Depolymerization Deprotonation Deterministic Deuterium Diastolic blood pressure

Epimerize Epinephrine Epithelial Equatorial Erythrocyte


Escherichia coli

Essential amino acid


Ester

Esterification Estradiol
Estrogen

Ethanol Ether Eukaryote


Excision

Differentiation Digestion Dilate Diploid Distal tubule DNA recombination DNA repair Dominant Dopamine Dorsal root ganglion Down syndrome (trisomY 21)
Copyright O by The BerkeleY Review

Excitatory
Excrete

Exocrine system Exocytosis Exogenous Exon


Exonuclease

External respiration

Extracellular medium

449

The BerkeleY Review Specializing in MCAT PreParation

Biotogy
F
Facilitated diffusion Facultative anaerobe FAD FADHz Fallopian tube Familial hypercholesterolemia Family
Fatty acid

Glossary
Glycoside Golgi apparatus Golgi body
Gonorrhea

MCAT llllords and phrases

Gradient Gramicidin
Gram-negative

Gram-positive

Growthhormone (GH)
Guanine Guanosine Guanosine diphosphate (GDp) Guanosine triphosphate (GTp)

Feedback-inhibited Feminization Fermentation

Fibrin Fibrinogen Fibroblast Fightor-fl ight response First filial (Fr) generation Follicle-stimulating hormone (FSH) Formaldehyde Frameshift mutation Fungus, fungi (pl.)

H
Habituation Haldane effect Haploid Heart contraction Heart rate
Heatstroke

G
G protein Galactose

Helper T cell (helper T lyrnphocyte)


Hematopoiesis, hemopoiesis

Hematopoietic stem cell (HSC)


Heme

Ganglion, ganglia (pl.)


Gastric enzyrne

Hemoglobin (Hb) Hemolysis

Gastrointestinal tract Gastrulation Gel chromatography


Gel electrophoresis
Gene

Hemophilia
Hemophilia factor VIII Hemorrhage Hepatic Hepatitis Heterotropic Heterotrophic Heterozygous

Genetic engineering Genital, genitata (pl.) Genome Genotype Genus, genera (plJ Germ cell Germ cell layer

Glopin Glomerular filtrate Glomerular filtration Glomerulus, glomeruli


Gluconeogenesis Glucose Glucose-6-phosphate Glutamic acid (Glu)
-.*

Hibernation High-density lipoprotein (HDL) Hippuric acid Histamine Histidine (His) Histidine-deficient Histone
(p1.)

Holocarboxylase slmthetase Homeostasis

Glutamine (Gln) Glycerol Glycine (Gly)


Glycogen

Homogenized Homozygous Hormone Host cell Human chorionic gonadotropin (hCG) Huntington's chorea

Glycolysis Glycoprotein
Copyright @ by The Berkeley Review

Hybrid Hydration Hydroboration 49o The Berkeley Review Specializing in MCAT Preparation

Biology
Hydrolysis Hydrophilic Hydrophilic amino acid Hydrophobic Hydrophobic amino acid Hydroxyl Hydroxylation Hydroxyprogesterone Hyperglycemia Hyperosmotic Hyperplasia Hyperpolarization Hypertension Hypoglycemia Hypotension Hypothalamus, hYPothalamic Hypovolemic shock

Glossary
K
Kanamycin

MCAT Words and Phrases

Ketone Killer T cell (killer T lymphocyte) Kinetochore microtubule

Kingdom
Krebs cycle

L
Lactalbumin
Lactase

Lactation
Lacteals Lactic acid

Lactoglobulin
Lactose Late gene

Left ventricle

I
Imidazole ring Immune reaction Immune response

ImmunitY Immunization immunoglobulin Imprinting Inhibitory Innervation Insulin Integration Intermolecular Internal resPiration Interstitial fluid Intestinal goblet cell Intestinal wall Intracellular medium
Tntramoleculat

Lethal mutation Leucine (Leu) Leukocyte Lewis acid Lewis base

Ligament Ligand

Intrauterine Intravenous infusion

lntron Invariant
Ionic Ionic imbalance
Isoleucine (Ile)

Light reactions Lipid Lipid bilayer Lipid-soluble Lipophilic Lipophobic Lipopolysaccharide Lipoprotein Liver Locus,loci (pl.) Logarithmic microbial growth Loop of Henle Low-densitY liPoProtein (LDL) Lumbar Luteinizing hormone (LH) Lymphatic sYstem
LymphocYte Lysine (Lys)

Lysis
Lysogen, lYsogenic Lysosome

Isomeric Isotonic

M
Macrophage

Juxtaglomerular aPParatus ]uxtaglomerular cells

Mammal

Medulla oblongata Meiosis, meiotic

Copyright

@ bY

The BerkeleY Review

491

The BerkeleY Review Specializing in MCAT PreParation

Biology
Membrane-bound enzyme Membrane-bound receptor
Menopause

Glossary

MCAT Words and phrases

Nonessential amino acid Nonkinetochore microtubule Nonsense mutation

Menstruation
Messenger RNA (mRNA)

Metabolism, metabolic Metabolize Methanol Methionine (Met) Microbe

Norepinephrine Nuclear membrane Nucleic acid Nucleophile, nucleophilic Nucleoside Nucleotide

Microcirculation Microfilament Microtubule Mitochondrial DNA (mtDNA) Mitochondrion, mitochondria (pl.) Mitosis, mitotic Molec u la r hyb rid i zation Monogastric
Monosaccharide

Nutrient Nutrition

o
Obligate aerobe Obligate anaerobe
Oleic acid Oncogene

Morphine Morphology Mosaic hypothesis Motiiity Motor neuron


Mucosa

Opiate Optical antipole Order


Organelle

Osmoregulatory
Osmosis, osmotic Osteoblast Osteoclast Osteocyte Osteoporosis

Mucus (n.), mucous (adj.) Multiple genes Mutagenic Mutation


Mr1 c ob a c t erium tub

Ovary

eruli sis

Oviduct
Ovulation Ovum, ova (p1.) Oxidation Oxidative phosphorylation Oxidizing condition Oxygenation Oxytocin P
Pso

Myelin Myoglobin Myosin Myosin light chain Myosin light chain kinase (MLCK)
Myxoairus influenzae

N
Na+/6+ -ATPase Na+/K+ pump NAD+ NADH Natural killer cell Negative feedback Nephron Neuroeffector Neuron
Neurotransmitter

Pancreas, pancreatic

Parasympathetic nervous system Parathyroid hormone (PTH) Parent cell Parent strand Parental (P) generation Parkinson's disease Partial pressure
Passive diffusion

Neurosecretory :
Neutrophil Nitrogenous waste Nitrous oxide
Copyright O by The Berkeley Review

Pathogen, pathogenic

Penicillin
Pepsin Peptidase Peptide

492

The Berkeley Review Specializing in MCAT preparation

Biology
Peptide bond Peptidoglycan Peripheral nervous system (PNS) Peritoneum

Glossary
Prototroph

MCAT Words and Phrases

Protozoon, protozoa (pl.)

pH optimum Phage progeny


Phenol Phenotype

Proximal tubule Pulmonary artery Pupillary light reflex Purine Pyridine

Phenylalanine (Phe)
Phosphatase

Pyrimidine Pyrrolidine Pyruvic acid

Phosphatidylcholine Phosphoprotein Phosphorylation Photophosphorylation


Photosvnthesis

a
Quinone

Phylum, piryla (pl.)


Physostigmine

R
Racemize

Pigment Pilomotor
Pilus, pili (p1.) Pineal gland PinealectomrPiperichne

Radial muscle Radioimmunoassay (RIA)


Rana sylaatica

Rate-determining step
Reabsorption Receptor
Recessive

Pituitarv gland
PKo Placebo effect Plasma Plasma clearance Plasma mernbrale

Recombination Red blood cell (RBC) Reduction Regulative hypothesis


Renal

Plasma osmolaritv Plasmid Plateau Pluripotent stem cell Point n'Lutatior-r Polar Polarized Polvmerize, polvmerization Pol;,'peptide Polvsaccharide Positir-e ieedback Postmenopausal Postsvnaptic membrane Premature stop codon

Renin-angiotensin system Replication

Repolarization Reproductive tract


Resonance

Respiratory tract Reticulum, reticula (pi.) Retina

Retrovirus
Reverse transcriptase Rhesus factor

Preprotein
Presvnaptic membrane

Rh-negative Rh-positive Ribonucleic acid (RNA) Ribonucleoside


Ribose

Primarv structure
Probe

Ribosomal RNA (IRNA)


Ribosomes, eukaryotic (40S, 50S, 80S) Ribosomes, prokaryotic (30S, 50S, 70S)

Progesterone .
Prokarvote Proline (Pro)
Prophage

Ribozyme

Right ventricle
RNAase Rough endoplasmic reticulum (RER) Rumen

Proteolytic Protonation
Copyright @ by The Berkeley Review

493,

The Berkeley Revieu' Specializing in MCAT Preparation

Biology
S
S

Glossary
Stereochemistry
Stereoselective Stochastic

MCAT llllords and Phrases

acchar omy ces c er eaisiae

Sacral

Salivary gland
S almonelln typhimur ium Sarcomere

Stomach acidity Stop codon

Streptomycin
Striated muscle Strict aerobe Strict anaerobe
Subspecies

Sarcoplasmic reticulum Saturated hydrocarbon


Second Secrete

filial

(Fz) generation

Secondary structure Secretory protein Semiconservative replication Seminal vesicle Seminiferous tubule
Sensory neuron Sepsis Septic shock

Succinic acid
Sucrose

Support cell Sweat gland Symbiotic


Sympathetic nervous system
Synapse

Septicemia
Serine (Ser)

Serotonin
Serum, sera (pl.)
Sex

pilus

Sex-linked trait Sickle-ceil anemia Signal hypothesis Signal peptidase Signal peptide Signal recognition particle (SRP) Simple diffusion Sinus venosus Skeletal muscle

Synapsin Synaptic transmission Synthesis, synthetic Synthesize Systemic artery Systemic circulation Systemic vein Systolic blood pressure

T
T cell

Tautomerism Tendon Tertiary strucfure Testis, testes (pl.)


Testosterone Tetanospasmin Tetanus Thermogenesis

Smallpox Smooth endoplasmic reticulum (SER)


Smooth muscle

Sodium-potassjum pump (Na+/K+ pump)

Solubility
Species

Thermoregulation
Thorax, thoracic Threonine (Thr)

$pectrophotometry
Spectroscopy Sperm

Spermatid
Spermatocyte, primary and secondary

Spermatogonium, spermatogonia
Spermatozoon, spermatozoa Spinal cord
(P1.)

(p1.)

Spleen
Splicing
Spore

-.:

Thrombin Thymine Thymus Thyroid gland Thyroid-stimulating hormone (TSH) Thyroxine Totipotent stem cell Toxin, toxic
Trans

Spontaneous mutation Start codon Stearic acid


Stem cell Copyright
@

Transcription Transcriptional regulation Transfer RNA (IRNA) Translation Triglyceride 49,4

by The BerkeleY Review

The BerkeleY Review Specializing in MCAT PreParation

Biology
True-breeding variety Trypanosome Trypanosomiasis Tryptophan (Trp) Tubal pregnancy Tubular load Tubular transport maximum (T6)

Glossary
X
X chromosome

MCAT Words and Phrases

Y
Y chromosome

Tubulin Tumer's syndrome Tyrosine (Tyr)

z
Zygote Zymogen

U
Ubiquinone

Uracil
Urea
lJrease

Uric acid Uridine triphosphate (UTP) Urine Uterus, uterine

V
Vaccine Vaccinia r-irus Vagus nen-e Valine (Val) ValinomJ-cin Variable surface glr-coproteins Vascular smooth m'.r-.cle

Vasoconstriction Vasodilation
Vasopression Vein, venous Vena cava

Ventricle
Vertebra, vertebrae Vertebrate
ipJ..
r

Very low-densiti- lipoprotein (VLDL)

Villus, villi (p1.) Viral envelope Viral genome Virion Virulence Virus Viscosity Visual centers Vitamin D

w
Water-soluble White blood cell (\\BCl Wild type

Copyright

by The Berkelel Revie,.r

495

The BerkeleY Review Specializing in MCAT Preparation

Biology
Biology Bubble Sheet (Make five copies, one for each section of the book.)

Section Answers

l.
,,

@@o@l
@@@@

37.
38.
l l

3. 4. 5.
6.

@@o@ @: @o@]

39.

@@@@ 7. @@@@ 8. @@@@ 9. @@@@ 10. @@@@ 11. @@@@ 12. @@@@ 13. @@@@ 14. @@@@ 15. @@@@ 16. @@@@ L7. @@@@ @ 18. @@ @ 19. @@ @ 20. @@ 2r. @@ @ )) @@ @ @ 23. @@ 24. @. @ @ @ 25. @@ @ 26. @@ 27. @@ @ 28. @@ @ 29. @@ @ 30. @@ @ 31. @@ a) @@ @ @ 33. @@ @ 34. @@ 1( @@ @ @ 36. @@

@G)(oq_l

40.
l

41.
42.

43. 44. 45. 46.


47.

48. 49.
50.
51".

52.
53.

@ @ @ @ @ @ @ @ @ @ @ @ @ @ @ @ @ @ @

54.
56.

57.
58. 59. 60.

61. 62.
63.

64.
65.

66.
67. 68.

69.

70.
7L. 72.

@@ >@l @@ .i@@l /-\ /,F\ @@ (o@ @@ @@ @@ @@ @@ @@ @@ o@ @ @@@ @ @@@ @ @@@ @ @@@ @ @@@ @ @@@ @ @@@ @ @@@ @ @o@ @ @@@ @ @@@ @ @o@ @ @@@ @ @@@ @ @@@ @ @@@ @ @@@ @ @@@ @ @@@ @ @@@ @ @@@ @ @@@ @ @@(Et @ @@@] @ @@@l @ @o@ @ G>(e)@i @ @@@l @ @(o@l
l l

zs.oo@@
7s. 76. 77.

74.'@ @ @ @

@ @ @ @ @ @ @ @ @ @ @ @ 78.@@@@ 7e.@@@@ 80.@@@@ 8t.@@@@ 82. @ @ @ @

ffi.I@@o@
84.@@@@ 8s.@@@@ 86.@@o@ 87.@@@@

88. 89. 90.

9r.
92. 93. 94.
95.

96. 97.
98.

99.
100.

@@@@ @@@@ @@@@ @@@@ @@@@ @@@@ @@@@ @@@@ @@@@ @@@@ @@@@ @@@@ @@@@

Raw

[]
I

Score

Estimated
Scaled Score

Copyright

The Berkeley Review

496

The Berkeley Review Specializing in MCAT Preparation

EKI{ELEY
R.D.V.I.E.W'"

PERIODIC TABLE OF TTIE ELEMENTS

He
-1.0

lri
i
I

Be
9.Cr

B
10.8
13

c
12.O

N
14.0
15

o
16.0
16 S

910
F\C

6.9

19.0

20.i

1l i i: Na | \tg
23.0
lQ

14

AI
27.0 TI

Si
28. I

P
31.0 JJ

l-1 ,i : lir

32.1

:s.s I re q

t7 18 Cl]r.
,i6

Rb Sr 85.5 I S- 6
_<6

37irs

n-i -<t, 5J.9i55.8 -r_ ': -1,i i 44 \h \Io Tc i Ru


;:; r:"
9_li 1101_l
i

c; lt'i;;

a5t)6

27

28

29

30

31

32

34
Se '79.0

Co
s8.9 45

Ni
58.7

Cu
63.5 47

Zn
65.4

Ga
69.7 49

Ge
72.6 50

As 74.9

79.9

35 Br

Itu
8-r,S

Rh
102.9

46 Pd
106.4 78

48

5l
sb
121.8 83

Ag Cd r0't.9 112.4
79 80

In
114.8 81

Sn
118.7

52 Te t2'7.6 84

53 51 IXe
126.9 1_I

Ba
l

l-

_l

Fr

87is.
lit
Rr
-

--t -,5 : ,16 Tr SReOs i,i, : , ir i 155: 190.2 I -r- ltrS - ": , "r Eho' _(E Bh tls
--: -:-f':l

77

lr
192.2
109
I

82

Pt
95.1
110

Au TI Bi Hg Pb Po t97.0 200.6 204.4 207.2 209.O (209)

(2t0)

85 At

86

Rn (222t

(223r

Uun Uuu Uub (266) (269) (2't2) (2'77)

llt

lll

112

:i
tfc
4:

Pr

;.- :.

\d
.l,1:
[-

:. 6l Pm Sm
' 1< I<

63

64

65

66

67

68

69

70

7l
Lu
175.0
103

Eu
l -i1,0

Gd
r57.3
96
r

Tb
158.9

Dy
t62.5
98

Ho
164.9 99

Er
t67.3
100

Tm
168.9
101

Yb
173.0

.l{" I

t:
ll'n

"".",...----'-Tt-

p\1

\p
:,:

-{m --:-1 r 'Jf

i-l Pu

95

97

l02
No

BK Es Cm Cf Fm (217) (247) (2st) (2s2) (2s7)

Md
(2s8)

Lr

(2se) (260)

Specializing

!rc"[T

FlurymMdixum

Você também pode gostar